You are on page 1of 579

Cordero O., y otros (2005). Curso Introductorio, Texto UNA. 2da edición.

Caracas: UNA

200

UNIVERSIDAD NACIONAL ABIERTA

Introducción a la
Ingeniería Industrial

Sólo para uso instruccional


Sin valor comercial

Ingeniería Industrial
UNIVERSIDAD NACIONAL ABIERTA

INTRODUCCIÓN A LA
INGENIERÍA INDUSTRIAL

CÓDIGO: 200
SEMESTRE: I

AUTOR:
MSc. Nanci Sánchez

DISEÑO DE INSTRUCCIÓN
Dr. Antonio Alfonso

CORRECCIÓN
MSc. Fortunato Ricci Méndez

DIAGRAMACIÓN Y MONTAJE
Lic. Alexandra Loginow

TRANSCRIPCIÓN
Johan Paredes
 

Cómo citar este documento:  
 

Sánchez Marchán, Nanci. (2014). Introducción a la Ingeniería Industrial, Texto 
UNA. Caracas: UNA 
 

 
Sánchez Marchán, Nanci. (2014).Introducción a la Ingeniería Industrial, Material Instruccional de Apoyo. Caracas: UNA

ICONOS EMPLEADOS EN EL MATERIAL INSTRUCCIONAL

A lo largo de la lectura de este material encontrará diversos iconos, cuyo


significado se explica a continuación.

AMPLIACIÓN DE CONOCIMIENTOS: está dirigido al estudiante que


desea profundizar más en sus conocimientos en determinado tema.

ATENCIÓN: se presenta cuando se quiere hacer una aclaratoria, una


advertencia o una reflexión sobre algún aspecto del contenido.

CASO DE ESTUDIO: es la exposición de una situación muy similar a la


realidad a la cual se le dará solución.

CONSULTA EN LA WEB: indica referencias a páginas Web

CONSULTA EN OTROS LIBROS: se refiere a un llamado a consulta en


libros que no figuran como textos de carácter obligatorio para el curso.

EJERCICIOS Y ACTIVIDADES PROPUESTAS: son ejercicios o


actividades sugeridas a manera de práctica sobre algún tema de la
unidad.
EJERCICIOS DE AUTOEVALUACIÓN: ejercicios que debe realizar
el estudiante y posteriormente verificar contra los resultados aquí
presentados.
EJEMPLO: es la exposición de un caso alusivo al tema en cuestión y
su resolución.

RECORDATORIO: indica algún aspecto a ser enfatizado, relacionado


con los conocimientos adquiridos previamente por el estudiante.

LECTURAS: indica un texto de carácter obligatorio para la consecución


de los objetivos del curso.

OBJETIVO: indica la finalidad de la unidad, que es lo que se quiere


lograr con el estudio de la misma.

RESPUESTA A EJERCICIOS DE AUTOEVALUACIÓN: presenta la


clave de respuestas a los ejercicios de autoevaluación, de manera que
pue-das reforzar tus conocimientos o corregir cualquier error.
Sánchez Marchán, Nanci. (2014).Introducción a la Ingeniería Industrial, Material Instruccional de Apoyo. Caracas: UNA

CONTENIDO: son los diferentes aspectos que se desarrollarán en el


aparte que se presenta.

TELECLASES: presenta información en formato audiovisual, proporcio-


nada por expertos en la material.

TIEMPO DE ESTUDIO: lapso que se considera suficiente para el


apren-dizaje del objetivo propuesto.

NOTAS MATEMÁTICAS: son informaciones complementarias sobre el


tema que se está tratando.

EVALUACIÓN

8
Sánchez Marchán, Nanci. (2014).Introducción a la Ingeniería Industrial, Material Instruccional de Apoyo. Caracas: UNA

ÍNDICE GENERAL DE CONTENIDO

PP
INTRODUCCIÓN 17
MÓDULO I
UNIDAD Nº 1: LA INGENIERÍA INDUSTRIAL.
CONCEPTOS E HISTORIA DE LA INGENIERÍA INDUSTRIAL
Origen de la ingeniería 23
Aspectos más resaltantes de la ingeniería en las culturas Egipcia, Mesopotámi- 25
ca, Griega, Romana, Oriental y Europea
Origen de la Ingeniería Industrial 28
Influencia de la ciencia, tecnología, técnica y arte en la ingeniería 30
Objetos técnicos 31
Definición de la Ingeniería Industrial 34
Aportes realizados por diferentes personalidades a la Ingeniería Industrial 35
Evolución y futuro de la Ingeniería Industrial 42
Áreas de conocimiento en la práctica moderna de los Ingenieros Industriales 44
La Ingeniería Industrial en Venezuela 45
Regiones de Venezuela donde se dicto la carrera Ingeniería Industrial 46
Misión e importancia de la ingeniería industrial en Venezuela 47
Perfil del Ingeniero Industrial en Venezuela 48
Perfil del TSU en Higiene y Seguridad Industrial en Venezuela 49
Mercado ocupacional del Ingeniero Industrial y del TSU en Higiene y Seguridad 50
Industrial
Valores del Ingeniero Industrial y TSU en Higiene y Seguridad Industrial 53
Ética profesional 53
Relación de la Ingeniería Industrial con otras disciplinas y ramas de la 55
ingeniería
RESUMEN DE LA UNIDAD Nº 1 61
AUTOEVALUACIÓN DE UNIDAD Nº 1 63
EJERCICIOS PROPUESTOS DE LA UNIDAD Nº 1 64

UNIDAD Nº 2: ÁREA OPERATIVA.


PRODUCCIÓN Y PRODUCTIVIDAD EN LA EMPRESA
Definición 73
Eficacia y Eficiencia 73
Clasificación de las empresas 77
Factores de la producción en la empresa 82
Áreas funcionales de la empresa 82
Comparación de funciones en una empresa de transformación y una comercial 84
Organización de la empresa 85
El funcionamiento económico de la empresa. 99
Toma de decisiones en la empresa 99
Cómo crear una empresa 100
PRODUCCIÓN
Origen de la producción 105
Evolución de la producción con el paso de la revolución industrial 105
Definición de proceso, producto y producción 108
Diseño de productos y procesos
Diseño de productos 110
Ciclo de vida de un producto 114
Otras formas de diseño 115
Sánchez Marchán, Nanci. (2014).Introducción a la Ingeniería Industrial, Material Instruccional de Apoyo. Caracas: UNA

Diseño de procesos 116


Etapas del diseño de procesos 117
Tipos de procesos de producción 117
Representación gráfica de los procesos 120
Análisis de procesos 121
Medición de procesos 121
Definición de sistema 123
Organización de un sistema de producción 125
Parámetros de un sistema de producción 127
Otros aspectos importantes a considerar en un sistema de producción 127
Clasificación de los sistemas productivos en atención a su proceso 128
Clasificación de los sistemas productivos sobre la base de su finalidad 129
Sistemas de planificación y control de la producción 130
Planeación y pronósticos de la producción
Pronósticos de la producción 137
Planeación de la producción 145
Inventario 153
Control de la producción 167
Productividad 171
ESTUDIO DEL TRABAJO: MÉTODOS, MOVIMIENTOS Y TIEMPO
Ingeniería de Métodos 179
Estudio de métodos 180
Medición del trabajo 202
CALIDAD EN LA EMPRESA
Concepto y evolución de la calidad 215
Concepto básicos relacionados con la calidad 220
Gestión de la calidad 221
Mejoramiento de la calidad 226
Costos de calidad 269
LOCALIZACIÓN Y DISTRIBUCIÓN EN PLANTA
Origen y definición de una planta industrial 273
Posibles escenarios para la localización de una planta 274
Localización de la planta 274
Pasos para la localización de una planta 276
Métodos de evaluación para la localización 277
Métodos cualitativos 277
Métodos cuantitativos 284
Manejo de materiales 293
El Proceso de logística en la planta 303
SEGURIDAD E HIGIENE EN EL TRABAJO
Origen y Evolución de la Seguridad e Higiene Industrial 307
Seguridad industrial 309
Higiene industrial 315
Programa de Seguridad e Higiene Industrial 323
La ergonomía 326
MANTENIMIENTO INDUSTRIAL DE MAQUINARIA Y EQUIPO
Evolución del mantenimiento 339
Objetivos del mantenimiento 340
Tipos de mantenimiento 340
Plan general de mantenimiento 347
Sánchez Marchán, Nanci. (2014).Introducción a la Ingeniería Industrial, Material Instruccional de Apoyo. Caracas: UNA

PROCESO DE MANUFACTURA
Desarrollo histórico de los procesos de manufactura 351
Concepto de manufactura 351
Materiales utilizados en manufactura 352
Operaciones básicas de manufactura 355
Clasificación de los procesos de manufactura     355
Etapas fundamentales de los procesos de manufactura 358
Energía en los procesos de manufactura 358
Métodos avanzados de manufactura 359
PREPARACIÓN DE PROYECTOS DE INVERSIÓN
Origen de los proyectos 361
Concepto de proyecto de inversión 362
Tipos de proyectos 363
Etapas del proyecto de inversión 364
Ciclo de un proyecto de inversión 366
Evaluación de un proyecto de inversión 366
INVESTIGACIÓN DE OPERACIONES
Origen de la Investigación de Operaciones 373
Aplicación del método científico en la Investigación de Operaciones 374
Definición de la Investigación de Operaciones 374
Ecuaciones 375
Inecuaciones o desigualdades 378
Los modelos matemáticos y su formulación o construcción 378
Usos y beneficios de los modelos de Investigación de Operaciones 382
Clasificación de los modelos de Investigación de Operaciones 383
Aplicación de la Investigación de Operaciones en la Ingeniería Industrial 384
RESUMEN DE LA UNIDAD Nº 2 386
AUTOEVALUACIÓN DE UNIDAD Nº 2 391
EJERCICIOS PROPUESTOS DE LA UNIDAD Nº 2 400

UNIDAD Nº 3: ÁREA DE DESEMPEÑO DE LA INGENIERÍA INDUSTRIAL.


ÁREA DE GERENCIA
Administración de recursos 415
Administración origen y desarrollo 415
Administradores 416
Objetivos de la administración 417
Definiciones de administración según algunos autores 418
Funciones de la administración y gerencia 418
Principios básicos de la administración 420
Teorías de la administración 421
La administración de recursos humanos 424
ECONOMÍA Y FINANZAS
Economía 429
Introducción a la economía 429
Definición de la economía 429
Campos de la economía 430
Microeconomía 430
Macroeconomía 431
Finanzas 433
Sánchez Marchán, Nanci. (2014).Introducción a la Ingeniería Industrial, Material Instruccional de Apoyo. Caracas: UNA

Introducción a las finanzas 433


Tipos de finanzas 434
Planeación financiera personal 434
Planeación financiera pública 436
Planeación de finanzas corporativas 437
Valor del dinero en el tiempo 442
Costos de la inversión 451
Contribución marginal y punto de equilibrio 456
Estados financieros 465
Depreciación 470
MERCADO Y MARKETING INDUSTRIAL
Origen del mercado 473
Definición de mercado 473
Tipos de mercado 474
Segmentación de mercado 475
Mercado meta 475
Origen del término marketing 476
Definición de marketing 477
Algunos elementos que han contribuido al desarrollo del marketing 478
Nueva visión del marketing 479
Marketing operativo o marketing mix 479
Plan de marketing, mercadeo o mercadotecnia 480
COMPORTAMIENTO ORGANIZACIONAL
Origen del comportamiento organizacional 491
Distribución del comportamiento organizacional 492
El individuo y las bases de su conducta 492
Bases de la conducta de grupos y equipos 499
Poder 505
Bases de la estructura de la organización 506
Diseño de la tecnología del trabajo 508
Políticas y prácticas de recursos humanos 508
Cultura organizacional 508
Clima organizacional 509
Diagnóstico y diseño de organizaciones 510
La comunicación en la organización 511
RESUMEN DE LA UNIDAD Nº 3 516
AUTOEVALUACIÓN DE UNIDAD Nº 3 518
EJERCICIOS PROPUESTOS DE LA UNIDAD Nº 3 522

UNIDAD N° 4: DISCIPLINAS AFINES A LA INGENIERÍA INDUSTRIAL


ESTADÍSTICA
Introducción a la estadística 529
División de la estadística 529
Etapas del Método estadístico 531
Importancia de la estadística para la Ingeniería Industrial 536
INGENIERÍA DE SISTEMAS
Introducción a la Ingeniería de Sistemas 539

12
Sánchez Marchán, Nanci. (2014).Introducción a la Ingeniería Industrial, Material Instruccional de Apoyo. Caracas: UNA

Influencia de la computadora e Ingeniería de Sistemas para la ingeniería 540


industrial y sus áreas
Definición de la Ingeniería de Sistemas 540
La Ingeniería de Sistemas en la actualidad 542
Elementos y vistas de un sistema 543
Sistema de información 545
Sistemas de información más utilizados para la gestión por la ingeniería indus- 548
trial
Algunos softwares de aplicación usados en la ingeniería industrial 549
CIENCIAS DE LA DECISIÓN
Origen y definición de la toma de decisiones 553
Modelos de criterios de decisión 554
Componentes de la decisión 554
Etapas de la toma de decisión para dar solución a un problema 555
Importancia de la toma de decisiones 558
Técnicas para la toma de decisión en grupo 558
Modelos y técnicas usadas en la toma de decisiones en las Diferentes áreas 559
de la Ingeniería Industrial
RESUMEN DE LA UNIDAD 4 565
AUTOEVALUACIÓN DE LA UNIDAD 4 566
EJERCICIOS PROPUESTOS 569

ÍNDICE DE FIGURAS Y TABLAS


Figuras
Figura 1. Transporte sobre rodillos 23
Figura 2. Nivel primitivo 24
Figura 3. Herramientas primitivas 24
Figura 4. Topo excavando un túnel 24
Figura 5. Etapas del desarrollo de los objetos técnicos 32
Figura 6. Relación de la ingeniería industrial con la ciencia, tecnología, técnica y 33
arte
Figura 7.Bases estructurales de la ingeniería industrial 34
Figura 8. Etapas de la evolución de la Ingeniería Industrial 43
Figura 9. Mapa de Venezuela con la ubicación de las regiones donde se dicta la 47
carrera Ingeniería Industrial
Figura 10. Áreas de desempeño de la Ingeniería Industrial 67
Figura 11. Medida de la eficacia y eficiencia en la organización 75
Figura 12. Áreas funcionales de la empresa 82
Figura 13. Comparación de funciones de una empresa de transformación y una 84
comercial, adaptación del texto contabilidad de costos de García
Colín
Figura 14. Tipos de organización atendiendo a su estructura 86
Figura 15. Estructura y niveles de la organización 87
Figura 16. Costos generados en una empresa 100
Figura 17. Proceso de desarrollo de nuevos productos 111
Figura 18. Generación de la idea en el diseño de un producto 111
Figura 19. Selección de la idea para el diseño de un producto 112
Figura 20. Diseño preliminar de un producto 113
Figura 21. Ciclo clásico de la vida de un producto 114

13
Sánchez Marchán, Nanci. (2014).Introducción a la Ingeniería Industrial, Material Instruccional de Apoyo. Caracas: UNA

Figura 22. Características de las etapas de la vida de un producto 114


Figura 23. Esquema de un proceso de producción 116
Figura 24. Tipos de procesos productivos 118
Figura 25. Tipos de sistemas 123
Figura 26. Representación gráfica de un sistema general de producción 125
Figura 27. Sistema de producción MRP 135
Figura 28. Caracterización de los conceptos de pronóstico y planeación de la 137
producción
Figura 29. Resumen de técnicas cuantitativas de pronóstico 142
Figura 30. Elementos de la planeación de la producción 151
Figura 31. Distribución de los modelos de inventario 160
Figura 32. Comportamiento del modelo básico de inventario 161
Figura 33. Estructura de costos del modelo básico de inventario 162
Figura 34. Manejo de inventarios 167
Figura 35. Técnicas de control de la producción 169
Figura 36. Factores necesarios para lograr el control de la producción 170
Figura 37. Fases principales de la planificación y el control de la producción 170
Figura 38. Ramas del estudio del trabajo 180
Figura 39. Diagrama de las incógnitas que pueden surgir al analista de métodos. 181
Figura 40. Ritmo de trabajo de un trabajador calificado 202
Figura 41. Estructura de un tiempo tipo o estándar 208
Figura 42. Organización de las técnicas de la medición del trabajo 214
Figura 43. Perspectivas externa e interna integradas en una perspectiva global 219
Figura 44. Interrelación entre las fases históricas de la Gestión de la Calidad. 222
Figura 45. Ciclo de Deming, llamado también modelo PDCA 227
Figura 46. Representación general de un histograma 230
Figura 47. Representación general de un diagrama de flujo 235
Figura 48. Representación general de un diagrama de causa y efecto 237
Figura 49. Factores triviales y vitales 240
Figura 50. Representación general de un diagrama de PARETO 240
Figura 51. Representación general de un diagrama de correlación 244
Figura 52.Resumen de gráficos de control por variables 248
Figura 53.Clasificación de gráficos de control por atributo 248
Figura 54. Representación general de un diagrama de control 249
Figura 55. Resumen de las siete herramientas básicas de la calidad 260
Figura 56. Representación general de un diagrama de afinidad 262
Figura 57. Representación general de un diagrama de relaciones 262
Figura 58. Representación general de un diagrama de árbol 263
Figura 59. Representación general de un diagrama matricial 263
Figura 60. Representación general de un diagrama de priorización 264
Figura 61. Representación general de un diagrama de proceso de decisión 264
Figura 62. Representación general de un diagrama de flechas 264
Figura 63. Resumen de nuevas herramientas de la calidad 266
Figura 64. Modelo tradicional de los costos de calidad 271

14
Sánchez Marchán, Nanci. (2014).Introducción a la Ingeniería Industrial, Material Instruccional de Apoyo. Caracas: UNA

Figura 65. Representación gráfica de una Matriz REL 282


Figura 66. Proceso de logística 306
Figura 67. Tipos de mantenimiento 341
Figura 68. Fases del proyecto de inversión 364
Figura 69. Ciclo de un proyecto de inversión 366
Figura 70. Modelos de Investigación de Operaciones 383
Figura 71. Interrogantes referentes a las acciones preestablecidas de la 416
administración
Figura 72. Diagrama de flujo de caja 449
Figura 73. Gráfico del punto de equilibrio 458
Figura 74. Elasticidad de la demanda 459
Figura 75. Fórmula para generar el precio de venta 463
Figura 76. Contenido de un plan de mercadeo 481
Figura 77. Distribución del comportamiento organizacional 492
Figura 78. Teoría de la jerarquía de las necesidades (Abraham Maslow) 497
Figura 79. Modelo de grupo de características que muestra la eficacia de un 501
equipo
Figura 80. Representación gráfica de la población y muestra 532
Figura 81. Sistema propuesto por Rechtin 544
Figura 82. Los sistemas de información y sus niveles 546

Tablas
Tabla 1. Aspectos más resaltantes de la ingeniería en las culturas Egipcia, 26
Mesopotámica, Griega, Romana, Oriental y Europea.
Tabla 2. Personajes fundamentales y sus aportes a la Ingeniería Industrial 36
Tabla 3. Áreas del sector público y sector privado 51
Tabla 4. Relación de la Ingeniería Industrial con otras disciplinas 56
Tabla 5. Relación que tiene la Ingeniería Industrial con otras ramas de la 59
ingeniería
Tabla 6. Relación entre eficacia y eficiencia 74
Tabla 7. Tamaño y límite numérico de la empresa 79
Tabla 8. Simbología utilizada para la construcción de los organigramas 88
Tabla 9. Tipos de organigrama, descripción y representación gráfica 89
Tabla 10. Tipos de diseño de estructura; definición, ventajas, desventajas y 93
ejemplo de cada uno de ellos
Tabla 11. Etapas evolutivas de la producción sobre la base de la revolución 106
industrial
Tabla 12. Diferencias entre sistemas cerrados y sistemas abiertos 124
Tabla 13. Ejemplos de sistemas de producción (primario, secundario, terciario) 129
Tabla 14. Técnicas cualitativas de pronóstico 139
Tabla 15. Definición de las técnicas cuantitativas de pronóstico 140
Tabla 16. Tipos de planificación 145
Tabla 17. Resumen de las fases de un sistema de planificación 152
Tabla 18. Modelo básico de inventario (EOQ): símbolos, fórmulas y significado 163
Tabla 19. Descripción y fórmulas utilizadas en los modelos derivados del modelo 165
EOQ.
Tabla 20. Avance en las definiciones de la productividad y sus creadores. 172

15
Sánchez Marchán, Nanci. (2014).Introducción a la Ingeniería Industrial, Material Instruccional de Apoyo. Caracas: UNA

Tabla 21. Símbolos a utilizar en la construcción de un diagrama de las 187


operaciones del proceso
Tabla 22. Símbolos para elaboración del diagrama de Flujo del Proceso 190
Tabla 23. Conjunto de estándares de símbolos para diagramas de proceso, 191
adaptación según la ASME
Tabla 24. Símbolos a utilizar en el diagrama bimanual 196
Tabla 25. Resumen de los Therbligs 198
Tabla 26. Factor de calificación de los elementos de una tarea según ritmo de 213
trabajo
Tabla 27. Evolución de la definición de calidad 217
Tabla 28. Patrones de gráficos de barra para comparar resultados de histogramas 231
Tabla 29. Significado y descripción de símbolos para elaboración del diagrama de 234
flujo (simbología ASME)
Tabla 30. Posibles patrones para comparar el grado de correlación 245
Tabla 31. Descripción de las variables usadas en los gráficos de control por Vari- 251
able
Tabla 32. Constantes para gráficos de control por variable 252
Tabla 33. Descripción de las variables de los gráficos de control por atributos 256
Tabla 34. Ventajas y desventajas de los gráficos de control 259
Tabla 35.Funciones de las herramientas básicas de la calidad 260
Tabla 36. Nomenclatura y codificación de la matriz de relaciones (REL) 281
Tabla 37. Dispositivos para el manejo de materiales 296
Tabla 38. Personajes y comentarios de los aportes que impulso la evolución de la 308
Seguridad e Higiene Industrial
Tabla 39. Enfermedades laborales y los agentes que la causan 316
Tabla 40. Los trabajadores expuestos a enfermedades pulmonares 317
Tabla 41. Clasificación de las aplicaciones de la robótica en el sector 357
manufacturero
Tabla 42.Procesos comunes de manufactura y sus requerimientos de energía 359
Tabla 43. Técnicas, métodos de solución, características y aplicación de la (IO) en 384
la Ingeniería Industrial
Tabla 44. Aspectos más resaltantes de las teorías clásicas de la administración, 422
sus enfoques y énfasis
Tabla 45. Teorías más recientes de la administración 424
Tabla 46. Funciones de las áreas de RRHH 425
Tabla 47. Simbología y fórmulas utilizadas en la Ingeniería Económica 450
Tabla 48. Diferencias entre costos y gastos 455
Tabla 49. Dimensiones de las habilidades intelectuales de la conducta de los 493
individuos
Tabla 50. Habilidades físicas 493
Tabla 51. Diferencia entre los conceptos de poder y liderazgo 505
Tabla 52. Algunas aplicaciones de la estadística en diferentes áreas de la 537
Ingeniería Industrial
Tabla 53. Modelos para las vistas del sistema de Rechtin 545
Tabla 54. Modelos y técnicas utilizadas en la toma de decisión en las diferentes 560
áreas de Ingeniería Industrial

16
Sánchez Marchán, Nanci. (2014).Introducción a la Ingeniería Industrial, Material Instruccional de Apoyo. Caracas: UNA

INTRODUCCIÓN

La ingeniería se considera como el arte de transformar la naturaleza para uso y bene-


ficio del hombre. Así, dentro de su desarrollo histórico se dieron, en primer lugar, las inge-
nierías unidas a elementos físicos tangibles, tales como la de minas y la civil, disciplinas que
modifican la naturaleza para obtener beneficios de sus recursos y la infraestructura necesa-
ria para su desarrollo. Con la aparición de la energía eléctrica, surge la Ingeniería Mecánica
y la Ingeniería Eléctrica que transforman las grandes fuentes de energía naturales para uso
y conveniencia del hombre.

En este desarrollo histórico, la Ingeniería Industrial es una de las últimas que se da y


surge como una necesidad de integrar los recursos humanos, materiales y económicos para
lograr una mejor y mayor productividad con calidad. El ingenio del hombre lo ha llevado a
buscar la máxima efectividad con el mínimo esfuerzo. La necesidad de integrar los recursos
humanos, materiales y económicos es el origen de este deseo de eficiencia del esfuerzo.

La Ingeniería Industrial surge durante el proceso de la transformación de la produc-


ción artesanal a la industrial, durante el siglo XVII. En este cambio adquieren significado
tres conceptos que forman la base de esta disciplina: organización, trabajo productivo y
tiempo. Los estudios del trabajo, la creación de nuevas formas de organización y el mejor
aprovechamiento del tiempo constituyen un nuevo campo de estudio que recibe el nombre
de Ingeniería Industrial; originada porque tales actividades se llevaron a cabo, precisamente,
en la organización más importante de esa época: la industria.

En un principio la disciplina estaba dirigida sólo a la planta de producción, por otra par-
te se notaba ya la inclusión de la aplicación de los principios de la administración en toda
organización. También se consideraban otros factores que salían fuera del alcance de la
administración, como era la iluminación, la ergonomía, la productividad, inventarios en todos
los procesos, etc. Todos estos aspectos se relacionaban constantemente con la gerencia.
Así surgían planteamientos como: el uso mínimo de trabajo con el mayor aprovechamiento
posible, hacer más con menos, hacer más con lo mismo, etc.

Sin duda alguna hoy el avance tecnológico ha marcado aún más de manera decisiva el
desarrollo de la Ingeniería industrial, constituyéndola en referencia del crecimiento de esta
época, aspecto que demuestra la importancia de sustentarla en nuevos principios diferencia-
dores como: la revolución del conocimiento, gestión del talento, la tecnología e innovación.
Esta sinergia operacional que constituye la competitividad sostenible de las organizaciones
marca el accionar y el grado de desempeño con el cual las empresas, de cualquier natura-
leza, participen en la dinámica pujante de la competencia global y regional. Entonces, co-
rresponde a las universidades del país, sin importar la modalidad en la cual se desempeñan;

17
Sánchez Marchán, Nanci. (2014).Introducción a la Ingeniería Industrial, Material Instruccional de Apoyo. Caracas: UNA

contribuir de manera decidida a la generación y desarrollo de profesionales, que puedan


estimular los sectores potenciales de la nación, mediante una continua investigación y ac-
tualización. En este sentido, resulta encomiable que en la carrera Ingeniería Industrial de la
Universidad Nacional Abierta (consciente de su rol fundamental) se asuma la real identidad
de fomentar la vocación estudiantil universitaria a través de la incorporación de materiales
instruccionales actualizados, que permitan promover la formación de ingenieros y TSU en
sus áreas, con alta capacidad creativa e innovadora, capaces de romper los paradigmas
tradicionales, y no de ser repetidores de los principios mecanicistas de la ciencia. Lograr
profesionales que generen aportes significativos en bien de la comunidad y la sociedad en
general.

En este texto se ofrece al estudiante, que inicia la carrera de Ingeniería Industrial (cód
280) y al Técnico Superior en Higiene y Seguridad Industrial (Cód 281) de la Universidad
Nacional Abierta (UNA) un panorama global sobre el contenido de las diferentes áreas que
contempla el estudio de su carrera, así como una visión genérica del funcionamiento de una
empresa, considerando en ella el equilibrio de los elementos que componen sus sistemas
para hacerlos más eficaces y eficientes; se citan los cambios que han experimentado duran-
te los últimos años con el uso de las nuevas tecnologías. El estudiante requiere un acerca-
miento específico de cómo la empresa se organiza, acorde a sus objetivos y establecimiento
de funciones en relación directa con las actividades del personal. De tal manera que siempre
se oriente hacia una mayor productividad, ya que es precisamente en una organización
productora de bienes o servicios donde debe ejercer su actividad profesional, optimizando
los recursos.

En este orden de ideas se desarrolla en este texto un módulo: el cual contiene cuatro
unidades, en la primera se describe una breve historia de la ingeniería; origen, evolución y
desarrollo de la Ingeniería Industrial; se destaca también en esta unidad el perfil, los valores
y ética que debe poseer el profesional de estas carreras.

La segunda unidad contempla el Área Operativa, donde se presentan los temas de: pro-
ducción y productividad; estudio de métodos, movimientos y tiempo; calidad en la empresa;
seguridad e higiene en el trabajo; mantenimiento industrial de maquinaria y equipo; proceso
de manufactura; evaluación económica de proyectos; localización y distribución en plantas,
investigación de operaciones.

En la tercera unidad se desarrolla el Área de Gerencia, que abarca temas como: admi-
nistración de recursos, economía y finanzas, marketing industrial, comportamiento organiza-
cional.

Por último la unidad cuatro, en la que se puede observar algunas de las disciplinas de
apoyo a la Ingeniería Industrial, como son: estadística, ciencias de la decisión e Ingeniería de
Sistemas, disciplinas importantes para el desarrollo de la Ingeniería Industrial. La estructura
completa del texto se puede observar en la siguiente tabla:

18
Sánchez Marchán, Nanci. (2014).Introducción a la Ingeniería Industrial, Material Instruccional de Apoyo. Caracas: UNA

ESTRUCTURA GENERAL DEL TEXTO INTRODUCCIÓN A LA INGENIERÍA


INDUSTRIAL (cód. 200)

MÓDULO TÍTULO UNIDAD CONTENIDO


LA INGENIERÍA
INDUSTRIAL CONCEPTOS E HISTORIA DE LA INGENIE-
UNIDAD 1
RÍA INDUSTRIAL

UNIDAD 2 ÁREA OPERATIVA

ÁREAS DE
DESEMPEÑO
DE LA
MÓDULO I INGENIERÍA
INDUSTRIAL UNIDAD 3
ÁREA DE GERENCIA

DISCIPLINAS • ESTADÍSTICA
AFINES A LA UNIDAD 4
INGENIERÍA • INGENIERÍA DE SISTEMAS
INDUSTRIAL
• CIENCIAS DE LA DECISIÓN

En síntesis este texto recoge el origen, los avances y el desarrollo futuro que se vislum-
bra para las diferentes áreas de esta disciplina, a través del uso de las nuevas Tecnologías,
tanto de la Información y Comunicación (TIC) como la Tecnología Industrial (TI); logrando
que los estudiantes de Ingeniería Industrial y TSU en Higiene y Seguridad Industrial, se moti-
ven y tengan un panorama global de los conocimientos que requieren para enfrentarse como
futuros profesionales, al campo laboral.

19
Sánchez Marchán, Nanci. (2014).Introducción a la Ingeniería Industrial, Material Instruccional de Apoyo. Caracas: UNA

MÓDULO I

UNIDAD 1

LA INGENIERÍA INDUSTRIAL

Objetivo:
Identificar, a través del tiempo, la influencia de la ciencia y la tecnología para el
desarrollo de la Ingeniería Industrial.

CONCEPTOS E HISTORIA DE LA INGENIERÍA INDUSTRIAL

•• Origen de la ingeniería.
•• Origen de la Ingeniería Industrial.
•• Origen de la Higiene y Seguridad Industrial.
•• Influencia de la ciencia, tecnología, técnica y arte en la ingeniería.
•• Objeto técnico.
•• Aspectos más importantes de los aportes realizados por diferentes per-
sonalidades a la Ingeniería Industrial.
•• Evolución y futuro de la Ingeniería Industrial.
•• Áreas de conocimiento en la práctica moderna de los Ingenieros Indus-
triales.
•• La Ingeniería Industrial en Venezuela.
•• Perfil del Ingeniero Industrial y TSU en Higiene y Seguridad Industrial
en Venezuela.
•• Mercado ocupacional del Ingeniero Industrial y TSU en Higiene y Segu-
ridad Industrial.
•• Ética profesional.
•• Valores del Ingeniero Industrial y TSU en Higiene y Seguridad Industrial.
Sánchez Marchán, Nanci. (2014).Introducción a la Ingeniería Industrial, Material Instruccional de Apoyo. Caracas: UNA

CONCEPTOS E HISTORIA DE LA INGENIERÍA INDUSTRIAL


En esta unidad se reseña el origen de la ingeniería como disciplina general, resaltando
los aspectos más importantes que estimularon la diversificación y aparición, a medida que
se fue requiriendo, de las diferentes ramas que la conforman; tal es el caso de la Ingeniería
Industrial, cuyo origen se relaciona con la revolución industrial, o cambio de la producción
artesanal a la industrial.

Asimismo se presentan aquí, los aportes más resaltantes de personajes como: Adam
Smith, Frederick W.Taylor, Henry L. Gant, los esposos Gilbreth, Henry Fayol, Henry Ford,
entre otros; que impulsaron el desarrollo de la Ingeniería Industrial. Por otra parte, se muestra
el florecimiento de esta disciplina en Venezuela, su evolución y futuro en el mundo.

El futuro Ingeniero Industrial y TSU en Higiene y Seguridad Industrial, además de adqui-


rir los conocimientos que fundamentan su disciplina, necesitan conocer el perfil ocupacional
a obtener, sobre la base de las necesidades empresariales; el mercado donde se pueden
desempeñar como profesionales; fortalecer los valores y la ética que deben prevalecer en
ellos, durante el ejercicio de su profesión. Todos estos aspectos están contemplados en esta
unidad.

ORIGEN DE LA INGENIERÍA

Antes de sumergirnos en el mundo de la Ingeniería Industrial, es importante conocer de


manera general, los hechos más resaltantes de la ingeniería como disciplina general, que
permita visualizar los orígenes de esta rama de estudio. Con lo anterior se logra tener un
panorama de las bases que sustentaron la aparición de la Ingeniería Industrial, además de
los aportes logrados por diferentes personalidades.

Se puede hablar de ingeniería desde el primer instante en que se dio forma a una piedra
para convertirla en una herramienta, o cuando los primeros seres humanos usaron la energía
de forma consciente para encender una hoguera. Desde entonces, el desarrollo de la inge-
niería ha caminado en paralelo con el de la humanidad.

Es evidente que la ingeniería adquiere un gran


adelanto con la revolución agrícola (año 8000 A.C.),
cuando los hombres dejaron de ser nómadas, y vivie-
ron en un lugar fijo para poder cultivar las plantas que
producían los frutos que consumían y criar animales
comestibles. Hacia el año 4000 A.C; con los asen-
tamientos alrededor de los ríos  Nilo, Éufrates e Indo,
se centralizó la población y se inició la civilización con
escritura y gobierno. Con el tiempo en esta civilización Figura 1. Transporte sobre rodillos
aparecería la ciencia. Extraído del texto Pablo Grech. 2000.
Introducción a la Ingeniería

23
Sánchez Marchán, Nanci. (2014).Introducción a la Ingeniería Industrial, Material Instruccional de Apoyo. Caracas: UNA

Las grandes construcciones de la antigüedad


asombran aún en la actualidad, por la exactitud de sus
medidas. El movimiento de grandes pesos tanto longi-
tudinal como verticalmente en esas construcciones, no
hubiera sido posible sin contar con: palancas, poleas,
rodillos, vehículos sobre ruedas, etcétera. (Véase figura
1). Figura 2. Nivel primitivo
Extraído del texto Pablo Grech.
Las grandes Pirámides Egipcias las ciudades me- 2000. Introducción a la Ingeniería
sopotámicas, la Muralla China, así como los templos
construidos por las civilizaciones Maya, Azteca e Inca
revelan que los antepasados de esas civilizaciones po-
seían instrumentos que permitían elevar sus edificios
verticalmente, asentarlos sobre terrenos planos, así
como, controlar las longitudes de manera que se logra-
ran efectos espectaculares, como los que se observa-
ban periódicamente en el templo egipcio de Abbu Sim-
bal. El sol entraba exactamente algunos días del año e Figura 3.
iluminaba la faz de Ramsés III. Los egipcios desarro- Herramientas primitivas. Extraí-
llaron una herramienta para medir la horizontalidad de do del texto Pablo Grech. Intro-
ducción a la Ingeniería
sus construcciones: consistía en un marco en forma de
A, de cuyo vértice pendía una plomada. Los dos pies de
la A se localizaban sobre la superficie del terreno; si la
plomada bisectaba el ángulo de la superficie del terre-
no, era horizontal. Este instrumento se usó en Europa
hasta mediados del siglo XIX.(Véase figura 2)

Tampoco la construcción de las grandes obras de


la ingeniería actual se hubiera logrado sin contar con
máquinas adecuadas para ello. Hay una gran distancia
entre las toscas herramientas (véase figura 3) usadas Figura 4. Topo excavando un túnel
por el hombre primitivo para cortar las pieles, construir Extraída del texto Pablo Grech.
sus arcos, etcétera, y el topo usado para taladrar el tú- 2000. Introducción a la Ingeniería
nel submarino que unió a Francia con Inglaterra (Véase
figura 4)

La ingeniería surgió desde el primer momento en el que se dio forma a una pie-
dra para convertirla en una herramienta o cuando los primeros humanos usaron
la energía de forma consciente al encender una hoguera, es tan antigua como el
hombre, es la profesión que clarifica los sueños y acoge los ingenios de todo tipo.

24
Sánchez Marchán, Nanci. (2014).Introducción a la Ingeniería Industrial, Material Instruccional de Apoyo. Caracas: UNA

Se puede definir la ingeniería como:

La profesión que aplica conocimientos y experiencias para que mediante diseños, mode-
los y técnicas se resuelva problemas que afectan a la humanidad. Se caracteriza por uti-
lizar principalmente el ingenio de una manera más experta y ágil a través del uso del mé-
todo científico, puesto que está limitada en cuanto a tiempo y recursos en los proyectos.

Las personas que se dedican a la ingeniería reciben el nombre de ingenieros. El tér-


mino ingeniero fue utilizado por los constructores italianos, para denotar el ingenio de los
fabricantes de máquinas, en aquella época fundamentalmente máquinas de guerra; pero
realmente este significado es original de los países de habla inglesa, mientras que en cas-
tellano y lenguas similares procede del latín ingenioso, en un sentido de capacidad mental
e innovación. El término evolucionó más adelante para incluir todas las áreas en las que se
utilizan técnicas para aplicar el método científico. Por ello el ingeniero se puede definir como:

El profesional que utiliza el conocimiento de las ciencias, la técnica y la experiencia pro-


pia para encontrar las mejores soluciones a problemas concretos, creando los modelos
matemáticos adecuados a los problemas, que le permitan analizarlos rigurosamente y
probar sus soluciones potenciales.

Si existen múltiples soluciones razonables, los ingenieros evalúan las diferentes opcio-
nes de diseño sobre la base de sus cualidades y eligen la solución que mejor se adapte a
las necesidades.

Los ingenieros tratan de probar si sus diseños logran sus objetivos antes de pro-
ceder a la producción en cadena. Para ello, emplean entre otras cosas prototipos,
modelos a escala, simulaciones, pruebas destructivas y pruebas de fuerza. Las
pruebas aseguran que los artefactos funcionarán como se había previsto.

Aspectos resaltantes de la historia de la ingeniería en las culturas: Egipcia,


Mesopotámica, Griega, Romana, Oriental y Europea
Se consideran a continuación los aspectos más resaltantes de las culturas antes men-
cionadas, con la finalidad de tener una visión global de la influencia de tales civilizaciones en
el adelanto de la ingeniería en pro de la búsqueda de supervivencia, protección y bienestar
del individuo (ver tabla 1).

25
Tabla 1.

Aspectos más resaltantes de la ingeniería en las culturas Egipcia, Mesopotámica, Griega, Romana, Oriental y Europea.

CULTURA Y COMENTARIO
FECHA

Los egipcios realizaron algunas de las obras más grandiosas de la ingeniería de todos los tiempos, siendo
INGENIERÍA una de las más antiguas el muro de la ciudad de Menfis. El arquitecto real de Menfis fue Kanofer, tuvo un
EGIPCIA hijo a quien llamó Imhotep, los historiadores lo consideraron como el primer ingeniero.
3000-1999 A. La construcción de pirámides realmente era algo notable, si se considera que no se conocían ni el tomillo ni
de C. la polea, no había otro mecanismo que la palanca. Construyeron diques, canales, y contaban con sistemas
complejos de irrigación

INGENIERÍA Los habitantes de Mesopotamia (tierra entre dos ríos) aprendieron a irrigar sus tierras y a amurallar sus
MESOPOTÁ- ciudades, fijaron su atención sobre todo a la construcción de templos.
MICA

26
2000-699 A.
de C.

INGENIERÍA Construyeron templos en la Acrópolis, monte rocoso que miraba a la ciudad de Atenas. Las vigas de már-
GRIEGA mol del cielo raso de esta estructura estaban reforzadas con hierro forjado, lo que constituye el primer uso
conocido del metal como componente en el diseño de un edificio.
700-509 a. de
C.

La ingeniería romana era civil, especialmente en el diseño y construcción de obras permanentes tales
INGENIERÍA
como: acueductos, carreteras, puentes y edificios públicos. Una excepción fue la ingeniería militar, y otra
ROMANA
menor, por ejemplo, la galvanización.
510-399 A.
La ingeniería romana fue cambiando sus métodos poco a poco, una vez que daban con una buena técnica,
de C
no buscaban inmediatamente otro método más sencillo o más barato para reemplazarlo.
Sánchez Marchán, Nanci. (2014).Introducción a la Ingeniería Industrial, Material Instruccional de Apoyo. Caracas: UNA
Una de las más grandes realizaciones de todos los tiempos fue la Gran Muralla de China. La distancia de un
extremo a otro del muro es de aproximadamente 2 240 Km.; sin embargo, hay más de 4 080 Km. de muro
en total. También construyeron el canal más largo, el Yunho o Gran Canal, tiene 1 920 Km. y corre desde
INGENIERÍA Tientsin hasta Hangchow; su construcción requirió más de mil años.
ORIENTAL Otro descubrimiento importante de los chinos fue la brújula, que rápidamente se extendió, para ser de uso
común alrededor de 1200 D. de C.
400-299 A. de Los chinos fueron de los primeros constructores de puentes, con características únicas. Algunos de sus
C. puentes más antiguos fueron de suspensión.
Se cree que los chinos inventaron la pólvora. Es irónico que esta invención china, junto con el cañón, eli-
minara la muralla. Los chinos desarrollaron maquinaria de engranaje desde fechas muy antiguas. Algunos
historiadores creen que hacia el año 400 A. de C., habían engranajes en China.

Durante este periodo no existieron las profesiones de ingeniero o arquitecto, de manera que esas activida-
des quedaron en manos de los artesanos, tales como los albañiles maestros, la mayoría de las personas
debía tener el mismo oficio que sus  padres. A este período se le denominó oscurantismo y se encuentra
entre los años 600 y 1000 D. de C.

27
INGENIERIA
Sin embargo, en la centuria de 1500 ocurrió una serie de descubrimientos científicos importantes en la
EUROPEA
ingeniería y matemáticas, una revolución en el razonamiento con relación a la naturaleza y actividad de la
materia. El movimiento, fuerza y gravedad recibieron considerable atención en plena Edad Media y más
adelante.
300 A de C
hasta 1500 D. La invención de los anteojos en 1286, y el incremento considerable en las obras impresas en Europa en el
de C. siglo XV, fueron dos acontecimientos trascendentales en la expansión del pensamiento ingenieril. Desde
luego, otro factor importante en todo momento fue la actitud de una sociedad hacia la profesión. Durante el
Renacimiento, los ingenieros nuevamente fueron miembros de una profesión respetada e incluso algunos
de ellos recibieron buena paga.
La República de Venecia estableció en 1474 la primera ley de patentes, y en 1594 se dio a Galileo una pa-
tente sobre un dispositivo para elevar el agua.
Sánchez Marchán, Nanci. (2014).Introducción a la Ingeniería Industrial, Material Instruccional de Apoyo. Caracas: UNA
Sánchez Marchán, Nanci. (2014).Introducción a la Ingeniería Industrial, Material Instruccional de Apoyo. Caracas: UNA

En las culturas estudiadas anteriormente se puede vislumbrar que la mayoría de los


campos de actividad económica recibieron grandes aportes de la ingeniería, como: la agri-
cultura, la minería, la construcción y obras públicas, el transporte, entre otros.

Cabe destacar que la ingeniería como disciplina general ha legado también grandes
aportes a sus diferentes ramas; como en el caso de la Ingeniería Industrial: para la cual el
sistema humano constituyó el aspecto más importante en su desarrollo, por ello se le deno-
minó también Ingeniería de los Sistemas de la Actividad Humana; no obstante, es bien
conocido que ésta se formó de manera tardía con respecto a otras ramas, las cuales se
producían de forma experimental o práctica con el transcurrir del tiempo.

La Ingeniería Industrial tiene sus inicios en los talleres y fábricas, donde la aplicación
del método científico se da dentro de los sistemas y la ciencia; aquí toma el nombre de In-
geniería Industrial, por su papel en la industria. A continuación se muestra de manera más
detallada, el interesante origen de la Ingeniería Industrial.

ORIGEN DE LA INGENIERÍA INDUSTRIAL

Antes de la revolución industrial, una gran cantidad de personas trabajaban en el cam-


po y en otras tareas de naturaleza artesanal. Con la aparición de la industrialización se
comienzan a utilizar máquinas que para ser accionadas empleaban la energía del agua, del
viento o de los animales; siendo al mismo tiempo necesario mucho esfuerzo humano para
la realización de todas estas actividades propias de fabricación; lo que propició la búsqueda
de una alternativa que permitiera disminuir dicho esfuerzo. Se genera entonces la revolución
industrial.

La Revolución Industrial no es otra cosa que el paso de una economía agraria y arte-
sanal a otra dominada por la industria y la mecanización. La expresión Revolución Industrial
fue sugerida por el historiador Arnold Toynbee, para referirse al desarrollo económico britá-
nico entre los años 1760 y 1840.

Los cambios generados por este acontecimiento de la época, se pueden considerar


como: tecnológicos, socioeconómicos y culturales. Los tecnológicos incluyen el uso de nue-
vos materiales como el hierro y el acero; de nuevas fuentes de energía como el carbón,
nuevas fuerzas motrices como la máquina a vapor creada en 1785, la cual trae consigo el
desarrollo de dispositivos que utilizan esta energía. Se inventaron nuevas máquinas para hi-
lar o para tejer, que  permitieron un mayor aumento de la producción con un mínimo gasto de
energía humana. Surgiría una nueva forma de organización de la producción, que soportaría
la división del trabajo y una mayor especialización de la mano de obra. También deben des-
tacarse las importantes mejoras de los transportes como trenes, barcos de vapor y la cre-
ciente interacción entre la ciencia y la industria. Estos cambios tecnológicos supondrán un
acelerado incremento del uso de recursos naturales y de la producción en masa de bienes
manufacturados. La tierra ya no es considerada como la principal fuente de riqueza, debido
al creciente papel que tomó la industria y el comercio internacional.

28
Sánchez Marchán, Nanci. (2014).Introducción a la Ingeniería Industrial, Material Instruccional de Apoyo. Caracas: UNA

Entre los cambios sociales y culturales son destacables: las mejoras en la agricultura
que producirá un crecimiento en la población urbana, el desarrollo del movimiento obrero,
el espectacular crecimiento de los conocimientos científicos, técnicos y tecnológicos, que
aceleraron la mecanización, la cual comenzó a considerarse como la principal fuerza de
trabajo de la Revolución Industrial, hecho que fue de interés para científicos, ingenieros y
especialistas de varias disciplinas en cuestión de productividad.

Actividad propuesta
¿Cuáles son a su juicio los cambios más importantes logrados con la Revolución
Industrial? Argumente su respuesta.

Sobre la base de los acontecimientos descritos se puede decir entonces, que el origen
de la Ingeniería Industrial coincide con la Revolución Industrial, donde se desarrollaron más
los aspectos como: la ciencia, la técnica, la tecnología y el arte. En aquellos años muy
pocos gerentes o dueños de empresas se preocupaban de las condiciones de trabajo y
salarios de los obreros que se encontraban a su servicio. El salario que recibía un obrero, a
menudo correspondía a trabajos a destajo; estipulándose un precio a cada pieza u objeto y
pagándosele el salario de acuerdo con lo que hubiese producido éste; motivo que demandó
del hombre ponerse en contacto con los problemas de la dirección del taller o de la fábrica
y, comenzó a aplicar métodos analíticos complementados con experiencias racionales de las
organizaciones humanas.

Cabe señalar que con la Revolución Industrial se genera un desarrollo pujante de la


industria, en detrimento de las condiciones laborales. Con lo cual surge la necesidad de pro-
mulgar leyes referentes a las condiciones de trabajo y jornada laboral, razón que impulsa el
desarrollo de la Higiene y Seguridad Industrial (ver detalles de su origen en unidad 2).

A través del cambio generado en la historia moderna de Europa, se desencadenó


el paso de una economía agraria y artesanal, a otra dominada por la industria y
la mecanización; es lo que se denomina Revolución Industrial. El término fue es-
tablecido por el historiador Arnold Toynbee para referirse al desarrollo económico
británico entre 1760 y 1840.

Antes de continuar con la descripción del origen y evolución de la Ingeniería Industrial,


es necesario reflexionar acerca de la relación que existe entre las bases, que fundamentaron
la evolución de las diferentes ramas de la ingeniería y de otras profesiones, es decir el de-
sarrollo de la ciencia, tecnología, técnica, arte; para así, establecer claramente los aspectos
que aceleraron el desarrollo y avance de esta disciplina.

29
Sánchez Marchán, Nanci. (2014).Introducción a la Ingeniería Industrial, Material Instruccional de Apoyo. Caracas: UNA

INFLUENCIA DE LA CIENCIA, TECNOLOGÍA, TÉCNICA Y ARTE EN LA


INGENIERÍA
Para abordar este punto es importante definir cada uno de los términos involucrados:

• Ciencia. Proviene del latín scientia que significa conocimientos, los cuales son
obtenidos mediante la observación y el razonamiento, metódicamente estructurado.
Se sustentan en hipótesis, teorías, leyes, modelos y postulados, de ellos se deducen
principios y leyes generales. La Ciencia no crea las cosas sino que brinda el cono-
cimiento y la explicación sobre ellas. Cajas, F. (2001). Alfabetización Científica y
Tecnológica.

• Tecnología. Es una palabra de origen griego, τεχνολογος, formada por (τεχνo) que
significa arte, técnica u oficio y logos (λογος) conjunto de saberes. Se define como
el conjunto de habilidades que permiten construir objetos y máquinas para adaptar
el medio y satisfacer las necesidades humanas. También se puede decir que es el
conjunto de reglas, procesos, conocimiento que se aplican en determinada área del
quehacer productivo humano. Cajas, F. (2001). Alfabetización Científica y Tec-
nológica.

• Técnica. (del griego τεχνo, que significa arte). La técnica es un conjunto de sabe-
res prácticos o procedimientos para obtener un resultado deseado, puede ser apli-
cada en cualquier ámbito humano: ciencias, arte, entre otros. Este procedimiento
exige orden, seguir reglas, requiere tanto destrezas manuales como intelectuales,
frecuentemente el uso de herramientas y siempre de conocimientos muy variados.
La técnica no se explica, opera y se transforma a través de reglas, normas y proce-
dimientos. Marrandi, A, Archenti, N y Piovani, J. (2007). Método, Metodología,
Técnica.

• Arte. Pericia o habilidad en la producción de algo, que se puede elaborar de diver-


sas maneras. A través de este medio una persona puede expresar sentimientos,
pensamientos, ideas o imágenes de la realidad física o humana. Onians, John
(2008): Atlas del arte.

• Ingeniería. Como se expresó en la definición anterior de ingeniería (pág.15), es


la profesión que aplica conocimientos y experiencias para que mediante diseños,
modelos y técnicas se resuelvan problemas que afectan a la humanidad. También
se puede definir como el arte de aplicar los conocimientos científicos a la invención,
perfeccionamiento o utilización de la técnica en todas sus determinaciones. Pablo
Grech (2000): Introducción a la Ingeniería

En la ciencia, técnica, tecnología y arte, se manifiesta una variable común: La


creatividad, la necesidad continua de hacer cosas, de inventar, de innovar.

30
Sánchez Marchán, Nanci. (2014).Introducción a la Ingeniería Industrial, Material Instruccional de Apoyo. Caracas: UNA

Actividad propuesta
Establezca algunas diferencias importantes entre: ciencia y tecnología; técnica y
arte; ciencia y arte; técnica y tecnología.

Conociendo la definición de estos términos, se puede decir que para resolver los proble-
mas ingenieriles se ha de considerar: la ciencia, la técnica, la tecnología y el arte. La ciencia
constituye la base o el conocimiento sobre la cual la ingeniería se fundamenta, conocimien-
to que ésta aplica de forma metódica con fines racionales para realizar o llevar a cabo la
Investigación científica. La investigación científica hace uso del método científico, el cual se
puede definir como:

La pauta que permite a los investigadores ir desde un punto A hasta un punto Z con la
confianza de obtener un conocimiento válido, en el desarrollo de nuevos procesos, instru-
mentos, herramientas, etcétera, para el bienestar de la sociedad.

Ingeniería significa inventar utilizando el ingenio y apoyándose en ciencia, técnica, tec-


nología y arte, como se expresó anteriormente, para resolver problemas y lograr satisfacer a
los clientes. Con el uso de la ciencia la ingeniería busca la verdad, buena correspondencia
entre la realidad y las ideas que se hace de ella. Con el arte busca el placer que da la ex-
presión y evocación de los sentimientos, la belleza de la forma, los sonidos y los conceptos
que conduce al placer intelectual. Mediante la aplicación de la tecnología satisface las ne-
cesidades y deseos humanos, para lo cual considera su funcionalidad. Emplea la técnica,
para obtener un resultado en cualquier otra actividad, resolviendo problemas prácticos que
pueden transformar el mundo, concibiéndolo más predecible y provocando grandes efectos
sociales y ambientales. El comportamiento de una técnica significa saber hacer algo, para
obtener objetos técnicos.

OBJETOS TÉCNICOS
La elaboración de los objetos técnicos requiere de la aplicación de técnicas adecuadas
para cada caso.

Los objetos técnicos se pueden definir como:

Instrumentos que no existen en la naturaleza y que han sido ideados tras un largo pro-
ceso. Han sido aprehendidos a través del trabajo humano, pensados y juzgados como
instrumentos de apoyo o productos del trabajo.

El trabajo es lo que da sentido a los objetos técnicos y no lo contrario. El trabajo es la


actividad por la cual el hombre realiza en sí mismo la mediación entre la especie humana y
la naturaleza.

31
Sánchez Marchán, Nanci. (2014).Introducción a la Ingeniería Industrial, Material Instruccional de Apoyo. Caracas: UNA

Los objetos técnicos han pasado por una serie de etapas de desarrollo como se mues-
tra en la figura 5:

Etapa del Artesano


Étapa Industrial
Etapa del Hombre Época cuando las
Prehistórico necesidades sociales Aparece la primera
obligan a la ejecución de máquina de vapor que
Se genera antes de la tareas de producción acciona los primeros
aparición de la escritura, más especializadas. El telares mecánicos,
técnico primitivo se
etapa destinada a naciendo así, la técnica
satisfacer necesidades convierte en especialista de la época moderna o
de supervivencia, como pasando a llamarse industrial. Caracterizada
las derivadas de los artesano, el cual eleva la por la aparición del
actos naturales (comer, importancia social de la ingeniero además de
técnica. Se le otorga a la
dormir, etc). No había motores que generan
especialización del construcción de objetos una mayor producción
trabajo.   técnicos el rango de de objetos técnicos con
actividad especializada. menor esfuerzo.
   
 
Etapa Actual
Caracterizada por la
aparición de máquinas
específicas o
computadoras donde
los niveles de
aplicación de técnicas
organizativas son
elevados, permitiendo
la comunicación con
todo el planeta a través
de satélites y la
producción masiva de
productos
estandarizados.

 
Figura 5. Etapas del desarrollo de los objetos técnicos.

Se observa en estas etapas cómo el hombre en su afán por mejorar sus condiciones
de vida; hace uso de mecanismos como: la ciencia, la técnica, la tecnología y el arte, para
mejorar el diseño y la funcionalidad de los objetos que le han permitido desenvolverse en
diferentes ambientes y épocas del acontecer humano, generando la evolución y la moderni-
zación de éstos. La evolución de estas etapas en los objetos técnicos se puede observar a
través del ejemplo que se muestra a continuación:

Ejemplo Ilustrativo:
En la tabla siguiente se contempla la evolución de cuatro objetos técnicos que
logró crear el hombre con la ayuda de la ciencia, la tecnología, la técnica y el arte.

32
Sánchez Marchán, Nanci. (2014).Introducción a la Ingeniería Industrial, Material Instruccional de Apoyo. Caracas: UNA

Primer elemento Segundo elemento Tercer elemento


Abanico Ventilador Aire acondicionado
Pluma y tintero Máquina de escribir Computadora
Bote de remo Velero Yate
Horno a leña Horno a gas Horno Microondas

Los primeros elementos muestran diseños muy rudimentarios que luego se fueron mo-
dificando, hasta obtener diseños altamente sofisticados y prácticos, que permitieron al hom-
bre mejorar su vida.

Se puede decir que la Ingeniería Industrial al igual que otras ramas de la ingeniería,
carreras afines y no afines, requirió del avance de la ciencia, tecnología, técnica y arte,
como también del ingenio del individuo, para su desarrollo. Con estos avances se acumuló y
sistematizó el conocimiento, lo que permite que esta rama de la ingeniería se institucionalice
como profesión.

En la Ingeniería Industrial se presentó la orientación intelectual y la creatividad necesa-


ria para el proceso de industrialización, lo que la establece como una carrera distinta a otras,
por el factor humano que la integra, el mecánico y el económico, así como, por su aplicación.
En el factor humano se empezó a considerar la higiene y seguridad industrial, por los cam-
bios que la industrialización provocó, en la salud y bienestar de los trabajadores.

Se puede agregar entonces que la Ingeniería Industrial como una de las ramas de la
ingeniería, se encarga de intermediar entre la ciencia y la tecnología, aplicando los principios
científicos, la técnica y el arte, para determinar la forma más efectiva de combinar personas,
máquinas, materiales, información y energía para el desarrollo de productos o procesos, que
mejoren el bienestar de la sociedad en general y del trabajador en particular. Esta relación
se muestra en la figura 6:

Ciencia

Técnica Ingeniería Tecnología


Industrial

Arte

Figura 6. Relación de la ingeniería industrial con la ciencia, tecnología, técnica y arte.

33
Sánchez Marchán, Nanci. (2014).Introducción a la Ingeniería Industrial, Material Instruccional de Apoyo. Caracas: UNA

Sobre la base de la relación antes señalada, se aceleró el proceso de transformación


de la producción artesanal a la industrial y se desarrollaron las bases estructurales de la
Ingeniería Industrial, como se contempla en la figura 7:

Transformación
de producción Organización Mejora de
artesanal a del trabajo Sistemas
la industrial Surge la
productivo Productivos ingeniería
mediante el uso aplicando con el uso
combinado de la Industrial.
métodos y de diferentes
Ciencia, Técnica, técnicas. tecnologías.
Tecnología y el
Arte.

Figura 7. Bases estructurales de la Ingeniería Industrial.

Se puede vislumbrar en la figura 7, el surgimiento de la Ingeniería Industrial con el paso


de la producción artesanal a la industrial, aplicando la ciencia y técnicas que le permitieron
organizar el trabajo y disminuir el tiempo de producción en la empresa, además de una
mejora en los procesos haciendo uso de la tecnología y del arte para demostrar la pericia o
habilidad en la producción.

DEFINICIÓN DE LA INGENIERÍA INDUSTRIAL


Considerando las bases estructurales mencionadas anteriormente, se puede definir la
Ingeniería Industrial como:

Una disciplina relacionada con el diseño, organización, mejoramiento, instalación y ma-


nejo de sistemas productivos simples o complejos, integrados por individuos, materiales,
equipos, dinero, información y tecnología, que permiten generar, controlar y mejorar pro-
cesos, productos o servicios, a un menor costo; que satisfagan a los clientes, y que estén
en consonancia con la sociedad y el medio ambiente.

Otras definiciones de Ingeniería Industrial


A continuación se presentan otras definiciones, con la finalidad de aportar al lector de
las carreras Ingeniería Industrial y TSU en Higiene y Seguridad Industrial, una mejor apre-
ciación y visión de la ingeniería industrial. Estas definiciones también pueden constituir un
instrumento para la actividad académica y profesional.

• “La Ingeniería Industrial es la profesión en la cual se aplica, juiciosamente, el cono-


cimiento de las ciencias matemáticas y naturales, obtenido mediante el estudio, la
experiencia y la práctica, con el fin de determinar las maneras de utilizar económi-
camente los materiales y las fuerzas de la naturaleza en bien de la humanidad” (Ac-

34
Sánchez Marchán, Nanci. (2014).Introducción a la Ingeniería Industrial, Material Instruccional de Apoyo. Caracas: UNA

creditation Board for Engineering and Technolgy (ABET) antes el Engineers


Council for Profesional Development ECPD) 1932)

• “La Ingeniería Industrial se ocupa de la planificación, el mejoramiento y la instala-


ción de sistemas integrados por hombres, materiales y equipos. Exige conocimien-
tos especializados y una sólida formación en ciencias, matemática, física y social,
junto con los principios y los métodos del análisis y del proyecto, para especificar,
predecir y evaluar los resultados que habrá de obtenerse de tales sistemas” (Ame-
rican Institute of Industrial Engineering. 1948)

• “La Ingeniería Industrial es aquella que se ocupa del diseño, mejoramiento e implan-
tación de sistemas integrados por personas, materiales, equipo y energía. Se vale
de los conocimientos y posibilidades especiales de las ciencias matemáticas físicas
y sociales, junto con los principios y métodos del análisis y el diseño de ingeniería,
para especificar, predecir y evaluar los resultados que se obtendrán de dichos siste-
mas” (Salvendy, Gabriel, Manual de Ingeniería Industrial. 1991)

La Ingeniería Industrial es una ingeniería de optimización de la industria, tiene


que ver con el costo, la rentabilidad, la calidad, la flexibilidad, la satisfacción de la
demanda y las oportunidades.

Actividad propuesta
Examine y discuta las definiciones anteriores. Compárelas y redacte su propia
definición.

Sobre la base de la definición de ingeniería Industrial se puede decir que el Ingeniero


Industrial:

Es un profesional con una formación integral que lo capacita para desempeñarse en las
áreas técnicas de una empresa, desarrollando soluciones tecnológicas en los problemas
generados por los procesos productivos, para cubrir necesidades sociales, industriales o
económicas.

Aportes fundamentales realizados por diferentes personalidades a la


ingeniería industrial
Mucho se ha escrito acerca de los pioneros de la Ingeniería Industrial, quienes surgie-
ron durante y después de la revolución industrial en Inglaterra y Estados Unidos. Antes de la
revolución industrial los bienes los producían los artesanos en el conocido sistema casero.
En aquellos días la administración de las fábricas no era problema. Sin embargo, a medida
que se desarrollaban nuevos aparatos y se descubrían nuevas fuentes de energía, se tuvo
la necesidad práctica de organizar las fábricas para que se pudieran aprovechar las inno-
vaciones.

35
Sánchez Marchán, Nanci. (2014).Introducción a la Ingeniería Industrial, Material Instruccional de Apoyo. Caracas: UNA

A continuación se puede apreciar una tabla con los personajes más importantes que
impulsaron el desarrollo de la Ingeniería Industrial, destacando sus aportes e implicaciones
en el futuro de esta disciplina.

Tabla 2. Personajes fundamentales y sus aportes a la Ingeniería Industrial


Personaje y Fecha Aportes a la Ingeniería Industrial
Sir Richard De origen británico, primero de todos los pioneros. Inventó en Ingla-
Arkwright terra el Torno de Hilar Mecánico en 1732. Además creó y estableció lo
que probablemente fue el primer sistema de control administrativo para
(1732-1792) regularizar la producción y el trabajo de los empleados de las fábricas.

Ambos de origen británico. Ellos instituyeron la capacitación técnica


para los artesanos que superó cualquier tipo de capacitación que exis-
James Watt tiera en esa época y también contribuyeron mucho a normalizar la admi-
(1736-1819) nistración de las fábricas. Establecieron la primera fábrica completa de
máquinas de manufactura en el mundo construyendo una instalación de
manufactura integrada, que se adelantó mucho a su época. Siguiendo
Matthew Boulton el ejemplo de sus padres en 1775 lograron perfeccionar la Máquina de
(1728-1809) Vapor iniciada por éstos, las mejoras que realizaron en la Máquina de
Newcomen dieron lugar a la conocida Máquina de Vapor, que resultaría
fundamental en el desarrollo de la Revolución industrial, tanto en el Rei-
no Unido como en el resto del mundo. También, instituyeron un sistema
de control de costes diseñado para disminuir el desperdicio y mejorar la
productividad.

Fue un escritor empresa- 1. Ideales definidos claramente.


Harrington rial norteamericano, de- 2. Sentido común.
fensor de las operaciones 3. Asesoría competente.
Emerson eficientes y del pago de
4. Disciplina.
premios para el incremen-
to de la producción. Su 5. Trato justo.
(1853-1931)
libro, The Twelve Princi- 6. Registros confiables, inmediatos y
ples of Efficiency (1911), adecuados.
presentaba las bases 7. Distribución de las órdenes de trabajo.
para obtener operacio- 8. Estándares y programas.
nes eficientes. Generó 12 9. Condiciones.
principios que de alguna 10. Operaciones estándar.
forma fueron paralelos a
las enseñanzas de Taylor; 11. Instrucción de la práctica estándar por
escrito.
estos eran los siguientes:
12. Recompensa a la eficiencia.

No cabe duda de que los 12 principios expuestos por Emerson en 1911


son tan válidos hoy como lo fueron entonces.

36
Sánchez Marchán, Nanci. (2014).Introducción a la Ingeniería Industrial, Material Instruccional de Apoyo. Caracas: UNA

Economista y filósofo británico, cuyo aporte fundamental lo compone


su célebre obra “Naturaleza y Causa de la Riqueza de las Naciones”
Adam Smith en 1776. Constituyó el primer intento de analizar los determinantes del
(1723-1790) capital y el desarrollo histórico de la industria y el comercio entre los paí-
ses europeos; lo que permitió crear la base de la moderna ciencia de la
economía.
También se le atribuye la Especialización del trabajo. Observó que al
dividir la tarea de fabricar alfileres en cuatro operaciones, aumentaba la
producción cinco veces. Estableció tres ventajas de la división del trabajo,
justificando el aumento de la eficiencia en las siguientes consecuencias:
1. El aumento de la destreza de cada trabajador.
2. El ahorro de tiempo que comúnmente se pierde al pasar de
una especie de trabajo a otro.
3. La invención de un gran número de máquinas que facilitan
y abrevian el trabajo.

Matemático e Ingeniero de origen británico, se licenció en la Universidad


de Cambridge en 1814. Inventor de las máquinas calculadoras programa-
Charles Babbage bles. En 1815, fundó con J. Herschel la Analytic Society con el propósito
(1791-1871) de la renovación de la enseñanza de las matemáticas en Inglaterra. En
1816 fue elegido miembro de la Royal Society. Charles Babbage, en su
libro “sobre la economía de la maquinaria en las fabricaciones” publicado
en Filadelfia en 1832 indicaría que el principio más importante del que
depende la economía de una fabricación, es la división del trabajo entre
las personas que lo realizan.
Al respecto fundamentó:
1. “El período ocupado en el adiestramiento de un arte cual-
quiera dependerá de la dificultad de su ejecución y mientras
mayor sea el número de procedimientos distintos, en conse-
cuencia más largo será el tiempo que el aprendiz emplea en
adquirirlo”.
2. “La repetición constante de un mismo procedimiento deter-
mina ‘un grado de dominio’ y rapidez (eficiencia), que no es
adquirido nunca por una persona que está obligada a ejecu-
tar muchas actividades diferentes”. Por ejemplo, Adam Smith
estableció que un herrero acostumbrado nada más que a ha-
cer clavos, producía tres veces más que un herrero general o
que elaboraba productos diferentes.
3. “Cuando cada procedimiento u operación por el que es pro-
ducido un artículo constituye la única ocupación del indivi-
duo, siendo dedicada toda su atención a una operación muy
limitada y simple, es mucho más probable que venga a su
mente alguna forma de mejorar sus herramientas o el modo
de usarlas”.

37
Sánchez Marchán, Nanci. (2014).Introducción a la Ingeniería Industrial, Material Instruccional de Apoyo. Caracas: UNA

De origen Estadounidense. Antes de la revolución industrial no era


práctica usual el uso de partes intercambiables, puesto que no existían
Eli Whitney patrones (estándares) de producción. Los conceptos sobre partes inter-
(1765-1825) cambiables son desarrollados por Eli Whitney; La industria naciente re-
cibe así un fuerte impulso que acelera su desarrollo, posteriormente mu-
chas otras personas contribuyen con sus aportes a sentar las bases de
esta nueva especialidad de la ingeniería. Eli Whitney fue un inventor y
fabricante, inventó la máquina para desgranar el algodón en 1793. Esta
máquina era una unidad mecánica que removía las semillas del algodón,
lo cual hasta entonces era un trabajo muy pesado. La mayor contribución
de éste para la industria norteamericana, fue el desarrollo e implementa-
ción del sistema de fabricación y la línea de montaje. Fue el primero en
usarla en la producción de mosquetes para el gobierno norteamericano.
Los conceptos de Whitney fueron explotados más adelante por Henry
Ford y otros en la industria.

Frederick W.Taylor, quien es considerado padre de la Ingeniería Industrial


nace en Filadelfia en 1856, suspende sus estudios y en 1878 es empleado
Frederick W. por la Acería Midvale Steel Works. Comienza a trabajar como un simple
Taylor obrero y asciende rápidamente en esa industria. Al mismo tiempo se rei-
1856-1915 nicia en sus estudios y obtiene el título de ingeniero mecánico en 1883, es
nombrado ingeniero jefe en esa empresa. En 1881 Taylor había iniciado
un estudio de las distintas actividades que se ejecutaban en las fabricas,
luego de 12 años de esfuerzo, desarrolla un concepto basado en la idea
de tarea. Taylor propone que la gerencia realice un plan de trabajo para
cada uno de sus empleados, que se escriba en detalle la tarea correspon-
diente y se le indiquen al operario las herramientas que deberá emplear
en la ejecución de la tarea.
Conocido el plan de trabajo que debe ejecutar cada empleado, se asig-
nará un tiempo determinado para cada actividad de ese plan, esto es lo
que se conoce como la formula de Taylor utilizada para máximo rendi-
miento laboral. Su formación y su capacidad personal permitieron a Taylor
pasar enseguida a dirigir un taller de maquinaria, donde observó minu-
ciosamente el trabajo de los obreros que se encargaban de cortar los
metales, descomponiéndo el trabajo en tareas simples, cronometrándolas
estrictamente y exigiendo a los trabajadores la realización de las tareas
necesarias en el tiempo justo, minimizando así tiempos muertos del traba-
jador por cambios de actividad o de herramientas; y establece un salario
a destajo (por pieza producida) en función del tiempo de producción esti-
mado, salario que debía actuar como incentivo para la intensificación del
ritmo de trabajo. La tradición quedaba así sustituida por la planificación
en los talleres, pasando el control del trabajo de manos de los obreros a
los directivos de la empresa y poniendo fin al forcejeo entre trabajadores
y empresarios en cuanto a los estándares de productividad.

38
Sánchez Marchán, Nanci. (2014).Introducción a la Ingeniería Industrial, Material Instruccional de Apoyo. Caracas: UNA

Ingeniero industrial de origen norteamericano, mecánico; discípulo de F.


W. Taylor, fue colaborador de éste en el estudio de una mejor organiza-
Henry L. Gant ción del trabajo industrial. Sus investigaciones más importantes se centra-
(1861-1919) ron en el control y planificación de las operaciones productivas mediante
el uso de técnicas gráficas, entre ellas el llamado diagrama de Gantt,
popular en toda actividad que indique planificación en el tiempo. Su obra
principal, publicada en 1913, se titula Work, Wages and Profits (Trabajo,
salarios y beneficios).
Propuso una selección científica de los trabajadores y una cooperación
armónica entre ellos y la gerencia. Recalcaba la necesidad de desarro-
llar una mutualidad de intereses entre la gerencia y los trabajadores. Al
hacer esto, subrayó la importancia de la enseñanza, de desarrollar una
comprensión de sistemas, tanto por parte de los obreros como de la di-
rección y de apreciar que en “todos los problemas de la Administración el
elemento humano es el más importante”. Siendo considerada por algunos
historiadores sociales como el invento social más importante del siglo XX.

Frank Gilbreth, fue el fundador en 1911 de la técnica moderna del estudio


de movimientos, la cual se puede definir como el estudio de los movimien-
Frank Gilbreth tos del cuerpo, que se utilizan para ejecutar una operación, con la idea de
(1868-1924) mejorarla, eliminando y simplificando los movimientos innecesarios y es-
tableciendo después, la secuencia de movimientos más favorables para
lograr una eficiencia máxima. Puso en práctica sus teorías por primera
vez en 1895, en el oficio de albañilería, donde éste estaba empleado.
Introdujo mejora de método por medio del estudio de movimientos y en-
trenamientos de operadores, con lo cual logró aumentar el promedio de
colocación de ladrillos a 350 por hombre por hora. Antes que Gilbreth
hiciera sus estudios, el promedio por hombre por hora era de 120 ladrillos
y se consideraba un promedio satisfactorio de trabajo.
Frank Gilbreth, con la ayuda de su esposa Lillian Gilbreth, desarrolló tam-
bién la técnica de la cámara de cine para estudiar los movimientos, la
cual ha contribuido a la fecha a la solución de muchos problemas. En
la industria, esta técnica se conoce con el nombre de Estudio de micro
movimientos. Pero el estudio de movimientos con la ayuda de la cámara
lenta, no se limita a aplicación industrial, es inapreciable, en el mundo de
los deportes como instrumento para mostrar el desarrollo de la habilidad
y de la forma.
El estudio de movimientos, en el sentido más amplio, contiene dos grados
de refinamiento con amplias aplicaciones industriales. Estas son, el estu-
dio visual de los movimientos y el estudio de micro movimientos.
El estudio visual de movimientos se aplica con mayor amplitud, ya que
la actividad que se estudia no necesita ser de tanta importancia, para
justificar económicamente su empleo. Este tipo de estudio comprende la
observación cuidadosa de la operación y la elaboración de un diagrama
de proceso del operador, teniendo en cuenta las leyes de la economía de
movimientos.
A causa de su costo considerable, el procedimiento de micro movimientos
resulta generalmente práctico sólo en trabajos de mucha actividad, en
donde la duración y la repetición son grandes. Estos dos tipos de estudios
pueden compararse con la vista de un objeto a través de una lupa, o por
medio de un microscopio. Los detalles que añade el microscopio solo
tienen aplicación en trabajos más productivos.

39
Sánchez Marchán, Nanci. (2014).Introducción a la Ingeniería Industrial, Material Instruccional de Apoyo. Caracas: UNA

Fue una de las primeras psicólogas industriales y recibió su doctorado en


esta disciplina en 1915. Después de la muerte repentina de su esposo en
Lillian Gilbreth 1924, continuó al frente de la firma consultora y fue muy aclamada como
(1878-1972) la “primera dama de la Administración”. La atención de Lilian Gilbreth por
los aspectos humanos del trabajo y el interés que tenía su esposo por
la eficiencia, dieron lugar a una rara combinación de talentos. Por esta
razón Frank Gilbreth insistió en que en la aplicación de los principios de
la administración científica, es necesario considerar a los trabajadores
primero y comprender sus necesidades y su personalidad. Frank afirmaba
que la administración científica no puede determinar normas de lo justo o
lo injusto, pero determina normas de eficiencia para lo cual la administra-
ción debe “primero conservar lo mejor del trabajo; segundo, organizar el
presente; y tercero prever y planear el futuro”.
Es interesante que los esposos Gilbreth llegaran a la conclusión de que no
es la monotonía en el trabajo la causa de tanta insatisfacción laboral, sino
más bien la falta de interés que muestra la gerencia por los trabajadores.

Henry Fayol Conocido como el “Padre de la Teoría Moderna de la Administración”,


cuya contribución se manifiesta a través de las siguientes hipótesis:
(1841-1925)
Necesidad de desarrollar una doctrina administrativa mediante la apari-
ción del método positivista.
El buen funcionamiento de la organización formal de la empresa depen-
de de la búsqueda del mejoramiento continuo mediante la aplicación “de
principios, de leyes o reglas” de carácter flexible, no absoluto y que deben
usarse independientemente de las condiciones cambiantes y especiales.
Se debe dar prioridad a la subordinación del interés particular al interés
general, lo cual puede leerse como: el interés de quienes tienen el poder
en las organizaciones deben subordinar los intereses individuales.
El modelo administrativo de Fayol tiene tres aspectos básicos:
Aplicación del criterio de división del trabajo para establecer una estructu-
ra básica de la empresa mediante operaciones o agrupación de funciones
básicas de la organización.
Proceso administrativo de gerencia, funciones básicas de los directivos o
elementos de la administración.
Formulación de criterios técnicos que deben orientar la actividad adminis-
trativa o los principios generales de la administración.
Fayol descubrió que las actividades de una empresa industrial podían di-
vidirse en 6 grupos:
Técnicas de producción: fabricación, transformación de insumos.
Comerciales: compras, ventas, búsqueda de mercados, intercambio.
Financieras: búsqueda y uso óptimo del capital.
De seguridad: protección de la propiedad y de las personas.
Contables: registro de ingresos y egresos, inventarios, balances, estadís-
ticas, precio de costo.
Administrativas o de gerencia: previsión, mando, organización, coordina-
ción y control.
Al señalar que estas actividades existen en los negocios de cualquier ta-
maño, Fayol observaba que las cinco primeras eran bien conocidas y, por
lo tanto, dedicó la mayor parte de su libro al análisis de las Administrativas.
Además, propone 14 principios, leyes, reglas o criterios para el buen fun-
cionamiento de la estructura social, denominados principios generales de
la administración.

40
Sánchez Marchán, Nanci. (2014).Introducción a la Ingeniería Industrial, Material Instruccional de Apoyo. Caracas: UNA

Henry Ford Fue un industrial de origen estadounidense, conocido por sus innovado-
res métodos en la industria de los vehículos a motor.
(1863-1947)
En 1913 Ford, introdujo en su fábrica la técnica de la Cadena de Montaje,
y empezó a utilizar piezas intercambiables para los coches. Aunque no
fue el creador, ni el primero en utilizar estas técnicas inherentes al sistema
fabril, sí fue el principal responsable de que estas prácticas se generali-
zaran, ayudando así a la gran expansión de la industria estadounidense.
A principios de 1914 esta innovación, aunque aumentaba la productividad,
redundó en una reducción del trabajo mensual en su fábrica de entre el 40
y el 60%, debido sobre todo a la monotonía de la cadena de montaje y a
los repetidos aumentos de las cuotas de producción asignadas a los tra-
bajadores. Ford superó esta dificultad duplicando el salario diario medio
de la industria, pasando de pagar dos dólares y medio a cinco dólares al
día. El resultado neto fue una creciente estabilidad en su fábrica y una re-
ducción importante de los costes corrientes. Estos hechos y gracias a los
mismos métodos, dieron como resultado un incremento en la producción.

41
Sánchez Marchán, Nanci. (2014).Introducción a la Ingeniería Industrial, Material Instruccional de Apoyo. Caracas: UNA

En estos aportes se puede destacar cómo el hombre con su capacidad creativa e inno-
vadora ha integrado la ciencia, la tecnología, la técnica y el arte con la finalidad de mejorar
y transformar cualquier sistema formado por hombres, materiales, recursos financieros y
equipos; para cambiar el entorno en beneficio del trabajador, de la empresa y de la sociedad
en su conjunto, considerando también el cuidado del medio ambiente.

Éste es el lema que debe impulsar a los ingenieros industriales en su proceder, ya sea
para continuar con las importantes investigaciones de estos personajes y mejorarlas o en-
contrando nuevos hallazgos que simplifiquen y mejoren las actividades dentro y fuera de la
empresa.

Eres dueño de tus acciones y de ellas depende tu tránsito por la vida, así como el
aporte al desarrollo de la disciplina que elegiste para tu preparación profesional
que recién inicias. Detén tu camino sólo para analizar, reflexionar y mejorar tu
proceder y continúa tu caminar buscando ideas y aportando soluciones para el
individuo en su ambiente laboral y social.

Actividad propuesta
Conocidos los aportes realizados por los personajes antes mencionados, selec-
cione cuál o cuáles de ellos a su parecer han contribuido más en el desarrollo de
la ingeniería industrial, así como, a mejorar las condiciones del medio ambiente
laboral. Argumente su selección.

EVOLUCIÓN Y FUTURO DE LA INGENIERÍA INDUSTRIAL


La Ingeniería Industrial ha experimentado diversos cambios, pasando de los métodos
mecánicos a los métodos electrónicos, de procedimientos de diseño cualitativo a nuevas
técnicas que requieren modelación, simulación y amplio empleo de estadísticas; de un en-
foque centrado en la producción a un enfoque integrador de sistemas. El interés pasó de
sistemas relativamente pequeños a macrosistemas y de la medición de las actividades y
diseños de espacio de trabajo, al diseño y análisis de sistemas más grandes y complejos.

Se pueden identificar cinco etapas en la evolución de la Ingeniería Industrial como apa-


rece en la figura 8:

42
Sánchez Marchán, Nanci. (2014).Introducción a la Ingeniería Industrial, Material Instruccional de Apoyo. Caracas: UNA

INGENIERÍA INDUSTRIAL
Profesión que integra tanto el objetivo técnico como el objetivo
social, característica particular de ésta rama de la ingeniería.
ETAPAS EN LA EVOLUCIÓN DE LA IINGENIERÍA INDUSTRIAL

COMPORTAMIENTO HUMANO EN BUSCA DE LA EXCELENCIA


Se genera un cambio de funciones que atienden los Ingenieros
Industriales, además de las consideradas en las etapas anteriores,
aparece la función social de incrementar la productividad, con el objeto
de generar bienestar compartido por los integrantes internos y externos
de una empresa.

SOBRE LA BASE DE LA CIBERNETICA Y


LA TEORÍA GENERAL DE SISTEMAS
Se trabaja en el análisis de sistemas más grandes y complejos.

APOYADA SOBRE LA BASE DE SISTEMAS DE INFORMACIÓN


En ésta estapa se da un salto espectacular de un enfoque centrado
en producción a un enfoque integrador de sistemas.

SE UTILIZAN MODELOS MATEMÁTICOS


(Investigación de Operaciones)
Se generan modelos mátematicos para la solución de problemas y
toma de decisiones.

ETAPA CONVENCIONAL
Predominio de los métodos mecánicos a través del estudio de
tiempos y movimientos.

Figura 8. Etapas de la evolución de la Ingeniería Industrial.

Actividad propuesta
Sobre la base de lo descrito anteriormente complete la línea de tiempo mostrada
en la figura que sigue, correspondiente a la evolución de la Ingeniería Industrial.
Señale las etapas que faltan y amplíe el contenido plasmado en cada una de
ellas, utilizando información impresa o la encontrada en internet.

43
Sánchez Marchán, Nanci. (2014).Introducción a la Ingeniería Industrial, Material Instruccional de Apoyo. Caracas: UNA

  Ingeniería
Industrial

Etapa 1 Etapa 2 Etapa 3 Etapa 4


  Utilización de modelos
matemáticos
(Investigación de
operaciones)

ÁREAS DE CONOCIMIENTO EN LA PRÁCTICA MODERNA DE LOS


INGENIEROS INDUSTRIALES
En 1993, el National Council of Examiner for Engineering and Surveying (NCEES) en
Estados Unidos , identificó como áreas de conocimiento en la práctica moderna de los In-
genieros Industriales: probabilidad y estadística; software, hardware y sistemas operativos;
medición e instrumentación, estándares e investigación de operaciones; principios de ad-
ministración, ergonomía e ingeniería humana, métodos de trabajo y técnicas de medición;
materiales y procesos de manufactura; ciencia de materiales y sus propiedades físicas y me-
cánicas; análisis y diseño de sistemas, así como análisis de costos e Ingeniería Económica;
confiabilidad, análisis de falla y Estadística para el Control de Calidad.

En general, la educación en Ingeniería Industrial tiende a cubrir un amplio espectro de


funciones técnicas y gerenciales, con incremento del poder de decisión a todos los niveles, y
en muchos casos, una ampliación de la práctica profesional fuera del campo de exclusividad
de la manufactura. Es por ello que en la actualidad la formación del Ingeniero Industrial se
inclina hacia integrar las habilidades propias de la ingeniería, con los métodos de la matemá-
tica y de la informática para formular y construir modelos para el diseño, análisis, evaluación
y predicción de sistemas productivos.

La práctica de la Ingeniería Industrial ha cambiado radicalmente en el es-


quema de la globalización productiva y, es sin duda una de las carreras de
gran importancia y futuro en nuestro país. Esta carrera también llamada la
profesión de la productividad, impacta positivamente la eficiencia y eficacia
de los sistemas productivos y de servicios.

Esta época muestra un mundo altamente competitivo, lo que reclama una Venezuela
dinámica. La modernización de nuestro país necesita mayor calidad, productividad y compe-
titividad, de tal forma que es necesario contar con tecnología de vanguardia. Esto requiere de
recursos humanos capaces de coordinar las relaciones entre el hombre, el medio ambiente
y la técnica; un agente de cambio cuyo reto no es sólo optimizar lo que existe sino crear e
implementar procesos, integrar los procesos globales de las organizaciones, haciendo espe-
cial énfasis en el rediseño de éstos; en el enfoque de procesos y sistemas y en la tendencia
hacia la administración de la tecnología y la información.

44
Sánchez Marchán, Nanci. (2014).Introducción a la Ingeniería Industrial, Material Instruccional de Apoyo. Caracas: UNA

En la Universidad Nacional Abierta (UNA), conscientes de este reto, se orienta el perfil


de la carrera hacia la realización de actividades en las áreas operativa y de gerencia; se pro-
picia el desarrollo de organizaciones de calidad que busquen competitividad a nivel nacional
e internacional, así como también reforzar aspectos como la ética y los valores del futuro
Ingeniero Industrial UNA.

LA INGENIERÍA INDUSTRIAL EN VENEZUELA


En Venezuela aparece la primera escuela de Ingeniería Industrial, en la Universidad de
Carabobo (UC), en el año de 1958. El 21 de marzo de 1958, la Junta de Gobierno que asu-
mió el poder, a raíz de la caída de Pérez Jiménez, refundó, mediante Decreto No. 100, a la
Universidad de Carabobo, tal como se conoce hoy.

A la nueva universidad se le anexaron 3 facultades: la Facultad de Derecho, que sería la


continuación de la existente Escuela de Derecho Miguel José Sanz y dos, totalmente nuevas:
la Facultad de Medicina y la Facultad de Ingeniería. De estas 3 facultades la que realmente
tuvo un alcance innovador fue la de ingeniería, porque con el nombre de Facultad de Ingenie-
ría Industrial, que se le dio al comienzo, se iniciaba una carrera que no existía en Venezuela.
El Comité Organizador de la universidad designó a un Ingeniero Mecánico suizo, de nombre
Guillermo Machler, Jefe de la UNESCO en Venezuela, para que organizara la primera Es-
cuela de Ingeniería Industrial del país, elaborara su pensum y se ocupara de dirigirla.

En octubre de 1958 se abrió un curso básico, fundamentado en el pensum diseñado por


el Dr. Machler, con el primer año de la carrera y cuyas clases comenzaron a dictarse en las
aulas del viejo edificio que ocupaba la Escuela de Derecho Miguel José Sanz. Paralelamente
se comenzaron a construir, en los terrenos que le habían sido cedidos a la Universidad en
Bárbula, tres galpones provisionales que fueron divididos en dos aulas de clases, una sala
de dibujo, una pequeña biblioteca, las oficinas del decanato, la dirección de la escuela y un
taller de carpintería que fue el primer laboratorio de la escuela. El pensum muy europeo que
propuso el Dr. Machler, incluía la enseñanza de carpintería, como una materia fundamental
del ciclo básico.

El primer decano de la Facultad fue el ingeniero Simón Medina Ortega quien renunció
ocho meses después, asumiendo el cargo el Ing. Gustavo Casson López, el 13 de diciembre
de 1959. Casson de profesión Ingeniero Industrial obtuvo su título en el Instituto Tecnológico
de Georgia (Georgia Tech. USA) que lo llevó a ser el primer Ingeniero Industrial venezolano.
Este cargo lo ocupó hasta septiembre de 1962.

De los 47 estudiantes que comenzaron sus estudios de ingeniería industrial en la UC, se


graduaron en la primera promoción 13 estudiantes. Uno de ellos a pesar de las penurias del
inicio, se fue, acabado de graduar, al mejor instituto tecnológico del mundo “MIT” y obtuvo un
máster en tiempo récord y con notas sobresalientes, el Ing. César Peña Vigas.

También, se graduaron en esa primera promoción, los ingenieros: Rolando Carrillo, Iván
Olaizola, Ramón Aldrufeu, Carlos y Víctor Alvarado, entre otros. Así surgió y creció la Facul-
tad de Ingeniería de la Universidad de Carabobo.

45
Sánchez Marchán, Nanci. (2014).Introducción a la Ingeniería Industrial, Material Instruccional de Apoyo. Caracas: UNA

Después, surgieron escuelas en las siguientes universidades: La Universidad Católica


Andrés Bello (UCAB) en Caracas, La Universidad del Zulia (LUZ), El Instituto Politécnico
Luis Caballero Mejías en Barquisimeto, Caracas y Puerto Ordaz; hoy Universidad Nacional
Experimental Politécnica “Antonio José de Sucre”, La Universidad Nacional Experimental del
Táchira, La Universidad Simón Bolívar en Caracas (USB), Universidad de Oriente (UDO),
Universidad Experimental Francisco de Miranda en Coro estado Falcón (UNEFM), Universi-
dad Nacional Abierta en todo el territorio nacional (UNA).

Posteriormente, fueron apareciendo nuevas instituciones tanto públicas como privadas,


las cuales incluyen dentro de sus ofertas de carreras a la Ingeniería Industrial, tal es el caso
de la Universidad José María Vargas y la Universidad Santa María en Caracas, la Universi-
dad Yacambú en Cabudare estado Lara, la Universidad Experimental de Guayana, el Institu-
to Universitario Politécnico Santiago Mariño en Caracas, la Universidad José Antonio Páez
en Valencia.

A continuación se presenta el mapa de Venezuela, en el cual se pueden visualizar las


regiones de la República Bolivariana de Venezuela en las cuales están ubicadas las univer-
sidades más importantes, donde se dicta la carrera Ingeniería Industrial:

REGIONES DE VENEZUELA DONDE SE DICTA LA CARRERA INGENIERÍA INDUSTRIAL

46
Sánchez Marchán, Nanci. (2014).Introducción a la Ingeniería Industrial, Material Instruccional de Apoyo. Caracas: UNA

Leyenda

Número Estado Universidades


Universidad Nacional Abierta (UNA)*
Universidad Nacional Experimental Antonio José de Sucre (UNEXPO).
Universidad Simón Bolívar (USB)
Miranda Universidad Santa María (USM)
Universidad José María Vargas (UJMV)
1
Instituto Universitario Politécnico Santiago Mariño (IUPSM)
Universidad Católica Andrés Bello (UCAB)
2 Carabobo Universidad de Carabobo (UC)
3 Táchira Universidad Nacional Experimental del Táchira (UNET)
4 Zulia Universidad del Zulia (LUZ)
5 Anzoátegui Universidad de Oriente (UDO)
6 Falcón Universidad Francisco de Miranda (UFM)
Universidad Nacional Experimental Antonio José de Sucre (UNEXPO).
7 Lara
Universidad Yacambu (UY)
8 Bolívar Universidad Nacional Experimental Antonio José de Sucre (UNEXPO).
* La Universidad Nacional Abierta (UNA) con Sede Administrativa en Caracas, imparte la carrera
Ingeniería Industrial en todo el Territorio Nacional en sus diferentes Centros Locales.

Figura 9. Mapa de Venezuela con la ubicación de las regiones donde se dicta la carrera
Ingeniería Industrial.

MISIÓN E IMPORTANCIA DE LA INGENIERÍA INDUSTRIAL EN VENEZUELA


La Ingeniería Industrial en Venezuela afronta el gran reto de generar modelos organiza-
tivos que se adapten a las necesidades del país, y además de convencer a los dueños y/o
directivos de las distintas empresas de la importancia de su aplicación, para poder sobrevivir
con éxito en un mundo cada vez más competitivo y exigente.

La Ingeniería Industrial ha traído beneficios importantes al país, tanto en el campo de


trabajo, como en el hogar y las ciudades. La necesidad de esta rama de la ingeniería en Ve-
nezuela se ha incrementado, en la medida en que los ingenieros industriales se han dado a
conocer en su campo, dirigiendo sus esfuerzos a:
•• Realizar las tareas dirigidas hacia la obtención de productos y servicios
con una alta calidad, disminución de costos de producción, el aumento
de la eficacia y eficiencia en la mano de obra, el incremento en la efi-
ciencia de los equipos, la procura de condiciones de trabajo saludables y
seguras, mejor remuneración entre otras.

47
Sánchez Marchán, Nanci. (2014).Introducción a la Ingeniería Industrial, Material Instruccional de Apoyo. Caracas: UNA

•• Concebir un instrumento de acción social en términos de ayudar a los


sectores con más necesidad de la población.
•• La protección, cuidado y saneamiento del medio ambiente, en la produc-
ción de bienes, por aquellas empresas cuyos desechos representen un
peligro para la sociedad.
•• Realizar el trabajo en la empresa bajo un ambiente SOL (Seguridad, Or-
den y Limpieza).
•• Otros.

PERFIL DEL INGENIERO INDUSTRIAL EN VENEZUELA


El Ingeniero Industrial es un profesional capaz de reconocer y optimizar los recursos
humanos, físicos, técnicos y operativos de todo el proceso productivo, desde su planeación
y producción hasta su distribución y consumo; e integrarlo a las necesidades propias de la
sociedad y la cultura en la que interactúa, aplicando para ello sus conocimientos y técnicas
especializadas basadas en las matemáticas, física, ciencias sociales, junto con los principios
y métodos de análisis y el diseño de la ingeniería industrial en pro de buscar la solución de
problemas en sistemas balanceados, que le permita lograr una posición empresarial de alto
nivel.

De acuerdo con su formación humanística, técnica y científica, el Ingeniero Industrial


realiza actividades en áreas como:

Dirección y administración de producción:


•• Control de producción.
•• Planeación de producción.

Gestión de calidad:
•• Planeación de calidad.
•• Organización de sistemas de control de calidad.

Investigación de operaciones:
•• Organización de sistema.
•• Estructuración de modelos y análisis de estos.

Higiene y seguridad industrial (estas actividades son propias del ingeniero in-
dustrial, así como del TSU en higiene y seguridad industrial):
•• Establece programas de educación para el adiestramiento del personal,
así como programar campañas de seguridad.
•• Crea programas de inspección de máquinas, equipos y lugares de tra-
bajo.

48
Sánchez Marchán, Nanci. (2014).Introducción a la Ingeniería Industrial, Material Instruccional de Apoyo. Caracas: UNA

Ingeniería de métodos o estudios de movimientos y tiempos:


•• Balance de líneas.
•• Medición de trabajos.
•• Estudios de métodos.
•• Ingeniería de productos.
•• Diseño del producto.
•• Diseño de procesos de producción.

Ingeniería de proyectos:
•• Estudio de mercados.
•• Determinación del tamaño y de la localización.

Administración de personal:
•• Evaluación de puestos.
•• Estudio de salarios.

PERFIL DEL TSU EN HIGIENE Y SEGURIDAD INDUSTRIAL EN VENEZUELA


Este profesional puede desempeñar las funciones de coordinación, supervisión, aseso-
ramiento e inspección para empresas o instituciones como miembro del Comité de Seguri-
dad y Salud Laboral, miembro de los Consejos Estadales y Municipales o en la rama de ac-
tividad económica de seguridad y salud en el trabajo, orientadas a desarrollar capacidades
relativas al ejercicio profesional en un área específica de las ciencias sociales.

Con el propósito de diferenciar las funciones de supervisor e inspector, se puede des-


tacar que el TSU en Higiene y Seguridad Industrial en su función como inspector, tiene la
responsabilidad con los fines y objetivos de garantizar la salud y seguridad en el entorno
laboral, velando por el cumplimiento de las normas y procedimientos de higiene y seguridad
industrial.

Además de todo lo expuesto anteriormente el Ingeniero Industrial y el TSU en Higiene y


Seguridad Industrial deben poseer las siguientes cualidades:
•• Inclinación por el trabajo científico; se expresa en la curiosidad, el disfrute
y dedicación de esfuerzos a los problemas científicos.
•• Actitud organizada y disciplinada en el trabajo; se refiere a la buena or-
ganización del tiempo, la planificación y organización de las actividades.
•• Cuidado; rapidez y exactitud en la realización de tareas simples.
•• Colaboración; capacidad de trabajar eficientemente en equipos para rea-
lizar tareas y proyectos específicos.
•• Capacidad numérica; aptitud verbal para trabajar con números y resolver
problemas de cálculo aritmético.

49
Sánchez Marchán, Nanci. (2014).Introducción a la Ingeniería Industrial, Material Instruccional de Apoyo. Caracas: UNA

•• Habilidad de comunicarse; facilidad para comunicarse de modo verbal y


no verbal sobre la base de la lógica y el calor humano.
•• Capacidad analítico-sintética; habilidad para observar los detalles de una
realidad o sistema de modo que las partes se relacionen y se visualicen
dentro de su totalidad.
•• Interés humanístico; capacidad de demostrar sensibilidad ante situacio-
nes humanas que necesiten de soluciones tecnológicas y científicas.
•• Pensamiento abstracto; capacidad demostrada de trabajar con símbolos,
relaciones, espacios, fórmulas, dentro de lenguajes formales.
•• Responsabilidad; asumir solo o en grupo un rol de participación efectiva
en la conducción de un proceso o en la realización de una tarea.
•• Compromiso ético-social; sensibilidad demostrada ante los problemas
sociales de un grupo o una comunidad manteniendo el respeto ante inte-
reses, apreciaciones, valores.
•• Retentiva; capacidad para retener información y recordar cosas que se
aprendieron en algún tiempo.
•• Creatividad e innovación; capacidad de proponer nuevas vías, nuevas
alternativas, resoluciones de problemas y situaciones, utilizando pensa-
mientos diversos.

Se puede decir entonces, que el Ingeniero Industrial y el TSU en Higiene y Seguridad


Industrial dada su sólida formación humanística, técnica y científica, además de las cualida-
des que lo caracterizan; son capaces de insertarse en la mayoría de las áreas de la industria,
para contribuir con el desarrollo y buen funcionamiento de éstas.

MERCADO OCUPACIONAL DEL INGENIERO INDUSTRIAL Y DEL TSU EN


HIGIENE Y SEGURIDAD INDUSTRIAL
El egresado en estas especialidades tiene la capacidad para trabajar en la mayoría de
las áreas de una industria, pueden trabajar en cualquier institución que tenga áreas relacio-
nadas con la evaluación económica, política y social.

Estas áreas se pueden clasificar como sector público y sector privado. Los elementos
de cada una de éstas áreas se pueden observar en la tabla siguiente:

50
Sánchez Marchán, Nanci. (2014).Introducción a la Ingeniería Industrial, Material Instruccional de Apoyo. Caracas: UNA

Tabla 3. Áreas del sector público y sector privado

Áreas
Sector público Sector privado
• Ministerios. • Oficinas de asesoramiento y asisten-
cia técnica.
• Fondo de Desarrollo Industrial.
• Sector manufacturero.
• Corporaciones regionales de de-
sarrollo. • Empresas consultoras.
• Corpoindustria, gobernaciones de • Universidades o instituciones educati-
estado y alcaldías. vas privadas.
• Entidades bancarias. • Plantas industriales de diferente natu-
raleza.
• Institutos de educación superior,
en docencia e investigación. • Entidades crediticias privadas que fi-
nancien al sector industrial.
• Empresas e institutos de servicios
en el Estado. • Empresas de mantenimiento indus-
trial.
• Agroindustrias.
• Empresas comerciales

Conocido el mercado ocupacional del Ingeniero Industrial y del TSU en Higiene y Segu-
ridad, se presentan a continuación algunos compromisos que estos profesionales deben
asumir con la organización donde laboren:

a. Mantener el Status Quo (estado del momento actual). Se trata de mantener un


cierto equilibrio en algunos asuntos que involucran a dos o más partes interesa-
das en una empresa, si así fuere necesario.

b. Respetar las normas establecidas. Contribuir al respeto y cumplimiento cabal


de las leyes, ordenanzas y reglamentaciones relacionadas con el ejercicio profe-
sional.

c. Implantar cambios. En la actualidad existe una necesidad de ingenieros indus-


triales que puedan manejar cambios rápidos de tecnología y altos niveles de inno-
vación. Un ingeniero industrial observa el sistema como un todo, busca la mejor
combinación de recursos humanos, recursos naturales, equipos y estructuras
hechas por el hombre y construye el puente entre la gerencia y el nivel operativo,
motivando a la gente para lograr alcanzar cambios espectaculares.

d. Contribuir al mejoramiento continúo. Los ingenieros industriales se preocupan


por hacer las cosas mejor cada día. Con sus conocimientos ingenieriles mejoran
procesos y sistemas para incrementar la calidad y la productividad en la empresa.

51
Sánchez Marchán, Nanci. (2014).Introducción a la Ingeniería Industrial, Material Instruccional de Apoyo. Caracas: UNA

Ejemplo Ilustrativo
Se presenta un ejemplo donde se pueden identificar las fortalezas que tienen los
egresados de una carrera de acuerdo al sitio donde se desarrollen como estu-
diantes:
José, Pedro y Ana van a estudiar Ingeniería Industrial, cada uno tiene pensado
hacer algo diferente cuando se gradúen como ingenieros. José tiene pensado
montar una empresa que distribuya equipos y herramientas de trabajo. Pedro as-
pira manejar el área de producción de una compañía que desarrolle equipos de
manejo de materiales, para posicionarlos en los mejores mercados del mundo. El
papá de Ana es uno de los más importantes mayoristas de víveres del país, le ha
dicho que se va a encargar de su empresa, para que aumente su patrimonio. En
la tabla mostrada, se dan a conocer cuatro universidades que ofrecen programas
de Ingeniería Industrial, pero cada uno con un énfasis diferente. Indique ¿cuál es
el que le conviene más a cada uno de esos jóvenes estudiantes?
Cuadro de énfasis porcentual por áreas en cuatro universidades diferentes (hipo-
téticas):
Áreas de la Universidad Universidad Universidad Universidad
carrera A B C D
Ciencias básicas 15% 25% 35% 20%
Ciencias
aplicadas de la 10% 15% 15% 20%
ingeniería
Asignaturas
25% 30% 20% 30%
profesionales
Ciencias
20% 15% 15% 10%
Sociales
Administración y
35% 15% 15% 20%
finanzas

Respuesta:

En atención al énfasis de los programas de las universidades que ofrecen la carrera de


Ingeniería Industrial, a los estudiantes les conviene la siguiente alternativa:

A José le conviene estudiar en la universidad “A”, por tener un énfasis mayor en el área
de administración y finanzas, éste se piensa dedicar a la distribución o venta y requiere del
conocimiento que le proporciona esta área para desenvolverse en el mundo de los negocios.
También posee esta universidad un énfasis importante en ciencias sociales lo que le permi-
tiría conocer el comportamiento humano en sus diferentes escenarios.

Pedro debe elegir la universidad “D” por poseer mayor énfasis en las ciencias aplicadas
de la ingeniería y las asignaturas profesionales que le aportarán el conocimiento requerido
para desarrollar los mejores diseños ingenieriles y procesos productivos, requeridos para
competir con su producto nacional e internacionalmente.

52
Sánchez Marchán, Nanci. (2014).Introducción a la Ingeniería Industrial, Material Instruccional de Apoyo. Caracas: UNA

Ana al igual que José debe seleccionar la universidad “A”, donde puede desarrollar de
una forma más amplia los conocimientos en las áreas de administración y finanzas, ciencias
sociales; requeridas para incursionar y triunfar en el mundo del comercio, escenario tan
competitivo hoy día, lo que le permitiría aumentar el patrimonio de la empresa de su padre.

VALORES DEL INGENIERO INDUSTRIAL Y EL TSU EN HIGIENE Y SEGURI-


DAD INDUSTRIAL
La Ingeniería Industrial se diferencia de otras ingenierías por su amplio sentido social y
económico. Una de las áreas de la ingeniería industrial donde se hace mayor énfasis en el
aspecto social es la higiene y seguridad industrial, lo que hace inevitable la utilización y de-
sarrollo de valores muy específicos que demandan tanto los Ingenieros Industriales como el
TSU en Higiene y Seguridad Industrial. Estos valores van encaminados al desarrollo exitoso
de estos profesionales, impulsando la mejora del entorno en el cual se desenvuelven.

Estos valores son:

1. Acatamiento de normas y especificaciones convenidas con el cliente en cuanto a


calidad, cantidad, costo, tiempo, puntualidad, entrega, garantía y servicio.

2. Poseer un buen diseño, amplio conocimiento, dominio y uso adecuado de las tecno-
logías más favorables.

3. Trabajar en un ambiente SOL (Seguridad, Orden y Limpieza).

4. Lograr en los procesos calidad, repetitividad o estandarización, confiabilidad, etc.

5. Trabajar con ética, honestidad, autenticidad, honradez, responsabilidad, y pasión.

ÉTICA PROFESIONAL
La ética profesional consiste en respetar las normas y leyes de la nación en el cumpli-
miento de cuidar los bienes de la empresa y velar por los derechos de los trabajadores. Se
refiere a reforzar y promover los principios que demanda la carrera. Estos principios tienen
un amplio sentido social, ya que el fin común de los ingenieros industriales es generar un
bienestar compartido.

A continuación se presenta la última versión del Código de Ética Profesional (1996) del
Colegio de Ingenieros de Venezuela:

Se considera lo contrario a la ética e incompatible con el digno ejercicio de la profesión,


para un miembro del Colegio de Ingeniero de Venezuela:

Primero (virtudes): actuar en cualquier forma que tienda a menoscabar el honor, la


responsabilidad y aquellas virtudes de honestidad, integridad y veracidad que deben servir
de base a un ejercicio cabal de la profesión.

53
Sánchez Marchán, Nanci. (2014).Introducción a la Ingeniería Industrial, Material Instruccional de Apoyo. Caracas: UNA

Segundo (ilegalidad): violar o permitir que se violen las leyes, ordenanzas y reglamen-
taciones relacionadas con el cabal ejercicio profesional.

Tercero (conocimiento): descuidar, el mantenimiento y mejora de sus conocimientos


técnicos, desmereciendo así la confianza que al ejercicio profesional concede la sociedad.

Cuarto (seriedad): ofrecerse para el desempeño de especialidades y funciones para


las cuales no tengan capacidad, preparación y experiencia razonables.

Quinto (dispensa): dispensar, por amistad, conveniencia o coacción, el cumplimiento


de disposiciones obligatorias, cuando la misión de su cargo sea de hacerlas respetar y
cumplir.

Sexto (remuneración): ofrecer, solicitar o prestar servicios profesionales por remune-


raciones inferiores a las establecidas como mínimas, por el Colegio de Ingenieros de Vene-
zuela.

Séptimo (proyectos): elaborar proyectos o preparar informes, con negligencia o ligere-


za manifiestas, o con criterio indebidamente optimista.

Octavo (firma): firmar inconsultamente planos elaborados por otros y hacerse respon-
sable de proyectos o trabajos que no están bajo su inmediata dirección supervisión.

Noveno (obras): encargase de obras, sin que se hayan efectuado todos lo estudios téc-
nicos indispensables para su correcta ejecución, o cuando para la realización de las mismas
se hayan señalado plazos incompatibles con la buena práctica profesional.

Décimo (licitaciones): concurrir deliberadamente o invitar, a licitaciones de estudio y/o


proyectos de obras.

Décimo primero (influencia): ofrecer, dar o recibir comisiones o remuneraciones in-


debidas y solicitar influencias o usar de ellas para la obtención u otorgamiento de trabajos
profesionales, o para crear situaciones de privilegio en su actuación.

Décimo segundo (ventajas): usar de las ventajas inherentes a un cargo reenumerado


para competir con la práctica independiente de otros profesionales.

Décimo tercero (reputación): atentar contra la reputación o los legítimos intereses de


otros profesionales, o intentar atribuir injustificadamente la comisión de errores profesionales
a otros colegas.

Décimo cuarto (intereses): adquirir intereses que, directa o indirectamente colindan


con los de la empresa o cliente que emplea sus servicios o encargase sin conocimiento de
los interesados, de trabajos en los cuales existan intereses antagónicos.

Décimo quinto (justicia): contravenir deliberadamente los principios de justicia y leal-


tad en sus relaciones con clientes, personal subalterno y obreros, de manera especial, con
relación a estos últimos, en lo referente al mantenimiento de condiciones equitativas de tra-
bajo y a su justa participación en las ganancias.

54
Sánchez Marchán, Nanci. (2014).Introducción a la Ingeniería Industrial, Material Instruccional de Apoyo. Caracas: UNA

Décimo sexto (el ambiente): intervenir directa o indirectamente en la destrucción de


los recursos naturales u omitir la acción correspondiente para evitar la producción de hechos
que contribuyen al deterioro ambiental.

Décimo séptimo (extranjeros): actuar en cualquier forma que permita o facilite la con-
tratación con profesionales o empresas extranjeras, de estudios o proyectos, construcción,
inspección y supervisión de obras, cuando a juicio del Colegio de Ingenieros, exista en Ve-
nezuela la capacidad para realizarlos.

Décimo octavo (autoría): utilizar estudios, proyectos, planos, informes u otros docu-
mentos, que no sean del dominio público, sin la autorización de sus autores y/o propietarios.

Décimo noveno (secreto): revelar datos reservados de índole técnico, financiero o pro-
fesional, así como divulgar sin la debida autorización, procedimientos, procesos o caracte-
rísticas de equipos protegidos por patentes o contratos que establezcan las obligaciones de
guardas de secreto profesional. Así como utilizar programas, discos, cintas u otros medios
de información, que no sea de dominio público, sin la debida autorización de sus autores y/o
propietarios, o utilizar sin autorización de códigos de acceso de otras personas, en provecho
propio.

Vigésimo (experimentación y servicios no necesarios): someter a su cliente o a su


empleador a la aplicación de materiales o métodos en experimentación, sin su previo y total
conocimiento y aprobación o recomendarle servicios no necesarios.

Vigésimo primero (publicidad indebida): hacer o permitir cualquier publicidad no ins-


titucional, dirigida a atraer al público hacia la acción profesional, personal o participar en pro-
gramas de televisión, radio u otros medios, que no tengan carácter divulgativo profesional,
o que en cualquier forma, atenten contra la dignidad y seriedad de la profesión. Así como,
valerse de posición para proferir declaraciones en los medios o hacer propaganda de mate-
riales, equipos y tecnologías.

Vigésimo segundo (actuación gremial): incidir con lo dispuesto en las “Normas de


Actuación Gremial del CIV”.

RELACIÓN DE LA INGENIERÍA INDUSTRIAL CON OTRAS DISCIPLINAS Y


RAMAS DE LA INGENIERÍA
Relación con algunas disciplinas

El Ingeniero Industrial iniciando el pregrado es preparado de modo similar a otros inge-


nieros. Estudian los mismos cursos básicos en matemática, física, química, humanidades
y ciencias sociales. La base en matemáticas y el conocimiento de los principios de física y
química deben ser consistentes, como herramientas necesarias para ayudarse en la resolu-
ción de problemas de la práctica ingenieril. También debe tomar algunas asignaturas de las
ciencias físicas básicas de la ingeniería tales como: termofluidos, electrotecnia, entre otras.

55
Sánchez Marchán, Nanci. (2014).Introducción a la Ingeniería Industrial, Material Instruccional de Apoyo. Caracas: UNA

Se presenta seguidamente una tabla donde se puede observar la definición de cada


una de las disciplinas mencionadas, además de la relación que guardan éstas con la Inge-
niería Industrial:

Tabla 4. Relación de la Ingeniería Industrial con otras disciplinas:

Disciplinas Definición Relación


La matemática (del latín Los cursos iníciales de la ingeniería industrial
mathematĭca, y éste del grie- emplean modelos matemáticos como conec-
go μαθηματικά, derivado de tor central para entender sus sistemas; lo que
μάθημα, conocimiento) es una le permite abordar con éxito asignaturas pos-
Matemática

ciencia que estudia las propie- teriores de ciencias específicas de ingeniería


dades y relaciones cuantitati- industrial tales como: los elementos tradicio-
vas entre los entes abstractos nales de la producción, análisis económico,
(números, figuras geométricas, planeación de la producción, diseños de re-
símbolos)así como su evolución cursos, manejo de materiales, procesos y sis-
en el tiempo. Partiendo de axio- temas de fabricación, análisis de puestos de
mas y siguiendo el razonamien- trabajo, y así sucesivamente.
to lógico.
Se define como la ciencia que El estudio de la física es la base para el de-
examina los conceptos funda- sarrollo de cualquier ingeniería, y la Ingeniería
mentales de la materia, la ener- Industrial no escapa a esta aseveración, pues
Física

gía, el tiempo y el espacio, así sin ella no se podría construir absolutamente


como las relaciones que hay nada que sea seguro. Se aplica en la solución
entre ellos. de problemas prácticos y retos técnicos del
sector productivo del país.
Del egipcio kēme (“tierra”), es La química permite a la Ingeniería Industrial el
la ciencia que se dedica al es- estudio de los métodos de producción de reac-
tudio de la composición, la es- tivos químicos de manera económica y más
Química

tructura y las propiedades de la beneficiosa, con un impacto menor al medio


materia, junto a los cambios que ambiente. También se logra el diseño, mante-
experimenta durante las llama- nimiento y operación de equipos de procesos
das reacciones químicas. químicos para evitar que las sustancias se
descompongan.

Humanidades (del latín humanis- Las humanidades constituyen una herra-


tas) son el conjunto de disciplinas mienta para el Ingeniero Industrial, a fin de
relacionadas con el conocimiento que éste pueda ubicarse históricamente en
humano y la cultura, cada cultura el mundo contemporáneo para que, con
experimenta la naturaleza de cierta sentido crítico y analítico, cumpla cabal-
Humanidades

manera, y lo va a expresar a través mente con su compromiso en la sociedad,


de la religión, la ciencia, la tecno- proporciona los juicios para poder tomar
logía, la filosofía, las artes, etc. La decisiones, asumir posiciones y aceptar res-
experiencia puede ser interna, o ponsabilidades frente a los valores que con-
sea consigo, o también puede ser dicionan la vida. Es decir la misión de las dis-
externa, es decir, con la naturaleza, ciplinas del comportamiento humano para la
las cosas y el entorno. Ingeniería Industrial es ayudar a construir y
transmitir un mayor conocimiento en lo políti-
co, económico, social y cultural.

56
Sánchez Marchán, Nanci. (2014).Introducción a la Ingeniería Industrial, Material Instruccional de Apoyo. Caracas: UNA

Son todas aquellas ciencias que Con las ciencias sociales el Ingeniero In-
se ocupan del estudio del aspecto dustrial puede conocer el comportamiento
social del hombre, como su com- de los seres humanos en la sociedad, tan-
portamiento, y su desempeño como to desde una perspectiva individual como
Ciencias Sociales

individuo dentro de la sociedad. La grupal. Este comportamiento se puede ob-


transformación constante del mun- servar a través de la organización social
do actual y la adaptación del indi- haciendo uso de la política, la sociología, el
viduo han impuesto a las ciencias derecho, la antropología, la geografía, entre
sociales nuevos retos y tareas. Por otros. O considerando Las disciplinas referi-
ello, tiene como finalidad potenciar das a la organización económica, que a su
el desarrollo económico y social de vez influyen en la organización social. Por
los grupos y de las personas. último se considera las disciplinas referidas
al comportamiento del individuo como es el
caso de la psicología.

La Ingeniería Industrial constituye pues una adecuada combinación de áreas del co-
nocimiento, que permite al egresado desarrollar ampliamente su capacidad creativa en la
solución de problemas en el área de su especialidad, por ello la formación académica del
Ingeniero Industrial se caracteriza por ser integral e interdisciplinaria, es decir aplica las cien-
cias exactas y naturales en armonía con las ciencias sociales y humanas; lo que le hace
entender el lenguaje de muchas especialidades. Esto es una particularidad de esta rama de
la ingeniería.

La particularidad antes mencionada permite a los Ingenieros Industriales estar presen-


tes en equipos multidisciplinarios que ayuden a asegurar ganancias, calidad total, costos
efectivos, manejo adecuado del tiempo y satisfacción de los clientes externos e internos, a
través de la mejora continua y de planes estratégicos a corto y largo plazo. Factores apropia-
dos para el desempeño profesional dentro del proceso productivo industrial y de servicios,
demandados por el país.

El equipo multidisciplinario, se puede definir como un conjunto de personas, con


diferentes formaciones académicas y experiencias profesionales, que operan en conjunto,
durante un tiempo determinado, abocados a resolver un problema complejo, es decir tienen
un objetivo común. Cada individuo es consciente de su papel y del papel de los demás, tra-
bajan en conjunto bajo la dirección de un coordinador. La composición de equipos multidisci-
plinares es muy variada, tanto en número de disciplinas involucradas como en el número de
miembros de cada especialidad.

Este equipo debe desarrollar una actitud investigadora de carácter continuo, que parta
de un relevamiento de la situación, diagnóstico y análisis, evaluación de alternativas y pre-
sentación de propuestas concretas. Se trata, de que por la asociación de diversas discipli-
nas, se logre una sinergia en el campo de la investigación, se analice la problemática en for-
ma más profunda y se desarrollen nuevas ideas; quizá no esperadas, que permitan atender
de manera eficiente problemas complejos de investigación.

Una característica de la investigación multidisciplinaria e interdisciplinaria es la de esta-


blecer objetivos en términos de proyectos con metas que incluyan un impacto social y eco-

57
Sánchez Marchán, Nanci. (2014).Introducción a la Ingeniería Industrial, Material Instruccional de Apoyo. Caracas: UNA

nómico, más que en términos de investigaciones aisladas restringidas a una disciplina que
tienen mayor limitación en sus metas.

Principales beneficios que se obtienen con la formación de equipos multidisciplinario en


la elaboración de proyectos:
•• Mayor integración entre los diversos agentes del proceso.
•• Reducción del tiempo en la elaboración de proyectos.
•• Mejora continua del  producto y del proceso.
•• Disminución de costos.

El carácter multidisciplinario implica la integración del conocimiento de diferentes


disciplinas con la idea de buscar soluciones a tareas, situaciones o problemas
que se presentan en la sociedad. Surge como consecuencia de la transformación
del enfoque individual y de las dificultades para la resolución de los problemas
complejos, que contemplan la participación activa de varias disciplinas, hacien-
do que los especialistas interaccionen entre sí, para conformar una respuesta
coherente que sintetice los aportes particulares, logrando que éstos no marchen
separados sino incluidos en una solución única.

Relación de la Ingeniería Industrial con otras ramas de la ingeniería


Los ingenieros mecánicos, eléctricos y químicos, entre otros, son especialistas en su
área, diseñan y operan máquinas y dispositivos de su especialidad. Los Ingenieros Indus-
triales, por su formación multidisciplinaria pueden entender los términos de estos especia-
listas y controlar administrativamente los procesos generados por éstos. El control para el
Ingeniero Industrial significa proporcionar todos los insumos necesarios para la pro-
ducción, entre los cuales se encuentra: programación, control del personal operativo, dar
mantenimiento a los equipos y preocuparse por elevar la eficiencia del trabajo. En general,
todas estas tareas las viene desempeñando el ingeniero industrial, desde su creación, lo que
le permite relacionarse con otras ingenierías.

A continuación se presenta una tabla donde se puede observar la definición de algunas


ramas de la ingeniería, así como, la relación que guardan éstas con la Ingeniería Industrial:

58
Sánchez Marchán, Nanci. (2014).Introducción a la Ingeniería Industrial, Material Instruccional de Apoyo. Caracas: UNA

Tabla 5. Relación que tiene la Ingeniería Industrial con otras ramas de la ingeniería.

Rama Definición Relación


Es la aplicación de la ciencia, en Muestra los sistemas y equipos necesarios para la
particular, química, física, biología transformación física, química y bioquímica, que su-
Ingeniería y matemática, al proceso de con- fre la materia durante un proceso productivo, anali-
Química vertir materias primas o productos zando los recursos disponibles y seleccionando en-
químicos en productos más útiles, tre las distintas series de procesos, las alternativas
aprovechables o de mayor valor. que mejor conduzcan a su obtención y prevención
de la contaminación ambiental.

Es la agrupación de recursos hu- Esta disciplina le da herramientas a la Ingeniería


manos y materiales a través de los Industrial para solucionar problemas en la organiza-
cuales se recolectan, almacenan, ción a través de modelos cualitativos y/o cuantitati-
recuperan, procesan y comunican vos, simulándolos y validándolos mediante la utiliza-
Ingeniería datos e información con el objetivo ción de paquetes computacionales para solucionar
de Sistemas de lograr una gestión eficiente de problemas específicos, acción que se puede ejecu-
las operaciones de una organiza- tar porque las empresas basan su funcionamiento
ción. en un sistema de información que es el pilar sobre
el que descansa la operatividad diaria de la misma.

La Ingeniería Eléctrica se encar- El uso de los principios de las ciencias básicas y


ga de la generación, transmisión, de las ciencias de la disciplina, especialmente las
distribución y procesamiento de referentes a la investigación operacional, el mode-
Ingeniería señales eléctricas para diferentes lamiento económico, y la comprensión de las orga-
aplicaciones, tales como: energía y nizaciones, de modo que puedan desempeñarse en
Eléctrica sistemas de potencia, sistemas de forma competente en el ejercicio profesional en In-
comunicación, control automático, geniería Industrial.
electrónica, diagnóstico médico,
robótica, etc.
Procedimiento para la obtención Permite a la Ingeniería Industrial solucionar los pro-
y tratamiento de metales y no me- blemas en la obtención de metales, aleaciones, es-
tales contenidos en los minerales, tudio de las propiedades y procesos de producción
mediante procesos físicos y quími- que se suscitan en la industria manufacturera.
Ingeniería cos, pudiendo éstos formar entre sí
o con otros elementos aleaciones.
Metalúrgica La metalurgia de los metales fé-
rreos (hierro y sus aleaciones), se
denomina siderurgia, permite obte-
ner materiales para construcción y
trabajo en metal-mecánica.
Se dedica esta rama de la inge- Utiliza conocimientos científicos, tecnológicos y de
niería al estudio del diseño y cons- gestión en actividades industriales que requieren de
trucción de elementos, máquinas, equipos mecánicos para la producción de bienes y
equipos y sistemas mecánicos servicios, de manera segura, eficiente y rentable.
Ingeniería para ponerlos al servicio de la hu-
Mecánica manidad, para fomentar el desa- Esta disciplina también soluciona los problemas de
rrollo social y económico de ésta. mantenimiento de máquinas y dispositivos mecáni-
cos en el área de producción de la empresa.

El campo de la Ingeniería Industrial tiene una amplia aplicación, por lo que también se
subdividió en una serie de especialidades como son: ingeniero en procesos de manufactura,
industrial administrador, industrial en administración y planeación de la producción, industrial
en control de calidad, industrial en sistemas, industrial en evaluación de proyectos y otras.
Hay que enfatizar que esta es una de las especialidades de la ingeniería que no sólo está

59
Sánchez Marchán, Nanci. (2014).Introducción a la Ingeniería Industrial, Material Instruccional de Apoyo. Caracas: UNA

relacionada con otras ingenierías en la misma industria, sino que está en contacto con todas
las áreas de la industria distintas de la ingeniería, por ello el énfasis como disciplina multidis-
ciplinaria.

A continuación se presenta una situación donde se puede observar este énfasis multi-
disciplinario de la Ingeniería Industrial.

Ejemplo Ilustrativo
El dueño de una empresa de productos lácteos, ubicada en el estado Apure ha de-
cidido crear otra planta en el estado Carabobo, para introducir nuevos productos
que le permitan aumentar su competitividad en el mercado. ¿Qué equipo de pro-
fesionales se requiere para la instalación y puesta en marcha de la nueva planta?

El equipo de profesionales necesario para emprender esta encomiable labor es el si-


guiente:

• Economistas. Estudian la factibilidad de introducir en el mercado los nuevos pro-


ductos y realizan los primeros cálculos de la inversión requerida para la puesta en
marcha. Estas actividades también pueden ser asumidas por el ingeniero Industrial,
dado su formación interdisciplinaria.

• Administrador. Representa a la organización, coordinando en el tiempo los recur-


sos disponibles (actuales y futuros; materiales, humanos e intangibles) en función de
las necesidades operativas en directa relación a los objetivos deseados. Puede ser
asumida también esta función por el ingeniero Industrial.

• Abogados. Preparan los documentos de constitución de la empresa, para luego


solicitar los créditos indispensables para la operación.

• Ingeniero Químico. Determinará el procedimiento requerido para no perder o alte-


rar los compuestos que definen las propiedades alimenticias de la leche.

• Ingeniero Mecánico. Se encargará de especificar las características del equipo y


máquinas requeridos para el procesamiento de la leche.

• Ingeniero Industrial. Determina la mejor ubicación de la planta tomando en consi-


deración la disminución de los costos de transporte de la materia prima a la planta
y de distribución de productos terminados a los mercados, también considerará la
existencia de mano de obra necesaria en el proceso. El Ingeniero Industrial se en-
cargará por otra parte de la organización de la empresa para hacerla funcional y
flexible en cuanto a su administración y dirección. Determinará el número óptimo de
operarios, empleados, máquinas y materia prima a almacenarse, para que el proce-
so productivo se lleve a cabo eficientemente y la planta opere todo el año.

60
Sánchez Marchán, Nanci. (2014).Introducción a la Ingeniería Industrial, Material Instruccional de Apoyo. Caracas: UNA

• Ingeniero Agrónomo. Mejora la calidad de los procesos de la producción de pastos


requeridos para la alimentación de las vacas, fundamentados en principios científi-
cos y tecnológicos

• Médico Veterinario. Propone la receta para una dieta equilibrada que aumente la
producción de leche de las vacas.

• Ingeniero de sistemas. Desarrolla los sistemas de información y programas, para


optimizar la solución de problemas en la empresa.

El equipo multidisciplinario requiere de la motivación, la participación y también


del consenso activo de distintos profesionales interrelacionados en un determi-
nado medio, donde en ocasiones surgen liderazgos que irradian valores; que
luego se vuelven compartidos y en otras oportunidades se suscitan hechos, con-
diciones o incluso síntomas a partir de los cuales el grupo toma conciencia de lo
que viene ocurriendo, para así solucionar los problemas y normalizar las distintas
situaciones que plantean los clientes. 

RESUMEN DE LA UNIDAD Nº 1
Tanto en el pasado como en la actualidad, la mayoría de los campos de actividad eco-
nómica han recibido apoyo de la ingeniería.

Estos aportes se pueden observar en culturas como la egipcia, la mesopotámica, la


romana, la griega, la oriental y la europea, donde se consigue verificar el adelanto de la
ingeniería, en pro de la búsqueda de protección, bienestar y supervivencia del individuo. La
ingeniería se sustenta en la ciencia, la técnica, la tecnología y el arte, para lograr avances y
realizar importantes aportes a sus diferentes ramas.

La Ingeniería Industrial, no está exenta de estos aportes, para la cual el sistema huma-
no constituyó el aspecto más importante para su desarrollo. Se formó de manera tardía con
respecto a otras ramas de la ingeniería.

Esta disciplina trata sobre el diseño, mejoramiento e instalación de sistemas integrados


de hombres, materiales y equipos, en beneficio de la sociedad. Entre las personas que ge-
neraron grandes aportes a la ingeniería industrial se encuentran:

Sir Richard Arkwright, Adam Smith, Frederick W. Taylor, los esposos Gilbreth, Henry
Ford. También se pueden citar personajes como Henry L. Gant, Henry Fayol, Eli Whitney,
entre otros.

En Venezuela aparece la primera escuela de Ingeniería Industrial, en la universidad de


Carabobo, en el año de 1958. Después, surgieron escuelas en las siguientes universidades:
La Universidad del Zulia (LUZ), el Instituto Politécnico Luis Caballero Mejias hoy UNEXPO,
la Universidad Nacional Experimental del Táchira, la Universidad Experimental Francisco de
Miranda en Coro, la Universidad Nacional Abierta, entre otras.

61
Sánchez Marchán, Nanci. (2014).Introducción a la Ingeniería Industrial, Material Instruccional de Apoyo. Caracas: UNA

El Ingeniero Industrial requiere una base consistente en matemáticas y el conocimiento


de los principios de física y química, como herramientas necesarias para ayudarse en la
resolución de problemas de la práctica ingenieril. También debe tomar algunas asignaturas
de las ciencias físicas básicas de la ingeniería como termofluidos y electrotécnica, así mis-
mo asignaturas de humanidades y ciencias sociales, para fortalecer su formación integral o
multidisciplinaria.

La Ingeniería Industrial ha evolucionado, experimentando diversos cambios, pasando


de los métodos mecánicos a los métodos electrónicos, de procedimientos de diseño cualitati-
vo a nuevas técnicas que requieren modelación, simulación y amplio empleo de estadísticas;
de un enfoque centrado en la producción a un enfoque integrador de sistemas. En la Univer-
sidad Nacional Abierta (UNA), conscientes de este reto, se orienta el perfil de las carreras
hacia la realización de actividades en las áreas operativa y de gerencia. También se refuer-
zan aspectos como la ética y los valores de los estudiantes para lograr un egresado integro.

El mercado laboral del Ingeniero Industrial y del TSU en Higiene y Seguridad Industrial
es amplio, se pueden desempeñar tanto en el sector público como en el privado, en el campo
de la docencia, investigación, asesoría, diseño y control de sistemas productivos de bienes
o servicios entre otros.

Se diferencian estas carreras, de otras profesiones por su amplio sentido social. Una de
las áreas de la Ingeniería Industrial donde se hace mayor énfasis en el aspecto social es la
higiene y seguridad industrial, lo que hace inevitable la utilización y desarrollo de valores muy
específicos que demandan tanto los ingenieros industriales como el TSU en Higiene y Segu-
ridad Industrial. Estos valores están encaminados al desarrollo exitoso de estas profesiones
y a la mejora del entorno en el cual se desenvuelven. También es de suma importancia la
ética profesional que debe prevalecer en estos profesionales, la cual consiste en respetar las
normas y leyes de la nación en el cumplimiento de velar por los derechos de los trabajadores
y cuidar los bienes de una empresa.

Seguidamente se muestra la autoevaluación de la unidad 1, así como un grupo de ejer-


cicios propuestos para que los estudiantes se ejerciten en función a la presentación de las
pruebas correspondientes al lapso académico actual.

62
Sánchez Marchán, Nanci. (2014).Introducción a la Ingeniería Industrial, Material Instruccional de Apoyo. Caracas: UNA

AUTOEVALUACIÓN DE LA UNIDAD 1
Se muestra un grupo de preguntas alusivas al tema historia de la Ingeniería Indus-
trial. Respóndalas y verifique sus respuestas, con las presentadas por la autora al
final del texto.
1. ¿Qué representa Frederick Winslow Taylor para la Ingeniería Industrial?
2. Se presenta la tarea investigar un tema, considere los conceptos que dieron
origen a la fórmula de Taylor y plantee dos métodos diferentes para realizar la
tarea, además seleccione el método que considere el mejor y más económico.
3. Escriba en el espacio señalado, la letra que identifica al autor responsable de la
acción formulada:
Desarrolló estudios sobre economía de movimientos ( )
a) Henry Gantt, b) Frank Gilbreth.
4. En qué consistieron las primeras ideas aportadas por Henry Ford en el estudio
de la Ingeniería Industrial y cómo contribuyeron en el avance de esta disciplina.
5. ¿Cuál es el aporte realizado por Eli Whitney referente a la iniciación de la Inge-
niería Industrial y en qué tipo de fábrica se introdujo estas ideas?
6. ¿Por qué se considera utilizar el enfoque multidisciplinario en la solución de los
problemas en las organizaciones?
7. ¿Cuál es la diferencia que existe entre arte y técnica y dé un ejemplo de cada
uno?
8. Indique la relación que existe entre ciencia e Ingeniería Industrial.
9. En una empresa de bienes o servicios el Ingeniero Industrial o el TSU en Higie-
ne y Seguridad Industrial puede encargarse de:
(a) Prevenir accidentes laborales.
(b) Mantener la higiene y seguridad del proceso y la empresa.
(c) Desarrollar capacidades relativas al ejercicio profesional en un área especí-
fica de las ciencias sociales.
(d) Todas las anteriores.
Seleccione la alternativa correcta y justifíquela.
10. Establezca la relación de los profesionales citados y las funciones que tienen en
una empresa. Escriba la letra que corresponda en el espacio señalado en las
diferentes funciones.
Realiza el control de inventarios en una fábrica de repuestos para ( )
Diseña las estructuras de una planta Industrial ( )
Determina los requerimientos de energía de las maquinarias y equipos ( )
a) Ingeniero Industrial, b) Ingeniero Mecánico, c) Ingeniero Eléctrico, d) Ingeniero
Químico, e) Ingeniero Civil.

63
Sánchez Marchán, Nanci. (2014).Introducción a la Ingeniería Industrial, Material Instruccional de Apoyo. Caracas: UNA

Ejercicios propuestos

Se presenta este grupo de preguntas con la intención de proporcionar al estudian-


te un material para ejercitarse aún más, antes de la presentación de las pruebas.
Estas preguntas no tienen respuestas en este texto, para su comprobación puede
apoyarse en la lectura de otros textos o buscar información en la web.
Responda las siguientes interrogantes:

1. ¿Cómo se producen las primeras manifestaciones de la ingeniería?


2. ¿Qué es Ingeniería Industrial?
3. ¿Cuál es la diferencia entre ciencia e ingeniería?
4. Cite el motivo que impulsó a Frank Gilbreth a iniciar el concepto de método estándar.
5. ¿Cuáles son los aportes de Henry Ford para la Ingeniería Industrial entre los años
(1863-1947), que permitió mejorar la eficiencia de los procesos productivos y los cos-
tos de los artículos producidos?
6. ¿Cuándo y quién construyo la primera bomba impulsada por vapor?
7. Identifique las disciplinas relacionadas con la Ingeniería Industrial
8. Describa los aportes que Charles Babbage dejó a la Ingeniería Industrial.
9. ¿Qué promovió la demanda de nuevos equipos en una sociedad en continuo desa-
rrollo?
10. Describa los tres factores que han contribuido más al desarrollo de la ingeniería.
11. Indique la relación que existe entre ciencia, técnica, tecnología, arte e Ingeniería In-
dustrial.
12. ¿Cómo fue el inicio de la revolución industrial?
13. ¿Considera que los ingenieros sólo deben abordar aspectos técnicos? Justifique su
respuesta.
14. ¿Qué hace necesario que el ingeniero se prepare para el cambiante escenario cien-
tífico?
15. Cómo se relaciona la ingeniería industrial con otras ramas de la ingeniería?
16. ¿Qué hace a la Ingeniería Industrial diferente de las otras disciplinas de la ingeniería? 
17. ¿Cuáles son las ciencias básicas para la ingeniería industrial? 
18. Describa tres de las disciplinas relacionadas con la Ingeniería Industrial: Administra-
ción, Investigación de Operaciones e Ingeniería de Sistemas.
19. ¿Cuál debe ser el perfil de un ingeniero industrial y de un TSU en Higiene y Seguridad
Industrial?
20. Explique brevemente las etapas del desarrollo de los objetos técnicos.
21. ¿Qué límites le pondría a la ingeniería para tener un desarrollo sostenible responsa-
ble?
22. ¿Cuál es la relación de las herramientas (piedra, bronce, hierro) con la ingeniera?

64
Sánchez Marchán, Nanci. (2014).Introducción a la Ingeniería Industrial, Material Instruccional de Apoyo. Caracas: UNA

MÓDULO I

ÁREA DE DESEMPEÑO DE LA INGENIERIA INDUSTRIAL

UNIDAD 2

ÁREA OPERATIVA

UNIDAD 3

ÁREA GERENCIA
Sánchez Marchán, Nanci. (2014).Introducción a la Ingeniería Industrial, Material Instruccional de Apoyo. Caracas: UNA

A continuación se desglosan cada una de estas áreas:

PRODUCCIÓN Y PRODUCTIVIDAD EN LA EMPRESA


ESTUDIO DEL TRABAJO: MÉTODOS, MOVIMIENTOS Y TIEMPO
CALIDAD EN LA EMPRESA
ÁREA SEGURIDAD E HIGIENE EN EL TRABAJO
OPERATIVA MANTENIMIENTO INDUSTRIAL DE MAQUINARIA Y EQUIPO
PROCESO DE MANUFACTURA
PREPARACIÓN DE PROYECTOS DE INVERSIÓN
LOCALIZACIÓN Y DISTRIBUCIÓN EN PLANTAS INDUSTRIALES
INVESTIGACIÓN DE OPERACIONES

ADMINISTRACIÓN DE RECURSOS
ÁREA DE
ECONOMÍA Y FINANZAS
GERENCIA
MARKETING INDUSTRIAL
COMPORTAMIENTO ORGANIZACIONAL

Una de las principales funciones de la ingeniería radica en reducir hechos a números,


los cuales pueden ser usados en fórmulas para mostrar la relación entre varias áreas de la
especialidad Ingeniería Industrial. La especialidad contempla una orientación profesional,
donde se encuentran sus áreas de desempeño, agrupadas en dos componentes. El primero
de ellos se denomina área operativa, el segundo es un componente económico-administra-
tivo o área de gerencia. Ver figura 10

Área Operativa
Áreas de
Desempeño de la
Ingeniería Industrial

Área de Gerencia

Figura 10. Áreas de desempeño de la Ingeniería Industrial.

67
Sánchez Marchán, Nanci. (2014).Introducción a la Ingeniería Industrial, Material Instruccional de Apoyo. Caracas: UNA

El área operativa, incluye el desarrollo de habilidades y el aprendizaje de herramientas


asociadas con la configuración y el diseño de ingeniería, aplicados a sistemas de producción
y a las actividades asociadas con éstos, en busca de soluciones innovadoras para los pro-
blemas generados en una empresa.

En el área de gerencia, se contempla la conformación y aplicación de herramientas


propias de la Ingeniería Industrial para la toma de decisiones de carácter económico y finan-
ciero en las organizaciones, considerando factores complejos como el riesgo y la fijación de
posiciones diversas respecto a las alternativas de decisión.

El estudio de este módulo le permitirá familiarizarse con las diferentes áreas de desem-
peño profesional que ejercerá como futuro ingeniero. Le invitamos a invertir tiempo y dedi-
cación en ese estudio, lo cual incidirá positivamente en su aprendizaje y en su formación.

68
Sánchez Marchán, Nanci. (2014).Introducción a la Ingeniería Industrial, Material Instruccional de Apoyo. Caracas: UNA

UNIDAD 2

ÁREA OPERATIVA

•• Producción y productividad en la empresa.


•• Estudio de métodos, movimientos y tiempo.
•• Calidad en la empresa.
•• Seguridad e higiene en el trabajo.
•• Mantenimiento industrial de maquinaria y equipo.
•• Proceso de manufactura.
•• Preparación de proyectos de inversión.
•• Localización y distribución en plantas industriales.
•• Investigación de operaciones.
Sánchez Marchán, Nanci. (2014).Introducción a la Ingeniería Industrial, Material Instruccional de Apoyo. Caracas: UNA

PRODUCCIÓN Y PRODUCTIVIDAD EN LA EMPRESA

Objetivo:
Indicar la importancia de la producción eficaz y eficiente de bienes y servicios en
una empresa, para lograr su mayor productividad.

LA EMPRESA
La Empresa:
•• Definición.

Eficacia y Eficiencia
•• Diferencias entre eficacia y eficiencia.
•• Relación entre eficiencia y eficacia.
•• Situaciones que pueden presentarse en una organización, cuando se
mide la eficacia y la eficiencia.
•• Clasificación de las empresas.
•• Funciones de la empresa.
•• Toma de decisiones en la empresa.

Factores y áreas funcionales de la empresa:


•• Organización de la empresa.

Aspectos generales a considerar en la organización de la empresa:


•• Estructura y niveles de la organización.
•• Instrumentos metodológicos de la organización.
•• Manuales administrativos.
•• Estructura organizativa.

El funcionamiento económico de la empresa.


Pasos necesarios para crear una empresa.
Sánchez Marchán, Nanci. (2014).Introducción a la Ingeniería Industrial, Material Instruccional de Apoyo. Caracas: UNA

Antes de abordar los diferentes temas que la producción y el resto de las áreas que
toma en cuenta la Ingeniería Industrial, se considerarán aspectos de una empresa, que per-
mitirán al estudiante poder distinguir estas áreas en ella, así como, su aplicabilidad en el
logro de obtener la eficacia y eficiencia en todos los procesos generados en la creación de
bienes y servicios.

LA EMPRESA

Definición
La empresa está compuesta por una serie de elementos, los cuales tienen su importan-
cia y connotación: empresario, factor humano, factores de producción, recursos económicos
y financieros, organización, relaciones con el entorno, marco técnico, económico, político,
social y por supuesto los objetivos, entre los cuales se consideran la obtención y distribución
de lucro.

Para llevar bien una empresa es necesario acelerar su buena marcha aplicando siste-
mas eficaces y eficientes que guarden equilibrio entre sí. Esto implica buscar maneras de
mejorar la productividad sin sacrificar la calidad de los productos (bienes y servicios) que
ofrece la empresa. La efectividad de cómo se administre la empresa, depende de la coordi-
nación balanceada entre las etapas del proceso administrativo y productivo.

La empresa se puede definir como:

Una entidad económica, en donde se coordinan factores de producción, financieros y co-


merciales de manera eficaz y eficiente, para obtener los bienes y servicios que se ofrecen
en el mercado. Implica una dimensión social importante como generadora de empleo y
riqueza, también es impulsora de las grandes innovaciones, además de compleja, cam-
biante y diversa.

De esta definición se pueden extraer dos términos importantes, los cuales tienen que
ver con el buen funcionamiento de la empresa y el logro de la productividad en ella. Estos
términos son eficacia y eficiencia, los cuales, se detallan seguidamente.

EFICACIA Y EFICIENCIA

Eficiencia (del latín effiicienta) acción, fuerza y virtud de producir. Hace énfasis en los
medios, hacer las cosas correctamente, resolver problemas para lograr ahorrar gastos, cum-
plir tareas y obligaciones, capacitar a los empleados. En la eficiencia se aplica un enfoque
reactivo.

Eficacia o efectividad (del latín efficax) que tiene el poder de producir el efecto deseado
en el menor tiempo. Hace énfasis en los resultados, en hacer las cosas correctas, lograr ob-
jetivos, crear más valores principalmente para el cliente. Se aplica aquí un enfoque proactivo,
en lugar de reaccionar, anticiparse a los acontecimientos.

73
Sánchez Marchán, Nanci. (2014).Introducción a la Ingeniería Industrial, Material Instruccional de Apoyo. Caracas: UNA

El planteamiento esencial de la eficiencia es la relación entre insumos (gastos) y resul-


tados (ingresos), su expresión es la reducción de los costos. Hay que producir con los costos
más bajos posibles, o dicho de otra forma, obtener los mayores resultados con los mismos
recursos. La eficiencia actúa en el ámbito interno de la empresa, ahí es donde se puede
trabajar para reducir gastos.

La eficacia considera el nivel de contribución al cumplimiento de los objetivos estratégi-


cos de la empresa, sólo puede rendir resultados dentro del marco de su definición.

En definitiva la pregunta básica de la eficiencia es ¿Cómo hacer mejor las cosas? la de


la eficacia es ¿Qué es lo que se debería estar haciendo?

Peter Drucker, dice que “un líder debe tener un desempeño eficiente y eficaz a la vez,
pero aunque la eficiencia es importante, la eficacia es aún más decisiva”, de igual forma ma-
nifiesta que la clave del éxito de una organización es la eficacia.

La práctica indica que el éxito y estabilidad a largo plazo de la empresa depende más
de los progresos en su eficacia, que en su eficiencia. La empresa debe primero identificar
qué es lo más conveniente que debe hacer para aprovechar una oportunidad o neutralizar
una amenaza; después, tratar de hacerlo de la forma más eficiente posible (con el mínimo
de gastos). Pero, no se puede limitar a esto, pues la competencia está haciendo lo mismo,
por tanto, debe tratar de innovar constantemente, de asumir el cambio como una necesidad.

Relación entre eficiencia y eficacia


La relación entre la eficacia y eficiencia se puede observar en la siguiente tabla:

Tabla 6. Relación entre eficacia y eficiencia


  
Objetivos Relación objetivos/
Eficacia= resultados bajo
Resultados Ambos conceptos requieren de
condiciones ideales estrategias operacionales tales
como: planes, políticas, reglas,
Relación recursos/ procedimientos y acción.
Recursos
Eficiencia= resultados bajo
Resultados condiciones reales

Objetivos Planes Políticas Reglas y Procedimientos Acción


Resultados
Eficiencia
Eficacia

74
Sánchez Marchán, Nanci. (2014).Introducción a la Ingeniería Industrial, Material Instruccional de Apoyo. Caracas: UNA

La relación más ilustrativa es la del esfuerzo y los resultados. Si aparecen más resulta-
dos al realizar una actividad o trabajo a menor costo, menor tiempo, sin desperdiciar recur-
sos y con la fijación de un alto nivel de calidad, la eficiencia se incrementará.

En las organizaciones no es suficiente ser eficientes, las organizaciones modernas bus-


can algo más que eso, y eso es la eficacia. Cuando una organización hace uso de estrate-
gias operacionales para alcanzar los resultados se dice que es eficiente. Pero si además de
alcanzar resultados, logra los objetivos, entonces se dice que es eficaz. Ambas tienden a
complementarse. Para ser eficaz, es preciso ser eficiente, dar prioridad a las tareas que se
deben realizar asegurándose que lo que se realiza es viable, contiene una adecuada finali-
dad y muestra calidad en los resultados.

Situaciones que pueden presentarse en una organización, cuando se mide la eficacia y


eficiencia. Véase figura 11.

Sobre la base de los conceptos de eficacia y eficiencia estudiados, se pueden presentar


las siguientes situaciones:
ALTA

MUY EFICIENTE
MUY EFICIENTE
POCO EFICAZ
MUY EFICAZ
Administra bien los re-
Logra los objetivos y admi-
cursos, pero no logra los
EFICIENCIA

nistra bien los recursos


objetivos
POCO EFICIENTE
POCO EFICIENTE
MUY EFICAZ
POCO EFICAZ
Logra los objetivos sin
No logra los objetivos, ni
escatimar los recursos utili-
administra bien los recursos
zados para alcanzarlo
BAJA

BAJA EFICACIA ALTA

Figura 11. Medida de la eficacia y eficiencia en la organización.

La incidencia que puede tener la eficacia y la eficiencia en una organización, de acuer-


do con la figura exhibida, depende del lugar donde se espera se ubique la misma, a fin de
medir su operatividad en cuanto a la administración de sus recursos y al logro de sus objeti-
vos. Para que una organización funcione debe tener al menos un nivel mínimo de eficacia y
eficiencia en sus procedimientos. Lo peor que le puede ocurrir a una organización es tener
bajos niveles de eficiencia y eficacia, pues sus posibilidades de competir en el mercado y
subsistir en este mundo altamente globalizado serían mínimas.

75
Sánchez Marchán, Nanci. (2014).Introducción a la Ingeniería Industrial, Material Instruccional de Apoyo. Caracas: UNA

La forma en cómo se obtiene un conjunto de resultados refleja la eficacia, mien-


tras que la forma en cómo se utilizan los recursos para lograrlos se refiere a la
eficiencia.

Se presentan dos ejemplos ilustrativos de cómo medir la eficacia y eficiencia en una


empresa:

Ejemplo Ilustrativo Nº 1

Eficacia y eficiencia en el proceso y los resultados de la empresa ESCOM


S.A.

El gerente general de la empresa ESCOM S.A, solicita con urgencia al gerente de


producción el informe anual respecto al desempeño del personal a su cargo, con
la finalidad de considerar la posibilidad de un ascenso para los integrantes de ese
equipo de trabajo. El gerente de producción le da las pautas a su secretaria para
que lo elabore y se lo entregue antes de finalizar la tarde. Después de una ardua
tarea la secretaria entrega el informe a su jefe, el cual lo revisa con detenimiento.
El gerente de producción se dirige hasta donde está ubicada la secretaria para
agradecerle y elogiar su trabajo, pero al acercarse observa la gran cantidad de bo-
rradores en la papelera, como parte del desperdicio generado durante el proceso.

¿Se puede decir que la secretaria y el gerente actuaron de manera eficaz y


eficiente?

La secretaria alcanzó un resultado eficaz al entregar el informe a tiempo y de acuerdo


a las pautas exigidas por su jefe. Logró alcanzar el objetivo. En cuanto al proceso, ella creó
desperdicio al utilizar los recursos para realizar el trabajo; por lo tanto no actuó de manera
eficiente. El gerente al igual que la secretaria logró alcanzar el objetivo, es decir, obtener el
informe a tiempo, pero tampoco actuó de manera eficiente, ya que se limitó a entregar las
pautas a la secretaria, pero no le brindó apoyo, ni supervisó el trabajo en pro de disminuir el
desperdicio.

En todo proceso se debe tratar de utilizar la menor cantidad de recursos posible, para
lograr disminuir los costos, sin desmejorar la calidad en el resultado. En conclusión es impor-
tante actuar de manera eficaz y eficiente simultáneamente, para lograr disminuir los costos,
alcanzar los objetivos y excelentes resultados.

76
Sánchez Marchán, Nanci. (2014).Introducción a la Ingeniería Industrial, Material Instruccional de Apoyo. Caracas: UNA

Ejemplo Ilustrativo Nº 2
Caso de eficacia e ineficiencia en una empresa de servicios
La compañía “MI HOGAR”, que opera una gran cantidad de restaurantes en el
país, abrió su primer “CHOCOHOGAR” en la ciudad de Mérida y está en vías de
inaugurar otros “CHOCOHOGAR” en algunas otras regiones del país de clima
similar al merideño, todo ello para aumentar sus utilidades en un 70%. Con la inau-
guración del resto de los CHOCOHOGAR, se superó las expectativas de aumento
de las utilidades en un 20%. Sin embargo, un sondeo efectuado entre sus clientes
reflejó que el servicio proporcionado por la compañía tiene grandes deficiencias.
Entre las que se pueden mencionar las siguientes: empleados malhumorados,
locales sucios, errores en el despacho de órdenes y lentitud en el servicio.

En la empresa “MI HOGAR”, se logró alcanzar el objetivo al conseguir aumentar las uti-
lidades y superar las expectativas, lo que demuestra su eficacia, pero el servicio proporcio-
nado por la compañía tiene grandes fallas, lo que manifiesta una gran ineficiencia. De lo que
se puede deducir, que no basta alcanzar el objetivo del aumento de utilidades, si a cambio
se está sacrificando la imagen, prestigio y lealtad de los clientes, que actualmente no suelen
tolerar ser ignorados por mucho tiempo.

Aclarados los conceptos de eficacia y eficiencia se proseguirá con el estudio del fun-
cionamiento de la empresa.

CLASIFICACIÓN DE LAS EMPRESAS


Por su finalidad: Por su función básica:
• Lucrativas: buscan beneficios económicos, • Industriales: transforman la mate-
se crean para producir bienes y servicios ria prima.
rentables y están constituidas por personas
que desean multiplicar su capital y obtener • Comerciales: compra/venta de
utilidades denominadas dividendos. productos.
• No lucrativas: son las que no persiguen fines • Servicios: asistencia a la comuni-
de lucro. Manejan recursos, obtienen benefi- dad.
cios pero no obtienen utilidades.

Por su actividad u objeto: Por la titularidad del capital:


• Sector primario: agropecuarias y extractivas. • Pública: sus objetivos son socia-
• Sector secundario: industriales y construc- les más que económicos, en pro de
ción. una comunidad.
• Sector terciario: servicios, transporte, edu-
• Privada: ésta genera beneficios
cación, seguros, entre otros.
económicos, sus objetivos están
más centrados en las utilidades
que en lo social.

77
Sánchez Marchán, Nanci. (2014).Introducción a la Ingeniería Industrial, Material Instruccional de Apoyo. Caracas: UNA

Por su naturaleza jurídica:

Todas las empresas deben contar con una forma jurídica que viene determinada por el
número de personas que la integran en su fundación, por el capital aportado y por el tamaño.
Entre estas formas podemos distinguir las siguientes

a) Empresario individual: cuando el propietario de la empresa es la única persona


que asume todo el riesgo y se encarga de la gestión del negocio.

b) Sociedades: cuando diversas personas deciden invertir en una empresa pueden


formar una sociedad. En la actualidad se pueden distinguir los siguientes tipos de
sociedades:
•• Sociedad Anónima (S.A.): el capital está formado por partes proporcio-
nales denominadas acciones que son valores negociables, y el socio es
el propietario de la acción en cada momento.
•• Sociedad de Responsabilidad Limitada (S.R.L.): la responsabilidad de los
socios se limita al capital aportado. El volumen total de dicho capital se
divide en partes proporcionales. No se pueden negociar sin el consenti-
miento de todos los socios.
•• Sociedad Regular Colectiva (S.R.C.): se caracteriza porque los socios
asumen una responsabilidad solidaria e ilimitada.
•• Sociedad Comanditaria (“S.Com.”): además de tener socios colectivos,
cuentan con un conjunto de socios que aportan un capital y su responsa-
bilidad se limita al capital aportado.
•• Sociedad Anónima Laboral (S.A.L.): son aquellas en las que los trabaja-
dores son los dueños de la propia empresa. Su responsabilidad se limita
al capital aportado y nadie puede tener más del 25% (para evitar que
ningún trabajador sea jefe).
•• Sociedad Cooperativa: supone la agrupación de pequeños empresarios
con el objetivo de hacerse más fuerte en el mercado. Comparten por
igual riesgos y beneficios.

Por su dimensión:
•• Micro empresa.
•• Pequeña y Mediana Empresa (PYME).
•• Grande.
•• Macroempresa.

Los factores que sirven para realizar esta clasificación son: capital social, número de
empleados y tecnología. En varios países se toma en cuenta un límite numérico de personas
que puede emplear la empresa para ser considerada como pequeña, mediana o grande. Ese
límite varía de un país a otro y de una industria a otra.

78
Sánchez Marchán, Nanci. (2014).Introducción a la Ingeniería Industrial, Material Instruccional de Apoyo. Caracas: UNA

El límite numérico más usado se muestra en la siguiente tabla:


Tabla 7. Tamaño y límite numérico de la empresa

Tamaño de la empresa Límite numérico

Una empresa que realiza una actividad regular y cuenta con


Microempresa
pocos recursos. Menos de 10 trabajadores.

Pequeña Menos de 50 trabajadores.

Son aquellas que cuentan con una plantilla menor a 250 traba-
Mediana
jadores.
Grande Menos de 500 trabajadores.
En estas empresas se pueden ubicar a las corporaciones,
transnacionales, consorcios. El número de trabajadores es su-
perior a 500. Una empresa de este tipo puede tener millones de
Macroempresa trabajadores repartidos en gran parte del mundo, por ejemplo:
Coca-Cola, McDonald, Nestlé, entre otras.

Para realizar esta clasificación los países consideran la realidad económica, social y
demográfica con la cual coexiste la empresa. Sin embargo, esta clasificación es adoptada
por la mayoría de ellos.

Por el origen del capital:


•• Nacionales: empresa pública constituida en forma societaria, controlada
por el Estado o por sus organismos y de las que se vale la administración
para la intervención directa en las actividades económicas.
•• Extranjeras: es una sociedad de responsabilidad limitada establecida
exclusivamente con capital extranjero.
•• Multinacionales o transnacionales: están establecidas en su país de
origen y también se constituyen en otros países, para realizar sus activi-
dades mercantiles tanto de venta y compra como de producción.

La pequeña empresa ha demostrado a lo largo del tiempo y en la mayoría de


países (desarrollados o no) que es un componente importante de la economía, y
en muchos casos, es la base de las medianas y grandes empresas.

Toda empresa se ha iniciado mediante cierto financiamiento, el cual puede provenir de:

a) Fondos propios: cuando el capital pertenece a los dueños y/o a los propietarios.

b) Fondos ajenos: financiamiento de capital externo a los propietarios, mediante


préstamos, para lo cual necesita una garantía o recursos que respaldan el prés-

79
Sánchez Marchán, Nanci. (2014).Introducción a la Ingeniería Industrial, Material Instruccional de Apoyo. Caracas: UNA

tamo del deudor para adquirir el financiamiento y delimitar las condiciones en las
cuales se cancelará la deuda.

Si la empresa posee un status en cuanto a la actualización, es decir, innovación, tecno-


logía que utiliza, etc., será más fácil la obtención de ese financiamiento.

Actividad propuesta
A continuación se presenta una serie de preguntas referentes a la clasificación de
las empresas. Seleccione una de las opciones de respuesta para cada pregunta.
1. ¿Cuáles empresas buscan beneficios económicos y se crean para producir bienes y
servicios rentables?
a) No lucrativas.
b) Lucrativas.
c) Constitutivas.
d) Mayorista.
2. ¿Cuál empresa brinda asistencia a la comunidad?
a) Empresa industrial.
b) Empresa de servicios.
c) Empresa pesquera.
d) Empresa comercial.
e) Empresa manufacturera.
3. Sociedad que no se pueden negociar sin el consentimiento de todos los socios.
a) Sociedad Anónima.
b) Sociedad de Responsabilidad Limitada.
c) Sociedad Regular Colectiva.
d) Sociedad Comanditaria.
e) Sociedad Anónima Laboral.
4. ¿Cuál empresa pertenece al sector primario?
a) Industriales y construcción.
b) Agropecuarias y extractivas.
c) Servicios, transporte, educación.
d) Bancos, seguros.
5. ¿Cuántos tipos de empresas pueden existir de acuerdo con su tamaño?
a) 2.
b) 3.

80
Sánchez Marchán, Nanci. (2014).Introducción a la Ingeniería Industrial, Material Instruccional de Apoyo. Caracas: UNA

c) 4.
d) 5.
e) 8.
6. ¿Qué tipo de empresa puede tener aproximadamente 100 trabajadores?
a) Mediana.
b) Grande.
c) Pequeña.
d) Microempresa.
e) Macroempresa.
7. Cuando una empresa tiene más de 500 trabajadores se dice que es:
a) Pequeña.
b) Mediana.
c) Grande.
d) Microempresa.
e) Macroempresa.
8. ¿Cuál empresa es una sociedad de responsabilidad limitada establecida
exclusivamente con capital extranjero?
a) Nacionales.
b) Extranjera.
c) Multinacionales.
9. Cuando el capital de una empresa pertenece a los dueños y/o a los propietarios se
dice que esta empresa trabaja con fondos:
a) Del Estado.
b) De capital externo a los propietarios.
c) Propios.
10. Cuando una empresa puede tener entre 300 y 500 trabajadores se le conoce con
el nombre de:
a) Pequena.
b) Mediana.
c) Grande.
d) Microempresa.
e) Macroempresa.

81
Sánchez Marchán, Nanci. (2014).Introducción a la Ingeniería Industrial, Material Instruccional de Apoyo. Caracas: UNA

FACTORES Y ÁREAS FUNCIONALES DE LA EMPRESA

Factores de producción de la empresa


Los factores de producción son insumos y elementos que se requieren para producir
una mercancía o servicio, identificándolos como:
•• Tierra
•• Trabajo 
•• Capital
Tierra. Espacio en el que se desarrolla el proceso productivo, concibiéndose como tal
el lugar donde se encuentra, por ejemplo, la empresa, la industria y los recursos naturales.

Trabajo. Es el esfuerzo físico y mental para la producción  de bienes y servicios.

Capital. Son los bienes con los que cuenta la empresa, que le permiten generar un
ingreso. Se pueden dividir en bienes de capital y bienes financieros.

Entre los bienes de capital se encuentran la tierra, los edificios, la maquinaria, los pro-
ductos almacenados, las materias primas que se posean, el potencial humano. Los bienes
financieros son las acciones, bonos y los saldos de las cuentas en los bancos, entre otros.

Áreas funcionales de la empresa


Generalmente una empresa consta de, al menos, seis áreas funcionales básicas de
trabajo, las cuales se observan en la figura 12.

Área de Dirección General


de la Empresa

Área de Producción
Área de Administración
ÁREAS
Funcionales de la
Empresa
Área de Mercadeo y Venta
Área de Investigación,
Desarrollo e Innovación
(I+D+I)
Área Contable y
Financiera

Figura 12. Áreas funcionales de la empresa.

82
Sánchez Marchán, Nanci. (2014).Introducción a la Ingeniería Industrial, Material Instruccional de Apoyo. Caracas: UNA

Área de dirección general de la empresa:


Está representada por una o varias personas que se encargan de las tareas propias de
la dirección, es decir, de la planificación, la organización, la coordinación y el control. Estas
personas disponen de todos los recursos, tanto humanos como técnicos y financieros que la
empresa tenga, con el fin de lograr los objetivos trazados. En las pequeñas empresas es el
propietario quien representa la función de dirección.

El éxito de la dirección, se fundamenta en una comunicación constante, respetuosa y


honesta entre trabajadores y la gerencia. Un trabajador que se identifica y se siente orgulloso
de trabajar en un lugar, transmitirá ese orgullo hacia el trato a los clientes.

Área de producción:
Área o departamento donde se ejecutan funciones de abastecimiento y transforma-
ción de insumos o recursos (energía, materia prima, mano de obra, capital, información) en
productos finales (bienes o servicios). No sólo las empresas manufactureras cuentan con
el área de producción, sino también muchas empresas de servicios, por lo que hoy día, se
suele utilizar más el término operaciones antes que el de producción, ya que el término pro-
ducción parece sólo implicar bienes tangibles y no a los bienes intangibles o servicios. Véase
tema de producción.

Área de administración:
Esta área toma en cuenta todo lo relacionado con el funcionamiento de la empresa.
Desde la contratación del personal hasta la compra de insumos, el pago del personal, la
firma de los cheques, verificar que el personal cumpla con su horario, la limpieza del local, el
pago a los proveedores, el control de los inventarios de insumos y de producción. La gestión
del negocio también es parte de esta área. Véase tema de administración de recursos.

Área contable y financiera:


Toda empresa debe llevar un sistema contable en el que se detallen los ingresos y egre-
sos monetarios en el tiempo. Además, se debe declarar y cancelar periódicamente, ante el
ente tributario, los impuestos según los resultados de los libros contables que la empresa
lleva. La emisión de facturas, las proyecciones de ingresos por ventas y los costos asocia-
dos con el desarrollo del negocio, son tomados en cuenta en esta área. La empresa puede
escoger ser una empresa individual o una sociedad, por lo que el mismo propietario puede
llevar los registros contables o bien contratar a una persona dedicada a esta disciplina: un
contador. Véase tema de economía y finanzas.

Área de mercadeo y ventas:


Se realiza la función comercial o de distribución. En esta área se detallan las funciones,
capacidades y cualidades de quien será el responsable, además del personal involucrado
en la estrategia de mercadeo del negocio, es decir, la publicidad, el diseño del empaque y la
marca del producto o servicio, la distribución del mismo y el punto de venta, la promoción y
la labor de ventas. Véase tema de marketing industrial.

83
Sánchez Marchán, Nanci. (2014).Introducción a la Ingeniería Industrial, Material Instruccional de Apoyo. Caracas: UNA

Área de Investigación, Desarrollo e Innovación (I+D+i)


En la actualidad el desarrollo tecnológico se manifiesta como una función importante
para el futuro de las empresas, con el propósito de mejorar los procesos productivos. Esta
función además incluye cualquier innovación que se realice en el resto de las áreas, para lo-
grar el desarrollo de la empresa. Véase tema de producción (diseño de productos y procesos).

Actividad propuesta
Enumere las funciones que le correspondería asumir, si manejara las áreas de
producción o administración en una empresa.

COMPARACIÓN DE FUNCIONES DE UNA EMPRESA DE TRANSFORMACIÓN


(BIENES O SERVICIOS) Y UNA COMERCIAL

Esta comparación se puede apreciar en la figura 13.

EMPRESA DE TRANSFORMACIÓN (BIENES SERVICIOS)


Adquiere materias primas para transformarlas y obtener un producto de acuerdo al
diseño establecido, que satisfaga las necesidades de los consumidores

Función compra: Función producción: Función distribución:


Adquisición de mercancias Conservación y Despacho y entrega de
a proveedores, las cuales transformación de materias productos empacados
generan costos inherentes primas, mediante esfuerzo a clientes, utilizando
al traslado como: fletes, humano y el conjunto publicidad y promoción.
gastos aduanales, de diversas erogaciones Generando gastos de
impuestos de importación, fabriles venta, administración y
seguros, entre otros. financieros.

EMPRESA COMERCIAL
Tiene como principal función económica actuar como
intermediaria, comprando artículos elaborados para
posteriormente revenderlos.

Figura 13. Comparación de funciones de una empresa de transformación y una comercial,


adaptación del texto contabilidad de costos de García Colín.

84
Sánchez Marchán, Nanci. (2014).Introducción a la Ingeniería Industrial, Material Instruccional de Apoyo. Caracas: UNA

En una empresa comercial no se desarrolla la función de producción, se ocupa sólo de


la venta y distribución de productos, como se observa en la figura anterior. En cambio si la
empresa produce un bien como: carros, queso, entre otros. O servicios como por ejemplo:
comida rápida, servicio de electricidad, reparación de vehículo, entre otros, se realiza una
transformación. En el ejemplo de comida rápida se puede observar la función de producción
cuando se preparan los alimentos (ver ejemplo ilustrativo en el tema de producción).

Al igual que en la empresa de transformación, en la empresa de servicios se generan


gastos de mano de obra, pago de salarios, pagos de alquiler, pagos por la publicidad, im-
puestos, entre otros.

ORGANIZACIÓN DE LA EMPRESA
En una empresa las operaciones para la obtención del producto son numerosas y se
necesitan personas especializadas para las diferentes tareas que deberán ser coordinadas;
por ello, se puede afirmar que la división del trabajo es la razón misma de la organización.
Organizar consiste en coordinar los diferentes factores o funciones de la empresa a través
de una estructura organizativa, en la que se establezcan las diferentes áreas o niveles je-
rárquicos; para ello se organizan las funciones a realizar, se establecen las relaciones entre
los diversos elementos y se forman los flujos de autoridad, así como los distintos niveles de
decisiones. Se determinan las funciones o roles que debe desempeñar cada persona inte-
grante de la organización, así como las relaciones de todo tipo que se establecen entre ellas.

Organizar supone crear una estructura, una unidad formada por partes o compo-
nentes, distintos unos de otros. Esa unidad es la empresa, las partes o divisiones
son las áreas diferentes de la empresa.

Toda empresa tiene organizados sus recursos de una determinada manera; el problema
consiste en saber si esa organización es la más eficiente, es decir, la que conduce a lograr
los objetivos propuestos con el mínimo costo posible, ya que una organización adecuada en
la empresa es capaz de multiplicar varias veces la capacidad productiva. En consecuencia
la organización, dentro de la empresa, ha de concretar las siguientes tareas:
•• Dividir el conjunto de actividades que debe ejecutar una empresa en
grupos de actividades homogéneas; es decir, definir los departamentos,
secciones, áreas, talleres que permiten estructurar la misma.
•• Asignar un administrador o responsable a cada uno de los grupos de
actividades establecidos, dotándole de la autoridad necesaria para su-
pervisar el trabajo de cada componente de su grupo.
•• Coordinar, tanto en sentido horizontal como vertical, toda la estructura
de la empresa.

85
Sánchez Marchán, Nanci. (2014).Introducción a la Ingeniería Industrial, Material Instruccional de Apoyo. Caracas: UNA

Tipos de organización atendiendo a su estructura


Se muestra una figura donde se pueden visualizar los tipos de organización (formal,
informal) atendiendo a su estructura:

• Estructura que planifica de forma deliberada para


alcanzar un objetivo especifico.
• Se caracteriza por tener una estructura bien defini-
da y contar con políticas y reglamentos de acción
FORMAL claros.
• La autoridad de cada miembro está delimitada.
• Es flexible, abierta a los cambios.
• Practica el principio de la eficiencia.

ORGANIZACIÓN
• Se refiere a las relaciones sociales y se desarrolla
espontáneamente entre los individuos libres y cuyas
actividades no siguen reglamentos ni estructuras es-
pecíficas.
• Las personas buscan ser aceptadas, sentirse solida-
INFORMAL rias con otras, ser escuchadas, buscan protección.
• Se producen reacciones individuales o colectivas de
los individuos, ante la organización formal.
• Conviene ser tenida en cuenta por los organizadores.
• Puede generar grupos de presión

Figura 14. Tipos de organización atendiendo a su estructura.

Las organizaciones deben ser flexibles en el sentido de tener capacidad para adaptarse
a las situaciones que pueden producirse en su entorno, de modo que sea posible minimizar
el impacto en éstas.

Aspectos generales a considerar en la organización de una empresa


Los aspectos generales a considerar en la organización de una empresa son:

1. Estructura y niveles de la organización.


La organización se divide en niveles, los cuales van a depender de la dimensión de la
empresa y el perímetro de supervisión de subordinados que el jefe pueda controlar. Si ésta
es pequeña y tiene pocos empleados podrán ser dirigidos por un solo jefe. Si la empresa va
creciendo y teniendo más trabajadores se tendrán que ir constituyendo mandos intermedios,
que irán aumentando conforme se incrementa el número de subordinados, como se muestra
en la figura 15:

86
Sánchez Marchán, Nanci. (2014).Introducción a la Ingeniería Industrial, Material Instruccional de Apoyo. Caracas: UNA

Consejo de
Administración

Alta dirección

Nivel intermedio

Nivel de gestión
(Empleados)

Figura 15. Estructura y niveles de la organización.

Según se va subiendo se amplían las tareas directivas y se reducen las tareas ejecuti-
vas. Cuando la tarea a ejecutar se realiza por una sola persona no se presentan problemas,
pero si se exige la participación de varias personas es necesario organizarlas.

Los grupos de personas incluidas en esta figura pueden estar entrelazadas entre sí de
distintas formas, lo que da lugar a diferentes estructuras.

2. Instrumentos metodológicos de la organización:


Los instrumentos metodológicos más importantes de la organización son los organigra-
mas y manuales.

Los organigramas: son representaciones gráficas de la estructura formal de una orga-


nización, que muestran las interrelaciones, las funciones, los niveles, las jerarquías, las obli-
gaciones y la autoridad existentes dentro de ella. Los organigramas describen gráficamente
la división del trabajo que se establecerá y la relación a lograr entre las distintas áreas de
actividades, a través de líneas que representan los canales de supervisión coordinación y
comunicación que pueden existir en la empresa.

87
Sánchez Marchán, Nanci. (2014).Introducción a la Ingeniería Industrial, Material Instruccional de Apoyo. Caracas: UNA

Ventajas de los organigramas:

1. Muestran la posición de cada departamento en la estructura interna de la orga-


nización.

2. Señalan la interrelación o enlace que debe existir entre cada departamento y


sección de la organización.

3. Facilitan los procedimientos con los que trabaja la organización, dirección y con-
trol.

4. Dejan claramente definidas las líneas de mando y de responsabilidad de la orga-


nización.

5. Muestran el organismo asesor y aquellos otros con los que se establecen las
coordinaciones dentro de la organización.

Tabla 8. Simbología utilizada para la construcción de los organigramas.

Nombre Simbología Significado


Departamento o división de
la organización que varían
Rectángulo
en importancia de acuerdo a
su tamaño

Línea de mando de una divi-


Línea continua vertical
sión superior a una inferior.

Línea continua Indica la unión entre órganos


horizontal de asesoría.

Significa coordinación, seña-


Línea discontinua la las relaciones de unidades
que no tienen claramente
horizontal definida su ubicación admi-
nistrativa.

Clasificación de los organigramas:


En atención a su disposición geométrica, los organigramas se adecuan a la naturaleza
del sistema organizativo que representan. En la tabla 9 se pueden observar los tipos de or-
ganigrama más usados, una breve descripción y la representación gráfica de éstos:

88
Sánchez Marchán, Nanci. (2014).Introducción a la Ingeniería Industrial, Material Instruccional de Apoyo. Caracas: UNA

Tabla 9. Tipos de organigrama, descripción y representación gráfica

Tipo Descripción Representación Gráfica


Organigra- Está compuesto por rectángulos, que represen-
ma Vertical tan los cargos u órganos unidos entre sí por lí-
neas, que trazan las relaciones de comunicación
entre ellos. Cuando las líneas son horizontales,
representan relaciones de comunicación entre
iguales. Cuando son verticales, representan re-
laciones de autoridad (del superior sobre el su-
bordinado), lo que no está unido por una línea,
no tiene relación entre sí. Es uno de los más usa-
dos.
Organigra- Contiene los mismos elementos del organigra-
ma Horizon- ma vertical, sólo que el nivel máximo jerárquico
tal comienza a la izquierda, agregándose sucesiva-
mente los niveles subordinados hacia la dere-
cha.

Organigra- Formados por un círculo central, que corres-


ma Circular ponde a la autoridad máxima de la empresa, a
cuyo derredor se trazan círculos o rectángulos
los cuales constituyen los diversos niveles de la
organización. Las líneas representan los canales
de comunicación existentes entre los órganos o
cargos. A medida que se aproximan al centro, los
niveles jerárquicos son gradualmente más eleva-
dos.
Organigra- Es también llamado organigrama sectorial y se
ma radial elabora mediante círculos concéntricos, cada
uno de los cuales representa un nivel jerárquico.
La autoridad máxima se localiza en el centro del
organigrama, disminuyendo el nivel jerárquico a
medida que se aproxime a la periferia.
Es de forma compacta su estructura organizacio-
nal. Sin embargo, presenta ciertas limitaciones
de orden técnico, pues no permite la representa-
ción de organismos auxiliares muy variados.
Organigra- En este tipo de organigrama se puede combinar
ma Mixto algunos de los organigramas anteriores (Verti-
cal, horizontal,…) en uno solo. Algunas organi-
zaciones utilizan este tipo de organigramas de-
bido al alto volumen y complejidad de puestos
que tienen bajo su administración, buscando la
optimización del espacio de trabajo.

Organigra- Representa la estructura organizacional median-


ma Escalar te barras horizontales de longitud variable, según
el nivel jerárquico. Cada órgano está representa-
do por una barra. Todas las barras comienzan en
la misma línea a la izquierda (o a la derecha) y se
prolongan en la dirección requerida, según sea
el caso. Cuanto más elevado sea el nivel jerár-
quico, mayor será la longitud de la barra.

89
Sánchez Marchán, Nanci. (2014).Introducción a la Ingeniería Industrial, Material Instruccional de Apoyo. Caracas: UNA

La principal utilidad del organigrama es apreciar de manera fácil, como se estructura


una organización, sin necesidad de explicaciones detalladas sobre la forma en como se ha
dividido; por ello deben ser revisados y actualizados con frecuencia. El organigrama más
común es el vertical, sin embargo, el que se utiliza para la mejora organizacional es el mix-
to, puesto que refleja un mecanismo independiente del sector empresarial o tamaño de la
empresa tratante.

Ejemplo Ilustrativo
Caso de selección del tipo de organigrama de acuerdo a las características
de la organización.
La organización YORJEKA S.A, es una empresa manufacturera grande, dada
esta característica maneja un alto volumen y complejidad de puestos bajo su ad-
ministración. Por ello debe buscar la optimización del espacio de trabajo. ¿Cuál es
el tipo de organigrama que le conviene a la empresa para generar su estructura
organizacional?

Dadas las circunstancias antes planteada a la empresa YORJEKA S.A. le conviene


elegir un organigrama de tipo mixto, el cual es una combinación de dos tipos diferentes de
organigrama, como se muestra a continuación:

Con la utilización de este organigrama la empresa lograría la optimización del espacio


de trabajo.

Actividad propuesta
Describa qué representan las líneas horizontales y verticales en un organigrama.

Los organigramas constituyen la base que fundamentan las representaciones gráficas


de la estructura formal de una organización.

90
Sánchez Marchán, Nanci. (2014).Introducción a la Ingeniería Industrial, Material Instruccional de Apoyo. Caracas: UNA

3. Estructura organizativa:
Elementos de la estructura organizativa:

•• Autoridad.
•• Especialización.
•• Departamentalización.
•• Cadena de mando.
•• Tramo de control.
•• Centralización / Descentralización.
•• Formalización.

Autoridad: derechos y deberes inherentes a una posición gerencial para dirigir el tra-
bajo de un subordinado.

Especialización: grado en la que las tareas en la organización se subdividen en pues-


tos separados, es decir, cada trabajador se especializa en realizar parte de una tarea com-
pleta en lugar de toda.

Departamentalización: una vez que los trabajos se dividen por medio de la especia-
lización del trabajo, deben agruparse de nuevo para que las tareas comunes se puedan
coordinar.

Cadena de mando: la cadena de mando es la línea continua de autoridad que se ex-


tiende de los niveles organizacionales más altos a los más bajos y define quién informa a
quién. Ayuda a los empleados a responder preguntas como ¿A quién recurro si tengo un
problema? y ¿Ante quién soy el responsable?

Tramo de control: se refiere a cuántos subordinados puede supervisar un gerente de


manera eficaz y eficiente.

Centralización y descentralización: la centralización detalla el grado en el que la


toma de decisiones se concentra en un solo punto de la organización, es decir:

•• Si los gerentes de alto nivel toman las decisiones clave de la organiza-


ción con una participación escasa o nula de niveles inferiores, entonces
esta organización está centralizada.
•• En contraste, cuanto más información proporcionen o las decisiones
sean tomadas por los empleados de niveles inferiores, habrá más des-
centralización.

91
Sánchez Marchán, Nanci. (2014).Introducción a la Ingeniería Industrial, Material Instruccional de Apoyo. Caracas: UNA

Formalización: es el grado mediante el cual una organización depende de reglas y


procedimientos para dirigir el comportamiento de los empleados. Este grado se manifiesta,
entre otras cosas, en la elaboración por escrito de todas las actividades que se desarrollan
en la dinámica empresarial.

Tipos de diseño de una estructura organizativa


Cada empresa tiene su propia estructura organizativa en función de sus objetivos, de
su tamaño, de sus productos, además de la circunstancia que atraviesa. A continuación se
muestra una tabla, que contiene los principales tipos de estructura, una definición breve de
cada tipo, sus ventajas y desventajas, así como, un ejemplo de cada uno de ellos:

92
Tabla 10. Tipos de diseño de estructura; definición, ventajas, desventajas y ejemplo de cada uno de ellos.

Tipo de diseño
Definición Ventajas Desventajas Ejemplo
de estructura
Se apoya al máximo • Simplicidad y claridad • La concentración de pode-
en el principio de je- para su aplicación. res requiere la especializa-
rarquía y de subordi- ción en numerosas tareas Director General
nación absoluta a su • Unidad de mando, cada y la realidad es que no
inmediato superior. subordinado responde se puede ser experto en
Es decir, existe una ante un único jefe. No hay todas ellas.
unidad de mando y interferencia de poderes.
cada persona recibe • Cuando la empresa crece
• La comunicación de y la cadena de órdenes Personal Producción Finanzas
Estructura órdenes de un solo información (ascendente) también, se incrementa la
jerárquica o jefe. tanto como la comunica- burocracia.
lineal de Fayol
ción de órdenes (descen-
(militar)
dente) es directa • Es rígida e inflexible y pue-
de dar lugar a un régimen
• Permite a los mandos dictatorial. Modelado Pintura Horno Acabado
inferiores tomar deci-
siones en ausencia de
superiores.

93
• La disciplina se mantiene
fácilmente.
Los subordina- • Se basa en la idea de F. • Pérdida de unidad de
dos, de los niveles W. Taylor según la cual mando.
inferiores, en vez de la mayor productividad
estar conectados a de un obrero se alcanza • Se reciben órdenes de Gerencia
la dirección a través rompiendo la unidad de varios jefes que pueden
de un único punto (el mando. ser contradictorias, lo cual
superior inmediato) conlleva a una posible
recibe las órdenes, • Se proporciona autoridad confusión en la ejecución
Estructura a los especialistas en las de tareas.
funcional instrucciones y la Dpto. de Dpto. de Dpto. de Personal:
asistencia que nece- tareas que son de su es-
de Taylor pecialidad, haciendo más • Menor disciplina. Producción: Contabilidad: Dirección y Gestión
(horizontal) sita directamente de Preparado y Facturación: de RRHH:
varios jefes diferen- fluida la estructura. • La especialización puede
Mezclado: Cuentas por Pagar Empleo
tes, cada uno de los • El subordinado recibe hacer olvidar los objetivos
Labrado Cuentas por Cobrar Admón. de Personal
cuales desarrolla órdenes, asistencia e globales de la empresa.
Horneado
una función parti- instrucciones de varios • Retrasos en la acción
cular, en la que es jefes, cada uno de los debido al fraccionamiento
especialista. cuales desarrolla una de control y responsabili-
función particular. dades.
Sánchez Marchán, Nanci. (2014).Introducción a la Ingeniería Industrial, Material Instruccional de Apoyo. Caracas: UNA
Continuación Tabla 10

Tipo de Definición Ventajas Desventajas Ejemplo


diseño de
estructura
Se basa en la • Se basa en la idea • Se puede debilitar Staff personal, para uso exclusivo del director general, sin
distinción entre de Hery Fayol quien a la autoridad de que nadie más de la empresa pueda hacer uso de él.
jefes con autoridad sugirió la incorpo- línea.
(tipo subordinado) ración de “estados
mayores” compues- • El staff sólo reco- Director General
y técnicos especia- mienda, no tiene
tos de asesores Staff
listas que deben especialistas, pre- autoridad.
ser oídos antes de servando la unidad • Se pueden crear
tomar una decisión de mando. problemas admi- Jefe de Jefe Jefe de
(tipo funcional). El nistrativos o sobre Comercial
• No se proporcio- Administración Distribución
jefe con autoridad quien ejerce el
na autoridad a los
recibe los informes especialistas para liderazgo.
de los técnicos dar órdenes a nadie
Estructura (llamados staff y Staff especializado: Está a disposición de todos los cargos
de línea. de la empresa. Sirve a un gran número de departamentos

94
mixta o bajo su responsa-
jerárquica- bilidad toman la • Subordinado recibe y funciones diferentes (personal, contabilidad, técnico-
funcional decisión pasando órdenes, asistencia informático, marketing, jurídico, fiscal, calidad, etc).
e instrucciones de
al encargado la un solo jefe, que
orden de ejecutar- toma decisiones tras Director General
las. La función del escuchar las opinio-
staff consiste en Staff
nes de los asesores
informar, aconsejar, especialistas.
asesorar y apoyar Staff Staff Staff Staff Staff
• El staff no critica,
técnicamente a las sino que sugiere
unidades de mando. Director de Director Director de Director de Director de
soluciones y brinda
No está autorizado Producción Comercial Administración Personal Marketing
apoyo para resolver
ni para dar órdenes los problemas.
ni para tomar deci-
siones.
Sánchez Marchán, Nanci. (2014).Introducción a la Ingeniería Industrial, Material Instruccional de Apoyo. Caracas: UNA
Sánchez Marchán, Nanci. (2014).Introducción a la Ingeniería Industrial, Material Instruccional de Apoyo. Caracas: UNA

Actividad propuesta
Elabore una tabla cuya distribución sea similar a la tabla N° 10. Considere para
ello las estructuras del tipo matricial, por comité, departamentalización, entre
otras.

Para seleccionar una estructura adecuada es necesario comprender que cada empre-
sa es diferente y puede adoptar la estructura organizacional que más se acomode a sus
prioridades y necesidades, es decir, la estructura se crea según el rubro o industria, deberá
acoplarse y responder a la planeación; además debe reflejar la situación de la organización,
por ejemplo: años de fundada, tamaño, tipo de sistema de producción, el grado en el cual su
entorno es complejo y dinámico, entre otros.

Ejemplo Ilustrativo
La empresa SANMAR S.A, requiere organizar el departamento de finanzas, para
ello se necesita elaborar la estructura organizacional de ese departamento ¿Qué
tipo de estructura seria la apropiada en este caso?
En este caso la estructura recomendada para organizar el departamento sería el
tipo funcional, como la siguiente:

Finanzas

Contabilidad Tesorería Presupuesto

Permite esta estructura agrupar a las personas y unidades sobre la base del tra-
bajo que realizan e incrementa las oportunidades para utilizar herramientas más
especializadas y personal mucho más calificado.

Las estructuras también pueden clasificarse de acuerdo a su presentación o disposición


gráfica, es decir, vertical u horizontal. (La estructura vertical se corresponde con la jerárqui-
ca y la estructura horizontal con la funcional). Ver tabla 10.

Las organizaciones con una estructura horizontal, se vuelven más rápidas para tomar
decisiones; pero también puede llegarse al extremo de que los administradores tengan de-
masiada responsabilidad, por no contar con los medios que les ayuden a resolver problemas
cotidianos del trabajo.

95
Sánchez Marchán, Nanci. (2014).Introducción a la Ingeniería Industrial, Material Instruccional de Apoyo. Caracas: UNA

Actividad propuesta
Sobre la base de la disposición gráfica que posee la siguiente figura, determine
su orientación. Razone su respuesta.

Tendencias actuales de diseños de estructuras


Según los nuevos paradigmas de la estructuración de una organización, no se deben
construir departamentos separados, sino que éstos deben configurarse como un todo o ser-
vir de apoyo a los procesos productivos básicos de la empresa. La gestión en la empresa se
realiza de abajo hacia arriba, una concepción que discrepa del modelo taylorista, en el cual
la gestión está orientada de arriba hacia abajo.

Entre las nuevas tendencias de estructuras organizacionales se pueden citar las


siguientes:

•• Equipos. El uso de equipos como base del diseño para coordinar y eje-
cutar las actividades de trabajo.
•• Virtual. Organización central, pequeña, que contrata externamente sus
principales funciones de negocios.
•• Organización sin fronteras. Busca eliminar la cadena de mando, tiene
tramos de control ilimitados y reemplaza los departamentos con equipos
facultados (que tiene poder sobre la base de la delegación para la toma
de decisiones, capacitación, entre otros)

96
Sánchez Marchán, Nanci. (2014).Introducción a la Ingeniería Industrial, Material Instruccional de Apoyo. Caracas: UNA

Actividad propuesta
¿Qué tipo de diseño de estructura tienen?
•• Amazon.
•• Yahoo.
•• Google.
•• En general los nuevos líderes empresariales de Internet.

4. Manuales administrativos:
El manual administrativo es un documento que incluye en forma sistematizada las ac-
tividades a ser cumplidas por los miembros de la organización y la forma en como deben
realizarse. El propósito fundamental de los manuales es el de la sistematización de funciones
y operaciones que realiza la organización, así como instruir al personal, objetivos, políticas,
funciones, autoridad, normas, procedimientos u otros aspectos de la institución.

Importancia y necesidad de los manuales administrativos:

La importancia del manual administrativo es que suministra información para la acción,


al servir de guía para la ejecución del trabajo. A su vez, el manual tiene la relevancia de faci-
litar el adiestramiento, la supervisión y el mejor desempeño de los ejecutivos.

Clasificación de los manuales administrativos:

Los manuales administrativos se suelen clasificar en tres categorías:


1) Manuales de organización.
2) Manuales de políticas.
3) Manuales de sistemas y procedimientos.

Manuales de organización: son instrumentos que contiene información de la empresa


u organización tales como:
1) Nivel estratégico corporativo de la empresa (misión, visión, objetivos corporati-
vos).
2) La estructura de la organización y la base sobre la cual se diseñó esa estructura.
3) Las relaciones entre la función de línea y la de asesoría.

4) Las responsabilidades específicas del nivel operativo.

97
Sánchez Marchán, Nanci. (2014).Introducción a la Ingeniería Industrial, Material Instruccional de Apoyo. Caracas: UNA

Manuales de políticas: incluyen las políticas de la empresa, que son la guía básica
para la acción; así mismo prescriben los límites generales dentro de los cuales deben rea-
lizarse las actividades. Una correcta formulación de las políticas en un manual, permitirá:

1) Agilizar el proceso decisorio.

2) Establecer líneas o guías al personal directivo para que pueda obrar y adecuar
las actividades a los objetivos de la institución.

3) Facilitar la descentralización.

4) Servir de base para una constante y efectiva revisión.

Manuales de sistemas y procedimientos: son llamados, también manuales de ope-


ración; incluyen los trámites y métodos de trabajo. Son los manuales más comunes en las
organizaciones. A través de ellos se orienta y sistematiza la actividad operativa de la empre-
sa. Los manuales de procedimientos incluyen la descripción de normas, de procedimientos,
los flujogramas de los procesos, los formularios de registro y sus respectivos instructivos.
Los manuales de sistemas y procedimientos, ofrecen una serie de ventajas entre las que
tenemos:

1) Constituyen una útil fuente de información, sobre las prácticas generales de la


organización. Para que la información sea útil se requiere que sea comunicada a
través de la línea de mando de la organización.

2) Constituyen una guía indispensable para el trabajo a ejecutar.

3) Son una herramienta útil para el adiestramiento de nuevos empleados y también


para todos aquellos funcionarios que perteneciendo a la organización son ascen-
didos de categoría.

4) Eliminan duplicaciones innecesarias de actividades.

5) Permiten la revisión constante y el mejoramiento de las políticas, procesos y pro-


cedimientos, de la organización, en la medida en que se consulten con la debida
frecuencia. Ayudan a producir un mejoramiento sostenido de las operaciones,
normas y procedimientos.

6) Permiten un ahorro de tiempo, al evitar las preguntas de rutina, ya que la respues-


ta rápida a las mismas, se obtienen a través de la lectura del manual.

7) Constituyen un freno a la improvisación, en sus distintas manifestaciones.

8) Permiten generar un legado histórico de la evolución administrativa de la empre-


sa.

9) Constituyen un elemento importante para la revisión y evaluación objetiva de las


prácticas institucionales.

98
Sánchez Marchán, Nanci. (2014).Introducción a la Ingeniería Industrial, Material Instruccional de Apoyo. Caracas: UNA

TOMA DE DECISIONES EN LA EMPRESA


La toma de decisiones es el proceso de seleccionar un curso de acción entre alternati-
vas; es la esencia de la planeación y el corazón del éxito. En una empresa se deben tomar
muchas decisiones todos los días. Algunas de ellas son decisiones de rutina, mientras que
otras tienen consecuencias drásticas en las operaciones que desarrolla la empresa.

Para la toma de decisiones se hace uso de la Ingeniería Económica, ya que a través de


sus técnicas matemáticas se optimiza este proceso, en beneficio de la empresa. También es
preciso considerar la factibilidad del proyecto, la cual estudia las condiciones para poder
resolver un problema, con la disponibilidad de recursos que se maneja (revisar temas Eco-
nomía y Finanzas, unidad 3; Toma de Decisiones, unidad 4)

EL FUNCIONAMIENTO ECONÓMICO DE LA EMPRESA

En el funcionamiento económico de una empresa se consideran los siguientes aspec-


tos:

1. Información patrimonial: indica cuál es el valor de la empresa, dicha información


se obtiene de los inventarios y los balances que presenta la empresa.

2. Información financiera: para saber cuánto posee la empresa y el grado de liqui-


dez del que se dispone, así como cuánto debe y a quién debe.

3. Información económica: para saber si gana o pierde dinero. Dicha información se


obtiene de la cuenta de resultados (diferencia entre ingresos y gastos).

La responsabilidad económica de la empresa viene definida por su razón de ser o de


realizar un negocio. Todo negocio, consiste básicamente en satisfacer necesidades y deseos
del cliente vendiéndole un producto o servicio por más dinero de lo que cuesta fabricarlo.
La ventaja que se obtiene con el precio, se utiliza para cubrir los costos y para obtener una
utilidad.

Conocer los costos de la empresa es un elemento clave de la correcta gestión empresa-


rial, para que el esfuerzo y la energía que se invierte en la empresa den los frutos esperados.
Por otra parte, no existen decisiones empresariales que de alguna forma no influyan en los
costos. Por eso es importante que las decisiones a tomarse tengan la suficiente calidad, para
garantizar el buen desenvolvimiento de la empresa. Seguidamente se presenta una figura
en la que se pueden observar los costos generados en una empresa. Ver tema economía y
finanzas, unidad 3.

99
Sánchez Marchán, Nanci. (2014).Introducción a la Ingeniería Industrial, Material Instruccional de Apoyo. Caracas: UNA

COSTOS
GENERADOS EN
UNA EMPRESA

Costos de Costos o Costos o Costos o


Producción Gastos de Gastos de Gastos de
Financiamiento Administración Distribución

Costos o Costos o
Gastos Gastos
Directos Indirectos

Figura 16. Costos generados en una empresa.

Actividad propuesta
Seleccione una empresa conocida y compare sus costos entre:
Diferentes departamentos de la empresa.
Con otras empresas.
En diferentes períodos.

CÓMO CREAR UNA EMPRESA


Se estudió previamente todo lo relacionado con el funcionamiento de la empresa, sobre
esta base se muestra a continuación un procedimiento sencillo de cómo crear una empresa:

Procedimiento para crear una empresa:

Paso 1 Para crear una empresa se debe partir de una idea (innovación de un
producto, servicio a realizar) es decir, descubrir sectores de la eco-
nomía que no estén suficientemente atendidos, y crear una empresa
para satisfacer la demanda. Se puede inventar algo, modificar algo ya
existente, copiar ideas que funcionan con éxito fuera de la región o en
otro país, o simplemente ofrecer un servicio que no se está prestando
adecuadamente.
Investigación Tomada la decisión del tipo de empresa a crear, se deberá, antes de
preliminar abrir las puertas del negocio, tramitar una serie de documentos refe-
rentes a las licencias necesarias para poner la empresa en marcha.

100
Sánchez Marchán, Nanci. (2014).Introducción a la Ingeniería Industrial, Material Instruccional de Apoyo. Caracas: UNA

Paso 3 Además de servir para comprobar si la idea puede ser puesta en prác-
tica, será útil a la hora de conseguir financiación.
Es un documento que especifica cuándo se pretende iniciar o que ya
se ha iniciado un negocio. En él se expone el propósito general de
una empresa y los estudios de mercado, técnico, financiero y de or-
ganización, incluyendo temas como: los canales de comercialización,
el precio, la distribución, el modelo de negocio, la ingeniería, la locali-
Crear un plan zación, el organigrama de la organización, la estructura de capital, la
de negocio evaluación financiera, las fuentes de financiamiento, el personal nece-
sario junto con su método de selección, la filosofía de la empresa, los
aspectos legales, y su plan de salida.

Pasos para elaborar el plan de negocio:

Paso 3.1 El estudio de mercado consiste en una iniciativa empresarial con el


fin de hacerse una idea sobre la viabilidad comercial de una actividad
económica.
A través de este estudio se puede verificar si hay amenazas o fortale-
zas en el negocio, observando a la competencia y su ubicación, si el
mercado de ese producto/servicio crece o decrece y cuánto estarían
dispuestos a pagar los posibles clientes. Véase tema de marketing
Estudio de
industrial.
mercado

Paso 3.2 El marketing es un conjunto de principios, metodologías y técnicas a


través de las cuales se busca conquistar un mercado, colaborar en la
obtención de los objetivos de la organización, y satisfacer las necesi-
dades y deseos de los consumidores o clientes.
Se puede determinar con este plan:
•• Qué gama de productos/servicios se van a ofrecer.
Plan de •• Qué precio se va a fijar (y su evolución futura), en fun-
marketing ción de la estructura de costos, la previsión de demanda
y el nivel de competencia.
•• Cómo es posible comunicarse con los clientes, cuál será
la publicidad y en qué formatos.
•• Cuáles van a ser los canales de distribución. Véase tema
de marketing industrial.

102
Sánchez Marchán, Nanci. (2014).Introducción a la Ingeniería Industrial, Material Instruccional de Apoyo. Caracas: UNA

Paso 3.3 Una vez realizadas las actividades de publicidad y mercadeo y el pro-
ceso de comunicación se haya logrado, se puede empezar a plani-
ficar la cantidad de productos a distribuir a través de los diferentes
canales.
Se ha de determinar qué y cuánto se necesita para poder fabricar
el producto, en qué cantidad, qué inversiones se deberá asumir, es
decir, detallar todos los aspectos técnicos y organizativos que toman
parte en la fabricación de los productos o en la prestación del servi-
cio, considerando para ello los siguientes aspectos:
Plan de
producción •• Proceso de fabricación.
•• Localización geográfica de las instalaciones.
•• Edificios y terrenos necesarios.
•• Equipos necesarios para la fabricación de los productos
o la venta de los servicios.
•• Estrategia del proceso productivo.
•• Descripción de los procesos de control de calidad, con-
trol de inventarios y demás procedimientos de inspec-
ción que garanticen la optimización de los recursos y la
satisfacción del cliente.

Paso 3.4 Cuántas personas se necesitan, distribuidas por departamentos y


funciones, con una formación adecuada, así como su remuneración
y tipo de contrato.

Recursos
humanos

Por último se elabora un plan financiero, que se encuentra también dentro


del plan de negocio.
Paso 3.5 Se traduce todo lo anterior a bolívares, y al estimar los ingresos y
gastos (Balance provisional de ganancias y pérdidas) más una es-
tructura de activo y pasivo también un balance provisional del mismo
y el presupuesto de tesorería, se podrá comprobar si se van ajus-
tando a las cifras reales.

Plan financiero

103
Sánchez Marchán, Nanci. (2014).Introducción a la Ingeniería Industrial, Material Instruccional de Apoyo. Caracas: UNA

La empresa es el instrumento universalmente empleado para producir y poner en ma-


nos del público la mayor parte de los bienes y servicios existentes en la economía. Para
crear una empresa es necesario visualizar ésta, como un conjunto de recursos (insumos en
su proceso de producción), capacidades (habilidades y conocimientos de sus empleados) y
aptitudes centrales (distinguen a una empresa en el nivel competitivo y reflejan su imagen)
que pueden utilizarse para crear una ventaja con relación a otras empresas del mercado.
Lo anterior supone que cada empresa tiene recursos y capacidades que no poseen otras
empresas, al menos no en igual combinación.

Los insumos en el proceso de producción de una empresa pueden ser tangibles o in-
tangibles. Los recursos tangibles son los activos que se pueden ver y contar, la capacidad de
pedir dinero prestado, las condiciones de su planta, entre otros. Generalmente el valor de los
recursos tangibles se establece a través de los estados financieros. Los recursos intangibles
van desde el derecho de propiedad intelectual, las patentes, las marcas registradas o depen-
den de ciertas personas con conocimientos prácticos, entre otros. Se utilizan como base de
las capacidades y aptitudes centrales.

Crear es un acto infinitamente humano, que exalta la condición de seres pensantes,


creativos, lo que coloca al hombre por encima de otras especies. Crear en todos los órdenes
de la vida no solo resulta un acto de inigualable importancia para el hombre, sino que desde
siempre ha servido para marcar límites entre lo productivo y lo improductivo, entre lo auten-
tico y lo falso.

La creación de una empresa es una vía de desarrollo económico y social para cualquier
individuo o comunidad, por ello la importancia de discutir y reflexionar todo lo pertinente a
esta actividad humana, consagrada al servicio del hombre.

A continuación se proponen dos ejercicios alusivos a la creación de una empresa:

Actividad propuesta N° 1
Para considerar los distintos elementos relevantes en el proceso de creación de
una empresa, estudie los casos de diversas organizaciones atendiendo a su éxito
o fracaso, en función principalmente de su plan de marketing.

Actividad propuesta N° 2
Tomando en cuenta las ofertas de empleo que se pueden encontrar en las di-
ferentes empresas del país, piense en varias opciones con las que se pueda
comenzar un nuevo negocio con perspectiva de futuro. ¿Qué necesidad actual o
futura, sin satisfacer o que no esté cubierta en la actualidad, se puede detectar
para crear su propia empresa?
Indique al menos tres opciones.

104
Sánchez Marchán, Nanci. (2014).Introducción a la Ingeniería Industrial, Material Instruccional de Apoyo. Caracas: UNA

PRODUCCIÓN
•• Origen de la producción.
•• Evolución de la producción con el paso de la Revolución Industrial.
•• Definición de proceso, producto y producción.
•• Diseño de productos y proceso.
•• Definición de sistema.
•• Sistema de producción.
•• El sistema Justo A Tiempo (JAT).
•• Los sistemas MPR: MRP- I, MRP- II y el sistemas CAM.
•• Organización de un sistema de producción.
•• Parámetros de un sistema.
•• Planeación de la producción.
•• Pronósticos de la producción.
•• Inventario.
•• Control de la producción.

ORIGEN DE LA PRODUCCIÓN
En la historia de la humanidad la mayoría de los habitantes de la tierra vivió en la mise-
ria, primero como cazadores o recolectores y luego como campesinos o jornaleros.
En su afán por mejorar sus condiciones de vida, el hombre inició de manera rudimenta-
ria la actividad productiva de forma individual, lo cual lo hacía débil y solitario, aumentando
su imposibilidad de producir y luchar contra las inclemencias de la naturaleza. Razón que
los impulsó a agruparse, para lograr el trabajo colectivo y la propiedad sobre los medios de
producción.
A medida que se avanzaba en el tiempo el hombre fue mejorando sus herramientas
manuales de trabajo, es a finales del siglo XVIII y principios del siglo XIX, con la incorpora-
ción de las máquinas a la producción, que se sustituye el trabajo manual y los tradicionales
sistemas de fabricación por otros nuevos.
El trabajo se trasladó desde los talleres artesanales con un reducido número de ope-
rarios a las fábricas, en donde máquinas y obreros fueron agrupados en grandes concentra-
ciones. La división del trabajo impulsada por Adam Smith (ver tabla 2) derivó en un notable
incremento de la productividad, así como la disminución de los costos de fabricación, lo que
genero la baja de los precios y el crecimiento del número de consumidores; además las
industrias se fueron diversificando para responder a las nuevas necesidades y tendencias.
Con esta naciente industria se desarrolló el intercambio de los productos, es decir, el
comercio y se dio paso a la Revolución Industrial, con la que algunas sociedades cambiaron
su pobreza por un desarrollo sorprendente.
A mediados del siglo XIX nace en Filadelfia Frederick Taylor (ver tabla Nº2) quien se
dedicó a estudiar detenidamente la organización de las operaciones realizadas en las nue-
vas industrias de producción masiva, surgidas a raíz de la Revolución Industrial. De estos
estudios se derivan los principios de las modernas técnicas de producción.

105
Sánchez Marchán, Nanci. (2014).Introducción a la Ingeniería Industrial, Material Instruccional de Apoyo. Caracas: UNA

EVOLUCIÓN DE LA PRODUCCIÓN CON EL PASO DELA REVOLUCIÓN


INDUSTRIAL
Con la Revolución Industrial la producción experimentó varios cambios en cuanto a ma-
teriales utilizados, maquinaria, países, sociedades entre otros, que se muestran a través de
las etapas mencionadas, en la siguiente tabla:

Tabla 11. Etapas evolutivas de la producción sobre la base de la Revolución Industrial.


Etapas evolutivas de la producción con el paso de la Revolución Industrial
Primera etapa Segunda etapa Tercera etapa
(1760 – 1830) (1870 – 1914) (1945 - Hasta nuestros días)
Materia prima inorgánicas: carbón, Las mismas de la pri- Siguen siendo las mismas que
petróleo, madera y mera etapa, así como, en las anteriores etapas, pero
algodón. químicas, el plástico, se realizan investigaciones
tejidos para la industria para lograr que sean más
textil, minerales. ligeras ejemplo; fibra óptica,
fibra de vidrio, nuevas cerá-
micas, aluminio, acero, cobre,
mercurio, etc.
Fuente de El carbón, agua, mecá- Hidráulica electricidad Se siguen usando las mismas
energía nica producida por el y los derivados del energías tradicionales de la
hombre. petróleo. segunda etapa, pero se le
añade la energía natural o
alternativa, cómo es: la eólica,
la solar, la hidráulica, etc. ener-
gías naturales, inagotables y
limpias.
Modos de Aparece la propiedad Los protagonistas en Los protagonistas en esta
producción privada y la pública. Los esta etapa son la alta etapa son las personas que
dueños de las empre- burguesía y los terrate- tenían alguna especialización
sas, eran los responsa- nientes, la clase media y que renovaban los conoci-
bles de los medios de y la pequeña burguesía, mientos de su profesión.
producción. Se fueron que regentaban peque-
El dominio lo tienen las em-
formando empresas ños negocios, y la clase
presas multinacionales por
más grandes que se di- popular que mejoró la
invertir un mayor capital.
vidían en acciones, para situación de la etapa
poder repartir de igual anterior a esta.
forma los beneficios.
Las empresas son
propiedad de grandes
grupos de empresarios,
que se agruparon for-
mando trusts, cárteles y
holdings.
Desarrollo Máquina de vapor, in- Las industrias más La maquinaria es cada vez
industrial o ventada por James Watt importantes de la an- más precisa y requiere la más
innovación de (1785) terior etapa siguen a la alta tecnología, como la aero-
la época cabeza de esta segunda náutica, la óptica, o la utilizada
Lanzadera volante o el
fase, pero además apa- en el mundo científico, que
telar mecánico.
rece la industria química requiere de una gran precisión.
Desarrollo de la indus- que incrementó adelan-
tria siderúrgica y la textil. tos en la agricultura y
medicina.

106
Sánchez Marchán, Nanci. (2014).Introducción a la Ingeniería Industrial, Material Instruccional de Apoyo. Caracas: UNA

Primera etapa Segunda etapa Tercera etapa


(1760 – 1830) (1870 – 1914) (1945 - Hasta nuestros días)
Se inventa el dinamo:
transformaba la energía
mecánica en energía
eléctrica. El motor de
explosión: extraía la
energía del petróleo
responsable de la
invención del automóvil.
El cinematógrafo y el
teléfono: fueron inventos
que revolucionaron el
mundo de las telecomu-
nicaciones.
Mercado La industrialización Alemania, Estados Uni- Con la globalización el merca-
comenzó en Inglaterra dos y Japón se convier- do se vuelve mundial apoyado
quien se convirtió en ten en nuevas potencias por las relaciones económicas
una potencia. Luego se en el mercado, destro- internacionales. Los mercados
empezó a extender por nando a Inglaterra. más dominantes en el mundo
Europa Occidental, Es- (una economía fuerte, según
tados Unidos y Japón. su Producto Interno Bruto) son:
Estados Unidos, China, Japón,
países de la Unión Europea,
Brasil, Canadá, Rusia, entre
otros.
objetivo Se busca la mecani- Se busca la automati- Sobre la basa de la auto-
zación para producir zación para mejorar la matización se desarrolla la
mucho y más barato producción. robotización.

La actualidad está marcada por una serie de adelantos que señalan tres palabras clave
en la producción: función de producción, estrategia de producción y operaciones de
producción. La función de producción existe desde que se inició la actividad productiva,
pero no ocurre así con la estrategia de producción. No fue sino hasta 1969 cuando Wickham
Skinner que realiza el primer trabajo referido a la necesidad de conceder un carácter estra-
tégico a la función de producción, con el título Manufacturing Missing Link in Corporate Stra-
tegy. Sin embargo, es en los años 70 y principios de los 80 cuando surge como tal el nuevo
paradigma de la estrategia de producción, desarrollado por profesores e investigadores de la
Facultad de Administración de Empresas de Harvard.

La década de los 80 fue testigo de una revolución en las filosofías de dirección y de las
tecnologías aplicadas a las operaciones de producción. Chase y Aquilano (1995) refieren a
la producción just-in-time (JIT) como el mayor adelanto en la filosofía de fabricación, com-
parable en su impacto con la cadena de montaje de Henry Ford. Al JIT se unió el Control de
Calidad Total (TQC) y juntos, sobre una visión estratégica del área de fabricación, forman
las prácticas industriales de numerosas empresas de excelencia. También en esta fecha se
incorporan nuevas tecnologías en el manejo de las fábricas, para mejorar las operaciones
de producción.

107
Sánchez Marchán, Nanci. (2014).Introducción a la Ingeniería Industrial, Material Instruccional de Apoyo. Caracas: UNA

En la década de los 90, aumentó el dinamismo en los sectores industriales, destacándo-


se una alta competitividad, una globalización de las operaciones y el desarrollo de redes de
fábricas, acentuándose la importancia del enfoque estratégico de la producción en el nuevo
escenario de los sistemas logísticos y las cadenas de suministros. El término estrategia
también se identifica como el arte de crear y proyectar planes para alcanzar una meta con-
creta. Tiene su origen en un proceso de toma de decisiones, e involucra una gama amplia
de actividades que van desde la asignación de recursos hasta las operaciones cotidianas de
producción.

La experiencia en muchas de las empresas industriales contemporáneas, no parecen


competir aún sobre una plataforma de excelencia y liderazgo en su función de producción.
En tal sentido, resta un gran trabajo por hacer para alcanzar el nivel superior de desarrollo en
sus procesos industriales, razón por la cual el Ingeniero Industrial, se prepara cada día mejor
para conocer, desarrollar y mejorar la producción de bienes y servicios, logrando la mayor
eficacia y eficiencia en los diferentes procesos y actividades que se generan en esta labor.
Es necesario que esté al tanto de todo lo relacionado con la producción y la productividad,
de las mejoras que se pueden lograr en estos aspectos, utilizando estrategias efectivas,
herramientas que proporcionan la Ingeniería Industrial las cuales pueden dividirse en varias
categorías a saber: para la definición del problema, para la recopilación de datos, para el
análisis de éstos, para el desarrollo de las soluciones técnicas, para la investigación de las
operaciones, la gestión y para proyectos de mejora continua.

También debe hacer uso de tecnología de punta como Diseño Asistido por Computado-
ra (CAD), Manufactura Asistida por Computadora (CAM), máquinas herramientas automati-
zadas, robots reprogramables, telecomunicaciones, entre otros.

El TSU en Higiene y Seguridad también tiene una gran responsabilidad en el desarrollo


de los procesos industriales, la cual tiene que ver con lograr mantener al trabajador en con-
diciones seguras dentro de su sitio de trabajo, así como, el desarrollo ergonómico de estos
espacios, para lograr la máxima productividad y eficiencia de los procesos.

Actividad propuesta
Construya una línea de tiempo donde se muestre la evolución de la producción,
con el paso de la Revolución Industrial, destacando los avances tecnológicos
más importantes en cada etapa.

DEFINICIÓN DE PROCESO, PRODUCTO Y PRODUCCIÓN


Del latín procesus o procedere, proceso es:

El conjunto de fases sucesivas donde se configuran uno o más insumos mediante un


procedimiento, para generar un producto.

108
Sánchez Marchán, Nanci. (2014).Introducción a la Ingeniería Industrial, Material Instruccional de Apoyo. Caracas: UNA

Un proceso se puede dividir en subprocesos o partes bien definidas de un proceso.


Ejemplo, ventas es un subproceso de solicitud de crédito.

Procedimiento: forma en como se desarrollan las etapas de un proceso, es decir, la


serie de pasos en los que se apoya el proceso de un producto, en un determinado tiempo y
hacia un fin determinado.

Un producto se define como:

El resultado que se logra mediante un proceso, que permite cumplir con unas especifica-
ciones determinadas que satisfagan las expectativas de los consumidores.

En un proceso también se pueden generar subproductos, es decir, el producto que se


obtiene además del principal, por ejemplo el afrecho es un subproducto del trigo; el ácido
sulfúrico es subproducto del azufre que se elimina del petróleo.

Por otra parte, del latín productĭo, el término producción hace referencia a la acción de
producir, a la cosa producida, al modo de producirse o a la suma de los productos del
suelo o de la industria, de allí que producción se puede definir como:

El grupo de operaciones que se emplean para transformar insumos en bienes o servicios


terminados o semielaborados, con la finalidad de incrementar su utilidad o su valor.

En este orden de ideas se puede decir que para la industria, un proceso de fabrica-
ción o de producción industrial es:

El conjunto de operaciones que se encuentran interrelacionadas de forma dinámica y


son necesarias para modificar las características de insumos, es decir, los elementos de
entrada (conocidos como insumos) pasan a ser elementos de salida (productos), tras
un proceso en el que se incrementa su valor para el cliente.

El proceso de producción requiere la consideración de aspectos importantes como es


el diseño que se va a utilizar en el desarrollo de productos, así como, en su proceso de fa-
bricación. En este orden de ideas, antes de abordar todo lo relacionado con el proceso de
producción, se estudiará la fase de diseño de productos.

DISEÑO DE PRODUCTOS Y PROCESOS


Lo que sostiene a cualquier organización es el producto que ofrece a la sociedad. Las
compañías que cumplen las necesidades de los clientes con bienes o servicios atractivos,
útiles y de alta calidad, encuentran clientes; aquéllos que no lo hacen no sobreviven. Así una
decisión crítica para los gerentes de una empresa es la selección, definición y diseño de los
productos que produce.

109
Sánchez Marchán, Nanci. (2014).Introducción a la Ingeniería Industrial, Material Instruccional de Apoyo. Caracas: UNA

DISEÑO DE PRODUCTOS
El diseño de un producto es:

La estructuración de las partes componentes o actividades que dan a un elemento un


valor específico, es considerado como un paso previo a la producción, al igual que el
pronóstico y la demanda de su volumen.

El resultado de la decisión del diseño del producto se transmite a las operaciones en


forma de especificaciones, en las cuales se indican las características que se desea tenga
el producto a elaborar.

Estrategias en la introducción de nuevos productos

Cuando se quiere fabricar un producto nuevo en una empresa, es preciso considerar


una serie de estrategias como las siguientes:

Impulso de mercado

•• Se debe fabricar lo que se puede vender.


•• Se identifican necesidades en el mercado a cubrir.
•• La realización del Marketing es clave para posicionar el producto en el
mercado, así como la retroalimentación.

Interfuncionalidad

•• La innovación de productos es el resultado de un esfuerzo coordinado


por todos los departamentos de la empresa.

Impulso tecnológico

•• Se debe vender lo que se puede fabricar.


•• Las nuevas tecnologías presentan oportunidades de fabricación de nue-
vos productos para los que habrá que crear mercado, Investigación y
Desarrollo (I+D) es clave.

Diseño o desarrollo de nuevos productos

Simultáneamente con los clientes y la empresa, existen otra serie de personas u orga-
nizaciones que se ven afectadas por la creación de un nuevo producto y por las actividades
de su ciclo de vida. Por ello, el objetivo del proceso de diseño de un producto es satisfacer el
conjunto de necesidades de todas las personas u organizaciones involucradas, de la forma
más eficiente. Se presenta a continuación una figura donde se muestra las etapas involucra-
das en el desarrollo de un nuevo producto:

110
Sánchez Marchán, Nanci. (2014).Introducción a la Ingeniería Industrial, Material Instruccional de Apoyo. Caracas: UNA

Tecnología de Investigación
Consumidores Generación de idea y Desarrollo T(I+D)

Selección del producto

Diseño preliminar del


Diseño preliminar proceso

Construcción del prototipo

Pruebas

Diseño definitivo del


Diseño definitivo proceso

Producción final

Figura 17. Proceso de desarrollo de nuevos productos.

Generación de la idea
Definición de producto:
•• por la necesidad que cubre y
•• por el modo de cubrirla (la tecnología utilizada)

De ahí que la fuente de generación de nuevos productos sea doble. Ver figura 18.

Interna Externa
Dentro de la Empresa Sugerencias de
I+D Clientes

Figura 18. Generación de la idea en el diseño de un producto.

111
Sánchez Marchán, Nanci. (2014).Introducción a la Ingeniería Industrial, Material Instruccional de Apoyo. Caracas: UNA

La identificación de las necesidades del mercado comprende el desarrollo de nuevas


tecnologías y productos, para satisfacer estas necesidades. Por otro lado las ideas también
pueden surgir de la tecnología disponible o nueva.

Selección de la idea

Las ideas generadas en la primera fase pasan por un triple filtro. Ver figura 19.

¿Se puede fabricar? ¿Se puede vender?


Filtro técnico Filtro mercado

¿Es rentable fabricarlo y venderlo?


Filtro financiero

Figura 19. Selección de la idea para el diseño de un producto.

Las ideas para nuevos productos deben pasar por lo menos tres pruebas: el potencial
del mercado, factibilidad financiera, compatibilidad con operaciones. Antes de colocar la idea
de un nuevo producto en el diseño preliminar se le debe someter a los análisis necesarios
que se establecen en torno a estas tres pruebas.

Método de selección
1. Escoger características relevantes.
2. Establecer importancia de cada una de ellas y asignarles peso específico conse-
cuentemente.
3. Valorar estas características respecto a cada producto potencial en una escala
concreta.
4. Obtener las calificaciones medias ponderadas para cada producto.
A continuación es recomendable someter los productos a algún criterio financiero de
jerarquización, por ejemplo el Valor Actual Neto (VAN).

Diseño preliminar

En el diseño preliminar se toma en cuenta costo, calidad y rendimiento del producto. El


resultado debe ser un diseño de producto que resulte competitivo en el mercado y que pueda
producirse. Ver figura 20.

112
Sánchez Marchán, Nanci. (2014).Introducción a la Ingeniería Industrial, Material Instruccional de Apoyo. Caracas: UNA

Coste
Especificaciones
completas del
producto a lo largo
de tres ejes

Calidad

Rendimiento

Figura 20. Diseño preliminar de un producto.

Esta etapa del diseño de un producto se relaciona con el desarrollo del mejor diseño
para la idea del nuevo producto. Cuando se aprueba un diseño preliminar, se puede construir
un prototipo para someterlo a pruebas adicionales y análisis.

Diseño de prototipo y prueba

El prototipo debe parecerse lo más posible al producto final.

Las pruebas son en dos frentes:


•• Mercado: aceptación por clientes potenciales.
•• Funcionamiento: comprobar cada una de sus especificaciones técnicas.

Las pruebas en los prototipos buscan verificar el desempeño técnico y comercial. Una
manera de apreciarlo es construir suficientes prototipos como para apoyar una prueba de
mercado.

Diseño definitivo del producto


•• Se incorporan los cambios considerados oportunos en las fases anteriores.
•• Se desarrolla la tecnología de proceso o fabricación.
•• Se desarrollan controles de calidad.
•• Se diseñan procedimientos de pruebas de rendimiento, entre otros.

En este sentido el producto puede someterse a pruebas adicionales para asegurar el


desempeño de éste. La atención se coloca en la terminación de las especificaciones de di-
seño para que se pueda proceder con la producción.

113
Sánchez Marchán, Nanci. (2014).Introducción a la Ingeniería Industrial, Material Instruccional de Apoyo. Caracas: UNA

Ciclo de vida del producto


 

Crecimiento

Madurez
Ventas

Introducción

Declive
Tiempo

Figura 21. Ciclo clásico de la vida de un producto.

La vida de un producto se puede concebir como una sucesión de varias fases en las que
el producto tiene un comportamiento distinto. Los productos también se pueden desarrollar y
sufrir cambios utilizando nuevos materiales, tecnologías o procesos durante su ciclo de vida,
para disminuir sus costos y mejorar su producción; a este proceso se le denomina rediseño
de un producto. Los productos se someten constantemente, en el uso, a rediseños e inno-
vaciones. Algunos buenos ejemplos son los automóviles, teléfonos y artículos domésticos.
Ver figura 21 y 22.

Características relevantes de las etapas de la vida de un producto:


Introducción Crecimiento Madurez Declive
•• Las ventas son •• Perfeccionamiento •• Las ventas •• La demanda
bajas. del proceso. aumentan hasta disminuye y las
•• Existen pocos •• Desarrollo del llegar el momento ventas descienden.
competidores. producto. en que se •• Se producen
•• Los gastos en •• Existe una política detienen. recortes en
promoción y adecuada de •• Las líneas de las líneas de
distribución son precios. productos se productos.
altos. •• Las ventas suben alargan para atraer •• Los precios
•• Las utilidades son con rapidez. a segmentos se estabilizan
bajas. •• Se potencia la de mercado a niveles
•• El objetivo principal imagen de la adicionales relativamente
de la promoción es empresa. •• Existe una intensa bajos.
informar. •• Los clientes son competencia de •• La promoción se
•• Los clientes son impulsivos. precios. reduce al mínimo.
innovadores. •• Los clientes •• Los clientes que
actúan de manera compran son los
concienzuda. rezagados.
Etapa 1 Etapa 2 Etapa 3 Etapa 4

Figura 22. Características de las etapas de la vida de un producto.

114
Sánchez Marchán, Nanci. (2014).Introducción a la Ingeniería Industrial, Material Instruccional de Apoyo. Caracas: UNA

Se trata del clásico ciclo de nacimiento, desarrollo, madurez y muerte aplicado a las
ventas de un producto, es decir, cuando se lanza un producto al mercado, las ventas normal-
mente no se disparan el primer día. El mercado no conoce el producto y por tanto hay que
hacer un esfuerzo en darlo a conocer y captar los primeros clientes.

Conocer la fase del ciclo en la que se encuentra el producto permitirá diseñar la estrate-
gia más eficaz para alargar su vida en un mercado cada vez más cambiante y rápido.

Otras formas de diseño


Diseño modular:
Es un tipo de diseño estandarizado que se emplea para lograr diferentes versiones de
productos a partir de combinaciones de módulos transportables, desarmables y reorganiza-
bles; impulsando múltiples funcionalidades, lo que permite optimizar el tiempo de construc-
ción.

El diseño modular de puede observar en automóviles, refrigeradores, edificios, mue-


bles, entre otros.

Diseño Asistido por Computadora (CAD):

El diseño asistido por computadora se asocia con gráficas interactivas en computadoras


para crear dibujos y modelos de productos en dos o tres dimensiones, a través de programas
informáticos (softwares).

Se utilizan los softwares CAD para elaborar modelos virtuales y planos detallados de la
mayoría de los productos manufacturados, así como, en la distribución de planta, diseño de
máquinas y herramientas entre otros. Ver planificación de sistemas de producción.

Diseño de un producto considerando la calidad:

La calidad en atención al diseño se refiere al conjunto de características inherentes a un


producto que cumplan con ciertas especificaciones para satisfacer las necesidades del con-
sumidor. En este sentido, la capacidad del proceso indica la tolerancia permitida de acuerdo
a las especificaciones señaladas para obtener una producción adecuada.

Factores relacionados con la calidad del diseño

Para conseguir excelente calidad en el diseño de un producto hay que tener en cuenta
las dimensiones básicas de la calidad:
1. Dimensión técnica: engloba los aspectos científicos y tecnológicos que afectan al
producto.
2. Dimensión humana: cuida las buenas relaciones entre clientes y empresas.
3. Dimensión económica: se ocupa de minimizar costos tanto para el cliente como
para la empresa.

115
Sánchez Marchán, Nanci. (2014).Introducción a la Ingeniería Industrial, Material Instruccional de Apoyo. Caracas: UNA

Otros parámetros de la calidad a considerar en el diseño de un producto son :


•• Calidad de diseño: es el grado en el que un producto se ve reflejado en
su diseño.
•• Calidad de conformidad: es el grado de fidelidad con el que es reprodu-
cido un producto respecto a su diseño.
•• Calidad de uso: el producto ha de ser fácil de usar, seguro, fiable, etc.
•• El cliente es el nuevo objetivo: la calidad de un producto no se determina
solamente por parámetros puramente objetivos, es necesario incluir tam-
bién las opiniones del cliente que usa el producto.

En conclusión, para obtener un producto de calidad, es importante asegurar sus ca-


racterísticas desde el momento de su diseño. Un producto excelente es aquel que satisface
las necesidades del cliente, por ello, para desarrollar y lanzar al mercado un producto, es
necesario considerar los aspectos relacionados tanto con el diseño como con su proceso de
producción.

Diseño de procesos
Ver definición de proceso de producción en pág. 108, ver figura 23.

   

 
 
 
  Figura 23. Esquema de un proceso de producción

El proceso de producción debe diseñarse en paralelo con el proceso de diseño del pro-
ducto. Se basa en la premisa fundamental de que todo es mejorable. No es usual realizar
un diseño de procesos partiendo de cero. Sólo se aborda ese problema cuando se trata de
una fábrica nueva o de un nuevo producto que nada tiene que ver con el que se fabrica en
la actualidad.

En este sentido el diseño de un proceso se efectúa:


1. Cuando se va a ofrecer un bien o servicio nuevo.
2. Cuando se modifica un producto.
3. Cuando surge una nueva tecnología.

En el diseño de un proceso es necesario estudiar el ambiente organizacional, las carac-


terísticas deseadas, actividades a realizar y técnicas a emplear:

116
Sánchez Marchán, Nanci. (2014).Introducción a la Ingeniería Industrial, Material Instruccional de Apoyo. Caracas: UNA

•• Ambiente. Se debe crear un ambiente dentro de la organización donde el diseñador


pueda trabajar de forma efectiva.
•• Características. Las que más se destacan son la funcionalidad, la integridad, la sim-
plicidad en la fabricación, la seguridad, el entorno, el medio ambiente y el ciclo de vida
del producto.
•• Actividades. Van desde la identificación de las necesidades hasta el lanzamiento del
producto, teniendo en cuenta las conexiones e implicaciones de otros procesos con
los que el diseño está altamente relacionado, es decir, el marketing, la fabricación y
ensamblado, las ventas, la ergonomía y estética de los productos, entre otros.
•• Técnicas. Se requieren técnicas específicas como la ingeniería concurrente, el des-
pliegue de la función de calidad (QFD), los diagramas de proceso, el CAD-CAM y
otras técnicas de mayor alcance de aplicación, como por ejemplo el benchmarking, el
cual permite optimizar en gran medida el diseño basándose en modelos implantados
en otras empresas similares.

Etapas del diseño de procesos:

1. Generación de la idea.
2. Selección del producto.
3. Diseño preliminar.
4. Diseño final.
5. Selección del proceso.
6. Decisiones finales.

Las cuatro primeras etapas se corresponden con las etapas del diseño de un producto.
Ver página 111.

Etapa cinco. Se evalúan las opciones tecnológicas y métodos para elegir la tecnología,
los equipos y el flujo del proceso más adecuado.

Etapa seis. es decir, decisiones finales; se determina la planificación de la producción,


distribución en planta, capacidad, transporte, mantenimiento, ciclo de vida del producto, tra-
tamientos de residuos y problemas medioambientales, seguridad y programación del traba-
jo. Es la etapa más laboriosa y por lo tanto la que tendrá un mayor costo asociado.

Tipos de procesos productivos

Los procesos se pueden clasificar según el tipo de flujo del producto: en línea, intermi-
tente, o por proyecto. Según el tipo de servicio al cliente: fabricación para inventarios, fabri-
cación para surtir pedidos.

La selección del tipo de proceso productivo es estratégica para la empresa, pues puede
elevar los costos, mejorar la calidad, mejorar el servicio al cliente, atender cambios rápidos
en los productos, entre otros.En la figura 24 se puede observar los diferentes tipos de pro-
cesos productivos.

117
Sánchez Marchán, Nanci. (2014).Introducción a la Ingeniería Industrial, Material Instruccional de Apoyo. Caracas: UNA

INTERMITENTE TIPO
TALLER

BAJO
Ejm. reparación de vehículo
PROCESO POR PROYECTO
Ejm. construcción de un PROCESO
avión, edificio. INTERMITENTE

VOLUMEN DE PRODUCCIÓN
INTERMITENTE TIPO
LOTE
Ejm. muebles, zapatos

PRODUCCIÓN EN MASA
Ejm. viveres

PROCESO
DE FLUJO LINEAL O POR
PRODUCTO

ALTO
FLUJO CONTINUO
Ejm. papel, petróleo, luz,
agua.

Figura 24.Tipos de procesos productivos.

Clasificación según el tipo de flujo del producto


Proceso por Proyecto (construcción de aviones, barcos, viviendas, entre otros)
•• Fabricación de productos únicos o exclusivos.
•• Una o muy pocas unidades.
•• Actividades que deben llevarse a cabo en un lapso y en secuencia.
•• Control: técnicas de administración y proyectos (PERT – CPM)
•• Períodos de fabricación largos y costos.

Proceso intermitente
•• Poca cantidad de productos con mucha variedad.
•• Utiliza las mismas instalaciones para la producción de distintos produc-
tos.
•• Equipos y trabajos similares, se agrupan en centros de trabajo.
•• Un producto pasa por distintos centros de trabajo.
•• Se dividen en dos tipos: taller y lotes.

Proceso intermitente tipo taller (reparación de automóviles o electrodomésticos, sa-


nidad, entre otros)
•• Se fabrica una cantidad pequeña de un producto.
•• Características del producto se ajustan a las exigencias del cliente.
•• Son procesos muy flexibles.
•• Un lote de producto puede hacerse una sola vez.

118
Sánchez Marchán, Nanci. (2014).Introducción a la Ingeniería Industrial, Material Instruccional de Apoyo. Caracas: UNA

Proceso intermitente por lotes (industria farmacéutica, de equipos pesados, entre


otros)
•• Se producen lotes de productos una y otra vez.
•• El cliente elige entre una amplia variedad de productos, pero no es a
medida.
•• Por lo general los lotes son más grandes que en los talleres.
•• Un lote pasa por distintos centros de trabajo.

Producción en masa (automóviles, calculadoras, bolígrafos...)


•• Se fabrican productos discretos.
•• Fabricación en serie.
•• Son más eficientes que los procesos intermitentes.
•• Productos técnicamente homogéneos.
•• Poca variedad de productos. El cliente participa poco.

Proceso continuo (la industria del acero y el papel, entre otros)


•• Siempre se ejecutan las operaciones, en las máquinas preparadas, para
la obtención de un producto igual.
•• Requiere alta automatización.
•• Hay estandarización y controles de calidad efectivo.
•• A veces los productos no son discretos.

Clasificación según el tipo de servicio al cliente:

Fabricación para inventario (refinación del petróleo, fábrica de vidrio, fábrica de mue-
bles, productos enlatados, alimentos rápidos, entre otros).No se asignan pedidos individua-
les.
•• Se cuenta con una línea de producción estandarizada.
•• No se asignan productos individuales.
•• La compañía acumulara inventario anticipándose a la demanda; por lo
tanto, los pronósticos, la administración de inventarios y la planeación de
la capacidad se vuelven esenciales.

Fabricación para suministrar pedido (restaurante, hospital, joyería, compañía telefó-


nica, entre otros)
•• Se responde a los requerimientos del cliente.
•• Se realizan pedidos individuales.
•• Se cuenta con una amplia gama de especificaciones.
•• Las actividades de procesamiento se relacionan con los pedidos indivi-
duales.

119
Sánchez Marchán, Nanci. (2014).Introducción a la Ingeniería Industrial, Material Instruccional de Apoyo. Caracas: UNA

Actividad propuesta
Seleccione la respuesta correcta sobre la base del siguiente planteamiento:
La producción por talleres se diferencia de la producción por proyecto debido a
que:
El apoyo de la producción no es el producto, sino el centro de trabajo por el que
circulan productos diferenciados.
Los productos no se diferencian unos de otros.
La producción es continua.

Representación gráfica de los procesos

En la representación gráfica de los pasos o etapas de un proceso se utilizan símbolos,


algunos son los propuestos por la American National Standard Institute (ANSI).

Inicio - Fin Operación Operación Manual Base de Datos Subproceso

Conectores en la mis-
Decisión Documento Almacenamiento ma y dista página
Línea de flujo
O Archivo

Ejemplo Ilustrativo
Pasos o etapas de un proceso productivo químico:
  Inicio

Pesar materia
prima

Agregar reactor
 

Reacción
química

No  
Sacar muestra

Si  
Enfriar y
Viscosidad   descargar
Medir viscosidad

Fin

120
Sánchez Marchán, Nanci. (2014).Introducción a la Ingeniería Industrial, Material Instruccional de Apoyo. Caracas: UNA

Ventajas de los diagramas de un proceso productivo

•• Favorecen la comprensión del proceso al mostrarlo como un dibujo.


•• Permiten identificar problemas y oportunidades de mejora del proceso
(pasos redundantes, reprocesos, conflictos de autoridad, cuellos de bo-
tella)
•• Excelente herramienta para capacitar a los empleados.

Análisis de procesos

Estudio de procesos existentes con la finalidad de mejorarlos.

Herramientas utilizadas:

- Diagramas de flujo (buen nivel de detalle).

- Mapas en función del tiempo o mapas de procesos, entre otros.

Medición de procesos

Productividad:
Prod. = Salidas / Entradas.
Prod. = Uni. Producidas / tiempo.
Capacidad diseñada o proyectada: es la tasa de producción ideal para la cual se di-
señó el sistema. Se expresa con una relación (Toneladas producidas por año, semana, mes;
volumen producido por año; clientes atendidos por hora)

Tasa de uso de la capacidad: grado en el que una empresa utiliza su capacidad produc-
tiva y es igual a:

Tasa de uso de capacidad = salida real /capacidad diseñada x 100%

Como ejemplo de ello se puede citar lo siguiente:

Tasa uso de capacidad = horas reales máquina /horas disponibles de máquina x


100%

Calidad:
•• Número de productos defectuosos identificados tanto internamente como
externamente.
•• Cantidad de desechos del proceso.

Velocidad de entrega: se mide en dos dimensiones:


•• Tiempo de suministro: tiempo desde que se encarga el producto hasta
que el cliente lo recibe.

121
Sánchez Marchán, Nanci. (2014).Introducción a la Ingeniería Industrial, Material Instruccional de Apoyo. Caracas: UNA

•• Variabilidad de los tiempos de entrega (disminuirla para así disminuir la


incertidumbre del cliente).

Flexibilidad:
•• Tiempo requerido por un proceso para pasar de elaborar un producto a
otro.
•• Capacidad del proceso para elaborar más de un producto a la vez. Cuan-
tos más productos puedan hacerse simultáneamente, más flexible es el
proceso.

En el desarrollo de los procesos se pueden presentar un cuello de botella, es decir,


cuando la operación que tiene la capacidad más baja, limita la salida del producto del
sistema. A continuación se presentan ejemplos sencillos de esta situación:

Ejemplo Ilustrativo Nº 1

200 unids/hora 50 unids/hora 200 unids/hora

La segunda operación es cuello de botella.

Ejemplo Ilustrativo Nº 2

200 unids/hora 200 unids/hora 200 unids/hora

Cualquiera de las operaciones puede ser cuello de botella si la demanda supere las 200
u /hora.

También se puede agregar con respecto a las operaciones cuello de botella que:
•• Cuando se procesan productos diferentes aparecerán cuellos de botella
específicos.
•• Cuando existe variabilidad en la demanda se pondrán de manifiesto cue-
llos de botella flotantes.

122
Sánchez Marchán, Nanci. (2014).Introducción a la Ingeniería Industrial, Material Instruccional de Apoyo. Caracas: UNA

Un buen diseño puede atraer la atención, mejorar el funcionamiento de un producto o


un proceso, disminuir los costos de producción y lograr una gran ventaja dentro del mercado
meta.

Para que un proceso productivo funcione bien es necesario considerar las relaciones
con su entorno e involucrar una serie de elementos, dispositivos, acciones y controles que lo
hagan parte de un sistema, es decir, considerar las especificaciones de entrada (materia pri-
ma, materiales, etc.), operaciones, flujos de trabajo, métodos, personal y equipos necesarios
para la producción de bienes y servicios (salida).

Un bien es un conjunto de tangibles tales como material, el envase, el etiquetado, etc.;


pero que también incluye elementos intangibles tales como las garantías, la marca, etc.; es
decir, un bien se compone de los atributos que pueda tener.

Mientras que un servicio es un conjunto de intangibles tales como desempeños, es-


fuerzos, atenciones, etc.; pero que también incluye elementos tangibles. Por ejemplo, en un
restaurante: los alimentos, las bebidas, muebles, etc. Un servicio se basa en la confiabilidad,
calidad o calidez, en la atención al cliente.

Definición de sistema

Un sistema se puede definir como:

Un conjunto de elementos interrelacionados, que actúan simultáneamente para alcanzar


un objetivo.

Los elementos que componen un sistema no son independientes, sino que están estre-
chamente relacionados, de forma que las modificaciones que se producen en uno de ellos
pueden influir en los demás.

Un sistema puede ser abierto o cerrado como se indica en la figura 25.

Sistema Abierto
(U Organico: células, hombre, plantas...)

SISTEMA
Sistema Cerrado
(El Universo)

Figura 25.Tipos de sistemas.

123
Sánchez Marchán, Nanci. (2014).Introducción a la Ingeniería Industrial, Material Instruccional de Apoyo. Caracas: UNA

Sistema cerrado, es un sistema físico (o químico) que trabaja de acuerdo con estable-
cidas relaciones de causa y efecto, donde determinadas entradas producen determinadas
salidas, reciben poca influencia del ambiente. En realidad, no existen sistemas cerrados, ya
que siempre se establece y conserva un ligero intercambio con el ambiente.

El universo entero considerado como un todo es probablemente el único sistema real-


mente cerrado, sin embargo, en la práctica muchos sistemas no completamente aislados
pueden estudiarse como sistemas cerrados, con un grado de aproximación muy bueno o
casi perfecto: tal es el caso de los sistemas completamente estructurados, donde los ele-
mentos y relaciones se combinan de una manera peculiar y rígida produciendo una salida
invariable, llamados también sistemas mecánicos, como por ejemplo: las máquinas, los sis-
temas físicos, el reloj, el termóstato, etc.

Sistema abierto (u orgánico) funciona dentro de relaciones causa-efecto desconoci-


das e imprecisas y mantienen un intercambio agudo con el ambiente a través de entradas y
salidas. Intercambian energía y materia con el ambiente; su estructura es óptima cuando el
conjunto de elementos del sistema se organiza, aproximándose a una operación adaptativa.

El concepto de sistema abierto puede ser aplicado al individuo en diversos escenarios:


en un escenario individual, grupal, en un escenario de una organización y en el de una so-
ciedad, por ello se puede hablar de un microsistema hasta de un suprasistema; en términos
más amplios, va de la célula al universo. Como ejemplo de sistemas abiertos se pueden citar
los siguientes: células, plantas, el hombre, la organización, la sociedad, entre otros.

Diferencias entre sistemas cerrados y sistemas abiertos

Existen ciertas diferencias entre un sistema abierto y un sistema cerrado, las cuales se
presentan en la siguiente tabla:

Tabla 12. Diferencias entre sistemas cerrados y sistemas abiertos

Sistemas abiertos Sistemas cerrados


El sistema abierto interactúa constantemente El sistema cerrado interactúa poco con el am-
con el ambiente en forma reciproca, o sea, lo biente.
influencia y es influenciado.
El sistema abierto puede crecer, cambiar, El sistema cerrado no
adaptarse al ambiente y hasta reproducirse
bajo ciertas condiciones ambientales
Es propio del sistema abierto competir con No así el sistema cerrado
otros sistemas
Mantiene así mismo, un continuo flujo de en- Su comportamiento es determinístico y progra-
trada y salida, logrando el mantenimiento y mado.
sustentación de los componentes
Pueden desarrollarse en dirección a un estado Mantienen un estado de equilibrio químico y ter-
en decreciente orden y organización (entropía modinámico, es decir cumplen el segundo prin-
negativa). cipio de la termodinámica (entropía positiva).

124
Sánchez Marchán, Nanci. (2014).Introducción a la Ingeniería Industrial, Material Instruccional de Apoyo. Caracas: UNA

En conclusión los sistemas cerrados (o mecánicos) trabajan de acuerdo con relaciones


establecidas de causa y efecto, además conservan un intercambio establecido también con
el ambiente, donde determinadas entradas producen determinadas salidas. En cambio un
sistema abierto (u orgánico) funciona dentro de relaciones causa-efecto y mantienen un in-
tercambio dinámico con el ambiente.

Actualmente, los ingenieros industriales son más emprendedores, creativos y por su-
puesto, sobre los avances de las diferentes tecnologías, mejoran cada vez más sus formas
de trabajar, así como, los sistemas de producción.

Organización de un sistema de producción


Para organizar un sistema de producción se han de considerar los siguientes aspectos:
•• Producción: se ocupa específicamente de la actividad de producción de
artículos, es decir, de su diseño, su fabricación y del control del personal,
los materiales, los equipos, el capital y la información para el logro de
esos objetivos.
•• Operaciones: es un concepto más amplio que el de producción. Se re-
fiere a la actividad productora de artículos o servicios de cualquier orga-
nización ya sea pública o privada, lucrativa o no. La gestión de operacio-
nes, por tanto, engloba a la dirección de la producción.
•• Producto: es el nombre genérico que se da al resultado de un sistema
productivo y que puede ser un bien o un servicio. Un servicio es una ac-
tividad solicitada por una persona o cliente.

A continuación se puede observar una representación gráfica general de un sistema de


producción y un ejemplo ilustrativo, el cual permitirá conocer cómo se conectan los paráme-
tros que lo conforman:

Insumos Procesamiento Producto

Control Realimentación

Figura 26. Representación gráfica de un sistema general de producción.

125
Sánchez Marchán, Nanci. (2014).Introducción a la Ingeniería Industrial, Material Instruccional de Apoyo. Caracas: UNA

Ejemplo ilustrativo de un sistema de producción:


Un ejemplo sencillo de sistema de producción, puede ser una comida rápida.
(Hamburguesa y papas fritas). Los materiales (materia prima, insumos) para la
preparación de esta comida son: carne, lechuga, tomate, pan redondo, papas
prefritas congeladas, queso amarillo. Los cuales constituyen la entrada.
El proceso de transformación consiste en: la cocción de la carne y las papas, así
como, ensamblar la hamburguesa.
Salida o producto: hamburguesa y papas fritas

Representación gráfica del ejemplo ilustrado:

Insumos: Procesamiento: Producto:


carne, lechuga, Cocción de la Hamburguesa
tomate, pan redon- carne y las papas. y papas fritas
do, papas prefritas Ensamblar la ham-
congeladas, queso burguesa.
amarillo

Control:
Inspección Realimentación:
de insumos Satisfacción de clientes
cocción y
ensamblaje

Actividad propuesta
En atención a la teoría expuesta en este punto y al ejemplo citado, construye con
tus palabras una definición de sistema de producción:

Compare su definición con la mostrada a continuación:

Sistema de producción

Un sistema de producción es el modo en el que se lleva a cabo la entrada de insumos


(materiales, información, etc.), el proceso de transformación de esos insumos para lograr
como resultado un producto (bien o servicio) que es entregado a clientes o consumidores,
teniendo en cuenta la retroalimentación y el control adecuado del propio sistema.

126
Sánchez Marchán, Nanci. (2014).Introducción a la Ingeniería Industrial, Material Instruccional de Apoyo. Caracas: UNA

Actividad propuesta
Muestre las relaciones de insumos–transformación–resultados, para los si-
guientes sistemas de producción: restaurante, fábrica de automóviles, universidad.

PARÁMETROS DE UN SISTEMA DE PRODUCCIÓN


Los parámetros del sistema de producción se pueden clasificar como sigue:

•• Entrada, insumo o impulso (input): es la fuerza de arranque del siste-


ma, que suministra el material o la energía para la operación del sistema.
•• Salida, producto o resultado (output): es la finalidad para la cual se
reunieron elementos y relaciones del sistema, los productos obtenidos o
servicios prestados. Se producen también ciertos productos no desea-
dos (residuos, contaminación, etc.).
•• Procesamiento, procesador o transformador (throughput): es el fe-
nómeno que produce cambios, es el mecanismo de conversión de las
entradas en salidas o resultados. Generalmente es representado como
la caja negra, en la que entran los insumos y salen cosas diferentes, que
son los productos.
•• Realimentación (feedback): es la función de retorno del sistema que
tiende a comparar la salida con un criterio preestablecido, manteniéndola
controlada dentro de un estándar o criterio, es decir, un mecanismo para
conocer si se están cumpliendo los objetivos.

OTROS ASPECTOS IMPORTANTES A CONSIDERAR EN UN SISTEMA DE


PRODUCCIÓN
Ruido en el sistema. Es cuando existe una deficiencia por mal funcionamiento del
sistema, este ruido puede ser producido por los componentes del sistema o por el medio
ambiente que rodea al sistema. En el caso de los sistemas insumo-producto, para eliminar
el ruido es necesario corregir o sustituir el componente del sistema que no funciona bien.

Estabilidad del sistema. Es la propiedad para resistir perturbaciones, evitando que se


deje de cumplir con el objetivo, por ejemplo la capacidad de una máquina para recobrar el
equilibrio después de sufrir una perturbación originada por una falla eléctrica.

Entorno o ambiente externo. Son todos aquellos elementos que no forman parte de
la función de producción pero que están directa o indirectamente relacionados con ella. Está
en constante interacción con el sistema, ya que éste recibe entradas, las procesa y efectúa
salidas. La supervivencia de un sistema depende de su capacidad de adaptarse, cambiar y
responder a las exigencias y demandas del ambiente externo. Aunque el ambiente puede
ser un recurso para el sistema, también puede ser una amenaza, como por ejemplo las or-
denanzas jurídicas del estado.

127
Sánchez Marchán, Nanci. (2014).Introducción a la Ingeniería Industrial, Material Instruccional de Apoyo. Caracas: UNA

Existen dos tipos:

1. Entorno genérico: es todo aquello que rodea a la empresa o concuerda con el


entorno de la empresa. Por ejemplo: las políticas, condiciones legales, la tecnolo-
gía.

2. Entorno específico: es aquel que engloba al resto de los departamentos de la


empresa.

Subsistema. Son los sistemas que componen un sistema total o global. Ejemplos: el
área de ingeniería es un subsistema del sistema Universidad Nacional Abierta; la empresa
es un sistema compuesto por diferentes áreas que son sus subsistemas.

CLASIFICACIÓN DE LOS SISTEMAS PRODUCTIVOS EN ATENCIÓN A SU


PROCESO
En atención a su proceso los sistemas productivos se clasifican en:

1. Sistemas continuos: son aquellos en los que las instalaciones se acondicionan


de acuerdo con las rutas y los flujos que se generan en el proceso de fabricación,
cuando los insumos son homogéneos. En consecuencia puede adoptarse un conjunto
semejante de procesos y de secuencia de procesos. La producción a gran escala de
artículos estándar es características de estos sistemas.

2. Sistemas intermitentes: son sistemas en lo que las organizaciones deben ser


suficientemente flexibles para manejar una gran variedad de productos con diversos
tamaños. Las instalaciones de transporte entre las operaciones deben ser también
flexibles, para acomodarse a una gran variedad de características de los insumos
y a la gran diversidad de rutas que pueden requerir éstos. En este tipo de sistema
la empresa habitualmente fabrica una gran variedad de productos en los que los
volúmenes de venta y constantemente los lotes de fabricación son pequeños en
relación con la producción total. El costo total de mano de obra especializado es
relativamente alto.

3. Sistemas modulares: se desarrollan una serie de componentes básicos de los


productos (módulos) los cuales pueden ensamblarse de tal forma que puedan
producirse un gran número de productos distintos (ejemplo bolígrafos).

4. Sistemas por proyectos: en este tipo de sistemas no existe flujo de producto, pero
sí existe una secuencia de operaciones, todas las tareas u operaciones individuales
deben realizarse en una secuencia tal que contribuya a los objetivos finales del
proyecto. Los proyectos se caracterizan por el alto costo y por el problema que
representa la planeación y control administrativo.

128
Sánchez Marchán, Nanci. (2014).Introducción a la Ingeniería Industrial, Material Instruccional de Apoyo. Caracas: UNA

CLASIFICACIÓN DE LOS SISTEMAS DE PRODUCCIÓN SOBRE LA BASE


DE SU FINALIDAD
De acuerdo a su afinidad los sistemas de producción pueden ser:
a) Primarios: están sujetos a factores incontrolables, es decir, generados por la
naturaleza (agrícola y de extracción). Estos sistemas pueden operar como sistemas
continuos o intermitentes, dependiendo de la demanda en el mercado. Cabe señalar
que la industria del petróleo forma parte no sólo del sistema de extracción, sino
también de la transformación.
b) Secundarios: son los de transformación y artesanal (industria del vidrio, del acero,
petroquímica, automotriz, papelera, la de alimentos, entre otras.). Estos sistemas
funcionan como continuos e intermitentes dependiendo de las necesidades y de la
demanda del mercado. La característica de la industria de la transformación es una
gran división del trabajo aplicado a la producción en masa.
c) Terciarios: engloban todo el sistema productivo de servicios, tienen una relación muy
directa con la mercadotecnia.
Se muestra a continuación una tabla en la que se observan ejemplos de algunos siste-
mas de producción: primario, secundario y terciario.

Tabla 13. Ejemplos de sistemas de producción (primario, secundario, terciario).


Primario Secundario Terciario

Agricultura. Aeroespacial. Banca.


Canteras. Automotriz. Bienes raíces.
Forestal. Bebidas. Comercio mayorista.
Ganadería. Bisutería y accesorios. Comercio minorista.
Minería. Computadoras. Comunicaciones.
Pesca. Electrónica. Educación.
Petróleo(extracción). Enseres domésticos. Entretenimiento.
Equipos. Gobierno.
Instituciones generadoras de energía. Hotelería.
Madera y muebles. Información.
Maquinaria pesada. Mantenimiento.
Materiales para la construcción. Restaurantes.
Metales (orfebrería). Salud.
Papel. Servicios sociales.
Plásticos (formado). Seguros.
Procesamiento de alimentos. Servicios financieros.
Productos de goma. Servicios legales.
Productos farmacéuticos. Transporte.
Productos químicos. Turismo.
Refinación de petróleo. Publicidad.
Textiles.
Vidrios y cerámicas.

129
Sánchez Marchán, Nanci. (2014).Introducción a la Ingeniería Industrial, Material Instruccional de Apoyo. Caracas: UNA

Actividad propuesta
La empresa IASENZA S.A., se dedica a la fabricación de muebles de oficina y tie-
ne sucursales en todo el territorio nacional. Seleccione de acuerdo a la afinidad,
el sector de la producción, donde se encuentra IASENZA:
a) Primario
b) Secundario
c) Terciario

SISTEMAS DE PLANIFICACIÓN Y CONTROL DE LA PRODUCCIÓN


Es importante citar en este punto algunos sistemas de planificación y control de la pro-
ducción que han generado cambios importantes en la industria, entre los que se encuentran:
el JIT o JAT, los sistemas MPR o MRP-I, MRP-II y CAM, entre otros.

El sistema Justo A Tiempo (JAT) de su traducción del inglés Just in Time (JIT);
ideado por los japoneses, es un sistema de organización de la producción para las fábricas.
También conocido como método Toyota, permite aumentar la productividad, reducir el costo
de la gestión por pérdidas en almacenes debido a stocks innecesarios. De esta forma, no se
produce bajo suposiciones, sino sobre pedidos reales.

La filosofía JIT o JAT es un nuevo enfoque en la dirección de operaciones, el cual


pretende que los clientes sean servidos justo en el momento preciso, exactamente en la
cantidad requerida con productos de máxima calidad y mediante un proceso de producción
que utilice el mínimo inventario posible, además que se encuentre libre de cualquier tipo de
despilfarro o costo innecesario. Debe ser considerado como un proceso de mejora continua
de la planificación y control de la producción, en la que diariamente se generan cambios en
la empresa para diseñar condiciones óptimas de fabricación JAT.

La meta que persigue la filosofía JIT es de cero:


•• Defectos.
•• Averías (tiempo inoperativo).
•• Stocks (inventario).
•• Plazos y papeleo o burocracia.
•• Demoras.
•• Entre otras.

Elementos del Sistema JAT:


•• Proveedores.
•• Distribuidores.
•• Inventario.
•• Programación.
•• Nivelado de producción.

130
Sánchez Marchán, Nanci. (2014).Introducción a la Ingeniería Industrial, Material Instruccional de Apoyo. Caracas: UNA

•• Kanban.
•• Calidad.
•• Delegación de poder a los empleados.

Proveedores: el comprador y proveedor deben trabajar juntos, buscando un objetivo


común al eliminar desperdicios y reducir costos, considerándose el proveedor como parte de
la organización, logrando que se establezca cerca de ésta. De esta manera se eliminan las
actividades innecesarias como la recepción, inspección e inventarios, entre otros.

Distribuidores: generar distribuciones en planta flexibles, que reduzcan el movimien-


to tanto del material como del operario. Esto se logra construyendo células de trabajo por
familias de productos, reduciendo distancias, diseñando poco espacio para el inventario,
mejorando la comunicación con los empleados, construyendo equipos flexibles o móviles,
formando a los empleados.

Inventario: es el mínimo necesario para mantener el sistema en funcionamiento. Con


un inventario JAT, llega la cantidad exacta de material en el momento en el que se necesita,
es decir, suprimir stocks o inventarios que esconden la variabilidad o los problemas del sis-
tema de producción.

Programación: la programación de la producción se da a conocer no sólo en la empre-


sa sino también en los proveedores. Es decir, una buena programación mejora la posibilidad
de atender los pedidos de los clientes, reduce el inventario al permitir lotes más pequeños y
reduce el trabajo en curso.

Nivelado de producción: la empresa toma medidas para ejecutar programas nivela-


dos, logrando una distribución uniforme de la producción y de los recursos con el paso del
tiempo establecido. Con el nivelado el cambio entre modelos de programas genera un costo
fijo o insignificante.

Kanban (palabra japonesa que significa tarjeta): es la señal que autoriza mover, produ-
cir o comprar. Consiste en empujar (PUSH), para llevar a cabo una acción con anticipación a
una necesidad y jalar (PULL) para lograr una acción cuando se necesite.El sistema Kanban
mueve las partes a fabricar o fabricadas a través del proceso de producción, como un siste-
ma de programación, en la que cada operación hala el material que necesita de la operación
anterior. Ver más detalles en el contenido del tema inventario.

Calidad: el Sistema JIT muestra todos los problemas de calidad, que quedan ocultos
en el inventario y limita el número de defectos a través de la generación de pequeños lotes.
La mejora de la calidad implica que se necesitan menos protecciones de seguridad, lo que
conduce a un sistema mejor y más fácil de usar.

Delegación de poder a los empleados: es importante involucrar a los empleados en


las mejoras del proceso y del producto, ya que ellos son los que mejor conocen el trabajo.
Por ello, el sistema JAT requiere de atribuciones de poder, formación en diferentes campos,
apoyo de formación y pocas clasificaciones de trabajo.

131
Sánchez Marchán, Nanci. (2014).Introducción a la Ingeniería Industrial, Material Instruccional de Apoyo. Caracas: UNA

Objetivos del sistema JAT o JIT:

El sistema Just-in-Time tiene cuatro objetivos esenciales que son:


•• Atacar los problemas fundamentales.
•• Diseñar sistemas para identificar problemas.
•• Buscar la simplicidad.
•• Eliminar despilfarros.

Atacar los problemas fundamentales. Esto se puede explicar a través de la analogía


del río con las existencias. Cuando una empresa intenta bajar el nivel del río (o sea reducir el
nivel de sus existencias) descubre rocas, es decir, problemas. El nivel de las existencias pue-
de reducirse entonces gradualmente hasta descubrir otro problema; este problema también
se debe resolver y así sucesivamente, ejemplo de esto es el caso de la máquina poco fiable,
la filosofía del JIT indicaría que había que resolver el problema, tal vez con un programa de
mantenimiento preventivo que mejore la fiabilidad de la máquina o, si éste fallara, comprando
una máquina más fiable.

Ejemplo Ilustrativo

PROBLEMAS (ROCAS) SOLUCIÓN JAT


Zonas con cuellos de botella. Aumentar la capacidad.
Tamaños de lote grandes. Reducir el tiempo de preparación.
Plazos de fabricación largos. Reducir colas, etc., mediante un sistema de arrastre.
Calidad deficiente. Mejorar los procesos y/o proveedores.

En el anterior ejemplo ilustrativo se muestran algunos problemas y sus respectivas so-


luciones Just-in-Time. Así el enfoque JIT ante una máquina o un proceso que constituye un
cuello de botella, consiste en reducir el tiempo de preparación para conseguir una mayor
capacidad, buscar máquinas o procesos alternativos, lograr capacidad adicional o incluso,
subcontratar el trabajo en exceso. Un directivo JIT reconoce que ni un aumento del stock de
seguridad ni una programación más compleja logrará resolver el problema fundamental; lo
único que hace es tapar temporalmente las rocas.

Diseñar sistemas para identificar problemas. El sistema de arrastre Kanban saca


los problemas a la luz, con el JIT, cualquier sistema que identifique los problemas se conside-
ra beneficioso y cualquier sistema que los enmascare, perjudicial. Los sistemas de arrastre
Kanban identifican los problemas y por tanto son beneficiosos. Los enfoques tradicionales
tendían a ocultar los problemas fundamentales y de esta forma, retrasar o impedir la so-
lución. Los sistemas diseñados con la aplicación del JIT deben pensarse de manera que
accionen algún tipo de aviso cuando surja un problema. Si realmente se quiere aplicar el JIT
en serio, hay que hacer dos cosas:

132
Sánchez Marchán, Nanci. (2014).Introducción a la Ingeniería Industrial, Material Instruccional de Apoyo. Caracas: UNA

1. Establecer mecanismos para identificar los problemas.


2. Estar dispuesto a aceptar una reducción de la eficiencia a corto plazo con el fin
de obtener una ventaja a largo plazo.

Buscar simplicidad. Los enfoques de la gestión productiva de moda durante la década


de los setenta y principio de los ochenta se basaban en la premisa de que la complejidad era
inevitable. El JIT pone énfasis en la búsqueda de la simplicidad, basándose en el principio de
que enfoques simples conducirán hacia una gestión más eficaz. El primer tramo del camino
hacia la simplicidad cubre dos zonas:

1. Flujo de material.
2. Control.

Eliminar despilfarros. En este contexto significa eliminar todo aquello que no añada
valor al producto. Ejemplos de operaciones que añaden valor son los procesos como cortar
metal, soldar, insertar componentes electrónicos, etc. Ejemplos de operaciones que no aña-
den valor son la inspección, el transporte, el almacenaje, la preparación, entre otros.

Con el JAT se busca la excelencia mediante la continua eliminación de todo lo que no


agrega valor al producto (desperdicio).

El mejor criterio de decisión para medir el valor no puede ser la rentabilidad económica
en los proyectos de desarrollo, sino aquello que diferencia a la empresa de la competencia
y es valorado por el cliente.

El valor mide:
•• Magnitud que crece conforme aumenta la satisfacción de las necesida-
des del cliente o usuario del producto o servicio.
•• La relación de la satisfacción de la necesidad contra el costo.
•• La percepción que tiene el cliente o usuario de obtener mayores bene-
ficios o satisfacer mejor sus expectativas o necesidades, respecto a un
producto o servicio que tiene más valor que uno generado por la compe-
tencia.
“La cadena del valor desagrega una empresa en cada una de sus activida-
des estratégicamente relevantes, para entender el comportamiento de los
costos, las fuentes actuales y potenciales de diferenciación. Una empresa
obtiene ventaja competitiva haciendo sus actividades estratégicamente im-
portantes mejor que sus competidores y/o a un costo menor que ellos” Por-
ter, M.E., Competitive Advantage.

La cadena del valor descompone a la empresa en nueve actividades que crean valor,
con el propósito de entender el comportamiento de los costos y las fuentes de diferenciación
con la competencia. Las nueve actividades que crean valor se descomponen en cinco pri-
marias y cuatro secundarias.

133
Sánchez Marchán, Nanci. (2014).Introducción a la Ingeniería Industrial, Material Instruccional de Apoyo. Caracas: UNA

Actividades primarias:
•• Logística interna.
•• Operaciones.
•• Logística externa.
•• Marketing y ventas.
•• Mantenimiento y servicios.

Actividades secundarias:
•• Adquisición de bienes y servicios.
•• Tecnología, Investigación y Desarrollo (I+D).
•• Administración de recursos humanos.
•• Infraestructura de la empresa.

LOS SISTEMAS MPR: MRP- I Y MRP-II Y EL SISTEMA CAM


El sistema de Planificación de Requerimientos de Materiales (MPR) surge en la década
de 1960 en los Estados Unidos, en la empresa IBM; debido a la necesidad de integrar la
cantidad de artículos a fabricar con un correcto almacenaje de inventario, ya sea de produc-
tos terminados, productos en proceso, materia prima o componentes. Puede decirse que el
MRP es un sistema de control de inventario y programación que responde a las interrogantes
¿Qué orden fabricar o comprar? ¿Cuánta cantidad de la orden? ¿Cuándo hacer la orden?

Su objetivo es disminuir el volumen de existencia a partir de lanzar la orden de compra


o fabricación en el momento adecuado, según los resultados del Programa Maestro de Pro-
ducción (PMP) en el cual se realiza la programación de las unidades que se han de producir
en un determinado período, dentro de una perspectiva de planeación. Se genera a partir de
los pedidos de los clientes de la empresa o de pronósticos de demandas anteriores al inicio
del MRP y puede llegar a ser un insumo del sistema.

La aplicación del MPR, es útil donde existan algunas de las condiciones siguientes:
•• El producto final es complejo y requiere de varios niveles de subensam-
ble y ensamble.
•• El producto final es costoso.
•• El tiempo de procesamiento de la materia prima y componentes, sea
grande.
•• El ciclo de producción (lead time) del producto final sea largo.
•• Se desee consolidar los requerimientos para diversos productos.
•• El proceso se caracteriza por ítems con demandas dependientes funda-
mentalmente y la fabricación sea intermitente (por lotes).

El Sistema de Planificación de Requerimientos de Materiales (MPR o MRP- I): con-


siste en traducir el Plan Maestro de Producción (PMP) o Plan Director como también se le
llama, en necesidades y órdenes de fabricación y/o compras detalladas de todos los produc-

134
Sánchez Marchán, Nanci. (2014).Introducción a la Ingeniería Industrial, Material Instruccional de Apoyo. Caracas: UNA

tos que intervienen en el proceso productivo. También proporciona resultados, tales como,
las fechas límite para los componentes, las que posteriormente se utilizan para la gestión de
taller. Una vez que estos productos del MRP están disponibles, permiten calcular los requeri-
mientos de capacidad detallada para los centros de trabajo en el área de producción (taller).
Se muestra seguidamente una figura que representa el sistema MRP:

PLAN MAESTRO
DE PRODUCCIÓN
(PMP)

LISTA DE CONTROL DE
MATERIALES PLANIFICACIÓN INVENTARIO
DE
REQUERIMIENTOS
DE MATERIALES
(MRP)

PRODUCCIÓN COMPRAS

CONTROL DE PRODUCCIÓN Y
COMPRAS

Figura 27. Sistema de producción MRP.

En conclusión el sistema MRP integra las actividades de producción y compras. Plani-


fica las adquisiciones a proveedores en función de la producción programada y surge de la
práctica y experiencia de la empresa.

El Sistema de Planificación de Recursos de Manufactura (MRP- II): es un sistema


de información integrado que va más allá del MRP de primera generación, para sincronizar
todos los aspectos del negocio. Además de contar con la funcionalidad del sistema MRPI,
el MRP- II coordina las ventas, compras, fabricación, finanzas e ingeniería, así como, la pro-
gramación de otros recursos para la producción como personas, instalaciones, entre otros.
Adopta un plan de producción centrado, que utiliza una sola base de datos, la cual está
ajustada para planear y actualizar las actividades de todos los sistemas de la empresa. Se
fundamenta principalmente sobre la base de la demanda y estudios de mercado.

La aplicación más importante del sistema MRP II es la evaluación de diversas propues-


tas de negocios, debido a que puede simular cómo realizar las adquisiciones, cómo se afec-
tan las cuentas por pagar, cuándo se entrega la mercancía a los clientes, verifica si existen
cuentas por cobrar, cuál debe ser la capacidad afectada por las revisiones de la demanda.

135
Sánchez Marchán, Nanci. (2014).Introducción a la Ingeniería Industrial, Material Instruccional de Apoyo. Caracas: UNA

Los sistemas MRP-I y MRP-II, representan una mejora con respecto a los sistemas
tradicionales de planeación y control de la producción. Sus aplicaciones aumentan a medida
que los gerentes de operaciones continúan implantando nuevos métodos para la administra-
ción de materiales. También están diseñados para disminuir los inventarios, tiempos de es-
pera en la producción y la entrega, disminuciones en los ajustes internos de producción para
compensar los materiales que no se tienen disponibles, además de incrementar la eficiencia.

El Sistema de Fabricación Asistida por Máquinas (CAM): controla máquinas y/o


procesos completos de fabricación utilizando las especificaciones procedentes del Sistema
de Fabricación Asistida por Computador (CAD).

Cuando la producción es intermitente y/o unitaria (operaciones de proyecto), donde el


artículo final está formado por varios subconjuntos y componentes complejos, la tendencia
es utilizar un sistema basado en la teoría de redes. En estos casos se emplean los sistemas
de planificación y control de proyectos que hacen uso de los caminos críticos: Técnica de
Revisión y Evaluación de Programas (Programme Evaluation and Review Technique, PERT),
el Método de Camino Crítico (Critical Path Method, CPM), fundamentalmente y los sistemas
que utilizan la denominada Línea de Balance (LOB; Line of Balance).

Otra técnica útil en la planeación y control de la producción, es la simulación del proceso


productivo a partir de varios softwares (SIMAN; SIN FACTORY, etc.), permitiendo conocer los
diferentes estados del proceso con sólo modificar las variables fundamentales del sistema.

La Gestión Integrada de Materiales (GIM), es otra técnica organizativa que últimamente


está recibiendo mucha atención. Esta técnica está dirigida a lograr una visión integrada del
flujo de materiales con el enfoque logístico.

La utilización de un sistema u otro depende de la estrategia de producción que siga la


organización y de la estructura espacial del proceso productivo. Los criterios de los sistemas
de gestión citados, son diferentes y se parte de unos datos de entrada (inputs) distintos.

Un sistema de producción le otorga a un empresario o fabricante una estructura


que facilita la descripción y ejecución de un proceso productivo; es decir, es la
automatización de la producción en sí misma.

PLANEACIÓN Y PRONÓSTICOS DE LA PRODUCCIÓN

Es importante que las empresas antes de proceder a la producción, cuenten con en-
foques eficaces de pronósticos, punto de partida de toda planeación; en otras palabras, es
estimar la demanda futura de productos y servicios, además de los recursos necesarios para
obtenerlos. A continuación se presenta una caracterización de los conceptos de pronóstico
y planeación, en la que se puede observar la relación que existe entre ambos, para luego
desarrollar cada uno de estos aspectos:

136
Sánchez Marchán, Nanci. (2014).Introducción a la Ingeniería Industrial, Material Instruccional de Apoyo. Caracas: UNA

PLANEACIÓN Y PRONÓSTICO DE LA PRODUCCIÓN

PLANEACIÓN PRONÓSTICO
PLANEACIÓN DE LA DE LA
AGREGADA PRODUCCIÓN PRODUCCIÓN

PLAN DE RECURSOS
PLAN
PLANEACIÓN MAESTRO DE PLAN DE
OPERACIONAL PLAN REQUERIMIENTO PRODUCCIÓN DISTRIBUCIÓN
MATERIALES (MRP) (PMP)

PLAN DE CAPACIDAD

EJECUCIÓN COMPRAS PRODUCCIÓN DISTRIBUCIÓN

Figura 28.
Caracterización de los conceptos de pronóstico y planeación de la producción.

Los pronósticos son el primer paso dentro del proceso de planificación de la produc-
ción, en la que se generan los planes de recursos y capacidad. Los pronósticos sirven
entonces como punto de partida, no sólo para la elaboración de los planes estratégicos,
sino además, para el diseño de los planes a corto, mediano y largo plazo, lo cual permi-
te a las organizaciones, visualizar de manera aproximada los acontecimientos futuros,
eliminar en gran parte la incertidumbre y reaccionar con rapidez a las condiciones cam-
biantes con algún grado de precisión.

PRONÓSTICO DE LA PRODUCCIÓN

Rigss (1998), Domínguez Machuca (1995), entre otros, afirman que:

“Los pronósticos son el primer paso en el proceso de planificación de la producción


y sirven como punto de partida, no sólo para la elaboración de los planes estraté-
gicos (largo plazo), sino además, para el diseño de los planes a mediano y corto
plazo. Lo que permite a las organizaciones visualizar de manera aproximada los
acontecimientos futuros y eliminar en gran parte la incertidumbre para reaccionar
con rapidez a las condiciones cambiantes con algún grado de precisión”

Considerando la afirmación anterior, se puede decir que los pronósticos de producción,


se han convertido en una herramienta indispensable para la planeación de la producción a
corto, mediano y largo plazo, ya que permiten captar de manera más cercana la dinámica
de actividades, para cuantificar, ubicar y determinar la estacionalidad de los volúmenes de
producción. Por ello antes de la planeación es importante el planteamiento de modelos de

137
Sánchez Marchán, Nanci. (2014).Introducción a la Ingeniería Industrial, Material Instruccional de Apoyo. Caracas: UNA

pronóstico con el fin de contar anticipadamente con elementos de juicio o una proyección en
el futuro  que permitan establecer mecanismos de apoyo, para evaluar la ocurrencia proba-
ble de cualquier acontecimiento o el desarrollo de una tendencia.

En lo referente a los tipos de pronósticos, éstos pueden ser clasificados de acuerdo a


tres criterios: según el horizonte de tiempo, según el entorno económico abarcado y según
el procedimiento empleado.

1. Los pronósticos según el horizonte de tiempo son de largo, mediano o corto pla-
zo, y su empleo va desde la elaboración de los planes a nivel estratégico hasta
los de nivel operativo.

2. Los pronósticos según el entorno económico pueden ser de tipo micro o de tipo
macro y se definen de acuerdo al grado en el cual intervienen pequeños detalles
contra grandes valores resumidos.

3. Los pronósticos según el procedimiento empleado se corresponden con un tipo


puramente cualitativo, en aquellos casos en los cuales no se requiere de una
abierta manipulación de datos y sólo se utiliza el juicio o la intuición de quien
pronostica o meramente cuantitativos, cuando se utilizan procedimientos mate-
máticos y estadísticos que no requieren los elementos del juicio.

Sobre la base de lo expresado anteriormente se puede definir el pronóstico como:

El proceso a través del cual los órganos directivos de la empresa diseñan continuamen-
te el futuro deseable y seleccionan las formas de hacerlo factible, es decir, formulan un
conocimiento probable sobre un evento futuro. En el lenguaje de empresa, se suele en-
tender como pronóstico la estimación anticipada del valor de una variable, por ejemplo:
la demanda de un producto.

La finalidad de los pronósticos es predecir el desarrollo futuro de un sistema, de sus


entornos, para ayudar a la toma de decisiones (de planificación) sobre medidas de apoyo,
contramedidas u otras acciones que influyan, en mayor o menor grado, sobre la tendencia
del objeto planificado.

Para hacer el cómputo anticipado del costo, de los gastos e ingresos o rentas de un ne-
gocio, es decir, los presupuestos la empresa se fundamenta en el conocimiento acumulado
de su actividad, de los cambios y pronósticos sobre las cantidades y precios de los inputs y
los outputs.

Los pronósticos que realiza una empresa con mayor frecuencia son:
•• Pronóstico de ventas.
•• Pronóstico de producción.
•• Pronóstico de compras.
•• Pronóstico de gastos.

138
Sánchez Marchán, Nanci. (2014).Introducción a la Ingeniería Industrial, Material Instruccional de Apoyo. Caracas: UNA

•• Pronóstico de inversiones.
•• Pronóstico de caja.

Las técnicas utilizadas en los pronósticos de producción, tema de interés en este texto
son:
•• Cualitativas.
•• Cuantitativas.

Técnicas cualitativas: una técnica cualitativa es aquella que no requiere de una abierta
manipulación de datos, sólo se utiliza el juicio de quien pronostica. Funcionan cuando hay
falta o escasez de datos históricos y cuando es difícil convertir en números las variables que
intervienen en la determinación de la demanda futura. La mayoría es bajo costo y no requie-
ren de equipo computacional para hacerse, aunque su planeación implica una gran inversión
de tiempo por parte de los directivos.

En la tabla que se muestra a continuación se presentan las técnicas cualitativas más


importantes:

Tabla 14. Técnicas cualitativas de pronóstico


Técnica Definición
Un grupo de expertos responde, de manera anónima, a un cuestio-
nario que pregunta sobre las proyecciones de ventas de la empre-
Delphi
sa. Un moderador lee en voz alta las respuestas y, entre todos,
buscan consenso.
TÉCNICAS CUALITATIVAS

Juicio informado
Se basa en la opinión general de un grupo de directivos o gerentes
u opiniones de los
de la empresa.
gerentes/ejecutivos

Se basa en la evaluación de las etapas de un producto o servicio


Analogía de ciclos para predecir su demanda en el mercado. Esto es, desde la intro-
de vida ducción, inicio y crecimiento, hasta las etapas de madurez y decli-
nación.
Investigación de Se propone recolectar datos de diversas maneras (entrevistas,
mercados cuestionarios) para probar hipótesis acerca del mercado.
Consiste en recopilar las estimaciones realizadas por los vendedo-
Información de los
res (o distribuidores) acerca de las ventas esperadas en sus territo-
vendedores
rios, con el fin de suponer la tendencia y cambios futuros.

Técnicas cuantitativas: no requieren de elementos de juicio; son procedimientos me-


cánicos que producen cantidades que relacionan variables internas o externas con los ni-
veles de demanda, lo que brinda una visión amplia del sector. Los costos que implican son
de medios a bajos y usualmente requieren de equipo de cómputo, ya que necesitan de una
manipulación de datos mucho más compleja. Son más útiles para elaborar pronósticos a me-

139
Sánchez Marchán, Nanci. (2014).Introducción a la Ingeniería Industrial, Material Instruccional de Apoyo. Caracas: UNA

diano plazo de productos o servicios existentes y para el diseño de estrategias de marketing,


producción y contratación de personal.

A continuación se definen las técnicas cuantitativas de pronósticos más utilizadas y un


diagrama en la que se puede observar un resumen de estas técnicas con sus respectivas
fórmulas. Ver tabla 15.

Tabla 15. Definición de las técnicas cuantitativas de pronóstico


Técnica Definición

TÉCNICAS Nº 1 Esta técnica sirve para calcular el pronóstico de ventas para el


CUANTITATIVAS Promedio siguiente período exclusivamente, como su nombre lo indica es un
Móvil Simple promedio donde se obtiene n datos; para definir en forma práctica cuál
(PMS)
será el mejor resultado, se deberá tomar en cuenta el de menor error
al cuadrado < (D-P)2. En donde D: demanda; P: pronóstico. Los datos
se toman en función de cómo se quieran promediar, es recomendable
utilizar 10 datos o más.

PMS=Σ Demanda de n períodos anteriores


n
Nº 2 Ésta es otra técnica cuantitativa que sirve para calcular el pronóstico
Promedio de la demanda o de las ventas para períodos futuros. Para su apli-
Modelos de Móvil Doble cación y cálculos es recomendable seguir el procedimiento que se
series de tiempo (PMD)
indica:
Se calcula el PMS, considerándose el conjunto de datos y los valores
Sobre la base de asignados para n. Se determina el mejor pronóstico con antecedente
la observación
en el menor error al cuadrado < (D-P)2. Se calcula el promedio móvil
utilizan datos
históricos para doble y los valores correspondientes: a = 2(PMS) – PMD, intersec-
pronosticar. ción con eje vertical y b = n/n-1 (PMS – PMD), pendiente de línea de
regresión. Se calcula el pronóstico para el período deseado: y= a+bx,
donde:y = pronóstico deseado o buscado (final) y x el periodo en el
que se desea el pronóstico.
Nº3 Permite calcular los pronósticos de las ventas de la demanda para el
Ajuste siguiente período únicamente, la aproximación exponencial. Es una
Exponencial ponderación o valor de ajuste con cierto grado de error, este valor de
Simple (AES)
ajuste fluctúa entre (0.1 y 1). Si el valor de ponderación es pequeño el
deslizamiento o ajuste será gradual y mínimo. Para asignar el valor de
ajuste o de ponderación se debe tener en cuenta lo siguiente: la de-
manda en condiciones: De estabilidad a = 0.1, 0.2 y 0.3. En condicio-
nes de promedio a = 0.4, 0.5, 0.6. En proceso de cambio o de nuevos
productos a = 0.7, 0.8, 0.9. Se calcula luego el AES: a (D-P); P=P’ + a
(D-P);(D-P)2
Nº4 Técnica cuantitativa que permite calcular los pronósticos de la deman-
Ajuste da para períodos futuros, teniendo como antecedente datos históricos
Exponencial en cuanto a períodos y demanda. Se sigue el siguiente procedimiento:
Doble (AED)
Se calcula el pronóstico mediante ajuste exponencial simple, conside-
rando los valores del factor de ajuste y se selecciona el mejor pronós-
tico obtenido teniendo en cuenta el menor error< (D-P)2 y se calcula
el ajuste exponencial doble. Luego se calculan los siguientes paráme-
tros: a= 2(AES) – AED, b = a /a-1 (AES – AED). Finalmente se obtiene
el pronóstico final y = a+bx

140
Sánchez Marchán, Nanci. (2014).Introducción a la Ingeniería Industrial, Material Instruccional de Apoyo. Caracas: UNA

Nº 5 Sirve para calcular el pronóstico de ventas cuando existe estacio-


Índices de nalidad o ciclos y también se utiliza cuando en cada período existen
Estacionali- diferencias de ventas muy marcadas, razón por la cual se hace nece-
dad sario calcular un índice que permitirá un ajuste por cada período. El
procedimiento es el siguiente:
1. Hacer sumatorias horizontales y verticales de los datos y
calcular sus promedios.
2. Calcular el promedio total
3. Calcular los índices de estacionalidad IE= X/Xtotal
4. Calcular el pronóstico y= a+ b x

Modelo Nº 6 Esta técnica permite el cálculo de los pronósticos para períodos futu-
asociativo o Mínimos ros, para lo cual requiere de registros históricos que sean consisten-
causal Cuadrados tes, reales y precisos.
Consiste en llevar el total de las desviaciones elevadas al cuadrado a
Incorpora los un valor mínimo: su objetivo es determinar los coeficientes a y b, que
factores que
son conocidos como coeficientes de regresión, donde x es la varia-
pueden influir en
la cantidad por ble independiente (tiempo), y es la variable dependiente (pronóstico
de la demanda). En la práctica se pueden utilizar dos métodos para
calcular los pronósticos a través de mínimos cuadrados: fórmula
general y método simplificado.

Fórmula general:

x=
∑x a = y − bx
n
n ∑ xy − ∑ x ∑ y
b=
y=
∑y n ∑ x 2 − (∑ x) 2
n
y = a + bx
Donde:

n = tamaño de la muestra o el número de períodos


x = período en el que se desea el pronóstico
y = el pronóstico

Método simplificado:

x=
∑x a = y − bx
n
b=
∑ xy
y=
∑y ∑x 2

n
y = a + bx

141
Sánchez Marchán, Nanci. (2014).Introducción a la Ingeniería Industrial, Material Instruccional de Apoyo. Caracas: UNA

Resumen de técnicas cuantitativas de pronóstico y las fórmulas usadas en su cál-


culo:

Se toma en cuenta el resultado de - Se calcula el PMS < (D-P)2


menor error al cuadrado < (D-P)2. -Se calcula el PMD y los valores
En donde: correspondientes:a = 2(PMS) –
D: demanda PMD y
P:pronóstico b = n/n-1 (PMS – PMD)
PMS = ∑ (Demanda de periodos -Se calcula el pronóstico para el
anteriores)/n periodo deseado
Se utilizan 10 datos o más. y= a¬+bx

PROMEDIO PROMEDIO
-Se calcula el AES
< (D-P)2
MÓVIL MÓVIL Estabilidad: a= 0.1,
-Se calcula el AED. SIMPLE DOBLE 0.2 y 0.3.
-Se calculan los valo- Promedio: a = 0.4,
res correspondientes 0.5, 0.6.
a:a= 2(AES) – AED, AJUSTE AJUSTE Proceso de cambio
b = a /a-1(AES – EXPONENCIAL EXPONENCIAL o de nuevos produc-
AED) DOBLE SIMPLE tos a = 0.7, 0.8, 0.9.
Finalmente se obtie- Se calcula luego el
ne el pronóstico final AES: a (D-P); P=P’
y = a+bx + a (D-P);(D-P)2

MÍNIMOS PRONÓSTICO ÍNDICES DE


CUADRADOS DE LA ESTACIONALIDAD
PRODUCCIÓN

Fórmula General: n=tamaño de la 1. Hacer sumatorias horizontales y


verticales.
y=
∑y muestra
2. Calcular el promedio total
n x=período en el
que se desea el 3. Calcular los índices de
x
x pronóstico. estacionalidad: IE=X/XTotal
n y = el pronóstico 4. Calcular el pronóstico y = a +bx

Método Simplificado
∑y a = y − bx
y=
∑y a = y − bx y=
n
n ∑ xy ∑ xy
b= x b=
∑x
x ∑x
2
2
x
x n
n y = a + bx

Figura 29. Resumen de técnicas cuantitativas de pronóstico.

En este texto sólo se desarrollará la técnica Nº 1 (Promedio Móvil Simple) como ejemplo
ilustrativo del cálculo de los pronósticos de la demanda para períodos futuros. Las técnicas
restantes se examinarán cuando el estudiante avance en el estudio de su carrera, pero si al
estudiante le motiva el tema y quiere ahondar en el uso de las otras técnicas, puede revisar
algunos textos de control de la producción.

142
Sánchez Marchán, Nanci. (2014).Introducción a la Ingeniería Industrial, Material Instruccional de Apoyo. Caracas: UNA

La Técnica Nº 1 Promedio Móvil Simple (PMS), está definida en la tabla Nº 15. Los
datos a utilizar están presentados en función de cómo se quieran promediar u obtener los
resultados, con menor o mayor exactitud; por ello (n) puede tomar valores comprendidos
entre 2, 3, 4, 5....etc. En la práctica es recomendable utilizar bloques de información que en
promedio tengan 10 ó más datos, lo cual permitirá una mejor interpretación o visión del com-
portamiento del producto en el pronóstico.

Ejemplo Ilustrativo
La empresa SABROSITOS S.A., desea elaborar el pronóstico de ventas para
uno de sus productos de mayor demanda en el mercado se le conoce como
“Platanitos Sabrositos”, este pronóstico se requiere para el mes de octubre de
2012, para lo cual se debe considerar que n= 2, 3, 4. Sabiendo que en los últimos
meses el área de marketing ha registrado la información histórica que se indica
en la siguiente tabla:
Períodos mensua-
Enero Febrero Marzo Abril Mayo Junio Julio Agosto Septiembre Octubre
les

Demanda (D) 30 35 28 20 25 30 35 40 50 ¿?

Pronósticos (P) 24 27.5 32.5 31.5 24 22.5 27.5 32.5 37.5 45

Solución:

Se completa la tabla con (D-P) y (D-P)2

Cuando n= 2, se comienzan los cálculos después del segundo mes, es decir, marzo.

Períodos Demanda Pronósticos


(D-P) (D-P)2
mensuales (D) (P)
Enero 30 - - -

Febrero 35 - - -

Marzo 28 32.5 -4.5 20.25

Abril 20 31.5 -11.5 132.25

Mayo 25 24 1 1

Junio 30 22.5 7.5 56.25

Julio 35 27.5 7.5 56.25

Agosto 40 32.5 7.5 56.25

Septiembre 50 37.5 12.5 156.25

Octubre ¿? 45 Suma = 478.5

143
Sánchez Marchán, Nanci. (2014).Introducción a la Ingeniería Industrial, Material Instruccional de Apoyo. Caracas: UNA

Cuando n= 3, se comienzan los cálculos en abril, es decir, después del tercer mes:

Períodos Pronósticos
Demanda (D) (D-P) (D-P)2
mensuales (P)
Enero 30 - - -
Febrero 35 - - -
Marzo 28 - - -
Abril 20 31 -11 121
Mayo 25 27.66 -2.66 7.07
Junio 30 24.33 5.66 32.14
Julio 35 25 10 100
Agosto 40 30 10 100
Septiembre 50 35 15 225
Octubre ¿? 41.66 Suma = 585.21

Cuando n= 4, se comienzan los cálculos en mayo, después del cuarto mes:

Períodos Pronósticos
Demanda (D) (D-P) (D-P)2
mensuales (P)
Enero 30 - - -
Febrero 35 - - -
Marzo 28 - - -
Abril 20 - - -
Mayo 25 28.25 -3.25 10.56
Junio 30 27 3 9
Julio 35 25.75 9.25 85.56
Agosto 40 27.5 12.5 156.25
Septiembre 50 32.5 17.5 306.25
Octubre ¿? 38.75 Suma = 567.62

Sobre la base de los resultados se puede concluir que el mejor pronóstico es el realiza-
do para n= 2, en el que se obtienen 45 unidades, ya que se debe tomar en cuenta la suma-
toria de menor error al cuadrado.

Los pronósticos, según el procedimiento empleado, pueden ser de tipo puramente


cualitativo, en aquellos casos en los que no se requiere de una abierta manipulación de
datos y sólo se utiliza el juicio o la intuición de quien pronostica. O puramente cuanti-
tativos, cuando se utilizan procedimientos matemáticos y estadísticos que no requieren
los elementos del juicio. Esta última clasificación es la más generalizada por los distintos
autores consultados.

144
Sánchez Marchán, Nanci. (2014).Introducción a la Ingeniería Industrial, Material Instruccional de Apoyo. Caracas: UNA

PLANEACIÓN DE LA PRODUCCIÓN
La planeación, para que sea práctica y eficaz deberá tomar en cuenta y adaptarse a las
reacciones de las personas a quienes afecta. Desde la dirección de una empresa se puede
emanar una decisión que pudiera resultar óptima, pero puede resultar quizás poco conve-
niente; lo que obliga a establecer estrategias en la planeación.

Se puede decir entonces de manera general que la planeación es:

La función de la dirección de la empresa que estima por anticipado los factores de mano
de obra, materias primas, maquinaria y equipo, para realizar la fabricación que está de-
terminada por lapsos, con relación a:
• Utilidades que deseen lograr.
• Demanda del mercado.
• Capacidad y facilidades de la planta.
• Puestos laborales que se crean.

En la tabla siguiente, se describe cómo es el proceso de planeación incluyendo su al-


cance y nivel en la organización:

Tabla 16. Tipos de planificación

Tipo de
Alcance Nivel Objeto
Planificación
La expresión estratégica se usa en el sentido de
un enfoque bien mediato del plan y de su aplica-
ción, de tal manera que se tropiece con un mínimo
de dificultades por las reacciones de las personas
a quienes afecta.
Determina este plan la forma de crecimiento de
Largo plazo

Institucional

las organizaciones, señalando los productos y


Estratégica

servicios a desarrollar para el logro de sus objeti-


vos, sobre los recursos que serán utilizados y las
políticas generales que orientarán la adquisición y
la administración de tales recursos, considerando
a la empresa como una entidad total. Se puede
hacer uso de una matriz FODA para determinar
las: debilidades, oportunidades, fortalezas y ame-
nazas, para disminuir la incertidumbre generadas
por lo prolongado del tiempo empleado, 10 años
o menos.

145
Sánchez Marchán, Nanci. (2014).Introducción a la Ingeniería Industrial, Material Instruccional de Apoyo. Caracas: UNA

La Planeación Táctica considera un concepto de


toma deliberada y sistemática de decisiones que
Mediano plazo incluyen propósitos más limitados, plazos más

Intermedio
cortos, áreas menos amplias y niveles medios en

Táctica
la jerarquía.
Estos planes, por lo general son aproximadamen-
te de cinco años de duración. Fluyen naturalmente
desde los objetivos, políticas y estrategias esta-
blecidas en el plan estratégico.
Se basa en la lógica del sistema cerrado, por ello
la planeación se orienta hacia la optimización y
maximización de los resultados.
Corto plazo

Operacional

Operacional
El grado de libertad es mínimo porque se deben
obtener resultados, es por ello que la adminis-
tración debe tomar decisiones a corto plazo. Su
amplitud es sólo hacia una tarea u operación. Los
planes a corto plazo son usualmente planes a de-
sarrollarse en un año o menos y contienen deta-
lles y calendarios del tipo de presupuesto o plan
de financiamiento para su realización.
Adaptación del texto: Administración “Procesos Administrativos,” Idalberto Chiavenato.
Segunda Edición. 1998

La ejecución de la planeación implica el empleo de la planeación estratégica o de la


planeación táctica. En cualquier empresa la estrategia se emplea en el manejo de los de-
sarrollos internos de la empresa y con las fuerzas externas que aceptan el cumplimiento
exitoso de objetivos estipulados. En contraste, la planeación táctica se refiere al empleo más
efectivo de los recursos que se han aplicado para el logro de objetivos dados y específicos.
La diferencia entre ambas consiste en el elemento tiempo implicado en los diversos proce-
sos; mientras más largo es el elemento tiempo, más estratégica es la planeación. Los planes
a corto plazo u operacional son usualmente planes a desarrollarse en un año o menos y
contienen detalles y calendarios del tipo de presupuesto o plan de financiamiento para su
realización.

Una planeación será estratégica si se refiere a toda la empresa; será táctica, si


se refiere a gran parte de un producto o de su publicidad, y operacional si se de-
termina el tiempo en el que se elabora un producto determinado en una máquina
específica.

Programación de la producción

Actividad que consiste en la fijación de planes y horarios de la producción, de acuerdo


con la prioridad de la operación por realizar, determinado así su inicio y fin, para lograr el

146
Sánchez Marchán, Nanci. (2014).Introducción a la Ingeniería Industrial, Material Instruccional de Apoyo. Caracas: UNA

nivel más eficiente. La función principal de la programación de la producción consiste en lo-


grar un movimiento uniforme y rítmico de los productos a través de las etapas de producción.
Incluye la carga de los productos a los centros de producción e instrucciones pertinentes a
la operación.

Este programa se inicia con la especificación de lo que debe hacerse, en función de la


descomposición del plan agregado de la producción, el cual es traducirlo a un Plan Maestro
de la Producción (PMP). Plan detallado que establece cuántos productos finales serán pro-
ducidos y en qué períodos.

El programa de producción es afectado por:

Materiales: no cumplir con las fechas comprometidas para su entrega.

Capacidad del personal: alterar las fechas de entrega, al disminuir el número de ope-
rarios en función de bajar los costos de mano de obra.

Capacidad de producción de la maquinaria: utilización inadecuada de ésta. Utilizar


la maquinaria adecuadamente depende de las condiciones ambientales, especificaciones,
calidad y cantidad de los materiales, la experiencia y capacidad de las operaciones realiza-
das en ellas.

Sistemas de producción: no contar con un estudio preliminar, que permita seleccionar


el sistema más adecuado, acorde con las necesidades y características de la empresa.

La función de la programación de producción tiene como finalidad lo siguiente:


•• Prever las pérdidas de tiempo o las sobrecargas entre los centros de
producción.
•• Mantener ocupada la mano de obra disponible.
•• Cumplir con los plazos de entrega establecidos.

Existen diversos medios de programación de la producción, entre los que se destacan


los siguientes:
•• Gráfica de Barras. Muestra las líneas de tendencia.
•• Gráfica de Gantt. Se utiliza en la resolución de problemas relativamente
pequeños y de poca complejidad. Ver diagramas de proceso.
•• Camino Crítico. Se conoce también como teoría de redes, es un mé-
todo matemático que permite una secuencia y utilización óptima de los
recursos.
•• Pert- Cost. Es una variación del camino crítico, en la cual además de
tener como objetivo minimizar el tiempo, se desea lograr el máximo de
calidad del trabajo y la reducción al mínimo de los costos.

147
Sánchez Marchán, Nanci. (2014).Introducción a la Ingeniería Industrial, Material Instruccional de Apoyo. Caracas: UNA

Con la programación se persigue la sucesión de una serie de tareas a realizar


acotadas en el tiempo, sin embargo la planificación es la sucesión de tareas ne-
cesarias para alcanzar un objetivo.

Planear la producción trae muchas ventajas para la empresa, algunas de ellas son:
•• Se define el número de unidades a  producir en un período.
•• Se pueden calcular en forma global, las necesidades de mano de obra,
materia prima, maquinaria y equipo, sobre la base de lo producido en
períodos anteriores.
•• Se planea el cumplimiento de los pedidos para las fechas estipuladas.
•• Se pueden calcular las compras de materia prima sobre la base de lo que
se necesita para una producción estimada.
•• Se pueden estimar los recursos económicos para financiar la producción.

A continuación se presenta una guía práctica para planear la producción:

1. Se define el período para el cual se va a planear la producción.

2. Se calcula la producción requerida.

3. Conociendo la cantidad estimada de productos a vender, se define el número de


unidades por producto a producir en el período. Se ha de tener en cuenta, si se
tienen existencias de ellos, como también el número de unidades que se acos-
tumbra a tener en inventario. Para efectos de facilitar el cálculo, se elabora el
siguiente formato:

Empresa:_________________________________________
Producto: _________________________________________
Producción requerida/perío-        
dos.
Producción requerida,        
número de unidades estima-
das en ventas.
(+) Número de unidades        
en inventario al finalizar el
período.
Total unidades disponibles.        
(-) Número de unidades en        
inventario al iniciar produc-
ción.
Total unidades a producir.        

148
Sánchez Marchán, Nanci. (2014).Introducción a la Ingeniería Industrial, Material Instruccional de Apoyo. Caracas: UNA

4. Se calculan las necesidades de materia prima para llevar a cabo la producción se


utiliza la siguiente forma:

Empresa:_____________________________
Producto:_____________________ Período:_________________
Materia prima requerida:
Lista de materia Cantidad necesaria Número de unidades Cantidad total de ma-
prima por producto a producir teria prima requerida
       
       
       

5. Se efectúa el cálculo de la necesidad de mano de obra, maquinaria y equipo. Se


puede determinar, en forma global,  sobre la base de la experiencia de períodos
pasados. O se puede hacer uso de los siguientes formatos:

Empresa:_____________________________
Producto:_____________________ Período:_________________
Mano de obra requerida:
1 2 3 4 Total
Producción
Trabajadores
Tiempo extra
Costo de contratación –
despidos
Salarios en Bs
Total

Empresa:_____________________________
Producto:_____________________ Período:_________________
Maquinaria requerida:
Tipo de máqui- Tiempo disponible   Nº de Costo de utiliza-
na o equipo (en horas-máquina) máquinas ción de máquina
Producto 1
Producto 2
Producto n
Total

149
Sánchez Marchán, Nanci. (2014).Introducción a la Ingeniería Industrial, Material Instruccional de Apoyo. Caracas: UNA

Ejemplo Ilustrativo
Carteras “Carlota” va a planear la producción de la referencia del Bolso “Materno”
para los meses de octubre, noviembre y diciembre.  De acuerdo con las esti-
maciones de ventas realizadas se espera vender en julio 530 bolsos, en agosto
530 bolsos, en septiembre 1.500 bolsos. Se cree que al final de cada período se
tendrá en inventario 50 bolsos maternos. Al comenzar la producción hay en inven-
tario 90 bolsos en julio, 50 en agosto y septiembre respectivamente.
La materia prima necesaria para fabricar una unidad del producto es:

Cantidad necesaria por


Lista de materia prima
producto
Cuero 1 metro
Asas de plástico 2 unidades
Broches 2 unidades
Tela para foro 1 metro
Hilo 2 unidades
Pega 1/53 galón
Hebillas 2 unidades
Cierre 1 unidad

¿Cuál es la producción requerida?

Solución:
•• Cálculo de la producción requerida (PMP)

Con la ayuda de la siguiente tabla, Carteras “Carlota” calcule las unidades a producir en
los meses de julio, agosto y septiembre.

Empresa: Carteras “Carlota”


Producto: Bolso “Materno”
Períodos Julio Agosto Septiembre

Producción requerida ? ? ?

Número de unidades estimadas en ventas 530 530 1500


(+) número de unidades en inventario al fi-
50 50 50
nalizar el período
Total unidades disponibles 580 580 1550
(-) número de unidades en inventario al ini-
90 50 50
ciar producción
Total unidades a producir 490 430 1500

150
Sánchez Marchán, Nanci. (2014).Introducción a la Ingeniería Industrial, Material Instruccional de Apoyo. Caracas: UNA

•• Cálculo de las necesidades de materia prima

Sobre la base del consumo de materia prima en los meses anteriores, se calculan las
necesidades para la fabricación del Bolso “Materno” durante el mes de julio.

Empresa: Carteras “Carlota”


Producto: Bolso “Materno”
Período: julio
Materia prima requerida
Cantidad total de
Lista de materia Cantidad necesaria Número de unidades
materia primera
prima por producto a producir
requerida
Cuero 1 metro 490 490 metros
Asas de plástico 2 unidades 490 980 unidades
Broches 2 unidades 490 980 unidades
Tela para foro 1 metro 490 490 metros
Hilo 2 unidades 490 1470 unidades
Pega 1/53 galón 490 9 galones
Hebillas 2 unidades 490 980 unidades
Cierre 1 unidad 490 490 unidades

El cálculo de materia prima para los meses de agosto y septiembre se efectúa de la


misma manera que para el mes de julio.
•• Cálculo de las necesidades de mano de obra, maquinaria y equipo. Sobre
la base de la experiencia de períodos anteriores, se considera que con la
maquinaria y equipo que se tiene y con 4 operarios se puede cumplir con la
producción estimada.

Se muestra seguidamente una figura que representa una caracterización, de los ele-
mentos involucrados en la planeación de la producción, estudiados anteriormente.

LA PLANEACIÓN ES PERIODO DE LA PLANEACIÓN:


AFECTADA POR: -Planeación corto plazo (de sema-
-Materiales nas a meses)
-Capacidad de personal -Planeación mediano plazo (de 6 a
-Sistema de producción 18 meses)
-Capacidad de la maquinaria -Planeación largo plazo (años)

PLANEACIÓN DE LA
SE PUEDE PLANIFICAR PRODUCCIÓN
CALCULANDO:
-Periodo de la producción
-Producción requerida
-Necesidades de materia prima
-Necesidades de mano de obra,
materiales y equipo

Figura 30. Elementos de la planeación de la producción.

151
Sánchez Marchán, Nanci. (2014).Introducción a la Ingeniería Industrial, Material Instruccional de Apoyo. Caracas: UNA

Es importante resaltar, que de acuerdo con Domínguez Machuca (1995), estas fases
se deberán llevar a cabo en cualquier empresa independientemente de su tamaño y ac-
tividad, aunque la forma en como éstas se desarrollen dependerá de las características
propias de cada sistema productivo.

En la tabla 17 se puede observar un resumen de las principales fases de un sistema


de planificación, junto con los planes que de él se derivan. Relacionando por un lado,
los niveles de planificación empresarial y por otro lado, la planificación y gestión de la
capacidad.

Tabla 17. Resumen de las fases de un sistema de planificación

FASES DE UN SISTEMA DE PLANIFICACIÓN


Período
Fase Finalidad Descripción
(Mínimo-máximo)
Plan estratégico de Definir nuevos productos 1 -10 años Ampliar las familias de
producción. o mercados. (largo plazo) productos y mercados.
Número importante de
opciones en cuanto a
Plan estratégico de Modificaciones de la tec- 1-7 años
tecnología y capacidad,
procesos. nología o capacidad. (largo plazo)
para alcanzar la produc-
ción.
Plan táctico de Coordinar proyectos de 1-5 años
Proyectos/actividades
inversiones. inversión. (mediano plazo)
Opciones de uso de
Plan maestro de Coordinar el uso de recur- 1–18 meses
recursos sobre la base
producción. sos. (corto plazo)
del inventario.
Opciones de producción
Plan detallado de Coordinar producción, 1-8 meses
sobre la base de ventas
producción. ventas e inventario. (corto plazo)
e inventario.
Asignación de operacio- Secuencia de operacio-
1–6 semanas
Programa de producción. nes en función de perío- nes para producir unida-
(corto plazo)
dos y lugares. des, piezas, elementos.

Actividad propuesta

Diseñar un plan de producción para los meses de enero a junio en una fábrica
de pantalones. Las ventas estimadas, inventarios, horas laboradas y número de
trabajadores se muestran en la tabla adjunta.

Ene Feb Mar Abr May Jun Jul Ago Sep Oct Nov Dic

Ventas 800 650 900 700 1050 1400 1600 1550 1350 1250 700 900
Inventario
300 200 300 100 400 400 400 300 200 200 100 100
inicial
Inventario
70 80 50 60 70 60 70 80 60 80 70 70
final
Horas
108 108 144 144 144 144 144 180 180 180 144 72
laboradas
Nº de
6 6 7 7 7 7 7 8 8 8 7 3
trabajadores

152
Sánchez Marchán, Nanci. (2014).Introducción a la Ingeniería Industrial, Material Instruccional de Apoyo. Caracas: UNA

En cada día de producción se pueden fabricar hasta 100 unidades, producción


que puede incrementarse en un 16% vía horas extras, con un costo adicional de 0,4
Bs/unidad. Un trabajador tiene un sueldo promedio de 2.000 Bs/ mes.

Las necesidades de materia prima para una unidad de producto son:

Lista de materia prima Cantidad necesaria por


producto
Tela 3 metros
Tela para forro 1 metro
Cierre 1 unidad
Cinturilla 1 metro
Hilo 2 unidades
Broche 1 unidad
botones 2 unidades
Etiqueta 1 unidad

En conclusión la función de planeación de la producción puede ser muy compleja. Va-


rios factores afectan esta complejidad, incluyendo el número de productos, el patrón de de-
manda y la incertidumbre, el número de períodos en el horizonte, los procesos alternativos
para hacer el producto, la subcontratación, el tiempo extra y el inventario, entre otros.

El inventario tiene una gran relevancia, para muchas empresas en el mundo, en este
sentido, las necesidades de producción son traducidas por los gerentes a través del pre-
supuesto de ventas y las políticas relativas a los inventarios. Los inventarios son un puente
de unión entre la producción y las ventas que equilibra la línea de producción. Esta acción
tiende a facilitar el flujo constante de la producción y su programación. Seguidamente se
contempla los aspectos más resaltantes de éste tópico.

INVENTARIO
El inventario significa para la producción una reserva de materia prima, productos en
proceso y productos terminados, que puede actuar como un colchón para la empresa en
momentos de contingencia. Para muchos los inventarios presentan una doble faz, es decir,
pueden ser buenos o malos.

Se consideran buenos cuando puede mejorar el servicio al cliente mediante un embar-


que rápido de sus pedidos, así como también la posibilidad de reducir algunos costos tales
como: costos por pedidos a proveedores, costos por faltantes, costos por adquisición, costos
de calidad por arranque.

Se consideran malos por el hecho de que al elevar sus niveles, muchos costos au-
mentan tales como: costos de almacenar, costos de coordinar la producción, costos de un
rendimiento hacia la inversión, costos por reducción en la capacidad, entre otros.

153
Sánchez Marchán, Nanci. (2014).Introducción a la Ingeniería Industrial, Material Instruccional de Apoyo. Caracas: UNA

Introducción al Inventario
A través de la historia de los negocios, en una empresa se ocultaba la imprecisión de
la información respecto a la producción y sus requerimientos, así como, su ignorancia del
mercado, manteniendo inventario adicional. Los japoneses fueron los primeros en hacer más
eficiente este proceso en la industria manufacturera, generando una técnica que denomina-
ron Sistema KANBAN.

El sistema KANBAN es un simple papel o tarjeta con un índice de artículos, cuando la


línea de producción nota una baja de inventario, se anexa este papel a una red de hilo soli-
citando más partes, es decir, cada operación hala el material que necesita de la operación
anterior.

KANBAN significa tarjeta en japonés, y es la señal que autoriza mover, producir o com-
prar. Para ello se requiere realizar dos actividades:
•• Empujar (PUSH), llevar a cabo una acción con anticipación a una nece-
sidad.
•• Jalar (PULL), llevar a cabo una acción cuando se necesite.

PROGRAMACIÓN CON KANBAN


Se procede de la siguiente forma:
•• Se recibe un pedido en la bodega.
•• Si es la última caja en la estantería se retira la tarjeta y se lleva al depar-
tamento de compras.
•• Se coloca la tarjeta en la pizarra de programación sobre la tarjeta madre.
•• Si coincide con la cantidad a comprar, se gira una nueva orden al pro-
veedor.
•• Se procesa el producto.
•• Se coloca una tarjeta en cada una de los estrados que existen.
•• Se traslada a la bodega del cliente.

Se pueden generar dos tipos de Kanban:


•• De transporte o movimiento. Se mueve entre dos puestos de trabajo, indi-
cando la cantidad de producto a retirar del proceso anterior. Debe figurar
toda la información que facilite la localización y transporte de los ítems
necesarios entre puestos de trabajo.
•• De producción. Se mueve dentro de un puesto de trabajo y funciona como
orden de fabricación. Es preciso que contenga toda la información reque-
rida para facilitar la fabricación de piezas a las que se hacen referencia.

154
Sánchez Marchán, Nanci. (2014).Introducción a la Ingeniería Industrial, Material Instruccional de Apoyo. Caracas: UNA

Hoy día esta metodología es utilizada en una red gigantesca de proveedores que utilizan
la conexión del hilo, es decir, halar lo que se necesita en el momento preciso. Es un sistema
utilizado en diferentes líneas de producción, como por ejemplo en la red de telefonía privada
o pública. Actualmente se evita mantener mayor inventario del necesario, ya que la informa-
ción en tiempo real suple el inventario.

Los beneficios del sistema KANBAN son mayores que la inversión requerida y los
conceptos son aplicables donde quiera y no requiere uso de equipo sofisticado.

Como se mencionó anteriormente, la tendencia sigue siendo la de sustituir el inventario


con información. El tipo de mercado denominado BS2C (Business to Consumer) fue el pri-
mero que demostró un acortamiento en la cadena de proveedores y a su vez un acercamien-
to con el cliente final, CRM (Customer Relationchips Management System). Los primeros
indicios de este mercado no ocurrieron directamente en internet, sino con la aparición de
kioscos donde era posible personalizar una tarjeta y obtenerla en cuestión de minutos, no
existía inventario innecesario. El siguiente paso fue la producción de un CD con la música
deseada.

Otro ejemplo donde se pueden observar los procesos más avanzados de este tipo de
producción es el implementado por Levis, el cual ha logrado reducir la cadena de distribución
de sus clientes, enviándolos directamente a la línea de producción; por medio de servidores
de páginas, bases de datos, etc. Su habilidad para realizar transacciones financieras, le ha
permitido llevar su producto final con las especificaciones que el cliente le indica al instante.

Conforme la importancia de la información continua, seguirán surgiendo compañías


donde su principal recurso será la información. Algunas compañías muy exitosas no poseen
ningún tipo de activo, tal es el caso de Visa Internacional, la cual procesa transacciones por
billones de dólares al año, sin embargo visa como tal no existe, es una organización com-
puesta por alianzas de bancos y otras instituciones financieras, cuyo fundador es Dee Hock.

En muchas empresas la administración del inventario de bienes físicos se basa en las


determinaciones intuitivas del gerente de compras, quien decide qué artículos comprar,
cuándo comprarlos y en qué cantidades. Generalmente esta situación se presenta cuando la
empresa es pequeña y el número de artículos a mantener en inventario es reducido, lo cual
permite que estos procedimientos informales puedan funcionar adecuadamente.

Sin embargo, conforme la empresa crece y comienza a requerir una mayor variedad de
inventarios con distintas proporciones de utilización, los sistemas informales tienden a crear
problemas que puedan resultar en mayores costos y en una interrupción de la producción
y el suministro del producto terminado. En cambio un sistema formal de administración de
inventarios puede producir ahorros sustanciales para una compañía en cuanto a un menor
costo de compra, menores gastos de intereses, un aumento en la disponibilidad de fondos
internos, menores costos de operación, menor costo unitario de producción, confiabilidad en
la entrega por parte de la producción y un mejor servicio a los clientes en el suministro de
productos.

155
Sánchez Marchán, Nanci. (2014).Introducción a la Ingeniería Industrial, Material Instruccional de Apoyo. Caracas: UNA

En muchos países las empresas siguen manteniendo stocks de reserva o inventa-


rios. Esta previsión resulta especialmente importante cuando un producto tiene una de-
manda fuertemente estacional o cuando la demanda ha de servirse en un período temporal
relativamente corto, tal es el caso de nuestro país, donde la mayoría de las empresas se ven
en la necesidad de mantener inventarios, sobre todo de materias primas por lo costoso de
sus importaciones; en este orden de ideas se puede definir el inventario como:

El conjunto de actividades y técnicas utilizadas para mantener la cantidad de artículos


(materiales, materias primas, producto en proceso y producto terminado) en el nivel de-
seado tal que ni el costo ni la probabilidad de faltante sean de una magnitud significativa.

Es decir, el inventario es el material o los suministros que se tiene para el uso o la venta
futura. En general, se trata de bienes terminados que esperan el pedido de un cliente o bie-
nes materiales destinados a la producción o transformación.

Existen cuatro tipos de stocks según la función que éstos desempeñen y son:

Stocks de ciclo: muchas veces no tiene sentido producir o comprar materiales


al mismo ritmo en que son solicitados, ya que resulta más económico lanzar una
orden de compra o de producción de volumen superior a las necesidades del
momento, lo que dará lugar a este tipo de stocks.

Stocks estacionales: algunos productos presentan una demanda muy variable


a lo largo del año, aumentando mucho en determinados meses y disminuyendo
en otros (juguetes, uniformes y útiles escolares, etc.). Así, es lógico que la pro-
ducción sea mayor que la demanda en determinados períodos, por lo que se
generará un stock de carácter estacional.

Stocks de seguridad: suponen una garantía frente a posibles aumentos repen-


tinos de la demanda.

Stocks de tránsito: su función es actuar como reserva, a fin de mantener el flujo


continúo de materiales entre las distintas fases del proceso productivo.

Básicamente, estos stocks pretenderán dar respuesta a las preguntas que normalmen-
te se plantean en la actividad generada por los inventarios:

(1) ¿Cuándo lanzar una orden de producción o de compra?

(2) ¿Cuál debe ser el tamaño óptimo de dicho pedido?

156
Sánchez Marchán, Nanci. (2014).Introducción a la Ingeniería Industrial, Material Instruccional de Apoyo. Caracas: UNA

OBJETIVOS DEL INVENTARIO

Entre los principales objetivos del inventario se encuentran:


•• Disminuir el costo de compra.
•• Reducir los gastos de intereses.
•• Aumentar la disponibilidad de fondos internos.
•• Lograr menores costos de operación.
•• Disminuir el costo unitario de producción.
•• Aumentar la confiabilidad en la entrega por parte de la producción.
•• Mejorar el servicio a los clientes en el suministro de productos.

Mejorar la administración del inventario es un objetivo clave en el impulso que


realiza cada empresa para controlar la inversión, optimizar el flujo de efectivo y
aumentar las utilidades y el rendimiento sobre la inversión.

CLASIFICACIÓN DEL INVENTARIO


Se puede mencionar el inventario de:
•• Materia primas: la empresa adquiere las materias primas sin procesar,
las cuales necesitan una mayor elaboración o transformación para con-
vertirlas en parte de un producto final (petróleo crudo, madera, sustancia
químicas, etc.)
•• Trabajo en proceso: incluye todos los materiales de producción sobre
los que la empresa ha realizado operaciones de fabricación, procesa-
miento o transformación, pero que no están en la forma terminada. Están
en proceso de transformación y el inventario es la protección contra la
variabilidad en el proceso de producción, como por ejemplo los compo-
nentes que se almacenan para usarse en el ensamble final de algunos
productos.
•• Productos terminados: cubre todos los productos o bienes terminados,
producidos y almacenados, que esperan venderse o enviarse a los clien-
tes finales.
•• Piezas de servicio: las partes que suelen llamarse piezas de repuesto,
se usan para mantener el producto o el equipo que la empresa vende o
atiende en óptimas condiciones. Este inventario puede almacenarse en
el sitio de producción de las piezas terminadas o distribuirse en los sitios
que están estrechamente ligados a la reparación o al mantenimiento del
producto final.
•• Distribución: los productos terminados, piezas de repuesto, entre otros,
que están ubicados en depósitos que se encuentran en toda la red de
distribución (sucursales, bodegas, entre otros) cerca de la planta princi-
pal.

157
Sánchez Marchán, Nanci. (2014).Introducción a la Ingeniería Industrial, Material Instruccional de Apoyo. Caracas: UNA

•• Suministros: los artículos que se usan para mantener las operaciones,


ya sea en la fábrica o en la oficina, pero que no llegan a formar parte del
producto terminado. Son artículos para el mantenimiento de la planta, la
reparación de máquinas, el consumo de la planta o la producción, artícu-
los de oficina, etc.

EL COSTO DEL INVENTARIO


Cuando se considera el costo de inventario, se debe mirar más allá de lo obvio. El in-
ventario siempre conlleva un costo indirecto al cual se le llama costo de mantenimiento o
adicional y, por tanto, es considerado un desperdicio; éste generalmente suele distribuirse
entre el costo de compra y el costo total de material. El mejoramiento de la administración
del inventario debería concentrarse en reducir los costos indirectos que se generan en este
proceso.

Son ocho los costos indirectos principales que se asocian con el inventario.
•• De adquisición.
•• Por la inspección.
•• De almacenaje.
•• Por el manejo de materiales.
•• Por el interés de transacciones.
•• Los generados por la obsolescencia.
•• Por la depreciación.
•• Los derivados por el importe de seguros.

VALORIZACIÓN DEL INVENTARIO


Para la valorización de los inventarios de mercancías se utilizan los métodos PEPS,
UEPS y Promedio Ponderado:
•• El Método PEPS (Primeras en Entrar, Primeras en Salir), basado en
que las primeras unidades en entrar al almacén son las primeras en ven-
derse y por lo tanto, se deberá tener el precio más antiguo de ingreso
como base para su valorización.
•• El Método UEPS (Últimas en Entrar, Primeras en Salir) propone la
valuación de acuerdo con los últimos precios con que ingresan las mer-
cancías. Las últimas unidades en entrar al almacén son las primeras en
venderse
•• El método de Promedio Ponderado requiere conocer el total de las
unidades de inventario inicial y de las unidades adquiridas durante el pe-
ríodo, así como el total en pesos de las mismas unidades de inventario.
El promedio ponderado es el producto de dividir el total en pesos por el
total en unidades con el objetivo de estimar el valor final de los productos
no vendidos

158
Sánchez Marchán, Nanci. (2014).Introducción a la Ingeniería Industrial, Material Instruccional de Apoyo. Caracas: UNA

MODELOS DE INVENTARIOS
Todos los sistemas de inventario revisan continuamente sus prácticas de pedir y modifi-
car stocks según se requiera en cuanto a sus cantidades de pedido, sus puntos de pedido y
sus intervalos de tiempo, para obtener el tipo de desempeño que se desea de los inventarios

El manejo de los inventarios es sin lugar a dudas un elemento crítico, para el buen de-
sarrollo de la empresa, si éste no se efectúa correctamente la posibilidad de tener problemas
de abastecimiento o mayores costos es muy alta, es por esto que permanentemente se
deben estar revisando los normas para su manejo dentro de la empresa y considerar una
realidad donde lo único constante es el cambio que puede ocasionar una alta posibilidad de
dejar de ser competitivo y salir del mercado.

Los modelos de inventario son modelos matemáticos que básicamente tienen dos divi-
siones y depende de un procedimiento (basado en supuestos) respecto al comportamiento
de la demanda, estos modelos son: determinísticos y probabilísticos. En este sentido se
puede hablar del modelo más sencillo y fundamental de inventarios EOQ (Economic Order
Quantity) que consiste en determinar la cantidad óptima a ordenar que minimice los costos
incrementales totales, relacionados con mantener un inventario y procesar órdenes.

Del modelo básico de inventario se derivan una serie de variantes, los cuales pueden
ser útiles a la hora de tomar decisiones importantes sobre inventarios, debido a que se ajus-
tan a diferentes situaciones. A continuación se ubican dentro de las divisiones señaladas
anteriormente (determinísticos o probabilísticos) cada uno de estos modelos:

Modelos determinísticos:
1. EOQ original con stock de seguridad.

2. EOQ con descuentos por cantidad.

3. EOQ con faltantes por surtir.

Modelos probabilísticos:

1. Modelos de revisión continua:

•• Modelo EOQ Probabilizado.


•• Modelo EOQ Probabilístico.

Se observa a continuación una representación gráfica donde se plasma la distribución


del modelo fundamental (EOQ) de Inventario y sus variantes:

159
Sánchez Marchán, Nanci. (2014).Introducción a la Ingeniería Industrial, Material Instruccional de Apoyo. Caracas: UNA

MODELOS DE INVENTARIO

Modelo básico de
Cantidad de Lote
Económico (EOQ)

Modelos
probabilísticos (EOQ)

Modelos de revisión
continúa

Modelos Modelo EOQ Modelo EOQ


determinísticos probabilizado probabilístico
(EOQ) (Probado)

Modelo EOQ Modelo EOQ con Modelo EOQ con


original con Stock descuentos por faltantes por
de Seguridad cantidad surtir

Figura 31. Distribución de los modelos de inventario.

En este texto sólo se desarrollará el modelo básico de inventario (EOQ) el cual se des-
cribe a continuación. El estudio de los modelos derivados del modelo EOQ, se profundizará
con el devenir del estudio de la carrera.

MODELO BÁSICO (EOQ)


El modelo básico de inventario es una técnica cuya política óptima es la de ordenar la
misma cantidad de productos cada cierto período. Describe la importancia entre los costes
de pedir y los de mantenimiento hasta alcanzar el equilibrio entre ambos, es un modelo de-
terminista, lo que induce a considerar que los parámetros se conocen con certeza. La unidad
de tiempo es el año, pero el análisis también es válido para cualquier otra unidad de tiempo
(día, semana, mes, etc.).

160
Sánchez Marchán, Nanci. (2014).Introducción a la Ingeniería Industrial, Material Instruccional de Apoyo. Caracas: UNA

Los supuestos sobre los que este modelo se construye son:


1. Demanda constante y conocida.
2. Los productos tienen duración prolongada.
3. Se utiliza un sistema de monitoreo continuo.
4. El costo fijo de emitir una orden es constante.
5. El tiempo de espera (L) es conocido y constante.

Y además en el modelo se consideran los siguientes parámetros:


D: demanda, unidades por año.
Q: cantidad a ordenar.
Co: costo de emitir una orden.
Ch: costo asociado a mantener una unidad en inventario en un año.
CT: costo total de inventario.

Considerando los parámetros antes mencionados el costo anual de mantener unidades


en inventario es Ch = Co(D/Q) y el costo de emitir órdenes para el mismo período es Co =
Ch(Q/2). Luego la función de costo total (anual) asociado al manejo de inventarios es CT (Q)
= Co (D/Q) +Ch (Q/2). Derivando esta función respecto a Q e igualándola a cero se obtiene
la fórmula para el modelo EOQ que determina la cantidad óptima de pedido:

Q* =

El comportamiento de este modelo se aprecia fácilmente en la figura 32:

Nivel de
Inventarios
Q

tiempo
L

Figura 32. Comportamiento del modelo básico de inventario.

161
Sánchez Marchán, Nanci. (2014).Introducción a la Ingeniería Industrial, Material Instruccional de Apoyo. Caracas: UNA

La altura del triángulo representa la cantidad máxima de producto pedido (Q); el núme-
ro de triángulos es el número de pedidos en el período; la base del triángulo corresponde al
tiempo entre pedido (T); (R) representa el punto de reorden o donde el pedido es óptimo (Q*);
(L) es el tiempo de reorden y (t) es el período de planeación. También se deben conocer los
siguientes datos:
•• Demanda, normalmente se trabaja anual, aunque el modelo permite
otros manejos, se calcula a partir de los presupuestos de la empresa.
•• Costo de pedido, éste se genera cada vez que la compañía efectúa una
compra, en su cálculo debe involucrarse desde el tiempo que se toma
para efectuar el pedido, hasta los gastos de transporte y recepción de
la mercancía, sin olvidar incluir los gastos administrativos pertinentes al
pago de la factura.
•• Costo de mantenimiento (conservación), éste indica cuánto vale tener la
unidad de inventario en bodega, debe tenerse en cuenta desde el costo
del dinero, hasta los seguros en caso de tenerlos, el de la bodega y del
personal que maneja los inventarios, este costo se debe dar en la misma
unidad de tiempo en que se estima la demanda.

La parte compleja del modelo es precisamente la definición de los costos anteriores, lo


importante es minimizar la suma de los costos de pedir y de mantener, lo que se conoce con
el nombre de costo asociado.

Estructura de costos
1. Cu: Costo del artículo (Bs/und.)
2. Cc o Ch: Costos de mantener inventarios (Bs/und./año)
3. Co: Costo de ordenar un lote (Bs/pedido)
4. Cb: Costos de inexistencias o pedidos no atendidos ( Bs/und./año)

Costo
Anual

CT (anual)

CH=co/2 Q

CTmin

CO=(Ch.D)/2 Q

Q*
2 x D x Co
Q* =
Ch

Figura 33. Estructura de costos del modelo básico de inventario.

162
Sánchez Marchán, Nanci. (2014).Introducción a la Ingeniería Industrial, Material Instruccional de Apoyo. Caracas: UNA

SÍMBOLOS Y FÓRMULAS DEL MODELO BÁSICO (EOQ) Y SUS VARIANTES

Tabla 18. Modelo básico de inventario (EOQ): símbolos, fórmulas y significado.

Símbolo Fórmulas Significado

Costo de mantener inventario o costo de


Cc =Ch almacenamiento (Bs/und./año) (bolívares
Ch = Co x por pedido).

Costo de ordenar un lote o de emitir una


Co = S
orden para el mismo período (Bs/pedido).
Co = Ch x
Cantidad óptima de pedido (unidades por
Q* pedido). El costo total del almacén y el
Q* = costo de ordenar son iguales.

Número de pedidos.

Tc 1 Tiempo entre pedidos.


Tc = x N° días hábiles del pedido
N
Costo asociado a la política de inventa-
CA
rios.
Costo total, involucra el valor de los artí-
CT culos y el costo asociado. (bolívares por
año).

R R= DL Punto de reorden.

Demanda anual de un material (unidades


D
por año).

Q Cantidad a ordenar.

L Tiempo de espera.
Costos de inexistencias o pedidos no
Cb
atendidos (Bs/und./año).

Ejemplo Ilustrativo Nº 1
La demanda de un artículo particular es 18.000 unidades/año. El costo de alma-
cenamiento por unidad es de Bs 1,20 por año y el costo de ordenar una compra
es de Bs 400, el tiempo de anticipación (L) es de 20 días, el costo de una unidad
es de Bs 1. (Se supone 1 año = 365 días). Determinar la cantidad óptima de pe-
dido y la demanda diaria de pedido.

163
Sánchez Marchán, Nanci. (2014).Introducción a la Ingeniería Industrial, Material Instruccional de Apoyo. Caracas: UNA

Solución:

Para determinar la cantidad a pedir se aplica la siguiente fórmula:

2 * D * CO 2(400)(18.000)
Q*    3.464 Unidades
Ch 1.2

La demanda diaria se obtiene dividiendo la demanda anual, es decir, 18.000 unidades


al año entre 365 días que tiene el año, entonces:

Demanda/día = 18.000/ 365= 49 Unidades/día

El valor de 3.464 unidades indica que es procedente hacer un pedido de 3.465 unidades
para mantener el stock.

Ejemplo Ilustrativo Nº 2
Una empresa tiene una demanda anual de 1.000 unidades de su principal pro-
ducto. El costo de emitir una orden es de 10 Bs y se ha estimado que el costo de
almacenamiento unitario del producto durante un año es de 2,5 Bs. Asuma que
el tiempo de espera (L) desde que se emite una orden hasta que se recibe es de
siete días. Determine la cantidad óptima de pedido utilizando EOQ que minimiza
los costos totales. ¿Cuál es el punto de reorden (R)?

Respuesta:

Nivel de
Inventario
7u
Q* = = = 89,4= 90
90 u
unids

D (diaria )= unids/ día

R = DL= 2,74 unidades/día x 7 días =19,18 20 u

= 20 unidades

Existe Existe
Orden Orden

El tamaño óptimo de pedido (Q*) que minimiza los costos totales es de 90 unidades.
Adicionalmente, cada vez que el inventario llega a 20 unidades se emite un nuevo pedido por
90 unidades.

164
Sánchez Marchán, Nanci. (2014).Introducción a la Ingeniería Industrial, Material Instruccional de Apoyo. Caracas: UNA

DESCRIPCIÓN Y FÓRMULAS DE LOS MODELOS DERIVADOS DEL MODELO


FUNDAMENTAL O BÁSICO (EOQ)
A continuación se muestra una tabla donde se puede apreciar una breve descripción
de los modelos derivados del modelo EOQ, además de las fórmulas que se utilizan en su
aplicación. Le será de utilidad para próximos cursos de la carreara.

Tabla 19. Descripción y fórmulas utilizadas en los modelos derivados del modelo EOQ.

Modelos Determinísticos
Son aquellos que se usan cuando se pronostican con precisión la demanda en períodos
futuros, se describe esta demanda como una variable aleatoria en vez de una constante
para cualquier período futuro.
Modelo Descripción Fórmulas a usar
Modelo EOQ Los artículos son producidos interna-
con stock de mente en lugar de ser adquiridos a un 2 Co D
seguridad proveedor externo, la demanda es de- Q* = Ch
terminística y ocurre a tasa constante.
El modelo presume que los productos CoD Ch
son fabricados a una tasa p constan- CT= Q + 2 Q+Ch(K)
te de unidades por unidad de tiempo
(normalmente al año), luego durante
un intervalo de longitud (t) se produ-
cen exactamente (pt) unidades.

Modelo Existen descuentos según la orden, el CoD Ch


CT= + Q+D Cu
EOQ con costo anual de compra o de produc- Q 2
Descuentos ción depende del volumen deman- Co D Ch  Cb  2
por Cantidad dado. Adicionalmente si el costo de CT= +  (Q-S)2 +  S
Q 2Q 2Q
mantener unidades en inventario se
Ch
expresa como un porcentaje del pre- S*=  Q*
Ch+Cb
cio de compra, el costo anual de man-
tener ordenes en inventario también 2CoD(Ch+Cb)
Q*=
dependerá del precio de compra. ChCb

Modelo EOQ En muchas situaciones reales la de- Co D Ch  Cb  2


CT= +  (Q-S)2 +  S
con faltantes manda no puede ser satisfecha a Q 2Q 2Q
por surtir tiempo, en cuyo caso ocurre escasez. 2CoD(Ch+Cb)
Cuando ocurre escasez se incurre en Q*=
ChCb
costos adicionales por pérdida de ne-
gocios, órdenes especiales, etc. En di- Ch
S*=  Q*
chas situaciones es preciso modificar Ch+Cb
el modelo EOQ usual. Q-S S
T1= T2=
D D

165
Sánchez Marchán, Nanci. (2014).Introducción a la Ingeniería Industrial, Material Instruccional de Apoyo. Caracas: UNA

Modelos Probabilísticos
Se clasifican en general bajo situaciones de análisis continuo y periódico. Los modelos de
análisis periódico incluyen casos de un solo período y de períodos múltiples.
Modelo Definición Fórmulas a usar
La dimensión de las exis- Se ha de tener en cuenta que:
tencias estabilizadoras se P (z>=Kα) = α y B>= σL. Kα
Modelos EOQ de Revisión Probabilizada

determina de modo que la


La primordial suposición del modelo es que la
probabilidad de agotamiento
demanda, XL, durante el tiempo de entrega L se
de las existencias durante el
distribuye normalmente con media μL y desviación
tiempo de entrega (el perio-
standard σL, es decir, N (μL, σL).
do entre colocar y recibir un
pedido) no exceda un valor Sean:
predeterminado. L = tiempo de entrega entre colocar y recibir un
pedido.
μL = demanda promedio durante el tiempo de entre-
ga.
σL = desviación standard de la demanda durante el
tiempo de entrega.
B = tamaño de la existencia estabilizadora.
α = máxima probabilidad disponible de agotamiento
de las existencias durante el tiempo de entrega.
XL = variable aleatoria que representa la demanda
durante el tiempo de entrega.
Este modelo permite faltantes Función de costo total:
en la demanda, la política
F (x) = fdp de la demanda x, durante el tiempo de
requiere ordenar la cantidad
entrega.
siempre que el inventario
caiga al nivel R (Reorden). D = demanda esperada por unidad de tiempo.
Los valores óptimos de Y y R, h = costo de manejo por unidad de inventario por
se determinan minimizando el unidad de tiempo.
costo esperado por unidad de
Modelos EOQ Probabilístico

p = costo de faltante por unidad de inventario.


tiempo que incluye la suma
de los costos de preparación, K = costo de preparación por pedido
conservación y faltante. 1. El costo de preparación por unidad de tiempo es
KD/y.
El modelo tiene 3 suposicio-
nes: 2. En el costo de manejo, el inventario promedio es:
I = y/2 + R – E(x).
la demanda no satisfecha
durante el tiempo de entrega 3. costo de faltante esperado es:
se acumula. S = x(x-R) f(x) dx
no se permite más de una La solución para obtener y* y R* óptimas se determi-
orden pendiente. na por: Y* = 2D (K+pE(x)
La integral de R* hasta α en función de (x) = hy*/pD
3. La distribución de la de-
manda durante el tiempo de Como Y* y R*, no se pueden determinar de forma
entrega permanece esta- cerrada se usa un algoritmo numérico desarrollado
cionaria (sin cambio) con el por Hadley y Whitin, para encontrar las soluciones.
tiempo.

Los símbolos que se muestran en las fórmulas de los modelos determinísticos es-
tán descritos en la tabla 18. Los símbolos de los modelos probabilísticos en su
mayoría se encuentran descritos en esta tabla.

166
Sánchez Marchán, Nanci. (2014).Introducción a la Ingeniería Industrial, Material Instruccional de Apoyo. Caracas: UNA

Los modelos de inventarios permiten utilizar métodos cuantitativos al momento de tomar


decisiones. Una cantidad importante del capital invertido en las empresas, se encuentra en
los inventarios de los que ésta dispone.

Los aspectos considerados en el tema de inventarios se pueden englobar en la siguien-


te figura:

Administración de Control de
inventarios inventario
Aumentar la rentabilidad Se refiere a la parte ope-
del producto a través de racional del inventario a
una correcta utilización todos aquellos aspectos
del inventario en cuanto al momento de almace-
a niveles de stock y mini- nar el producto
mización del costo

MANEJO
DE Nivel de stock Almacenamiento
INVENTARIOS
control
Costos de
y registro
mantener
inventarios

Figura 34. Manejo de inventarios.

Un correcto manejo del inventario, permite generar políticas óptimas de reabastecimien-


to que logren disminuir los costos de su mantenimiento.

Conocidas las diferentes fases de la producción, por las cuales debe transitar un geren-
te de operaciones en una empresa, es necesario considerar ahora la fase de control de la
producción o examen a la producción.

CONTROL DE PRODUCCIÓN
El control de producción consiste en examinar todo el proceso relacionado con la pro-
ducción en una empresa, desde la requisición de materia prima hasta la facturación y envío
de los productos. Se basa en información vital que es utilizada por el almacén, distribución y
finanzas. Por ejemplo, la integración de la producción con el departamento de contabilidad
asegura que el trabajo en proceso sea actualizado continuamente al registrar los materiales,

167
Sánchez Marchán, Nanci. (2014).Introducción a la Ingeniería Industrial, Material Instruccional de Apoyo. Caracas: UNA

mano de obra y recursos a su respectiva orden de trabajo, lo cual asegura que esta informa-
ción será reflejada automáticamente en los estados financieros parciales o de fin de mes.

El Control de producción se puede definir como:

“La función de dirigir o regular el movimiento metódico de los materiales por  todo el
ciclo de fabricación, desde la requisición de materias primas, hasta la entrega o venta
del producto terminado, mediante la transmisión sistemática de instrucciones a los su-
bordinados, según el plan que se utiliza en las instalaciones del modo más económico”.
Diccionario de términos para el control de la producción y el inventario.

El control se refiere a la verificación para que se cumpla con lo planeado, redu-


ciendo a un mínimo las deficiencias del plan original, por los resultados y práctica
obtenidos.

OBJETIVO Y FUNCIONES DEL CONTROL DE PRODUCCIÓN EN PLANTA


El objetivo del control de producción en planta es asegurar, mediante procedimientos
de control interno de la producción, que el producto fabricado y puesto a la venta en el mer-
cado está conforme con las especificaciones de la norma correspondiente y cumple con los
valores señalados.

Las funciones del control de la producción son:


•• Pronosticar la demanda del producto, indicando la cantidad en función
del tiempo.
•• Comprobar la demanda real con la planificada y corregir los planes si
fuere necesario.
•• Establecer volúmenes económicos de partidas de los artículos que se
han de comprar o fabricar.
•• Determinar las necesidades de producción y los niveles de existencias
en determinados puntos de la dimensión del tiempo.
•• Comprobar los niveles de existencias, comparándolas con los que se han
previsto y revisar los planes de producción si fuere necesario.
•• Elaborar programas detallados de producción y planear la distribución de
productos.

TÉCNICAS MÁS UTILIZADAS EN EL CONTROL DE LA PRODUCCIÓN


Son diferentes las técnicas a utilizar en el control de la producción para lograr una revi-
sión interna y continua efectuada por el fabricante, con la finalidad de obtener conformidad
con las especificaciones de cada producto. Entre ellas se pueden citar las siguientes. Ver
figura 35.

168
Sánchez Marchán, Nanci. (2014).Introducción a la Ingeniería Industrial, Material Instruccional de Apoyo. Caracas: UNA

TÉCNICAS DE CONTROL DE LA PRODUCCIÓN

Sistema de información
Contabilidad, evaluación financiera, presupuestos,
reportes, informes, formatos.

Gráficos y diagrámas
Procesos, procedimientos.
Diagramas: Gantt, del operario
gráficos de control

Estudio de métodos
Tiempos y movimientos, estándares.

Métodos cuantitativos
Redes: camino critico, PERT.
Investigación de operaciones.
Estadistica.

Control interno, Softwares.

Figura 35. Técnicas de control de la producción.

FACTORES NECESARIOS PARA LOGRAR QUE EL CONTROL DE PRODUC-


CIÓN TENGA ÉXITO
Se puede hablar de tres tipos de factores:

1) Los factores creativos: son ajustados a la ingeniería de diseño y permiten confi-


gurar los procesos de producción.

2) Los factores directivos: se centran en la gestión del proceso productivo y preten-


den garantizar el buen funcionamiento del sistema.

3) Los factores elementales: son las entradas al sistema productivo (inputs), ne-
cesarios para obtener el producto (output). Estos son los materiales, energía,
capital, trabajo, entre otros. Ver figura 36.

169
Sánchez Marchán, Nanci. (2014).Introducción a la Ingeniería Industrial, Material Instruccional de Apoyo. Caracas: UNA

Factores de Producción

Factores Creativos: Factores Factores Directivos:


• Investigación y Elementales • Planificación
Desarrollo. • Dirección
• Configuración • Control
de Productos y
Procesos.

Materiales Energía Capital Trabajo Información Energía

Figura 36. Factores necesarios para lograr el control de la producción.

Éstos son factores evidentes pero muchas veces no reflejan la totalidad de la situación.
La empresa moderna debe manejar eficientemente estos factores con el objeto de controlar
los procesos de una manera productiva y con orientación hacia la calidad.

El control de producción es algo más que planeación, es la aplicación de varios


modos y medios, para asegurar la ejecución del programa de producción deseado.

El proceso de planificación, producción y el control de producción deben seguir un enfo-


que jerárquico que permita la coordinación entre objetivos, planes y actividades de los nive-
les estratégicos, tácticos y operativos. Es decir, cada uno de estos trata de lograr su propia
meta, teniendo en cuenta los del nivel superior, de los cuales depende y los de nivel inferior
que restringen. Ver figura 37:

Proyecto de Planeación de la Control de la


producción producción producción

Formulación Programación
del plan de de la
producción producción

Figura 37. Fases principales de la Planificación y el Control de la Producción.

170
Sánchez Marchán, Nanci. (2014).Introducción a la Ingeniería Industrial, Material Instruccional de Apoyo. Caracas: UNA

La planificación y el control de la producción dependen del tipo de proyecto de produc-


ción que se va a realizar. Cuando este proceso se planea por anticipado, se conocen los in-
sumos que se van a requerir o a comprar; satisfaciéndose las necesidades y expectativas de
productores, proveedores, trabajadores y consumidores, lográndose una máxima economía.
Acción que se traduce en un aumento en la productividad.

PRODUCTIVIDAD
•• Evolución de la productividad.
•• Tipos de productividad.
•• Factores que afectan a la productividad.
•• Medición de la productividad a nivel de la empresa.

EVOLUCIÓN DE LA PRODUCTIVIDAD
Tal vez la primera vez que se mencionó la palabra productividad de manera formal,
fue en un artículo de Quesnay en el año de 1766. Más de un siglo después, en 1883, Littre
definió la productividad como la facultad de producir, es decir, el deseo de producir. Sin em-
bargo, no fue sino hasta principios del siglo veinte que el término adquirió un significado más
preciso como una relación entre lo producido y los medios empleados para hacerla.

En 1950, la Organización para la Cooperación Económica Europea (OCEE) ofreció una


definición más formal de productividad:

Productividad, es el cociente que se obtiene al dividir la producción por uno de los


factores de producción. De esta forma es posible hablar de la productividad del capital, de
la inversión o de la materia prima según si lo que se produjo se toma en cuenta respecto
al capital, a la inversión o a la cantidad de materia prima, etc.

Se observa en la tabla que se presenta a continuación, un resumen de las definiciones


más importantes, referentes a la productividad. También se muestra en ella fechas y los
nombres de los personajes que las crearon.

171
Sánchez Marchán, Nanci. (2014).Introducción a la Ingeniería Industrial, Material Instruccional de Apoyo. Caracas: UNA

Tabla 20. Avance en las definiciones de la productividad y sus creadores.

Siglo Personaje Año Definición productividad


La palabra productividad aparece por primera vez,
fisiócrata Francois en el artículo de Quesnay “la riqueza proviene de la
XVII 1766
Quesnay naturaleza y que ésta es más productiva en cuanto
produce mayor riqueza”.
Observa en el incremento de la plusvalía la motiva-
XIX Carlos Marx 1860
ción capitalista para incrementar la productividad.
Littre 1883 Facultad de producir, asociada a la acción de hacer.
Década
Early Relación entre producción y los medios.
de 1900
Relación entre productividad obtenida y trabajo
Taylor, Henry Ford 1908
empleado.
Organización para la
Cooperación Econó- Cociente que se obtiene al dividir la producción por
mica Europea 1950 uno de los factores de producción, capital, inversión,
materia prima.
(OCEE)
Cambio en el producto obtenido por los recursos
Davis 1955
gastados.
Siempre una razón entre la producción y los insu-
XX Fabricant 1962
mos.
Definiciones funcionales para la productividad par-
Kendrick & Creamer 1965
cial y total.

Una familia de razones entre la producción y los


Siegel 1975
insumos.
Productividad total es la razón de producción tangi-
Sumanth 1979
ble entre insumos tangibles.

La Productividad es considerada como el motor que está detrás del progreso económi-
co y de las utilidades de la empresa. Es esencial para incrementar los salarios y el ingreso
personal. Un país que no mejora su productividad pronto reducirá su estándar de vida.

Por esta razón la productividad es considerada como uno de los indicadores más im-
portantes de los resultados económicos. Se presenta en forma de fracción, en la cual el
numerador es la producción física y el denominador los factores que han sido necesarios
para obtenerla.

En consecuencia la Productividad se puede definir como:

La correlación entre la cantidad producida y los insumos utilizados en un sistema de


producción.

Se puede hablar de productividad parcial o total.

Productividad parcial. Es la razón entre la cantidad producida y un insumo determi-


nado.

172
Sánchez Marchán, Nanci. (2014).Introducción a la Ingeniería Industrial, Material Instruccional de Apoyo. Caracas: UNA

PP = El insumo puede ser: mano de obra, capital,


energía, materiales, entre otros.

Ejemplos Ilustrativo N° 1
Productividad = PIB. /M.O. (Producto Interno Bruto/Mano de Obra)
Productividad = PIB. /Capital
Productividad = Ventas / Pagos
Productividad = PIB. / M.O + Capital
PIB (Producto Interno Bruto): es la suma del valor de todos los bienes y servi-
cios finales producidos en un país en un año.

No existe ninguna actividad humana que no se beneficie de la producti-


vidad, razón por la cual, la mayor parte del aumento del ingreso nacional
bruto se produce mediante el mejoramiento de la eficacia y la calidad de
mano de obra y no mediante la utilización de más trabajo y capital.
Ejemplo Ilustrativo N° 2
La pastelería CASCANUECES, elaboran pasteles de diferentes tipos que ven-
den a supermercados y restaurantes. Para realizar este trabajo requiere de tres
empleados que invierten 6 horas diarias para producir 150 pasteles. Calcule la
productividad en función del recurso tiempo empleado:
a) La productividad para la producción de 1500 pasteles
b) La productividad si aumenta su producción a 1550 pasteles.
c) ¿Cuál ha sido la variación porcentual de la productividad?

Solución
a) Productividad = 1.800 pasteles/ 6 hs = 300 pasteles/h
b) Productividad = 1.850 pasteles/ 6 hs = 308,33 pasteles/h
c) Variación de la productividad (308 – 300) pasteles = 8.33 pasteles = 8,33%

De lo anterior se puede decir que la variación en la productividad experimentó un au-


mento leve. Se considera una situación favorable, debido a que el insumo utilizado (tiempo)
se mantuvo constante.

Ejemplo Ilustrativo N° 3
Identifique en términos de qué se mide la productividad de un:
a) Periodista.
b) Traductor.
c) Vendedor.
d) Fábrica de calzado.
e) Proveedor de servicios de internet.

173
Sánchez Marchán, Nanci. (2014).Introducción a la Ingeniería Industrial, Material Instruccional de Apoyo. Caracas: UNA

La productividad para los ejemplos señalados se mide en términos de:

Periodista En hacer noticias veraces en el menor tiempo posible.


En traducir mayor números de documentos por hrs, día, sema-
Traductor
na, mes o años.
Vendedor En hacer todas las ventas posibles en un tiempo determinado.
Zapatos elaborados, con la menor cantidad de insumos utiliza-
Fábrica de calzado
dos.
Proveedor de servi-
Usuarios atendidos en hrs/día, hrs/semana, hrs/mes.
cios de internet

Productividad Total. Es la relación entre la producción total y la suma de todos los


factores de insumo. La medida de productividad total, refleja el costo conjunto de todos los
insumos al fabricar los productos. Tanto la producción como los insumos se expresan en
términos reales o físicos, convirtiéndolos en moneda constantes de un período de referencia.

PT = Insumos totales: mano de obra + capital + materiales +


energía, entre otros.

Ejemplo Ilustrativo N° 1
Productividad Total = PIB. / Mano de obra + Capital+ energía.

Ejemplo Ilustrativo N° 2
Una empresa obtiene una producción de 3.568 unidades empleando 360 horas
de trabajo, 35 unidades de material A y 698 unidades de material B. Los datos
sobre precios se refieren a continuación:
Precio unitario del producto: 15 Bs; precio de la hora del trabajo: 50 Bs; precio
unitario del material A: 120 Bs; precio unitario del material B: 3 Bs.
Se pide calcular la productividad total.

PT = = = = 2,2

El valor monetario de la producción es 2,2 veces el valor monetario de los recur-


sos empleados para obtenerla.

En términos cuantitativos, la producción es la cantidad de productos que se pro-


dujeron, mientras que la productividad es la razón entre la cantidad producida y
los insumos utilizados.

174
Sánchez Marchán, Nanci. (2014).Introducción a la Ingeniería Industrial, Material Instruccional de Apoyo. Caracas: UNA

La productividad implica la mejora del proceso productivo y ella aumenta cuando:


•• Existe una reducción de los insumos mientras las salidas permanecen
constantes.
•• Existe un incremento de las salidas, mientras los insumos permanecen
constantes.
Con frecuencia se confunden entre si los términos productividad, eficiencia y
efectividad. La productividad es una combinación de eficiencia y efectividad, ya
que la efectividad está relacionada con el desempeño y la eficiencia con la utili-
zación de recursos.

FACTORES QUE AFECTAN A LA PRODUCTIVIDAD


Estos factores se pueden clasificar de la siguiente manera: factores externos y factores
internos.

FACTORES EXTERNOS Incluyen: leyes gubernamentales, competitividad industrial, de-


manda de los clientes, cambios demográficos, comunicaciones,
(No son controlados por
entre otros. Como están fuera del control de la empresa pueden
la empresa)
perturbar tanto el volumen de la salida como la distribución de las
entradas.

Calidad
 Se debe reflejar tanto en el producto como en el proceso, es decir
un producto se debe fabricar con la mejor calidad posible aten-
diendo a su diseño, variedad, costos y precio.
En el diseño, la investigación y desarrollo de productos, condu-
cen a la utilización de nuevas tecnologías que pueden mejoran la
productividad.
El proceso se selecciona adecuadamente de acuerdo con el pro-
ducto y el mercado, para lograr un aumento en la productividad.

FACTORES INTERNOS
Entradas de insumos
(Son controlados por la
empresa) Mano de obra: su productividad está asociada a un gran núme-
ro de factores: selección y ubicación, capacitación, diseño del
trabajo, supervisión, estructura organizacional, remuneraciones,
objetivos, sindicatos y sistemas de motivación, entre otros.
La entrada de insumos como: materia prima, maquinaria, energía,
capital, capacidad técnica; está asociada a la planificación de la
capacidad de producción, gestión de compras e inventarios, entre
otros.
Salidas de bienes o servicios
Las salidas están ajustadas al incremento o disminución de las
entradas. Ejemplo aumento de la producción, manteniendo los in-
sumos constantes, se traduce en un aumento de la productividad.

175
Sánchez Marchán, Nanci. (2014).Introducción a la Ingeniería Industrial, Material Instruccional de Apoyo. Caracas: UNA

MEDICIÓN DE LA PRODUCTIVIDAD A NIVEL DE LA EMPRESA


La productividades un indicador económico, que mide la calidad en una empresa. Es
utilizada por los gerentes de operaciones para evaluar la capacidad de un sistema que ela-
bora los productos requeridos por los clientes y, a la vez el grado en que se aprovechan
los recursos empleados, es decir, el valor agregado. Pero para mejorar la productividad, no
basta sólo con mejorar la producción, es preciso considerarla como un asunto de toda la
organización y tener presente áreas tan importantes como ventas, finanzas, personal, pro-
cesamiento de datos, entre otros.

Una de las ventajas de contar con una excelente productividad a nivel de la empresa es
que:

1. Ayuda a disminuir los costos e incrementar los beneficios económicos y sociales.

2. Permite la competitividad de una empresa. Una empresa es competitiva en relación


con otras, cuando produce con mejor calidad y a bajos costos.

Medidas para aumentar la productividad


•• A largo plazo. Inversión en factores fijos: nuevas instalaciones, nuevas
maquinaria y equipos, etc.
•• A mediano plazo y corto plazo.Mejor aprovechamiento de los recursos
disponibles: estudio de tiempos y movimientos, mejora de métodos de
trabajo, redistribución del personal, entre otros.

Se pueden comparar dos cifras de productividad global de una empresa midiendo la


Tasa de Variación de la Productividad Global Total (TVPGT). Se colocan ambas cifras con
los mismos precios, es decir, se obtiene la productividad del año actual valorando la produc-
ción y sus entradas (imputs) a precios del año anterior.

La Tasa de Variación de Productividad Total (TVPGT) debe ser > 1

La productividad no se considera favorable para valores menores que 1.

176
Sánchez Marchán, Nanci. (2014).Introducción a la Ingeniería Industrial, Material Instruccional de Apoyo. Caracas: UNA

Ejemplo Ilustrativo
En la empresa del ejemplo ilustrativo Nº 2 de producción total, se presenta un
año después los siguientes datos: producción 3.500 unidades empleando 500
horas de trabajo, 40 unidades de material A y 200 unidades de material B.
El precio unitario del producto es de 18 Bs; precio de la hora del trabajo: 55 Bs;
precio unitario del material A: 121 Bs; precio unitario del material B: 8 Bs.
Se pide calcular la Tasa de Variación de la Productividad Global Total (TVPGT)
del año pasado con respecto al presente.
Tasa de Variación de Productividad total (TVPGT) = =

= = 1,73

TVPG = = 0,2136

Significa que la productividad global de la empresa ha caído 21,36%.

El sistema de producción guarda una relación estrecha con la productividad, ya


que el mismo tiene como objetivo primordial conseguir un aumento de ésta. En la fa-
bricación, el concepto de productividad sirve para evaluar el rendimiento de los talleres, los
equipos de trabajo, las máquinas y los empleados.

Para los empresarios, productividad es sinónimo de rendimiento y simplifican la defi-


nición diciendo que un buen sistema de producción, utilizando una cantidad adecuada de
recursos debe obtener el máximo de productos en el menor tiempo posible.

177
Sánchez Marchán, Nanci. (2014).Introducción a la Ingeniería Industrial, Material Instruccional de Apoyo. Caracas: UNA

ESTUDIO DEL TRABAJO: MÉTODOS, MOVIMIENTOS Y TIEMPO

Objetivo:
Señalar la importancia del Estudio del Trabajo, sobre la base del Estudio de Méto-
dos y de la medición de este, en la realización de una tarea.

•• La Ingeniería de Métodos
•• Estudio del trabajo
•• Estudio de métodos
•• Medición del trabajo

INGENIERÍA DE METODOS

La Ingeniería de Métodos se puede definir como:

Un conjunto de procedimientos sistemáticos para someter a todas las operaciones de


producción a un análisis minucioso,  con vista a introducir mejoras que faciliten más la
realización del trabajo y que permita que éste sea hecho en el menor tiempo e inversión
productiva posible, facilitándose el estudio del trabajo.

Se puede definir el trabajo como: el esfuerzo que realiza un individuo o grupo de indi-
viduos para desarrollar una serie de tareas a nivel físico o intelectual, que permitan generar
un producto.

En atención a la definición antes expuesta se puede decir que:

El estudio del trabajo, es la aplicación de ciertas técnicas, para examinar el esfuerzo que
realiza una persona o grupo en todos sus contextos; lo que conduce a investigar los fac-
tores que influyen en la eficacia, eficiencia y economía de la situación estudiada, con el
fin de generar mejoras.

El estudio de trabajo se divide en dos ramas: el estudio de métodos y la medición del


trabajo. La primera se ocupa del modo de hacer un trabajo; la segunda tiene como meta
averiguar cuánto tiempo se requiere para ejecutarlo.

La medición del trabajo y el estudio de métodos tienen su origen en la actividad de la


administración científica. Federick Taylor optimizó los métodos de trabajo mediante el estu-
dio minucioso de movimientos y fue el primero en utilizar él cronometro para medir el trabajo.
Otro aporte de Taylor fue la idea de cómo un estándar de producción (ejemplo, minutos por
pieza) debe establecerse por cada trabajo. Un estándar determina la cantidad de salida es-
perada de producción de un trabajador y se utiliza para planear y controlar los costos direc-
tos de mano de obra. Ver tabla 2.

179
Sánchez Marchán, Nanci. (2014).Introducción a la Ingeniería Industrial, Material Instruccional de Apoyo. Caracas: UNA

Se muestra a continuación una representación gráfica donde se puede diferenciar las


ramas de la división del trabajo:

Figura 38. Ramas del estudio del trabajo.

El estudio del trabajo permite incrementar la productividad sin recurrir a grandes in-
versiones de capital y sin exigir un mayor esfuerzo a la mano de obra. Este incremento de
productividad se conseguirá racionalizando el trabajo, para eliminar el tiempo suplementario
y el tiempo improductivo.

ESTUDIO DE MÉTODOS
El estudio de métodos se considera como el registro y examen crítico de los modos de
realizar actividades con el fin de efectuar mejoras. Abarca la formulación y selección de los
mejores métodos, procesos, herramientas, equipos diversos y características necesarias
para generar un producto o servicio.

El procedimiento básico para la realización de un estudio de métodos consta de los


siguientes pasos:
1. Seleccionar la tarea objeto del estudio.
2. Registrar todos los hechos del método actual por medio de observación directa.
3. Examinar críticamente y en forma ordenada cada detalle.

180
Sánchez Marchán, Nanci. (2014).Introducción a la Ingeniería Industrial, Material Instruccional de Apoyo. Caracas: UNA

4. Desarrollar el método más práctico y económico.


5. Implementar el nuevo método como práctica uniforme.
6. Mantener el nuevo método mediante comprobaciones periódicas.

El punto álgido de la mejora de métodos es examinar críticamente cada hecho o ele-


mento del método actual, para ser cuestionado, contestando la interrogante por qué:
¿Por qué es necesaria esta operación?
¿Por qué esta operación se efectúa de esta manera?
¿Por qué son tan estrechas estas tolerancias?
¿Por qué se ha especificado este material?
¿Por qué se ha asignado este operario para ejecutar el trabajo?

La interrogante ¿Por qué? indica de inmediato otras como ¿Qué?, ¿Cómo?, ¿Quién?
¿Dónde? y ¿Cuándo? Por ende, al analista le pueden surgir incógnitas como las plasmadas
en la figura 39:

Figura 39. Diagrama de las incógnitas que pueden surgir al analista de métodos.

181
Sánchez Marchán, Nanci. (2014).Introducción a la Ingeniería Industrial, Material Instruccional de Apoyo. Caracas: UNA

DIAGRAMAS DE PROCESO
Todo operario debe tener las herramientas necesarias que le ayuden a efectuar un me-
jor trabajo en el menor tiempo posible. Uno de los instrumentos de trabajo más importante
en la Ingeniería de métodos es el diagrama de proceso. Se define como:

Una representación gráfica relativa a un proceso industrial o administrativo.

En el análisis de métodos se usan generalmente ocho tipos de diagramas de proceso,


cada uno de los cuales tiene aplicaciones específicas. Ellos son:

1. Diagrama de operaciones de proceso o cursograma sinóptico: permite vi-


sualizar sólo operaciones e inspecciones que se ejecutan durante la elaboración
de un producto, a fin de analizar las relaciones existentes entre operaciones.

2. Diagrama de curso (o flujo) de proceso: representa gráficamente todas las


actividades que se realizan durante la elaboración de un producto, es decir, vi-
sualiza operaciones, inspecciones, transportes, almacenajes y demoras a fin de
analizar costos ocultos, actividades ocultas en el proceso productivo. Permite un
análisis completo de la fabricación de una pieza o componente.

3. Diagrama de recorrido de actividades: es la representación del diagrama de


proceso en un plano, donde se indica el recorrido y el descongestionamiento (si
existe) durante el proceso productivo, además permite revisar la distribución del
equipo en la planta. Existen dos tipos:

a. Tipo “Material”: presenta el proceso según los hechos ocurridos al material.


b. Tipo “Hombre”: presenta el proceso referido a las actividades del hombre.

4. Diagrama de interrelación hombre-máquina: representación gráfica de la se-


cuencia de elementos que componen las operaciones donde intervienen hom-
bres y máquinas. Este diagrama se emplea para estudiar, analizar y mejorar sólo
una estación de trabajo cada vez. Indica la relación exacta en tiempo entre el
ciclo de trabajo de la persona y el ciclo de operación de su maquina

5. Diagrama de proceso para grupo o cuadrilla: este diagrama es una adap-


tación del diagrama de proceso de hombre y máquina. Con el empleo de esta
herramienta el analista debe estar en condiciones de poder calcular el número
más económico de máquinas a atender por un operario; sin embargo, varios pro-
cesos y máquinas llegan a ser de tal magnitud que las preguntas a contestar no
es cuántas máquinas debe operar un trabajador, sino cuántos operarios se nece-
sitan para operar eficientemente una máquina. El diagrama muestra la relación
que se tiene entre el ciclo de inactividad y de operación de máquina, además del
tiempo muerto y efectivo.

6. Diagrama de proceso para operario: también conocido como diagrama bima-


nual, simo, mano izquierda-mano derecha, movimientos simultáneos. Este dia-

182
Sánchez Marchán, Nanci. (2014).Introducción a la Ingeniería Industrial, Material Instruccional de Apoyo. Caracas: UNA

grama muestra todos los movimientos realizados por la mano izquierda y por la
mano derecha, indicando la relación entre ellas.

6. Diagrama PERT y CPM: los métodos PERT y CPM están básicamente orienta-
dos a la determinación de un programa de tiempo. Aunque los dos métodos fue-
ron desarrollados casi independientemente, ambos son asombrosamente simila-
res. Quizá la diferencia más importante es que originalmente las estimaciones en
el tiempo para las actividades se supusieron determinantes en CPM y probables
en PERT. Ahora PERT y CPM comprenden realmente una técnica y las diferen-
cias, si existe alguna, son únicamente históricas.

La programación de proyectos por PERT-CPM consiste en tres fases básicas:


planeación, programación y control.

7. Diagrama de Gantt: es una forma fácil para calendarizar proyectos. Esta gráfica
se elabora utilizando barras que representan cada tarea o actividad. La longitud
de cada barra simboliza el tiempo relativo a cada una de ellas. Los cronogramas
de barras o Diagrama de Gantt fueron concebidos por el ingeniero norteamerica-
no Henry L. Gantt, uno de los precursores de la ingeniería industrial, contempo-
ráneo de Taylor. Ver tabla 2.

Se considera el estudio de esta gráfica en este texto, porque contribuye en la planifi-


cación y control de una serie de actividades de un proyecto en estudio, descritas para un
período determinado como se observa a continuación:

Diagrama de Gantt

Plan de Trabajo:
Semana 1 2 3 4 5 6 7 8 9
Actividad 1

Actividad 1
Actividad 2
Actividad 3
Actividad 4
Actividad 5
Actividad 6

Tiempo programado para actividad 1


Avance de la actividad 1
Observaciones:

183
Sánchez Marchán, Nanci. (2014).Introducción a la Ingeniería Industrial, Material Instruccional de Apoyo. Caracas: UNA

Se puede presentar un plan por actividad para mostrar el avance de la misma a la per-
sona responsable de revisar el trabajo, como se observa en el diagrama anterior; o exponer
el plan completo como en el ejemplo ilustrativo siguiente:

Ejemplo Ilustrativo
El profesor de la asignatura Introducción a la Ingeniería Industrial, asignó la ela-
boración de un trabajo de investigación. Se dispone de cuatro meses para reali-
zar dicha tarea. A través de un diagrama de GANTT, planifique las actividades a
realizar para alcanzar este objetivo.

Se definen las actividades a realizar:


•• Recopilación de la información.
•• Organización de la información.
•• Desarrollo del trabajo.
•• Transcripción del trabajo.

Luego se deben planificar las tareas en función de la complejidad que tengan, se le


asigna un tiempo a cada una de ellas para ejecutarlas y cumplir así con la meta establecida
por el estudiante, como sigue:

Diagrama de Gantt
Plan de trabajo: elaborar trabajo de investigación
Semanas
Semana 1 2 3 4 5 6 7 8 9 10 11 12
Selección del tema y título
del trabajo.
Recopilación de la informa-
ción.
Organización de la informa-
ción.

Desarrollo del trabajo.


Transcripción del trabajo.

Tiempo programado para actividades


Observaciones:

184
Sánchez Marchán, Nanci. (2014).Introducción a la Ingeniería Industrial, Material Instruccional de Apoyo. Caracas: UNA

Una vez calendarizadas las actividades de un proyecto se procede al estudio profundo


de los procesos que lo conforman, presentándolos de forma organizada, para lo cual se utili-
zan los diagramas de la operación, de flujo o curso del proceso, así como, el diagrama de
recorrido o representación en un plano de este diagrama de flujo (opcional). Muchas veces
se precisan conocer muchos más detalles en una determinada operación, en este caso se
emplea el diagrama bimanual o mano izquierda-mano derecha. Diagrama que conjuntamen-
te con los antes mencionados, también se abordarán en este texto.

Estos diagramas, excepto el bimanual, se emplean principalmente para exponer un pro-


blema de forma organizada. Por lo general, un problema no puede resolverse correctamen-
te, si no se presenta en forma adecuada. De manera que conviene describir aquí, estos
medios gráficos.

La utilización de los diagramas a estudiar en este texto sigue un orden. El primer diagra-
ma a realizar es el de operaciones del proceso, el cual permite conocer todos los detalles
de fabricación o administración de forma global; luego se elabora el de flujo del proceso,
donde se describe el proceso completo y su interrelación entre trabajos y actividades, con-
juntamente con este diagrama se puede preparar un diagrama de recorrido (opcional). Al-
canzado este punto, se presta atención al operario con el estudio de movimientos para una
tarea o un elemento del trabajo en particular, construyéndose así el diagrama bimanual.

Actividad propuesta
Indique cuál es la utilidad de cada uno de los siguientes diagramas.

Diagrama Utilidad
Diagrama de proceso para operario.
Diagrama de curso o flujo de proceso.
Diagrama de interrelación hombre-máquina.
Diagrama de operaciones de proceso.

PRINCIPIOS GENERALES PARA LA ELABORACIÓN DE LOS DIAGRAMAS


Para elaborar los diferentes diagramas, es necesario cumplir una serie de principios a
fin de presentar correctamente todas las actividades de modo que permita una clara interpre-
tación de lo ocurrido, durante un determinado proceso o procedimiento.

Los diagramas de proceso de la operación difieren ampliamente entre sí, como una con-
secuencia de las diferencias entre las actividades que reflejan, por tanto, no resulta práctico
preparar un único formato, ya que estos diagramas sólo coincidirían en la información de
identificación.

185
Sánchez Marchán, Nanci. (2014).Introducción a la Ingeniería Industrial, Material Instruccional de Apoyo. Caracas: UNA

Identificación: ésta se indica a través de un título, colocado en la parte superior del for-
mato, referido al tipo de diagrama de proceso a elaborar, además debe contener información
referida al método (si es actual o propuesto), así como, datos concernientes a:
•• Número de la pieza,
•• Número de plano,
•• Descripción del proceso,
•• Fecha y nombre de la persona que elabora el diagrama, entre otros.

Algunas veces hacen falta datos adicionales para identificar por completo el trabajo que
se diagrama. Tales datos pueden ser:
•• Los nombres de la planta, edificio o departamento.
•• Número de diagrama.
•• Cantidad de producción.
•• Información sobre costos.

Inicio del diagrama:


•• Se inicia en el extremo derecho del formato preparado para el diagrama
correspondiente, hacia el extremo izquierdo del mismo, conforme se va
desarrollando.
•• La primera vertical corresponderá al componente principal de la pieza a
elaborar, o aquél donde se realice el mayor número de operaciones.
•• En el caso del diagrama de operaciones, para indicar el material que se
utilizará en la obtención de cada componente (materia prima) se designa
una línea horizontal, sobre la cual se coloca una breve descripción del
material. En cuanto al diagrama de curso del proceso se coloca al inicio,
un símbolo de almacenaje el cual indica que el material se encuentra en
el almacén de materia prima o de componentes.
•• Luego se procede a colocar cada una de las actividades según el orden
en el que ocurran, al lado de cada actividad, se anota una breve descrip-
ción de ella.
•• Conforme se indican las actividades se enumeran de acuerdo con su
ocurrencia. Dicha numeración debe colocarse dentro de los símbolos de
cada actividad.

Los diagramas no deben hacerse de memoria sino a medida que se observa el


trabajo. Deben elaborarse con precisión y ajustándose a la realidad que se ob-
serva. Lo importante es que resulte una herramienta que pueda ser interpretada
por otras personas en el momento de hacer la mejora del método.

186
Sánchez Marchán, Nanci. (2014).Introducción a la Ingeniería Industrial, Material Instruccional de Apoyo. Caracas: UNA

DIAGRAMA DE OPERACIONES DEL PROCESO O CURSOGRAMA


SINÓPTICO
Un diagrama de operaciones del proceso es una representación gráfica del momen-
to en el que se introducen materiales en el proceso, del orden de las inspecciones y de todas
las operaciones, excepto las incluidas en el manejo de los materiales. Puede además com-
prender cualquier otra información que se considere necesaria para el análisis, por ejemplo
el tiempo requerido, la situación de cada paso o si ayudan los ciclos de fabricación.

Se usan líneas verticales para indicar el flujo o curso general del proceso a medida que
se realiza el trabajo y se utilizan líneas horizontales que se vinculan con las líneas de flujo
verticales para indicar la introducción de material, ya sea proveniente de compras o sobre el
que ya se ha hecho algún trabajo durante el proceso.

Los objetivos del diagrama de las operaciones del proceso son:


•• Dar una imagen clara de toda la secuencia de los acontecimientos del
proceso.
•• Estudiar las fases del proceso en forma sistemática.
•• Mejorar la disposición de los locales y el manejo de los materiales. Esto
con el fin de disminuir las demoras, comparar dos métodos, estudiar las
operaciones, para eliminar el tiempo improductivo.
•• Finalmente, estudiar las operaciones y las inspecciones en relación unas
con otras, dentro de un mismo proceso.

En la tabla 21 se pueden observar los símbolos a utilizar en la construcción de un dia-


grama de las operaciones del proceso.

Tabla 21. Símbolos a utilizar en la construcción de un diagrama de las operaciones del pro-
ceso, adaptación de la simbología utilizada por la Sociedad Americana de Ingenieros Mecá-
nicos (American Society of Mechanical Enginners, ASME)

Símbolo Actividad Significado


Se produce o efectúa algo.
Operación

Inspección Se verifica calidad o cantidad.

Indica actividades conjuntas de operación e


Actividad combinada inspección realizada por el mismo operario
en el mismo punto de trabajo.
Flujo o curso general del Indica que no hay intersección entre una
proceso a medida que se línea de flujo vertical y una línea de mate-
realiza el trabajo. riales horizontal

187
Sánchez Marchán, Nanci. (2014).Introducción a la Ingeniería Industrial, Material Instruccional de Apoyo. Caracas: UNA

Ejemplo Ilustrativo
En la imagen mostrada y con la información suministrada, se pide elaborar el dia-
grama del proceso de la operación correspondiente:

Actividades requeridas en el eje: Actividades requeridas en la moldura de plás-


1. Cepillar, tornear, muescar y cortar en torno re- tico:
vólver (0.025 hr). 1. Cepillar la parte de plástico (0.80 hr).
2. Cepillar extremo opuesto (0.010 hr). 2. Taladrar para el pernete de tope (0.022 hr).
3. Inspección. 3. Inspección.
4. Fresar (0.070 hr). 4. Montar el moldeado en la parte pequeña del
5. Eliminar rebaba (0.020 hr). eje y taladrar de lado para el pernete de tope.
6. Inspección del fresado.
7. Desengrasar (0.0015 hr).
8. Cadminizar (0.008 hr).
9. Inspección.

Actividades a realizar en el pernete de tope:


1. Tornear una espiga de 2 mm; biselar extremo y cortar en torno revólver (0.025 hr).
2. Quitar rebaba con una pulidora (0.005 hr).
3. Desengrasar (0.0015 hr).
4. Cadminizar (0.006 hr).
5. Inspección.
6. Fijar el pernete al montaje (0.045 hr).
7. Inspección.

Procedimiento:

1. Identificar el diagrama a través de un título colocado en la parte superior, además


debe contener información referida al método (si es actual o propuesto), fecha, per-
sona que lo elaboró, etc.
2. Se inicia el diagrama generalmente de Izquierda a derecha colocando líneas hori-
zontales que indican entrada del material, la cantidad de líneas depende del número
de partes que conforma el producto.
3. Una vez reflejada la entrada de materiales, debajo de las mismas se van indicando
las operaciones e inspecciones necesarias para desarrollar cada parte del producto.
4. A medida que se va requiriendo, se reflejan los ensambles de las piezas, hasta lo-
grar completar el producto final.
5. Las descripciones de las operaciones e inspecciones se pueden colocar al lado
izquierdo de cada símbolo si así lo prefiere.

188
Sánchez Marchán, Nanci. (2014).Introducción a la Ingeniería Industrial, Material Instruccional de Apoyo. Caracas: UNA

6. Al final del diagrama se puede elaborar un cuadro resumen donde se muestre el nº


total de operaciones e inspecciones.
Diagrama del Proceso de la Operación

Proceso: Fecha:
Elaborado por:
Diagrama Actual ( X ) Propuesto ( ) Nºde Diagrama:

Pernete de Tope Moldura de Plástico Eje


5mm de diámetro Fernolformaldehido 10mm de diámetro de Acero
Acero BSS 32/4 s.69

1 5 7
0.025hr 0.080hr 0.025hr

0.010hr 8
0.025hr 2 0.022hr 6

1 3

0.070hr 9

0.0015hr 3 Fresar
0.020hr 10

5
0.006hr 4

0.015hr 11

0.008hr 12

0.020hr 13

0.045hr 14

Cuadro Resumen
N° Operaciones N° Inspecciones
14 7

189
Sánchez Marchán, Nanci. (2014).Introducción a la Ingeniería Industrial, Material Instruccional de Apoyo. Caracas: UNA

Se debe colocar el nombre de cada actividad, al lado derecho de estas, como se mues-
tra en la operación Nº 3. Como ejercicio coloque el nombre al resto de las actividades, según
el número y definición que tiene en el problema.
El diagrama de operaciones ayuda  a promover y explicar un método propuesto deter-
minado. Como proporciona claramente una gran cantidad de información, es un medio de
comparación ideal entre dos soluciones competidoras.
Este diagrama ya terminado ayuda a visualizar en todos sus detalles el método presen-
te, logrando imaginar nuevos y mejores procedimientos; indica al analista qué efecto tendría
un cambio en una operación dada, sobre las operaciones precedente y subsiguiente. La
sola elaboración del diagrama de operaciones del proceso señala diversas posibilidades de
mejoramiento al analista.

DIAGRAMA DE FLUJO O CURSO DE PROCESO


Este diagrama constituye el instrumento más eficaz para el análisis y mejora de mé-
todos. Indica las diversas actividades a que da lugar la realización de una tarea, la cual fue
seleccionada previamente; es decir describe el proceso completo y su interrelación entre
trabajos y actividades.
Una vez definido claramente dónde comienza y dónde termina la tarea, las diversas ac-
tividades que componen el diagrama son registradas bajo los símbolos de operación, trans-
porte, inspección, demora y almacenamiento. El diagrama se elabora para registrar los re-
sultados de un relevamiento de las tareas que implica el proceso en estudio y de este modo
tener un panorama de lo que está sucediendo en el lugar de trabajo.

Símbolos a utilizar para elaborar el diagrama de flujo del proceso


Los símbolos a utilizar en la elaboración de este diagrama se observan en la tabla 22.
Tabla 22. Símbolos para la elaboración del diagrama de flujo del proceso. Según la
American Society of Mechanical Enginners (ASME)

190
Sánchez Marchán, Nanci. (2014).Introducción a la Ingeniería Industrial, Material Instruccional de Apoyo. Caracas: UNA

Se muestra seguidamente una adaptación de una tabla, también elaborada por la


ASME, donde se puede observar cada uno de estos símbolos y ejemplos alusivos de lo que
representan.

Tabla 23. Conjunto de estándares de símbolos para diagramas de proceso, adaptación


según la ASME

191
Sánchez Marchán, Nanci. (2014).Introducción a la Ingeniería Industrial, Material Instruccional de Apoyo. Caracas: UNA

El diagrama de flujo, no representa un fin como el diagrama de operaciones del


proceso, sino sólo un medio para lograr una meta. Se utiliza como instrumento
de análisis para eliminar los costos ocultos de un componente. Como el diagrama
muestra claramente todos los transportes, retrasos y almacenamientos, es conve-
niente para reducir la cantidad y la duración de estos elementos.

Actividad propuesta
Sobre la base de los símbolos antes estudiados se plantea el siguiente ejercicio:
En la siguiente tabla se muestra una serie de actividades correspondientes al
proceso productivo de la elaboración de un cuaderno. Seleccione con una (X) el
símbolo que corresponde a cada actividad de acuerdo a sus características.

Actividad O D

Almacenar materia prima


Trasladar materia prima desde el alma-
cén hasta el sitio de trabajo
Colocar materiales en mesa de trabajo
Medir portada, dependiendo del tamaño
del cuaderno
Inspeccionar medidas

Cortar portada

Dibujar motivo de portada

Imprimir motivo en portada


Esperar mientras seca la impresión del
dibujo
Medir hojas en función de tamaño de
portada
Cortar hojas

Ensamblar hojas con portada

Inspeccionar producto final

Transportar el producto final al almacén.

El diagrama de flujo se puede complementar con un diagrama de recorrido, para lo-


grar una mejor distribución en planta al ahorrar distancias y, por tanto, tiempo. El diagrama de
recorrido de actividades se efectúa sobre un plano donde se sitúan las máquinas a escala.
En él se traza una línea que indica la secuencia que seguirá el producto.

192
Sánchez Marchán, Nanci. (2014).Introducción a la Ingeniería Industrial, Material Instruccional de Apoyo. Caracas: UNA

Ejemplo Ilustrativo
Elabore el diagrama de flujo del proceso y su respectivo diagrama de recorrido de
la elaboración de un cinturón para un vestido de dama, el cual está compuesto de
las siguientes actividades:

Cinto: Hebilla:
1. Transportar entretela a máquina. 1. Forrar alambre.
2. Coser cinto. 2. Transportar a cortadora.
3. Coser a tamaño. 3. Cortar a tamaño.
4. Coser punta. 4. Doblar hebilla.
5. Cortar punta. 5. Transportar a prensas.
6. Transportar pieza a máquina. 6. Poner grapas (material de compra).
7. Perforar ojal. 7. Poner aguijón (material de compra).
8. Perforar 5 ojillos. 8. Esperar ensamble.
9. Poner 5 ojillos. 9. Transportar a ensamble.
10. Esperar ensamble.
11. Transportar a ensamble.

Trabilla:
1. Coser trabilla.
2. Esperar ensamble.
3. Llevar a ensamble.
4. Armar cinturón (junta cinto, hebilla y trabilla).
5. Transportar al almacén de productos termi-
nados.
6. Almacenado.

Solución:

Al igual que en el diagrama de operaciones, este diagrama inicia con una identificación,
la cual debe colocarse como un encabezado, donde se observa el tipo de diagrama, la ac-
tividad en estudio, número de diagrama, el nombre de la persona que preparó el diagrama,
entre otros. Seguidamente se procede a ubicar la descripción de las actividades a realizar
en una columna, al lado izquierdo del formato correspondiente. Los símbolos que represen-
tan cada una de estas actividades están ubicados en el centro de este formato. En el lado
derecho se encuentra otra columna, donde se plasman datos como distancias recorridas,
unidades en proceso y tiempo requerido para cada actividad.

La secuencia del proceso diagramado se refleja uniendo los símbolos de las diferentes
actividades con una flecha, asignándole un número a cada actividad, según su orden de
ejecución. Cada tipo de actividad comienza con el número 1, aumentando progresivamente,
según se repita la acción.

Al final se elabora un cuadro resumen con las diferentes actividades y la frecuencia con
la cual se repite cada una de ellas.

193
Sánchez Marchán, Nanci. (2014).Introducción a la Ingeniería Industrial, Material Instruccional de Apoyo. Caracas: UNA

Diagrama del Proceso de Flujo


Proceso: cinturón para vestido modelo 40-20 Fecha: enero de 2014
Elaborado por: Pedro López N° de diagrama: 15
Inicio: departamento de cintos Unidad de costo:
Culminación: almacén de productos terminados Producción anual:
Método: Actual ( X ) Propuesto ( )
Sánchez Marchán, Nanci. (2014).Introducción a la Ingeniería Industrial, Material Instruccional de Apoyo. Caracas: UNA

Diagrama de Recorrido

1 2

11
9
7
12

4 10
5

8 3

14

2 5
6

13 3

1 4
7
6

A 2
l
m
a
c
e
n
1
6 1 1

195
Sánchez Marchán, Nanci. (2014).Introducción a la Ingeniería Industrial, Material Instruccional de Apoyo. Caracas: UNA

DIAGRAMA MANO IZQUIERDA-MANO DERECHA (BIMANUAL, DEL


PROCESO DEL OPERARIO)
Presenta todos los movimientos y pausas realizadas por la mano derecha y la izquierda,
además de las relaciones entre las divisiones básicas relativas a la ejecución del trabajo
efectuado por ambas manos. El objeto de este diagrama es poner de manifiesto una ope-
ración específica con los detalles suficientes, de modo que se pueda mejorar mediante un
análisis. Este análisis  es recomendado sólo en operaciones manuales altamente repetitivas.
Se busca descubrir patrones de movimientos ineficientes en los que puedan observarse
transgresiones a las leyes de la economía de movimientos. El resultado de la aplicación de
este diagrama y las posteriores correcciones que se efectúen, será un ciclo de trabajo más
regular y rítmico que ayudará a minimizar las demoras y la fatiga del operario.

Frank Gilbreth fue el fundador de la técnica moderna del estudio de movimientos, la cual
se puede definir como el estudio de los movimientos del cuerpo, que se utilizan para ejecutar
una operación; con la idea de mejorarla, eliminando y simplificando los movimientos innece-
sarios y estableciendo después, la secuencia de movimientos más favorable para lograr una
eficiencia máxima. Ver tabla Nº 2.

ELABORACIÓN DEL DIAGRAMA


Para representar las actividades en este diagrama se emplean los mismos símbolos que
en los diagramas anteriores, pero se les atribuye un sentido ligeramente distinto. Los detalles
se muestran en la siguiente tabla:

Tabla 24. Símbolos a utilizar en el diagrama bimanual


Actividad / Definición Símbolo
Operación: se emplea para los actos de asir, sujetar, utilizar, soltar, etc.,
una herramienta, pieza o material.
Transporte: se emplea para representar el movimiento de las manos hasta
el trabajo, herramienta o material en proceso.
Espera: se emplea para indicar el tiempo en el que una o ambas manos
están en reposo.
Sostenimiento: en los diagramas bimanuales no se emplea el término
almacenamiento, y el símbolo que le correspondía se utiliza para indicar el
acto de sostener alguna pieza, herramienta o material con la mano cuya
actividad se está consignando.
Inspección: este símbolo casi no se emplea. Cuando un objeto se sujeta,
mira o calibra, los movimientos de la mano vienen a ser operaciones, para
los efectos del diagrama. Sin embargo, a veces resulta útil emplearlo, para
resaltar que se examina algo.

El diagrama Bimanual puede aplicarse a una gran variedad de trabajos de montaje, de


elaboración a máquina y también de oficina. Los ajustes apretados y la colocación en posi-
ciones difíciles pueden presentar ciertos problemas, para la elaboración de este diagrama

196
Sánchez Marchán, Nanci. (2014).Introducción a la Ingeniería Industrial, Material Instruccional de Apoyo. Caracas: UNA

Montar pequeñas piezas ajustadamente y ponerlas en posición antes del montaje puede
ser la parte más prolongada de un ciclo. En tal caso la puesta en posición deberá exponerse
como un movimiento en sí de operación, aparte del que se efectúa para hacer el montaje
propiamente dicho. Por ejemplo: colocar un destornillador en la cabeza de un tomillo peque-
ño. Realizando el procedimiento descrito anteriormente se resalta dicho movimiento y si se
muestra en relación con una escala de tiempos, se podrá evaluar su importancia relativa.

Se pueden lograr economías considerables con este tipo de diagrama, si es posible re-
ducir el número de colocaciones, por ejemplo: avellanando ligeramente el orificio y biselando
más la punta de la herramienta, o utilizando un destornillador neumático.

MOVIMIENTOS FUNDAMENTALES DEL DIAGRAMA MANO IZQUIERDA-


MANO DERECHA
El concepto de división básica del trabajo, desarrollado por Frank Gilbreth en sus pri-
meros ensayos, se aplica a todo trabajo productivo ejecutado por las manos del operador.
El llamo “therbligs” (su apellido leído al revés) a estos movimientos fundamentales; concluyó
que todas y cada una de las operaciones de un trabajo, se componen de una serie de 17
divisiones básicas. Seguidamente aparece una clasificación y una tabla con estos 17 movi-
mientos fundamentales de las manos, así como sus símbolos. Estos movimientos aparecen
un tanto modificados de acuerdo con un resumen de Gilbreth.

De acuerdo al resumen de Gilbreth, los therbligs se clasifican en una serie de divisiones


básicas: físicas, objetivas, semimentales o mentales y de retrasos. Idealmente, todo centro
de trabajo debería componerse, exclusivamente, de therbligs físicos y objetivos.

Efectivos:
Divisiones físicas básicas: Divisiones objetivas básicas:
• Alcanzar. • Usar.
• Mover. • Ensamblar.
• Sujetar. • Desensamblar.
• Soltar.
• Colocación previa.

Inefectivos:
Divisiones mentales y semimetales Retrasos:
básicas: • Retrasos inevitables.
• Buscar. • Retrasos evitables.
• Seleccionar. • Descanso para sobrellevar la fatiga.
• Colocar. • Sostener.
• Inspeccionar.
• Planear.

En la tabla 25 se observa un resumen de los Therbligs efectivos e inefectivos desarro-


llados por Gilberht, con sus respectivos símbolos.

197
Sánchez Marchán, Nanci. (2014).Introducción a la Ingeniería Industrial, Material Instruccional de Apoyo. Caracas: UNA

Tabla 25. Resumen de los Therbligs:

Therbligs efectivos
Therblig´s Símbolo Descripción
Movimiento con la mano vacía desde y hacia el ob-
Alcanzar AL jeto; por lo general en esta tarea se toma el objeto y
luego se suelta y así sucesivamente.
Movimiento con la mano llena, el tiempo depende del
Mover M
peso, la distancia y el tipo de movimiento.
Cerrar los dedos alrededor del objeto, este tipo de
movimiento empieza en el momento en el que los de-
Tomar T
dos hacen contacto con el objeto y termina cuando
está bien sujeto el mismo.
Dejar el control de un objeto.
Soltar S
Posicionar un objeto en un lugar predeterminado para
Preposicionar PP
su uso posterior, casi siempre ocurre junto con mover.
Manipular, utilizar una herramienta al usarla para lo
Usar U que fue hecha.
Unir dos partes que van juntas.
Ensamblar E
Opuesto al ensamble, separación de partes que es-
Desensamblar DE tán juntas.

Therbligs no efectivos o inefectivos


No ayudan al trabajo, por lo tanto hay que eliminarlos
Therblig´s Símbolo Descripción
Ojos o manos que deben encontrar un objeto, este mo-
Buscar B vimiento inicia al momento de mover los ojos para loca-
lizar el objeto.
Elegir un artículo entre varios.
Seleccionar SE

Posicionar P Orientar un objeto durante el trabajo.


Comparar un objeto con el estándar, casi siempre con
Inspeccionar I la vista.
Hacer una pausa para determinar la siguiente acción,
Planear PL por lo general se detecta como una duda antes del mo-
vimiento.
Más allá del control del operario debido a la naturaleza
Retraso Inevitable RI de la operación, por ejemplo la mano izquierda espera
mientras la derecha termina un alcance más lejano.
Sólo el operario es responsable del tiempo ocioso.
Retraso Evitable R
Descanso para Con- Aparece en forma periódica, no en todos los ciclos. De-
D
trarrestar la Fatiga pende de la naturaleza del trabajo.
Una mano detiene un objeto mientras la otra realiza un
Sostener SO
trabajo provechoso.

198
Sánchez Marchán, Nanci. (2014).Introducción a la Ingeniería Industrial, Material Instruccional de Apoyo. Caracas: UNA

Es importante tratar de eliminar los Therblings inefectivos, ya que no ayudan al trabajo,


en su mayoría constituyen demoras o retrasos.

“Siempre hay un método mejor”

Ejemplo Ilustrativo
A continuación se muestra un ejemplo de la construcción del diagrama bima-
nual, referente al recargo de cartuchos:

En el recargo de cartuchos calibre 32 se utilizan una serie de operaciones para el forma-


do completo de éstos. Se necesitan fulminantes, casquillos, pólvora y balas. Los fulminantes
se encuentran en una caja, así como los casquillos y las balas. La pólvora se encuentra en
un recipiente y es necesario pesarla antes de meterla al cartucho. Se tiene la siguiente área
de trabajo.

1 2 3 4

El procedimiento para recargar cartuchos calibre 32 es el siguiente:


•• La mano izquierda obtiene el fulminante y la mano derecha obtiene el
casquillo (0.5 seg).
•• La mano izquierda coloca el fulminante en el casquillo y la mano derecha
sostiene el casquillo (0.8 seg).
•• La mano izquierda obtiene la pólvora y la mano derecha coloca el cas-
quillo.
•• La mano izquierda coloca la pólvora en la pesadora y la mano derecha
espera (1 seg).

199
Sánchez Marchán, Nanci. (2014).Introducción a la Ingeniería Industrial, Material Instruccional de Apoyo. Caracas: UNA

•• La mano derecha toma la pólvora y la deposita en el casquillo, la mano


izquierda sostiene el casquillo (2 seg).
•• La mano izquierda sostiene el casquillo con pólvora y la mano derecha
obtiene la bala (0.5 seg).
•• La mano izquierda sostiene el casquillo y la mano derecho coloca la bala
y presiona (1.5 seg).
•• Obténgase el diagrama del Operador Bimanual actual.

Solución:

Para la elaboración del diagrama se procederá inicialmente de la misma forma que los
dos anteriores, es decir, identificándolo y colocando información relevante para entender lo
que contiene éste, como la siguiente: el proceso en estudio, número de diagrama, el nombre
de la persona que elaboró el diagrama, la fecha de elaboración, entre otros.

También en este diagrama los símbolos forman parte del formato y están ubicados en
su centro. La descripción de las actividades de la mano derecha, se encuentran en el lado
derecho del diagrama y las de la mano izquierda en el lado izquierdo. Luego se señalan los
símbolos en función de las actividades efectuadas por las manos, uniéndolos seguidamente
con una línea o flecha. Al final se ubica un cuadro resumen con el número de actividades
ejecutadas por ambas manos, así como el área de trabajo (opcional).

200
Sánchez Marchán, Nanci. (2014).Introducción a la Ingeniería Industrial, Material Instruccional de Apoyo. Caracas: UNA

Diagrama Bimanual

Proceso: recargo de cartucho Fecha: enero de 2014


Elaborado por: Pablo Leiva N° de diagrama: 35
Descripción: trabajo completamente manual Hoja: 1 de 1
Método: Actual ( X ) Propuesto ( )
   
Descripción de las Descripción de las  
operación  

Inspección  

Transporte  

Demora  

Sostenimiento  

operación  

Inspección  

Transporte  

Demora  

Sostenimiento  
actividades de la actividadades
mano izquierda de la mano derecha  

Toma fulminante             Toma casquillo

Coloca fulminante                   Sostiene casquillo

            Mueve casquillo a
Toma pólvora mesa

                  Espera
Coloca pólvora en
pesadora
Toma pólvora y             Sostiene casquillo
coloca en casquillo
            Toma bala
Sostiene casquillo

Sostiene casquillo                     Coloca bala    


 
                         
Cuadro resumen   Área de trabajo  
Actividad M.  Actual   M.  Propuesto    
Izq Der   Izq   Der    
5 3      
         
  1        
  1        
  2 2      
Total   7 7      
 

Finalizado el diagrama se revisan los Therbligs eficientes y los ineficientes, con la finali-
dad de eliminar los ineficientes y proponer otro diagrama de ser necesario.

201
Sánchez Marchán, Nanci. (2014).Introducción a la Ingeniería Industrial, Material Instruccional de Apoyo. Caracas: UNA

MEDICIÓN DEL TRABAJO


La medición del trabajo implica un estudio de tiempo que determine con la mayor exac-
titud posible, el lapso necesario para llevar a cabo una tarea determinada. Para realizar este
estudio es preciso observar y medir un número abundante de ciclos, tanto del tiempo de pro-
ceso, como del tiempo manual de una operación, que permita calcular un tiempo promedio
correcto.

La medición del trabajo se considera como la parte cuantitativa del estudio del trabajo,
indica el resultado del esfuerzo físico desarrollado por un operario en función del tiempo; es
decir, la utilización de técnicas para establecer el tiempo que invierte un trabajador califi-
cado en llevar a cabo una tarea definida, ejecutándola según una norma de cumplimiento
preestablecida, siguiendo un ritmo normal y un método predeterminado. Se estima el ritmo
del operador ajustándolo al tiempo promedio, dependiendo si este ritmo ha sido rápido o más
lento de lo normal.

Trabajador calificado

Un trabajador es calificado cuando demuestra tener las aptitudes físicas necesarias,


inteligencia e instrucción, además de las destrezas y conocimientos fundamentales para
efectuar el trabajo en curso, a un ritmo normal y según normas satisfactorias de seguridad
y calidad.

El ritmo al cual opera un trabajador tiene que ver con la velocidad de trabajo. Este ritmo
puede ser lento, normal o rápido y se calcula en porcentajes, como se muestra en la figura
40:

Ritmo normal
Ritmo lento Es una velocidad de trabajo, que
Es una velocidad de trabajo por se puede mantener a lo largo de
debajo de lo normal, se califica en una jornada sin producir una fatiga
porcentajes menores al 100%, excesiva, se califica con un 100 %
95%, 90%,85%, etc.

RITMO

Ritmo rápido
Es una velocidad de trabajo, por
encima de lo normal, se califica en
porcentajes superiores al 100%,
105%, 110%,115%, etc.

Figura 40. Ritmo de trabajo de un trabajador calificado.

202
Sánchez Marchán, Nanci. (2014).Introducción a la Ingeniería Industrial, Material Instruccional de Apoyo. Caracas: UNA

USOS DE LA MEDICIÓN DEL TRABAJO


De manera general revela la existencia y las causas del tiempo improductivo. Se puede
usar para dos objetos principales: en primer lugar, se puede emplear retrospectivamente
para valorar el rendimiento en el pasado. En segundo lugar, se puede utilizar mirando hacia
adelante, para fijar los objetivos futuros.

A continuación se esbozan algunos usos importantes de la medición del trabajo: 


•• Comparar la eficiencia de varios métodos, seleccionando aquel que lleve
menos tiempo.
•• Repartir el trabajo concerniente al proceso de manera equitativa, dentro
de los equipos de trabajo (Balanceo de Línea)
•• Determinar mediante diagramas actividades múltiples, el número de má-
quinas que puede atender un operario.
•• Obtener información sobre equipos y mano de obra, necesarios para
cumplir el plan de trabajo y aprovechar la capacidad de producción.
•• Presupuestos de ofertas, ventas y plazos de entrega.
•• Fijar normas sobre uso de maquinaria y desempeño de mano de obra,
como base de sistemas de incentivos.
•• Controlar costos de mano de obra, fijar y mantener costos estándar.

La medición del trabajo proporciona la información básica necesaria para llegar


a organizar y controlar las actividades de la empresa donde interviene el factor
tiempo.

PROCEDIMIENTO BÁSICO PARA LA MEDICIÓN DEL TRABAJO


Etapas necesarias para efectuar sistemáticamente la medición del trabajo:

1. Seleccionar el trabajo que va a ser objeto de estudio

2. Registrar todos los datos relativos a las circunstancias en las cuales se efectúa
el trabajo, es decir, a los métodos y elementos de actividad que contempla.

3. Examinar los datos registrados y el detalle de los elementos con espíritu crítico,
para verificar si se utilizan los métodos y movimientos más eficaces, con el pro-
pósito de separar los elementos improductivos de los productivos.

4. Medir la cantidad de trabajo de cada elemento, expresándola en tiempo, median-


te la técnica más apropiada de medición del trabajo.

5. Compilar el tiempo tipo de la operación, previendo suplementos para breves des-


cansos, necesidades personales, etc.

203
Sánchez Marchán, Nanci. (2014).Introducción a la Ingeniería Industrial, Material Instruccional de Apoyo. Caracas: UNA

6. Definir con precisión la serie de actividades y el método de operación a los que


corresponde el tiempo computado, notificar que ése será el tiempo tipo para las
actividades y métodos especificados.

Estas etapas sólo tendrán que seguirse en su totalidad cuando se desee fijar tiempos
estándar, también llamados tiempos tipo. Si la medición del trabajo se utiliza para averiguar
los tiempos improductivos antes o en el curso de un estudio de métodos o para comparar la
eficacia de varios métodos posibles, probablemente basten las cuatro primeras etapas.

Tiempo estándar o tipo no es lo mismo que tiempo real. Es el tiempo promedio en


que una tarea puede ser completada por una persona competente en su trabajo.
No por el mejor trabajador, sino por un obrero medio. Incluye un margen adecua-
do para relajación y contingencias.

TÉCNICAS DE MEDICIÓN DEL TRABAJO


Las técnicas de medición más comunes incluyen comparaciones, registros históricos,
estimaciones, estudio de tiempos cronometrados, muestreo del trabajo, sistemas de tiempos
predeterminados, y técnicas estadísticas.

Se pueden clasificar estas técnicas de la siguiente forma:

Intuitivas ( Basada en la experiencia):


• Comparaciones.
• registros históricos.
• Estimaciones.
• Calificación objetiva.
Estudio de Tiempo con cronómetro Técnicas
Muestreo del trabajo Directas

Estándares de tiempos predeterminados


• Sistemas de Normas de Tiempo Predeterminadas Técnicas
• (NTPD: MTM,MOST, MODATPS, entre otros) Indirectas

Técnicas estadísticas.

Técnicas intuitivas

Comparaciones: las comparaciones se generan de la experiencia y se utilizan para


cotejar el trabajo de un área con el mismo trabajo realizado en otra área, es decir, lograr una
referencia para evaluar estudios similares.

204
Sánchez Marchán, Nanci. (2014).Introducción a la Ingeniería Industrial, Material Instruccional de Apoyo. Caracas: UNA

Registros históricos: los registros históricos son los documentos que se tienen sobre
experiencias pasadas. Por ejemplo: las horas de mano de obra necesarias para producir una
determinada cantidad de trabajo, se indican en los registros históricos del equipo y se utilizan
más tarde cuando se realiza la misma operación.

Estimaciones: supone que un trabajador calificado puede determinar una aproxima-


ción razonable del tiempo que en realidad toma realizar un trabajo. Las estimaciones pueden
ser combinaciones de comparaciones e historias internas o manuales de estimación, como
las que frecuentemente se utilizan en proyectos de construcción.

Calificación objetiva: es un procedimiento según el cual se califica el ritmo y la dificul-


tad del trabajo. Bajo este procedimiento, el operador se califica exactamente de la misma for-
ma que un método anterior; pero posteriormente se selecciona un segundo factor de ajuste,
que toma en cuenta la dificultad del trabajo.

Las comparaciones, los registros históricos y las estimaciones, se basan en tiem-


pos reales y pueden ocasionar error e inconsistencia. Tienden a perpetuar retra-
sos experimentados con anterioridad.

ESTUDIO DE TIEMPO CON CRONÓMETRO


Un estudio de tiempos cronometrado se lleva a cabo cuando:
•• Se va a ejecutar una nueva operación, actividad o tarea ya estandariza-
da.
•• Se presentan quejas de los trabajadores o de sus representantes sobre
el tiempo de una operación.
•• Se encuentran demoras causadas por una operación lenta, que ocasiona
retrasos en las demás operaciones.
•• Se pretende fijar los tiempos estándar de un sistema de incentivos.
•• Se encuentren bajos rendimientos u excesivos tiempos muertos de algu-
na máquina o grupos de máquinas.

Existen dos métodos de cronometraje para registrar los tiempos elementales durante la
medición del trabajo, estos son:
•• Cronometraje continuo o acumulativo. Se deja correr el cronometro mien-
tras dura el estudio, se pone en marcha al principio del primer elemento
del primer ciclo, al final de cada elemento se apunta la hora que marca
el cronometro, y los tiempos de cada elemento se obtienen haciendo las
respectivas restas después de terminar el estudio.
•• Cronometraje con vuelta a cero. Los tiempos se toman directamente al
acabar cada elemento se hace volver el segundero a cero y se le pone
de nuevo en marcha inmediatamente para cronometrar el elemento si-
guiente, sin que el mecanismo del reloj se detenga en ningún momento

205
Sánchez Marchán, Nanci. (2014).Introducción a la Ingeniería Industrial, Material Instruccional de Apoyo. Caracas: UNA

En ambos métodos cronometrados, antes de realizar el estudio de tiempos, es nece-


sario considerar un número de lecturas, lo que permitirá que el estudio sea confiable. Estas
lecturas se obtienen aplicando la siguiente fórmula:

s t2
n=
kx

t = Valor en tabla “t student”


s = Desviación estándar
k = Porcentaje de error
x = Tiempo medio
n = Número de ciclos a observar

Al efectuar un estudio de tiempo, se seleccionan los elementos de la operación, de tal


manera que al menos un grupo de ellos se correspondan con el trabajo del operador a un
nivel normal de actuación; obteniendo así un factor de corrección que se aplica a todos los
demás elementos:

MUESTREO DEL TRABAJO


Es una técnica de medición que se ejecuta al reunir una serie de observaciones ins-
tantáneas a un grupo de máquinas, procesos u operarios, realizadas al azar durante un
período. Estas observaciones se pueden clasificar en actividades tales como trabajo directo,
recepción de instrucciones, transporte de material, demoras, entre otras. Cada observación
registra lo que está ocurriendo en ese instante. El porcentaje de observaciones registradas,
para una actividad particular o demora es una medida del porcentaje de tiempo; durante el
cual éstas ocurren.

El método de muestreo de trabajo tiene sus ventajas y desventajas. Entre las ventajas
se encuentran las siguientes:
•• No requiere observación continua por parte de un analista durante un
período largo.
•• El tiempo de trabajo de oficina disminuye.
•• El total de horas-trabajo a desarrollar por el analista es generalmente
mucho menor.
•• El operario no está expuesto a largos períodos de observaciones riguro-
sas.
•• Las operaciones de grupos de operarios pueden ser estudiadas fácil-
mente por un solo analista.

206
Sánchez Marchán, Nanci. (2014).Introducción a la Ingeniería Industrial, Material Instruccional de Apoyo. Caracas: UNA

Una desventaja es que las observaciones deben continuar mientras la medición del
desempeño así lo exija. Por lo general se asigna un número de observaciones de tiempo
completo de acuerdo con las exigencias del proyecto, considerando el tamaño de las insta-
laciones y la frecuencia de observación deseada.

Ésta técnica es muy útil para investigar las proporciones del tiempo total dedicada a las
diversas actividades que componen una tarea o trabajo. Los resultados del muestreo sirven
para determinar tolerancias o márgenes aplicables al trabajo, para evaluar la utilización de
las máquinas y para establecer estándares de producción.

ESTÁNDARES DE TIEMPOS PREDETERMINADOS (NTPD)


Los estándares de trabajo se pueden crear por medio de valores de tiempo predeter-
minado, que establezcan el tiempo normal necesario para realizar los movimientos manua-
les básicos y los ciclos de proceso de todos los trabajos. Se utilizan técnicas como las de
Maynard’s Operation Sequence Techniques (MOST), Methods Time Measurement (MTM),
Modular Arrangement of PTS (MODAPTS), entre otros; con el fin de automatizar el proceso
de establecer estándares mediante el uso de una base de datos de tiempos predetermina-
dos.

Al estudiar esta técnica se supone que cada movimiento ya ha sido examinado y se le


ha fijado un tiempo de actuación. La suma de los tiempos de todos los movimientos emplea-
dos en una operación, da el tiempo valorado para ésta.

Si en lugar de obtener el tiempo valorado para toda la operación se determina tan sólo
el de un elemento, es posible comparar este tiempo, con el que emplea actualmente el tra-
bajador y determinar el nivel de actuación de ese trabajador. Como el nivel de actuación se
expresa en porcentaje, interviene el factor de corrección para todos los demás elementos.
Para aplicar este método, debe suponerse que el nivel de actuación del trabajador es cons-
tante en la ejecución de toda la operación.

El factor de corrección puede expresarse como: P= Ft/tm

En donde:

P = Factor de actuación.
Ft = Tiempo de movimiento fundamental.
tm =Tiempo elemental medio observado en los mismos elementos que se haya usa-
do en Ft.

Tal vez una de las mayores objeciones a la aplicación de esta técnica es el tiempo que
se requiere para construir un diagrama bimanual de los elementos seleccionados, en el es-
tablecimiento de los tiempos de los movimientos básicos.

207
Sánchez Marchán, Nanci. (2014).Introducción a la Ingeniería Industrial, Material Instruccional de Apoyo. Caracas: UNA

ENFOQUES ESTADÍSTICOS
Los enfoques estadísticos resultan de técnicas matemáticas modernas tales como: frac-
cionar, muestreo de Monte Carlo, regresión lineal o la simulación por computadora, entre
otros; utilizando un modelo de precisión estadístico a través de la medición directa de los
datos. La técnica de fraccionar se puede utilizar en combinación con los tiempos predeter-
minados (medición indirecta) para producir un sistema que sea rápido y preciso, cuando se
aplique al trabajo en pequeñas cantidades pero con ciclos largos.

ESTÁNDAR DE TIEMPO
Los resultados principales de la medición del trabajo es un estándar de producción,
llamado también tiempo tipo o estándar de tiempo.

Un tiempo tipo o estándar de tiempo se puede definir como:


Una cantidad de tiempo que se requiere para ejecutar una tarea o actividad, cuando un
operador capacitado trabaja desarrollando una velocidad normal que pueda mantener día
tras día, incluyendo en ello los síntomas de fatiga; bajo los procedimientos de un método
preestablecido.

Un estándar se puede expresar en dos formas: ya sea como el tiempo requerido por
unidad de producción o el reciproco, producción por unidad de tiempo. En el tiempo tipo o
estándar están incluidos los tiempos de los elementos cíclicos: repetitivos, constantes, varia-
bles; así los elementos causales o contingentes que fueron observados durante el estudio de
tiempos, a estos tiempos ya valorados se le agregan los suplementos siguientes: personales,
por fatiga y especiales.

La figura mostrada indica la estructura de un tiempo tipo o estándar.

Tiempo Observado Calificación

Tiempo Normal (1 + Tolerancia Total)

Tiempo tipo o estándar

Figura 41. Estructura de un tiempo tipo o estándar.

Características de un estándar de tiempo:

208
Sánchez Marchán, Nanci. (2014).Introducción a la Ingeniería Industrial, Material Instruccional de Apoyo. Caracas: UNA

•• Un estándar es normativo, es decir, establece la cantidad de tiempo


requerido para trabajar bajo ciertas condiciones.
•• Un estándar también requiere que se preestablezca un método para
el trabajo o actividad. Generalmente el mejor método, el cual debe po-
nerse por escrito.
•• Por último, un estándar requiere que un operador capacitado, rea-
lice el trabajo a un paso normal, es decir aquél que desarrolle un ritmo
que pueda ser seguido por la mayoría de los trabajadores en su jornada
de trabajo diaria.

Aplicaciones del tiempo estándar:

•• Permite lograr una mejor planeación de la producción, eliminando en ella


las conjeturas o adivinanzas.
•• Es una herramienta que contribuye a establecer estándares de produc-
ción precisos y justos.
•• Apoya en el establecimiento de las cargas de trabajo.
•• Ayuda a formular un sistema de costos estándar.
•• Proporciona costos estimados, es decir presupuestar los costos de artí-
culos que se planea producir.
•• Proporciona bases sólidas para establecer sistemas de incentivos y su
control.
•• favorece el entrenamiento de nuevos trabajadores.

Cálculo del tiempo tipo o estándar:

Para calcular el estándar de tiempo se debe asignar un suplemento o margen al trabaja-


dor para que el estándar resultante sea justo y fácil de mantener por la actuación del operario
medio, a un ritmo normal y continuo.

Un suplemento es:
El tiempo que se concede al trabajador con el objeto de compensar los retrasos, las de-
moras y elementos contingentes que son partes regulares de la tarea.

Los suplementos pueden ser:

Asignables al trabajador:

1. Contantes:
•• Satisfacer necesidades personales (idas al bebedero o baño, entre otros)
•• Reponerse de la fatiga básica del trabajo (descanso)

209
Sánchez Marchán, Nanci. (2014).Introducción a la Ingeniería Industrial, Material Instruccional de Apoyo. Caracas: UNA

2. Variables
•• Trabajar de pie, postura anormal, uso de la fuerza física, intensidad de
luz, calidad del aire, tensión visual, auditiva y/o mental, monotonía mental
y/o física, etc.

No asignables al trabajador:
•• Por contingencia: averías, faltas de suministro de materiales, manteni-
miento, etc.

Pasos preliminares al cálculo del estándar de tiempo:


1. Obtener y registrar información de la operación.
2. Descomponer la tarea y registrar los elementos
3. Tomar las lecturas.
4. Nivelar el ritmo de trabajo.
5. Calcular los suplementos de estudios de tiempos.

Una vez superados los pasos preliminares se procede a calcular el estándar de tiempo
como sigue:

a) Se analiza la consistencia de cada elemento, de acuerdo con las consideraciones


siguientes:
•• Si las variaciones son debidas a la naturaleza de los elementos se
conservan todas las lecturas.
•• Caso contrario, la lectura anterior o posterior donde se observa la va-
riación de ambas son consistentes, la inconsistencia en el elemento
estudiado se deberá a la falta de habilidad del trabajador. Si un gran
número de observaciones son consistentes se pueden eliminar las
variaciones extremas.

b) En cada uno de los elementos se suman las lecturas que han sido consideradas
como consistentes.
c) Se registra el número de lecturas que han sido consideradas para cada elemento.
d) Se divide para cada elemento, la suma de las lecturas entre el número de lecturas
consideradas. El resultado es el tiempo promedio por elemento.
Te = SXi / n

210
Sánchez Marchán, Nanci. (2014).Introducción a la Ingeniería Industrial, Material Instruccional de Apoyo. Caracas: UNA

e) Se multiplica el tiempo promedio (Te) por el factor de valoración. Esta cifra debe
aproximarse hasta el milésimo de minuto, obteniéndose el tiempo base elemen-
tal:
Tn = Te (Factor de valoración en %)
f) Al tiempo base elemental se le suma la tolerancia por suplementos concedidos,
obteniéndose el tiempo normal o concedido por elemento:
Tt = Tn (1 + Tolerancias)
g) Se calcula la frecuencia por operación o pieza, de cada elemento cíclico o contin-
gente.
h) Se multiplica el tiempo concedido elemental por la frecuencia obtenida del ele-
mento. A este producto se le denomina tiempo total concedido.
i) Se suman los tiempos concedidos para cada elemento y se obtiene el tiempo tipo
o estándar por operación/ pieza, etc.
j) Al efectuarse el cálculo del tiempo tipo debe tenerse en cuenta las siguientes
consideraciones:
•• Si debe concederse el tiempo de preparación y retiro
•• El factor interferencia cuando se presenta un ciclo de trabajo estu-
diado.

La mayoría de las mejoras resultantes de la medición del trabajo radican en los


estudios fundamentales de métodos, que anteceden a los estudios de tiempo en
sí.

Ejemplo Ilustrativo de un estudio de tiempos completo hasta obtener el


estándar, utilizando la técnica medición y observaciones directas con cro-
nometro.
En uno de los departamentos de un laboratorio de pruebas de materiales se
determina la resistencia a la compresión de cilindros de concreto. Esos cilindros
son tomados del lugar de la construcción e indican la calidad del concreto usado.
Los constructores los envían al laboratorio, en donde se conservan en un cuarto
húmedo bajo temperatura y humedad controladas. Después de un período de 7
días, los cilindros se rompen para ver si tienen la resistencia especificada. Antes
de romper los cilindros, a éstos se le colocan unas tapas. Esta tarea consiste
en poner un compuesto químico líquido caliente en un molde, en el extremo del
cilindro. El líquido seca rápidamente formando una tapa muy dura. La finalidad
de las tapas es dejar una superficie lisa en los extremos del cilindro, para la apli-
cación uniforme de la fuerza que romperá el concreto.
Se quiere realizar un estudio de tiempos de la tarea “colocar tapas”, para calcular
el costo de mano de obra requerido de colocar las tapas para probar los cilindros.

211
Sánchez Marchán, Nanci. (2014).Introducción a la Ingeniería Industrial, Material Instruccional de Apoyo. Caracas: UNA

Solución:

Paso 1: definir los elementos que componen la tarea.

1. Sujetar abrazadera al cilindro.


2. Vaciar compuesto caliente en el molde.
3. Colocar cilindro en el molde.
4. Dejar que la tapa se enfríe en el molde.
5. Poner el cilindro en la mesa.
6. Vaciar compuesto caliente en el molde.
7. Colocar el otro extremo del cilindro en el molde.
8. Dejar que la tapa se enfríe en el molde.
9. Poner cilindro en la mesa y retirar abrazadera.

Paso 2: usando un cronómetro, medir el tiempo de cada elemento (aproximadamente


10 veces), se observan en la tabla mostrada.

Paso 3: calcular el tiempo medio de cada elemento.

Te
Elemento 1 2 3 4 5 6 7 8 9 10
(medio)
1. Sujetar
abrazadera al 0,08 0,09 0,09 0,1 0,08 1,01 0,09 0,08 0,09 0,09 0,09
cilindro.
2. Vaciar com-
puesto caliente 0,25 0,24 0,31 0,28 0,3 0,27 0,33 0,25 0,31 0,32 0,29
en el molde.
3. Colocar cilin-
0,18 0,19 0,18 0,17 0,19 0,19 0,19 0,18 0,18 0,19 0,18
dro en el molde.
4. Dejar que la
tapa se enfríe en 0,51 0,55 0,55 0,61 0,6 0,51 0,54 0,53 0,57 0,59 0,56
el molde.
5. Poner el cilin-
0,16 0,15 0,15 0,16 0,18 0,17 0,17 0,16 0,15 0,17 0,16
dro en la mesa.
6. Vaciar com-
puesto caliente 0,28 0,29 0,31 0,29 0,3 0,27 0,31 0,25 0,26 0,26 0,28
en el molde.
7. Colocar el otro
extremo del cilin- 0,19 0,18 0,2 0,19 0,2 0,21 0,2 0,2 0,19 0,2 0,20
dro en el molde.
8. Dejar que la
tapa se enfríe en 0,54 0,6 0,51 0,53 0,55 0,52 0,58 0,55 0,61 0,56 0,56
el molde.
9 Poner cilindro
en la mesa y reti- 0,38 0,36 0,41 0,42 0,49 0,52 0,41 0,44 0,58 0,39 0,44
rar abrazadera.
Tiempo Total 2,75

212
Sánchez Marchán, Nanci. (2014).Introducción a la Ingeniería Industrial, Material Instruccional de Apoyo. Caracas: UNA

Paso 4: calcular el tiempo total de la tarea.

Para ello se requiere del tiempo normal y el tiempo estándar de la tarea “poner tapas”

En este caso, cada elemento de la tarea se califica por separado, es decir, en cada
elemento el operador mostró un ritmo de trabajo diferente. El factor de calificación que el
observador asignó en cada elemento se puede observar en la siguiente tabla:

Tabla 26. Factor de calificación de los elementos de una tarea según ritmo de trabajo
Elemento Te (min.) Factor de Tiempo Tolerancias:
calificación normal
(min.)
1. Sujetar abrazadera al Necesidades perso-
cilindro. 0.09 1.2 0.11 nales: 5%
2. Vaciar compuesto ca-
liente en el molde. 0.29 1.1 0.32 Manejo de los cilin-
3. Colocar cilindro en el dros de 30 lbs. y del
molde. 0.18 1 0.18 material caliente: 8%
4. Dejar que la tapa se
enfríe en el molde. 0.56 1 0.56
Interrupciones por
5. Poner el cilindro en la
demoras: 7%
mesa. 0.16 1 0.16
6. Vaciar compuesto ca-
liente en el molde. 0.28 1.1 0.31 Tolerancia total =
7. Colocar el otro extremo 5% + 8% + 7% = 20%
del cilindro en el molde. 0.20 1 0.20
= 0.2
8. Dejar que la tapa se
enfríe en el molde. 0.56 1 0.56
9. Poner cilindro en la
mesa y retirar abrazadera. 0.44 1.2 0.53

Tiempo normal de la tarea = 2.93

Tiempo estándar: TE = TN (1 + Tolerancia Total) TE = 2.93 (1 + 0.2)


= 3.52 minutos.

Para realizar el estudio de tiempo es necesario establecer el tiempo que invierte un tra-
bajador calificado en llevar a cabo una tarea definida, como se mencionó anteriormente, se
pueden utilizar para ello diferentes técnicas para realizar el estudio. En este texto se abordó
sólo la técnica de medición y observación directa, por ser una de las más sencilla y utilizada.

A continuación se puede observar la figura 42, la cual muestra un resumen de las téc-
nicas expuestas:

213
Sánchez Marchán, Nanci. (2014).Introducción a la Ingeniería Industrial, Material Instruccional de Apoyo. Caracas: UNA

Figura 42. Organización de las técnicas de la medición del trabajo.

Se puede concluir que en toda empresa, el análisis sistemático del estudio del trabajo,
permite la obtención de estándares que se utilizan para mejorar los aspectos relacionados
con las actividades operativas y gerenciales de la producción.

214
Sánchez Marchán, Nanci. (2014).Introducción a la Ingeniería Industrial, Material Instruccional de Apoyo. Caracas: UNA

CALIDAD EN LA EMPRESA

Objetivo:
Explicar la importancia de la calidad en la producción de bienes y servicios,
haciendo énfasis en la mejora continua de productos y procesos, para lograr la
satisfacción de las expectativas de los clientes.

•• Concepto y evolución de la calidad.


•• Gestión de la calidad.
•• Mejoramiento de la calidad.

CONCEPTO Y EVOLUCIÓN DE LA CALIDAD


El concepto de calidad ha evolucionado desde que en 1924 W. A. Shewhart (físico,
ingeniero, y estadístico) estadounidense, de la Bell Telephone, publicara “Economic Control
of Quality of Manufactured Products” (Control Económico de la Calidad de los Productos
Manufacturados).

Seguidamente a este hecho se produjo la producción en cadena, surgiendo el primer


problema de calidad, las tolerancias dentro de las especificaciones. En consecuencia los
elementos defectuosos se rechazaban y posteriormente eran reprocesados o, sencillamen-
te, desechados como desperdicios. En cualquiera de los dos casos, los costos provocados
se elevaban.

De este modo surge la primera definición de calidad: conformidad con las especifica-
ciones. Las tolerancias se utilizan como una guía de fabricación de productos, al mantener
un estándar de especificaciones para los fabricantes. Cuando la conformidad es alta, corres-
ponderá un número menor de desechos y reprocesos, lo cual reduce el costo del proceso
productivo y del producto. Esto puede significar un mayor margen comercial o un menor
precio de venta, con el consiguiente aumento de la competitividad en el mercado. De ahí la
importancia de aplicar el control de calidad, como procedimiento para asegurar el ajuste
a las especificaciones de los productos y dispositivos.

Pero la idea de calidad como ajuste a las especificaciones, únicamente era útil en un
entorno en el que la relación demandaba una capacidad superior a la unidad, es decir, se
tenía asegurada la venta del producto. Esta situación era la que se daba en los años 50 y
60. En la medida en que fue cristalizando un mayor equilibrio entre estos dos términos, ven-
der se tornó más difícil, siendo necesario emplear otras estrategias. Por ello, la función de
Marketing alcanzó un importante desarrollo, como un medio para colocar productos que se
enfrentaban a una mayor competencia (años 70).

215
Sánchez Marchán, Nanci. (2014).Introducción a la Ingeniería Industrial, Material Instruccional de Apoyo. Caracas: UNA

En la actualidad, pueden encontrarse mercados saturados junto a consumidores cada


vez más exigentes. Ya no basta con producir bien sino que además se debe dar a los clientes
lo que esperan. Ajustarse, en suma, a sus necesidades y expectativas. Por esta razón es
preciso desarrollar un nuevo concepto de calidad que tenga en cuenta el cliente al que va
dirigido el producto y, más que tenerlo en cuenta, lo sitúe en el eje central de las actividades
de la organización.

Sobre la base de la explicación anterior se puede definir calidad como:

El conjunto de particularidades inherentes a un producto o servicio que permiten lograr


que se cumpla con las especificaciones de diseño, así como con la función para la cual
fue creado, logrando con ello la satisfacción de las expectativas de los clientes.

Actividad propuesta
Seleccionando las palabras que considere necesarias del grupo que se presenta
a continuación: propiedades, percepción, grado, conformidad, cliente, capacidad,
especificaciones, satisfacen, productos, servicios, herramienta, sustenta, exce-
lencia. Construya una definición de calidad:

Son numerosas las definiciones que en la actualidad existen de calidad, en este sentido
se presenta una tabla en la que se sintetizan de forma cronológica las más importantes con
su autor:

216
Sánchez Marchán, Nanci. (2014).Introducción a la Ingeniería Industrial, Material Instruccional de Apoyo. Caracas: UNA

Tabla 27. Evolución de la definición de calidad


Evolución de la definición de calidad
W. Eduards Deming Producto. Conformidad con especificaciones a bajo costo
(1900-1993, Estados Unidos) que satisfaga al cliente y aumente la productividad.

Joseph M. Juran Administración de la calidad. Se incluye: los problemas de


organización, comunicación, coordinación de funciones y el
(1904 - 2008, Rumania) elemento humano.
Philip B. Crosby Cero defectos. Cumplir con los requisitos, un error prevenido
(1926 – 2001, Estados Unidos) no necesita reparación.

Kauoro Ishikawa Proceso. La calidad tiene que ser construida en cada diseño
y cada proceso. No puede ser creada por medio de la inspec-
(1915-1989, japón) ción.
Armando V. Feigenbaun Calidad total. Abarca toda la organización e involucra la
(1922-2012, Estados Unidos) puesta en práctica de actividades orientadas hacia el cliente.

Genichi Taguchi Función de pérdidas. Son diferentes a las causadas por la


función intrínseca del servicio y está ocasionada por la varia-
(1924-2012, Japón) bilidad.
Shigeo Shingo JIT – Poka Yoke (Justo a Tiempo – A prueba de errores). El
proceso se debe detener siempre que ocurra un defecto, se
(1909-1999, Japón) debe identificar la fuente u origen para prevenir su recurrencia.
Masaaki Imai Kaisen. Significa mejoramiento continuo de trabajadores y
gerentes en la vida personal, en la casa, la vida social y en el
(1930-2012, Japón) trabajo.
Claus Moller Personas. “El mejor aspecto para comenzar a desarrollar la
calidad en una compañía u organización es en la actuación y
(1942-2012 ,Dinamarca) actitud de los individuos, con respecto a ella”

Se considera en este libro el análisis del texto “Gestión de la Calidad y Diseño de Orga-
nizaciones” de Moreno, Luzón y Peris (2000). Los cuales se apoyaron en las investigaciones
de Garvin (1988), Reeves y Bednar (1994), quienes sintetizaron muchas de las definiciones
reflejadas en la tabla 27, para enmarcar la definición de calidad en los siguientes enfoques:

1. Calidad como cumplimiento con las especificaciones.


2. Calidad como satisfacción de las expectativas del cliente.
3. Calidad como relación valor-precio.
4. Calidad como excelencia.

1. Calidad como conformidad con especificaciones

El crecimiento de la fábrica y la producción en masa aparecen con la llegada de la fase


monopolista del capitalismo a finales del siglo XIX y comienzos del XX, en esta fase el obje-
tivo de calidad de las empresas manufactureras era conseguir la producción a gran escala
de productos estandarizados e intercambiables y sin defectos.

217
Sánchez Marchán, Nanci. (2014).Introducción a la Ingeniería Industrial, Material Instruccional de Apoyo. Caracas: UNA

Esta visión de la calidad se fundamentó en los estudios de Taylor y Ford (Ver tabla Nº
2) cuyo aporte fue el control estadístico de procesos, el cual trata de reducir al mínimo los
altos costos como resultado de la inspección de toda la producción. En esta era la calidad
fue medida por indicadores cuantitativos para expresar la conformidad de los productos con
las especificaciones o requisitos. Bajo estos conceptos fue relativamente fácil gestionar la
calidad para medir y controlar con mayor exactitud los resultados.

Una de las desventajas de este enfoque es que los clientes no están al tanto de los
elementos técnicos y específicos; lo cual dificulta la aplicación del concepto acorde con la
insistencia de sus autores (Deming, Juran y Crosby, ver tabla Nº 27). Otra desventaja del en-
foque, es la de estar fundamentado sólo en la eficiencia y no estima debidamente la eficacia.

2. Calidad como satisfacción de las expectativas del cliente

Las fuerzas productivas que genera la lucha competitiva por los mercados han exigido
pasar de un enfoque en la gestión de la calidad basado en el cumplimiento de las especi-
ficaciones y la estandarización, hacia un enfoque centrado en la eficacia que contempla el
movimiento de los mercados sobre la base del cambio en las expectativas de los clientes.
Por ello, el productor no vende lo que produce según sus estándares, sino que produce lo
que se vende según las exigencias de los clientes. Así la calidad evolucionó desde el prin-
cipio de la conformidad con las especificaciones hacia el cumplimiento de las expectativas
de los clientes, propuesto por Deming (Ver tabla Nº 27). Las expectativas de cualquier ser
humano están matizadas por una fuerte carga subjetiva, lo que hace difícil su medición y
ponderación; siendo éste el principal obstáculo a la aplicación de este concepto de la calidad.

3. Calidad como relación valor-precio

Varios autores Abbott, Feigenbaum, Ishikawa (Ver tabla 27), defienden que aspectos
como condiciones de competencia, los conceptos de valor y precio estén incluidos en la
definición de calidad.

La calidad se entiende aquí como un concepto subordinado y relativo, lo que implica


que se tratará de obtener la mejor calidad posible a un precio dado. Agregando un conjunto
de atributos como confort, durabilidad, fiabilidad, seguridad, entre otros, que se traducen en
indicadores que posibilitan el establecimiento de comparaciones efectivas entre productos,
servicios y experiencias de consumo distintas. Este enfoque induce a la empresa a orientar
coherentemente su eficiencia económica interna y su eficacia con respecto a su entorno
competitivo.

La desventaja de la calidad en su relación valor-precio se evidencia debido a que los


componentes del valor de un producto o un servicio, particularmente aquellos que forman
parte de las preferencias del consumidor, así como la ponderación de cada uno de ellos,
son difíciles de precisar. Sumando también a esta situación que los componentes del valor
son variables en dependencia de los productos y/o servicios, las coyunturas del mercado,
entre otros.

218
Sánchez Marchán, Nanci. (2014).Introducción a la Ingeniería Industrial, Material Instruccional de Apoyo. Caracas: UNA

4. Calidad como excelencia

Esta última definición es la más completa y factible para trabajar bajo los conceptos de
la calidad.

Fig. 43. Perspectivas externa e interna integradas en una perspectiva global.


Fuente. Adaptado de Moreno – Luzón-Peris (2001).

El concepto de calidad como excelencia es el más general e integrador de las formas


de entender la calidad. La calidad como excelencia es un objetivo que permite y exige in-
corporar el compromiso de todos los miembros de la organización, y que, si es reconocida
por el mercado, será fuente de ventaja competitiva (Ver tabla 27). Este concepto se aplica a
aquellos productos y servicios que reúnen los máximos estándares de calidad en sus dife-
rentes características.

El concepto de calidad implica aquí no admitir, en la realización de cualquier tarea, todo


aquello que no sea lo mejor. Supone la inversión de las mejores habilidades y materiales en
la realización de una tarea, para alcanzar el mejor resultado posible.

Actividad propuesta
Estudiados los enfoques de calidad, resuma los argumentos esenciales de cada
uno de ellos y establezca las diferencias fundamentales que se perciben entre
estos enfoques.

Ejemplo Ilustrativo
Seguidamente se presentan las expectativas de calidad que puede tener un clien-
te, cuando compra un producto (vehículo) o utiliza el servicio que presta un banco
o un taller automotor.

219
Sánchez Marchán, Nanci. (2014).Introducción a la Ingeniería Industrial, Material Instruccional de Apoyo. Caracas: UNA

Producto/
Expectativa del Cliente
Servicio
• El cliente es tratado con cortesía.
• Se terminan y completan cada una de las transacciones del cliente
con precisión.
Servicio prestado
• Los estados de cuenta del cliente reflejan con exactitud sus tran-
por un Banco
sacciones.
• El banco cumple con las reglamentaciones gubernamentales.
• El entorno físico es agradable para el cliente.
• El desempeño del automóvil es como se pretende.
• Es agradable su apariencia.
• Están cada uno de los componentes del vehículo dentro de las
tolerancias de manufactura.
Producto
• El diseño es seguro en su operación.
(vehículo)
• El vehículo tiene la confiabilidad pretendida.
• El consumo de gasolina, por kilómetro, el control de la contami-
nación y el equipo de seguridad del vehículo están dentro de las
normas gubernamentales.

• Efectividad en la solución de fallas u otros trabajos en el vehículo.


• Repuestos de la más alta calidad.
• Duración del servicio y puntualidad.
Servicio de un
• Costo del servicio.
taller automotor
• Limpieza del vehículo a la entrega.
• Organización de la atención al cliente.
• Aspectos de las instalaciones y los empleados.
• Confianza que inspira el personal.

CONCEPTOS BÁSICOS RELACIONADOS CON LA CALIDAD


Se definen a continuación algunos de los conceptos básicos relacionados con la cali-
dad, de acuerdo con la norma ISO 9000 vigente (vocabulario).

•• Calidad. Es el conjunto de propiedades y características de un producto,


servicio o proceso, que le confieren su aptitud para satisfacer unas nece-
sidades expresadas o implícitas.
•• Control de calidad. Técnicas y actividades de carácter operativo utiliza-
das para satisfacer los requisitos relativos a la calidad.

220
Sánchez Marchán, Nanci. (2014).Introducción a la Ingeniería Industrial, Material Instruccional de Apoyo. Caracas: UNA

•• Gestión de la calidad. Aspectos de la función general de la gestión que


determinan y aplican la política de la calidad.
•• Sistema de la calidad. Conjunto de la estructura de una organización,
de responsabilidades, de procedimientos, de procesos y de recursos que
se establecen para llevar a cabo la gestión de la calidad.
•• Planificación de la calidad. Es el proceso por el cual una empresa de-
fine su razón de ser en el mercado, su estado deseado en el futuro y
desarrolla los objetivos y las acciones concretas para llegar a alcanzar
ese estado. Se refiere, en esencia, al proceso de preparación necesario
para alcanzar los objetivos de la calidad.
•• Política de la calidad. Directrices y objetivos generales de una empresa
relativos a la calidad, expresados formalmente por la dirección general.
•• Plan de calidad. Es el  conjunto de actuaciones que se necesitan desa-
rrollar en una organización,   para lograr alcanzar la eficacia definida en
la política de calidad. Debe plasmarse en un documento escrito donde se
definan los objetivos, las acciones, los indicadores y los estándares de
calidad que se pretenden lograr en un plazo establecido.
•• Manual de calidad. Detalla la política de calidad de la empresa, así
como la organización precisa para conseguir los objetivos de asegura-
miento de la calidad de una manera similar en la empresa en su conjun-
to. En él se describen la política de calidad de la empresa, la estructura
organizacional, la misión de todo elemento involucrado en el logro de la
calidad, entre otros.
•• Normas ISOO 9000. Son normas de calidad y gestión continua de cali-
dad, creadas por la Organización Internacional para la Estandariza-
ción (ISO) que se pueden aplicar en cualquier tipo de organización o
actividad sistemática, orientada a la producción de bienes o servicios.
Se componen de estándares y guías relacionadas con sistemas de ges-
tión, además de herramientas específicas como los métodos de audito-
ría (el proceso de verificar que los sistemas de gestión cumplen con el
estándar).

GESTIÓN DE LA CALIDAD
La gestión de la calidad es un enfoque global, que incorpora la formulación de la política,
los objetivos y las responsabilidades de calidad. Asimila el control, el aseguramiento, el
mejoramiento del sistema de calidad, mediante la participación del personal y su trabajo en
equipo, el aprendizaje y mejoramiento continuo con un enfoque hacia el cliente. Ver defini-
ción de la ISO.

221
Sánchez Marchán, Nanci. (2014).Introducción a la Ingeniería Industrial, Material Instruccional de Apoyo. Caracas: UNA

Evolución de la gestión de la calidad

La evolución del concepto de calidad en la industria muestra que se avanzó de una eta-
pa donde la calidad solamente se refería al control final. Para separar los productos malos
de los productos buenos, a una etapa de control de calidad en el proceso, con el lema: “La
calidad no se controla, se fabrica”.

Hacia finales de los años 70, la gestión de la calidad se fundamentó en el control y la ins-
pección, las empresas se conformaban con controlar los productos uno por uno o mediante
muestreo, asegurándose más tarde que los controles habían sido bien efectuados. Fue tras-
cendente en esta evolución la aportación de Juran (Ver tabla 27) y del enfoque japonés del
control de la calidad extendido a toda la empresa o Company Wide Quality Control (CWQC).

La gestión de la calidad ha transitado por cuatro etapas principales, como se observa


en la figura 44:

Figura 44. Interrelación entre las fases históricas de la Gestión de la Calidad.

Inspección

La etapa de inspección es la más rudimentaria. La calidad está definida por el produc-


tor y controlada por un inspector, cuya función es verificar el 100% de las piezas elaboradas
para separar los productos conformes con los requisitos de calidad establecidos, de los pro-
ductos no conformes. Se puede definir como:

La actividad de examinar, medir, contrastar o ensayar las características de calidad de


un producto o servicio, para determinar su conformidad con los requisitos especifica-
dos, es decir, detectar características no conformes mediante previo análisis de fallas.

Está apoyada en los sentidos, en instrumentos de medición, en patrones de compa-


ración o en equipos de pruebas y ensayos. La metrología (ciencia de las mediciones) es
también otro soporte importante para la inspección.

Partiendo de la mencionada etapa de inspección se fueron desarrollando modelos o


enfoques de gestión de la calidad que han sido generalmente reconocidos. Estos modelos
son: control de calidad, aseguramiento de la calidad y gestión de calidad total, éstos
han evolucionado históricamente hacia una visión cada vez más general, pero sin dejar de
contener elementos de los modelos anteriores. A continuación se analizan cada uno de ellos:

222
Sánchez Marchán, Nanci. (2014).Introducción a la Ingeniería Industrial, Material Instruccional de Apoyo. Caracas: UNA

Control de calidad

El control de calidad se inicia en Estados Unidos a principios del siglo XX con la finali-
dad de verificar los requisitos relativos a la calidad del producto. Ver definición ISO.

Aunque primeramente el control de calidad se aplicó sólo a la fabricación industrial,


enseguida se extendió su uso a la prestación de servicios.

En los años 30 se introduce la estadística en la inspección y en el control de calidad


(control estadístico del proceso) el cual es un elemento fundamental en la filosofía del
control de calidad moderno. Se puede definir como:

La utilización de procedimientos científicos, incluyendo los métodos estadísticos en la


organización, así como en la recogida y análisis de datos, para que las decisiones no se
sostengan bajo suposiciones.

Utiliza fundamentalmente gráficos que permiten monitorear la estabilidad (calidad) de


un proceso de producción, de forma que se detecte con anticipación cualquier situación
inadecuada; lo que permitirá eliminar las causas especiales de variabilidad en la obtención
del resultado final.

El control de calidad recae sobre el producto y el proceso. Se reparan los productos


defectuosos y no conformes, de modo que al cliente sólo se le entreguen los aceptables. Los
temas convencionales del control de calidad son el muestreo de aceptación, los gráficos
de control y el uso de las normas militares.

Aseguramiento de la calidad

El aseguramiento de la calidad es un aspecto importante de las operaciones de pro-


ducción en toda la historia, pero es en la década de los años veinte cuando se consolida el
término. Fueron pioneros de este hecho Walter Shewnart, Harold Dodge y George Edwards,
quienes con sus aportes, como por ejemplo las gráficas de control por parte de Shewnart,
las técnicas de muestreo por Dodge y las técnicas de análisis económicos de Edwards para
resolver problemas; constituyeron la base del moderno aseguramiento de la calidad.

Este enfoque de calidad se centra en proporcionar la confianza derivada del sistema


de calidad (ver definición ISO de sistema de calidad), para que internamente la gerencia,
los integrantes de la organización y externamente los clientes, es decir, todas las partes in-
teresadas puedan desempeñarse con la tranquilidad resultante de seguir una gestión por
procesos, de manera tal que se tenga una actitud preventiva frente a los problemas y se
garantice la conformidad de los productos y servicios. El sistema de calidad contempla todo
lo relacionado con la documentación de los procesos, los procedimientos y en el Manual de
Calidad.

223
Sánchez Marchán, Nanci. (2014).Introducción a la Ingeniería Industrial, Material Instruccional de Apoyo. Caracas: UNA

El aseguramiento de la calidad, se puede definir como:

El esfuerzo total para plantear, organizar, dirigir y controlar la calidad en un sistema de


producción con el objetivo de dar al cliente productos con la calidad adecuada. Es sim-
plemente asegurar que la calidad sea lo que debe ser.

En el ámbito de la gestión empresarial, la calidad ha dejado de ser una descripción del


grado de satisfacción que proporciona un producto o servicio para convertirse en toda una
cultura, una filosofía que rige los principios de la gestión de la organización y que está pre-
sente en todas sus áreas. Para ello son importantes considerar reglas, guías o definiciones
de características, las cuales aseguran que los materiales, productos, procesos o servicios
que son apropiados para sus propósitos, mantengan las configuraciones estándar (normas).

El aseguramiento de la calidad contempla todos los aspectos relacionados con la nor-


malización, tiene una cobertura desde el contacto con el cliente en el mercadeo, pasando
luego a la determinación de especificaciones detalladas en el diseño, posteriormente lleva
sus técnicas y normas a la adquisición de suministros o compras. Por ello se desarrollan
las primeras normas ISO (9000:1987) que han ido evolucionando hasta la versión actual.

La International Organization for Standardization (ISO), con sede en Ginebra, es una


federación internacional de organismos nacionales de normalización de más de 140 países
(en Venezuela: FONDO NORMA). Son el resultado de acuerdos internacionales entre las
organizaciones miembros de la federación.

La gran mayoría de estas normas ISO, son muy específicas de un producto, material o
proceso particular (ejemplo: tamaños de cojinetes). De esta serie de normas, la ISO 9000
e ISO 14000 se configuran como sistemas estándar de normalización.

Como ejemplo de la aplicación de esta norma se puede mencionar, el formato de las


tarjetas de crédito, tarjetas de teléfono y otras tarjetas inteligentes, que se ha convertido en
común, como un resultado de una norma ISO. Adherirse a este estándar ha permitido que
estas tarjetas puedan ser utilizadas en todo el mundo.

Por todo lo anteriormente expuesto las normas ISO son las más importantes a nivel
internacional.

En el sentido amplio el aseguramiento de la calidad quiere decir cualquier


acción que se toma con el fin de dar a los consumidores productos (bienes
y servidos) de calidad adecuada.

Se hizo insuficiente aplicar sólo una inspección, control de calidad o asegurar la cali-
dad. Razón por la cual hoy día se habla de la calidad en la atención al cliente, camino que
conduce hacia la calidad total. Donde se crea una nueva cultura, se establece y mantiene un

224
Sánchez Marchán, Nanci. (2014).Introducción a la Ingeniería Industrial, Material Instruccional de Apoyo. Caracas: UNA

liderazgo; desarrollando al personal para trabajar en equipo, además de enfocar los esfuer-
zos hacia el cliente y planificación de cada uno de los pasos, para lograr la excelencia en las
operaciones de la empresa.

Control total de la calidad (TQM, siglas en inglés)

La última etapa en la evolución de la calidad es la calidad total y se puede definir como:

Una metodología de gestión a través de la cual la empresa satisface las necesidades


y expectativas de sus clientes, empleados, accionistas y de la sociedad en general,
utilizando los recursos de que dispone: personas, materiales, tecnología, sistemas pro-
ductivos, información, entre otros.

A principios de los años 50 Deming comienza a desarrollar, en Japón, sus ideas sobre
la gestión de la calidad basada en la planificación, el control y la mejora. La extensión de las
actividades de mejora a todos los ámbitos de la empresa y a todo tipo de ellas, da lugar al
término calidad total, obteniéndose en 1951 un Premio Nacional de la Calidad en este país.
Luego en 1987 se crea el Malcolm Baldrige, el primer modelo de gestión de la calidad total
occidental. En Europa la calidad total se introduce en 1988, con la creación de la EFQM (Eu-
ropean Foundation for Quality Management). El primer premio EFQM data de 1991 y ocho
años después (1999) en Iberoamérica se crea el Premio Iberoamericano de la Calidad.

Existen diversos modelos de Control Total de la Calidad (TQM) tales como: el ISO 9000,
el modelo integral ISO y el modelo de excelencia (FQM).

Si la calidad consiste en facilitar productos satisfactorios para el cliente, la gestión total


de la calidad comprende todas las actividades mediante las cuales se alcanza esta satis-
facción, independientemente del lugar de la organización en la que ocurran. Esto significa
obtener:
•• La calidad de los productos.
•• La calidad de los insumos.
•• La calidad de los procesos.
•• La calidad de los recursos, técnicos, humanos y materiales.
•• La calidad de las actividades de gestión.

Para comprender la calidad total en toda su amplitud, hay que citar un conjunto de fun-
damentos básicos tales como:
•• Orientar la organización hacia el cliente. Satisfacer los requerimientos
del cliente es lo principal. Con este objetivo, la organización debe girar
en torno a los procesos que son importantes para este fin y que añaden
valor.

225
Sánchez Marchán, Nanci. (2014).Introducción a la Ingeniería Industrial, Material Instruccional de Apoyo. Caracas: UNA

•• Ampliar el concepto de cliente. Se concibe a la organización como un


sistema integrado por proveedores y clientes internos. Aplicar la calidad,
significa que hay que satisfacer, también, las necesidades del cliente in-
terno.
•• Poseer liderazgo en costes. Las necesidades y expectativas del cliente
serán mejor atendidas, si el costo trasladado al cliente es más bajo. Esta
reducción de costos permite competir en el mercado con posibilidades
reales de éxito, por tanto es necesario reducir los costos de no calidad.
•• Gestionar basándose en la prevención. La idea es hacer las cosas
bien a la primera. Se reduce la necesidad de aplicar acciones de control,
minimizando los costos.
•• Potenciar el factor humano. La calidad es asumida por todas las per-
sonas que conforman la organización. Por lo tanto es imprescindible es-
tablecer una gestión de los recursos humanos desde la motivación para
la calidad y la participación.
•• Mejora permanente. No se llega a la calidad total, se persigue un ho-
rizonte que se amplía a medida que se avanza. Aquí está implícita esta
idea de mejora continua. Siempre es posible hacer las cosas mejor y
adaptarse más a las necesidades y expectativas dinámicas del cliente.

La filosofía de calidad total no es, únicamente, un modo de pensar, es sobretodo, un


conjunto de principios y métodos que procuran la meta de la satisfacción del cliente al menor
costo.
Los enfoques o modelos de gestión antes estudiados (Inspección, control
de calidad, aseguramiento de la calidad y control total de la calidad) han
sido utilizados como marcos conceptuales o para la aplicación práctica de
la gestión de la calidad. Siguen muy vigentes hoy los enfoques de asegura-
miento de la calidad y gestión de calidad total.

Actividad propuesta
Investigue (en medio impreso o electrónico) y responda a la siguiente interrogante
¿qué debe considerar una empresa al momento de implantar un sistema de ges-
tión de calidad?

Mejoramiento de la calidad

Si algo caracteriza a la Gestión de la Calidad es la aplicación de la mejora continua.


No se trata de obtener resultados espectaculares en los procesos y sistemas, sino ir alcan-
zando mejoras incrementales de manera continua.

La calidad es un proyecto interminable, cuyo objetivo es detectar disfunciones tan rápido


como sea posible. Así, la calidad puede representarse en un ciclo de acciones correctivas y
preventivas. El ciclo de Deming o modelo PDCA, se refiere a las iníciales en inglés, de los
siguientes pasos:

226
Sánchez Marchán, Nanci. (2014).Introducción a la Ingeniería Industrial, Material Instruccional de Apoyo. Caracas: UNA

•• Planear (plan): definir los objetivos a alcanzar y planificar cómo imple-


mentar las acciones.
•• Hacer (do): implementar las acciones correctivas.
•• Controlar (check): verificar que se logre el conjunto de objetivos.
•• Actuar (act): según los resultados obtenidos en el paso anterior, tomar
medidas preventivas.

En la figura 45 se ilustra la estructura de este ciclo:

Figura 45. Ciclo de Deming, llamado también modelo PDCA.

Mejorar la calidad interna y externa permite que una organización trabaje con sus em-
pleados en las mejores condiciones, lo que se traduce en una relación de confianza y ga-
nancias tanto financiera (aumento de los ingresos) como personal (clarificación de roles,
necesidades y oferta, motivación del empleado, entre otras). La mejora de la calidad es un
proceso que requiere la participación de toda la organización y generalmente, conduce a
cambios en los hábitos de trabajo e incluso en la misma organización.

En general, la mejora continua es un proceso pero, sobretodo, una actitud que debe ser
interiorizada por la cultura de la organización y aplicada en todos sus ámbitos.

Existen siete herramientas básicas que han sido ampliamente adoptadas en las activi-
dades de mejora continua de la calidad, utilizadas como soporte para el análisis y solución
de problemas operativos en los más distintos contextos de una organización, estas son:

1. Hoja de Control 4. Diagrama de causa - efecto


(Hoja de recogida de datos) 5. Diagrama de PARETO
2. Histograma 6. Diagrama de correlación
3. Diagrama de flujo 7. Gráfico de control

227
Sánchez Marchán, Nanci. (2014).Introducción a la Ingeniería Industrial, Material Instruccional de Apoyo. Caracas: UNA

Estas herramientas sirven para:


•• Detectar problemas.
•• Delimitar el área problemática.
•• Determinar factores que probablemente provoquen el problema.
•• Determinar si el efecto tomado como problema es verdadero o no.
•• Prevenir errores debido a omisión, rapidez o descuido.
•• Confirmar los efectos de mejora.
•• Detectar desfases.

La experiencia de los especialistas en la aplicación de estas herramientas señalan que


bien aplicadas y utilizando un método estandarizado de solución de problemas pueden ser
capaces de resolver hasta el 95% de estos problemas.

A continuación se detalla cada una de estas herramientas:

1. Hoja de registro de datos o de control

Permite reunir y clasificar las informaciones según determinadas categorías, mediante


la anotación y registro de sus frecuencias, bajo la forma de datos. Los cuales se registran en
una plantilla o formato, diseñadas especialmente para cada caso, indicando la frecuencia de
observación. Estas plantillas o formatos pueden tener distintos caracteres según el tipo de
datos, como por ejemplo, el lugar y el número de observaciones a recogerse, entre otros. La
recolección de datos debe efectuarse de manera cuidadosa y exacta.

Los formatos de registro de datos pueden tener varias finalidades: controlar las varia-
bles de un proceso, llevar un control de productos defectuosos, estudiar la localización de
defectos en un producto, estudiar las causas que originan los defectos o realizar la revisión
global de un producto.

Procedimiento a seguir:

1. Identificar el elemento de seguimiento.


2. Definir el alcance de los datos a recoger.
3. Fijar la periodicidad de los datos a recolectar.
4. Diseñar el formato de la hoja de recogida de datos, de acuerdo con la cantidad
de información a utilizar, dejando espacio para totalizar los datos, que permita
conocer las fechas de inicio y término, las probables interrupciones, las perso-
nas que recogen la información, la fuente etc.

228
Sánchez Marchán, Nanci. (2014).Introducción a la Ingeniería Industrial, Material Instruccional de Apoyo. Caracas: UNA

Ejemplo Ilustrativo Nº 1
Resumen de los defectos encontrados en aislantes térmicos
Control de Aisladores
Identificación
Tipo: Fecha:
Lote: Línea:
Hoja de ruta: Operario:
Total revisado:
Defectos:
Tipo Total
Soldadura
Poro
Deformado
Incompleto
Otros

Notas e incidencias:

Ejemplo Ilustrativo Nº 2
Resumen de las lesiones ocurridas durante un mes de trabajo en una plan-
ta:
Lesiones ocurridas durante un mes de trabajo en una planta
Identificación
Nombre de la empresa: Mes estudiado:
Nombre del inspector: Frecuencia de lesiones:
Total general de lesiones:
Implementos de seguridad usados:
Semanas Total
Lesiones frecuentes 1 2 3 4
Cabeza
Tronco
Extremidades inferiores
Extremidades superiores
Observaciones:

2. Histograma: es una representación gráfica de una variable en forma de barras, don-


de la superficie de cada barra es proporcional a la frecuencia de los valores representados.
En el eje de las abscisas (X) se presentan las clases o características y en el eje de las or-
denadas (Y) la frecuencia.

Generalmente se utilizan con dos propósitos:

229
Sánchez Marchán, Nanci. (2014).Introducción a la Ingeniería Industrial, Material Instruccional de Apoyo. Caracas: UNA

•• Para comprender la variabilidad que hay en un proceso, con el fin de


evaluar su capacidad de producir resultados aceptables en forma con-
sistente.
•• Para ayudar a generar teorías sobre la dinámica del proceso y las causas
de los problemas, confirmándolas o descartándolas posteriormente por
medio de observaciones o análisis adicionales.

Procedimiento para construir un diagrama de barra o histograma:


•• Determinar el rango de los datos: es igual al dato mayor (D1) menos el
dato menor (D2); R = D1 - D2.
•• Cálculo del número de clase: un criterio usado frecuentemente es que el
número de clases debe ser aproximadamente la raíz cuadrada del núme-
ro de datos, por ejemplo, la raíz cuadrada de 30 (número de artículos) es
mayor que cinco, por lo que se seleccionan seis clases. Nc = 6 o también
se puede calcular utilizando la siguiente fórmula:
•• K= 1+ 3,322 log 10n
•• Construir los intervalos de clases: los intervalos resultan de dividir el ran-
R
go de los datos en relación al número de intervalos I= Cálculo de an-
K
cho de clase: es igual a la división entre el rango y el número de interva-
los (aproximar a número más cercano) AC = R / Nc.
•• Realizar tabla de frecuencias: en ella se exponen la frecuencia de cada
intervalo de clase y en ocasiones, según sea necesario, la frecuencia
acumulada (se obtiene de dividir cada frecuencia entre el total de la
muestra y se suman progresivamente los resultados. La frecuencia es el
número de veces que se repite un dato.
•• Graficar el histograma: se hace un gráfico de barras, las bases de las ba-
rras son los intervalos de clases y la altura es la frecuencia de las clases.
Si se unen los puntos medios de las bases superiores de los rectángulos
se obtiene el polígono de frecuencias.
•• Identificar y clasificar el patrón de variación; desarrollar una explicación
lógica y pertinente del patrón.

De acuerdo al procedimiento planteado se obtiene una figura como la observada a


continuación:

Figura 46. Representación general de un histograma.

230
Sánchez Marchán, Nanci. (2014).Introducción a la Ingeniería Industrial, Material Instruccional de Apoyo. Caracas: UNA

Una vez obtenida la figura o gráfico se realiza un análisis de los resultados de éste y se
emiten las observaciones y conclusiones al respecto.

A continuación se presenta la tabla 28 con diferentes patrones de gráficos, que permiten


realizar comparaciones con los resultados obtenidos del problema en estudio, analizando las
causas que lo generan y facilitando una conclusión de éste.

Tabla 28. Patrones de gráficos de barra para comparar resultados de histogramas:

35
30
25
20
15
10
5
0

45
40
35
30
25
20
15
10
5
0

45
40
35
30
25
20
15
10
5
0

45
40
35
30
25
20
15
10
5
0

231
Sánchez Marchán, Nanci. (2014).Introducción a la Ingeniería Industrial, Material Instruccional de Apoyo. Caracas: UNA

Ejemplo Ilustrativo
Suponga que un central azucarero desea determinar cómo varía el peso de 400
sacos de su producto en un día. Seleccione una muestra de 50 sacos y registre
sus pesos en kilogramos. Los datos obtenidos son los siguientes:

65 63 65 63 69 67 53 58 60 61
64 65 64 72 68 66 55 57 60 62
64 65 64 71 68 66 56 59 61 62
63 65 63 70 67 66 57 59 61 62
64 64 63 69 67 66 58 60 61 62

Elabore el histograma y escriba una conclusión, comparando los resultados con


los patrones de gráficos de barra presentados.

Pasos:

1. Se realizan los siguientes cálculos:

Rango = Dato máximo – Dato mínimo = 72 – 53 = 19

Número de clases o intervalos:

K= 1+ 3,322 log 10n

K= 1+3,322 log 1050 = 7

Ancho de Clase (Tamaño del intervalo) = = =3

2. Se organiza la información en la tabla de frecuencias, de la siguiente manera:

Pesos (Kg) Conteo Total


53 - 55 II 2
56 - 58 IIII I 5
59 - 61 IIII IIII I 9
62 - 64 IIII IIII IIII III 15
65 - 67 IIII IIII IIII 12
68 - 70 IIII I 5
71 - 73 II 2

232
Sánchez Marchán, Nanci. (2014).Introducción a la Ingeniería Industrial, Material Instruccional de Apoyo. Caracas: UNA

3. Se construye la siguiente tabla:

Frecuencia Frecuencia
Frecuencia Frecuencia
absoluta relativa
absoluta relativa
acumulada acumulada
Pesos (Kg) fi Fi hi Hi
53 - 55 2 2 4,00% 4,00%
56 - 58 5 7 10,00% 14,00%
59 - 61 9 16 18,00% 32,00%
62 - 64 15 31 30,00% 62,00%
65 - 67 12 43 24,00% 86,00%
68 - 70 5 48 10,00% 96,00%
71 - 73 2 50 4,00% 100,00%
50 100,00%

Fi

5. Conclusión:

Los datos están hacia la izquierda de la media. La distribución no es normal y el proceso


debe ser investigado.

3. Diagrama de flujo

Son representaciones gráficas de la secuencia o relación lógica correspondiente a la


secuencia de etapas, operaciones, movimientos, decisiones y otros eventos que ocu-
rren en un proceso. Es una herramienta muy apropiada para representar secuencias o
pasos complejos. El Diagrama de Flujo hace más fácil el análisis de un proceso para la iden-
tificación de las entradas de proveedores, las salidas de sus clientes y de los puntos críticos
del proceso.

233
Sánchez Marchán, Nanci. (2014).Introducción a la Ingeniería Industrial, Material Instruccional de Apoyo. Caracas: UNA

Procedimiento para su elaboración:

Esta representación se debe desarrollar a través de formas y símbolos gráficos como


los que se muestran en la tabla 29.

Tabla 29.
Significado y descripción de símbolos utilizados por la ASME (Sociedad Americana de Inge-
nieros Mecánicos), para elaboración del diagrama de flujo. Esta simbología es similar a la
utilizada por la organización ISO (Organización Internacional de la Normalización)

Significado Símbolo Descripción

Este símbolo se usa para identifi-


Inicio-Fin car el inicio y el fin de un proce-
so.

Representa una etapa del pro-


ceso. El nombre de la etapa y
Operación
de quien la ejecuta se registra al
interior del rectángulo.

Simboliza el documento resul-


tante de la operación respectiva.
Documento
En su interior se anota el nombre
que corresponda.

Representa el punto del proceso


donde se debe tomar una de-
SI
cisión. La pregunta se escribe
Decisión dentro del rombo. Dos flechas
que salen del rombo muestran la
NO
dirección del proceso, en función
de la respuesta real.

Línea de flujo o fluido de direc-


ción, son flechas que conectan
Flecha (Flujo) pasos del proceso. La punta de
la flecha indica la dirección del
flujo del proceso.

Se utiliza un círculo para indicar


el fin o principio de una página
Conector que conecta con otra. El núme-
ro de página que precede se
conecta dentro del círculo.

234
Sánchez Marchán, Nanci. (2014).Introducción a la Ingeniería Industrial, Material Instruccional de Apoyo. Caracas: UNA

Obteniendo un gráfico de la forma siguiente:

Figura 47. Representación general de un diagrama de flujo.

Ejemplo Ilustrativo
A continuación se plantea un ejemplo sencillo, correspondiente al proceso produc-
tivo de una central de almacenamiento y distribución de frutas de hueso (ciruelas).

235
Sánchez Marchán, Nanci. (2014).Introducción a la Ingeniería Industrial, Material Instruccional de Apoyo. Caracas: UNA

236
Sánchez Marchán, Nanci. (2014).Introducción a la Ingeniería Industrial, Material Instruccional de Apoyo. Caracas: UNA

4. Diagrama causa-efecto o de Ishikawa: es una técnica gráfica ampliamente utiliza-


da, que permite apreciar con claridad las relaciones entre un tema o problema y las posibles
causas que pueden estar contribuyendo para que ocurra. Sirve para solventar problemas de
calidad y actualmente es ampliamente utilizado alrededor de todo el mundo. Ver figura 48.

Figura 48. Representación general de un diagrama de causa y efecto.

Procedimiento

1. Trazar una flecha y escribir el “efecto” del lado derecho

2. Identificar las causas principales a través de flechas secundarias que terminan en


la flecha principal.

3. Identificar las causas secundarias a través de flechas que terminan en las flechas
secundarias, así como las causas terciarias que afectan a las secundarias.

237
Sánchez Marchán, Nanci. (2014).Introducción a la Ingeniería Industrial, Material Instruccional de Apoyo. Caracas: UNA

Ejemplo Ilustrativo N° 1
Elaborar un diagrama de Causa y Efecto cuya causa principal es detectar el mal
funcionamiento de la generación de luz electrica.

Ejemplo Ilustrativo N° 2
Construir un diagrama de Causa y Efecto cuya causa principal es detectar los
defectos en la elaboración de camisas para caballeros.

238
Sánchez Marchán, Nanci. (2014).Introducción a la Ingeniería Industrial, Material Instruccional de Apoyo. Caracas: UNA

5. Diagrama de PARETO: a principios del siglo XX, Wilfredo Pareto (1848-1923) realizó
un estudio sobre la riqueza y la pobreza en Italia y se dio cuenta de que el 20% de las perso-
nas controlaba el 80% de las riquezas. Posteriormente, Joseph Juran descubrió en muchas
situaciones esta misma evidencia de la regla 80-20.

Es una gráfica para organizar datos de forma que éstos queden en orden descendente,
de izquierda a derecha y separados por barras. Permite, pues, asignar un orden de priori-
dades. Es una técnica que separa los “pocos vitales de los muchos triviales”. Se utiliza para
separar los aspectos significativos de un problema, con el fin de que el equipo sepa a dónde
dirigir sus esfuerzos para mejorar.

Los problemas más significativos son las barras más grandes en la gráfica de PARETO
y ésos son los que hay que reducir. La meta es concentrar la atención en lo “poco pero esen-
cial” en lugar de lo “mucho pero trivial”.

Usos del diagrama:


•• Para concentrar la atención en lo poco pero esencial de los problemas o
procesos que la requieran con el propósito de generar un impacto visual.
•• Para seleccionar una oportunidad de mejora.
•• Para decidir cuál problema o problemas son más grandes, permitiéndole
al grupo establecer prioridades.
•• Para identificar las causas más influyentes del problema.
•• Para mostrar cambios en el desempeño a lo largo del tiempo.

Procedimiento para construir un gráfico de PARETO


•• Seleccionar los problemas a ser comparados y ordenarlos por categoría.
•• Seleccionar la unidad de medición (categoría) o criterio de clasificación
de los datos.
•• Determinar un período para la recolección de datos.
•• Diseñar una hoja para recoger los datos.
•• Recopilar datos y registrarlos en la hoja de verificación que se diseñó.
•• Ordenar los datos de la mayor categoría a la menor.
•• Elaborar una gráfica de PARETO que muestre los resultados en barras,
en forma descendente, de izquierda a derecha.
•• En el eje Y de la izquierda, registrar la frecuencia.
•• En el eje X, representar tipos de problemas, causas, etc.
•• En el eje Y de la derecha, indicar el porcentaje del total.
•• Iniciar en el eje Y de la izquierda, la línea del porcentaje acumulativo de
tal forma que sobre cada barra exista un punto, que representa el valor
acumulado de su resultado más el de la barra anterior, hasta llegar a
100% y luego, unir los puntos.
•• Indicar el título de la gráfica y cómo se calcularon los datos para refe-
rencia futura.

239
Sánchez Marchán, Nanci. (2014).Introducción a la Ingeniería Industrial, Material Instruccional de Apoyo. Caracas: UNA

•• Reelaborar la gráfica a lo largo del tiempo para que se puedan ver los
cambios.
•• Analizar la gráfica para determinar los “pocos vitales”.

De los pocos vitales se deriva el siguiente principio:

“Todo grupo de elementos o factores que contribuyen a un mismo efecto, unos


pocos son responsables de la mayor parte de dicho efecto”.

Figura 49. Factores triviales y vitales.

En calidad se utiliza este principio para priorizar los problemas o las causas que los
generan, a partir de una representación gráfica de los datos obtenidos. De acuerdo con este
principio, si se tiene un problema con muchas causas, se puede decir que en torno al 20%
de las causas se origina el 80% de los problemas y el 80% de las causas sólo resuelve el
20% de los problemas.

Una vez ejecutados los pasos anteriores se obtiene un gráfico como el siguiente:

Figura 50. Representación generalde un diagrama de PARETO.

240
Sánchez Marchán, Nanci. (2014).Introducción a la Ingeniería Industrial, Material Instruccional de Apoyo. Caracas: UNA

Algunos consejos para la interpretación:

En la interpretación de un diagrama de PARETO se ha de observar cuáles son los


defectos donde existe mayor frecuencia de recurrencia y así determinar los pocos vitales,
para inferir que, si se eliminan las causas que provocan estos pocos vitales, desaparecería
la mayor parte de los defectos. 

Ejemplo Ilustrativo
En la compañía de seguros “La Fortaleza”, con el objeto de mejorar el servicio a
sus clientes, se tomaron los datos de las causas por las cuales se devuelven los
reclamos de las pólizas HCM. Los datos han sido tomados durante un lapso deter-
minado.
En la tabla anexa se presentan estos datos, en la que los pasos del primero al
cuarto ya han sido ejecutados y se continúa con el quinto paso:

SEGUROS LA FORTALEZA
CAUSAS DE DEVOLUCIÓN DE RECLAMOS DE PÓLIZAS DE HCM
Julio–diciembre 2013
(Caso hipotético)

NÚMERO DE
CAUSA
CASOS
A Descripción incompleta del siniestro. 52
B Falta de factura original. 20
C Descripción del informe médico incompleta. 16
D Reporte del siniestro fuera del plazo establecido. 10
Falta del sello de la compañía de seguros que
E 6
pagó parte del siniestro.
F Falta del sello de la clínica. 4
G Siniestro no cubierto por la póliza. 4
TOTALES 112

241
Sánchez Marchán, Nanci. (2014).Introducción a la Ingeniería Industrial, Material Instruccional de Apoyo. Caracas: UNA

PASO 5: trazar los ejes del gráfico. PASO 6: sobre el eje horizontal enumere en
orden decreciente las categorías de izquierda
a derecha.

PASO 7: dibujar las barras, la altura de cada barra corresponderá al valor de la frecuen-
cia de casos observada en cada categoría.

PASO 8: trazar la línea quebrada para indicar sobre el gráfico el total acumulado de
cada categoría en términos porcentuales, para lo cual previamente deben calcularse los
porcentajes de las diferentes categorías, con respecto al total de los casos.

a. Cálculo de los porcentajes:

Para el cálculo de los porcentajes de cada categoría se emplea la siguiente relación:

%= x 100

La categoría A, tiene 52 casos de un total de 112% = 52/ 112 x 100

= 46,43

242
Sánchez Marchán, Nanci. (2014).Introducción a la Ingeniería Industrial, Material Instruccional de Apoyo. Caracas: UNA

En el siguiente cuadro se han realizado los cálculos de cada categoría:

Categoría
Número de casos % de casos % Acumulado
Causa
A 52 46.43 46.43
B 20 17.86 64.29
C 16 14.29 78.58
D 10 8.93 87.51
E 6 5.35 92.86
F 4 3.57 96.43
G 4 3.57 100.00
Total 112

b. Se procede a elaborar el Diagrama de PARETO con los porcentajes.

SEGUROS LA FORTALEZA
CAUSAS DE DEVOLUCIÓN DE RECLAMOS DE PÓLIZAS DE HCM
Julio–Diciembre 2013
(Caso hipotético)

243
Sánchez Marchán, Nanci. (2014).Introducción a la Ingeniería Industrial, Material Instruccional de Apoyo. Caracas: UNA

Interpretación del gráfico:


El esfuerzo de mejoramiento de la calidad para ofrecer un excelente servicio a los clien-
tes debe orientarse sobre las áreas descripción insuficiente del siniestro ocurrido. Presentar
la factura original adjunta a la solicitud del reclamo y una mejor descripción del informe mé-
dico, ya que en ellas se encuentran concentradas aproximadamente el 80 % de los casos de
los reclamos devueltos a los productores de seguro, es decir, el 78,58 %.

6. Diagrama de correlación: es una herramienta que se aplica para comprobar (acep-


tar o rechazar) teorías respecto a la supuesta existencia de una relación entre dos variables.
Se realiza una serie de medidas en pares (25 o más) para dos variables X y Y, y se grafican.

Se utiliza durante la fase de diagnóstico, para ensayar teorías sobre causas e identificar
su raíz. También es útil durante la fase de corrección, en el diseño de soluciones y para el
diseño de un sistema de control que mantenga los resultados de una acción de mejora de
la calidad.

Pasos para su elaboración:

1. Obtener una teoría aceptable y relevante sobre la supuesta relación entre dos
variables.
2. Obtener los pares de datos correspondientes a las dos variables (X,Y).
3. Decidir sobre qué eje se representará cada una de las variables.
4. Trazar y rotular los ejes horizontal y vertical.
5. Marcar sobre el diagrama los pares de datos.
6. Se analiza el gráfico para determinar si las variables están correlacionadas.

Finalizados los pasos anteriores se obtiene un gráfico como el que sigue:

Figura 51. Representación generalde un diagrama de correlación

Análisis:

Se deben tomar en cuenta aspectos como:

244
Sánchez Marchán, Nanci. (2014).Introducción a la Ingeniería Industrial, Material Instruccional de Apoyo. Caracas: UNA

1. La existencia de algún patrón o correlación entre las variables X e Y.


2. Establecer si la dirección es positiva o negativa.
3. Determinar la fortaleza de la relación (fuerte, débil)

Se muestra seguidamente una tabla en la que se pueden visualizar algunos patrones


para comparar la relación entre dos variables.

Tabla 30: posibles patrones para comparar el grado de correlación

245
Sánchez Marchán, Nanci. (2014).Introducción a la Ingeniería Industrial, Material Instruccional de Apoyo. Caracas: UNA

Ejemplo Ilustrativo:
Se quieren mejorar los procesos de selección llevados a cabo por una organiza-
ción. Para ello, se considera necesario evaluar la validez de las pruebas de selec-
ción empleadas. A tal efecto, se quiere determinar la relación existente entre las
puntuaciones obtenidas por un grupo de candidatos en una prueba selectiva, con
la que han alcanzado estos mismos individuos en la evaluación del rendimiento,
realizada un año después de su ingreso en la organización. Los pares de datos
obtenidos, se reflejan en la tabla que aparece a continuación.

Tabla de datos:

Nº X Y Nº X Y
1 42 45 16 98 75
2 42 51 17 76 67
3 51 46 18 66 66
4 47 52 19 90 75
5 53 60 20 59 70
6 62 45 21 95 79
7 74 71 22 81 80
8 56 46 23 92 85
9 80 65 24 83 80
10 67 59 25 75 80
11 65 72 26 64 50
12 75 54 27 71 63
13 73 59 28 60 46
14 55 75 29 59 62
15 50 64 30 85 92
El coeficiente de correlación para los datos referidos, es: r = 0,71.

Gráfico:

246
Sánchez Marchán, Nanci. (2014).Introducción a la Ingeniería Industrial, Material Instruccional de Apoyo. Caracas: UNA

Interpretación:

En el gráfico se puede apreciar una correlación positiva débil entre las dos variables, no
se evidencia una línea clara y definida desde el origen, es decir, la puntuación obtenida en la
prueba de evaluación del rendimiento no es totalmente determinante, para la selección de
los individuos.

7. Gráficos de Control o de Shewhart: se utilizan como técnicas de diagnóstico para


supervisar procesos de producción e identificar inestabilidades y circunstancias anormales,
fueron propuesto originalmente por W. Shewart en 1920, y en ellos se representa a lo largo
del tiempo el estado del proceso que se desea monitorear. Un gráfico de control es una grá-
fica lineal en la que se han determinado estadísticamente un límite superior (límite de control
superior) y un límite inferior (límite de control inferior) a ambos lados de la media o línea
central. La línea central refleja el producto del proceso. Los límites de control proveen seña-
les estadísticas para que la administración actúe, indicando la separación entre la variación
común y la variación especial.

Se aclara en estos gráficos cómo las medidas están relacionadas con los límites de
control superior e inferior del proceso, los puntos fuera de los límites de control revelan que el
proceso se encuentra fuera de control. Ningún proceso se halla espontáneamente en estado
de control, conseguir llegar a él supone un éxito, así como mantenerlo. Ése es el objetivo del
control de la calidad, en el cual se usa habitualmente muestras pequeñas de tamaño entre
5 a 10 elementos; tomadas a lo largo de un tiempo representativo, normalmente de 20 a 30
ocasiones.

Los gráficos de control de Shewart son básicamente de dos tipos: gráficos de control
por variables y gráficos de control por atributos.

Gráficos de control por variables

Se denominan gráficos por variables, cuando las medidas pueden adoptar un intervalo
continuo de valores; por ejemplo, la longitud, el peso, la concentración, entre otros. Se divi-
den en (ver figura 52):

247
Sánchez Marchán, Nanci. (2014).Introducción a la Ingeniería Industrial, Material Instruccional de Apoyo. Caracas: UNA

Resumen de la clasificación gráficos de control por variables

Figura 52. Resumen de gráficos de control por variables.

Gráficos de control por atributos.

Se dice que un gráfico es por atributo, cuando las medidas adoptadas no son continuas;
ejemplo, tres tornillos defectuosos cada cien, 3 paradas en un mes en la fábrica, seis perso-
nas cada 300, entre otros. (ver figura 53):

Resumen de clasificación de gráficos por atributo:

Figura 53. Clasificación de gráficos de control por atributo.

248
Sánchez Marchán, Nanci. (2014).Introducción a la Ingeniería Industrial, Material Instruccional de Apoyo. Caracas: UNA

Los gráficos de control de Shewart se pueden representar como sigue:

Figura 54. Representación general de un diagrama de control.

El interés de los gráficos de control radica en que son fáciles de usar e interpretar, tanto
por el personal encargado de los procesos como por la dirección y lo que es más importante;
la utilización de criterios estadísticos permite que las decisiones se basen en hechos y no en
intuiciones o en apreciaciones subjetivas, que tantas veces resultan falsas.

A la hora de analizar los datos en un proceso de control de calidad hay que diferenciar
tres casos, según la característica medida:
•• La variable es medible numéricamente, por ejemplo un tiempo.
•• Se estudia un atributo o característica cualitativa que el proceso posee o
no posee, por ejemplo, el paciente cumple o no cumple adecuadamente
el tratamiento.
•• Se cuenta el número de defectos en el producto o situaciones inadecua-
das en la prestación del servicio.

Para el funcionamiento del control de procesos productivos, se fijan los valores de va-
riables como:
•• Variables controlables: originan una variabilidad muy grande (ajuste
incorrecto de la máquina, errores humanos, entre otros) siendo posible
eliminar la causa o causas que las han producido.
•• Variables no controlables: son responsables de la variabilidad de las
características de calidad del producto y sus efectos son aleatorios. No
pueden ser asignadas a una causa única sino al efecto combinado de
muchas otras. Por ejemplo, las oscilaciones en la corriente que alimenta
las máquinas.

249
Sánchez Marchán, Nanci. (2014).Introducción a la Ingeniería Industrial, Material Instruccional de Apoyo. Caracas: UNA

Los cambios ocasionados por las variables controlables se denominan Causas asig-
nables. Los debidos a las variables no controlables se denominan Causas no asignables.
Cuando el proceso es afectado sólo por causas no asignables, se dice que está bajo con-
trol estadístico.

Los gráficos de control son muy útiles para estudiar las propiedades de los productos,
los factores variables del proceso, los costos, los errores y otros datos administrativos.

Un gráfico de control muestra:


•• Si un proceso está bajo control o no.
•• Indica resultados que requieren una explicación.
•• Define los límites de capacidad del sistema, los cuales previa compara-
ción con los de especificación pueden determinar los próximos pasos en
un proceso de mejora.

Cuando además de las causas no asignables, aparece una o varias causas asignables,
se dice que el proceso está fuera de control.

Interpretación de los gráficos de control

El objetivo de los gráficos de control es determinar de forma visual y por tanto sencilla
cuándo un proceso se encuentra fuera de control, con una probabilidad de error pequeña.

La primera indicación de que el proceso puede estar fuera de control viene dada por
la presencia de algún punto fuera de los límites de control. Para facilitar la detección de
patrones anómalos o poco probables en un proceso en estado de control, conviene revisar
algún patrón sobre implantación de procesos de calidad; en el que se presenta una serie de
reglas para detectar diferentes sucesiones de datos improbables, como la que se muestra a
continuación.

Se considera que existen anomalías en las gráficas de control cuando:

1. Existen puntos fuera de los límites de control.


2. Existen 7 puntos consecutivos a un mismo lado de la línea central.
3. Existen 7 puntos consecutivos ascendiendo o descendiendo.
4. Existen 7 puntos consecutivos alternando ascensos y descensos
5. De once puntos consecutivos, 10 por lo menos están a un mismo lado de la línea
central
6. Existen 2 de 3 puntos consecutivos muy cercanos a uno de los límites de control
(inferior o superior)
7. Cuando el 75% de los datos se encuentra alrededor del límite central de control.
8. Tendencias al incremento de la variabilidad o viceversa.
9. Se tienen por lo menos 4 puntos consecutivos con el mismo valor.

250
Sánchez Marchán, Nanci. (2014).Introducción a la Ingeniería Industrial, Material Instruccional de Apoyo. Caracas: UNA

En cualquier caso, siempre hay que estar atento a la presencia de patrones o tenden-
cias en los gráficos de control.

En este texto sólo se abordarán los gráficos de medias y rangos en el caso de control
por variables y la gráfica (P) en el caso de control por atributos.

Procedimiento para la elaboración de los gráficos de control por variables

Antes de iniciar la enumeración de los pasos a seguir en este procedimiento, se reseña


una tabla en la que se observa una breve descripción de los gráficos más usados en este
tipo de control.

Tabla 31. Descripción de las variables usadas en los gráficos de Control por Variables:
Gráfico Descripción Campo de aplicación.
Control de características para

X−R
Medias y rangos
datos agrupados
− Medias y desviación Control de características para
X−S estándar datos agrupados.
control de puntos indi- Control de características cuyas
PI - Rm viduales y rangos mó- muestras individuales se distri-
viles buyen normalmente.
Procesos en los que se desea
Promedios móviles ex- investigar causas que provo-
EWMA - R
ponenciales quen desplazamientos peque-
ños y constantes del centro.

Pasos:

1. Recaudar los datos:

Los datos son el resultado de la medición de las características del producto, los cuales
deben ser registrados y agrupados de la siguiente manera:
•• Se toma una muestra (subgrupo) de 2 a 10 piezas consecutivas y se
anotan los resultados de la medición (se recomienda tomar 5). También
pueden ser tomadas en intervalos de ½ - 2 hrs., para detectar si el pro-
ceso puede mostrar inconsistencia en breves períodos.
•• Se realizan las muestras de 20 a 25 subgrupos.

2. Calcular el promedio X y R para cada subgrupo

X 1 + X 2 .... X N R = X mayor − X menor


X =
N

251
Sánchez Marchán, Nanci. (2014).Introducción a la Ingeniería Industrial, Material Instruccional de Apoyo. Caracas: UNA

3. Calcular el rango promedio ( ) y media del proceso ( ).

R1 + R2 + ......RK X 1 + X 2 + ....... X K
R= X=
K K
4. Calcular los límites de control

Los límites de control son calculados para determinar la variación de cada subgrupo, es-
tán basados en el tamaño que tienen y se determinan según el tipo de variable que se utilice:

Límites X- R: Límites X - S: Límites P I- Rm : Límites EWMA - R :

LIC X = X − A2 R LIC X = X − A3 S Rm L
LSCX = X + 3 LSCX = µ + Lσ
d2 0 2−λ
LSC X = X + A2 R LIC X = X − A3 S LCX = X L
LSCX = µ − Lσ
LIC R = D3 R 0 2−λ
LCX = X Rm
LICX = X − 3
d2 LCX = X
LCX = X
LIC S = B4 S LSC R = D4 R
0 <λ ≤ 1
LSC R = D4 R LIC R = D3 R
LSC S = B3 S µ 0
=X

Donde D4, D3, A 2 son constantes que varían según el tamaño de muestra. A continuación
se presentan los valores de algunas constantes a utilizar en estos gráficos, para tamaños
de muestra de 2 a 10.

Tabla 32. Constantes para gráficos de control


n
Constante
2 3 4 5 6 7 8 9 10
D3 0 0 0 0 0 0.08 0.14 0.18 0.22
D4 3.27 2.57 2.28 2.11 2.00 1.92 1.86 1.82 1.78
A2 1.88 1.02 0.73 0.58 0.48 0.42 0.37 0.34 0.31
A3 2.659 1.954 1.628 1.427 1.287 1.182 1.099 1.032 0.975
B3 0.000 0.000 0.000 0.000 0.030 0.118 0.185 0.239 0.284
B4 3.267 2.568 2.266 2.089 1.970 1.882 1.815 1.761 1.716
E2 2.66 1.77 1.46 1.29 1.18 1.11 1.05 1.01 0.98

252
Sánchez Marchán, Nanci. (2014).Introducción a la Ingeniería Industrial, Material Instruccional de Apoyo. Caracas: UNA

5. Seleccionar la escala para las gráficas de control


Para la gráfica X la amplitud de valores en la escala debe de ser al menos del tamaño
de los límites de tolerancia especificados o dos veces el rango promedio ( ) .
Para la gráfica R la amplitud debe extenderse desde un valor cero hasta un valor supe-
rior equivalente a 1½ - 2 veces el rango.

6. Trazar la gráfica de control


Dibuje las líneas de promedios y límites de control en las gráficas.
Los límites de control se dibujan con una línea discontinua y los promedios con una línea
continua para ambas gráficas.
Marcar los puntos en ambas gráficas y unirlos para visualizar de mejor manera el com-
portamiento del proceso.

7. Analizar la gráfica de control

Ejemplo Ilustrativo:
El control de la temperatura es importante en un almacén destinado a guardar
cierto tipo de material. La temperatura debe ser mantenida dentro de los siguientes
valores: 70º ± 5º F. Existe la norma de leer la temperatura cuatro veces al día. En
la tabla mostrada se encuentra el registro correspondiente a los últimos 25 días,
determine:
a) Si el proceso está bajo control estadístico
b) Si el proceso es capaz de cumplir con las especificaciones.
Medidas Rango Media
Día 1 2 3 4 R x
1 70 66 72 68 6 69.00
2 70 68 69 68 2 68.75
3 70 71 75 76 6 73.00
4 70 69 73 66 7 69.50
5 70 70 71 65 6 69.00
6 66 71 69 71 5 69.25
7 70 74 73 70 4 71.75
8 70 69 72 74 5 71.25
9 69 71 73 67 6 70.00
10 67 70 66 64 6 66.75
11 71 72 63 66 9 68.00
12 69 72 69 70 3 70
13 68 64 73 74 10 69.75
14 66 72 71 73 7 70.50
15 71 71 69 65 6 69.00
16 68 67 71 71 4 69.25
17 67 70 70 72 5 69.75
18 69 70 70 69 1 69.50
19 67 70 68 68 3 68.25
20 70 68 67 64 6 67.25
21 70 71 68 68 3 69.25
22 64 68 72 69 8 68.25
23 70 72 76 70 6 72.00
24 69 71 70 72 3 70.50
25 69 69 70 75 6 70.75

253
Sánchez Marchán, Nanci. (2014).Introducción a la Ingeniería Industrial, Material Instruccional de Apoyo. Caracas: UNA

Cálculo de la línea central de control:


•• = Σ x / #M= 1740,25/25= 69,6

Cálculo de los límites de control:


•• LSC = X + A 2 * R = 69, 6+0, 73*5, 3= 73, 5
•• LIC = X - A 2 *R = 69, 6 - 0, 73*5, 3= 65, 7

Gráfico de medias:

De igual forma se puede construir un gráfico de control para la evolución del rango. En
este caso los límites de control vienen dados por las fórmulas:

LCSR=D4R LCIR=D4R

Donde D4 se obtiene de la tabla 2, y como antes R es el rango medio.

Cálculo de la línea central de control:

R= = 133/25= 5,3

Cálculo de los límites de control

LSC = D4 * R = 2, 28*5, 3= 12, 14


LIC = D3 *R = 0 * 5, 3= 0
LIC = X - A 2 * R = 69,6 - 0,73*5,3= 65,7

254
Sánchez Marchán, Nanci. (2014).Introducción a la Ingeniería Industrial, Material Instruccional de Apoyo. Caracas: UNA

Gráfico de rangos:

Como los gráficos de rangos y medias muestran estar bajo control estadístico, por no
presentar anomalias, se puede realizar su análisis.

Media del universo:

= = 69,6 69,6 ≠ 70,0 El proceso está descentrado.

Desviación estándar (estimada) del universo: s’= R/ d2.


σ’ = 5,3/2,059 = 2,57

Capacidad del proceso:


CP = IRP = LSE – LIE / 6s’
CP= 75 – 65 / 6x2.57= < 1

El proceso no es capaz de cumplir con las especificaciones por la gran dispersión.

Cálculo del coeficiente de variación:

Es una medida de dispersión relativa, que permite comparar la variabilidad entre varias
series de datos. Viene expresada por la relación existente entre una medida de dispersión
absoluta y una medida de tendencia central. Esta medida es independiente de las unidades
en que se expresan los datos.

CV = σ/Ẋ (100 %)

Nota: se considera que el promedio aritmético (X) es representativo de un conjunto de


datos cuando el coeficiente de variación es menor que diez (valor referencial, depende de la
naturaleza del proceso).

CV = 3,69 % < 10%, por ello la media del proceso es una buena representante de éste.

255
Sánchez Marchán, Nanci. (2014).Introducción a la Ingeniería Industrial, Material Instruccional de Apoyo. Caracas: UNA

Procedimiento para la elaboración del gráfico de control por atributos (p, np, u, c)

Antes de abordar este aspecto, se considera una breve descripción de las variables que
conforman los diferentes gráficos que se utilizan en este tipo de control.

Tabla 33. Descripción de las variables de los gráficos de control por atributos:

Gráficos Descripción Campo de aplicación


Control de la fracción global de defectuosos de un
P Proporciones. proceso.

NP Número de defectuosos. Control del número de piezas defectuosas.

C Defectos por unidad. Control de número global de defectos por unidad.

Promedio de defectos por


U Control del promedio de defectos por unidad.
unidad.

Pasos:

1. Frecuencia y tamaño de la muestra:

Establecer la frecuencia con la cual los datos serán tomados (hora, diaria, semanal…).
Los intervalos cortos entre tomas de muestras permitirán una rápida retroalimentación del
proceso ante la presencia de problemas. Se sugiere tomar tamaños de muestra iguales aun-
que no necesariamente se tiene que dar esta situación, el tamaño de muestra debería ser
mayor a 30. El tamaño de los subgrupos será de 25 o más.

2. Cálculo del porcentaje defectuoso (p) del subgrupo:

Registrar la siguiente información para cada subgrupo:


•• El número de partes inspeccionadas – n
•• El número de partes defectuosas – np

np
Calcule la fracción defectuosa (p) mediante: p =
n

3. Cálculo de porcentaje defectuoso promedio y límites de control:

El porcentaje defectuoso promedio para los k subgrupos se calcula con las siguientes
fórmulas:

np1 + np2 + .... + npk p(1 − p) p(1 − p)


p= LSC p = p + 3 LIC p = p − 3
n1 + n2 + ..... + nk n n

256
Sánchez Marchán, Nanci. (2014).Introducción a la Ingeniería Industrial, Material Instruccional de Apoyo. Caracas: UNA

Donde n es el tamaño de muestra promedio.

Nota: Cuando p y/o n es pequeño, el límite de control inferior puede resultar negativo,
en estos casos el valor del límite será = 0

Límites de np: Límites de C: Límites de U:


c
u= , donde
LSC = np + 3 np (1 − p) n
LSC = c + 3 c
c = número de defectos
LIC = np − 3 np (1 − p)
LSC = c − 3 c n = cantidad de piezas
inspeccionadas

u
LSC = u + 3
n
u
LIC = u − 3
n

4. Trazar la gráfica y analizar los resultados.

Como se mencionó anteriormente sólo se estudiará en este texto el gráfico p, el pro-


cedimiento a seguir para el resto de los tipos de gráficos de control por atributo (np, c, u) es
idéntico.

Ejemplo Ilustrativo:
Un fabricante de latas de aluminio registra el número de partes defectuosas, to-
mando muestras cada hora de n = 50, con 30 subgrupos. Realizar la gráfica de
control para la siguiente serie de datos obtenida durante el muestreo.

Muestra Latas defectuosas Muestra Latas defectuosas


1 12 16 8
2 15 17 10
3 8 18 5
4 10 19 13
5 4 20 11
6 7 21 20
7 16 22 18
8 9 23 24
9 14 24 15
10 10 25 9
11 5 26 12
12 6 27 7
13 17 28 13
14 12 29 9
15 22 30 6

257
Sánchez Marchán, Nanci. (2014).Introducción a la Ingeniería Industrial, Material Instruccional de Apoyo. Caracas: UNA

Calcular la fracción defectuosa para cada muestra:

Muestra Latas defec- Fracción Muestra Latas defec- Fracción


tuosas defectuosa tuosas defectuosa
1 12 0.24 16 8 0.16
2 15 0.30 17 10 0.20
3 8 0.16 18 5 0.10
4 10 0.20 19 13 0.26
5 4 0.08 20 11 0.22
6 7 0.14 21 20 0.40
7 16 0.32 22 18 036
8 9 0.18 23 24 0.48
9 14 0.28 24 15 0.30
10 10 0.20 25 9 0.18
11 5 0.10 26 12 0.24
12 6 0.12 27 7 0.14
13 17 0.34 28 13 0.26
14 12 0.24 29 9 0.18
15 22 0.44 30 6 0.12

Σp 3.6
p= =
n 15 p = 0.2313

p(1 − p) 0.2313*.77
LSC p = p + 3 0.2313 + 3
n = 50 = 0.4102

p(1 − p) 0.2313*.77
LIC p = p − 3 0.2313 − 3
n = 50 = 0.05243

Trazado de gráfica:

258
Sánchez Marchán, Nanci. (2014).Introducción a la Ingeniería Industrial, Material Instruccional de Apoyo. Caracas: UNA

Interpretación del gráfico:

Existen puntos fuera de los límites de control, por tanto, se puede decir que existen
ciertas anomalías en el proceso, de lo que se podría inferir que el mismo se encuentra fuera
de control estadístico.

A continuación se muestra un conjunto de ventajas y desventajas de los gráficos de


control las cuales se pueden visualizar en la siguiente tabla.

Tabla 34. Ventajas y desventajas de los gráficos de control

Ventajas Desventajas
• Conducen a un mejor procedimiento de control. • No se entienden a
menos que se dé capaci-
• Suministran una indicación de problemas impe-
tación.
riosos y permiten al personal operativo tomar
Gráficos de acciones correctivas antes de que ocurra la • Pueden causar confusión
control por producción real de artículos defectuosos. entre los límites de espe-
variables cificación y los límites de
• Proporcionan una utilización máxima de la infor-
tolerancia.
mación disponible de datos.

• Son potencialmente aplicables a cualquier pro- • No proporcionan infor-


ceso. mación detallada del
control de características
• Hacen posible considerar varias características
individuales.
de calidad al mismo tiempo y clasificar el artícu-
lo como disconforme si no satisface la especifi- • No reconocen distintos
Gráficos de cación de cualquier característica. grados de defectos en las
control por unidades de producto.
atributos • Los datos están a menudo disponibles.
• Son rápidos y simples de obtener.
• Son fáciles de interpretar.
• Son frecuentemente usados en los informes a la
Gerencia.

Las siete herramientas básicas de la calidad sirven para la recopilación sis-


temática de datos y para la visualización y análisis de los resultados. El Dr.
Kaoru Ishikawa llevó a cabo una recopilación de estas siete herramientas
para ser utilizadas de forma sistemática con el fin de lograr con ello una
gran efectividad.

En la tabla que a continuación se presenta, se puede observar un grupo de funciones


de las herramientas básicas de la calidad:

259
Sánchez Marchán, Nanci. (2014).Introducción a la Ingeniería Industrial, Material Instruccional de Apoyo. Caracas: UNA

Tabla 35. Funciones de las herramientas básicas de la calidad

Funciones Herramientas
Recoger datos. Hoja de recogida de datos.
Interpretar datos. Histograma.
Estudiar la relaciones causa-efecto. Diagrama de causa–efecto.
Fijar prioridades. Diagrama de PARETO.
Estratificar los datos. Estratificación.
Determinar correlaciones. Diagrama de correlación.
Determinar si un proceso está bajo control o no. Gráficos de control.

Las herramientas de calidad en algunos casos se pueden utilizar para determinar las ra-
zones por las cuales el proceso salió de control. Pero usualmente se utilizan tanto en control
como en mejora de procesos.

En el siguiente gráfico se muestra un resumen donde están involucradas las siete he-
rramientas básicas estudiadas:

Figura 55. Resumen de las siete herramientas básicas de la calidad.

260
Sánchez Marchán, Nanci. (2014).Introducción a la Ingeniería Industrial, Material Instruccional de Apoyo. Caracas: UNA

En el camino para mejorar la calidad se utilizan las siete herramientas básicas mos-
tradas en este trabajo, su estudio y aplicación sistemática permitirá el análisis y control de
los procesos, además de evaluar los resultados de éstos, con lo cual se podrán hacer las
modificaciones pertinentes que puedan contribuir a mejorar las calidad de los productos y
servicios donde se apliquen.

NUEVAS HERRAMIENTAS (ADMINISTRATIVAS) APLICADAS EN LA


MEJORA DE LOS PROCESOS EN LA EMPRESA
Existen también un grupo de 7 nuevas herramientas aplicadas en la mejora de la calidad
de los procesos de la empresa estas son: diagrama de afinidad, diagrama de relaciones,
diagrama de árbol, diagrama matricial, diagrama de priorización, diagrama de proceso de
decisión y diagrama de flechas.

Surgen estas herramientas en los años 70, cuando el reto en esa época consistía en
que todas las áreas de la organización se implicaran y así mismo asumieran responsabilidad
sobre la calidad y no sólo el área de producción. Por ello el comité de la JUSE (Japanese
Union of Scientist and Engineers) analizó y realizó una investigación profunda para su crea-
ción. Estas son denominadas como las siete nuevas herramientas de la calidad, las cuales
resultaron ser aplicables a todos los sectores empresariales. El objetivo de estas herramien-
tas es la resolución de pocos problemas e importantes, a diferencia de las herramientas
básicas, las cuales son utilizadas para muchos problemas triviales.

En la utilización de las nuevas herramientas para la mejora de la calidad, también se uti-


liza el proceso tradicional de resolución de problemas, que consta de las siguientes etapas:
•• Identificar el problema.
•• Analizar sus causas.
•• Formular alternativas para su resolución.
•• Seleccionar la mejor alternativa.
•• Planificar su puesta en práctica.
•• Ejecutar la alternativa seleccionada.
•• Verificar los resultados obtenidos.

Las nuevas herramientas ofrecen:


•• Procedimientos sistemáticos para identificar la visión y el futuro que pre-
fiere una organización.
•• Un proceso que permite analizar de manera crítica la visión y comprobar
su validez y las posibilidades de hacerse realidad en el futuro estratégico
de la organización.
•• Medios para reducir la distancia que existe entre la visión y el lugar que
ocupa la organización en ese momento.

261
Sánchez Marchán, Nanci. (2014).Introducción a la Ingeniería Industrial, Material Instruccional de Apoyo. Caracas: UNA

En este texto se abordarán estas herramientas de manera breve, presentando sólo la


definición de cada una de ellas, así como su representación gráfica general.

1. Diagrama de afinidad: es una herramienta que sintetiza un conjunto de datos


verbales agrupándolos en función de la relación que tienen entre sí. Se basa, por
tanto, en el principio de que muchos de estos datos son afines por lo que pueden
reunirse bajo unas pocas ideas generales. Es considerado como una clase especial
de “tormenta de ideas”, constituyendo, frecuentemente, esta técnica de creatividad
el punto de partida para la elaboración del diagrama, cuya estructura se muestra a
continuación:

Figura 56. Representación generalde un diagrama de afinidad.

2. Diagrama de relaciones:es una herramienta que ayuda a percibir la relación lógica


que existe entre una serie de problemas, actividades o departamentos encadenados
como causas y efectos. Se representa como sigue:

Figura 57. Representación generalde un diagrama de relaciones.

262
Sánchez Marchán, Nanci. (2014).Introducción a la Ingeniería Industrial, Material Instruccional de Apoyo. Caracas: UNA

3. Diagrama de árbol: es una técnica que permite obtener una visión del conjunto de
los medios necesarios para alcanzar una meta o resolver un problema. Se representa
de la siguiente forma:

Figura 58. Representación generalde un diagrama de árbol.

4. Diagrama matricial: este tipo de diagrama facilita la identificación de relaciones que


pudieran existir entre dos o más factores, sean éstos problemas, causas y procesos;
métodos y objetivos; o cualquier otro conjunto de variables. Su representación gráfica
es la siguiente:

Figura 59. Representación generalde un diagrama matricial.


Nota: Car 1, Car 2, Car 3. Car 4, Car 5,… significan características del grupo A y grupo B.

5. Diagrama de priorización: también llamado diagrama portafolio, Herramienta


empleada para la toma de decisiones sobre la base de la priorización de actividades;
es decir, se debe priorizar una serie de soluciones a un problema en función de dos
variables que normalmente suelen ser el beneficio que se espera de la implantación
de la solución y del esfuerzo que supone su implantación. Se utiliza una combinación
de las técnicas de diagrama de árbol y diagrama matricial. La representación gráfica
de esta herramienta se indica a continuación:

263
Sánchez Marchán, Nanci. (2014).Introducción a la Ingeniería Industrial, Material Instruccional de Apoyo. Caracas: UNA

Problema
Frecuencia

Importancia

Factibilidad

Total
Figura 60. Representación generalde un diagrama de priorización.

d) Diagrama de proceso de decisión: es similar al diagrama de árbol, tanto en su


estructura como en su propósito. Tiene la ventaja de que permite determinar contramedidas
y planificar su utilización con anticipación suficiente a la generación de los problemas.

Figura 61. Representación generalde un diagrama de proceso de decisión.

e) Diagrama de flechas: es una representación gráfica en forma de red, de la planifica-


ción de un proyecto, la cual muestra las relaciones existentes entre las distintas actividades.
Su forma es la siguiente:

Figura 62. Representación generalde un diagrama de flechas.

264
Sánchez Marchán, Nanci. (2014).Introducción a la Ingeniería Industrial, Material Instruccional de Apoyo. Caracas: UNA

Ejemplo Ilustrativo
En la tabla que se muestra se observan algunas preguntas referentes a las eta-
pas necesarias en la solución de problemas, aplicando las nuevas herramientas
de gestión de la calidad. Ésta se completa colocando el objetivo y la herramienta
a utilizar en cada una de ellas.

Etapa Objetivo Herramienta

¿Cuál es el problema? Identificar problemas. Diagrama de Afinidad.

¿Cuáles son las causas del Identificar causa raíz Diagrama de Relacio-
problema? del problema. nes.

¿De qué forma se resuelve Identificar todas las Diagrama de Árbol.


el problema? soluciones posibles
del problema.

¿Qué opción tomar? Seleccionar la mejor Matrices de Prioriza-


solución. ción Diagrama Matri-
cial.

¿Cuándo y cómo actuar? Planificar la puesta en Diagrama de proceso


práctica de la mejor de decisión, Diagramas
solución. de Flechas.

Actividad propuesta
Investigue: a) ¿Quién creó las nuevas herramientas de la calidad; b) ¿Cuál es la
finalidad de haberlas creado?

Se muestra seguidamente un resumen donde se puede observar las siete nuevas he-
rramientas de la calidad:

265
Sánchez Marchán, Nanci. (2014).Introducción a la Ingeniería Industrial, Material Instruccional de Apoyo. Caracas: UNA

Sistetiza un conjunto de datos


verbales, agrupándolos en
función de la relación que tiene
entre sí.

Identificación de relaciones
que pudieran existir entre dos o
más factores.

Figura 63. Resumen de nuevas herramientas de la calidad.

Las siete nuevas herramientas (administrativas) para el mejoramiento de la calidad


permiten predecir el futuro, proporcionando un método sencillo de ayuda a los gestores
y directivos de una forma similar a cómo las herramientas básicas sirven de apoyo a los

266
Sánchez Marchán, Nanci. (2014).Introducción a la Ingeniería Industrial, Material Instruccional de Apoyo. Caracas: UNA

departamentos de fabricación. Son técnicas que permiten descubrir las complicadas rela-
ciones entre los diferentes elementos de un problema. Usándolas, se hace más fácil aclarar
la situación, establecer un plan y llegar a la raíz del problema. Así mismo, hace más fácil
explicar la situación a otros y conseguir su cooperación. Además de lograr reducir costos y
tiempos en pro de optimizar las operaciones de producción.

Estas nuevas herramientas de gestión y planificación, no sustituyen a las


básicas, sino que las complementan.

OTRAS HERRAMIENTAS PARA MEJORAR LA CALIDAD


Existe también otro grupo de herramientas para el mejoramiento de la calidad entre las
cuales se encuentran:

1. 5S. Es una técnica referida al mantenimiento integral de la empresa con el objeto


de lograr lugares de trabajo más organizados, ordenados y aseados en cuanto
a maquinaria, equipo e infraestructura, así como también del entorno de trabajo.
El significado de cada (S) que conforma la técnica (5S) es el siguiente: Seri (cla-
sificación y descarte), Seiton (organización), Seiso (limpieza), Seiketsu (higiene
y visualización) Shitsuke (disciplina y compromiso).
2. Seis sigma. Es un método basado en datos para llevar la calidad hasta niveles
cercanos a la perfección, se diferencia de otros enfoques debido a que corrige
los problemas antes de que se presenten. Más específicamente se trata de un
esfuerzo disciplinado para examinar los procesos repetitivos de las empresas.
Este método se denomina DMAMC (Definir, Medir, Analizar, Mejorar, Controlar)
utiliza herramientas estadísticas, además de dispositivos que observan las varia-
bles de los procesos y sus relaciones, para gestionar sus características.
3. Lean seis sigma. Seis sigma se ha integrado con otras herramientas, como
lean manufacturing (manufactura esbelta) con la cual comparte algunos objeti-
vos, lo que las hace ser complementarias, generándose una nueva metodología
conocida como Lean Seis Sigma (LSS). Lo que distingue a lean 6 sigma, aparte
de sus componentes individuales es el reconocimiento de que no se debe con-
siderar sólo la calidad o sólo la velocidad en un proceso; que se necesita un
equilibrio entre los dos, que pueda ayudar a una organización para centrarse en
la mejora de la calidad definida por el cliente.
4. QDF. Es un método de gestión de calidad que se fundamenta en transformar las
demandas del usuario en la calidad del diseño, es decir; implementar métodos
para lograr calidad en el diseño del proceso de fabricación.
5. Poka-Yoke (sistema anti–errores). Está asociado a la posibilidad de utilizar le-
yes físicas o dispositivos que hagan imposible la aparición del error, y por lo tanto

267
Sánchez Marchán, Nanci. (2014).Introducción a la Ingeniería Industrial, Material Instruccional de Apoyo. Caracas: UNA

del defecto. Como todas las herramientas de calidad, esta técnica es un medio
para hacer a la empresa más competitiva. Ver tabla Nº 26.
6. Círculos de calidad. El concepto de los círculos de calidad permite que los
empleados participen más, solucionando en forma organizada sus propios pro-
blemas de trabajo. Incrementa la productividad de la organización y la calidad de
sus productos, a través de la participación directa de sus empleados; asumiendo
que tal participación derivará en sugerencias útiles para mejorar los métodos de
trabajo y el control de la calidad.
El enfoque es meramente voluntario en todos los niveles de la organización. Si
un gerente objeta el concepto, no habrá Círculos de Calidad en su departamento
hasta que él cambie de actitud.
7. AMFE-AMFEC (Análisis Modal de Fallos, Efectos y su Criticidad). Aunque la
técnica se aplica fundamentalmente para analizar un producto o proceso en su
fase de diseño, este método es válido para cualquier tipo de proceso o situación
de la empresa, desde el diseño y montaje hasta la fabricación, comercialización
y la propia organización en todas las áreas funcionales de la empresa. Eviden-
temente la primordial utilidad del AMFE es resaltar los puntos críticos de un pro-
ceso con el fin de eliminarlos o establecer un sistema preventivo para evitar su
aparición o minimizar sus consecuencias. Puede ser un riguroso procedimiento
de detección de defectos potenciales, si se aplica de manera sistemática.
8. Benchmarking. Es un proceso en el cual se identifican las mejores prácticas en
un determinado proceso o actividad, se analizan y se incorporan a la operativi-
dad interna de la empresa. El Benchmarking es un proceso continuo de gestión
y automejora.
9. La reingeniería de procesos. Es una técnica en la cual se analiza en profun-
didad el funcionamiento de uno o varios procesos fundamentales dentro de una
empresa, con el fin de rediseñarlos por completo y mejorarlos radicalmente.
La reingeniería de procesos sigue el siguiente método estructurado: 1) identificar
los procesos clave de la empresa 2) asignar responsabilidades de dicho proceso
a un propietario 3) definir los límites del proceso 4) medir el funcionamiento del
proceso 5) rediseñar el proceso para mejorar su funcionamiento.

Actividad propuesta
Determine las fortalezas y debilidades que tienen estas herramientas (5S, 6 Sig-
ma…) para mejorar la calidad ¿Cuándo es pertinente su utilización?

268
Sánchez Marchán, Nanci. (2014).Introducción a la Ingeniería Industrial, Material Instruccional de Apoyo. Caracas: UNA

fortalezas debilidades utilidad


5s
6 Sigma
Lens 6 Sigma
QDF
Poka – Yoke
.
.
.
La Reingeniería
de Procesos

En un negocio es necesario medir en términos económicos las áreas de oportunidad


y el impacto monetario de los avances del programa de mejora que está implementando la
organización, para optimizar los esfuerzos y lograr mejores niveles de calidad. Por ello, es
importante considerar los costos de calidad involucrados.

Al considerar estos costos se ha de tener presente su valor y peso específico dentro


de la administración de un negocio, como por ejemplo los costos incurridos en el diseño,
implementación, operación y mantenimiento de los sistemas de calidad de una organización,
aquellos costos de la organización comprometidos en los procesos de mejoramiento conti-
nuo de la calidad, así como los costos de sistemas, productos y servicios frustrados o que
han fracasado al no tener en el mercado el éxito que se esperaba. A continuación se detallan
estos costos de calidad.

COSTOS DE CALIDAD
Representan el dinero destinado a la obtención de la calidad requerida en los productos;
evitando ineficiencias, problemas y pérdidas que se originan en la producción, además de la
insatisfacción del cliente.

Costos de calidad o de conformidad

Son los gastos generados para asegurar que los productos, los servicios, los procesos
y/o los sistemas cumplan con los requerimientos. Se subdividen en:
•• Costos de prevención.
•• Costos de evaluación.

Costos de prevención

Son los costos de todas las actividades específicamente diseñadas para prevenir fallas
de calidad en productos o servicios, tales como:

269
Sánchez Marchán, Nanci. (2014).Introducción a la Ingeniería Industrial, Material Instruccional de Apoyo. Caracas: UNA

•• Revisión de nuevos productos.


•• Planeación de la calidad (manuales, procedimientos, etc.)
•• Evaluación de capacidad de proveedores.
•• Esfuerzos de mejora a través de trabajo en equipo.
•• Proyectos de mejora continua.
•• Educación y entrenamiento con calidad…etc.

Costos por evaluación

Son los costos asociados con las actividades de medir, evaluar y auditar los productos o
servicios, para asegurar su conformidad con los estándares de calidad y requerimientos de
desempeño. Se pueden mencionar los siguientes:
•• Inspecciones con el proveedor en recibo.
•• Pruebas e inspecciones en proceso y al producto terminado.
•• Auditorias al producto, proceso o servicio.
•• Calibración de equipos de prueba y medición.
•• Costos de materiales de prueba.

Costos de no calidad o no conformidad

Son los costos ocasionados por no cumplir con los requerimientos de los productos,
servicios, procesos y/o sistemas. Se subdividen en:

Costos por fallas internas

Son los costos resultantes de productos o servicios no conformes a los requerimientos


o necesidades del cliente, antes del embarque del artículo o la realización del servicio. Entre
éstos podemos citar los siguientes:
•• Desperdicio.
•• Retrabajo.
•• Reinspección y repetición de pruebas.
•• Revisión de materiales no conformes.
•• Reducción de precio por calidad reducida.

Costos por fallas externas

Son los costos resultantes de productos o servicios no conformes a los requerimientos


o necesidades del cliente, después de la entrega del producto o durante y después de la
realización del servicio, como por ejemplo:
•• Proceso de quejas y reclamaciones.
•• Devoluciones del cliente.
•• Garantías.
•• Campañas por productos defectuosos.

270
Sánchez Marchán, Nanci. (2014).Introducción a la Ingeniería Industrial, Material Instruccional de Apoyo. Caracas: UNA

Costos totales de calidad

Es la suma de los costos de prevención, evaluación, falla interna y falla externa.

Costos ocultos adicionales

Son los costos intangibles resultantes de productos o servicios no conformes a los re-
querimientos o necesidades del cliente, a veces pueden ser de 3 ó 4 veces el costo de cali-
dad, es decir tiene un efecto multiplicador. Por ejemplo:
•• Tiempo de ingeniería, tiempo de gerentes.
•• Tiempos muertos en producción.
•• Incrementos en inventarios.
•• Reducción de capacidad, órdenes pérdidas, entre otros.

Modelo tradicional de los costos de calidad

Figura 64.Modelo tradicional de los costos de calidad.

Las tres curvas que representantes de los costos de calidad, de no calidad y del costos
total de la calidad, están funcionalmente relacionadas con la calidad de conformidad. En el

271
Sánchez Marchán, Nanci. (2014).Introducción a la Ingeniería Industrial, Material Instruccional de Apoyo. Caracas: UNA

gráfico se observa que los costos por fallos decrecen de modo continuo con el aumento de
la calidad de conformidad, dibujando un recorrido que va desde cero, cuando el 100% de los
productos cumplen las especificaciones y hasta el infinito, cuando el 100% de los productos
son defectuosos. Los costos de prevención y de evaluación son cero cuando el 100% de las
unidades son defectuosas y se elevan asintóticamente conforme se alcanzan niveles próxi-
mos a cero defectos. La curva del costo total de la calidad tiene un mínimo. Dicha minimiza-
ción se alcanzaría cuando esfuerzos adicionales de prevención y evaluación no se justifican
porque producen ahorros más pequeños en costos de fallos.

Actividad propuesta
Investigue hacia dónde se dirige la medición de los costos de calidad en una
empresa.
La empresa de producciones metálicas, METALICAR C.A., sólo cuenta con un
modelo, en el que se registran algunos de los costos de no conformidades tanto
internas (cuando se detecta dentro de la empresa) como externas (cuando lo de-
tecta el cliente). Éste no permite realizar el control, es engorroso su registro y casi
nunca se utiliza la información. Además se desconoce en la empresa, cualquier
gasto por concepto de evaluación y prevención. Investigue cuáles podrían ser
estos costos que se omiten.

Los costos de la calidad constituyen la herramienta para el programa de mejora conti-


nua de la calidad, dado que permiten conocer dónde están los fallos internos, dónde están
los fallos externos y por tanto, si se logran reducir éstos, se reduce el costo total en la orga-
nización. Lo que permite elevar la productividad del trabajo, además de aumentar la satis-
facción del cliente.

En un mundo en constante evolución, las técnicas y métodos de gestión deben actuali-


zarse para dar respuesta a las nuevas inquietudes y necesidades de la industria. La creativi-
dad es el mayor recurso existente, debe ser convenientemente utilizado para generar nuevas
y poderosas herramientas e instrumentos de gestión.

272
Sánchez Marchán, Nanci. (2014).Introducción a la Ingeniería Industrial, Material Instruccional de Apoyo. Caracas: UNA

LOCALIZACIÓN Y DISTRIBUCIÓN EN PLANTA

Objetivo:
Señalar la importancia de la localización y distribución de una planta, para lograr
su mejor ubicación, así como, un acomodo de sus recursos en pro de obtener
bienes o servicios de manera eficiente.

•• Origen y definición de una planta industrial.


•• Localización de la planta industrial.
•• Distribución física de planta (Layout).
•• El Proceso de logística en la planta.
•• Manejo de materiales:

- Evolución del manejo de materiales.


- Factores que afectan las decisiones sobre el manejo de los materiales.
- Dispositivos para el manejo de materiales.
- Estandarización en el manejo de materiales.
- Automatización.
- Robots.

ORIGEN Y DEFINICIÓN DE UNA PLANTA INDUSTRIAL


En el pasado la distribución en planta era producto del hombre que realizaba el trabajo,
o del arquitecto que diseñaba el edificio, se mostraba un área de trabajo para una tarea o
servicio específico, pero no se manifestaba la aplicación de ningún principio. Esta situación
cambió con la llegada de la Revolución Industrial, se convirtió en un objetivo económico para
los dueños de fábricas, estudiar la ordenación de sus instalaciones. Las primeras mejoras
fueron dirigidas hacia la mecanización del equipo, luego comprobaron que un taller limpio y
ordenado era una ayuda efectiva para el movimiento de personas y materiales.

Con el tiempo, los dueños de fábricas y/o sus administradores empezaron a formar
grupos de especialistas para estudiar los problemas de distribución. Con estos profesionales
surgieron los principios de distribución en planta que se conocen hoy día.
Sánchez Marchán, Nanci. (2014).Introducción a la Ingeniería Industrial, Material Instruccional de Apoyo. Caracas: UNA

Una planta industrial es:

Un sistema formado por máquinas, equipos y otras instalaciones dispuestas adecuada-


mente en edificios o lugares apropiados, cuyo cometido es transformar materias o ener-
gías de acuerdo a un proceso básico preestablecido.

La función del hombre dentro de este sistema es la utilización racional de estos elemen-
tos, para obtener mayor rendimiento de los espacios y equipos.

La planeación de la localización y distribución en planta se encuentran dentro de los


riesgos industriales antes de operar. Estos riesgos en la fase de planificación generalmente
son mínimos, pero también existe la posibilidad de que una planificación errónea pueda cau-
sar grandes pérdidas.

POSIBLES ESCENARIOS PARA LA LOCALIZACIÓN DE UNA PLANTA


Entre los posibles escenarios para localizar una planta se encuentran:

Súper localización: cuando se presentan casos de organizaciones transnacio-


nales que deben escoger a nivel mundial una nación o país con ciertos patrones
atractivos, para la instalación de una planta productora o una sucursal, con la idea
de expandir sus operaciones.

Macro localización: se concibe cuando una empresa de carácter nacional ana-


liza varias regiones dentro de su nación o país, para fijar sus operaciones de
producción o de servicios.

Micro localización: se presenta cuando una empresa analiza varios lugares es-
tratégicos de una ciudad, para realizar la instalación de su planta.

LOCALIZACIÓN DE UNA PLANTA INDUSTRIAL


En la fase de operación, el proceso de ubicación del lugar adecuado para instalar una
planta industrial requiere el análisis de diversos factores tales como los puntos de vista eco-
nómico, social, tecnológico y del mercado entre otros. Realizar una localización industrial
significa que se puede ir de una localización macro (en una región de un país) a una locali-
zación micro (la comunidad de esa región).

La localización de la planta se debe planear cuidadosamente, ya que es costoso cam-


biarla después. Con los avances de la comunicación y el transporte, la distancia tiene ahora
menos importancia en esta tarea. Un aspecto que si tiene gran importancia, es la escogencia
de un sitio para instalar la planta lo suficientemente grande para permitir su ampliación futura.

274
Sánchez Marchán, Nanci. (2014).Introducción a la Ingeniería Industrial, Material Instruccional de Apoyo. Caracas: UNA

Procedimientos previos para la planeación de la ubicación de una planta:

Estudio preliminar: es un proceso de planeación para determinar sitios viables,


después de identificar ciertos factores clave. La administración debe emprender
una búsqueda de disposiciones opcionales que parezcan compatibles con los
requerimientos generales.

Análisis detallado: puede llevarse a cabo una investigación sobre la mano de


obra, para evaluar la disponibilidad de la ubicación más práctica. La reacción de
la comunidad es importante para evaluar las actitudes prevalecientes e instru-
mentar estrategias que acarreen una aceptación favorable dentro de ésta, las
encuestas de opinión pueden ser de gran utilidad en este sentido.

El problema de la localización se puede abordar en dos (2) etapas:

En la primera se decide la zona general en la cual se instala la empresa y se puede


considerar que consta de tres (3) pasos los cuales son:
•• Elegir el territorio o región en general.
•• Escoger la localidad particular dentro de la región.
•• Seleccionar dentro de la localidad el lugar en específico para la planta.

En la segunda etapa se elige el sitio preciso de ubicación de la planta, considerando


los detalles de costo de terrenos, facilidades administrativas, entre otras. El estudio del des-
plazamiento consiste en analizar las variables, que se llaman Fuerzas de locación, a fin de
buscar la localización donde la resultante de estas fuerzas produzca una máxima tasa de
ganancia o un mínimo costo unitario. Los principales elementos de juicio a considerar son:
•• La suma de los costos de transporte de insumos.
•• La disponibilidad y costos relativos de los recursos.
•• La influencia de factores como tributación y problemas legales, condi-
ciones generales de vida, facilidades adquisitivas, políticas de descen-
tralización o de centralización, disposición de aguas residuales, olores y
ruidos molestos, entre otros.

Es importante saber que los factores que influyen en la localización industrial han sido
agrupados de muchas maneras, pero en el fondo con la misma base conceptual. Los facto-
res básicos que gobiernan corrientemente la evaluación para la localización de fábricas son:

•• Localización de materiales de producción. •• Agua.


•• Mano de obra. •• Condiciones de vida.
•• Terrenos disponibles. •• Leyes y reglamentos.
•• Combustible industrial. •• Estructura tributaria.
•• Facilidad de transporte. •• Clima.
•• Mercado. •• Entre otras.
•• Facilidades de distribución.
•• Energía.

275
Sánchez Marchán, Nanci. (2014).Introducción a la Ingeniería Industrial, Material Instruccional de Apoyo. Caracas: UNA

Pasos y criterios para la localización de una planta

En cada paso que se presenta a continuación se puede visualizar también un grupo de


criterios a considerar en el momento de localizar una planta:

Paso 1) Seleccionar un país concreto

Criterios para seleccionar un país o países:


•• Estabilidad gubernamental. •• Disponibilidad de materias primas.
•• Regulaciones gubernamentales. •• Número y proximidad de proveedores.
•• Sistema político y económico. •• Sistema de transporte.
•• Estabilidad económica. •• Mercado laboral y su costo.
•• Tipos de cambio. •• Tecnología disponible.
•• Cultura. •• Personal comercial.
•• Clima. •• Conocimientos técnicos.

Paso 2) Determinar la región del país donde se ubicará la planta

Criterios para seleccionar regiones:


•• Trabajo (disponibilidad, costo y sindi- •• Regulaciones gubernamentales.
calización). •• Regulaciones ambientales.
•• Proximidad de los clientes. •• Disponibilidad de materia prima.
•• Número de posibles clientes. •• Personal comercial.
•• Costos de construcción y de alquiler. •• Clima.
•• Sistemas y calidad de transporte. •• Servicios públicos.
•• Costos de terrenos.

Paso 3) Selección de la ciudad donde se instalará la planta

Criterios a utilizar en la selección de ciudades:

•• Gobierno de la comunidad. •• Impuestos.


•• Regulaciones locales sobre negocio. •• Costos de construcción y alquiler.
•• Regulaciones ambientales. •• Costos de terrenos.
•• Servicios de la administración (cá- •• Disponibilidad de lugares.
mara de comercio, etc). •• Servicios financieros.
•• Ambiente de negocio. •• Mercado laboral.
•• Servicios de la comunidad. •• Estímulos de la comunidad.
•• Sistema de transporte. •• Proximidad de los clientes.

Paso 4) selección del lugar especifico donde se instalará la planta. Los factores que
influyen en la selección del sitio, son más específicos y dependen del sector industrial (pri-
mario, secundario, terciario) al cual pertenezca la planta.

276
Sánchez Marchán, Nanci. (2014).Introducción a la Ingeniería Industrial, Material Instruccional de Apoyo. Caracas: UNA

Criterios para selección de lugares especificos dentro de la ciudad:

•• Base de clientes. •• Restricciones en la división de parcelas.


•• Costos de construcción y alquiler. •• Transporte.
•• Costos de terrenos. •• Vialidad.
•• Tamaño del lugar. •• Seguridad.
•• Servicios. •• Competencia.

Los criterios de evaluación que prevalecen son la comodidad o los accesos para la
atención al cliente.

MÉTODOS DE EVALUACIÓN PARA LA LOCALIZACIÓN


Permiten evaluar alternativas de localización. Pueden ser cualitativos o cuantitativos.

Métodos cualitativos

Entre los métodos cualitativos más importantes se encuentran los siguientes:

1. Método Delfi.

2. Distribución física de planta (layout)

1) Método Delfi

Es aplicado en situaciones complejas de problemas de ubicación y distribución de plan-


ta. El modelo es desarrollado por un equipo coordinador, el equipo pronosticador y el equipo
estratégico. Se identifican aquí tendencias, desarrollo y oportunidades; además de los pun-
tos fuertes y débiles de la organización.

Desarrollo del método Delfi:

1. Formar dos grupos Delfi: un grupo para vaticinar las tendencias en los ambientes
social y físico que afecten a la organización (grupo vaticinador), y el segundo grupo
es para identificar las metas y prioridades estratégicas de la organización (el grupo
estratégico Delfi).

2. Identificar amenazas y oportunidades: el equipo coordinador, mediante varias


aplicaciones de cuestionarios y de retroalimentación, le solicita al equipo vaticinador
Delfi que identifique las principales tendencias y oportunidades del mercado, así
como, las posibles amenazas que se debe prevenir la organización.

277
Sánchez Marchán, Nanci. (2014).Introducción a la Ingeniería Industrial, Material Instruccional de Apoyo. Caracas: UNA

3. Determinar la(s) dirección(es) y las metas estratégicas de la organización: el grupo


estratégico utiliza las conclusiones de la investigación Delfi del grupo vaticinador.

4. Desarrollar alternativas: se inicia una vez establecida la meta a largo plazo por el
grupo estratégico, el cual debe ser capaz de generar diversas alternativas de solu-
ción al problema.

5. Jerarquizar las alternativas: el conjunto de alternativas del paso anterior se presen-


tan al grupo estratégico Delfi para que se le asignen juicios subjetivos de valor.

Posterior a la localización se plantea la siguiente incógnita ¿qué tipo de distribución


se debe tener?, en este punto es preciso definir el tipo de proceso de fabricación para elegir
el mejor tipo de distribución del equipo y maquinaria.

2. Distribución física de planta (Layout)

Las decisiones sobre disposición de planta (Layout) se refieren al arreglo de equipo,


personas, materiales y facilidades dentro de una planta para producir bienes o servicios en
forma óptima. Sus objetivos son:

•• Permitir que los materiales, personas e información fluyan en el proceso


de manera eficiente y segura.
•• Minimizar tiempos muertos y esfuerzos redundantes.
•• Minimizar inventarios en proceso.
•• Minimizar costos de operación y mantenimiento.
•• Proveer lugar de trabajo adecuado y seguro.

La distribución física en plantas se puede clasificar de la siguiente forma:

Distribución de planta orientada al proceso: son adecuadas para operaciones in-


termitentes cuando los flujos de trabajo no están normalizados para todas las unidades de
producción. Los centros o departamentos de trabajo involucrados se agrupan por el tipo de
función que realizan.

Distribución en planta orientada al producto: se adopta cuando se fabrica un pro-


ducto estandarizado, por lo común en gran volumen. Cada una de las unidades en produc-
ción requiere de la misma secuencia de operaciones de principio a fin.

Distribución en planta por componente fijo: se requiere cuando, a causa del tama-
ño, conformación, o cualquier otra característica, no es posible desplazar el producto. En una
distribución de planta fija el producto no cambia de lugar; herramientas, equipo y fuerza de

278
Sánchez Marchán, Nanci. (2014).Introducción a la Ingeniería Industrial, Material Instruccional de Apoyo. Caracas: UNA

trabajo se llevan hasta él, según se requiere, a fin de ejecutar etapas apropiadas de elabo-
ración progresista.

Distribución de planta combinada: comúnmente no existen las distribuciones de


planta puras y se tiene que adoptar una distribución de planta combinada. Esto es lo más
usual en el caso de procesos y productos.

Se muestran a continuación ejemplos generales de algunos tipos de distribución o La-


yout:

Ejemplo Ilustrativo 1: Distribución por proceso.

Los procesos y servicios similares son dispuestos en zonas comunes. En general se


usa en procesos tipo Job Shop (taller de trabajo), ya que el bajo y volumen de producción así
lo justifica. Esta distribución es característica en empresas de servicios.

279
Sánchez Marchán, Nanci. (2014).Introducción a la Ingeniería Industrial, Material Instruccional de Apoyo. Caracas: UNA

Ejemplo Ilustrativo 2: Distribución por producto.

El volumen de producción es grande. Los equipos y servicios auxiliares se disponen de


acuerdo a la secuencia de elaboración del producto. Un buen ejemplo son las líneas de pro-
ducción o de montaje. Distintas partes de la planta se especializan en familias de productos
diferentes.

Un objetivo importante en el Layout por producto es que las etapas de la secuen-


cia del diagrama de recorrido, tengan la misma tasa de utilización.

HERRAMIENTAS PARA EL DISEÑO DEL LAYOUT


•• Matriz de relaciones (REL).
•• Planificación Sistemática de Layout ( SLP).
•• Teoría de grafos.
•• Otras.

Matriz de relaciones (REL)

280
Sánchez Marchán, Nanci. (2014).Introducción a la Ingeniería Industrial, Material Instruccional de Apoyo. Caracas: UNA

Generalmente, las relaciones entre departamentos son expresadas en términos cuan-


titativos de costos de transporte, sin considerar los no menos importantes factores cualita-
tivos. Para superar este problema, se construye la matriz de relación de actividades REL,
aplicada considerando tanto los factores cuantitativos como los factores cualitativos.

Características:
•• La matriz REL es una metodología que condensa la información respecto
a la importancia relativa de las ubicaciones entre parejas de secciones.
•• Se identifica la importancia relativa que tienen entre sí cada pareja de
departamentos, para estar ubicados en forma contigua.
•• Las relaciones entre departamentos se clasifican mediante factores cua-
litativos de puntuación (A, E,...X)
•• Dentro de un mismo factor de puntuación, se asignan códigos (distintos
números) para identificar la relación particular de cercanía.
•• Todas las relaciones se evalúan. Para N actividades, se realizan N(N-1)/2
evaluaciones.

Tabla 36. Nomenclatura y codificación de la matriz de relaciones (REL):

Clave Valor Identificación Razón de relación Ejemplo de actividades

1 A Absolutamente necesario Contacto personal Recepción de materiales


Continuidad en
2 E Especialmente importante Almacén de materiales
proceso
Operación( Torneado, fresa-
3 I Importante Ruido molesto do, cepillado, corte, ensam-
blado, entre otras)
4 O Ordinario Luz Traslado
Comportamiento
5 U No importante Material en espera
de suministros
6 X No deseable Ventilación Inspección
.
..
n

La razón y actividades pueden variar, dependen del proceso productivo y distribución.

281
Sánchez Marchán, Nanci. (2014).Introducción a la Ingeniería Industrial, Material Instruccional de Apoyo. Caracas: UNA

Representación gráfica general de una Matriz REL:

Figura 65.Representación gráfica de una Matriz REL.

Actividad propuesta
Indague respecto a la importancia que tiene la matriz REL, en la planificación del
Layout.

Planificación Sistemática de LAYOUT (SLP)

Es recomendable cuando el flujo numérico de artículos o recursos entre departamentos


no resulta práctico, o no se revelan otros factores cualitativos que resultan decisivos para la
disposición relativa final.

Etapas del SLP

1. Localización: determinar el área de estudio

2. Layout general: asignación de las secciones para cada uno de los niveles (pisos
- planta) en el área de estudio

3. Layout de detalle: distribución de secciones para cada nivel (piso)

4. Instalación: distribución el interior de cada sección

282
Sánchez Marchán, Nanci. (2014).Introducción a la Ingeniería Industrial, Material Instruccional de Apoyo. Caracas: UNA

Requiere de los siguientes pasos:


•• Elaborar un diagrama con la importancia relativa entre los departamen-
tos adyacentes (matriz REL)
•• Hacer un diagrama de relación de actividades
•• Ajustarlo mediante prueba y error, hasta encontrar un diagrama de adya-
cencia satisfactorio (disposición relativa)
•• Finalmente, el diagrama se ajusta a las restricciones de espacio físico de
la instalación (dimensionamiento)

Teoría de grafos

Corresponde a una herramienta matemática heurística para el diseño de Layout. Un


grafo está constituido por:
•• Nodos (N): secciones (departamentos)
•• Arcos (A): flujo de recursos

Ejemplo Ilustrativo: Layouty su versión en grafo:

Actividad propuesta
Sobre la figura del grafo mostrado, elabore el Layout correspondiente.

Método de asignación relativa a la computarización de instalaciones (CRAFT).

Programa heurístico cuyo resultado está condicionado por el Layout inicial que se le da
como punto de partida. Lo usual es correrlo con varios Layouts iníciales distintos. Maneja
hasta 40 departamentos.

Requiere la siguiente información:


•• Layout inicial.
•• Flujo de recursos entre departamentos.
•• Costo de transporte entre departamento.
•• Número y ubicación de departamentos fijos.

283
Sánchez Marchán, Nanci. (2014).Introducción a la Ingeniería Industrial, Material Instruccional de Apoyo. Caracas: UNA

Otras herramientas de diseño de Layout

Estas herramientas consideran un número mayor de detalles, aspectos más sofistica-


dos y son aplicadas en función del problema del Layout a tratar. Se valen de evaluaciones de
preferencia, las que poseen la ventaja de ser aplicables incluso en términos de las caracte-
rísticas que tienen importancia para la instalación de un departamento.

Esencialmente constituidas por algoritmos implementados a través de las computado-


res. Estos algoritmos se clasifican en:

Algoritmos Constructivos: en ellos se realiza una selección sucesiva de la ubicación


de los distintos departamentos hasta lograr un Layout final.

Ejemplos:
•• CORELAP: Computarized Relationship Layout Planning.
•• ALDEP : Automated Layout Design Program.
•• Otros : PLANET, RMA, LSP, LAYOPT.

Algoritmos optimizadores: se parte de un Layout inicial, luego se evalúan distintos in-


tercambios entre los departamentos, según algún criterio y si el cambio es favorable, se hace
permanente. El algoritmos más conocido de este tipo es el CRAFT (Computarized relative
allocation of facilities technique).

Algoritmos apoyados en la teoría de grafos: se realizan algoritmos aplicando la teo-


ría de grafos descrita anteriormente y se genera un programa para utilizar en la computadora
y obtener una solución.

Las buenas distribuciones se proyectan a partir de la maquinaria y el equipo, los cuales


a su vez, están basadas en los procesos y métodos, siempre que un proyecto de distribu-
ción esté en marcha, se deberán reexaminar los métodos y procesos y cuando se necesite
adoptar nuevos métodos o instalar nueva maquinaria, será un buen momento para evaluar
de nuevo toda la distribución.

MÉTODOS CUANTITATIVOS PARA LA LOCALIZACIÓN


Entre los métodos cuantitativos más usados se encuentran los siguientes:

1. Método de los factores ponderados.

2. Método de punto muerto

3. Método del centro de gravedad.

4. Método de transporte.

284
Sánchez Marchán, Nanci. (2014).Introducción a la Ingeniería Industrial, Material Instruccional de Apoyo. Caracas: UNA

Método de los factores ponderados

Pasos:

1. Determinar una relación de los factores relevantes.

2. Asignar un peso a cada factor que refleje su importancia relativa.

3. Fijar una escala a cada factor. Ejm: 1-10 ó 1-100 puntos.

4. Hacer que los directivos evalúen cada localización para cada factor.

5. Multiplicar la puntuación por los pesos para cada factor y obtener el total para
cada localización.

6. Hacer una recomendación basada en la localización que haya obtenido la mayor


puntuación, sin dejar de tener en cuenta los resultados obtenidos a través de
métodos cuantitativos.

Ejemplo Ilustrativo
El equipo de estudio creado para la localización de una nueva planta de fabrica-
ción ha identificado un conjunto de criterios importantes para el éxito de la deci-
sión; al mismo tiempo, ha distinguido el grado de importancia de cada una de las
alternativas en una escala de 0 a 10. Todo esto se recoge en la tabla observada.

Factores Peso relativo Alternativas


(%) A B C
Proximidad a proveedores 30 7 7 10
Costos laborales 30 5 9 7
Transportes 20 9 6 6
Impuestos 15 6 6 7
Costos de instalación 5 7 8 2
Puntuación total 6,65 7,3 7,45

•• La puntuación total para cada alternativa se calcula como la suma de las


puntuaciones para cada factor ponderadas según su importancia relativa.
•• Así, por ejemplo, la puntuación total recibida por la alternativa A se ob-
tendría como:
•• PA = 7 x 0,30 + 5 x 0,30 + 9 x 0,20 + 6 x 0,15 + 7 x 0,05 = 6,65.

285
Sánchez Marchán, Nanci. (2014).Introducción a la Ingeniería Industrial, Material Instruccional de Apoyo. Caracas: UNA

•• Las alternativas B y C parecen ser mejores que A, por lo que se podrá


rechazar esta última.
•• Entre las dos restantes, hay una pequeña diferencia a favor de C, aunque
quizás no definitiva.
•• Vemos que C tiene la ventaja principal de estar muy próxima a la fuen-
te de abastecimiento de materia prima, lo cual es un factor importante,
mientras que su punto débil es el costo de instalación, que es bastante
elevado. 

Método de punto muerto

Examina los costos y volúmenes de producción en un gráfico, para establecer compa-


raciones entre alternativas.

Pasos:

1. Se determina los costos fijos y variables para cada localización, además de es-
tablecer la ecuación de la recta que representa el costo total.
2. Se elabora un gráfico donde el volumen de producción se ubica en el eje Y, y
los costos en el eje X.
3. Se selecciona la localización que ofrezca el costo total mínimo.

Ejemplo Ilustrativo
Una empresa desea realizar el estudio para la ubicación de una nueva planta,
sobre la base de los costos, ya que el ingreso por ventas no se verá afectado
por la misma, es decir, se supone que venderá la misma cantidad, independien-
temente de donde se instale. La empresa estudia cuatro posibles alternativas,
para los cuales ha estimado los costos fijos y variables que aparecen en la
siguiente tabla:
Tipos de costos Sitios a elegir
A B C D
Alquileres 140 200 300 250
Fijos Impuestos 100 300 400 300
Producción 360 400 500 350
Otros 300 400 400 350
Totales 900 1300 1600 1250
Materiales 5 3 4 5
Mano de obra 6 5 8 8
Variables Transporte 7 6 2 3
Otros 3 3 1 3
Totales 21 17 15 19
CT = Costos fijos+ Costos variables

286
Sánchez Marchán, Nanci. (2014).Introducción a la Ingeniería Industrial, Material Instruccional de Apoyo. Caracas: UNA

Sobre la base de los datos y cálculos de costos totales presentados se puede decir que:
•• La opción A es la que provoca menores costos fijos en lo que se refiere
a impuestos y alquileres. Por el contrario, el costo variable es bastante
alto al tratarse de una zona más alejada, lo que provoca mayores costos
de transporte de materias primas, personal, etc.
•• La ubicación B tiene la ventaja de ofrecer mano de obra más barata, así
como aprovisionamiento bastante económicos.
•• Por lo que respecta a la alternativa C, resulta ser justamente lo contrario
de A; sus costos fijos son más elevados pero los variables son los más
reducidos.
•• La opción D, por su parte, está en una posición intermedia tanto en cos-
tos fijos como en variables.

El análisis realizado a las diferentes opciones, demuestra que la opción más atractiva es
la (B) ya que tiene el costo de mano de obra y aprovisionamientos más baratos, además los
costos totales representan una media.

Método del centro de gravedad

Es una técnica que se utiliza para la localización de plantas de fabricación o almacenes


de distribución, en la que se consideran las instalaciones existentes, las distancias que las
separan y los volúmenes de artículos que se han de despachar.

287
Sánchez Marchán, Nanci. (2014).Introducción a la Ingeniería Industrial, Material Instruccional de Apoyo. Caracas: UNA

Procedimiento:

Se empieza colocando ubicaciones existentes en un sistema de coordenadas. El ob-


jetivo es establecer las distancias relativas entre las ubicaciones. El centro de gravedad se
encuentra calculando las coordenadas (X) e (Y) que dan por resultado el costo mínimo de
transporte.

Fórmulas:

•• Cx = Coordenada X del centro de gravedad.


•• Cy = Coordenada Y del centro de gravedad.
•• dix = Coordenada X de ubicación.
•• diy = Coordenada Y de ubicación.
•• Vi = Volumen de artículos movilizados hasta la ubicación o desde ella.

Ejemplo Ilustrativo
PDVSA necesita ubicar una instalación de distribución intermedia entre sus refine-
rías, Bajo Grande, Amuay, Cardón, el Palito y Puerto la Cruz. En la tabla adjunta
aparece el mapa de coordenadas. La cantidad de gasolina despachada desde las
refinerías hasta la nueva instalación para su distribución.

Ubicaciones Galones de
gasolina /mes

(000,000)

Pto la Cruz 1500

El Palito 250

Bajo Grande 450

Amuay 350

Cardón 450

Solución

CX = (325 x 1,500) + (400 x 250) + (450 x 450) + (350 x 350) + (25 x 450)
1,500 + 250 + 450 + 350 + 450

288
Sánchez Marchán, Nanci. (2014).Introducción a la Ingeniería Industrial, Material Instruccional de Apoyo. Caracas: UNA

CX = 923,750 →Cx= 307.9


3,000

Cy= (75 x 1,500) + (150 x 250) + (350 x 450) + (400 x 350) + (450 x 450)
1,500 + 250 + 450 + 350 + 450

Cy = 650,000 →Cy =216.7


3,000

Centro de gravedad = (3.08, 217)

Interpretación del problema

La gerencia tiene como coordenadas X e Y en valores aproximados de 308 y 217 res-


pectivamente. Y proporciona un punto de arranque para buscar un nuevo sitio.
•• Según el centro de gravedad en el mapa: podría ser más eficiente en
materia de costos ubicarla nueva instalación para su distribución cerca
de la planta de Puerto la Cruz, por encontrarse más cerca del punto de
gravedad (325, 200)
•• La gerencia puede volver a calcular el centro de gravedad, analizando
otras situaciones.

Método de transporte
•• El método de transporte de la programación lineal, puede emplearse
para someter a prueba el impacto que en materia de costos, tienen las
diversas ubicaciones posibles.
•• Encontrar el plan de distribución para satisfacer la demanda con el míni-
mo costo de transportes entre plantas y almacenes.

Ejemplo Ilustrativo

Planteamiento

1. Variables:
Xij : cantidad a transportar desde i hasta j (km) constantes
Cij : costos de transporte desde i a j (Bs/ km)
Di : disponibilidades en las diferentes plantas
Bj : demandas en los almacenes
2. Función Objetivo: Min Z = Cij*Xij

289
Sánchez Marchán, Nanci. (2014).Introducción a la Ingeniería Industrial, Material Instruccional de Apoyo. Caracas: UNA

3. Restricciones:
•• Xij → Dij Xij → Bj
•• ∑Cij *Xij =Bij
•• ∑Cij *Xij =Dij
•• ∑Bj =∑Di
•• Xij ≥ 0

Matriz de transporte:

ORIGEN/DESTINO 1 2 3 j
PLANTAS

1 C1 X1 D1
2 D2
3 D3

i Cij X Di
B1 B2 B3 Bj

Es de hacer notar que el método de transporte es una herramienta de la Investigación


de Operaciones, el cual se abordará cuando el estudiante avance en el estudio de su carrera.

Actividad propuesta
Investigue respecto a la aplicación e importancia que tiene el método de transpor-
te para la Ingeniería Industrial.

PASOS A SEGUIR PARA MEJORAR LA DISTRIBUCIÓN EN PLANTA

1. Elaborar el plano actual de la planta de producción, dibujando la ubicación de las


máquinas y puestos de trabajo. Teniendo en cuenta las siguientes instrucciones:

a. Tomar las medidas de la planta de producción y dibujar un plano de ella. Para


facilitar la manipulación del plano, se reducen las medidas, por ejemplo un metro
se puede hacer igual a un centímetro.
b. Sobre este dibujo técnico se resaltan las partes que no se pueden mover como
columnas, baños, entre otras.
c. Medir el área de cada una de las máquinas y puestos de trabajo que se pueden
movilizar. Reducir estas medidas de igual forma que se redujo las medidas de la
planta, se trazan sobre un papel.

290
Sánchez Marchán, Nanci. (2014).Introducción a la Ingeniería Industrial, Material Instruccional de Apoyo. Caracas: UNA

d. Recortar las áreas trazadas y escribir sobre ellas el nombre de la máquina o


equipo que representa.

2. Hacer un análisis de la distribución de planta actual de la empresa. Colocar las


máquinas y puestos de trabajo sobre el plano, en la secuencia de operaciones que
sigue el producto para ensayar diferentes colocaciones.

3. En caso de que las máquinas y puestos de trabajo en la situación actual estén mal
ubicados, se elige la distribución de planta que se considere más conveniente para
mejorar esa situación, es decir, se debe rediseñar la planta. Esta mejora permitirá al
operario realizar su labor, sin interrumpir el trabajo de los demás.

Causas del rediseño de una planta (Layout)

Una planta puede requerir de un rediseño o redistribución de sus espacios, cuando ex-
perimenta cambios como:
•• Variación en la cartera de productos.
•• Cambio de procesos o tecnologías.
•• Cambios en el volumen de producción:
– Periódicos.
– Continuos.
– Esporádicos - Circunstancial.

Ejemplo Ilustrativo Nº 1
En la tienda por departamento YORJEKA, se clasifica la mercancía a exigir en
cuatro ambientes: alimentos, hogar, cuerpo y mercería. Para la tienda es primor-
dial la asociación mental que se haga de los productos, como por ejemplo ubicar
los destapadores cerca de las botellas. La distribución en planta o Layout que
tiene el almacén actualmente es la siguiente:

HOGAR CONTROL
(1)
ENTRADA

(2)
CUERPO

(3)
MERCERÍA

(4) CAJAS (8)


(6)

ALIMENTOS SALIDA
(5) (7)

Dentro de cada ambiente existen atractivos y se busca ubicar la mercancía más deman-
dada a la vista, siguiendo los manuales de procedimiento de la empresa. La reposición se
realiza a diario. Existen fechas y eventos especiales, los cual influyen expandiendo o dismi-

291
Sánchez Marchán, Nanci. (2014).Introducción a la Ingeniería Industrial, Material Instruccional de Apoyo. Caracas: UNA

nuyendo el espacio de cada ambiente. El almacén también persigue aumentar las ventas y
disminuir los costos de manejo de la mercancía, para aumentar la rentabilidad.

Sobre la base del análisis descrito y en función de los objetivos que persigue YORJEKA,
considerar primero las necesidades del cliente facilitándole los productos que éste solicita
y no los que el almacén quiere vender, se plantea la siguiente propuesta de Layout para la
empresa YORJEKA:

HOGAR CUERPO MERCERÍA


(1)
ENTRADA

(8)
SALIDA
(2)
ALIMENTOS
(3) (4) (5)

CONTROL
CAJAS (6) (7)

Ejemplo Ilustrativo Nº 2
Se puede observar a continuación un ejemplo de una distribución en planta (La-
yout) por proceso:

Recepción Fresadoras Planeación Esmeriladoras Embarque


(1) (3) (4) (5) (11)

Almacén de Ensamble Tornos Inspección


materias primas
(8) (6) (9)
(2)

Almacén de productos Maquinaria automática


terminados
(7)
(10)

Actividad propuesta Nº 1
Proponga un Layout para el ejemplo ilustrativo Nº 2, considerando como criterio
que los procesos finales, se encuentren más cerca del embarque.

292
Sánchez Marchán, Nanci. (2014).Introducción a la Ingeniería Industrial, Material Instruccional de Apoyo. Caracas: UNA

Actividad propuesta Nº 2
Elabore una posible distribución en planta o Layout para una carnicería.

Como se expresó anteriormente con una correcta distribución en planta, se puede lo-
grar el mejor desplazamiento de materiales en el proceso productivo. Seguidamente se estu-
diarán los aspectos más importantes del manejo de materiales en una planta.

MANEJO DE MATERIALES
El manejo de materiales se corresponde con la estructura de dirección y control de todo
lo relativo a inventarios, para asegurar la eficiencia productiva de la organización.

Éste debe garantizar que las partes, materias primas, materiales en proceso, productos
terminados y suministros se desplacen periódicamente de un lugar a otro sin choques.

Cada operación del proceso requiere materiales y suministros a tiempo en un punto en


particular. El eficaz manejo de materiales asegura que éstos serán entregados en el momen-
to y lugar adecuado, en la cantidad correcta y además contar con el espacio apropiado para
el almacenamiento.

EVOLUCIÓN DEL MANEJO DE MATERIALES


La humanidad en su esfuerzo por alcanzar su desarrollo industrial inició el uso de los
principios mecánicos como fuente principal en el manejo de materiales (uso de la palanca, la
rueda, plano inclinado, etc.) y posteriormente incorporó la neumática, la hidráulica, la electri-
cidad y la electrónica y sus combinaciones como una forma más fácil, rápida y segura.

En la actualidad los procesos industriales hacen uso de tecnologías de punta en los


equipos y sistemas de manejo de materiales, lo cual permite incrementar la productividad en
la empresa y lograr una ventaja competitiva en el mercado. El manejo de materiales implica
movimiento, tiempo, cantidad y espacio, lo que genera costos, por ello es conveniente lograr
un modelo automatizado para este fin.

DEFINICIÓN
El manejo de materiales se puede definir como:

La orientación de todo lo relativo al flujo de materiales en cuanto a preparación y coloca-


ción de éstos, para facilitar su movimiento desde su adquisición hasta el almacenamiento
y distribución del producto terminado a los clientes, reforzando la productividad de la
organización.

293
Sánchez Marchán, Nanci. (2014).Introducción a la Ingeniería Industrial, Material Instruccional de Apoyo. Caracas: UNA

El manejo de materiales debe asegurar que las materias primas, material en proceso,
productos terminados y suministros se desplacen periódicamente de un lugar a otro. La
planificación es el principio fundamental de este proceso, a través de la cual se realizan las
consideraciones de espacio disponible, capacidades de las máquinas, técnicas aplicables,
entre otros. Todo ello asegura la implementación de un sistema de manejo de materiales
adaptado a las necesidades de la planta.

Las siguientes son algunas recomendaciones para la planeación de la disposición de


los materiales:
•• Evitar la ubicación de materiales directamente sobre el piso, sin una pa-
leta u otros soportes debajo de éstos.
•• Asegurar un espacio adecuado de almacenamiento en el lugar de traba-
jo para la cantidad apropiada de material, antes y durante la operación
planificada.
•• Planear el uso del mismo contenedor en todo el sistema, evitando así los
daños por cambios frecuentes de contenedores.
•• Considerar la capacidad de carga del piso, la altura del techo y el espa-
ciamiento entre columnas.
•• Cumplir con el ancho de los pasillos y alterarlo sólo de ser necesario y
posible.
•• Proveer el espacio necesario dentro y alrededor del área de trabajo para
el apropiado manejo de materiales y la maniobra del equipo de trabajo.
•• Disponer de instrucciones debidamente elaboradas para cada operador.

Esta planeación puede verse afectada por decisiones erróneas. Existen cuatro factores
que pueden perturbar las decisiones en este sentido, éstos son:
•• El tipo de sistema de producción.
•• Los productos que se van a manejar.
•• El tipo de edificio dentro del cual se van a manejar los materiales.
•• El costo que genera el manejo de los dispositivos (grúas, montacargas,
bandas transportadoras, entre otros).

Una desventaja que presenta el manejo de materiales para la producción es que agrega
poco valor al producto y consume una parte importante del presupuesto de manufactura.
Una ventaja puede ser el movimiento de hombres, máquinas, herramientas e información
como flujo que debe apoyar los objetivos de la recepción, la selección, la inspección, el in-
ventario, la contabilidad, el empaque, el ensamble y otras funciones de la producción.

Este proceso puede ser más productivo y consumir menos presupuesto de manufac-
tura, si se fundamenta en principios para lograr el mejor beneficio en cuanto a seguridad,
eficacia y eficiencia en la empresa. Se presentan 10 principios para el manejo de materiales
desarrollados por Handling Institute en 1998:

294
Sánchez Marchán, Nanci. (2014).Introducción a la Ingeniería Industrial, Material Instruccional de Apoyo. Caracas: UNA

•• Principio de planeación: todo el manejo de materiales debe ser el re-


sultado de un plan deliberado en el que se definan por completo nece-
sidades, objetivos de desempeño y especificaciones funcionales de los
métodos propuestos.
•• Principio de estandarización: métodos, equipos, controles y software
para el manejo de materiales, deben estandarizarse dentro de los límites
que logran los objetivos globales de desempeño y sin sacrificar la flexibili-
dad, modularidad y producción.
•• Principio del trabajo: el trabajo de manejo de materiales debe minimi-
zarse sin sacrificar la productividad o el nivel de servicio requerido de la
operación.
•• Principio de ergonomía: debe reconocerse la capacidad y las limitacio-
nes humanas; respetarse el diseño de las tareas, así como, de equipos de
manejo de materiales, para lograr operaciones seguras y efectivas.
•• Principio de carga unitaria: las cargas unitarias deben ser de tamaño
adecuado y configurarse de manera que logren el flujo de material y los
objetivos del inventario, en cada etapa de la cadena de proveedores.
•• Principio de utilización del espacio: debe hacerse uso efectivo y efi-
ciente de todo el espacio disponible.
•• Principio de sistema: las actividades de movimiento y almacenaje de
materiales deben estar integrados por completo para formar un sistema
operativo que abarque todo lo concerniente a recepción, inspección, al-
macenamiento, producción, ensamble, empaque, unificación, selección
de órdenes, envíos, transporte y manejo de reclamaciones.
•• Principio de automatización: las operaciones de manejo de materiales
deben mecanizarse y/o automatizarse cuando sea posible, para mejorar la
eficiencia operativa, incrementar la respuesta mejorando la consistencia.
•• Principio ambiental: el impacto ambiental y el consumo de energía son
criterios a considerar al diseñar o seleccionar el equipo en los sistemas de
manejo de materiales.
•• Principio del costo del ciclo de vida: un análisis económico exhaustivo
debe tomar en cuenta todo el ciclo de vida del equipo de manejo de mate-
riales y los sistemas que resulten.

USO ADECUADO DE LOS MATERIALES EN MOVIMIENTO

Es necesario el uso adecuado de los materiales en movimiento para no tener grandes


pérdidas de capitales, por ello se debe considerar lo siguiente:
1. Eliminar distancias. Hacer las distancias del transporte tan cortas como sea posible.
Los movimientos más cortos requieren de menos tiempo y dinero que los movimien-
tos largos, además ayudan a que la producción sea más eficiente.
2. Mantener el movimiento. Reducir el tiempo de permanencia en las terminales de una
ruta tanto como se pueda.

295
Sánchez Marchán, Nanci. (2014).Introducción a la Ingeniería Industrial, Material Instruccional de Apoyo. Caracas: UNA

3. Emplear patrones simples. Se deben reducir los cruces y otros patrones que con-
ducen a una congestión, ya que la reducción de cruces hace que la producción se
haga más ligera, tomando en cuenta la capacidad de las instalaciones.
4. Transportar cargas en ambos sentidos. Pueden lograrse sustanciales ahorros si se
pueden diseñar sistemas para el manejo de materiales que solucionen el problema
de ir o regresar sin una carga útil.
5. Transportar cargas completas. Se debe considerar un aumento en la magnitud de
las cargas unitarias disminuyendo la capacidad de carga, reduciendo la velocidad o
adquiriendo un equipo más versátil.
6. Evitar el manejo manual. Disponer de medios mecánicos que puedan hacer el tra-
bajo en forma más efectiva.
7. Los materiales deberán estar etiquetados. Es fácil colocar mal o perder los artículos
por no etiquetarlos.

DISPOSITIVOS PARA EL MANEJO DE MATERIALES


El número de dispositivos para el manejo de materiales que existen actualmente es de-
masiado grande, por lo que se describirán brevemente sólo algunos de ellos. El manejo de
materiales en masa, se realiza de manera horizontal o vertical y se puede clasificar en tres
categorías, como se observa en la tabla 37:

Tabla 37.Dispositivos para el manejo de materiales


Dispositivo Tipos Imagenes
Grúas torres: usadas para la
construcción de edificios.
Grúas: máquina de eleva-
ción de movimiento discon- Grúas auto - desplegables:
tinuo destinado a elevar y Son grúas capaces de des-
distribuir cargas en el espa- montarse por sí mismas sin
cio suspendido de un gan- necesidad de requerir otra grúa
cho. Manejan el material en para el montaje.
el aire, arriba del nivel del Grúas Telescópicas: consis-
suelo, a fin de dejar libre el ten de muchos tubos que se
piso para otros dispositivos encuentran uno dentro de otro.
de manejo. Los objetos pe- Se adaptan fácilmente a las
sados, voluminosos y pro- aplicaciones móviles.
blemáticos son candidatos
para el movimiento en el Grúas Luffing o Transtainer:
aire con grúas. son utilizadas en puertos para
el trasporte y orientación de
contenedores.

296
Sánchez Marchán, Nanci. (2014).Introducción a la Ingeniería Industrial, Material Instruccional de Apoyo. Caracas: UNA

Transportador de tobogán:
su forma ergonómica permite
deslizar cualquier tipo de pro-
ducto de manera óptima
Transportador de cinta: es-
tán constituidos por una banda
sin fin. La cinta puede ser móvil
Transportador: es un apa- o fija.
rato relativamente fijo dise- Transportador de rodillo: fun-
ñado para mover materia- ciona por medio de un motor
les, pueden tener diferentes que acciona cadenas u otros
formas. elementos.
Transportadores en forma de
ductos: utilizados para el flujo
de líquidos, gases o material
en polvo a presión o a granel.
Se colocan en el interior de las
paredes, debajo del piso o en
tendidos aéreos.

Carros operados manual-


mente (carretillas, patines, en-
tre otros): adecuados para car-
gas ligeras, desplazamientos
Los carros: son medios cortos y lugares pequeños.
de transporte de materiales
industriales livianos o pesa- Carros operados con motor:
dos, que pueden ser opera- Monta carga: pueden cargar,
dos de forma manual o por descargar, levantar y trasladar
medio de un motor. diversos materiales.
Tractores: se utilizan para mo-
ver objetos pesados y volumi-
nosos.

Ejercicio Ilustrativo
Situación: El señor Manuel Correa piensa instalar una empresa procesadora de
maíz, la cual va a registrar con el nombre de “Mazorca C.A”.
¿Qué equipo de manejo de materiales debería adquirirse en la empresa Mazorca?

Para seleccionar el equipo es necesario considerar el aspecto económico y el tipo de


material a transportar.

297
Sánchez Marchán, Nanci. (2014).Introducción a la Ingeniería Industrial, Material Instruccional de Apoyo. Caracas: UNA

El equipo que más se adapta al tipo de material que se procesará en la empresa Mazor-
ca, es aquel que utiliza acción neumática para transportar material a granel. Es un sistema
bastante práctico que origina pocos gastos de mantenimiento, ya que toda la energía para el
transporte proviene de un potente ventilador o aspirador que puede ser bien resguardado del
polvo y la intemperie. Es el sistema más usado en los molinos de cereales, ya que elimina la
producción de polvillo que puede originar contaminación ambiental.

Actividad propuesta N° 1
El señor Juan Pérez tiene pensado instalar un taller mecánico y necesita adquirir
un equipo de manejo de materiales que le facilite el trabajo en éste. ¿Qué equipo
le recomendaría usted? Por qué.

Actividad propuesta N° 2
Las piezas en un palé o paleta (plataforma o bandeja de madera o plástico) ocu-
pan normalmente posiciones predeterminadas, procurando asegurar la estabili-
dad, facilitar su manipulación y optimizar su extensión.
¿Qué tipos de equipos se utilizan para moverlas paletas?

ESTANDARIZACIÓN EN EL MANEJO DE MATERIALES


La estandarización en el manejo de materiales se fundamenta en la normalización de
los métodos, tipos y tamaños de los equipos dentro del espacio disponible. En cualquier
campo de actividad es sabio y provechoso normalizar de la mejor manera los métodos y
procesos involucrados, después de que éstos han sido determinados.

La normalización de las operaciones y los equipos permite fijar modelos productivos


que maximicen el aprovechamiento racional de los espacios y el uso de los equipos para el
manejo de materiales, convirtiéndose en el primer paso para la automatización de procesos.

De esta forma, la estandarización no significa mantener un sólo tipo o marca de equi-


po asignado a una función en particular, sino que debe ser interpretada como la utilización
práctica y eficiente del menor número de tipos, marcas, modelos y tamaños de equipos a
objeto de normalizar a nivel mundial los campos relacionados con el manejo de material en
los procesos de producción y transformación.

MECANIZACIÓN, AUTOMATIZACIÓN Y ROBOTIZACIÓN


La mecanización fue el paso inicial hacia la evolución de la automatización. La simplifi-
cación del trabajo, permitida por la división de éste, también posibilitó el diseño y construc-
ción de máquinas que reproducían los movimientos del trabajador. A medida que evolucionó

298
Sánchez Marchán, Nanci. (2014).Introducción a la Ingeniería Industrial, Material Instruccional de Apoyo. Caracas: UNA

la tecnología de transferencia de energía, estas máquinas especializadas se transformaron,


aumentando así su eficacia productiva.

Se entiende por mecanización:

La incorporación de máquinas para ejecutar determinadas tareas que son realizadas de


modo manual por el operario, logrando una mayor rapidez en el trabajo y una mejor cali-
dad en el resultado.

Se puede considerar la mecanización de procesos para:

a. Grandes cantidades o volúmenes de material.


b. Movimientos frecuentes o repetitivos (aún cuando sean cortos).
c. Movimientos de gran longitud.
d. Movimientos de gran esfuerzo, riesgosos o difíciles.
e. Actividades de carga que requieran de dos operarios.
f. Eliminar exceso de manejo manual.
g. Reemplazar personal de manejo de materiales.
h. Alimentar o remover materiales de las máquinas.
i. Mover contenidos pesados.
j. Actividades de manejo hechas directamente por operarios.
k. Remoción de escombros.
l. Reducir el tiempo de manejo.
m. Proveer tasas de manejo uniforme, controladas y máximas.

Por otra parte la automatización se puede definir como:

La acción de eliminar de forma parcial o total, la intervención de personas en la ejecución


del trabajo, a través de sistemas o elementos computarizados y electromecánicos para
controlar maquinarias y/o procesos industriales.

El propósito de la automatización es realizar ciertas tareas, con poca o ninguna partici-


pación del hombre. Las máquinas automáticas pueden repetir continuamente la acción para
la que están diseñadas, pero sin posibilidad de variar su funcionamiento. Las consecuencias
más directas son el aumento de productividad y evitar la realización de tareas desagradables
o peligrosas para el operario.

Los sistemas de procesos industriales y de control, así como de administración, han he-
cho de la automatización una herramienta indispensable para todo tipo de empresas, desde
pequeños negocios  hasta grandes compañías.

299
Sánchez Marchán, Nanci. (2014).Introducción a la Ingeniería Industrial, Material Instruccional de Apoyo. Caracas: UNA

Se puede considerar la automatización en la industria para:

1. Diseñar o seleccionar contenedores calificados para el manejo de materiales.


2. Utilizar equipos auto-controlados y auto-programables cuando sea útil.
3. Considerar automatización del flujo de personal o equipos, así como, de materiales.
4. Facilitar la comunicación en el manejo de materiales.
5. Utilizar acoplamientos, suiches y transferencias automatizadas.
6. Mover herramientas, sierras, muebles, etc.

La automatización se puede observar en casi todos los aspectos de la vida, tanto a nivel
personal como industrial. En la industrial es sumamente importante, debido a que agiliza los
procesos, aumenta la calidad de los productos y en consecuencia, el desarrollo de la com-
petitividad. El día a día de la vida de las personas no podría entenderse sin la automati-
zación, desde factores tan sencillos como programar el despertador, hasta la posibilidad de
hacer operaciones bancarias desde internet, o sacar dinero de un cajero automático.

Es por estas razones que al hablar de inversión en tecnología de automatización, no se


le debe considerar solamente como una inversión más, sino como una estrategia esencial
para la competitividad. No invertir en esta tecnología implica un riesgo alto de rápido despla-
zamiento por parte de la competencia.

Los elementos básicos de un sistema automatizado son:


•• Energía: para completar el proceso y operar el sistema.
•• Programa: para dirigir el proceso.
•• Sistema de control: para ejecutar las instrucciones

Actividad propuesta N° 1
Investigue cuál fue el primer y auténtico controlador realimentado.

Actividad propuesta N° 2
Pregunte a personas de su sitio de trabajo u hogar ¿qué se necesita para auto-
matizar una oficina? Analice qué tienen en común sus definiciones.

De todo lo descrito anteriormente se puede decir que la mecanización y automatización


son conceptos distintos. El primero se refiere a la sustitución de la energía física del hombre
por la de una máquina. El trabajo del hombre queda reducido a dirigir los materiales y a regu-
lar las máquinas que ejecutan el trabajo sin flexibilidad alguna, debido a que el programa se
ha fijado previamente. En cambio en la automatización el objetivo es sustituir al operador por
un sistema de realimentación, creado con tecnologías derivadas de la informática.

300
Sánchez Marchán, Nanci. (2014).Introducción a la Ingeniería Industrial, Material Instruccional de Apoyo. Caracas: UNA

El uso de nuevas tecnologías derivadas de la informática permite ampliar la capacidad


para controlar la producción con máquinas de control computarizado, propiciando el avance
hacia sistemas más grandes y complejos; unas de cuyas expresiones más sofisticadas y
más ahorradoras de trabajo humano directo, son los sistemas flexibles de producción, los
sistemas de automatización integrada de la producción, o los robots.

La robotización también es una automatización (no intervienen personas) pero, en este


caso, las máquinas utilizadas son capaces de realizar diferentes acciones según se las pro-
grame; además, serán capaces de captar información de su alrededor y procesarla, para
variar posteriormente su acción en función del resultado obtenido. La evolución de la robótica
está orientada a conseguir robots con mayor movilidad, con sistemas de visión avanzados,
con capacidad para entender y reproducir el lenguaje humano, con capacidad para tomar
decisiones inteligentes, etc.

El término Robot fue introducido por el escritor de ciencia ficción Karel Capek, en su
obra de teatro: Robots Universales Rossun (RUR) en 1920, desde entonces fue acogido en
la industria y sus diseños han variado, para buscar la eficiencia como auxiliares, en trabajos
que implican altos grados de mecanización y riesgo para el trabajador.

Algunas definiciones de robot:


•• “Un robot es un manipulador reprogramable y multifuncional concebido
para transportar materiales, piezas, herramientas o sistemas especia-
lizados, con movimientos variados y programados, con la finalidad de
ejecutar tareas diversas” El Robot Institute America.
•• “Se trata de un aparato mecánico que se parece y hace el trabajo del ser
humano ”El Oxford Englihs Dictionary.

El principal papel del robot es articular diferentes máquinas y funciones productivas:


transporte, manejo de materiales, maquinado, carga y descarga. También permite ejecutar
tareas tan complejas como selección de piezas por calidad, decoración dentro del molde,
paletizado, empaque automático, subensambles, inserción de piezas metálicas o plásticas
dentro del molde para sobre-inyección, etcétera. Es posible interconectar sistemas externos
de automatización con el robot y hacerlos interactuar de manera inteligente.

Se puede decir entonces que un robot industrial es:

Un dispositivo o grupo de dispositivos con controladores rápidos basados en micropro-


cesadores que ejecuta tareas de forma automática en las diferentes etapas de la produc-
ción industrial, minimizando el esfuerzo humano y los tiempos de producción.

Para programar un robot, como para cualquier programa informático, lo primero es de-
sarrollar el algoritmo que permita indicarle qué acción ha de realizar. Posteriormente será
necesario elaborar el programa informático que lo manejará, mediante las instrucciones o
comandos propios del lenguaje utilizado.

301
Sánchez Marchán, Nanci. (2014).Introducción a la Ingeniería Industrial, Material Instruccional de Apoyo. Caracas: UNA

Actividad propuesta:
Investigue cómo se obtienen las señales de entrada y salida de un robot.

Al robot lo conforman dos subsistemas: la estructura mecánica, hidráulica o eléctrica,


que comprende sus funciones de movimiento y manipulación y su estructura electrónica o
informática que le provee de memoria programable y permite sincronizarlo con otras má-
quinas. Ésta es la famosa inteligencia del robot, que le otorga flexibilidad y versatilidad, así
como capacidad para realizar diversas tareas. Esta evolución ha dado origen a una serie de
tipos de robots, que se citan a continuación:

– Manipuladores: permiten gobernar el movimiento de sus elementos, de los si-


guientes modos:
•• Manual: cuando el operario controla directamente la tarea del mani-
pulador.
•• De secuencia fija: cuando se repite, de forma invariable, el proceso de
trabajo preparado previamente.
•• De secuencia variable: Se pueden alterar algunas características de
los ciclos de trabajo.
– Robots de repetición y aprendizaje: se limitan a repetir una secuencia de mo-
vimientos, previamente ejecutada por un operador humano, haciendo uso de un
controlador manual o un dispositivo auxiliar.

– Robots con control por computador: Son manipuladores o sistemas mecáni-


cos multifuncionales, controlados por un computador, que habitualmente suele
ser un microordenador.

– Robots inteligentes: son similares a los del grupo anterior, pero además son ca-
paces de relacionarse con el mundo que les rodea a través de sensores y tomar
decisiones en tiempo real (auto programable).

– Micro-robots: se utilizan con fines educacionales, de entretenimiento o investi-


gación. Su estructura y funcionamiento son similares a los de aplicación indus-
trial.

Utilizar equipos de manejo automatizado como los robots permite incrementar la eficien-
cia del proceso de manejo de materiales. Sin embargo, la automatización debe ser justifica-
da, para no caer en autoregulaciones excesivas o innecesarias.

Puesto que el éxito de un negocio radica en satisfacer las necesidades de los


clientes, es indispensable que haya un buen manejo de materiales para evitar las
causas de las inconformidades.

302
Sánchez Marchán, Nanci. (2014).Introducción a la Ingeniería Industrial, Material Instruccional de Apoyo. Caracas: UNA

El manejo de materiales requiere almacenamiento, transporte y entrega a tiempo, en


excelentes condiciones. Para ello es necesario un proceso logístico en la planta, el cual se
abordará a continuación.

PROCESO DE LOGÍSTICA EN LA PLANTA


El uso del término logística industrial es prácticamente nuevo. La palabra logística en
otras épocas era usada principalmente en el ámbito militar, este término definía las opera-
ciones que se relacionaban con el almacenamiento, transporte y también alojamiento de los
ejércitos durante el proceso bélico. Hoy día, el término logística se relaciona directamente
con las tareas de abastecimiento, depósito y distribución de productos.

Se entiende por logística:

Al proceso de gerenciar estratégicamente el movimiento y almacenamiento de materias


primas, partes y productos terminados, desde los proveedores a través de la empresa
hasta el usuario final, debido a que se afirma que el producto adquiere su valor cuando el
cliente lo recibe en el tiempo, forma adecuada y al menor costo posible.

La logística determina y coordina, en forma óptima, el producto correcto, el cliente co-


rrecto, el lugar correcto y el tiempo correcto. Si se asume que el rol del mercadeo es estimu-
lar la demanda, el rol de la logística será precisamente satisfacerla.

De un tiempo para acá, la función logística empresarial ha tomado fuerza debido a que
los mercados se han vuelto más exigentes, las firmas tienen que competir con empresas de
todo el mundo, atendiendo de la mejor manera a todos y cada uno de sus clientes, además,
la aparición de nuevas tecnologías de información han traído como consecuencia menores
tiempos y costos de transacción, esto ha obligado a las empresas a tomar más en serio la
gestión logística si es que quieren seguir siendo competitivas.

Anteriormente la logística era solamente, tener el producto justo, en el sitio justo, en


el tiempo oportuno, al menor costo posible. Actualmente estas actividades, aparentemente
sencillas, han sido redefinidas y ahora conforman todo un proceso.

Existen dos formas básicas de logística:


•• Optimizar un flujo de material constante a través de una red de enlaces
de transporte y de centros del almacenaje.
•• Coordinar una secuencia de recursos para realizar un determinado pro-
yecto.

Sin embargo las actividades clave para tener una buena gestión logística, son las si-
guientes:
•• El servicio al cliente.
•• Los inventarios.

303
Sánchez Marchán, Nanci. (2014).Introducción a la Ingeniería Industrial, Material Instruccional de Apoyo. Caracas: UNA

•• Los suministros.
•• El transporte y la distribución.
•• El almacenamiento.

En conjunto estas actividades lograrán la satisfacción del cliente y una reducción de


costos de la empresa, además de la alta competitividad que le dará con otras empresas.

Es necesario tener en cuenta que las fuentes de abastecimiento, las redes de trans-
porte y distribución, así como, el costo de los mismos, son tres factores realmente
fundamentales para un sistema de logística industrial en correcto funcionamiento. Si
se hace lo que se conoce en la gestión de logística industrial, como por ejemplo un estudio
de localización, se podrán reducir los costos de materias primas y productos manufactura-
dos.

Se debe analizar la ubicación de la fábrica y los depósitos, examinando las redes de


abastecimiento y distribución, ya que éstos son los nexos de la empresa con el exterior, es
decir, con proveedores o clientes. También es recomendable tener en cuenta la accesibilidad
del personal, servicios públicos, así como las condiciones fiscales.

Otro aspecto importante en el proceso de logística industrial, es garantizar la


facilidad de empaque con la que cuenta el producto, para mantenerse en óptimas condi-
ciones y a su vez tener en cuenta los aparatos de preservación, las modalidades de transpor-
te, las condiciones en las que los paquetes o bultos son trasladados de un lugar a otro. Por
otro lado el espacio físico que ocupe el producto tendrá incidencia cuando se planifique el
acondicionamiento de éste, no es lo mismo una caja de forma cúbica que un barril de forma
cilíndrica.

FUNCIONES DEL ÁREA DE LOGÍSTICA

La función logística se encarga de la gestión de los flujos físicos (materias primas, pro-
ductos acabados…) y se interesa en su entorno. El entorno corresponde en este caso a:
•• recursos (humanos, consumibles, electricidad…)
•• bienes necesarios a la realización de la prestación (almacenes propios,
herramientas, camiones propios, sistemas informáticos…)
•• servicios (transportes o almacén subcontratados, …)

La función logística gestiona directamente los flujos físicos e indirectamente los flujos fi-
nancieros y de información asociados. Los flujos físicos son generalmente divididos entre los
de compra (entre un proveedor y su cliente), de distribución (entre un proveedor y el cliente
final), de devolución (logística inversa).

304
Sánchez Marchán, Nanci. (2014).Introducción a la Ingeniería Industrial, Material Instruccional de Apoyo. Caracas: UNA

LOGÍSTICA DE COMPRA
La logística de compra incluye la gestión de los flujos físicos, de información y adminis-
trativos siguientes:
•• la planificación del aprovisionamiento desde los proveedores de acuerdo
con las previsiones de producción o venta.
•• la ejecución del aprovisionamiento y de los transportes y otras operacio-
nes de importación relacionados.
•• la gestión de la relación con los proveedores (con objetivo de la mejora
del servicio y la reducción de los costes logísticos).

LOGÍSTICA DE DISTRIBUCIÓN
La logística de distribución conocida como DFI (Distribución Física Internacional)
incluye la gestión de los flujos físicos, de información y administrativos siguientes:
•• la previsión de la actividad de los centros logísticos.
•• el almacenamiento.
•• el traslado de mercancías de un lugar a otro del almacén con los recur-
sos y equipos necesarios.
•• la preparación de los pedidos o la ejecución de cross docking (tránsito).
•• algunas veces, la realización de pequeñas actividades de transformación
del producto (kitting, etiquetado…).
•• el transporte de distribución hasta el cliente.

LOGÍSTICA INVERSA
La logística inversa incluye la gestión de los flujos físicos, de información y administra-
tivos siguientes:
•• recogida del producto en las instalaciones del cliente
•• puesta en conformidad, reparación, reintegración en stock, destrucción,
reciclaje, embalaje y almacenaje.

Actividades de logística

Se pueden mencionar las siguientes:


•• Localización de instalaciones.
•• Compras.
•• Empaque.
•• Planeación y control de producción e inventarios.
•• Manejo de materiales.
•• Almacenamiento.

305
Sánchez Marchán, Nanci. (2014).Introducción a la Ingeniería Industrial, Material Instruccional de Apoyo. Caracas: UNA

•• Tráfico y transporte.
•• Administración de pedidos/ órdenes.
•• Soporte de servicio y refacciones.
•• Retorno de productos y manejo de desperdicios.
•• Programas de distribución con proveedores y clientes.
•• Movimiento de personal.
•• Sistema de comunicación en la red de distribución.

Las actividades de logística se llevan a cabo en dos etapas básicas:


•• Optimizar un flujo de material constante a través de una red de enlaces
de transporte y de centros del almacenaje (administración de distribu-
ción).
•• Coordina una secuencia de recursos para realizar un determinado pro-
yecto (administración de materiales)

Consideradas las actividades de logística antes mencionadas, se pueden resumir con


una representación gráfica de este proceso, la cual se muestra a continuación:

Figura 66. Proceso de logística en la planta.

El proceso de logística muestra todas las actividades involucradas físicamente con la


adquisición, movimiento y almacenamiento de materia prima, inventario en proceso y de
producto terminado, desde el origen hasta el punto de consumo.

306
Sánchez Marchán, Nanci. (2014).Introducción a la Ingeniería Industrial, Material Instruccional de Apoyo. Caracas: UNA

SEGURIDAD E HIGIENE EN EL TRABAJO

Objetivo:
Indicar la trascendencia de la Seguridad e Higiene Industrial en el campo de
la Ingeniería Industrial, a los fines de prevenir los riesgos a los que están expues-
tos los trabajadores en sus diferentes ambientes de trabajo.

•• Evolución de la Seguridad e Higiene Industrial.


•• Seguridad Industrial.
•• Higiene Industrial.
•• Agentes contaminantes.
•• Medicina del trabajo.
•• Programa de Seguridad e Higiene Industrial.
•• Historia de la Ergonomía.
•• Clasificación de la Ergonomía.
•• Aplicaciones de la Ergonomía.
•• Ventajas de la Ergonomía.
•• Medidas preventivas en el área de trabajo.

Este tema se desarrolla tanto para el estudiante que inicia la carrera Ingeniería Indus-
trial, como para el estudiante de la carrera TSU en Higiene y Seguridad Industrial. El tema
tiene una gran relevancia para ambas carreras, dado que estos profesionales deben velar
por la seguridad, el ambiente, la salud y el bienestar del trabajador, en la ejecución de sus
responsabilidades laborales.

ORIGEN Y EVOLUCIÓN DE LA SEGURIDAD E HIGIENE INDUSTRIAL


Desde el inicio del origen de la especie humana surge la necesidad natural de proveerse
de alimentos y medios de sustento, aparece el trabajo y en consecuencia, la existencia de
accidentes y enfermedades generadas por la actividad laboral. En la tabla 38 se sintetizan
los estudios más importantes realizados en este campo.

307
Sánchez Marchán, Nanci. (2014).Introducción a la Ingeniería Industrial, Material Instruccional de Apoyo. Caracas: UNA

Tabla 38.
Aportes y comentarios de personajes que impulsaron la evolución de la
Seguridad e Higiene Industrial.
Personaje
o país Época o año Aporte a la Seguridad e Higiene Industrial
promotor
Promulgación de leyes por el Rey Hammurabi, referentes a pro-
Antigua Año 2000
teger la salud de los esclavos ante los accidentes que pudieran
Babilonia A.C.
sufrir.
Conocido como el padre de la medicina, realizó las primeras
Hipócrates 460 -377 A.C.
observaciones sobre enfermedades laborales.
Médico romano. Hizo referencia a los peligros inherentes en el
manejo del zinc y del azufre y propuso lo que puede haber sido el
Plinio 400 - 1472
primer equipo de protección respiratoria, fabricado con vejigas de
(El viejo) años.
animales, que se colocaban sobre la boca y nariz para impedir la
inhalación de polvos.
Escribió un libro sobre las enfermedades relacionadas con el
Ulrich Ellem-
1473 ambiente de trabajo y cómo prevenirlas, en el cual hizo renacer el
bog
interés en esta área.
Publicó un libro en el que se hacen sugerencias para mejorar la
ventilación en las minas, fabricación de máscaras protectoras, se
discuten ampliamente los accidentes en las minas y sus causas;
George Bauer 1556
describe los defectos del pie de trinchera (enfermedad por expo-
sición de los pies a la humedad de las minas). También muestra
las enfermedades que afectan a los mineros.
Dr Escribió un libro que reforzó el interés en el estudio sobre la toxi-
1567
Paracelso cidad del mercurio y otros metales.
Bernardino Luchó porque el ejercicio de la medicina del trabajo se llevara a
1700
Ramazzini cabo en los lugares de trabajo y no en el consultorio médico.
Inicio de la Promulgación de leyes referentes a las condiciones de trabajo y
Revolución jornada laboral. También para eliminar efectos contaminantes de
Inglaterra Industrial polvo y humo, así como prohibir el consumo de alimentos en las
instalaciones de trabajo.
1833
Promulgación de ley para indemnizar al trabajador que sufriere
Alemania 1869
daño en la práctica de su trabajo.
Estados ~Legislación referente a protección contra uso máquina peligro-
Unidos ~1877 sas. +Inicio Consejo Nacional de Seguridad (prever accidentes de
+1915 toda índole). *Fundación del Consejo Interamericano, para prever
(1938)*EUA y
accidentes en países de habla hispana.
Países Ameri- *1938
canos
Primera Primeros intentos científicos de protección al trabajador, anali-
Guerra 1914-1918 zando las enfermedades, condiciones ambientales, distribución y
Mundial diseño de maquinaria y equipos para evitar accidentes laborales.
Países Creación de la Oficina Internacional del Trabajo (OIT), para la
integrantes de 1919 asistencia técnica y respaldo a políticas sociales y de trabajo.
la ONU
Asociaciones Creación de la Sociedad Internacional de Seguros
1927
de Seguros
1909 Inicio seguridad industrial. *Promulgación de la ley de Sanida So-
*1936 cial.\Creación del instituto de Seguro Social de Venezuela. + Re-
glamento de condiciones de Higiene y Seguridad en el Trabajo
Venezuela \1936 ºReglamento de Prevención de Incendio. ~ COVENIN trabaja en la
+1944 estandarización de normas en materia de seguridad.

º1968 ~1974
Ley Orgánica de Prevención, Condiciones y Medio ambiente de
Venezuela,
2008 Trabajo (LOPCYMAT), actualmente utilizada en todo el territorio
época actual
nacional, promulgada en 1986 y activada en enero de 2008.

308
Sánchez Marchán, Nanci. (2014).Introducción a la Ingeniería Industrial, Material Instruccional de Apoyo. Caracas: UNA

CONCEPTOS DE SEGURIDAD E HIGIENE INDUSTRIAL


La seguridad y la higiene aplicadas a los centros de trabajo tienen como objetivo sal-
vaguardar la vida, preservar la salud y la integridad física de los trabajadores, por medio del
dictado de normas encaminadas tanto a proporcionarles las mejores condiciones en el sitio
de trabajo, como a capacitarlos y adiestrarlos para que se eviten, dentro de lo posible, las
enfermedades y los accidentes laborales.

SEGURIDAD INDUSTRIAL

La seguridad industrial se puede definir como:

Un conjuntos de principios leyes, normas y mecanismos de prevención de los riesgos


inherentes al sitio laboral, que pueden causar un accidente de trabajo, con daños severos
a la vida de los trabajadores o a las instalaciones o equipos de las empresas en todos
sus ramos.

Ejemplo Ilustrativo
Qué sustenta la siguiente afirmación: “La Seguridad Industrial tiene como
propósito la prevención de accidentes y siniestros”.
La razón fundamental que sustenta esta afirmación se deriva del hecho probado
por la experiencia, que prevenir es más económico que pagar los gastos que se
derivan de la ocurrencia de los accidentes y siniestros. Se consumen menos re-
cursos, si se toman medidas apropiadas para disminuir la cantidad de accidentes
laborales que pueden presentarse si no se toma alguna medida.

Algunos puntos a considerar dentro de la Seguridad Industrial son los siguientes:


•• Accidentes de trabajo y enfermedad laboral o profesional.
•• Causas y costos de los accidentes.

ACCIDENTE DE TRABAJO
Se define como:

La lesión orgánica o perturbación funcional inmediata o posterior, o la muerte producida


repentinamente en el ejercicio o con motivo del trabajo. Cuando no existe lesión orgánica
o perturbación funcional se considera un incidente.

Entre las causas que dan origen al accidente de trabajo existen dos:

1. Directas o próximas: dependen del ambiente de trabajo donde se generó el acci-


dente y de las condiciones biológicas intrínsecas del propio accidentado. Existen dos formas:

309
Sánchez Marchán, Nanci. (2014).Introducción a la Ingeniería Industrial, Material Instruccional de Apoyo. Caracas: UNA

•• Condiciones inseguras: son las causas que se derivan del medio donde
los trabajadores realizan sus labores, tales como materiales, maquinaria,
etc.; y que presenta un gran peligro de accidente.
•• Prácticas inseguras: son las causas que dependen de las acciones del
propio trabajador y que pueden dar como resultado un accidente.

Los factores principales que pueden dar origen a un acto inseguro son:
•• La falta de capacitación y adiestramiento para el puesto de trabajo.
•• La confianza excesiva.

2. Indirectas o remotas: el trabajador es víctima inocente del riesgo que sufre.

Clasificación de los accidentes

Accidentes de trabajo según su forma:


•• Caída de personas.
•• Caída de objetos.
•• Pisado de objetos.
•• Aprisionamiento entre objetos.
•• Esfuerzos excesivos.
•• Exposición de temperaturas extremas.
•• Exposición a la corriente eléctrica.
•• Exposición a sustancias nocivas.

Accidentes de trabajo según el agente material que lo genera:


•• Máquinas.
•• Medios de transporte y elevación.
•• Otros aparatos y equipos.
•• Materiales sustancias y radiaciones.
•• Ambiente de trabajo.

Accidentes de trabajo según la ubicación de la lesión.


•• Cabeza y cuello.
•• Tronco.
•• Miembro superior e inferior.
•• Ubicaciones múltiples.
•• Lesiones generales.

310
Sánchez Marchán, Nanci. (2014).Introducción a la Ingeniería Industrial, Material Instruccional de Apoyo. Caracas: UNA

CAUSAS Y COSTOS DE LOS ACCIDENTES


Causas

Son eventos indeseados que se producen cuando un conjunto de factores se combinan


en circunstancias propicias, que generan consecuencias graves o fatales para la integridad
de los trabajadores. Las consecuencias de los accidentes pueden ser lesiones, daños, pér-
didas, entre otras.

Actividad propuesta: Interprete la siguiente situación:


Manuel Carvajal, es el gerente de producción de la empresa constructora NAE-
LE, C.A., en donde es necesario elevar y usar a grandes alturas herramientas,
trozos de piedra, partes metálicas, entre otras, que pueden generar accidentes
¿Qué medidas mínimas deben aplicarse a los trabajadores para que mantengan
un comportamiento adecuado, frente a las normas de seguridad existentes?

Costos

Los accidentes tienen costos directos o subjetivos, como el sufrimiento de la víctima y


el dolor de su familia y costos indirectos o de recursos, como los daños a la propiedad, la
destrucción de máquinas o la pérdida de la producción, incapacidad en el trabajador, entre
otras cosas.

Clasificación de los costos de accidentes de trabajo:

1. Costos del tiempo perdido por el trabajador lesionado.

2. Costos del tiempo perdido por otros trabajadores que interrumpen sus tareas:
•• Por curiosidad.
•• Por compasión.
•• Por ayudar al trabajador lesionado.
•• Por otras razones.

3. Costo del tiempo perdido por supervisores para:


•• Presentar asistencia al trabajador.
•• Investigar las causas del accidente.
•• Disponer tiempo para que otro trabajador realice las labores del otro tra-
bajador lesionado.
•• Preparar los informes sobre el accidente.

4. Costo del tiempo de la persona que prestó los primeros auxilios.

5. Costo de los daños ocasionados por máquinas, herramientas u otros bienes.

311
Sánchez Marchán, Nanci. (2014).Introducción a la Ingeniería Industrial, Material Instruccional de Apoyo. Caracas: UNA

6. Costos por la imposibilidad de entregar los pedidos en la fecha convenida.

7. Costos de las prestaciones al personal.

8. Costos por el pago completo.

9. Costos de beneficios pendientes por sustituir máquina averiada.

10.Costos de debilitamiento que causa el personal moralmente, al ver el accidente.

INCAPACIDADES GENERADAS POR LOS ACCIDENTES


Incapacidad temporal: es la imposibilidad de trabajar durante un período limitado.

Incapacidad parcial permanente: incapacidad del cuerpo de un sujeto para efec-


tuar un trabajo y que permanece prácticamente durante el resto de su vida.

Incapacidad total permanente: es la incapacidad plena o de funciones de un


lesionado, que permanece durante toda su vida.

PREVENCIÓN DE ACCIDENTES

Existe un mejor desempeño cuando las metas están definidas específicamente y no


cuando se han quedado sin definir. Para prevenir los accidentes se debe considerar lo si-
guiente:

Capacitación

El adiestramiento de la seguridad supone dos fases:

1. El obrero debe aprender a comportarse y efectuar su trabajo de un modo más


seguro.
2. Debe ser estimulado a poner en práctica sus conocimientos.

Fomentar la seguridad a través de:

1. Realizar concursos basados en el espíritu de competencia.


2. Entregar distinciones por haber cursado alguna materia de seguridad.
3. Proponer metas de los días trabajados sin accidentes.
4. Motivar a la participación de todas las personas.
5. Informar de casos reales.
6. Realizar ceremonias y festejos de seguridad en donde se resalte la importancia de
seguridad.

312
Sánchez Marchán, Nanci. (2014).Introducción a la Ingeniería Industrial, Material Instruccional de Apoyo. Caracas: UNA

Utilizar carteles

Algunos son humorísticos, otros dan consejos de orden general, otros demuestran peli-
gros que se corresponden con una acción determinada, etc.

Emplear equipos de protección

Entre el equipo protector necesario para disminuir los riesgos se encuentran:


•• Protección de cabeza: El uso constante y estricto de los llamados som-
breros duros.
•• Protección del rostro y ojos: gafas con cubiertas laterales, gafas con es-
cudo plástico para los ojos, gafas fundidores, gafas comunes de seguri-
dad.
•• Protección del oído: los métodos que suelen emplearse comprenden re-
visiones de diseño, cubiertas para amortiguar el sonido, entre otros.
•• Equipo protector respiratorio.

Motivar a los trabajadores en el logro de su protección personal

Se debe llevar a cabo una campaña refiriendo los beneficios del uso del equipo en
casos reales donde su uso ha prevenido las lesiones. Si el equipo ha sido seleccionado
adecuadamente y su beneficio ha sido comunicado en forma clara al personal, la etapa del
rechazo será muy corta.

Investigar y analizar las causas de los accidentes

Para realizar la investigación de análisis de accidentes se pueden sugerir los pasos que
a continuación se mencionan:

1. Obtener el reporte del accidente elaborado por el patrón.


2. Obtener de ser posible y de inmediato, la declaración del trabajador accidentado,
acerca de las circunstancias de lo ocurrido.
3. Obtener la declaración de los testigos en su caso.
4. Obtener el informe médico.
5. Hacer un reconocimiento del lugar del accidente.
6. Observar y registrar los hechos captados en los puntos anteriores.
7. Estudiar los hechos en conjunto, los esenciales y los secundarios, con el objeto de
precisar los factores que provocaron el accidente.
8. Sugerir algunas acciones correctivas a seguir.

313
Sánchez Marchán, Nanci. (2014).Introducción a la Ingeniería Industrial, Material Instruccional de Apoyo. Caracas: UNA

Ejemplo Ilustrativo
Se presenta a continuación un grupo de riesgos con las medidas necesarias para
solventarlos:

Riesgo Medidas
Escalera especial.
Incendio de edificio.
Extinguidores.
Incendio de planta industrial. Equipo automático contra incendios.

Objetos que caen. Casco de seguridad.

Deslizamiento en superficies lisas. Usar botas antiresbalantes.


Exposición prolongada a productos
Uso de guantes, mascarillas, otros.
químicos.

Actividad propuesta Nº 1
Seleccione la respuesta correcta
Todos los polvos, con excepción de los provenientes de los metales pesados,
producen la enfermada profesional denominada:
a ( ) Saturnismo
b ( ) Silicosis
c ( ) Neucomiosis
d ( ) Hidrargirismo

Actividad propuesta Nº 2
Seleccione la respuesta correcta ¿Cuál de los siguientes enunciados relacionados
con la Higiene y Seguridad Industrial es falso? Justifique su respuesta.
a. Los riesgos deben disminuirse aunque los gastos implicados en tal acción no
sean económicamente aceptables para la empresa.
b.Son departamentos encargados en la formación de recursos humanos para pre-
venir riesgos de accidentes empleando técnicas de motivación, educación y pro-
tección en las organizaciones.
c.Resulta más económico prevenir con antelación los accidentes y siniestros que
puedan darse, que pagar los gastos derivados de la ocurrencia de éstos.
d.La silicosis se produce frecuentemente en trabajadores de fábricas de cemen-
to que no usan mascarillas protectoras debidamente diseñadas.

314
Sánchez Marchán, Nanci. (2014).Introducción a la Ingeniería Industrial, Material Instruccional de Apoyo. Caracas: UNA

HIGIENE INDUSTRIAL
Se puede definir como:

La disciplina encargada de la investigación, evaluación y control de los agentes conta-


minantes de origen laboral, los cuales pueden deteriorar la salud de los trabajadores y
causarles una enfermedad de trabajo,en el ejercicio debido a su actividad laboral.

Ejemplo Ilustrativo
Uno de los objetivos más importantes de la Higiene Industrial es la prevención de
los efectos de los agentes perjudiciales de la salud y bienestar de los trabajadores
¿Cuál es el procedimiento a seguir para alcanzar este objetivo?
Este objetivo se puede lograr reduciendo las tensiones ejercidas sobre el indivi-
duo a límites compatibles y permisibles, con el mantenimiento de la salud, razón
por la cual es necesario conocer las propiedades químicas y físicas de los agen-
tes, sus efectos fisiológicos, como también los principios básicos para su control.

Actividad propuesta: Interprete la siguiente situación:


En el campo de aplicación de la higiene industrial se consideran las siguientes
fases: identificar, evaluar y controlar los contaminantes.
Describa cada una de estas fases, apoyándose a través de medios impresos o
electrónicos.

AGENTES CONTAMINANTES
De acuerdo con la diversidad de propiedades, los agentes contaminantes presentan
una gran diversidad de riesgos a la salud de los trabajadores, si entran en contacto directo
con el organismo. Se considera el riesgo como:

La probabilidad de ocurrencia de un siniestro, el cual puede ser casual o inseguro, se


corresponde este efecto con una imprudencia, impericia o negligencia de quien la realiza

Las lesiones laborales se presentan por diferentes causas externas, originadas por es-
tos agentes contaminantes. El daño que pueden causar los contaminantes depende del
material del que se trate y su forma de entrada al organismo, pudiendo ser éste de efecto
inmediato o prolongado, generando daños irreversibles.

Para determinar el daño que se produce, se utilizan diferentes criterios, tales como:
•• Tipo de agente.
•• Vía de entrada.
•• Tiempo de exposición.
•• Grado de concentración de los agentes contaminantes.

315
Sánchez Marchán, Nanci. (2014).Introducción a la Ingeniería Industrial, Material Instruccional de Apoyo. Caracas: UNA

Los agentes contaminantes son un grupo de factores o sustancias que están presentes
en el medio ambiente y que causan efectos desfavorables para la salud o el mismo medio.
Se pueden clasificar de la siguiente manera:
1. Agentes contaminantes químicos.
2. Agentes contaminantes físicos.
3. Agentes contaminantes biológicos.
4. Agentes ergonómicos.

Agentes químicos

Los agentes contaminantes químicos pueden ser sustancias orgánicas e inorgánicas,


naturales o sintéticas que tienen probabilidades de lesionar la salud de las personas o causar
un efecto negativo en el medio ambiente, pueden aparecer en los siguientes estados:
1. Sólidos.
2. Polvo (sílice).
3. Fibras (asbesto).
4. Humos (combustibles).
5. Líquidos: aerosoles (plaguicidas).
6. Gaseosos: vapores (disolventes).

Agentes contaminantes físicos

Los contaminantes físicos son formas de energía (energía mecánica, energía térmica,
radiaciones) y entre ellos se pueden distinguir:
•• Ruido.
•• Radiaciones ionizantes (ultravioleta, infrarrojos, láser, radio, etc.)
•• Vibraciones.
•• Ventilación.
•• Iluminación.
•• Presión.
•• Temperatura.
•• Nieblas.
•• Humos.
•• Vapores.
•• Gases.
•• Polvos.

Agentes contaminantes biológicos

Son todos aquellos agentes representados por organismos vivos, tales como:
•• Bacterias.
•• Hongos.
•• Insectos.
•• Polen.

316
Sánchez Marchán, Nanci. (2014).Introducción a la Ingeniería Industrial, Material Instruccional de Apoyo. Caracas: UNA

Agentes ergonómicos

Calificados como tales aquellos estados, posiciones y circunstancias que se generan


por realizar un trabajo y que puedan producir lesión o deterioro a la salud del trabajador.
•• Mal diseño.
•• Operaciones inadecuadas.
•• Condiciones inadecuadas.
•• Relaciones laborales inadecuadas.

Vías de entrada de los agentes contaminantes al organismo

Algunas de las vías de penetración de los productos químicos pueden ser:


1. Por contacto directo: absorción a través de la piel, generando quemadu-
ras.
2. Mediante ingestión (vías digestivas): generando envenenamiento e intoxi-
cación.
3. Por inhalación (vías respiratorias): produce igualmente envenenamiento
e intoxicación.
4. Por absorción cutánea: A través de la piel, causando intoxicación.
5. A través de vía parenteral: es decir, inyectado ocasionando la muerte.
6. Vía auditiva: ruido alto y bajo, golpeteo, vibraciones.
7. A través de vía visual: poca iluminación, radiaciones, temperatura y lu-
ces resplandecientes.

Medidas de detección de agentes contaminantes:

En el medio ambiente laboral

Para la medición y detección de los contaminantes, algunos fabricantes han puesto a


disposición del comercio instrumentos que permiten la detección y evaluación de sus con-
centraciones. Estos instrumentos detectan una zona determinada cuando los agentes exce-
den los niveles de contaminación, activando una alarma.

En el organismo

Lo más importante para la detección son los exámenes médicos físicos periódicos. Él
médico que los realiza trata de evaluar los antecedentes familiares que tenga el trabajador.
Por otro lado, la medicina del trabajo o la higiene industrial deben investigar el medio ambien-
te para poder detectar las enfermedades de trabajo.

Pasos para evaluar y controlar los contaminantes:


1. El primer paso será identificar cuáles son los contaminantes que pueden
afectar la salud del trabajador.

317
Sánchez Marchán, Nanci. (2014).Introducción a la Ingeniería Industrial, Material Instruccional de Apoyo. Caracas: UNA

2. Una segunda fase, en todo campo de aplicación de la higiene industrial,


sería la medición, es decir, cuantificar el contaminante.
3. En función de la medición, se valorará el nivel de contaminante, para
decidir si existe una situación segura o una situación peligrosa, para ello
se utilizan criterios de valoración.
4. Si se decide que se está ante una situación segura, se llevará a cabo
un control periódico para determinar la continuidad de la línea. En caso
contrario, se tendrá que actuar sobre los contaminantes, mediante un
control ambiental.

Tiempo de exposición de un agente contaminante

El tiempo de exposición a un contaminante es el tiempo que los trabajadores están


sometidos durante su jornada laboral a la acción del mismo; habitualmente se expresa en
horas o minutos por jornada.

A la definición anterior se le debe agregar el factor de susceptibilidad individual, que


corresponde a la reacción propia que cada individuo puede desarrollar frente a los efectos
de un agente o estímulo externo.

Los contaminantes químicos pueden provocar un daño de forma rápida (intoxicación


aguda), o generar una enfermedad profesional con el transcurrir del tiempo (intoxicación
crónica). A menor tiempo de exposición menor riesgo, por eso es muy importante que sola-
mente esté en el lugar donde se manipulan productos químicos el tiempo necesario y que las
tareas de otro tipo (realizar cálculos, escribir, uso del ordenador, entre otros) las realice en un
lugar sin presencia de contaminantes químicos en el aire.

Grado de concentración del contaminante

El Grado Concentración se refiere a la proporción entre el agente contaminante y el


medio que lo contiene. El grado de concentración de un contaminante depende de factores
tales como:
•• Naturaleza física y química de los efluentes.
•• Condiciones meteorológicas del ambiente.
•• Grado de turbulencia en la atmósfera.
•• Localización de chimeneas respecto al movimiento del aire y naturaleza
del terreno que se encuentra ubicado en la dirección del viento que pro-
porciona la chimenea.

Para que un contaminante pueda tener la máxima dispersión, es necesario que los
efluentes cuenten con el movimiento y la máxima flotación posible.

318
Sánchez Marchán, Nanci. (2014).Introducción a la Ingeniería Industrial, Material Instruccional de Apoyo. Caracas: UNA

ENFERMEDADES LABORALES O PROFESIONALES


Enfermedades laborales o profesionales, es todo estado patológico que tiene su origen
en el trabajo o en el medio en que el trabajador presta sus servicios.

Las enfermedades laborales o profesionales se propagan por causa de agentes quími-


cos como gases y partículas; y biológicos, como virus, bacterias, hongos, materiales en es-
tado de descomposición. Que se encuentran en el ambiente de trabajo y que representan un
alto riesgo para la salud humana. Por esta razón se debe asegurar una atmósfera adecuada,
para la vida humana antes y durante la realización de los trabajos.

Medidas de prevención de enfermedades laborales


•• Conocer las características de cada uno de los contaminantes.
•• Vigilar el tiempo máximo.
•• Mantener ordenado y limpio el lugar de trabajo.
•• Usar adecuadamente el equipo de protección personal.
•• Someterse a exámenes médicos iníciales y periódicos.

Las enfermedades causadas por temperaturas altas se pueden prevenir así:


•• Evitando la existencia de temperaturas altas.
•• Proporcionando pastillas de sal a los trabajadores, acompañados por
grandes cantidades de líquidos.
•• Concediendo descansos periódicos.
•• Usando el equipo de protección más adecuado.

Las enfermedades causadas por la expansión de polvos, gases, humos o vapores se


pueden prevenir:
•• Identificando la sustancia contaminante.
•• Limitando la exposición.

Las enfermedades causadas por ruidos se pueden prevenir:


•• Eliminando las fuentes de ruido.
•• Aislando al personal.
•• Seleccionando las características de ruido y vibraciones bajas.
Ruido: sonido no diseñado, es una forma de vibración. Los efectos del ruido sobre los
trabajadores incluyen:
•• Efectos psicológicos como alarmar, distraer, etc.
•• Interferencias de la comunicación hablada.
•• Efectos fisiológicos como pérdida de la capacidad auditiva.

Se observa a continuación la tabla 39, que contiene algunas enfermedades laborales o


profesionales, así como los agentes que la causan.

319
Sánchez Marchán, Nanci. (2014).Introducción a la Ingeniería Industrial, Material Instruccional de Apoyo. Caracas: UNA

Tabla 39. Enfermedades laborales y los agentes que la causan


Agente que la
Enfermedad Descripción Síntoma Prevención  
causa
Silicosis Partículas de Depósito de partículas Falta de aire, tos, Uso de mascari-
Sílice(rocas de de Sílice, en el pulmón normalmente con lla o dispositivo
quarzo, oro, plo- ocasionando la destruc- flemas, bronquitis que purifique el
mo, Zin, cobre) ción de éste y la forma- crónica, enfisema, aire.
ción de fibrosis (cica- dolor en el pecho,
trización) de los tejidos labios y lóbulos de
pulmonares incluidos las orejas azula-
los vasos sanguíneos y dos.
linfáticos.
Beriliosis Berilio Infección pulmonar cau- Acceso repentino Uso de masca-
sada por la aspiración de tos, dificultad rilla, control del
de polvo o vapores que para respirar y per- polvo.
contienen berilio. dida de peso.
Bisinosis algodón sin Estrechamiento de las Sibilancias al respi- Uso de masca-
procesar, lino y vías respiratorias cau- rar y opresión en el rilla, control del
cáñamo. sado por la aspiración pecho. polvo.
de partículas de algo-
dón, lino o cáñamo.
Asma Polvo, gases, Tipo de asma causa- Tos u otros sínto-
Uso de mas-
Ocupacional humos, vapores, do por la exposición mas del asma, tal
carilla. Cambio
productos anima- a irritantes inhalados como respiración
en el proceso
les, desechos de en el lugar de trabajo, sibilante y opresión
de trabajo para
pájaros, polen de como fabricantes de en el pecho.
manejar mejor
plantas, deter- colchones, materiales la exposición
gentes, metales. para empacar, plastifi- al irritante que
cadores. permita mante-
ner los niveles
de exposición al
mínimo.
Neumonitis Sustancias Inflamación de los Ardor en el pecho, Uso de másca-
químicas como el pulmones o dificultad dificultad respira- ras protectoras,
cloro, desinfec- respiratoria causada toria, necesidad de evitar inhalar de-
tantes de ino- por inhalación de quími- aire, tos. rrames químicos.
doro, cloruro de cos nocivos.
metileno.
Bronquitis Polvos, emana- Inflamación de las vías Tos continua que Máscaras facia-
ciones, ácidos respiratorias principales produce flema, les, ropa protec-
fuertes y otros de los pulmones de los sibilancias. tora, tratamiento
químicos. trabajadores expuestos de textiles.
a polvos, emanaciones
de humo, vapores o
brumas.
Neumoconiasis polvo del carbón. También llamada como Dificultad respirato- Uso de másca-
enfermedad del pulmón ria, tos crónica. ras protectoras
negro, es una enferme- como medida
dad respiratoria causa- preventiva para
da por la inhalación del minimizar la ex-
polvo del carbón por posición al polvo
períodos prolongados. de carbón.

320
Sánchez Marchán, Nanci. (2014).Introducción a la Ingeniería Industrial, Material Instruccional de Apoyo. Caracas: UNA

Asbestosis el cloro, el fosge- Es una fibrosis pul- Produce graves Uso de máscaras
no, el dióxido de monar causada por quemaduras en de gas con su
azufre, el sulfato la inhalación de polvo los ojos, la nariz, propia provisión
de hidrogeno, que contiene fibras de la garganta, la de aire, para
el dióxido de amianto. tráquea. manipular gases
nitrógeno y el y sustancias
amoniaco. químicas
Dermatitis Sustancias quími- Es una intoxicación Produce calor, Uso de guantes
cas como ácido sistémica de absorción enrojecimiento, protectores, ropa
crómico, bicroma- cutánea. hinchazón, forma- y equipos ade-
to potásico con- ción de vesículas y cuados.
centrado, trióxido exudación.
de Arsenio, así
como plantas y
agentes fotoreac-
tivos irritantes y
sensibilizantes.

Se puede observar en la tabla que sigue, algunos trabajadores típicos que corren el
riesgo de contraer enfermedades pulmonares.

Tabla 40. Los trabajadores expuestos a enfermedades pulmonares


Enfermedad pulmonar Trabajadores expuestos
• Mineros del plomo, cobre, plata y oro
Silicosis • Ciertos mineros del carbón (por ejemplo, tamizado, techos).
• Obreros de las fundiciones.
• Alfareros.
• Cortadores de granito o piedra arenisca.
• Obreros que construyen túneles.
• Obreros que fabrican jabones abrasivos.
• Los que trabajan con inyección de chorros de arena.
Pulmón negro • Obreros del carbón.
Asbestosis • Obreros que extraen, muelen y manufacturan amianto.
• Obreros de la construcción que instalan o extraen materiales que
contienen amianto.
Beriliosis • Trabajadores aeroespaciales.
Neumoconiosis • Soldadores.
benigna • Mineros de hierro.
• Operarios del bario.
• Trabajadores del estaño.
Asma profesional • Sujetos que trabajan con granos, madera de cedro rojo, ricino,
tinturas, antibióticos, resinas, té y enzimas utilizadas en la fabri-
cación de detergentes, malta y artículos de cuero.
Bisinosis • Trabajadores del algodón cáñamo y lino.
Enfermedad de los tra- • Granjeros.
bajadores de silos

321
Sánchez Marchán, Nanci. (2014).Introducción a la Ingeniería Industrial, Material Instruccional de Apoyo. Caracas: UNA

Exámenes periódicos

Un aspecto importante en la prevención de enfermedades profesionales es la realiza-


ción de exámenes periódicos, los cuales tienen por objetivo la detección precoz de afeccio-
nes producidas por agentes de riesgo a los cuales el trabajador se encuentre expuesto con
motivo de sus tareas, con el fin de evitar el desarrollo de enfermedades profesionales. La
realización de estos exámenes es obligatoria en todos los casos donde exista exposición a
los agentes de riesgos y tienen frecuencias de realización y contenidos mínimos.

La supervisión, como una actividad planeada, sirve para conocer oportunamente


riesgos a los que están expuestos los trabajadores, antes de que ocurra un acci-
dente o una enfermedad de trabajo, que puedan provocar una lesión o la pérdida
de la salud del trabajador.

Ejemplo Ilustrativo Nº 1
Situación: Federico Casorla tiene una pequeña empresa que se ocupa de la ins-
talación de cocinas empotradas, en la cual está constantemente a la exposición
del polvo que genera el corte frecuente de mármol y el granito. Federico no utiliza
ningún tipo de protección. ¿Qué enfermedad puede contraer el señor Casorla con
el transcurrir del tiempo?

Respuesta:
Federico Casorla puede, con el tiempo, adquirir una enfermedad pulmonar deno-
minada SILICOSIS, la cual se contrae en espacios donde se genera gran canti-
dad de polvo proveniente del corte de este tipo de material (mármol, granito, entre
otros).

Ejemplo Ilustrativo Nº 2
Situación: los operadores de una planta procesadora de agua, están expuestos
durante 8 horas/día, al ruido ensordecedor que generan las bombas utilizadas en
este proceso. Con el paso de los años ¿Qué puede producir este contaminante a
los operarios, sin el uso de la protección adecuada?

Respuesta
El ruido puede ser dañino para aquellas personas que trabajan durante muchos
años en ambientes muy ruidosos, sin la debida protección. Puede producirles
desarreglos nerviosos, incapacidad auditiva, efectos psicológicos como alarmar,
distraer, interferencias de la comunicación hablada, entre otros.

322
Sánchez Marchán, Nanci. (2014).Introducción a la Ingeniería Industrial, Material Instruccional de Apoyo. Caracas: UNA

Para lograr la máxima protección a todo el personal, así como de las instala-
ciones y propiedades de cualquier empresa, es necesario que todos los traba-
jadores estén preparados para seguir un plan de control bien organizado y así
efectuar la evacuación de todo el personal.

Actividad propuesta
Investigue a través de medios electrónicos o impresos ¿cuáles son los sistemas
portátiles de extinción de incendios?

PROGRAMACIÓN DE SEGURIDAD E HIGIENE INDUSTRIAL


Según el Artículo 82 del Reglamento de la LOPCYMAT (LEY ORGÁNICA DE PREVEN-
CIÓN, CONDICIONES Y MEDIO AMBIENTE DE TRABAJO), el Programa de Seguridad,
“es el conjunto de objetivos, acciones y metodologías en materia de promoción, prevención y
vigilancia de la seguridad en el trabajo.” ¿Para qué sirve? Éste debe ser elaborado para que
sea una guía en lo que se refiere a la gestión de seguridad de la empresa, es decir, los pasos
que deben ser dados por la empresa para mantener la Seguridad y la Salud Laboral. ¿Por
qué NO debe faltar en ninguna empresa?, el programa es la documentación más importante
en la gestión de seguridad de cualquier empresa, su elaboración está contemplada en el Ar-
ticulo 61 de la LOPCYMAT y en el artículo 119 de la misma Ley está contemplado como una
Infracción Grave, que puede ser sancionada con multas que van desde 25 a 75 Unidades
Tributarias, por Trabajador afectado, por tanto, este será el primer documento que le exijan
al momento de una inspección del Inpsasel.

Puntos que debe contener el programa:

1. Descripción del proceso de trabajo (producción o servicios): Se debe realizar una


descripción del proceso que lleva a cabo la empresa, puede realizarse a través de
flujogramas, diagramas o cualquier otro y especificar en cada paso del proceso
productivo los productos o servicios que se generan a través de éste.

2. Identificación y evaluación de los riesgos y peligros existentes: A través de la


descripción del proceso productivo de la empresa se deben extraer e identificar
los peligros existentes en la empresa y evaluar los riesgos que representen a los
trabajadores de la misma.

3. Planes de trabajo para abordar los diferentes riesgos y procesos peligrosos, los
cuales deben incluir como mínimo:

323
Sánchez Marchán, Nanci. (2014).Introducción a la Ingeniería Industrial, Material Instruccional de Apoyo. Caracas: UNA

a) Información y capacitación permanente a los trabajadores y asociados: Deben


especificarse los planes de capacitación que la empresa lleva a cabo, mostrando
por ejemplo si se realiza la capacitación a través de una charla mensual o reparte
volantes con información de seguridad.

b) Procesos de inspección y evaluación en materia de seguridad y salud en el trabajo:


Los procesos de inspección deben comprender los formatos de inspecciones, la
periodicidad, quién las realizará y cómo se manejarán los resultados, entre otros
detalles.

c) Monitoreo y vigilancia epidemiológica de los riesgos y procesos peligrosos: El


programa debe especificar quién realizará la vigilancia y monitoreo epidemiológico
de los riesgos y los procesos peligrosos existentes en la empresa, anteriormente
descritos. En este caso será el servicio de seguridad y salud laboral quien se
encargue de este aspecto.

d) Monitoreo y vigilancia epidemiológica de la salud de los trabajadores: Este punto


comprende la descripción del proceso referente a las historias médicas de los
trabajadores, desde su ingreso a la empresa.

e) Reglas, normas y procedimientos de trabajo seguro y saludable: Aquí debe estar


descrita toda la normativa de seguridad establecida en la empresa, así como
también la normativa de seguridad establecida para la realización de tareas
específicas dentro de la empresa.

f) Dotación de equipos de protección personal y colectiva: Deben describirse los


procedimientos que se llevan a cabo para la dotación de equipos de protección
personal de acuerdo con la tarea que realice el trabajador, así como los formatos
y registros de estas dotaciones.

g) Atención preventiva en salud ocupacional: En este punto se debe describir


el procedimiento que se lleva a cabo en la empresa para la prevención de
enfermedades ocupacionales en los trabajadores de la Empresa.

h) Planes de contingencia y atención de emergencias: Se deben describir los


procedimientos llevados a cabo en caso de una emergencia, a quien llamar, las
rutas de escape de la empresa y todo lo inherente en planes de emergencia.

i) Personal y recursos necesarios para ejecutar el plan: Aquí se debe detallar el


personal de la empresa que estará a cargo de la seguridad en la empresa y
las responsabilidades que tienen cada uno de los trabajadores, en materia de
seguridad.

324
Sánchez Marchán, Nanci. (2014).Introducción a la Ingeniería Industrial, Material Instruccional de Apoyo. Caracas: UNA

j) Recursos económicos precisos para la consecución de los objetivos propuestos:


En este punto la empresa debe hacer una relación de los gastos que realiza en
materia de seguridad contra la facturación o cualquier otro indicador de entrada
de la empresa, por ejemplo, cuánto invertirá en el plan de capacitación anual,
cuánto invertirá en el mantenimiento de los sistemas de emergencia, cuánto
invertirá en la dotación de equipos de protección personal, en el servicio de
seguridad y salud laboral, entre otros.

k) Las demás que establezcan las normas técnicas.

4. Identificación del patrono y compromiso de hacer cumplir los planes establecidos: Al


final debe haber un compromiso firmado, por parte del patrono para hacer cumplir
todo lo indicado en el programa. También recuerde que todos los formatos que tengan
que ver con la seguridad, deben ser presentados en el programa. Este programa
debe ser revisado y aprobado tanto por el Comité de Seguridad y Salud Laboral, los
empleados de la empresa, así como INPSASEL.

Actividad propuesta: Interprete la siguiente situación:


Investigue por qué el Programa de Seguridad e Higiene es considerado como
todo la compilación de la Gestión de Seguridad que una empresa lleva a cabo.

Recuerde que el programa de seguridad debe ser un reflejo de lo que debe cum-
plirse en toda empresa en materia de seguridad.

La seguridad y la higiene son instrumentos de prevención de los riesgos y deben consi-


derarse sinónimos por poseer la misma naturaleza y finalidad. En esencia, el aspecto central
de la seguridad e higiene del trabajo reside en la protección de la vida y la salud del traba-
jador, el ambiente de la familia y el desarrollo de la comunidad. Sólo en segundo término, si
bien muy importantes por sus repercusiones económicas y sociales, se deben colocar las
consideraciones sobre pérdidas materiales y quebrantos en la producción, inevitablemente
que acarrean también los accidentes y la insalubridad en el trabajo.

Abordados los conceptos de higiene y seguridad industrial, se puede decir que el traba-
jador necesita un ambiente de trabajo confortable y un puesto de trabajo donde se contem-
plen los principios ergonómicos, que permitan mejorar los espacios de trabajo para asegurar
la salud y bienestar de los trabajadores. En este orden de ideas, se presentan a continuación
los fundamentos que dan origen a la ergonomía.

325
Sánchez Marchán, Nanci. (2014).Introducción a la Ingeniería Industrial, Material Instruccional de Apoyo. Caracas: UNA

ERGONOMÍA
La Ergonomía se considera como el estudio científico de las relaciones entre el hombre
y su ambiente de trabajo, donde el término ambiente, es utilizado en un sentido más global,
incluyendo también equipos, aparatos, herramientas, materiales, métodos y la propia orga-
nización del trabajo; para hacer que el trabajo se adapte al trabajador en lugar de obligar al
trabajador a adaptarse a él.

Por ello su alcance abarca las distintas condiciones laborales que pueden influir en la
comodidad y la salud del trabajador, abarca aspectos como la iluminación, el ruido, la tem-
peratura, las vibraciones, el diseño del lugar en el que se trabaja, el de las herramientas, el
de las máquinas, el de los asientos y el calzado, así como el puesto de trabajo, que involucra
elementos como el trabajo en turnos, las pausas y los horarios de comida.

Ergonomía es una palabra compuesta por dos partículas griegas: ergo y nomos, que
significan respectivamente, actividad y normas o leyes naturales. Una traducción literal sería
las normas que regulan la actividad humana. Es un campo de conocimiento que integra la
información de ciencias como la psicología, la fisiología la antropometría, la biomecánica,
entre otras.

La lógica que utiliza la ergonomía se basa en el principio de que las personas son más
importantes que los objetos o que los procesos productivos; por tanto, en aquellos casos en
los que se plantee cualquier tipo de conflicto de intereses entre personas y cosas, deben
prevalecer los de las personas. Se trata de adaptar los productos, las tareas, las herramien-
tas, los espacios y el entorno en general a la capacidad y necesidades de las personas; de
manera que mejore la eficiencia, seguridad y bienestar de los consumidores, usuarios o
trabajadores.

Por ejemplo, una de las consecuencias del trabajo manual, además del aumento de la
mecanización, es que cada vez hay más trabajadores que padecen dolores de la espalda,
dolores de cuello, inflamación de muñecas, brazos, piernas y tensión ocular. La aplicación
de la ergonomía al lugar de trabajo reporta muchos beneficios evidentes. Para el trabajador,
unas condiciones laborales más sanas y seguras; para el empleador, el beneficio más inne-
gable es el aumento de la productividad.

Para lograr sus objetivos, la ergonomía utiliza diferentes técnicas en las fases de plani-
ficación, diseño y evaluación. Algunas de esas técnicas son los estudio de mercado, análisis
funcionales, biomecánicos, antropométricos, psicológicos y fisiológicos.

“La ergonomía es una ciencia aplicada de carácter multidisciplinario que tiene como
finalidad la adecuación de los productos, sistemas y entornos artificiales a las caracte-
rísticas, limitaciones y necesidades de sus usuarios, para optimizar su eficiencia, segu-
ridad y confort” La Asociación Internacional de Ergonomía (1999).

326
Sánchez Marchán, Nanci. (2014).Introducción a la Ingeniería Industrial, Material Instruccional de Apoyo. Caracas: UNA

CLASIFICACIÓN DE LA ERGONOMÍA
La ergonomía se centra en dos ámbitos: el diseño de productos y el puesto de trabajo.
Su aplicación al ámbito laboral ha sido tradicionalmente la más frecuente; aunque también
está muy presente en el diseño de productos y en ambientes relacionados como la actividad
del hogar, el deporte, entre otros. Asimismo está presente en el diseño de entornos para
personas con limitaciones funcionales.

•• Ergonomía del producto


El objetivo de este ámbito son los consumidores, usuarios y las características del con-
texto en el cual el producto es usado. El estudio de los factores ergonómicos en los produc-
tos, busca crear o adaptar productos y elementos de uso cotidiano o específico de manera
que se adapten a las características de las personas que los van a usar. Es decir, la ergo-
nomía es transversal, pero no a todos los productos, sino a los usuarios de dicho producto.

El diseño ergonómico de productos trata de buscar que éstos sean eficientes en su uso,
seguros, que contribuyan a mejorar la productividad sin generar patologías en el humano,
que en la configuración de su forma indiquen su modo de uso, etc.

Para lograr estos objetivos, la ergonomía utiliza diferentes técnicas en las fases de pla-
nificación, diseño y evaluación. Algunas de esas técnicas son: análisis funcionales, biome-
cánicas, datos antropométricos del segmento de usuarios objetivo del diseño, ergonomía
cognitiva y análisis de los comportamientos fisiológicos de los segmentos de los cuerpos
comprometidos en el uso del producto.

•• Ergonomía en el trabajo
El diseño ergonómico del puesto de trabajo intenta obtener un ajuste adecuado entre las
aptitudes o habilidades del trabajador y los requerimientos o demandas del trabajo. El obje-
tivo final, es optimizar la productividad del trabajador y del sistema de producción, al mismo
tiempo que garantizar la satisfacción, la seguridad y salud de los trabajadores.

El diseño ergonómico del puesto de trabajo debe tener en cuenta las características an-
tropométricas de la población, la adaptación del espacio, las posturas de trabajo, el espacio
libre, la interferencia de las partes del cuerpo, el campo visual, la fuerza del trabajador y el
estrés biomecánico, entre otros aspectos. Los aspectos organizativos de la tarea también
son tenidos en cuenta.

Para diseñar correctamente las condiciones que debe reunir un puesto de trabajo se
debe tener en cuenta, entre otros, los siguientes factores:
•• Los riesgos de carácter mecánico que puedan existir.
•• Los riesgos causados por una postura de trabajo incorrecta fruto de un
diseño incorrecto de asientos, taburetes, etc.

327
Sánchez Marchán, Nanci. (2014).Introducción a la Ingeniería Industrial, Material Instruccional de Apoyo. Caracas: UNA

•• Riesgos relacionados con la actividad del trabajador (por ejemplo, por las
posturas de trabajo mantenidas, sobreesfuerzos o movimientos efectua-
dos durante el trabajo de forma incorrecta o la sobrecarga sufrida de las
capacidades de percepción y atención del trabajador).
•• Riesgos relativos a la energía (la electricidad, el aire comprimido, los ga-
ses, la temperatura, los agentes químicos, etc.)

El diseño adecuado del puesto de trabajo debe servir para:


•• Garantizar una correcta disposición del espacio de trabajo.
•• Evitar los esfuerzos innecesarios. Los esfuerzos nunca deben sobrepa-
sar la capacidad física del trabajador.
•• Evitar movimientos que fuercen los sistemas articulares.
•• Evitar los trabajos excesivamente repetitivos.

Ejemplo Ilustrativo de diseños de puestos de trabajo


Al determinar la altura adecuada de la superficie de trabajo, como la mostrada en
la figura, es importante tener en cuenta los factores siguientes:
•• la altura de los codos del trabajador.
•• el tipo de trabajo que habrá de desarrollar.
•• el tipo de producto con el que se trabaja.
•• las herramientas y el equipo que se habrán de usar.

Cuando se diseña un puesto de trabajo surge la inquietud del tipo de postura que será
más conveniente para el trabajador: trabajar de pie o trabajar sentado. Por ello surge la si-
guiente interrogante ¿Cuál de estas dos posturas es la menos dañina para el operario?

Se puede responder que ninguna de las dos posturas de trabajo es totalmente segura.
Cada una tiene sus ventajas y desventajas. Generalmente se considera que el diseño del
puesto de trabajo le puede permitir a la persona cambios frecuentes de posición, de pie,
luego sentado, luego caminar. Sin embargo, dada las características de los ambientes de
manufactura, esto es difícil de lograr.

328
Sánchez Marchán, Nanci. (2014).Introducción a la Ingeniería Industrial, Material Instruccional de Apoyo. Caracas: UNA

RECOMENDACIONES PARA TRABAJAR SENTADO


•• Utilizar sillas y mesas ergonómicas. Las sillas con un espaldar acolchado
y firme, que se ajuste vertical y horizontalmente y la mesa que también
sea regulable.
•• Colocar la silla a una altura, de tal forma que los pies estén planos sobre
el piso al estar sentado, con las rodillas a la altura de la cadera y la su-
perficie de trabajo levemente por encima de la cadera.
•• Asegúrese de estar sentado cerca del escritorio, para evitar encorvarse
al trabajar.
•• Los reposabrazos también son necesarios para dar descanso a éstos.
•• El ángulo entre tronco y muslos ha de rondar entre 90 y 120º.
•• Si la mesa o la silla son regulables conviene adaptar la altura de uno u
otra a la posición correcta.

Ejemplo Ilustrativo:
En los diseños de puestos de trabajo presentados seleccione (el) o (los) que son
correctos.

A B C

Existen en este ejemplo dos diseños correctos, es decir, los correspondientes


a la letra B y C. En ellos se puede observar que la silla y mesa tienen diseños
ergonómicos, la operaria utiliza reposabrazos y está sentada cerca del escritorio,
para evitar el encorvamiento del cuerpo. Todas estas características correspon-
den a un buen diseño para la realización de trabajos en posición sentado.

Actividad propuesta
Indage cuáles son las recomendaciones pertinentes para leer o escribir sobre la
mesa durante tiempos prolongados.

329
Sánchez Marchán, Nanci. (2014).Introducción a la Ingeniería Industrial, Material Instruccional de Apoyo. Caracas: UNA

RECOMENDACIONES PARA TRABAJAR DE PIE


•• Debe disponer de un asiento o taburete para poder sentarte cada cierto
tiempo.
•• Trabajar con los brazos a lo largo del cuerpo y sin tener que encorvarse
ni girar la espalda excesivamente.
•• Si se trabaja tras una mesa, mostrador o banco lo mejor es que sean
ajustables a distintas alturas y a distintas tareas. Se tomará la altura de
los codos como referencia. Algunas medidas de referencia son las si-
guientes:
- La altura del mostrador debe quedar unos 5-10 cm por encima de la
altura de sus codos para tareas de precisión.
- Entre 10 y 30 cm por debajo de los codos para tareas muy pesadas
- Para tareas normales con un nivel medio de fuerza y precisión, don-
de cargará con objetos no muy pesados, se toma la altura de codos,
menos 5-10 cm.
•• Una tarima para los pies, o escabel, ayudará a reducir la presión sobre la
espalda y cambiar de postura.
•• Si el suelo es de cemento o metal, se puede tapar para que no se ab-
sorban los choques. El suelo debe estar limpio, liso y no ser resbaladizo.
•• Es recomendable llevar zapatos con empeine reforzado y tacos bajos
cuando trabaja de pie.
•• El trabajo que se realiza debe estar a unos 20 ó 30 centímetros frente al
cuerpo.

Ejemplo Ilustrativo

Mondelo 2001; Wilson 2001; Chaffin 1999, recomienda que un


trabajo sea realizado de pie cuando existen las siguientes situa-
ciones laborales:
1. Puestos móviles
2. Tareas que impliquen el uso de más de 3 kg de fuerza
3. Tareas que impliquen cargar objetos de más de 4 kg de peso
4. Falta de espacio para pies y rodillas
5. Tareas que requieran alcances máximos mayores a 50 cms

RECOMENDACIONES PARA EL LEVANTAMIENTO DE CARGA


1. Si la carga está a nivel del piso, sujétela firmemente, acérquela a su cuerpo y luego
levántela utilizando la fuerza de sus piernas.

330
Sánchez Marchán, Nanci. (2014).Introducción a la Ingeniería Industrial, Material Instruccional de Apoyo. Caracas: UNA

2. Al desplazarse y bajar la carga, mantenga la espalda recta. En este último (bajar)


aplique la fuerza de sus piernas.

3. Al levantar una carga colocada en una estantería (nivel superior), colocar una esca-
lera firmemente en el piso. Subir y ubicarse en la misma de forma segura, sujete la
carga, acérquela a su cuerpo y luego levántela e inicie el descenso, con la espalda
recta.

Ejemplo Ilustrativo
Para levantar pesos sin lesionar la columna, es necesario separar los pies,
flexionar las rodillas, mantener la espalda recta y colocar la carga cerca del
cuerpo, con sujeción firme. A continuación se muestra en la figura siguiente, la
forma correcta de cómo realizar esta actividad:

ERGONOMÍA VISUAL
Permite esta parte de la ergonomía, prevenir problemas visuales mediante consejos
referentes a las posturas y ambientes de trabajo. Se dirige a todas aquellas personas que
realizan continuamente trabajos en visión cercana como por ejemplo: estudiantes, informáti-
cos, diseñadores, administradores, etc.

A continuación se presentan algunas recomendaciones para la realización de tareas en


el trabajo con una iluminación apropiada, sin perjudicar la vista:

331
Sánchez Marchán, Nanci. (2014).Introducción a la Ingeniería Industrial, Material Instruccional de Apoyo. Caracas: UNA

•• La iluminación debe estar equilibrada, para que el operador pueda ver


los objetos o leer documentos.
•• Determinar fuentes de luz intensa en los puestos de trabajo (reflejos
luminosos brillantes) con la utilización de un espejo
•• Las lámparas no deben producir brillos intensos o sombras en el pues-
to de trabajo. El brillo y las sombras causan vista cansada y fatiga.
•• Ubicar el puesto de trabajo para que la ventana quede al lado del tra-
bajador.
•• No se debe colocar la mesa de trabajo frente a lámparas.
•• Ubicar las cortinas o persianas de ventanas para controlar niveles de
luz y brillo.
•• Las paredes y mobiliario se deben pintar con colores mate y neutros.
El color y acabado de una superficie determina la cantidad de reflejo
de luz.
•• Es preciso controlar el parpadeo de lámparas fluorescentes y revisar
que se les dé mantenimiento apropiado.
•• El operario debe levantar la vista frecuentemente fuera del trabajo para
descansar los ojos.
•• Los almacenes, corredores y escaleras necesitan estar bien ilumina-
dos, para evitar accidentes.

Niveles de luz recomendados (lux):


Corredores 50 - 10
Escaleras 100- 200
Cuartos 100 - 200
Almacenes 200 -400
Oficinas de trabajo 500 - 750
Salas de Conferencia 300 - 750
Oficinas de dibujo 500 - 1000

Tiempo de exposición a la luz

Procurar descansar la visión durante unos dos o más minutos, al haber estado durante
30 minutos utilizando la visión cercana. Es muy aconsejable mirar por una ventana y variar
el enfoque.

Distancia de trabajo:

Entre los ojos y la mesa tiene que haber como mínimo la distancia que hay entre el
codo apoyado a la mesa hasta el puño de la mano bajo la barbilla (unos 30 - 40 cm)

La evaluación periódica de la capacidad visual tiene un importante papel dentro de las


estrategias para mejorar el rendimiento en el trabajo.

332
Sánchez Marchán, Nanci. (2014).Introducción a la Ingeniería Industrial, Material Instruccional de Apoyo. Caracas: UNA

Ejemplo Ilustrativo
¿Cuáles son las consideraciones necesarias para trabajar con el monitor?
El monitor ha de estar justamente detrás del teclado, es decir, en la zona de
visión recomendada. Para calcular la posición del monitor, se aconseja realizar
una prueba: en primer lugar ponerse frente al monitor (que ha de estar apagado);
cerrar los ojos y situar la cabeza en posición natural con los músculos relajados.
Después se deben abrir los ojos y localizar el punto del monitor que se visualiza
primero. Este punto tiene que estar unos 5 centímetros debajo del extremo supe-
rior de la pantalla (no del monitor)
Practique en casa este consejo.

ALGUNAS CONSIDERACIONES PARA LOGRAR UN PUESTO DE TRABAJO


ERGONÓMICO
En ocasiones los puestos de trabajo pueden estar bien diseñados, pero la salud y los
problemas de seguridad surgen al dar poca atención a la manera cómo se realiza el trabajo.
A continuación se presentan algunas recomendaciones para lograr un diseño de un puesto
de trabajo ergonómico:
•• Realizar los ajustes necesarios para que la altura de la superficie de
trabajo sea correcta y lo suficientemente amplia para contener los mate-
riales.
•• Hay que tener en cuenta qué trabajadores son zurdos y cuáles no y fa-
cilitarles una superficie de trabajo y unas herramientas que se ajusten a
sus necesidades.
•• Es conveniente facilitar a cada puesto de trabajo un asiento cuando el
trabajo se efectué de pie. Las pausas periódicas y los cambios de pos-
tura del cuerpo disminuyen los problemas que causa el permanecer de-
masiado tiempo de pie. Es importante usar un descanso para los pies, el
descanso debe ser ajustable y apoyar el pie entero.
•• Si se trabaja sentado la silla debe tener un respaldo en el que se apoye
la parte más baja de la espalda.
•• Se requiere eliminar los reflejos y las sombras. Una buena iluminación
es esencial. Por ello es necesario amoldar las cortinas o persianas de
ventanas para controlar niveles de luz y brillo.

CLASIFICACIÓN AMPLIA DE LA ERGONOMÍA

Existe una clasificación más amplia de la Ergonomía y es la siguiente:


•• Ergonomía de sistemas y/o ergonomía de puestos.
•• Ergonomía preventiva y de corrección.
•• Ergonomía geométrica, ambiental y temporal.

333
Sánchez Marchán, Nanci. (2014).Introducción a la Ingeniería Industrial, Material Instruccional de Apoyo. Caracas: UNA

Ergonomía de sistemas y ergonomía del puesto de trabajo

La Ergonomía de sistemas estudia conjuntos de elementos, humanos y no humanos,


sometidos a interacciones, lo que implica una gran cantidad de variables; mientras que la del
puesto de trabajo se refiere al estudio concreto y exhaustivo de las relaciones entre un solo
hombre y una máquina, medios o instrumentos que utiliza para trabajar.

Ergonomía preventiva y ergonomía correctiva

La preventiva se aplica cuando el sistema estudiado todavía no existe. Se trata de la er-


gonomía en la etapa de proyecto que busca conseguir el diseño óptimo de sistemas antes de
su puesta en funcionamiento, dada la dificultad que representa modificar los ya existentes.

La correctiva es menos eficaz que la anterior aunque más fácil, puesto que se puede
apoyar en la observación de errores de un sistema ya realizado en lugar de analizar las ta-
reas de una forma vaga.

Ergonomía geométrica, ambiental y temporal


•• Ergonomía geométrica
Puede definirse como el estudio de las relaciones entre hombre y condiciones métri-
cas y posiciónales de su puesto, con una tendencia a conseguir el máximo confort. Al ser
el hombre una estructura móvil, sus necesidades serán satisfechas al alcanzar un confort
geométrico definido por:

Confort posicional: resultado de la correcta interacción entre el puesto de trabajo y el


cuerpo, por lo que hay que considerar los datos antropométricos relevantes. Los estudios en
este terreno se dirigen fundamentalmente al diseño de puestos de trabajo y elementos que lo
constituyen (asientos, herramientas, entre otros), así como a las posturas adecuadas.

Confort cinético-operacional: que estudia el movimiento muscular en relación con su


acoplamiento a la tarea y analiza y diseña los mandos y mecanismos de operación en fun-
ción del rendimiento, del consumo energético, el esfuerzo y la fatiga, condicionados por la
flexibilidad, precisión, esfuerzo, rapidez y fatiga muscular.

•• Ergonomía ambiental
Es la parte de la Ergonomía que estudia y desarrolla las relaciones entre el hombre y los
factores ambientales que condicionan su estado de salud y de confort.

En este ámbito se estudian dos grandes grupos de factores aparte de los de tipo psico-
social que son:

1. Factores físicos: térmicos, luminoso-visuales, auditivos y dinámicos (vibraciones)


2. Factores físicos y biológicos.

334
Sánchez Marchán, Nanci. (2014).Introducción a la Ingeniería Industrial, Material Instruccional de Apoyo. Caracas: UNA

•• Ergonomía temporal
Busca el bienestar del trabajador en relación con los tiempos de trabajo, teniendo en
cuenta el tipo de organización, las cargas y los contenidos del mismo. Estudia los horarios de
trabajo, la duración de las jornadas, optimización de pausas y descansos, ritmos de trabajo,
evaluando la relación fatiga-descanso en sus aspectos físicos y psicológicos.

APLICACIONES DE LA ERGONOMÍA
La Ergonomía se puede aplicar en:
•• Diseño de máquinas.
•• Diseño de herramientas.
•• Diseño del puesto de trabajo.
•• Posición de trabajo.
•• Organización de la empresa.

Diseño de máquinas

Aunque los principios ergonómicos deben aplicarse en el proceso de diseño de las má-
quinas, dado que ese es el momento en el que se pueden eliminar y/o corregir los riesgos
que dan lugar a enfermedades, se deben ampliar en el proyecto de implantación de éstas, lo
cual contribuye a lograr su correcta localización e instalación.

En las condiciones de utilización previstas deben reducirse al mínimo las molestias,


fatiga y tensión psíquica del trabajador además de eliminar los posibles riesgos de lesiones,
cumpliendo con los siguientes resultados:
•• Mantener la muñeca rígida.
•• Mantener la espalda relajada.
•• Mantener el codo pegado al cuerpo.
•• Mantener aproximadamente 90° entre brazo y antebrazo.
•• Evitar actividades por detrás de la línea media del torso.

En ocasiones, el diseño de las protecciones colectivas de las máquinas, por estar mal
hecho, puede inducir riesgos de lesiones musculoesqueléticas por la necesidad del trabaja-
dor de adoptar posturas incómodas y/o forzadas para realizar su trabajo. En este sentido se
debe tener en cuenta la amplitud del gesto que delimita los siguientes movimientos a realizar
respecto a un obstáculo:
•• Hacia arriba.
•• Por encima del obstáculo.
•• Alrededor de un obstáculo.
•• Hacia el interior de un recipiente.
•• A través del obstáculo.

335
Sánchez Marchán, Nanci. (2014).Introducción a la Ingeniería Industrial, Material Instruccional de Apoyo. Caracas: UNA

Diseño de herramientas

Las herramientas fueron los primeros objetos creados para conseguir un ahorro de
energía y son nuestras necesidades las que determinarán su forma y su función. Por ello
habrá que basar su diseño en la función para la que estarán destinadas, así como en la an-
tropometría (estudio de las dimensiones humanas) y en la biomecánica (estudio de los mo-
vimientos), contemplando el cuerpo humano como un conjunto integrado y no como varios
segmentos directamente relacionados con la utilización de las herramientas.

Desde el punto de vista ergonómico, una herramienta debe cumplir básicamente los
siguientes requisitos:
•• Desempeñar con efectividad la función para la que está hecha.
•• La operación de la herramienta debe ser tal que reduzca al mínimo la
fatiga.
•• Los movimientos que debe realizar tienen que ser compatibles con los
del brazo y la mano sin provocar sobrecargas excesivas.
•• La muñeca manual debe permitir a la muñeca permanecer recta durante
la realización del trabajo.

Estar proporcionada a las dimensiones del trabajador:


•• Tener en cuenta el tipo de ropas utilizadas en el momento de la tarea, así
como el material y su grosor.
•• Satisfacer las necesidades de presión de fuerza y de precisión.
•• Si es posible, adaptarse a diestros y zurdos.
•• Proporcionar efecto de retroalimentación (feed-back) a su usuario (textu-
ra, presión, temperatura,...)

Diseño del ambiente laboral

Trata del diseño de las condiciones de trabajo que rodean a la actividad que realiza el
trabajador. Puede referirse a aspectos como:
•• Condiciones ambientales: temperatura, iluminación, ruido, vibraciones,
entre otros.
•• Distribución del espacio y de los elementos dentro del espacio.
•• Factores organizativos: turnos, salario, relaciones jerárquicas, etc.

Todo diseño ergonómico ha de considerar los objetivos de la organización,


teniendo en cuenta aspectos como la producción, rentabilidad, innovación
y calidad en el servicio.

336
Sánchez Marchán, Nanci. (2014).Introducción a la Ingeniería Industrial, Material Instruccional de Apoyo. Caracas: UNA

VENTAJAS DE LA ERGONOMÍA
Algunas de las ventajas de la ergonomía son:
•• Mayor efectividad de los sistemas.
•• Menos índice de error en el trabajo.
•• Menor número de accidentes que ocasionen lesiones o daños a la pro-
piedad.
•• Reducción de las necesidades de nuevo diseño de los sistemas una vez
que éstos son ya operativos, siempre y cuando las medidas adecuadas
se tomen en la fase del diseño.
•• Reducción del costo y del tiempo necesario para la formación de los
operadores.
•• Mayor eficiencia en el empleo del personal y menor necesidad de efec-
tuar una selección rigurosa.

FACTORES DE RIESGO Y MEDIDAS ERGONÓMICAS PREVENTIVAS EN EL ÁREA


DE TRABAJO

Factores de riesgo en la adopción de posiciones incorrectas en el trabajo.

Todas las posiciones que se mencionan a continuación son arriesgadas y pueden pro-
vocar lesiones por esfuerzos repetidos en actividades laborales:
•• Curvar y girar repetidamente las muñecas.
•• Girar repetidamente los brazos.
•• Mantener repetidamente los codos alejados del cuerpo.
•• Utilizar repetidamente pinzas.
•• Alcanzar o levantar repetidamente objetos por encima de los hombros.
•• Utilizar repetidamente una herramienta que vibra.
•• Utilizar repetidamente la mano para hacer fuerza.
•• Girar o presionar repetidamente la espalda.
•• Levantar repetidamente objetos situados más abajo de las rodillas.
•• Trabajar repetidamente con la cabeza agachada.

De acuerdo a los riesgos existentes en el área de trabajo se pueden mencionar las si-
guientes medidas preventivas:
•• Analizar cuidadosamente cuáles son los riesgos potenciales a los que
se pueda estar expuesto al efectuar un trabajo en particular.
•• Minimizar los riesgos identificados, tomando medidas para reducir los
movimientos repetitivos y el esfuerzo excesivo.
•• Neutralizar la posición incómoda que pueda causar tensión, colocando
el cuerpo en una posición relajada y neutral.

337
Sánchez Marchán, Nanci. (2014).Introducción a la Ingeniería Industrial, Material Instruccional de Apoyo. Caracas: UNA

Actividad propuesta
Responda las preguntas planteadas en la tabla que sigue, referentes a los riesgos
que generan algunas posiciones al realizar un trabajo:

Problema ¿Cuáles son las caracte- ¿Qué problemas


rísticas ergonómicas de de salud provoca?
este problema?
Trabajar repetida-
mente con la cabeza
agachada
Curvar y girar repeti-
damente las muñecas
Girar repetidamente
los brazos.

La ergonomía surge del concepto en el cual la actividad laboral no es sólo la máquina,


sólo el individuo, o el individuo manipulando la máquina; sino más bien de la concordancia
entre las posibilidades físicas de la máquina y las propiedades psicológicas del individuo. Es
una filosofía que, si se aplica con eficacia, puede mejorar considerablemente las condiciones
de trabajo. Se pueden hacer mejoras diseñando o rediseñando correctamente la manera en
la cual se afectan las tareas, el contenido de éstas, los métodos con los que se manipula o
instala el equipo, la manera en como se fijan los horarios laborales, el equipo para efectuar
un trabajo, entre otros.

338
Sánchez Marchán, Nanci. (2014).Introducción a la Ingeniería Industrial, Material Instruccional de Apoyo. Caracas: UNA

MANTENIMIENTO INDUSTRIAL DE MAQUINARIA Y EQUIPO

Objetivo:
Reconocer la relevancia del Mantenimiento en una empresa, en función del ahorro
en tiempo y dinero cuando se previene la paralización por fallas de maquinarias y
equipos en el proceso productivo.

•• Evolución del Mantenimiento.


•• Objetivos del mantenimiento.
•• Tipos de mantenimiento:
-Correctivo.
-Preventivo.
-Predictivo.
-Proactivo.
-PM y TPM.
•• Plan general de mantenimiento.

EVOLUCIÓN DEL MANTENIMIENTO

Al principio sólo se hacía mantenimiento cuando ya era imposible seguir usando el equi-
po, lo que se denominó mantenimiento reactivo o correctivo.

Fue en 1950, cuando un grupo de ingenieros japoneses iniciaron un nuevo concepto en


mantenimiento que simplemente seguía las recomendaciones de los fabricantes de equipos
acerca de los cuidados que se debían tener en la operación, mantenimiento de máquinas y
sus dispositivos. Esta nueva tendencia se llamó mantenimiento preventivo. Como resultado,
los gerentes de planta se interesaron en hacer que sus supervisores, mecánicos, electricis-
tas y otros técnicos; desarrollaran programas para lubricar y hacer observaciones clave para
prevenir daños al equipo. Aún cuando ayudó a reducir pérdidas de tiempo, era una alter-
nativa costosa, ya que se reemplazaban partes componentes de la maquinaria que podían
haber durado más tiempo y se aplicaban horas innecesarias de trabajo.

Por la creciente automatización de los procesos productivos y la complejidad de su


mantenimiento, en la década de los 60 surge en Estados Unidos el concepto de manteni-
miento productivo (PM) en el seno de General Electric. Este concepto hacía referencia a
que el objetivo del mantenimiento no es sólo mantener los equipos, sino mejorar la calidad
mediante modificaciones de diseño que mejoren la fiabilidad y la mantenibilidad de éstos. De
esta manera el mantenimiento productivo (PM) engloba el mantenimiento correctivo, mante-
nimiento preventivo y la gestión de la calidad. A partir de 1964 se introduce el PM en Japón,

339
Sánchez Marchán, Nanci. (2014).Introducción a la Ingeniería Industrial, Material Instruccional de Apoyo. Caracas: UNA

no sin antes haberle dotado del toque característico japonés, mientras en la mayoría de las
empresas americanas el mantenimiento y la producción se mantenían separados, los japo-
neses consiguen que todos los operadores participen en el mantenimiento de los equipos de
producción.

Se asignaron más altas responsabilidades a la gente relacionada con el mantenimiento


y se hacían consideraciones acerca de la confiabilidad y el diseño del equipo.

Diez años después, tuvo lugar la globalización del mercado creando nuevas y más fuer-
tes necesidades de excelencia en todas las actividades. Los estándares mundiales en térmi-
nos de mantenimiento del equipo se comprendieron y dan lugar a un sistema más dinámico:
el Mantenimiento Productivo Total (TPM).

El TPM es un concepto de mejoramiento continuo que ha probado ser efectivo primero


en Japón y luego a América. Se trata de la participación e involucramiento de todos y cada
uno de los miembros de la organización hacia la optimización de cada máquina.

Sobre la base de lo antes expuesto se puede definir el mantenimiento como:

Un proceso compuesto por una serie de operaciones técnicas que son de aplicación di-
recta, estructural y de control económico y que debe ser capaz de prolongar el ciclo vital
de la maquinaria, las instalaciones y los edificios para que el valor de las inversiones se
mantenga activo durante el tiempo de amortización e incluso después.

Las acciones u operaciones que se consideran directas y propias del trabajo de man-
tenimiento son las inspecciones, revisiones y pruebas, la lubricación y engrase y las repara-
ciones de cualquier tipo o nivel.

OBJETIVOS DEL MANTENIMIENTO

Los objetivos del mantenimiento son:


•• Atenuar y combatir el desgaste y la destrucción tanto de la infraestructura
como de las instalaciones.
•• Restaurar una operación y funcionamiento original.
•• Garantizar una operación y funcionamiento continuo, confiable y seguro
sin interrumpir los servicios que se prestan.

TIPOS DE MANTENIMIENTO

Existen cinco tipos reconocidos de operaciones de mantenimiento, los cuales están en


función del momento en el tiempo cuando se realizan, el objetivo particular para el cual son
puestos en marcha y en función a los recursos utilizados, así se puede hablar de manteni-
miento, ver figura 67:

340
Sánchez Marchán, Nanci. (2014).Introducción a la Ingeniería Industrial, Material Instruccional de Apoyo. Caracas: UNA

Figura 67. Tipos de mantenimiento.

MANTENIMIENTO CORRECTIVO
Este mantenimiento también es denominado mantenimiento reactivo, tiene lugar luego
que ocurre una falla o avería, es decir, sólo actuará cuando se presenta un error en el siste-
ma. En este caso si no se produce ninguna falla, el mantenimiento será nulo, por lo que se
tendrá que esperar hasta que se presente el desperfecto para tomar medidas de corrección
de errores. Este mantenimiento trae consigo las siguientes consecuencias:
•• Paradas no previstas en el proceso productivo, disminuyendo las horas
operativas.
•• Afecta las cadenas productivas, es decir, que los ciclos productivos pos-
teriores se verán parados a la espera de la corrección de la etapa ante-
rior.
•• Presenta costos por reparación y repuestos no presupuestados, por lo
que se dará el caso que por falta de recursos económicos no se podrán
comprar los repuestos en el momento deseado.
•• El tiempo que estará el sistema fuera de operación no es predecible.

341
Sánchez Marchán, Nanci. (2014).Introducción a la Ingeniería Industrial, Material Instruccional de Apoyo. Caracas: UNA

Clasificación del mantenimiento correctivo

Se clasifica en:
•• No planificado: es el mantenimiento de emergencia (reparación de ro-
turas). Debe efectuarse con urgencia ya sea por una avería imprevista a
reparar lo más pronto posible o por una condición imperativa que hay que
satisfacer (problemas de seguridad, de contaminación, de aplicación de
normas legales, etc.).
•• Planificado: se sabe con antelación qué es lo que debe hacerse, de
modo que cuando se pare el equipo para efectuar la reparación, se dis-
ponga del personal, repuesto y documentos técnicos necesarios para
realizarla correctamente.

Ejemplo Ilustrativo
Cuando fallan los bombillos en los hogares, empresas, instituciones públicas y
privadas, calles y avenidas, etc, se deben reponer para corregir la falla, de ello se
deduce que se aplica un mantenimiento correctivo, ya que no se tomaron previsio-
nes para cambiarlos antes de que dejaran de funcionar.

MANTENIMIENTO PREVENTIVO
Este mantenimiento también es denominado mantenimiento planificado, tiene lugar an-
tes de que ocurra una falla o avería, se efectúa bajo condiciones controladas sin la existencia
de algún error en el sistema. Se realiza a razón de la experiencia y pericia del personal a
cargo, los cuales son los encargados de determinar el momento necesario para llevar a cabo
dicho procedimiento; el fabricante también puede estipular el momento adecuado a través de
los manuales técnicos. Presenta las siguientes características:
•• Se realiza en un momento cuando no se está en producción, por lo que
se aprovechan las horas ociosas de la planta.
•• Se lleva a cabo siguiendo un programa previamente elaborado donde se
detalla el procedimiento a seguir además de las actividades a realizar, a
fin de tener las herramientas y repuestos necesarios a la mano.
•• Cuenta con una fecha programada, un tiempo de inicio y de terminación
preestablecidos, aprobados por la directiva de la empresa.
•• Está destinado a un área en particular y a ciertos equipos específicamen-
te. Aunque también se puede llevar a cabo un mantenimiento generaliza-
do de todos los componentes de la planta.
•• Permite a la empresa contar con un historial de todos los equipos, ade-
más brinda la posibilidad de actualizar la información técnica de los equi-
pos.
•• Permite contar con un presupuesto aprobado por la directiva.

342
Sánchez Marchán, Nanci. (2014).Introducción a la Ingeniería Industrial, Material Instruccional de Apoyo. Caracas: UNA

Para realizar la prevención se consideran las fuentes internas de las que dispone la em-
presa, las cuales están constituidas por los registros o historiales de reparaciones existentes
en ésta, informan sobre las tareas de mantenimiento que el equipo ha tenido durante su per-
manencia en las instalaciones. Forman parte de estas fuentes, los archivos de los equipos e
instalaciones, con sus listados de partes, especificaciones, planos generales, de detalle, de
despiece, los archivos de inventarios de piezas y partes de repuesto, y por último, los archi-
vos del personal disponible en mantenimiento con el detalle de su calificación, habilidades,
horarios de trabajo, sueldos, entre otros..

Ejemplo Ilustrativo
¿Cuál es el mantenimiento preventivo que se puede hacer a un teclado de
computadora?
El mantenimiento preventivo que se puede hacer a un teclado consiste básica-
mente en la limpieza exterior, ya que éste acumula bastante suciedad producida
por la manipulación de los usuarios y el medio ambiente. Esta limpieza se debe
hacer con un compuesto especial, creado para este propósito, generalmente su
presentación es en forma de crema. También existen espumas que permiten lim-
piar las teclas sin que se produzca humedad en el teclado, ya que la humedad
puede causar cortocircuitos.
Para realizar el mantenimiento interior, se debe destapar con cuidado el teclado,
observando la forma en como está armado, debido a que su desarme varía no-
tablemente de una marca a otra. Es necesario tener mucho cuidado con los tor-
nillos; estos generalmente vienen en diferentes tamaños y si se ubican en forma
equivocada pueden dañar el sistema de cierre

MANTENIMIENTO PREDICTIVO
El mantenimiento predictivo es el seguimiento del desgaste de una o más piezas o com-
ponentes de equipos principales a través de análisis de síntomas, o estimación realizada por
evaluación estadística, tratando de deducir el comportamiento de esas piezas o componen-
tes y determinar el punto exacto de cambio.

Es un mantenimiento programado que se realiza con el fin de prevenir la ocurrencia


de fallas. Se basa en la confiabilidad de los equipos que mide el tiempo medio entre fallas
consecutivas (MTTF) sin considerar las peculiaridades de una instalación dada. Ejemplos:
limpieza, lubricación, recambios programados. Este tipo de mantenimiento trata de antici-
parse a la aparición de las fallas, evaluando sus consecuencias para así aplicar las tareas
adecuadas de mantenimiento (preventivas o correctivas).

El sustento tecnológico de este mantenimiento consiste en la aplicación de algoritmos


matemáticos agregados a las operaciones de diagnóstico, que juntos pueden brindar infor-
mación referente a las condiciones del equipo. Tiene como objetivo disminuir las paradas por
mantenimientos preventivos y de esta manera minimizar los costos por mantenimiento. La

343
Sánchez Marchán, Nanci. (2014).Introducción a la Ingeniería Industrial, Material Instruccional de Apoyo. Caracas: UNA

implementación de este tipo de método requiere de inversión en equipos, en instrumentos y


en contratación de personal calificado.

Los aparatos e instrumentos que se utilizan son de naturaleza variada y pueden en-
contrarse incorporados en los equipos de control de procesos (automáticos), a través de
equipos de captura de datos o mediante la operación manual de instrumental específico. Es-
tos aparatos permiten analizar ruidos, vibraciones, aceites aislantes o espesores de chapa,
mediante las aplicaciones de la electrónica con equipos de ultrasonido, cromatografía líquida
y gaseosa y otros métodos.

A continuación se presentan las técnicas más utilizadas para la estimación del mante-
nimiento predictivo:
•• Analizadores de Fourier (para análisis de vibraciones)
•• Endoscopia (para poder ver lugares ocultos)
•• Ensayos no destructivos (a través de líquidos penetrantes, ultrasonido,
radiografías, partículas magnéticas, entre otros)
•• Termovisión (detección de condiciones a través del calor desplegado)
•• Medición de parámetros de operación (viscosidad, voltaje, corriente, po-
tencia, presión, temperatura, etc.)

Para llevar a cabo el mantenimiento predictivo, los equipos son monitorizados y evalua-
dos a intervalos. Las ventajas principales de llevar a cabo el mismo sin sacrificar la fiabilidad
del equipo son las siguientes:
•• Identificar fallos en sus primeros estados de aparición y anticipar la ne-
cesidad de reparaciones.
•• Reducir el stock de recambios, que se pueden solicitar sólo cuando se
necesiten.
•• Poder planificar y reservar los recursos para llevar a cabo las reparacio-
nes.
•• Reducir labores de mantenimiento preventivo.

MANTENIMIENTO PROACTIVO
El mantenimiento proactivo es una técnica enfocada en la identificación y corrección de
las causas que originan las fallas en equipos, componentes e instalaciones industriales, esta
técnica implementa soluciones que atacan la causa de los problemas no los efectos. Cam-
bios en los procedimientos, los productos, las cargas o el diseño del equipo para cambiar el
nivel de desgaste y extender la vida útil del equipo. El mantenimiento proactivo reduce los
gastos de mantenimiento en un porcentaje considerable

Este tipo de mantenimiento implica contar con una planificación de operaciones, la cual
debe estar incluida en el plan estratégico de la organización; a su vez debe brindar indicado-
res (informes) hacia la gerencia, respecto del progreso de las actividades, logros, aciertos y
también errores.

344
Sánchez Marchán, Nanci. (2014).Introducción a la Ingeniería Industrial, Material Instruccional de Apoyo. Caracas: UNA

El uso de aceites inferiores, repuestos baratos y rellenos de piezas no forma parte de un


programa de mantenimiento proactivo.

PM (MANTENIMIENTO PRODUCTIVO) Y TPM (MANTENIMIENTO


PRODUCTIVO TOTAL)

Mantenimiento productivo
El mantenimiento productivo o mantenimiento planificado (PM) es la etapa anterior al
mantenimiento productivo total (TPM) y es la evolución del mantenimiento correctivo. Se
caracteriza por la creciente mentalización, por la calidad y el consiguiente desarrollo de
técnicas para el control y aseguramiento de la calidad. En esta etapa, se produce un gran
desarrollo tecnológico en los medios de producción, impulsado por la necesidad de diseñar
equipos que puedan producir bienes de calidad exigidos por el mercado.

Este mantenimiento tiene como fundamento los principios de solidaridad, colaboración,


iniciativa propia, sensibilización, trabajo en equipo. Lo que conlleva que todos los involucra-
dos directa o indirectamente en la gestión del mantenimiento deben conocer la problemática
existente en esta materia, es decir, técnicos, profesionales, ejecutivos y directivos deben
estar conscientes de las actividades que se llevan a cabo para desarrollar las labores de
mantenimiento, asumiendo un rol en las prioridades del mantenimiento en forma oportuna y
eficiente.

Integra los tipos de mantenimiento preventivo y predictivo, aprovecha sus fortalezas


para corregir antes de fallar, pero permitiendo a cada componente acercarse al final de su
vida útil; logrando que los costos de mantenimiento se reduzcan. Las técnicas de manteni-
miento predictivo más usadas son: inspecciones VOSO (ver, oír, sentir y oler), mediciones y
monitoreo de vibraciones mecánicas, termografía, análisis de fallas, entre otras.

Mantenimiento productivo total (TPM).


El TPM corresponde a las siglas en inglés de mantenimiento productivo total, también
se puede considerar como mantenimiento de participación total. El objetivo es transformar la
actitud de todos los miembros de una organización involucrándolos en las labores de man-
tenimiento. La participación de gente que no está familiarizada con el funcionamiento de los
equipos favorece los resultados, pues ellos pueden detectar detalles que pasan desapercibi-
dos para quienes operan diariamente estos artefactos.

Entre los objetivos principales y fundamentales del TPM se tienen:


•• Reducción de averías en los equipos.
•• Reducción del tiempo de espera y de preparación de los equipos.
•• Utilización eficaz de los equipos existentes.
•• Control de la precisión de las herramientas y equipos.

345
Sánchez Marchán, Nanci. (2014).Introducción a la Ingeniería Industrial, Material Instruccional de Apoyo. Caracas: UNA

•• Promoción y conservación de los recursos naturales y economía de


energéticos.
•• Formación y entrenamiento del personal.

En el Mantenimiento Productivo Total o TPM se dividen las actuaciones del manteni-


miento según su nivel de complejidad. Se asignan las operaciones con menor carga técnica
al personal de operación y el resto de operaciones las lleva a cabo el personal de manteni-
miento propiamente dicho.

El TPM adopta como filosofía el principio de mejora continua desde el punto de vista del
mantenimiento y la gestión de equipos. Este tipo de mantenimiento ha recogido también los
conceptos relacionados con el mantenimiento basado en el tiempo (MBT) y el mantenimiento
basado en las condiciones (MBC).

Los principios fundamentales sobre los cuales se fundamenta el TPM son:


•• Participación de todo el personal, desde la alta dirección hasta los ope-
rarios de planta. Incluir a todos y cada uno de ellos permite garantizar el
éxito del objetivo.
•• Creación de una cultura corporativa orientada a la obtención de la máxi-
ma eficacia en el sistema de producción y gestión de los equipos y ma-
quinarias, de esta forma se puede llegar a la eficacia global, que facilite
la eliminación de las pérdidas antes de que se produzcan y se consigan
los objetivos.
•• Implantación del mantenimiento preventivo como medio básico para al-
canzar el objetivo de cero pérdidas, mediante actividades integradas en
pequeños grupos de trabajo y apoyado en el soporte que proporciona el
mantenimiento autónomo.
•• Aplicación de los sistemas de gestión de todos los aspectos de la produc-
ción, incluyendo diseño, desarrollo, ventas y dirección.
•• Garantiza a las empresas resultados en cuanto a la mejora de la pro-
ductividad de los equipos, mejoras corporativas, mayor capacitación del
personal y transformación del puesto de trabajo.

Ejercicio Ilustrativo
En la tabla mostrada se observan ciertas operaciones de mantenimiento que per-
miten alcanzar algunos objetivos concernientes a la preservación de maquinarias
y equipos utilizados en la producción:

346
Sánchez Marchán, Nanci. (2014).Introducción a la Ingeniería Industrial, Material Instruccional de Apoyo. Caracas: UNA

Operación Objetivo
Cambio de aceite en los motores des- Preservarlos del desgaste prematuro.
pués de cierto recorrido de los vehícu-
los.
Reajuste de las tensiones de las co- Para lograr que la transmisión entre
rreas en poleas. poleas y correas siga siendo eficiente
(deslizamiento).

Engrase en las máquinas de una indus- Para evitar la fricción, el desgaste entre
tria. piezas.
Reposición de cauchos deteriorados en Evitar accidentes por deslizamientos en
vehículos. el pavimento.

Todas las operaciones de mantenimiento estudiadas anteriormente tienen un único fin,


anticiparse a la aparición de las fallas, evaluando sus consecuencias para así, aplicar las
tareas adecuadas y prevenir antes de que ocurra una falla o avería. Es decir se aplica man-
tenimiento predictivo y preventivo.

Actividad propuesta Nº 1
a. ¿Qué relación tiene el mantenimiento con la seguridad laboral?
b. Considera que el mantenimiento sólo debe ser realizado por el departamento
encargado de esta acción.

Con la aplicación del TPM Todos los miembros de la organización se deben


hacer responsables de la conservación del equipo, el cual se vuelve más
productivo, seguro y fácil de operar.

PLAN GENERAL DE MANTENIMIENTO


El plan general de mantenimiento incorpora elementos que permiten realizar un segui-
miento detallado para garantizar eficacia y eficiencia en las mejoras a realizar. La jerarquiza-
ción de las acciones implicará especificar tareas y asignar responsables.

Para estructurar el plan de acción hay que:

1. Identificar las tareas necesarias y su posible secuencia para cumplir las acciones
planteadas.
2. Determinar quién es el responsable de la puesta en marcha y de la ejecución de las
tareas a desarrollar.

347
Sánchez Marchán, Nanci. (2014).Introducción a la Ingeniería Industrial, Material Instruccional de Apoyo. Caracas: UNA

3. Identificar los recursos humanos y materiales necesarios para ejecutar las tareas.
4. Definir la fecha de inicio y culminación de cada acción.
5. Identificar los indicadores de seguimiento o productos visibles de las tareas que
muestran cómo una acción es implementada y cómo se avanza en su ejecución.
6. Tomar en cuenta la información acumulada por el departamento de control de cali-
dad de la empresa.
7. Evaluar los costos involucrados.
8. Llevar a cabo un seguimiento y evaluación del plan de acción. El monitoreo de las
acciones de mejora y el cumplimiento de las tareas a ejecutar, son aspectos impor-
tantes a tener en cuenta.

Ejemplo de un formato para registrar información de un plan general de mantenimiento:

SECTOR ESTRATÉGICO: ÁREA COMÚN:

Acciones Tiempos Responsable


Responsa- Recursos Indicador
de mejora Tareas Costos de
ble de tarea inicio final necesarios seguimiento
(jerarquía) seguimiento
1 a
b
c
.
.
.
n
2 a
b
c
.
.
.
n
.
.
.
n

348
Sánchez Marchán, Nanci. (2014).Introducción a la Ingeniería Industrial, Material Instruccional de Apoyo. Caracas: UNA

Actividad propuesta Nº 2
Investigue en medios impresos o electrónicos ¿por qué un plan de manteni-
miento debe tomar en cuenta la información acumulada por el departamento de
control de calidad de la empresa?

La industria de hoy exige no sólo la inversión en nuevas instalaciones de producción


para aumentar su productividad, es también imperante utilizar las instalaciones eficazmente,
donde uno de los requisitos más importante es el establecimiento de un servicio ordenado
y técnico de mantenimiento, que sea eficiente, seguro y económico al utilizar los equipos,
herramientas e instalaciones.

349
Sánchez Marchán, Nanci. (2014).Introducción a la Ingeniería Industrial, Material Instruccional de Apoyo. Caracas: UNA

PROCESOS DE MANUFACTURA

Objetivo:
Indicar el efecto del Proceso de manufactura en el desarrollo de bienes indus-
triales, a partir de la necesidad de lograr procedimientos eficaces y eficientes para
el procesamiento de materiales de acuerdo a sus propiedades.

•• Desarrollo de los procesos de manufactura.


•• Materiales utilizados en manufactura.
•• Propiedades de los materiales.
•• operaciones básicas de manufactura.
•• Clasificación de los procesos de manufactura.
•• Métodos avanzados de manufactura.

DESARROLLO DE LOS PROCESOS DE MANUFACTURA


La manufactura es el origen fundamental de la industria, la palabra significa hacer a
mano, pero en economía significa transformar la materia prima en un producto de utilidad
específica. Casi todos los bienes que usa el hombre son fruto del proceso manufactura rea-
lizado en grandes fábricas o de forma artesanal.

El origen de los procesos de manufactura pueden acreditarse a Eli Whitney, Taylor, en-
tre otros. (Ver tabla 2).

Miron L. Begeman y otros investigadores, lograron nuevos avances en las técnicas de


fabricación. Estudios que se han aprovechado en la manufactura, asociados al conocimiento
de los principios y la aplicación de electricidad, electrónica y las computadoras; permitiendo
la producción de máquinas avanzadas que agilizan y mejoran los procesos de fabricación.

CONCEPTO DE MANUFACTURA
La definición de manufactura se origina del latin manus que significa mano y factura que
se traduce como hechura. Sobre esta base se puede decir que la manufactura es:

351
Sánchez Marchán, Nanci. (2014).Introducción a la Ingeniería Industrial, Material Instruccional de Apoyo. Caracas: UNA

El proceso a través del cual se crean productos de forma manual, mecánica, automática
o a través de combinaciones de estas técnicas de procesamiento.

La definición moderna de manufactura integra todas las actividades requeridas para


transformar las materias primas en productos terminados hasta su funcionamiento. En este
sentido se puede hablar de una definición de manufactura tecnológica y otra económica:

Manufactura tecnológica: es el uso de procesos físicos y químicos para alterar la


geometría, propiedades o apariencia de un material inicial para hacer piezas o productos.

Manufactura económica: es la transformación de materiales en artículos de mayor


valor por medio de una o más operaciones de procesamiento.

MATERIALES UTILIZADOS EN MANUFACTURA


La selección del equipo, así como, del proceso de producción está influenciada por los
materiales a utilizar. Se pueden mencionar cuatro tipos de materiales básicos:
•• Metales.
•• Cerámicas.
•• Polímeros.
•• Compuestos, consisten generalmente de la combinación de dos tipos
básicos de materiales.

Propiedades de los materiales

Soldabilidad:

Procedimiento por el cual dos o más piezas de metal se unen por aplicación de calor,
presión, o una combinación de ambos, con o sin el aporte de otro metal, cuya temperatura
de fusión es inferior a la de las piezas que han de soldarse. La mayor parte de procesos de
soldadura se pueden separar en dos categorías: soldadura por presión, que se realiza sin
la aportación de otro material, se efectúa solo mediante la aplicación de presión suficiente y
normalmente ayudada con calor; y soldadura por fusión, realizada mediante la aplicación de
calor a las superficies, que se funden en la zona de contacto, con o sin aportación de otro
metal. En cuanto a la utilización de metal de aportación se distingue entre soldadura ordina-
ria y soldadura autógena. Esta última se realiza sin añadir ningún material.

Templabilidad:

Proceso de baja temperatura en el tratamiento térmico del material, especialmente el


acero, con el cual se obtiene el equilibrio deseado entre la dureza y la tenacidad del producto

352
Sánchez Marchán, Nanci. (2014).Introducción a la Ingeniería Industrial, Material Instruccional de Apoyo. Caracas: UNA

final. Las piezas de acero endurecidos se calientan a una temperatura elevada, pero bajo el
punto de fusión del material. Luego se enfrían rápidamente en aceite o en agua para lograr
un material más duro, con menos estrés interno, pero más frágil. Para reducir la fragilidad, el
material pasa por un recocido que aumenta la tenacidad y disminuye su dureza,

Recocido:

Proceso de tratamiento térmico a través del cual el vidrio, ciertos metales y aleaciones
se hacen menos quebradizos y más resistentes a la fractura. Este procedimiento minimiza
los defectos internos en la estructura atómica del material y elimina posibles tensiones inter-
nas provocadas en las etapas anteriores de su procesado.

El recocido es necesario como paso intermedio en procesos de manipulación de meta-


les, como la fabricación de alambre o el estampado en latón, para recuperar la ductilidad que
el metal a tratar pierde debido al endurecimiento producido durante la operación de modela-
do y para obtener los más bajos valores de resistencia a la deformación.

Dureza:

Propiedad de un material sólido relacionada con la resistencia a la deformación o abra-


sión de la superficie. También se describe como la resistencia a la penetración del material
en cuestión.

La dureza está relacionada con la solidez, durabilidad y resistencia de los sólidos. Exis-
ten diversas pruebas para determinar el valor de la dureza, tales como:
•• Prueba Brinell (BHN): En una prensa se coloca una probeta con la su-
perficie superior plana y se presiona esa superficie con un balín de acero
que tiene una carga de 500 Kg (materiales blandos) o 3.000 Kg (materia-
les duros). El diámetro de la huella impresa determina el valor de dureza.
•• Prueba Vickers (VHN): Una pirámide de diamante se presiona contra
una probeta, bajo cargas más livianas que la prueba Brinell. La diagonal
de la impresión determina el número de la dureza.
•• Prueba Rockwell(Ra, Rb, etc.): un cono de diamante (ensayo Rc) es
presionada en una probeta. La profundidad de la huella determina el nú-
mero de dureza. Para materiales más blandos se utiliza la prueba Rb, la
cual reemplaza al cono de diamante por un balín y se reduce la carga
empleada.

Existen otras pruebas y escalas de dureza como la escala de Mohs (resistencia a las
ralladuras) y la realizada con un escleroscopio.

353
Sánchez Marchán, Nanci. (2014).Introducción a la Ingeniería Industrial, Material Instruccional de Apoyo. Caracas: UNA

Maquinabilidad:

Propiedad que determina la capacidad de mecanización de un material. Está relaciona-


da con los procesos en los cuales existe arranque de material o viruta como:
•• cizallado: proceso por el cual se corta una plancha o una pieza metálica
en frío por medio de tijeras o cizallas.
•• torneado: operación que consiste en trabajar una pieza en un torno,
máquina-herramienta en la que se asegura y se hace girar la pieza a
trabajar, para pulirla o labrarla. Existen varios tipos de torneado como el
simple o recto y el cónico y horadado.
•• taladrado: operación que consiste principalmente en la abertura, agran-
damiento, corte y acabado de agujeros en una pieza.

También están el fresado, el cepillado y el rectificado entre otros procesos que involu-
cran maquinabilidad.

Isotropía:

Un material o pieza es isotrópico cuando presenta exactamente las mismas propieda-


des en todas las direcciones. Lo contrario es que sea anisotrópico, o sea, que tenga propie-
dades distintas para cada dirección (propiedades direccionales).

Colabilidad:

Propiedad que tiene relación con la fluidez que adquiere un material una vez alcanzada
la temperatura de fusión. Tiene gran importancia en procesos de fundición, en los cuales a
través del vertido de metal fundido sobre un molde hueco, por lo general hecho de arena,
se obtienen piezas metálicas. La fundición implica tres procesos diferentes: en primer lugar
se construye un modelo de madera, plástico o metal con la forma del objeto terminado; más
tarde se realiza un molde hueco rodeando el modelo con arena y retirándolo después; luego
se vierte metal fundido en el molde (este último proceso se conoce como colada).

Para que un material logre una fluidez adecuada y el proceso de fundición se lleve a
cabo con éxito, es necesario que la temperatura de colada sobrepase unos 110 ºC tempera-
tura de fusión; para evitar problemas de endurecimiento precoz del material.

Conformabilidad:

Propiedad del material que determina su moldeabilidad. En estado líquido tiene relación
con el tipo de fundición que se emplee (molde-vaciado, pieza fundida, etc.). En estado sólido
está relacionada con procesos de deformación plástica del material (trefilado, laminado, etc.).

354
Sánchez Marchán, Nanci. (2014).Introducción a la Ingeniería Industrial, Material Instruccional de Apoyo. Caracas: UNA

En estado granular, está ligada a la presión y a la temperatura que se apliquen a los granos
o polvo del material.

La conformabilidad en estado sólido presenta 3 casos: conformabilidad con conserva-


ción de masa (deformación plástica para materiales dúctiles y maleables), con reducción de
masa (torneado, cepillado, rectificado, taladrado, etc.) y de unión (remaches, soldaduras,
pegamentos, presión). Esta propiedad es de gran importancia a la hora de decidir el proceso
para lograr la pieza final proyectada.

OPERACIONES BÁSICAS DE MANUFACTURA


Se puede hablar de dos tipos básicos de operaciones de manufactura:

Operaciones de procesamiento, las cuales transforman el material de trabajo


de forma o estado, mediante cambios en su geometría o propiedades.

Operaciones de ensamblaje, donde se combinan las piezas para formar una


nueva entidad.

CLASIFICACIÓN DE LOS PROCESOS DE MANUFACTURA    


De manera general los procesos de manufactura se clasifican en cinco grupos:

1. Procesos que cambian la forma del material:


•• Metalurgia extractiva.
•• Fundición.
•• Formado en frío y en caliente.
•• Metalurgia de los polvos.
•• Moldeo de plásticos.
2. Procesos que provocan desprendimiento de viruta para obtener la forma, terminado
y tolerancias de las piezas deseadas:
•• Maquinado con arranque de viruta convencional.
•• Torno.
•• Fresado.
•• Cepillado.
•• Taladrado.
•• Brochado.
•• Rimado.
3. Procesos para acabado de superficies:
•• Por desprendimiento de viruta.
•• Por pulido.
•• Por recubrimiento.

355
Sánchez Marchán, Nanci. (2014).Introducción a la Ingeniería Industrial, Material Instruccional de Apoyo. Caracas: UNA

4. Procesos para el ensamble de materiales:


•• Ensambles temporales.
•• Ensambles permanentes.
5. Procesos para cambiar las propiedades físicas de los materiales:
•• Tratamientos térmicos.
•• Tratamientos químicos.

En un proceso de manufactura, los materiales entran en el almacén; salen de él para


su transformación a una y otra fase posterior de elaboración, según se requiera. Hasta que,
por último, vuelven al almacenamiento en forma de producto final ya acabado y a punto para
ser enviado.

Ejemplo Ilustrativo
A continuación se presentan las etapas correspondientes a dos procesos de ma-
nufactura, referentes a empresas donde se obtienen productos de marroquinería y
joyería, listos para ser enviados al cliente:

Marroquinería
Proceso de manufactura: cortado del cuero, secado, confección de producto, al-
macenamiento, oferta.

Joyería
Proceso de manufactura: explotación de minerales, eliminación del estéril, extrac-
ción y lavado, corte, oferta.

En la actualidad la manufactura se ha automatizado y un elemento indispensable en


gran parte de los procesos de manufactura es el robot, considerado como el rey de la auto-
matización, el cual se usan de manera extensa en la industria. Este aspecto fue estudiado
con profundidad en el tema manejo de materiales.

La Federación Internacional de la Robótica (IFR) estableció en 1998 una clasificación


de las aplicaciones de la robótica en el sector manufacturero y es la siguiente:

356
Sánchez Marchán, Nanci. (2014).Introducción a la Ingeniería Industrial, Material Instruccional de Apoyo. Caracas: UNA

Tabla 41. Clasificación de las aplicaciones de la robótica en el sector manufacturero


Aplicaciones de la robótica en el sector manufacturero

Manipulación en fundición:
•• Manipulación en moldeado de plásticos.
•• Manipulación en tratamientos térmicos.
•• Manipulación en la forja y estampación.
•• Soldadura:
- Por arco
- Por puntos
- Por gas
- Por láser
- Otros

Aplicación de materiales:
•• Pintura.
•• Adhesivos y secantes.
•• Otros.

Mecanización:
  •• Carga y descarga de máquinas.
•• Corte mecánico, rectificado, desbardado y pulido.
•• Otros.

Otros procesos:
•• Láser.
•• Chorro de agua.
•• Otros.

Montaje:
•• Montaje mecánico
•• Inserción
•• Unión por adhesivos.
•• Unión por soldadura.
•• Manipulación para montaje.
•• Otros.

Paletización:
•• Medición, inspección, control de calidad.
•• Manipulación de materiales.
•• Formación, enseñanza e investigación.
•• Otros.

357
Sánchez Marchán, Nanci. (2014).Introducción a la Ingeniería Industrial, Material Instruccional de Apoyo. Caracas: UNA

ETAPAS FUNDAMENTALES DE LOS PROCESOS DE MANUFACTURA


Dentro de las etapas fundamentales de los procesos de manufactura se encuentran:

1. Almacenamiento. Disposición de materiales en un lugar determinado, estrechamen-


te relacionado con orden, aseo y condiciones de seguridad.
2. Transporte. Movimiento de materiales en diferentes sentidos y dirección.
3. Mecanizado. Proceso de fabricación que contempla varias operaciones, para elimi-
nar material por arranque de viruta o por abrasión.
4. Transformación de materias primas: actividad ejecutada por operarios con el apoyo
de diferentes energías, para obtener productos elaborados.
5. Acabados. Calidad final que contribuye a la mejor presentación de un producto.
6. Ensamble. Unión entre dos o más partes para formar un conjunto o subconjunto
completo.
7. Línea de producción. Distribuir tareas y procesos individuales en estaciones y cel-
das de trabajo, para ahorrar recursos y generar ganancias.
8. Empaque. Actividades necesarias para diseñar y producir el recipiente o la envoltura
para un producto. Ejercen un papel importante en la publicidad y en la venta.

Actividad Propuesta Nº 1
Elabore un listado de procesos industriales de manufactura que estén auto-
matizados.

ENERGÍA EN LOS PROCESOS DE MANUFACTURA


La energía es la capacidad de un sistema físico para realizar trabajo. La materia posee
energía como resultado de su movimiento o de su posición en relación con las fuerzas que
actúan sobre ella. Por ejemplo: la energía mecánica procede de la transformación de la
energía potencial del agua almacenada en un embalse. La energía eléctrica es producida
por los generadores acoplados a las turbinas. La energía térmica es la energía calorífica que
se produce mediante la quema de un combustible, que puede ser sólido, líquido o gaseoso.

En la tabla que se presenta a continuación, se menciona el tipo de energía requerida por


algunos procesos de manufactura:

358
Sánchez Marchán, Nanci. (2014).Introducción a la Ingeniería Industrial, Material Instruccional de Apoyo. Caracas: UNA

Tabla 42.Procesos comunes de manufactura y sus requerimientos de energía

Forma de energía
Proceso Acción lograda
utilizada

Funde el metal antes de colocarlo dentro de


Moldeado Térmica
moldes donde ocurre su solidificación.
Maquinado A través de descargas eléctricas, se remue-
por descarga Eléctrica ve material de la pieza ocasionado por altas
eléctrica temperaturas.
Se logra a través de modificar la forma original
Forjado Mecánica de un metal presionado entre dados, general-
mente a altas temperaturas.
La piezas se calientan por debajo de su punto
Templado Térmica de fusión para que las moléculas unifiquen su
estructura interna.
Un polímero transformado a consistencia plás-
Moldeado por
Térmica y mecánica tica mediante calor, se inyecta en un molde
inyección
para que tome la forma de éste.
El rayo láser crea una vaporización y fundición
Corte por láser Luminosa y térmica de los metales por los que pasa, haciendo
cavidades al paso de su haz.
Se elimina el material sobrante mediante el
Maquinado Mecánica movimiento relativo de las piezas contra las
herramientas y viceversa.
Mediante dados y sellos, las partes metálicas
Troquelado Mecánica
toman su forma.
A través del calor se funde una parte del metal
Soldadura Térmica
de la pieza metálica para adherirse a otra.

Algunas formas de energía pueden llegar a ser muy costosas, en la realización de estos
procesos, lo que induce a investigar otras fuentes alternas de energía tales como: la natural,
la atómica, la eólica, la solar, la hidráulica, entre otras. Todas tienen en común que son na-
turales, inagotables y limpias.

MÉTODOS AVANZADOS DE MANUFACTURA

Se refiere a un enfoque para el diseño de producto en el cual las empresas intentan


reducir el tiempo que se requiere para llevar acabo un nuevo producto al mercado, en este
sentido se dice que se aplica la ingeniería concurrente (o también conocida como Ing. Si-
multánea) donde la implantación de manufactura empieza cuando el diseño de producto se
está desarrollando.

359
Sánchez Marchán, Nanci. (2014).Introducción a la Ingeniería Industrial, Material Instruccional de Apoyo. Caracas: UNA

El diseño para la manufactura y el ensamble, es el aspecto más importante de la inge-


niería concurrente, debido a que tiene el mayor impacto en los costos de producción y en el
tiempo de desarrollo del producto; por ello se desarrolla la capacidad para diseñar y realizar
productos de alta calidad en el tiempo mínimo. En este sentido se implementan tres técnicas,
las cuales dependen de los datos generados en el diseño, a través de un sistema gráfico
computarizado. Hablar de esto implica mencionar la gran precisión con la que se realizan
los trazos, gracias al modelo gráfico computarizado de la geometría de partes. Las técnicas
antes mencionadas son:

1. Estereolitografía. Es un proceso para fabricar una parte plástica sólida a partir de un


archivo de datos. Generando un modelo sólido mediante un sistema grafico compu-
tarizado de la geometría de partes, controlado por un rayo láser.

2. Sinterizado selectivo con láser. Este proceso es similar al anterior sólo que en lugar
de utilizar un polímero liquido se utilizan polvos y se comprime por el rayo láser has-
ta formar las capas que van a formar la pieza.

3. Modelado por deposición fundida: Este proceso se basa en irle dando forma con el
rayo láser a una pieza cuyo material es similar a la cera.

En la manufactura existen productos que se fabrican en varias etapas y necesitan di-


versos procesos automatizados, haciendo uso de robots como por ejemplo los automóviles:
requieren piezas que pueden ser elaboradas por la empresa automotriz en cuestión o son
adquiridas en empresas nacionales o internacionales.

Así como productos más sencillos que requieren menos elaboración como el caso de
los artículos fabricados con piedra tallada, arcilla, madera, entre otros.

En los procesos de manufactura existen filosofías, metodologías y herramientas que


facilitan su avance. Con el transcurrir del tiempo, los equipos, máquinas y sistemas utilizados
para organizar e integrar las operaciones desde la planta hasta las transacciones comercia-
les de negocio, se han agilizado con la mecanización y automatización de los procesos.

360
Sánchez Marchán, Nanci. (2014).Introducción a la Ingeniería Industrial, Material Instruccional de Apoyo. Caracas: UNA

PREPARACIÓN DE PROYECTOS DE INVERSIÓN

Objetivo:
Determinar la relevancia que tiene la Evaluación de Proyectos para la Ingeniería
Industrial, al fundamentar una decisión de aceptación o rechazo de un proyecto en
estudio.

•• Origen de los proyectos.


•• Definición de proyecto de inversión.
•• Tipos de proyectos.
•• Etapas del proyecto de inversión.

ORIGEN DE LOS PROYECTOS


Los proyectos existen desde siempre, partiendo de las antiguas pirámides de Egipto
o los monumentales edificios de la antigua Roma o la antigua Grecia, hasta las complejas
plataformas tecnológicas de hoy día.

Numerosas técnicas empleadas actualmente por la mayoría de los directores de proyec-


tos de todo el mundo, tienen ya muchos años de historia desde el momento en que se utili-
zaron por primera vez. Un buen ejemplo de ello es el Diagrama de Gantt, técnica popular de
planificación gráfica que fue desarrollada por Henry Lawrence Gantt y aplicada por primera
vez entre 1931 y 1935. Ver tema estudio del trabajo.

Fue en los años 50 y 60 del siglo XX, cuando la gestión de proyectos comenzó a profe-
sionalizarse, hasta entonces había sido meramente una disciplina. En los años 70 se puede
observar una gestión de proyectos tradicional. Superados los años 70 hasta nuestros días,
las cosas han cambiado, se asume que se vive en un mundo predecible para desarrollar los
proyectos, tomando en cuenta la opinión y satisfacción del cliente.

En este sentido surgen nuevas técnicas y herramientas para gestionar los proyectos:
gestión de contratos, análisis de costo-beneficio, análisis de riesgos del entorno, negociación
y gestión de cambios, entre otros. Además, se incorporan dos fases adicionales al ciclo de
vida tradicional: la de mantenimiento y la de operación.

Una de las mayores innovaciones en los últimos años es, la humanización del gestor
de proyectos, el cual se ha preocupado por adaptar los proyectos a las necesidades de la
sociedad.

361
Sánchez Marchán, Nanci. (2014).Introducción a la Ingeniería Industrial, Material Instruccional de Apoyo. Caracas: UNA

CONCEPTO DE PROYECTO DE INVERSIÓN


Un proyecto de inversión es:

La aplicación de conocimientos, herramientas y técnicas para dar respuesta a una nece-


sidad económica o social.

La aplicabilidad o viabilidad de un proyecto obedece a estimar las ventajas y desventa-


jas de asignar recursos a su realización, garantizando así la satisfacción de objetivos. Razón
por la cual se puede distinguir dos variantes de proyectos: económicos y sociales.

Se puede definir un proyecto económico como:

Un plan que busca generar una rentabilidad económica, sobre la base de la formulación
y evaluación de una inversión para implementar la producción económica de un bien o
servicio. Es promovido por empresas e individuos.

Ejemplo Ilustrativo de un proyecto económico


Caso: una cadena de establecimientos minoristas dedicada a la  comercializa-
ción de confites, estudia la conveniencia de comprar una propiedad y trasladar
una de sus tiendas al centro de la ciudad, la cual actualmente se encuentra
funcionando en las afueras de dicha ciudad, en un local arrendado por un costo
de 10.000 Bs. mensuales.

Proyecto: adquisición de una propiedad para la reubicación de la tienda.


Costos: el precio de adquisición de la tienda es de 700.000 Bs que incluye:
Terreno: 200.000 Bs.
Edificio: 400.000 Bs.
Obras adicionales por un monto de 100.000 Bs.

Beneficios: con el traslado de la tienda se espera incrementar las ventas en un


80% y eliminar el costo de arrendamiento.

Actividad propuesta
Formule un proyecto de inversión referente a la instalación de una planta de ener-
gía alterna (eólica, hidráulica, entre otras) hacia el oriente del país.

Un proyecto social es:

Un plan que busca generar un impacto sobre la calidad de vida de una población objeto
de estudio. Promovido por entes gubernamentales, las ONG, así como las empresas a
través de sus políticas de responsabilidad social.

362
Sánchez Marchán, Nanci. (2014).Introducción a la Ingeniería Industrial, Material Instruccional de Apoyo. Caracas: UNA

Ejemplo Ilustrativo de un proyecto social


Caso: actualmente las aguas del Lago de Valencia se encuentran contaminadas
por compuestos químicos de plantas industriales y descarga de aguas servidas.
Proyecto: tratamiento de aguas servidas (proyecto de saneamiento)
Beneficios: mejoramiento de la salud de la población y recuperación del lago.
Costos de inversión: planta de tratamiento y recursos necesarios para operarla.

Actividad propuesta
Formule un proyecto de inversión referente a la creación de un centro recreacio-
nal familiar.

Cuando una empresa hace una inversión incurre en un desembolso de efectivo


con el propósito de generar en el futuro, beneficios económicos que ofrezcan un
rendimiento atractivo para quienes invierten o un beneficio social que mejore la
calidad de vida en una comunidad o sociedad en general.

Existen dos motivos para que la empresa realice una inversión:


•• Por excedente de recursos. La empresa puede invertir estos recursos
por ejemplo, en nuevas tecnologías, para mejorar los procesos y produc-
tos.
•• Por necesidades específicas. Cuando se requiere resolver situaciones
generadas en las diferentes áreas de la empresa: producción, adminis-
tración, calidad, entre otras. O resolver problemas sociales existentes en
las comunidades.

La conveniencia económica de un proyecto se mide a través del beneficio que logra


quien o quienes lo emprenden.

TIPOS DE PROYECTOS

Los proyectos surgen de la necesidad de crear un nuevo negocio o para mejorar uno en
marcha. En este sentido se puede hablar de proyectos:

Según la categoría:
•• Productivos: agrícolas, petroleros, pecuarios, frutícolas, pesqueros, fi-
nancieros, entre otros.
•• Infraestructura: energía, carreteras, puentes, puertos, hospitales, hote-
les, entre otros.
•• Sociales: salud, educación, vivienda, saneamiento ambiental, esparci-
miento, capacitación, entre otros.

363
Sánchez Marchán, Nanci. (2014).Introducción a la Ingeniería Industrial, Material Instruccional de Apoyo. Caracas: UNA

Según el ámbito geográfico: nacional, regional, local.

Según su utilización:
•• De bienes: que producen artículos o mercancías. Los cuales pueden ser
de consumo final, intermedio y de capital.
•• De servicios: cuyo producto final es un servicio público o privado. Se
pueden dividir en servicios de infraestructura económica, social, educa-
tiva, entre otros.
•• De investigación: el producto no es un bien o un servicio, sino conoci-
mientos. Pueden ser de investigación teórica, experimental o aplicada.

Actividad propuesta
Formule un proyecto de inversión para cada uno de los tipos de proyectos indi-
cados anteriormente.

ETAPAS DEL PROYECTO DE INVERSIÓN

Figura 68. Fases del proyecto de inversión.

A continuación se explican cada una de estas fases.

364
Sánchez Marchán, Nanci. (2014).Introducción a la Ingeniería Industrial, Material Instruccional de Apoyo. Caracas: UNA

Abarca todas las etapas que se desarrollan antes de que se realice la


inversión propiamente dicha. Se consideran las siguientes etapas:
• Idea: se enuncia el proyecto, se plantea sus características funda-
mentales y se analizan.
• Perfil de un proyecto: se aborda la información existente a través
del juicio o sentido común y la experiencia. Permite seleccionar diver-
sas alternativas que por razones de peso sea la más atractiva.
Preinversión
(Anteproyecto) • Estudio de prefactibilidad: significa proponer el plan para el logro
Formulación o de un objetivo, sobre la base de fuentes secundarias de información,
no muestra la profundidad y exactitud de un proyecto. Sus etapas
preparación del
son: estudio de mercado, técnico, administrativo y financiero, es decir
proyecto se realiza una evaluación del proyecto (se detallan más adelante, ver
evaluación de proyecto).
• Estudio definitivo (factibilidad): se realiza un análisis amplio de
los resultados financieros, económicos y sociales de una inversión,
es decir, contiene toda la información del anteproyecto de manera
FASES DEL PROYECTO DE INVERSIÓN

más precisa y detallada, que permite tomar una decisión de acepta-


ción o de rechazo, así como una reducción en el margen de error de
las estimaciones de los costos y beneficios. Se consideran también
aquí, los criterios de evaluación para llevar a cabo el proyecto. (Ver
evaluación de un proyecto).
Considera todos los requerimientos y especificaciones que exige la natu-
raleza del proyecto. Abarca desde el momento de invertir hasta la puesta
en marcha del proyecto, es decir, hasta que éste se encuentre en condi-
ciones de operar normalmente. Se clasifica en dos grandes secciones:
Inversión fija, la cual está integrada por:
• Gastos preoperativos (como estudios, patentes, organización, permi-
sos, entre otros)
Inversión o diseño • Terrenos y edificios.
(Proyecto) • Maquinaria y equipo de producción.
• Equipo y mobiliario de oficinas.
• Equipo de transporte para ventas, entre otros.
Inversión de capital de trabajo: inversión adicional que debe aportarse
para que la empresa empiece a elaborar el producto: comprar materias
primas, pagar mano de obra directa que la transforme, otorgar crédito en
las primeras ventas y contar con cierta cantidad de efectivo para sufragar
los gastos diarios de la empresa, entre otros.
Se realiza el funcionamiento u operación en el que se espera alcanzar los
objetivos y metas que dieron origen al proyecto de inversión, de acuerdo
Operación a los planes previstos en los estudios de pre inversión. Se caracteriza por
la vida productiva y operativa del proyecto y está determinada por la vida
útil de éste, además se empieza a generar los beneficios que revierten
los gastos ocasionados en la etapa de implementación o inversión.
El proyecto se presenta como la acción que genera una respuesta al
Evaluación de problema, oportunidad o necesidad.Se verifica después de un tiempo
resultados razonable de la operación, si efectivamente el problema ha sido solucio-
nado por la puesta en marcha del proyecto.

365
Sánchez Marchán, Nanci. (2014).Introducción a la Ingeniería Industrial, Material Instruccional de Apoyo. Caracas: UNA

CICLO DE UN PROYECTO DE INVERSIÓN

Figura 69. Ciclo de un proyecto de inversión.

Como se puede observar en la figura 64, la evaluación de un proyecto de inversión se


realiza en todas sus fases, desde la preinversión a través de un análisis financiero para de-
terminar su viabilidad hasta después de su puesta en marcha para determinar que se está
ejecutando de acuerdo a lo planeado. En el resto de las fases se generan puntos de control
y evaluación para lograr el seguimiento de los procesos.

EVALUACIÓN DE UN PROYECTO DE INVERSIÓN

Se deben tener en cuenta las diferentes fases de un proyecto, para determinar dónde
establecer los puntos de control y evaluación del mismo. Se establece la viabilidad del pro-
yecto considerando estudios referentes a: entorno, mercado, técnico, administrativo, legal,
económico y/o financiero, impacto ambiental; de una iniciativa de negocio nuevo o en mar-
cha.

Sobre la base de lo antes mencionado se puede definir la evaluación económica de


proyectos como:

Un instrumento o herramienta para determinar, mediante un análisis de costo-beneficio


si se genera o no el rendimiento deseado para entonces tomar la decisión de realizarlo o
rechazarlo.

La evaluación de un proyecto genera información que permite realizar un plan de acción


para la utilización productiva de los recursos económicos de los que dispone una empresa,
emitiendo una crítica sobre la conveniencia y confiabilidad de la estimación preliminar del be-
neficio que genera el proyecto en estudio. Se detalla a continuación los diferentes aspectos
que contempla la evaluación de un proyecto:

366
Sánchez Marchán, Nanci. (2014).Introducción a la Ingeniería Industrial, Material Instruccional de Apoyo. Caracas: UNA

Estudio del entorno


Escenario en el cual se desarrollará el proyecto, da una primera aproximación sobre la
pertinencia de avanzar en éste.

Estudio de mercado (Ver tópico de Marketing Industrial)

Los principales puntos que debe considerar un estudio de mercado son:


•• Perfil de los productos o servicios.
•• Análisis de la demanda.
•• Análisis de los precios.
•• Análisis de la oferta.
•• Análisis de los canales de distribución.
•• Promoción.

Estudio técnico
En esta etapa se diseña como producir lo que se venderá, es decir cómo fabricar o ge-
nerar los productos o servicios. Se consideran aquí los siguientes aspectos:
•• Procesos de fabricación.
•• Localización general y específica del proyecto.
•• Materia primas e insumos requeridos.
•• Equipo, maquinaria e instalaciones necesarias.
•• Capacidad de producción.
•• Tecnología.
(Ver temas: producción, localización y distribución en planta, inventarios, entre otros)

Estudio administrativo
El estudio debe comprender las siguientes etapas (ver tópico de administración de re-
cursos):
•• Supuesto.
•• Planeación.
•• Organización.
•• Integración.
•• Dirección.
•• Control.

Estudio legal

Se han de considerar aquí aspectos como:


•• Responsabilidad de la empresa.
•• Aspectos tributarios.
•• Riesgo que se debe asumir.
•• Persona natural o jurídica.

367
Sánchez Marchán, Nanci. (2014).Introducción a la Ingeniería Industrial, Material Instruccional de Apoyo. Caracas: UNA

Estudio del impacto ambiental

Tener en cuenta un desarrollo sostenible entre los consumidores y el medio ambiente de


los productos o servicios que proporciona la inversión, asociado a los menores costos, bajo
la Norma ISO 14.000.

Estudio económico-financiero (ver tema economía y finanzas)

El estudio económico-financiero utiliza la información de los estudios de mercado y téc-


nico para conseguir los flujos de efectivo positivos y negativos en el lapso de la planeación,
el monto de la inversión fija y flexible, las formas de financiamiento para la operación y la
evaluación para conocer el beneficio y la calidad de la inversión del proyecto. Se demuestra
en esta etapa que el proyecto puede ejecutarse con los recursos disponibles.

Usualmente la decisión final sobre la realización efectiva del proyecto dependerá


de las conclusiones de los estudios de mercado, técnico y financiero.

Este estudio es llamado también cuantitativo o de viabilidad y la información que se


debe conocer es:

Vida útil del proyecto. Es el tiempo en el que se considera que el proyecto generará
beneficios (ingresos) y deberá estar considerado en el espacio de planeación del proyecto.

Valor de rescate al final de la vida útil del proyecto. Es el monto de efectivo a recu-
perar por la inversión realizada al final de la vida útil del proyecto, es decir, genera un flujo de
efectivo positivo al final de la evaluación del proyecto.

Costo de capital o tasa mínima requerida. Es la tasa de ganancia anual que solicita
ganar el inversionista para llevar a cabo el proyecto. Esta tasa debe cubrir al menos el costo
de capital de los recursos utilizados por la empresa, en la adquisición de activos requeridos
para realizar el proyecto de inversión.

Flujos netos de efectivo. Un proyecto debe generar beneficios económicos futuros


que justifiquen la inversión. Estos beneficios deben ser calculados al hacer las proyecciones
financieras del proyecto y no son otra cosa que el importe obtenido por la operación del pro-
yecto. Existen dos métodos para determinarlo:
•• A través del estado de resultados presupuestado, adicionándole a la uti-
lidad neta las partidas virtuales.
•• A través del presupuesto de caja.

368
Sánchez Marchán, Nanci. (2014).Introducción a la Ingeniería Industrial, Material Instruccional de Apoyo. Caracas: UNA

Métodos cuantitativos para evaluar las diferentes alternativas de solución de un


proyecto

Se clasifican en:
•• Estáticos. No consideran el cambio de valor del dinero en el tiempo.
•• Dinámicos. Si lo consideran.

Métodos estáticos
•• Período de Recuperación de Capital, PRC. Su objetivo es determinar el
tiempo en el que se recupera la inversión.
•• Tasa de Rendimiento Contable, TRC. Considera la Utilidad Contable Pro-
medio, UCP y lo compara con la inversión.

Métodos dinámicos
•• Valor Actual Neto (VAN). Donde al beneficio neto actualizado (BNA) es
el valor actual del total del flujo de caja proyectado actualizado a través
de una tasa de descuento (tasa de oportunidad, de rendimiento o de
rentabilidad mínima esperada) se le resta la inversión. Si la diferencia es
positiva el proyecto puede resultar atractivo (VAN= BNA- Inversión).
•• Tasa Interna de Retorno (TIR). es la máxima tasa de descuento que pue-
de tener un proyecto para ser considerado rentable. Para hallar la TIR se
debe encontrar la tasa que permita que el (BNA) sea igual a la inversión
(VAN igual a cero). Al compararla con la Tasa Mínima de Retorno (TMR)
se puede apreciar la calidad de la inversión. La TMR se obtiene: TMR=
% de la inversión a financiar x Tasa de préstamo x(1- tasa de impuesto).
Para considerar la inflación se debe corregir la TMR y se calcula como
sigue:

(1 + TMR)
i = (1 + Tasa de inflación)

Criterios de aceptación de un proyecto

Dependerá del tipo de proyecto, si existe más de una alternativa o si hay más de un
proyecto. Los criterios a considerar en la aceptación de un proyecto son:
•• Período de recuperación: PR < vida útil.
•• Retorno sobre la Inversión (ROI) > 1, mientras mayor sea el ROI un ma-
yor porcentaje del capital se va a recuperar.
•• Tasa de Rendimiento Contable: TRC > TMR.
•• Valor Actual Neto: VAN > 0.
•• Tasa Interna de Rendimiento: TIR > TMR.
•• Relación costo (egresos)– beneficios(ingresos)

369
Sánchez Marchán, Nanci. (2014).Introducción a la Ingeniería Industrial, Material Instruccional de Apoyo. Caracas: UNA

Para calcular la relación Costo – Beneficio se utiliza la siguiente expresión:

B/C = VAI / VAC

Donde:
•• B/C: relación costo-beneficio.
•• VAI: valor actual de los ingresos netos o beneficios.
•• VAC: valor actual de los costos de inversión.
Si B/C > 1, el proyecto es rentable, significa que los beneficios serán mayores que los
costos de inversión.

Si B/C < 1, no es rentable, significa que los beneficios serán iguales o menores que los
costos de inversión.

Calificación del proyecto

Al considerar más de un proyecto los valores obtenidos por cada proyecto en PR, TRC,
VAN y TIR, en cada uno de estos métodos se le asigna un porcentaje, según la importancia
que tengan para la empresa. De manera ponderada se califica cada proyecto y se seleccio-
na el de mayor puntuación. Ver tema de economía y finanzas.

Selección del proyecto

En la selección de un proyecto, la decisión cuantitativa debe ser complementada con


aspectos cualitativos; considerando el rendimiento que genera el proyecto, el riesgo que en-
cierra, la premura, la necesidad de llevarlo a cabo, entre otros. La rentabilidad debe ir acorde
a los criterios cualitativos.

Ejemplo Ilustrativo de la evaluación de un proyecto de inversión


La empresa láctea LECHEKARIN S.A., se dedica a la venta de leche refrigerada.
Hace poco la empresa realizó un análisis de marketing el cual arrojó la posibili-
dad de ingresar al mercado con algunos derivados de la leche como por ejemplo:
yogurt, mantequilla, natilla, quesos, suero, entre otros. De estos productos se es-
cogió para la inversión el yogurt líquido, debido a que no existen muchos compe-
tidores en el mercado. El mercado potencial no cubierto en la zona es de 100.000
hogares. El canal de distribución y venta sería supermercados y abastos. En pro-
medio se estima que cada hogar en este mercado consume 0.8 litros mensuales
de yogurt líquido. El precio sugerido es de 10,00 Bs por litro. La empresa piensa
elevar el precio en un 10% anual por la creciente demanda.
Para iniciar esta línea de producción se requiere invertir para comprar equipos de
refrigeración por 15 mil Bs, línea de envasado por 8 mil Bs, empacadora por 7 mil
Bs, una batidora mezcladora por 10 mil Bs, invertir en una zona de almacén con
túnel de refrigeración 5 mil Bs, en dos vehículos repartidores Bs 300.000 mil c/u.
Los precios de los equipos incluyen la instalación o puestos en planta. (Tipo de
cambio a la fecha de compra 4,30 Bs por dólar).

370
Sánchez Marchán, Nanci. (2014).Introducción a la Ingeniería Industrial, Material Instruccional de Apoyo. Caracas: UNA

Los costos en la fase de operación de la producción son:


• Cada litro de yogurt emplea 1,50 litros de leche, el litro de leche se compra pues-
to en planta en 0.70 Bs.
• Por cada litro de yogurt se emplea 125 grs de fruta, el kilo de fruta en promedio
cuesta 0.5 Bs (fresa, mora, durazno únicos sabores en venta) y aditivos como:
preservantes, saborizantes, entre otros a 0.05 Bs por litro.

Los impuestos en el país son 30% de la utilidad neta, la empresa actualmente tie-
ne como tasa de rentabilidad mínima el 20% anual (tasa de descuento). La depre-
ciación de activos fijos es de 20% anual. La vida útil de los activos es de 5 años,
el valor de venta de todos los activos fijos al finalizar los 5 años es de 20.000 Bs
(la devaluación cambiaria se estima en 0.5% anual). La Tasa Interna de Retorno
(TIR) = 34% y el Valor Actual Neto (VAN)= 246, 051.
Determine según los criterios de rentabilidad, la aceptación o rechazo del pro-
yecto.

Solución:
Se determina la rentabilidad a partir del valor actual neto (VAN) y la tasa interna
de retorno (TIR) aplicadas al flujo de caja económico. En este caso están refleja-
das en el problema y son: TIR = 34% y VAN= 246, 051. (Ver tema de economía
y finanzas)
Sobre esta base, para decir que la inversión es rentable hay que evaluar de ma-
nera comparada los resultados con la tasa de descuento, si el VAN supera a cero,
significa que el flujo de caja es mayor que la inversión en valor actual al 20% de
tasa de interés o de descuento (o retorno mínimo exigido por el inversionista). Asi-
mismo el 34% de TIR supera el 20% de tasa de interés o costo de capital. Como
ambos criterios se satisfacen, se puede decir que la inversión es rentable (flujo de
caja obtenido compensa todos los egresos hechos, sea por inversión o costos),
se acepta el proyecto.

Para verificar la viabilidad de un proyecto se pueden aplicar más de un criterio de acep-


tación.

Actividad propuesta
En el ejemplo ilustrativo, referente a la empresa LECHEKARIN S.A., aplique otros
criterios de aceptación de un proyecto, para comprobar el resultado obtenido con
el criterio de Tasa Interna de Rendimiento (TIR).

La preparación de un proyecto en la fase de preinversión permite emitir un juicio sobre


la conveniencia del proyecto, es decir, si es rentable llevarlo a la fase de operación, si los
beneficios obtenidos son mayores a los costos que se generan.

La gran mayoría de los temas tratados hasta el momento en el Área Operativa de este
texto, necesitan de la Investigación de Operaciones, como una herramienta útil en la solu-
ción de sus problemas. Es por ello que se estudiará esta materia a continuación.

371
Sánchez Marchán, Nanci. (2014).Introducción a la Ingeniería Industrial, Material Instruccional de Apoyo. Caracas: UNA

INVESTIGACIÓN DE OPERACIONES

Objetivo:
Señalar la aplicabilidad de la Investigación de Operaciones dentro del campo de la
Ingeniería Industrial, sobre la base de la importancia de la construcción de mode-
los matemáticos para vislumbrar las posibles soluciones a un problema.

•• Origen de la Investigación de Operaciones.


•• Aplicación del Método Científico en la Investigación de Operaciones.
•• Definición de la Investigación de Operaciones.
•• Ecuaciones.
•• Inecuaciones o Desigualdades.
•• Los modelos matemáticos y su formulación o construcción.
•• usos y beneficios de los Modelos de Investigación de Operaciones.
•• Clasificación de los modelos de Investigación de operaciones.
•• Aplicación de la Investigación de Operaciones en la Ingeniería Indus-
tria.

ORIGEN DE LA INVESTIGACIÓN DE OPERACIONES (IO)


Empezó a utilizarse durante la Segunda Guerra Mundial en Gran Bretaña, donde la
administración militar llamó a un grupo de científicos de distintas áreas del saber para que
estudiaran los problemas tácticos y estratégicos asociados a la defensa del país, situación
que dio origen al nombre Investigación de Operaciones (IO).

La aplicación de la IO se extendió luego a las organizaciones industriales, las cuales se


sintieron motivadas para investigar en este campo; como resultado se lograron importantes
avances tales como el Método Simplex para resolver problemas de programación lineal, de-
sarrollado por George Dantzing en 1947; la Programación Dinámica, por Bellman en 1957; la
Teoría de Colas y la de Inventario por Lieberman 1968, entre otros.

Otro aspecto que impulsó aún más el crecimiento de la investigación de operaciones


fue la aparición de las computadoras que con la capacidad de realizar cálculos aritméticos
más rápido que el ser humano, solucionó de una manera más eficaz problemas complejos
en empresas industriales y administrativas.

373
Sánchez Marchán, Nanci. (2014).Introducción a la Ingeniería Industrial, Material Instruccional de Apoyo. Caracas: UNA

Hoy día la Investigación de Operaciones se aplica a casi todos los problemas, busca el
óptimo resultado en la utilización de recursos escasos a través del método científico, con el
cual se comienza describiendo algún sistema mediante un modelo matemático, que luego se
guía para determinar la mejor forma de operar éste. Se buscan las posibles técnicas a utilizar
para solucionar problemas de producción, administración, organización y control que se pro-
ducen en los diversos sistemas que existen en la naturaleza y los creados por el ser humano
(sistemas organizados, sistemas físicos, económicos, educacionales, de servicio social, etc.)

APLICACIÓN DEL MÉTODO CIENTÍFICO EN LA INVESTIGACIÓN DE


OPERACIONES
Considerando el método científico, se presentan las siguientes fases para el estudio
operativo de la Investigación de Operaciones:

1. Definición del problema. Para iniciar la búsqueda de la solución de un problema


es preciso estar claro, saber correctamente lo que se busca. La definición del pro-
blema debe ser precisa, concisa y con palabras sencillas que no dejen lugar a varias
interpretaciones.

2. Construcción del modelo matemático. Se busca obtener la mejor representación


de una realidad o de una idealidad de un problema.

3. Deducción de una o varias soluciones. Cuando el modelo ha sido bien escogido


o construido, se espera que la solución del problema real sea teórica. Algunas ve-
ces no es posible obtener soluciones exactas para el problema original, entonces se
aceptan soluciones aproximadas.

4. Pruebas del modelo y contraste. Se utiliza información histórica e información de


la fase de construcción del modelo para verificar si los resultados son coincidentes
con los resultados reales del funcionamiento del proceso.

5. Ajustes del modelo y monitoreo de resultados Se debe seguir haciendo compa-


raciones con varias generaciones de resultados para lograr afinar el modelo.

6. Implementación de la solución. Superadas todas las pruebas que proporcionan


seguridad sobre el funcionamiento del instrumento, se capacita a las personas que
tendrán a su cargo la operación del modelo, preparando todas las herramientas de
cómputo para que se elaboren automáticamente los reportes, que permitirán que las
personas indicadas tomen las decisiones pertinentes.

DEFINICIÓN DE LA INVESTIGACIÓN DE OPERACIONES

La Investigación de Operaciones se fundamenta en el método científico y utiliza mo-


delos matemáticos y estadísticos, así como algoritmos con el objeto de realizar un proceso

374
Sánchez Marchán, Nanci. (2014).Introducción a la Ingeniería Industrial, Material Instruccional de Apoyo. Caracas: UNA

de toma de decisiones. Frecuentemente, se realiza el estudio de sistemas complejos reales,


con la finalidad de mejorar u optimizar su funcionamiento.

Sobre la base de la información hasta el momento plasmada se puede definir la Investi-


gación de Operaciones como:

Una metodología utilizada en la toma de decisiones, mediante la cual se logran solucio-


nes óptimas a problemas relacionados con la distribución eficaz y eficiente del dinero,
materia prima, mano de obra, energía, entre otros, con la ayuda de modelos matemáticos
y el enfoque de sistemas.

La IO puede considerarse tanto un arte como una ciencia. Como arte refleja los concep-
tos eficientes y limitados de un modelo matemático definido para una situación dada. Como
ciencia comprende la deducción de métodos de cálculo para resolver los modelos.

En este texto los esfuerzos se van a concentrar en la formulación o construcción de los


modelos matemáticos, por considerar que los mismos constituyen la base para la solución
de los problemas de IO.

Para plantear un modelo matemático es preciso tener conocimientos de cómo diseñar


un sistema de ecuaciones, así como también, conocer las expresiones matemáticas deno-
minadas inecuaciones. A continuación se presenta una explicación breve (repaso de mate-
mática de enseñanza media) de cada uno de estos aspectos.

ECUACIONES E INECUACIONES

ECUACIONES

Las ecuaciones son expresiones algebraicas. Una expresión algebraica es una


combinación de números y símbolos. Por ejemplo: 5x2 + 3x3y3z.

Un término es una combinación de números y símbolos, unidos por operaciones de mul-


tiplicación o división. Por ejemplo: 5x2, 3x3y3z, son los términos de la expresión algebraica
5x2 + 3x3y3z.

Un factor es cada uno de los componentes de un término. Por ejemplo: 5 y x2, son los
factores del término 5x2 de la expresión algebraica 5x2+ 3x3y3z. Elegido un factor, un coefi-
ciente, es lo queda del término. Por ejemplo: 3 es el coeficiente de x3y3z, x3 es el coeficiente
de 3y3z, z es el coeficiente de 3x3y3 y así sucesivamente. Si el coeficiente es un número se
le llama coeficiente numérico.

Se dice que dos términos son similares cuando sólo se diferencian en el coeficiente
numérico.

El grado de un término es la suma de los exponentes de las variables. Por ejemplo: el


grado del término 3x3y3z es 7. El grado de una constante es cero.

375
Sánchez Marchán, Nanci. (2014).Introducción a la Ingeniería Industrial, Material Instruccional de Apoyo. Caracas: UNA

Las ecuaciones son igualdades y es preciso recordar siempre esto. Es necesario distin-
guir entre identidades y ecuaciones. Cuando dos expresiones son iguales para cualesquiera
valores que se pongan en lugar de las letras que figuran en la expresión, es una identidad.
Cuando la igualdad sólo se cumple para determinados valores de la expresión es una ecua-
ción.

Por ejemplo: 2x2 + 5x2 + x2 = 8x2 es una identidad y 2x2 + 3x = 5 es una ecuación.

Clasificación

Las ecuaciones se pueden clasificar de varias formas:

a) Por el número de incógnitas:

Las ecuaciones pueden tener una o más incógnitas. Por ejemplo la ecuación 3x
+ 4 = 10, sólo tiene una incógnita, la ecuación 3x - y = 5, tiene dos y 5xy - 3x2 + z
= 8 tiene tres incógnitas.

Las ecuaciones con una incógnita se pueden imaginar cómo puntos sobre el eje
x. Las de dos incógnitas como curvas en un plano. Las de tres incógnitas como
curvas en un espacio de tres dimensiones.

b) Por el grado de la incógnita:

Las ecuaciones de una incógnita se pueden clasificar por el grado de la incógnita


(el grado es el exponente más alto de la incógnita).

Hay fórmulas generales para resolver las ecuaciones de grado 1 a 4 (pero las fór-
mulas son complicadas y difíciles de recordar para grado mayor que 2). Si no se
puede descomponer la ecuación en factores, cualquier ecuación, sea del grado
que sea, se puede resolver de esta forma:

Sea la ecuación: xn + a1xn-1 + a2xn-2 +... + an = 0

Si x1, x2,..., xn son las soluciones de la ecuación, se cumplen las siguientes ecuaciones:

x1 + x2 + ... + xn = -a1

x1x2+ x1x3+...+x1xn + x2x3+...+ x2xn + ...+ xn-1xn = a2 

x1x2x3 + x1x2x4 + ...+ x1x2xn + x2x3x4 +...+ x2x3xn + ...+ xn-2xn-1xn = -a3

x1x2...xn = (-1)nan

Utilizando estas ecuaciones, se formularía un sistema de ecuaciones que permitiría


obtener las soluciones deseadas.

376
Sánchez Marchán, Nanci. (2014).Introducción a la Ingeniería Industrial, Material Instruccional de Apoyo. Caracas: UNA

Las ecuaciones de pueden diferenciar por el número de términos que posean:

1. Ecuaciones binómicas: son aquellas que poseen dos términos

2. Ecuaciones polinómicas: las que tienen tres o más términos.

Ejemplos Ilustrativos:
1. Calcule dos números de forma que su diferencia sea igual a 43 y el triple del
menor supere en 5 unidades al mayor. Plantee el sistema.
x - y = 43
3y = x + 5
2. Entre Pedro y yo tenemos 12 Bs, si yo le diera 1,7 Bs entonces él tendría el
doble que yo. ¿Cuánto tenemos cada uno? Plantee el sistema.
x + y = 12
2(y - 1,7) = x + 1,7
3. Raquel compra  5 melones y 2  sandías por 13 Bs. Alfredo compra 3 melones y
4 sandías por 12 Bs ¿Cuánto vale un melón? ¿Y una sandía? Plantear el sistema
de ecuaciones.
5x + 2y = 13
3x + 4y = 12
4. El doble de la edad de Sara coincide con la cuarta parte de la edad de su
padre. Dentro de 2 años la edad de Sara será la sexta parte de la de su padre
¿Qué edad tiene cada uno?
2x = y/4
x + 2 = 1/6(y + 2)
5. Vicente  hace un viaje en automóvil de 1.300 km. Divide el recorrido en dos eta-
pas, de forma que en la primera recorre 72 km más que en la segunda. ¿Cuánto
recorrió en cada etapa? Plantee el sistema.
x + y = 1.300
x - y = 72
6. ¿Cuánto miden los lados de un rectángulo si su perímetro es 130 cm, y mide
19 cm más de largo que de ancho?
x + y = 65
x - y = 19

377
Sánchez Marchán, Nanci. (2014).Introducción a la Ingeniería Industrial, Material Instruccional de Apoyo. Caracas: UNA

INECUACIONES O DESIGUALDADES
La región factible en un problema de programación lineal está definida por un juego de
inecuaciones.

Una inecuación es una expresión matemática la cual se caracteriza por tener los sig-
nos de desigualdad. Siendo una expresión algebraica, muestra como resultado un conjunto
en el cual la variable independiente, puede tomar cualquier valor de ese conjunto, cumplien-
do la desigualdad. A este conjunto se le conoce como intervalo.

En matemática, una inecuación es una expresión referida al tamaño u orden relativo


a dos objetos. La notación a < b significa que a es menor que b y la notación a > b quiere
decir que a es mayor que b. Estas relaciones son conocidas con el nombre de inecuaciones
estrictas, contrastando con a ≤ b (a es menor o igual a b) y a ≥ b (a es mayor o igual que b),
llamadas inecuaciones no estrictas.

Si el signo comparativo de la inecuación es el mismo para cualquier valor que tomen


las variables por las que está definida, entonces se hablará de una inecuación “absoluta” o
“incondicional”

Si por el contrario, el signo comparativo es el mismo sólo para ciertos valores de las
variables, pero se invierte o cambia para otros valores, será una inecuación “condicional”.

El signo comparativo de una inecuación no se cambia si a ambos miembros se les


suma o resta el mismo número real, o si se les multiplica o divide por un número positivo; en
cambio, se invierte si a ambos miembros se les multiplica o divide por un número negativo.

Actividad propuesta Nº 1
Identifique y detalle conceptualmente cinco ejemplos de funciones matemáticas
aplicables en actividades empresariales de producción de bienes o servicios.

LOS MODELOS MATEMÁTICOS Y SU FORMULACIÓN O CONSTRUCCIÓN


Un modelo es una representación ideal de un sistema y la forma en la que este opera. El
objetivo es analizar el comportamiento del sistema o bien predecir su comportamiento futuro.
Evidentemente los modelos no son tan complejos como el sistema mismo, de tal manera que
se hacen las suposiciones y restricciones necesarias para representar las porciones más
relevantes de éste.

En atención a la definición de modelo se puede decir que un modelo matemático es:

El producto de la abstracción de un sistema real que elimina las complejidades haciendo


uso de suposiciones pertinentes, así como, de la aplicación de una técnica matemática,
para lograr su representación simbólica.

378
Sánchez Marchán, Nanci. (2014).Introducción a la Ingeniería Industrial, Material Instruccional de Apoyo. Caracas: UNA

Un modelo matemático consta al menos de tres conjuntos básicos de elementos:

1. Variables de decisión y parámetros: Las variables de decisión son incógnitas que


deben ser determinadas a partir de la solución del modelo. Los parámetros repre-
sentan los valores conocidos del sistema o bien que se pueden controlar.

2. Restricciones: Las restricciones son relaciones entre las variables de decisión y


magnitudes que dan sentido a la solución del problema y las acotan a valores facti-
bles. Por ejemplo si una de las variables de decisión representa el número de em-
pleados de un taller, es evidente que el valor de esa variable no puede ser negativo.

3. Función objetivo: La función objetivo es una relación matemática entre las varia-
bles de decisión, parámetros y una magnitud que representa el objetivo o producto
del sistema.

Por ejemplo si el objetivo del sistema es minimizar los costos de operación, la función
objetivo debe expresar la relación entre el costo y las variables de decisión. La solución ópti-
ma se obtiene cuando el valor del costo sea mínimo para un conjunto de valores factibles de
las variables. Es decir hay que determinar las variables x1, x2,..., xn que optimicen el valor de
Z = f(x1, x2,..., xn) sujeto a restricciones de la forma g(x1, x2,..., xn) b. Donde x1, x2,..., xn
son las variables de decisión, Z es la función objetivo y f es una función matemática.

Tanto el objetivo como las restricciones son funciones de dos tipos de variables: las
controlables (de decisión) que pueden ser directamente reguladas por el decisor y el valor
de éstas, debe determinarse como elemento de la decisión. Las incontrolables son aque-
llas que no están bajo el control directo del decisor. Tales como las ganancias por unidad,
costo por unidad, cantidades producidas y tiempos de producción.

Ejemplos ilustrativos de formulación de un modelo matemático:

A continuación se presentan varios ejemplos sencillos de la formulación de un modelo


matemático, dada la situación. Para plantear un modelo matemático sobre la base del enun-
ciado de un problema es conveniente tabular la información disponible en él, de manera que
se pueda visualizar mejor los datos.

Ejemplo Ilustrativo N° 1
Sean x1 y x2 la cantidad a producirse de dos productos 1 y 2, los parámetros son
los costos de producción de ambos productos, 3 Bs para el producto 1 y 5 Bs
para el producto 2. Si el tiempo total de producción está restringido a 500 horas y
el tiempo de producción es de 8 horas por unidad para el producto 1 y de 7 horas
por unidad para el producto 2, entonces se puede representar el modelo como
sigue:

379
Sánchez Marchán, Nanci. (2014).Introducción a la Ingeniería Industrial, Material Instruccional de Apoyo. Caracas: UNA

3 Bs/unidad por X1 unidad + 5 Bs/unidad por X 2 unidad =Bs. Que representan


los costos totales de producción para ese objetivo. Este objetivo, corresponde a
una ecuación con dos términos, dos incógnitas. Los términos son: 3X1 y 5X2 y las
incógnitas son X1 y X2. Matemáticamente se escribe:
Z = 3x1 + 5x2 (Costo total de Producción)

Como se expresa en el enunciado el tiempo de producción está restringido a


500h y para cada unidad de cada producto se dispone respectivamente de 8 y 7
hrs, se obtendrá la siguiente ecuación:
8hrs/unidad por X1 unidades + 7 hrs/ unidad, tiene que ser menor o igual a 500
hrs. Esto expresado en forma matemática se escribe:
8x1 + 7x2 500

Función Objetivo: minimizar el costo de producción


Minimizar Z=3x1 + 5x2
Con la condición, limitante o restricción del número de horas disponibles:
8x1 + 7x2 500

Otra condición que se debe establecer es que los valores correspondientes a X1


y X2, deben ser positivos, es decir:
X1 y X2 ≥ 0. Estas variables representan unidades y deben ser positivas.
Por tanto el modelo matemático correspondiente es el siguiente:
Minimizar Z=3x1 + 5x2
8x1 + 7x2 500
X1 y X2 ≥ 0

Ejemplo Ilustrativo N° 2
Una compañía fabrica dos productos, A y B. La ganancia para el A es de 1000
Bs por unidad y para el B es de 1500 Bs por unidad. Se requieren 20 horas para
fabricar una sola unidad de A y 30 horas para fabricar una sola unidad de B.
El tiempo total de manufactura disponible por año es de 1200 horas. La cantidad
producida de A no debe ser mayor que 10 unidades por año y la cantidad de B no
mayor que 30 unidades por año.
Si se desea maximizar las ganancias por año, el modelo de la situación descrita
será:
Maximizar 1000 X A + 1500 XB

380
Sánchez Marchán, Nanci. (2014).Introducción a la Ingeniería Industrial, Material Instruccional de Apoyo. Caracas: UNA

Sujeto a las restricciones:


20 XA+ 30 XB<1200
XA <10
XB <30

Donde XA y XB son las variables controlables, es decir el número de unidades


producidas por año.
Ejemplo Ilustrativo N° 3
La industria metalúrgica “REX” fabrica dos tipos de productos “A” y “B” para abas-
tecer el mercado nacional. Cada producto debe pasar por dos máquinas para su
completa elaboración.
La máquina (1) tarda para producir el producto “A” una hora por cada unidad y
para el “B” 2 horas/unidad.
La máquina (2) tarda para producir el producto “A” tres horas por unidad y para
el “B” una hora por unidad. Las ganancias para “A” son de 400 Bs. y para el “B”
son de 300 Bs.
El tiempo disponible en horas-semanal es de 100 tanto para la máquina (1) como
para la (2) ¿Qué cantidades de cada producto se deben producir para maximizar
las ganancias? Construir el modelo correspondiente.
La información disponible es:
Sean X1: la cantidad del producto “A” a fabricar.
X2: la cantidad del producto “B” a fabricar.
Z: las ganancias totales.
La función objetivo será maximizar las ganancias obtenidas y se expresa lineal-
mente de la forma siguiente:
Máx Z = 400 X1 + 300 X2
Sujeta a las condiciones de disponibilidades de las máquinas en el tiempo sema-
nal.
X1 + 2 X2< 100 hrs/sem.
3 X1 + X2< 100 hrs/sem.
X1, X2 >0 (No negatividad de los productos)
El modelo lineal correspondiente es:
Máx Z = 400 X1 + 300 X2
S.A 1) X1 + 2 X2< 100
2) 3 X1 + X2< 100
3) X1, X2 >0
Donde, Z = 400 X1 + 300 X2 es una función lineal, Z es la variable incontrolable
(dependiente) y X1, X2 son las variables controlables (independientes).

381
Sánchez Marchán, Nanci. (2014).Introducción a la Ingeniería Industrial, Material Instruccional de Apoyo. Caracas: UNA

Estos tipos de modelos pueden ser resueltos utilizando la programación lineal (P.L.) que
permite asignar una cantidad fija de recursos para satisfacer varias demandas, de tal forma,
que mientras se optimiza algún objetivo se satisfacen otras condiciones definidas.

LOS MODELOS MATEMÁTICOS Y FORMULACIÓN O CONSTRUCCIÓN

USOS Y BENEFICIOS DE LOS MODELOS DE INVESTIGACIÓN DE OPERACIONES

En general, los modelos matemáticos se usan para tomar decisiones. Las cuales se
pueden dividir en estratégicas y operacionales.

Las estratégicas: generalmente se toman decisiones de una sola vez, tienen efecto
sobre intervalos relativamente largos.

Ejemplo Ilustrativo
• Reemplazar un sistema existente por uno nuevo.
• Reordenar inventarios a intervalos regulares, en lugar de cada vez que el nivel
caiga por debajo de una cantidad especificada.

Es probable que las decisiones estratégicas tengan un impacto importante en la orga-


nización por lo tanto hay que hacer esfuerzos para que el modelo sea válido y los datos lo
más exactos posible.

Las operacionales: afectan procesos en curso sobre períodos más cortos.

Ejemplo Ilustrativo
• Determinar el plan de producción mensual más óptimo.
• Decidir cuál el plan de embarque más efectivo en costos, desde las plantas
hasta los mercados.

A diferencia de las decisiones estratégicas, las operacionales se usan repetidamente,


así que es necesario identificar los procedimientos de solución más eficientes, ya que pue-
den proporcionar ahorros significativos en costos y en tiempo.

Los modelos en sí, proporcionan los siguientes beneficios:


•• Un método para determinar la mejor forma de lograr un objetivo.
•• Un método para evaluar el impacto de un cambio o un nuevo sistema, sin
el costo y el tiempo de llevarlo a cabo primero.
•• Una forma de evaluar la “fortaleza” de la solución óptima.
•• Un procedimiento para lograr un objetivo que beneficie a la organización
en forma global.

382
Sánchez Marchán, Nanci. (2014).Introducción a la Ingeniería Industrial, Material Instruccional de Apoyo. Caracas: UNA

CLASIFICACIÓN DE LOS MODELOS DE INVESTIGACIÓN DE OPERACIONES


Los modelos se pueden clasificar en abstractos y matemáticos.

Según su grado de abstracción pueden ser:


•• Abstractos (no físicos).
•• Concretos (físicos).

Los modelos abstractos no se abordarán en este texto, sólo los modelos matemáticos.

Los modelos matemáticos se clasifican en:

Figura 70. Modelos matemáticos de la Investigación de Operaciones.

No habría ventaja alguna de utilizar modelos si estos no simplificaran la situación


real. En muchos casos se puede utilizar modelos matemáticos que, mediante le-
tras, números y operaciones, representan variables, magnitudes y sus relaciones.

383
Sánchez Marchán, Nanci. (2014).Introducción a la Ingeniería Industrial, Material Instruccional de Apoyo. Caracas: UNA

APLICACIÓN DE LA INVESTIGACIÓN DE OPERACIONES EN LA INGENIERÍA


INDUSTRIAL
La Investigación de Operaciones se ha aplicado en los negocios, la industria, la milicia,
el gobierno, los hospitales, entre otras. La gama de aplicaciones es extraordinariamente
amplia.

Se muestra seguidamente una tabla que contiene los modelos de Investigación de Ope-
raciones, así como, los métodos de solución, características y su aplicación en las Áreas de
Ingeniería Industrial.

Tabla 43. Técnicas, métodos de solución, características y aplicación


de la (IO) en la Ingeniería Industrial
Procedimientos utilizados en la Investigación de Operaciones
Modelos Métodos de Características Aplicación en la
solución Ingeniería Industrial
Gráfico Se tienen 2 variables Microeconomía y la
administración de
(dos productos)
empresas, ya sea para
aumentar al máximo
los ingresos o reducir al
mínimo los costos de un
sistema de producción
tales como: mezcla de
Programación lineal alimentos, la gestión de
formulados a través de inventarios, la cartera y la
ecuaciones lineales, optimi- gestión de las finanzas,
zando la función objetivo, la asignación de recursos
también lineal humanos y recursos de
máquinas, la planifica-
ción de campañas de
publicidad, etc.
Simplex Se presentan más de La misma aplicación del
2 variables (más de método anterior.
dos productos)
Transporte Determinar el envió En el área de producción
de material desde para la distribución de
varios orígenes hasta materias primas, produc-
varios destinos al tos en proceso, termina-
costo de transporte dos , entre otros
mínimo.
Asignación Asignación de re- En el área de producción
cursos en diferentes para la elaboración de
máquinas en un deter- varios productos en dos
minado tiempo. o más máquinas.

384
Sánchez Marchán, Nanci. (2014).Introducción a la Ingeniería Industrial, Material Instruccional de Apoyo. Caracas: UNA

Programación lineal Simplex Se tienen más de 2 La misma aplicación del


entera: Para determinar variables (más de dos método gráfico
Ramificar y acotar
cantidades que no pueden productos)
fraccionarse
Programación no lineal • Optimización con- Es requerida por em- En empresas de cons-
PNL) resolución de un vexa. presas que generan trucción de tanques
sistema de igualdades y productos que necesi- de cualquier material y
• Formulaciones es-
desigualdades sujetas a un tan de varios procesos, forma, entre otras.
peciales de proble-
conjunto de restricciones es decir, se generan
mas de programa-
sobre un conjunto de varia- modelos no lineales
ción lineal.
bles reales desconocidas, (x3, x4 , x5 …)
con una función objetivo a • Ramificación y
maximizar, cuando alguna poda.
de las restricciones o la fun-
ción objetivo no son lineales
Programación dinámica: Regresión: Es práctico cuando se Igual aplicación al méto-
quiere obtener el ópti- do de transporte
modelo para reducir el • Sucesión Fibonacci.
mo teniendo un punto
tiempo de ejecución de un
• Coeficientes bino- de origen y diversos
algoritmo. Se utiliza para op-
miales. destinos, logrando
timizar problemas complejos
determinar el mejor re-
que pueden ser discretiza-
corrido en kilometraje.
dos y secuencializados.

Teoría de cola: El estudio Distribución de pro- Se establece la lon- Lo utilizan empresas


matemático del comporta- babilidad. Las más gitud de una cola, como bancos, supermer-
miento de líneas de espera, usadas: determinando la tasa cados, metro, entre otras,
que se presenta cuando los de salida y la tasa de en atención al cliente.
• Distribución expo-
clientes llegan a un lugar llegada.
nencial (markovia-
demandando un servicio, el
na).
cual tiene una cierta capaci-
dad de atención. • Distribución dege-
nerada.
• Distribución Erlang,
otras.
Simulación: el modelo Simulación Monte Practica en procesos En todas las áreas de la
usado para la simulación Carlo, Software SCA- donde existe incerti- Ingeniería Industrial, para
seria teórico, conceptual o DA, entre otros. dumbre y riesgos. simular procesos como
sistémico. producción de textiles,
alimentos, juguetes, cons-
trucción de infraestructu-
ras por medio de maque-
tas, entre otros.
Flujo de redes: grafo dirigi- • Minimización de Determina el flujo de Diseño de redes de trans-
do, donde la fuente es quien redes. redes inalámbricas porte, en distribución de
produce o inicia el traspaso en procesos de esta productos, para minimizar
• Ruta más corta.
de algún material o produc- índole. sus costos.
to por los arcos, tomando • Flujo máximo.
en cuenta la ley de corrien-
tes de Kirchoff, donde, la • flujo del costo mí-
suma de flujos entrantes a nimo.
un vértice debe ser igual a
la suma de flujos saliendo
del vértice.

385
Sánchez Marchán, Nanci. (2014).Introducción a la Ingeniería Industrial, Material Instruccional de Apoyo. Caracas: UNA

Procesos markovianos: Estocásticos Permite vislumbrar Economía y Finanzas.


serie de eventos, en los discrepancias entre
cuales la probabilidad de variables aleatorias de
que ocurra uno de ellos de- un proceso, ya sea de
pende del evento inmediato un momento a otro, de
anterior .Proceso estocásti- un estado a otro, en un
co discreto que cumple con período determinado.
la propiedad de Márkov
Teoría de juegos: área Suma cero Determinar estrategias Igual al anterior.
de la matemática aplicada a seguir entre dos com-
que utiliza modelos para petidores de un mismo
estudiar interacciones en ramo.
estructuras formalizadas
de incentivos (los llamados
juegos) y llevar a cabo pro-
cesos de decisión.
Secuenciación: permite Menor tiempo Si el número de máqui- Planeación y control de la
encontrar una secuencia nas es superior a tres producción, ampliaciones
de (n) tareas en un ambien- se puede utilizar la Ing. de fábrica, entre otros.
te de (m) máquinas, para de Métodos o PER y
minimizar las tareas que se CPM.
entregan con retraso y así
alcanzar los objetivos del
sistema.

Debe recordarse que, un modelo es una aproximación de un sistema real


por consiguiente todas las variables pueden no estar incluidas en él.

Los modelos más importantes para la investigación de operaciones, son los modelos
matemáticos que emplean un conjunto de símbolos y funciones para representar las varia-
bles de decisión y las relaciones que describen el comportamiento del sistema. El uso de las
matemáticas para representar el modelo, el cual es una aproximación de la realidad, permite
aprovechar las computadoras de alta velocidad y técnicas de solución con la matemática
avanzada.

En la actualidad es importante que una empresa considere la Investigación de Opera-


ciones para fundamentar la comprensión, modelación y resolución óptima de los problemas
relacionados con procesos tales como planificación, diseño, toma de decisiones, desarrollo
de softwares, entre otros; logrando así eficacia y eficiencia en sus operaciones, para aumen-
tar sus utilidades y posicionarse mejor en el mercado.

RESUMEN DE LA UNIDAD 2
Área operativa

En esta unidad se desarrolla el área operativa de la Ingeniería Industrial, en la cual


se está en contacto directo con el personal que hace y genera la actividad principal de la

386
Sánchez Marchán, Nanci. (2014).Introducción a la Ingeniería Industrial, Material Instruccional de Apoyo. Caracas: UNA

empresa (producción), también se verifica el ambiente de trabajo de este personal, además


se conocen y analizan las fallas en los equipos para prevenirlas, minimizarlas o eliminarlas;
logrando optimizar los procesos de producción. Se contemplan en esta área los siguientes
temas:

La empresa

La empresa está compuesta por una serie de elementos, los cuales tienen su impor-
tancia y connotación: empresario, factor humano, bienes de capital, recursos económicos
y financieros. Se compromete a producir para el consumo, a partir de los elementos que la
componen y en particular de los factores de la producción.

El tamaño de la empresa varía en atención al número de trabajadores que posee.

Los países deben considerar la realidad económica, social y demográfica con la cual
coexiste la empresa diariamente, para lograr su desarrollo y productividad.

Producción y productividad

La producción se ocupa de la obtención de bienes y servicios, es decir, de su diseño,


control del personal, los materiales, los equipos, el capital y la información para el logro de
los objetivos planteados.

La productividad, es el cociente que se obtiene al dividir la producción por uno de los


factores de producción. De esta forma es posible hablar de la productividad parcial o total.

Para los empresarios productividad es sinónimo de rendimiento y simplifican la defi-


nición diciendo que un buen sistema de producción, utilizando una cantidad adecuada de
recursos debe obtener el máximo de productos en el menor tiempo posible.

Ingeniería de Métodos

Son procedimientos sistemáticos para someter a todas las operaciones de producción


a un análisis minucioso, con miras a introducir mejoras que faciliten la realización del trabajo
y que permita que éste sea hecho en el menor tiempo, lo que conlleva la realización de un
estudio de movimiento y tiempo.

El estudio de métodos abarca la formulación y selección de los mejores métodos, proce-


sos, herramientas, equipos diversos y características necesarias para generar un producto
o servicio.

Uno de los instrumentos de trabajo más importante en la Ingeniería de Métodos es el


diagrama de proceso o representación gráfica relativa a un proceso industrial o administrativo.

387
Sánchez Marchán, Nanci. (2014).Introducción a la Ingeniería Industrial, Material Instruccional de Apoyo. Caracas: UNA

Calidad

Se introduce en EUA a principios del siglo XX y puede definirse como el conjunto de


técnicas y actividades de carácter operativo utilizadas para verificar los requisitos relativos a
la calidad del producto.

En la obtención de la calidad se hizo insuficiente aplicar sólo una inspección, control de


calidad o asegurar la calidad. Razón por la cual hoy día se habla de la calidad en la atención
al cliente, camino que conduce hacia la calidad total.

Mejorar la calidad de productos y procesos requiere de la aplicación de diferentes herra-


mientas, así como, de la participación de toda la organización. Situación que en la mayoría
de los casos, conduce a cambios en los hábitos de trabajo e incluso en la misma estructura
de la organización.

Seguridad e Higiene Industrial

La Seguridad Industrial considera los principios, leyes, normas y mecanismos de pre-


vención de los riesgos inherentes al sitio laboral, que pueden causar un accidente ocupacio-
nal, con daños severos a la vida de los trabajadores o a las instalaciones o equipos de las
empresas.

Los puntos a considerar dentro de la Seguridad Industrial son los siguientes:


•• Accidentes de trabajo y enfermedad profesional.
•• Riesgos.
•• Causas y costos de los accidentes.
•• Salud ocupacional:
•• Ergonomía
•• Biomecánica

La Higiene Industrial se encarga de la investigación, evaluación y control de los agentes


contaminantes de origen laboral, los cuales pueden deteriorar la salud de los trabajadores y
causarles una enfermedad de trabajo, en el ejercicio o con el motivo de su actividad laboral.

Ergonomía

La Ergonomía ayuda a adaptar el trabajo a la persona, dejando atrás el viejo concepto


de forzar a la persona a adaptarse al trabajo. Adaptar la labor al trabajador puede ayudar a
reducir el estrés y eliminar potenciales problemas físicos.

La lógica que utiliza la ergonomía se basa en el principio de que las personas son más
importantes que los objetos o que los procesos productivos; por tanto, en aquellos casos en
los que se plantee cualquier tipo de conflicto de intereses entre personas y cosas, deben
prevalecer los de las personas.

388
Sánchez Marchán, Nanci. (2014).Introducción a la Ingeniería Industrial, Material Instruccional de Apoyo. Caracas: UNA

La Ergonomía se puede aplicar en:


•• Diseño de máquinas
•• Diseño de herramientas
•• Diseño del puesto de trabajo
•• Posición de trabajo
•• Organización de la empresa

Mantenimiento Industrial

Es un proceso compuesto por una serie de operaciones técnicas que son de aplicación
directa, estructural, de control económico, el cual debe ser capaz de prolongar el ciclo vital
de la maquinaria, las instalaciones y los edificios para que el valor de las inversiones se man-
tenga activo durante el tiempo de amortización e incluso después.

Objetivos del mantenimiento son:


•• Mitigar y combatir el desgaste y la destrucción tanto de la infraestructura
como de las instalaciones.
•• Restaurar una operación y funcionamiento original.
•• Garantizar una operación y funcionamiento continuo, confiable y seguro
sin interrumpir los servicios que se prestan.

Preparación de proyectos de inversión

Un proyecto de inversión se genera con el propósito de resolver una necesidad huma-


na, es indispensable entender que tal acción debe tomarse con una base de decisión que
justifique su aplicabilidad. Cuando se evalúa un proyecto se genera información, permitiendo
emitir una crítica sobre la conveniencia y confiabilidad de la estimación preliminar del bene-
ficio que genera éste.

Existen dos motivos para que la empresa realice las inversiones:


•• Por excedente de recursos.
•• Por necesidades específicas.

Inventarios

En muchos países las empresas siguen manteniendo stock de reserva o inventarios


tal es el caso de nuestro país. El inventario se compone de actividades y técnicas utilizadas
para mantener una cantidad de artículos (materiales, materias primas, producto en proceso
y producto terminado) en el nivel deseado; logrando que ni el costo, ni la probabilidad de
faltante sean de una magnitud significativa.

389
Sánchez Marchán, Nanci. (2014).Introducción a la Ingeniería Industrial, Material Instruccional de Apoyo. Caracas: UNA

Los japoneses fueron los primeros en hacer más eficiente este proceso en la industria
manufacturera, generando una técnica que denominaron Sistema KAMBAN, un simple pa-
pel o tarjeta que se anexa a una red de hilo solicitando las partes que una operación requiere
de la anterior.

Entre los principales objetivos del inventario se encuentran:


•• Disminuir el costo de compra.
•• Reducir los gastos de intereses.
•• Aumentar la disponibilidad de fondos internos.
•• Lograr menores costos de operación.
•• Disminuir el costo unitario de producción.
•• Aumentar la confiabilidad en la entrega por parte de la producción.

Localización y distribución en plantas

En la fase de operación, el proceso de ubicación del lugar adecuado para instalar una
planta industrial requiere el análisis de diversos factores desde los puntos de vista eco-
nómico, social, tecnológico y del mercado, entre otros. Realizar una localización industrial
significa ir de una localización macro (en una región de un país) a una localización micro (la
comunidad de esa región).

El problema de la localización se puede abordar en dos (2) etapas:

En la primera se decide la zona general en la que se instalará la empresa. En la se-


gunda se elige el punto preciso, considerando los problemas de detalle, costo de terrenos,
facilidades administrativas, entre otras.

La localización de la planta se debe planear cuidadosamente, ya que es costoso cam-


biarla después.

Investigación de Operaciones

Esta área permite realizar investigaciones sobre las operaciones que realiza la empresa,
por ello se aplica a problemas que se refieren a la conducción y coordinación de operacio-
nes o actividades en ella. La Investigación de Operaciones se ha aplicado en los negocios,
la industria, la milicia, el gobierno, los hospitales, entre otros. Así, la gama de aplicaciones
es extraordinariamente amplia. Muchas industrias, incluyendo la aérea y de proyectiles, la
automotriz, la de comunicaciones, computación, energía eléctrica, electrónica, alimenticia,
metalúrgica, minera, del papel, del petróleo y del transporte, han empleado la Investigación
de Operaciones. Las instituciones financieras, gubernamentales y de salud están incluyendo
cada vez más estas técnicas.

Para formular un problema lineal hay que seguir los siguientes pasos:
1. Identificar las variables de decisión.
2. Identificar la función objetiva.
3. Identificar las restricciones.

390
Sánchez Marchán, Nanci. (2014).Introducción a la Ingeniería Industrial, Material Instruccional de Apoyo. Caracas: UNA

Después de formular el problema hay que asegurarse de que es lineal para poder re-
solverlo, aplicando un algoritmo conocido. En La actualidad se desarrollan programas de
computadoras que permiten resolver la casi totalidad de los problemas que involucran la
investigación de operaciones siendo de gran utilidad para problemas grandes y complejos.

Seguidamente se presenta la autoevaluación de la unidad 2, en la que encontrará un


conjunto de ejercicios, cuyas respuestas se presentan al final del texto. También se puede
observar una serie de ejercicios propuestos, para que el estudiante se ejercite más.

AUTOEVALUACIÓN DE LA UNIDAD 2
Una vez estudiados los contenidos concernientes al área operativa, es recomen-
dable resolver los ejercicios correspondientes a la autoevaluación, sobre la base
de los ejemplos ilustrativos. Resueltos estos ejercicios podrá comprobar sus re-
sultados con las respuestas aportadas por la autora al final del texto. En caso
de no realizar los ejercicios correctamente, repase de nuevo los contenidos e
inténtelo una vez más hasta lograr su cometido. Si aún persisten las fallas en sus
respuestas, consulte a su asesor, para que le ayude a despejar sus dudas.

ÁREA OPERATIVA

LA EMPRESA
Ejercicio Nº 1.

Seleccione la respuesta correcta en cada uno de los siguientes ejercicios:

Ejercicio 1.1: ¿Cuántos trabajadores pueden tener una empresa mediana?

a. 50 a 250.
b. 100 a 300.
c. 300 a 200.
d. 80 a 450.
e. 600 a 1500.

Ejercicio 1.2. Se define como la unidad económica o social en la cual se reúne capital,
entre otros, para lograr un objetivo común.
a. Empresa.
b. Costos.
c. Empresa chica.
d. Empresa grande.
e. Contabilidad.

391
Sánchez Marchán, Nanci. (2014).Introducción a la Ingeniería Industrial, Material Instruccional de Apoyo. Caracas: UNA

Ejercicio 1.3. Esta empresa puede tener más de 20 trabajadores.


a. Grande.
b. Mediana.
c. Pequeña.
d. Microempresa.

Ejercicio 1.4. La empresa_____ puede tener ________ trabajadores.


a. Pequeña-300.
b. Grande-500.
c. 25-Pequeña.
d. 1500.

Ejercicio 1.5. A qué sociedad pertenecen las siguientes siglas: SRC


a. Sociedad Anónima.
b. Sociedad de Responsabilidad Limitada.
c. Sociedad Comanditaria.
d. Sociedad Anónima Laboral.
e. Ninguna de las anteriores.

Ejercicio Nº 2
Caso: aplicación de estrategias operacionales en el logro de la eficacia y
eficiencia, en una empresa automotriz.
Una empresa automotriz, previo análisis, determinó que los trámites para la do-
cumentación en la compra de vehículos podrían tener un efecto directo sobre los
precios de venta del producto, e inició un trabajo para eliminar en lo posible parte
del papeleo y la considerable duplicación del esfuerzo del personal en estas ta-
reas burocráticas.
Después de seis meses la empresa había logrado los siguientes resultados:
Los gastos administrativos se redujeron en un 50%.
Una reducción en la plantilla administrativa en la misma proporción.
Un ahorro considerable de material de papelería.

Analice la situación planteada.

Ejercicio Nº 3.

Describa los pasos a seguir en la toma de decisiones en una empresa.

392
Sánchez Marchán, Nanci. (2014).Introducción a la Ingeniería Industrial, Material Instruccional de Apoyo. Caracas: UNA

Ejercicio Nº 4

Realice una caracterización de los organigramas horizontal y circular.

Ejercicio Nº 5

Represente gráficamente las estructuras jerárquica o lineal y departamentalización por


producto.

PRODUCCIÓN

Ejercicio Nº 6

La empresa NANEL S.A., se dedica a la extracción de mineral de hierro en el territorio


guayanés. Seleccione de acuerdo con la afinidad, el sector de la producción, donde se en-
cuentra NANEL:

a.Primario.
b.Secundario.
c.Terciario.

Ejercicio Nº 7

¿Qué métodos usaría para pronosticar lo siguiente?

a. Ventas del próximo año de televisores de alta definición.


b. Número de suscriptores a un periódico matutino.
c. Producción de arroz para el próximo año.

Ejercicio Nº 8

La demanda de un artículo se da en la tabla siguiente:

t dt t dt t dt t dt
1 239 4 345 7 227 10 352
2 325 5 254 8 221 11 241
3 268 6 216 9 208 12 420

Calcule el pronóstico utilizando promedio móvil simple.

393
Sánchez Marchán, Nanci. (2014).Introducción a la Ingeniería Industrial, Material Instruccional de Apoyo. Caracas: UNA

INVENTARIOS
Ejercicio Nº 9
Mencione los objetivos necesarios para alcanzar las metas del inventario.

Ejercicio Nº 10
Una empresa puede generar diferentes tipos de inventario. Describa los más importantes.

PRODUCTIVIDAD

Ejercicio Nº 11

En una empresa fabricante de cuadernos, se producen 2.000.000 de unidades al mes.


Para lograr esta producción se utilizan los siguientes insumos:

Insumos Cantidad
Número de trabajadores 20
Horas trabajadas promedio 170 horas al mes
Papel 350.000 resmas al mes
Energía 300.000 kwh /mes
Tinta 100 unidades/ mes
Horas- maquina trabajadas en promedio 190 horas al mes
Número de máquinas 10
Cartulina 250 unidades

Calcule el indicador de productividad para las horas-máquina.


Recuerde que el indicador de productividad es la relación entre producción e insumo y
se puede calcular de manera parcial (para cada uno de los insumos) o total (todos los insu-
mos).

Ejercicio Nº 12
En la siguiente tabla se presenta la información simplificada, de una empresa manufac-
turera (X):

Factor Cantidad
Número de trabajadores 100
Horas trabajadas promedio 155 horas al mes
Unidades producidas por mes 1.000.000
Unidades de materia prima por mes 150.000

Calcule el indicador de productividad para las horas–hombre trabajadas.

394
Sánchez Marchán, Nanci. (2014).Introducción a la Ingeniería Industrial, Material Instruccional de Apoyo. Caracas: UNA

Ejercicio Nº 13
Un fabricante elabora el mismo número de unidades de un producto (X) en las plantas
A, B y C, ubicadas en diferentes regiones del país. La planta A requiere mayor número de
operarios para alcanzar su producción. ¿Considera usted que la productividad de la planta A
es igual a la productividad de las plantas B y C?

ESTUDIO DEL TRABAJO: MÉTODOS TIEMPOS Y MOVIMIENTOS


Ejercicio Nº 14
Represente los símbolos utilizados para elaborar un diagrama de proceso e indique el
significado y definición de cada uno de ellos.

Ejercicio Nº 15
Elabore el diagrama bimanual actual y propuesto del proceso necesario para cortar
tubos de vidrio en trozos, cortados con la ayuda de una plantilla.

CALIDAD EN LA EMPRESA
Ejercicio Nº 16
¿Por qué la calidad es importante en el ámbito de la empresa?

Ejercicio Nº 17
¿Qué significa la siguiente figura?

Excelencia

Conformidad con las Satisfacer expectativas del


especificaciones cliente
(Eficiencia) (Eficacia)
Perspectiva Interna Valor – Precio
Perspectiva de Mercado

Perspectiva Global
(Eficiencia y Eficacia)

395
Sánchez Marchán, Nanci. (2014).Introducción a la Ingeniería Industrial, Material Instruccional de Apoyo. Caracas: UNA

Ejercicio Nº 18

El grafico de control X - R, es llevado a cabo en una resistencia tensora de un cierto


hilado. El tamaño de la muestra es 5, los valores de X y R están computados para cada
grupo y después de 25 muestras se tiene que la suma de las medias muestrales es de 514,8
y la suma de los rangos de todas las muestras es de 120. Calcular los límites de control para
la gráfica X - R.

Ejercicio Nº 19

Elabore un diagrama de proceso donde refleje las actividades que usted realiza como
estudiante regular de la UNA.

Ejercicio Nº 20

Complete la tabla mostrada, con la orientación de los diferentes enfoques de la gestión


de la calidad:

Enfoque Orientación del enfoque


Inspección
Control
Aseguramiento de la calidad
Gestión de la calidad total

SEGURIDAD E HIGIENE EN EL TRABAJO


Ejercicio Nº 21

Marcos Salazar es un ingeniero químico y fue contratado para trabajar en una planta
industrial donde se emplean gases venenosos. ¿Qué medidas mínimas debe exigir Marcos
para resguardar su salud?

Ejercicio Nº 22

Determinar los agentes contaminantes que pueden entrar al organismo por las siguien-
tes vías:
•• Auditiva.
•• Respiratoria.
•• Cutánea.
•• Visual.
•• Digestiva.

396
Sánchez Marchán, Nanci. (2014).Introducción a la Ingeniería Industrial, Material Instruccional de Apoyo. Caracas: UNA

Ejercicio Nº 23

Seleccione la respuesta correcta ¿Cuál de los siguientes enunciados relacionados con


la Higiene y Seguridad Industrial es falso? Justifique su respuesta.

a. Los riesgos deben disminuirse aunque los gastos implicados en tal acción no
sean económicamente aceptables para la empresa.

b. Son departamentos encargados en la formación de recursos humanos para pre-


venir riesgos de accidentes empleando técnicas de motivación, educación y pro-
tección en las organizaciones.

c. Resulta más económico prevenir con antelación los accidentes y siniestros que
puedan darse, que pagar los gastos derivados de la ocurrencia de los mismos.

d. La silicosis se produce frecuentemente en trabajadores de fábricas de cemento


que no usan mascarillas protectoras debidamente diseñadas.

ERGONOMÍA
Ejercicio Nº 24

Todos los años gran cantidad de personas sufren lesiones por el uso de la computado-
ra que pueden parecer no tan serias, pero pueden llegar a ser complicaciones de cuidado
e incluso incapacitantes. Si se hace uso de la computadora con frecuencia o por períodos
prolongados. ¿Cuáles son las precauciones a considerar?

Ejercicio Nº 25

Considera que los diseños de puestos de trabajo mostrados a continuación son correc-
tos:

MANTENIMIENTO INDUSTRIAL DE MAQUINARIA Y EQUIPOS

Ejercicio Nº 26

Considere que la fiabilidad de una máquina o equipo puede ser igual al 100%.

397
Sánchez Marchán, Nanci. (2014).Introducción a la Ingeniería Industrial, Material Instruccional de Apoyo. Caracas: UNA

Ejercicio Nº 27

¿Qué tipo de mantenimiento es necesario aplicar cuando un equipo que proporciona


servicio vital ha dejado de funcionar, por cualquier causa y es necesario actuar bajo un plan
contingente? Justifique su respuesta.

Ejercicio Nº 28

La empresa NASAN C.A, fabrica dos tipos de productos “A” y “B” para abastecer el mer-
cado nacional. Cada producto debe pasar por dos máquinas para su completa elaboración.
La máquina 1 presenta algunas fallas por falta de mantenimiento y se decidió interrumpir el
proceso por dos días para su reparación. ¿Cuáles son los costos involucrados con la parali-
zación de la máquina? ¿Cómo se podría disminuir o evitar este tipo de incidente?

PREPARACIÓN DE PROYECTOS DE INVERSIÓN


Ejercicio Nº 29

Sobre la base del título y evaluación del proyecto planteado, escriba una posible conclu-
sión al respecto.

TÍTULO: DIMENSIÓN, ORÍGENES Y CARÁCTER DEL SECTOR DE LA ECONOMÍA


INFORMAL EN VENEZUELA

Evaluación: este estudio examina las tendencias y actual dimensión del sector informal
en Venezuela; las razones para el crecimiento de la informalidad; el entorno legal y norma-
tivo que enfrentan los informales y el enfoque hacia la economía informal adoptado por los
funcionarios.

Ejercicio Nº 30

Para aceptar un proyecto se requiere la consideración de ciertos criterios. Detalle los


más utilizados.

LOCALIZACIÓN Y DISTRIBUCIÓN EN PLANTAS


Ejercicio Nº 31

Elabore un plano sencillo de la distribución en planta de una panadería.

398
Sánchez Marchán, Nanci. (2014).Introducción a la Ingeniería Industrial, Material Instruccional de Apoyo. Caracas: UNA

Ejercicio Nº 32

Mencione los factores básicos que gobiernan la evaluación para la localización de fá-
bricas.

INVESTIGACIÓN DE OPERACIONES
Ejercicio N° 33

La empresa “La Casa de la Ventana” ubicada en Caracas, está dedicada a la elabo-


ración de artículos de vidrio de alta calidad (puertas y ventanas) los cuales se hacen en 3
plantas diferentes.
•• Planta 1. Se fabrican molduras y marcos de aluminio.
•• Planta 2. Se fabrican molduras y marcos en madera.
•• Planta 3. Se hace y se ensambla el vidrio.

Se tiene un programa de cambio en la producción y se propone incursionar con 2 nue-


vos productos.
•• Producto 1. Puerta de vidrio con marco en aluminio.
•• Producto 2. Ventana de vidrio con marco en madera.

Según el departamento de comercialización toda la producción de éstos puede colocar-


se en el mercado.

En la siguiente tabla se presenta la información recogida de la situación planteada:

Planta Tiempo de Producción por Tiempo de Producción


Lote (horas) Disponible a la Semana (horas)
Producto 1 Producto 2
1 1 0 4
2 0 2 12
3 3 2 18
Ganancia por 6.000 10.000
Lote (Bs)
Se fabrican lotes de 20 productos por semana.

a) Formule dos (2) preguntas alusivas a la situación expuesta.


b) Defina las variables a estudiar.
c) ¿Cuál es la función objetivo?

399
Sánchez Marchán, Nanci. (2014).Introducción a la Ingeniería Industrial, Material Instruccional de Apoyo. Caracas: UNA

Ejercicio N° 34

El director de servicio de agua de una ciudad encuentra una forma de proporcionar al


menos 10 millones de litros de agua potable al día (10 mld). El suministro puede ser propor-
cionado por el depósito local o por medio de unas tuberías desde una ciudad vecina (por
bombeo). El depósito local tiene un rendimiento diario de 5 millones de litros de agua diarios
(5 mld), que no puede ser sobrepasado. La tubería no puede abastecer más de 10 millones
de litros diarios (10 mld), debido a su diámetro. Por otra parte; por acuerdo contractual, se
bombearía como mínimo 6 millones de litros diarios (6 mld). Finalmente el agua del depósito
cuesta 300 Bs por millón de litros de agua (ml) y 500 Bs por tubería (por bombeo), lo que
conduce a minimizar los costos de suministro diario de agua. Se pide:

Establecer las restricciones sin llegar al sistema de ecuaciones.

construir la función objetivo

Ejercicio N° 35

Una persona dispone de 4000 Bs para invertir y se le presentan tres opciones de in-
versión. Cada opción requiere de depósitos en cantidades de 1000 Bs, puede invertir lo que
desee en las tres opciones.

Las ganancias esperadas son las siguientes:

Inversión
Ganancias 0 1000 2000 3000 4000
Opción 1 0 2000 5000 6000 7000
Opción 2 0 1000 3000 6000 7000
Opción 3 0 1000 4000 5000 8000

¿Cuál es el modelo con el cual se puede determinar el dinero que deberá invertirse en
cada opción para maximizar las ganancias?

EJERCICIOS PROPUESTOS DE LA UNIDAD Nº 2


Estos ejercicios se proponen con la finalidad de proporcionar al estudiante, otro
material que le permita ejercitarse, para la presentación de las pruebas. No se
muestran las respuestas de estos ejercicios en el texto. Puede apoyarse con los
ejemplos ilustrativos mostrados o con la lectura de otros textos, así como, con la
búsqueda de información en la web.

400
Sánchez Marchán, Nanci. (2014).Introducción a la Ingeniería Industrial, Material Instruccional de Apoyo. Caracas: UNA

LA EMPRESA
Ejercicio Nº 1

Analice una empresa que usted conozca bien o haya trabajado en ella y:

a) Considere en qué forma trabaja desde el punto de vista de la eficacia y de la efi-


ciencia.

b) Proponga cambios para resolver los problemas en su funcionamiento.

Ejercicio Nº 2

Complete la siguiente figura:

Tiene a su cargo la dirección total de la


empresa

Alta dirección

Nivel intermedio

Desarrollo de los programas, ejecución y


control de las actividades.

Ejercicio Nº 3

Elabore la representación gráfica de un sistema de producción de la explotación de una


mina de hierro, de acuerdo con los siguientes datos:

Los elementos del insumo son la mina, la dinamita y la energía. Para la producción del
mineral de hierro; se requieren varias operaciones para su transformación: excavar, dinami-
tar, recoger el mineral, transportarlo, triturarlo, almacenarlo. Existen también inspecciones
regulares con el objeto de medir el contenido de hierro del material como un control. El pro-
ducto deseado es el mineral de hierro.

401
Sánchez Marchán, Nanci. (2014).Introducción a la Ingeniería Industrial, Material Instruccional de Apoyo. Caracas: UNA

Ejercicio Nº 4
Responda las siguientes preguntas:
4.1. Distinga entre eficacia y eficiencia en la gestión empresarial.
4.2. ¿Cuándo se produce la eficiencia positiva?
4.3. ¿A qué se deben principalmente los problemas de ineficiencia organizativa?
4.4. ¿Cómo se clasifican los manuales administrativos?

PRODUCCIÓN
Ejercicio Nº 5
Cite tres ejemplos de producción, donde se pueda reconocer el insumo, el producto y
las operaciones de transformación.

Ejercicio Nº 6

Utilizando los términos proceso, elementos, producto, específico, otros, elabore una
definición de sistema de producción. Dé un ejemplo de producción que se encuentre dentro
del sector primario de la economía. Identifique el consumo el producto y las operaciones de
transformación.

Ejercicio Nº 7
Alimentos IASENMAR, prepara galletas de miel y arroz que se venden a supermerca-
dos. La tabla mostrada, contiene las observaciones mensuales de las ventas de galletas
durante 2010 y 2011 en miles de bolsas. Alimentos IASENMAR ha vendido galletas de miel y
arroz durante 10 años y el departamento de planeación maneja la demanda como un proce-
so constante. Con los datos mostrados determine el pronóstico para enero de 20012 usando
promedio móvil simple con N = 6.
OBS MES DEMANDA OBS MES DEMANDA
1 Enero 48,5 13 Enero 48,9
2 Febrero 46,0 14 Febrero 49,5
3 Marzo 54,4 15 Marzo 59,0
4 Abril 49,8 16 Abril 56,0
5 Mayo 48,1 17 Mayo 49,3
6 Junio 55,0 18 Junio 58,5
7 Julio 47,7 19 Julio 53,0
8 Agosto 45,2 20 Agosto 48,6
9 Septiembre 51,0 21 Septiembre 50,8
10 Octubre 47,5 22 Octubre 53,4
11 Noviembre 49,1 23 Noviembre 49,8
12 Diciembre 50,8 24 Diciembre 56,3

402
Sánchez Marchán, Nanci. (2014).Introducción a la Ingeniería Industrial, Material Instruccional de Apoyo. Caracas: UNA

Ejercicio Nº 8

La empresa “HUESITOS S.A.”, se dedica a la producción de huesos de cuero que sir-


ven en el entretenimiento de mascotas. Los gerentes de producción y marketing se reunieron
informalmente para poder determinar el rumbo a seguir por la compañía en materia de cum-
plimiento con las demandas dadas para el segundo semestre del año 2005:

Julio Agosto Septiembre Octubre Noviembre Diciembre


100.000 120.000 420.000 150.000 100.000 250.000

Por el nivel de ventas estipulado, la empresa necesita estimar cuáles serán los recursos
que tendrán que poseer con sus respectivos costos para poder cumplir a la perfección con
esta nueva ola de requerimientos de huesitos.

La empresa es informada justo antes de comenzar con este análisis que los operarios
de la fábrica no pueden trabajar más de un máximo del 15% de horas extras y que las mis-
mas deben abonarse el doble que las horas normales trabajadas, sabiendo también que
cada operario trabaja 8 horas durante 20 días al mes.

La compañía no puede contar en planta con más de 350 operarios para la producción
del producto terminado. Los datos con los que la empresa cuenta para decidir la producción
del siguiente semestre son los siguientes:
•• Costo de la hora normal: 30 Bs.
•• Tiempo de fabricación por empleado: 10 huesos por cada hora trabajada.
•• Contratar cada empleado cuesta el doble de un sueldo mensual en horas
normales.
•• Despedir un empleado es equivalente a abonar tres sueldos mensuales
al operario (también en horas normales).
•• Se estima que la empresa a julio de 2005 contaba en almacén con
50.000 huesos y que deseaban mantener durante el próximo semestre
un mínimo de 25.000 huesitos por mes.
•• La política de la empresa no acepta que existan faltantes de producción
durante los ejercicios que se realizan.
•• Los costos de almacenaje ascienden a $10 por cada 100 unidades de
producto.
•• La empresa contaba con una dotación de 56 empleados al 31 de julio de
2005.
•• Tanto los costos de contratación y despido como los de inventarios se
imputan por lo devengado.

“HUESITOS S.A.” quiere determinar qué estrategia seguir durante el próximo semestre.
Para ello, Ud. deberá elaborar los planes correspondientes a los tipos de estrategias puras
de planeación agregada que conoce. Redacte un informe para la compañía. Avale y funda-
mente su elección.

403
Sánchez Marchán, Nanci. (2014).Introducción a la Ingeniería Industrial, Material Instruccional de Apoyo. Caracas: UNA

INVENTARIOS
Ejercicio Nº 9

El gerente de producción de una empresa manufacturera, es abordado por el encarga-


do del almacén y le expresa que la disponibilidad de espacio para almacenar la producción
de una semana es un área de 50 m 2 , con una altura de 3,8 m . La producción es de 12
cajas / hora . El volumen de las cajas es de 1 m 3 y se venden 70 cajas / dia . Determine si el
área del almacén es suficiente para almacenar la producción obtenida en la semana.

Ejercicio Nº 10

Son ocho los costos indirectos principales que se asocian con el inventario. Describa
cada uno de ellos.

PRODUCTIVIDAD
Ejercicio Nº 11

Considérese la compañía X, a continuación se muestran datos sobre los productos que


se fabricaron y los insumos que se consumieron en un período específico.
Producción = 1000 Bs
Mano de Obra = 300 Bs
Materiales = 200 Bs
Insumo de Capital = 300 Bs
Energía = 100 Bs
Otros Gastos= 50 Bs
Suponiendo que estos valores están en bolívares respecto al período base, calcule los
valores de la productividad parcial, la productividad total y el factor total.

Ejercicio Nº 12

El gerente general de la empresa CARAMELOS LAFERMAC, detectó que para lograr la


producción en la línea de caramelos masticables en el año 2004, se consumió una propor-
ción mayor de recursos que en el año 2003. Identifique el concepto que encierra lo expresa-
do por el gerente.

ESTUDIO DEL TRABAJO: METODOS, TIEMPOS Y MOVIMIENTOS


Ejercicio Nº 13

Usted fue contratado para realizar un estudio de métodos en una empresa manufacture-
ra. a) ¿Qué información requiere conocer para realizar el mismo? b) ¿Dónde puede encontrar
parte de esa información?

404
Sánchez Marchán, Nanci. (2014).Introducción a la Ingeniería Industrial, Material Instruccional de Apoyo. Caracas: UNA

Ejercicio Nº 14

Realice un gráfico de operaciones del proceso para cada una de las siguientes activi-
dades:
a. Colocarle minas a un portaminas.
b. Sujetar dos hojas de papel con un clip.
c. Poner papel a la impresora.

Ejercicio Nº 15

¿Cuáles de los movimientos propuestos por Gilbreht utilizaría usted para enroscar un
tornillo en la pared?

Ejercicio Nº 16

Se desea realizar un estudio de tiempos referente a la siguiente tarea: limpieza de la


superficie externa de una máquina, la cual después de cierto tiempo de operación, se llena
de polvo y residuos provenientes de la línea de producción donde se encuentra, dificultando
su uso.

La tarea en estudio puede dividirse en los siguientes elementos:


- Recoger el material de limpieza.
- Limpiar la superficie externa.
- Guardar material.
En la siguiente tabla se muestran para cada elemento, los tiempos tomados en 8 obser-
vaciones.

Tiempos observados (min)

Elemento 1 2 3 4 5 6 7 8
1. Recoger el material de limpieza 0.5 0.48 0.51 0.5 0.48 0.47 0.52 0.51
2. Limpiar la superficie externa 0.94 0.85 0.80 0.81 0.92 0.85 0.89 0.82
3. Guardar material 0.75 0.72 0.78 0.74 0.71 0.72 0.76 0.75

a. Calcular el tiempo de operación para cada elemento.


b. Calcular el tiempo normal de la tarea, considerando que el operador obser-
vado tuvo un desempeño del 125% para el elemento 1, 110% para el elemen-
to 2 y 80% para elemento 3.
c. Calcular el tiempo estándar de la tarea, tomando en cuenta que la empresa
tiene como política aplicar los siguientes suplementos:
•• Necesidades personales 4%
•• Fatiga general 2%
d. Calcular el número de observaciones necesario, para asegurar un nivel de
confianza del 95% y un 3% de precisión.

405
Sánchez Marchán, Nanci. (2014).Introducción a la Ingeniería Industrial, Material Instruccional de Apoyo. Caracas: UNA

Ejercicio Nº 17

Una tarea se dividió en 4 elementos A, B, C y D. Se quiere calcular el tiempo normal y


el tiempo estándar de la tarea. En la siguiente tabla se muestran los tiempos de operación
para cada elemento (obtenidos después de efectuar 15 observaciones) y el nivel de desem-
peño observado.

Elemento Tiempo de operación Nivel de desempeño


(min)
A 1.3 90
B 0.9 95
C 0.5 100
D 1.1 95

Considere que la tarea en estudio implica trabajar de pie y una monotonía física bastan-
te alta. También tome en cuenta un porcentaje de tiempo adicional necesario para satisfacer
necesidades personales (utilice la tabla de suplementos dada).

Ejercicio 18

Se ha realizado un estudio para estimar el tiempo que un trabajador se tarda en producir


una unidad. Se observó a un trabajador por un período de 45 minutos. Durante dicho perío-
do produjo 30 unidades. El analista valora ese rendimiento en un 90%. La empresa permite
un 15% del tiempo de trabajo perdido por fatiga y retrasos. ¿Cuál es el tiempo normal y el
tiempo estándar para producir una unidad?

Ejercicio 19

Los siguientes tiempos de operación se han obtenido a partir de la realización de un


estudio de tiempos de la tarea X, mediante la observación de un trabajador en 15 ciclos de
operaciones.

Número de Tiempo Número de Tiempo


observación (seg.) observación (seg.)

1 15 9 14
2 12 10 20
3 16 11 13
4 11 12 15
5 13 13 16
6 14 14 15
7 16 15 11

406
Sánchez Marchán, Nanci. (2014).Introducción a la Ingeniería Industrial, Material Instruccional de Apoyo. Caracas: UNA

8 12

El trabajador ha alcanzado un rendimiento del 115%. El tiempo permitido para necesida-


des personales y por fatiga es del 10% total de trabajo. ¿Cuál es el tiempo normal y el tiempo
estándar de esa tarea?

Ejercicio Nº 20

Sacar punta a un lápiz es una operación que puede dividirse en 8 pequeños movimien-
tos elementales. Usando las tablas MMT, se puede asignar a cada elemento un cierto nú-
mero de TMU, tal como se muestra en la siguiente tabla

Movimiento Tiempo (TMU)


Alcanzar el lápiz situado a 4 pulgadas (10 6
cm.)
Agarrar el lápiz. 2
Desplazar el lápiz 6 pulgadas. 10
Colocar el lápiz cerca del orificio del 20
sacapuntas.
Insertar el lápiz en el sacapuntas. 4
Sacar punta al lápiz. 120
Sacar el lápiz del sacapuntas. 10
Desplazar el lápiz 6 pulgadas (15 cm.) 10

¿Cuál es el tiempo necesario para sacar punta a un lápiz? convierta la respuesta a mi-
nutos y segundos.

Ejercicio Nº 21

Un estudio por muestreo del trabajo va a llevarse a cabo en los próximos 30 días, res-
pecto al tiempo perdido por retrasos inevitables en una línea de producción. Por experiencia
se estima que los retrasos inevitables ocurren el 10% del tiempo, pero se quiere actualizar
esta estimación ya que algunas condiciones han cambiado.

a. Calcule el número de observaciones que deben efectuarse si se quiere un 3.5%


de precisión y un nivel de confianza del 95%.

b. ¿Cuántas observaciones diarias deben hacerse si se dispone de 30 días para


realizar el estudio?

c. Establezca un horario para llevar a cabo las observaciones correspondientes a


un día. Considere una jornada laboral de 8 horas, de 8 a.m. a 12 m. y de 2 p.m.
a 6 p.m.

407
Sánchez Marchán, Nanci. (2014).Introducción a la Ingeniería Industrial, Material Instruccional de Apoyo. Caracas: UNA

d. Explique cómo llevaría a cabo el muestreo del trabajo y cómo calcularía el tiempo
perdido por retrasos inevitables.

Ejercicio Nº 22

Tarea a estudiar: limpiar de resina las cuchillas de una troceadora.

La tarea consiste en limpiar la resina que se acumula en las cuchillas durante el troceo,
para poderlas afilar. Se le coloca disolvente a la resina y luego se raspa la cuchilla con una
espátula, limpiándola a continuación.

En la siguiente tabla se muestran los elementos en los que se dividió la tarea y los tiem-
pos tomados en 10 observaciones.

Tiempo en minutos

Elemento 1 2 3 4 5 6 7 8 9 10
1. Colocar disolvente. 1.7 1.6 2.0 2.7 2.3 3.0 1.6 2.4 3.5 2.6
2. Raspar resina. 2.5 3.3 2.9 3.0 3.0 3.4 3.0 2.4 2.5 3.6
3. Limpiar cuchilla. 1.2 2.4 2.3 2.4 2.2 3.1 2.5 2.7 2.6 2.8

a. Calcular el tiempo de operación para cada elemento.

b. Calcular el tiempo normal y el tiempo estándar. Considere que el factor de califi-


cación asignado por el observador para cada elemento es:

Elemento 1 → 95%
Elemento 2 → 95%
Elemento 3 → 110%
También considere que se aplican los siguientes suplementos:

Necesidades personales 5%
Trabajar de pie 2%
Interrupciones 3%
Fatiga general 2%
c. Calcular el número de observaciones (n), para un nivel de confianza del 95% y un
5% de precisión.

CALIDAD EN LA EMPRESA
Ejercicio Nº 23

De acuerdo con su criterio, ¿Cuáles son los requerimientos de calidad de los clientes
en un taller de servicio automotor?

408
Sánchez Marchán, Nanci. (2014).Introducción a la Ingeniería Industrial, Material Instruccional de Apoyo. Caracas: UNA

Ejercicio Nº 24

Existen muchas definiciones para el término calidad, una de ellas es: cumplir con los
requisitos exigidos por el cliente. Sobre la base de esta definición, identifique los requisitos
del cliente en los siguientes productos o servicios:
•• Un auto rojo.
•• Una tonelada de azúcar.
•• Una cita médica a las 5 PM.
•• Un apartamento tipo estudio

SEGURIDAD E HIGIENE EN EL TRABAJO


Ejercicio Nº 25

Mencione los implementos que debe contener el equipo de seguridad de los obreros
que trabajan en una empresa constructora. Justifique su respuesta

Ejercicio Nº 26

El ruido es un contaminante del ambiente, de acuerdo con su criterio mencione algunas


medidas preventivas para proteger el oído del mismo.

Ejercicio Nº 27

Identifique qué debe hacerse para lograr que las medidas preventivas contempladas en
un plan de seguridad, operen con eficacia.

ERGONOMÍA
Ejercicio Nº 28

Estudie la posición de trabajo mostrada.

Diga si es ergonómica. Justifique su respuesta.

409
Sánchez Marchán, Nanci. (2014).Introducción a la Ingeniería Industrial, Material Instruccional de Apoyo. Caracas: UNA

Ejercicio Nº 29

¿Cómo debe ser el puesto de trabajo para trabajadores que realizan su labor de pie?

MANTENIMIENTO INDUSTRIAL DE MÁQUINARIA Y EQUIPO


Ejercicio Nº 30

En la empresa TACATA, CA., ocurrió una avería en el sistema eléctrico, atendiéndose


ésta de manera inmediata, para evitar la paralización de la planta. Identifique el tipo de man-
tenimiento aplicado.

Ejercicio Nº 31

La empresa MANAURE CA., produce alimentos lácteos; es una empresa que se preocu-
pa mucho por el mantenimiento de los equipos que conforman su aparato productivo y para
ello cuenta con una plataforma estadística que le permite predecir las fallas que se pueden
presentar en la planta ¿Qué tipo de mantenimiento aplica en este caso la empresa?

Ejercicio N° 32

Muchos de los problemas que se presentan en maquinarias y equipos de la empresa,


se pueden evitar o prevenir si se realiza un mantenimiento periódico a cada uno de sus com-
ponentes, sobre esta base responda las siguientes preguntas:

a) Defina qué es el mantenimiento preventivo.

b) De acuerdo a su criterio, mencione algunas ventajas que se pueden logran, al


aplicar este tipo de mantenimiento.
c) Como ejemplo diga cómo puede realizar mantenimiento preventivo, al teclado de
su computadora.

PREPARACIÓN DE PROYECTOS DE INVERSIÓN


Ejercicio Nº 33

Sobre la base del título y evaluación del proyecto planteado, exprese una posible con-
clusión al respecto.

410
Sánchez Marchán, Nanci. (2014).Introducción a la Ingeniería Industrial, Material Instruccional de Apoyo. Caracas: UNA

Título: INFLUENCIA SOCIOECONÓMICA DEL DETERIORO DE AUTOPISTAS Y


CARRETERAS EN EL TERRITORIO NACIONAL.

Evaluación: este estudio, apunta a entender el impacto de las carreteras y autopistas


en el territorio nacional. El estudio focalizó los impactos en la infraestructura y servicios de
transporte, servicios sociales y medio ambiente. También se evaluaron los beneficios econó-
micos de las posibles mejoras y su sostenibilidad.

Ejercicio Nº 34

¿Qué sucede cuando se encuentra una solución para un problema planteado?

LOCALIZACIÓN Y DISTRIBUCIÓN EN PLANTAS


Ejercicio Nº 35

Mejore la siguiente distribución de planta, correspondiente al proceso de fabricación de


baldosas:

INVESTIGACIÓN DE OPERACIONES
Ejercicio N° 36

Una compañía vende dos mezclas diferentes de nueces. La mezcla más barata contie-
ne un 80% de chocolate y un 20% de nueces, mientras que la más cara contiene 50% de
cada tipo. Cada semana la compañía obtiene 1800 kilos de chocolate y 1200 kilos de nueces

411
Sánchez Marchán, Nanci. (2014).Introducción a la Ingeniería Industrial, Material Instruccional de Apoyo. Caracas: UNA

de sus fuentes de suministros. ¿Cuál es el modelo que permite saber la cantidad de mezcla
que se debería producir a fin de maximizar las utilidades si las ganancias son de 10 Bs por
cada kilo de la mezcla más barata y de 15 Bs por cada kilo de la mezcla más cara?

Ejercicio N° 37

La compañía “EXACTITUD” es una empresa que suministra partes y componentes


electrónicos muy especializados para la industria de artículos de consumo electrónico. Las
órdenes son obtenidas por consumo en las que los factores de decisión son precio, calidad
y tiempo de entrega. Este último es el factor decisivo ya que la calidad y precio son bastante
similares entre competidores.

Esta manera de operar forzó a la compañía a fluctuaciones severas en personal, pro-


ducción y tiempos extra pagados, a la vez que se detecte una disminución de la productivi-
dad al aumentar la producción, ya que la efectividad del personal disminuye en los tiempos
extra y, el personal recién contratado no tiene la misma habilidad que el personal ya entre-
nado. Defina un objetivo para este problema y las actividades que se realizan en el sistema.

Ejercicio N° 38

Un fabricante de dos productos A y B dispone de 6 unidades de material y 28 horas para


su ensamblaje, el tipo A requiere 2 unidades de material y 7 horas de ensamblaje, el tipo B
requiere una unidad de material y 8 horas de ensamblaje, los precios de los productos son
120 y 80 Bs respectivamente. Elabore el modelo que permita determinar cuántos productos
de cada tipo debe fabricar para maximizar su ingreso.

412
Sánchez Marchán, Nanci. (2014).Introducción a la Ingeniería Industrial, Material Instruccional de Apoyo. Caracas: UNA

UNIDAD 3

ÁREA DE DESEMPEÑO DE LA INGENIERÍA INDUSTRIAL

ÁREA DE GERENCIA

Objetivo:
Explicar los aportes a la Ingeniería Industrial del área de gerencia, para lograr la
solución de problemas, utilizando la administración de recursos, a partir de diversas
teorías, modelos, métodos y técnicas gerenciales.

El área de gerencia determina el manejo ADMINISTRACIÓN DE RECURSOS


óptimo de los recursos humanos, finan-
cieros y físicos que forman parte de las
organizaciones a través de: la adminis-
tración de recursos, economía y finan-
ECONOMÍA Y FINANZAS
zas, marketing industrial, comportamien-
to organizacional, entre otros. Las tareas
gerenciales son una parte importante de
las funciones de una empresa, en este
sentido, es necesario manejar los as- MARKETING INDUSTRIAL
pectos básicos de la solidez fiscal y tri-
butaria, el ingreso operativo neto y los
fondos procedentes de las operaciones
fundamentales de la empresa, para mi- C O M P O R T A M I E N T O
nimizar los gastos y maximizar el flujo de
caja, procedente de la administración.
Contribuyendo a la estabilidad de la es-
tructura financiera. ORGANIZACIONAL

413
Sánchez Marchán, Nanci. (2014).Introducción a la Ingeniería Industrial, Material Instruccional de Apoyo. Caracas: UNA

ADMINISTRACIÓN DE RECURSOS

•• Administración: origen y desarrollo.


•• La administración.
•• Administradores.
•• Objetivos de la administración.
•• Diferentes conceptos de administración.
•• Funciones de la administración y de la gerencia.
•• Principios de la administración.
•• Diferencias entre administración y gerencia.
•• Teorías de la administración.
•• Teorías más recientes de la administración.
•• Recursos humanos:
–Origen de la administración de recursos humanos.
–Evolución de la administración de recursos humanos.
–Planificación de plantillas.
–Selección de personal.
–Política salarial.

ADMINISTRACIÓN: ORIGEN Y DESARROLLO


Algunos escritores, remontan el desarrollo de la administración a los comerciantes
sumerios y a los egipcios, antiguos constructores de las pirámides, o a los métodos organi-
zativos de la iglesia y las milicias antiguas. Su origen genérico es Ad-minis-ter y significa:
Ad: dirección, Minis: menor que, Ter: subordinación, obediencia.

Las innovaciones tales como la extensión de los números árabes(entre los siglos V y
XV) y la aparición de la contabilidad de partida doble en 1494, proporcionaron las herramien-
tas para el planeamiento y el control de las organizaciones y de esta forma el surgimiento
formal de la administración.

Algunos conciben la administración moderna como una disciplina que comenzó como
una ramificación de la economía en el siglo XIX. Los economistas clásicos tales como Adam
Smith y John Stuart Mill proporcionaron una base teórica sobre la asignación de los recur-
sos, la producción y la fijación de precios. Al mismo tiempo, innovadores como Eli Whitney,
James Watt y Matthew Boulton, desarrollaron herramientas técnicas de producción tales
como la estandarización, procedimientos de control de calidad, contabilidad analítica y pla-
neamiento del trabajo.

415
Sánchez Marchán, Nanci. (2014).Introducción a la Ingeniería Industrial, Material Instruccional de Apoyo. Caracas: UNA

A Frederick Winslow Taylor, se le considera el padre moderno de la Administración, el


cual la considera una ciencia. Estudió al factor humano, así como a la mecánica y a los ma-
teriales dentro de un sistema de producción (ver tabla Nº 2).

La Administración es la base de todo en una empresa, ya que si no se sabe adminis-


trar no se obtendrán los resultados deseados. En términos generales, cuando se habla de
administración implica saber mantener el orden de todo, hasta de la vida, tiempo, entre otros.
Por ello, la administración está en todos lados hasta en los hogares.

Es decir, implica saber:

Figura 71. Interrogantes referentes a las acciones preestablecidas de la Administración.

Para lograr responder a las interrogantes anteriores, es necesario un ambiente donde


las personas trabajando juntas, puedan desempeñarse bien, lo cual exige una interacción de
aportes y condiciones de tipo económico, psicológico, político, social y técnico.

En la administración, las personas trabajando en grupo alcanzan con eficiencia metas


seleccionadas. Esto se aplica a todo tipo de organizaciones bien sean pequeñas o grandes
empresas, lucrativas y no lucrativas, a las industrias manufactureras y a las de servicio. Las
organizaciones cuentan con personas que tienen el encargo de servirle para alcanzar sus
metas, llamados gerentes o administradores.

ADMINISTRADORES

Son profesionales que en una organización, dirigen las actividades de otros. Estos tam-
bién podrán tener algunas responsabilidades operativas. Se pueden dividir en dos grandes
grupos:
•• Los administradores operativos, son los que trabajan directamente en
un puesto o actividad y no tienen responsabilidad de supervisar el trabajo
de otros empleados.
•• Los administradores gerentes, dirigen las actividades de otras perso-
nas. Estos tienen la responsabilidad de realizar acciones que permitan

416
Sánchez Marchán, Nanci. (2014).Introducción a la Ingeniería Industrial, Material Instruccional de Apoyo. Caracas: UNA

que los trabajadores y/o personal aporten sus mejores esfuerzos indivi-
duales, para alcanzar los objetivos del grupo.

Los administradores gerentes se clasifican en:


•• Gerentes de primera línea. Son las personas que ocupan el primer ni-
vel gerencial de una organización, supervisan y coordinan las activida-
des de los trabajadores operativos. Actividad que no realizan los adminis-
tradores altos y medios. Generalmente se le llama capataz, supervisor o
administrador de oficina.
•• Gerentes medios. El término gerencia media incluye varios niveles de
una organización. Son los responsables de la implementación de las po-
líticas y planes desarrollados por los administradores de más alto nivel.
Su responsabilidad es supervisar y coordinar las actividades de los ad-
ministradores de primera línea. Se denominan comúnmente gerentes de
planta, gerentes de operaciones, gerentes de división.
•• Alta gerencia. La alta gerencia está compuesta por un grupo de perso-
nas comparativamente pequeño el cual es responsable de administrar
toda la organización. Establecen los objetivos, la estrategia organizacio-
nal, las políticas operacionales y dirigen la interacción de la organización
con su entorno.

En síntesis se puede decir que el término gerente, se usa para referirse a la persona
que es responsable de cumplir con las cuatro actividades básicas de la administración (pla-
nificación, organización, dirección y control) en el desarrollo de sus relaciones y que pueden
trabajar en diferentes niveles de una empresa, así como, en diferentes rangos de actividades
dentro de ella, por ello un gerente es:

Una persona responsable de dirigir las actividades que ayudan a las organizaciones en el
logro de sus metas. La medida de la eficiencia y la eficacia de un gerente se miden con
el alcance de metas y consecución de objetivos apropiados en su gestión.

A nivel operativo, los gerentes están dejando de seguir las directrices marcadas y
pasan a ser emprendedores de ideas de negocio. Asimismo, los gerentes de nivel
intermedio están pasando de ejercer labores de control para apoyar de modo con-
tinuo las distintas iniciativas empresariales. Por último, los de alto nivel, en vez de
diseñar estrategias, empiezan a dotar a las organizaciones de nuevas direcciones
y crean ambientes donde se busca el desafío constantemente.

OBJETIVOS DE LA ADMINISTRACIÓN
•• Alcanzar en forma eficiente y eficaz los objetivos de una organización.
Recuerde que la eficacia consiste en alcanzar las metas y la eficiencia
se obtiene cuando se logran los objetivos con el mínimo de recursos. Son

417
Sánchez Marchán, Nanci. (2014).Introducción a la Ingeniería Industrial, Material Instruccional de Apoyo. Caracas: UNA

dos conceptos presentados por Peter Drucker, uno de los autores más
reconocidos en el campo de la administración. Ver unidad 2.
•• Permite a la empresa ampliar la perspectiva sobre el medio donde se
desarrolla.
El éxito que puede tener la organización al alcanzar sus objetivos y también al
satisfacer sus obligaciones sociales, depende en gran medida, de sus gerentes.
Si los gerentes realizan debidamente su trabajo, es probable que la organización
alcance sus metas.

DEFINICIONES DE ADMINISTRACIÓN SEGÚN ALGUNOS AUTORES


Administración se define como:
•• “El arte o técnica de dirigir e inspirar a los demás, con base en un profun-
do y claro conocimiento de la naturaleza humana” (J. D. Mooney).
•• “Es la dirección de un organismo social y su efectividad en alcanzar los
objetivos, fundada en la habilidad de conducir a sus integrantes” (Koontz
y O’Donnell).
•• Es el proceso mediante el cual se planifica, organiza, controla y dirigen
los esfuerzos de los miembros de una organización, así como la utiliza-
ción de los demás recursos existentes en ella, para alcanzar los objeti-
vos, a través de metas preestablecidas (la autora).

Otras definiciones:

Administración científica. Término que se utilizó originalmente para referirse al trabajo


y el enfoque de F. W. Taylor y de sus colaboradores para analizar la administración. Implica
que los métodos de investigación, análisis y resumen científicos se pueden aplicar a las ac-
tividades de los administradores.

Administración como arte. Administrar es un arte que exige inteligencia, experiencia,


decisión y mesura.

Administración como ciencia. Conocimiento organizado (conceptos, teoría, principios


y técnicas) en la que se sustenta la práctica de la administración.

Gerencia. Es un tipo particular de administración, que orienta los procesos hacia la


búsqueda de eficiencia, es decir, el logro de fines a un mínimo costo.

FUNCIONES DE LA ADMINISTRACIÓN Y GERENCIA

Los conceptos, principios, teorías y técnicas de la administración se agrupan en cuatro


funciones:

418
Sánchez Marchán, Nanci. (2014).Introducción a la Ingeniería Industrial, Material Instruccional de Apoyo. Caracas: UNA

Planeación Implica seleccionar misiones y objetivos, así como las


acciones necesarias para cumplirlos y requiere por lo
tanto de la toma de decisiones; esto significa la elección
de planes de acción a partir de diversas alternativas.

Organización Es la parte de la administración que supone el estable-


cimiento de una estructura intencionada de los papeles
que los individuos deberán desempeñar en una empre-
sa. La estructura es intencionada en el sentido de garan-
tizar la asignación de todas las tareas necesarias para
el cumplimiento de las metas, asignarlas a las personas
mejor capacitadas para realizar esas tareas.

Dirección Es el hecho de influir en las personas para que contribu-


yan a favor del cumplimiento de las metas organizaciona-
les y grupales; tiene que ver con el aspecto interpersonal
de la administración.

Control Consiste en medir y corregir el desempeño individual y


organizacional para garantizar que los hechos se ape-
guen a los planes, es decir, la medición del desempeño
sobre la base de metas y planes, la detección de des-
viaciones respecto de las normas y la contribución a la
corrección de éstas.

Ejemplo Ilustrativo Nº 1
¿Cuál es la importancia de la dirección en la administración?
La dirección es la parte central, la esencia, el corazón de la administración, a la
cual se debe disciplinar para desarrollar el resto de las funciones: planificación,
organización y control, para conseguir los objetivos de la empresa utilizando efi-
cientemente los recursos disponibles.
Otra importancia radica en que este elemento de la administración está en con-
tacto directo con los empleados, tanto a nivel individual, como grupal. Conside-
rando que los empleados no están solamente interesados en los objetivos de la
empresa, también tienen sus propios objetivos. Razón por la cual, la dirección
debe encaminar el esfuerzo humano hacia los objetivos de la empresa, sin des-
cuidar los intereses de los empleados, es decir, el administrador debe pensar en
términos de los resultados relacionados con la orientación, la comunicación, la
motivación y la dirección.

419
Sánchez Marchán, Nanci. (2014).Introducción a la Ingeniería Industrial, Material Instruccional de Apoyo. Caracas: UNA

Ejemplo Ilustrativo Nº 2
La señora Maritza López gerente de la empresa ARANT S.R.L., necesita verificar
si los resultados obtenidos en las operaciones de la empresa, son iguales o pa-
recidos a los planes presentados. ¿Cuál es la función y las acciones que debería
considerar la señora López para definir este caso?

Respuesta
Para este caso la señora López debería considerar la aplicación de un control, es
decir, la medición y corrección de la ejecución sobre la base de las metas estable-
cidas, comparando lo realizado con lo planeado, así como cuentas de las tareas
y actividades asumidas. Algunas de las acciones a considerar son las siguientes:
•• Establecimiento de patrones de comparación en puntos estra-
tégicos.
•• Medición de lo realizado.
•• Comparación de lo que se ha realizado con lo que se ha pla-
neado.
•• Corrección de las fallas encontradas.

PRINCIPIOS BÁSICOS DE LA ADMINISTRACIÓN


Son catorce principios propuestos por Fayol (1916), considerados como el pilar de la
administración:

1. División del trabajo. Las personas más especializadas se desempeñan eficien-


temente en su oficio.

2. Autoridad. Los gerentes emiten órdenes para que se realicen las cosas.

3. Disciplina. Respeto de las normas y reglas de la organización.

4. Unidad de mando. Recibir instrucciones de una sola persona para ejecutar una
operación en particular.

5. Unidad de dirección. Las operaciones que persiguen un mismo fin deben ser
contempladas en un solo plan y dirigidas por la misma persona.

6. Subordinación. Los intereses de los empleados no deben ser más importantes


que los intereses de la organización como un todo.

7. Remuneración. La compensación debe ser equitativa para todos los miembros


de la organización.

8. Centralización. Los subordinados no tiene participación en la toma de decisio-


nes. En caso contrario se habla de descentralización.

420
Sánchez Marchán, Nanci. (2014).Introducción a la Ingeniería Industrial, Material Instruccional de Apoyo. Caracas: UNA

9. La jerarquía. Representada por la línea de autoridad en orden de rango.

10. Orden. Ubicación de personas y materiales en el tiempo y espacio adecuado.

11. Equidad. Administradores justos con sus subordinados.

12. Estabilidad del personal. Evitar una tasa alta de rotación del personal, en fun-
ción de la buena marcha de la empresa.

13. Iniciativa. Otorgar libertad a los subordinados para la puesta en marcha de sus
planes.

14. Espíritu de equipo. Promover el compañerismo para lograr un sentido de unidad


en el seno de la organización.

La administración se basa en el esfuerzo cooperativo que el hombre desa-


rrolla en las organizaciones, su tarea básica es alcanzar los mejores resul-
tados, utilizando la capacidad de las personas. En cualquier tipo de organi-
zación humana se busca el logro de determinados objetivos con eficiencia
y eficacia. La administración dirige el esfuerzo de los grupos organizados.

TEORÍAS DE LA ADMINISTRACIÓN
Las teorías de la administración se construyen con distintos enfoques que intentan co-
nocer, comprender, describir, explicar y predecir el comportamiento de las organizaciones,
entre las que se encuentran: las teorías clásica, neoclásica y científica de la administración,
teoría burocrática, teoría estructuralista de la administración, teoría de sistemas, teoría ma-
temática, teoría de relaciones humanas, teoría conductista, teoría del comportamiento or-
ganizacional, teoría del desarrollo organizacional entre otras. También se presentan algunas
teóricas de origen más reciente, como teoría de la dependencia de recursos, teoría ecológica
evolucionista y teoría de costos de transacción.

A continuación se muestran dos tablas:


•• Tabla 44. Aspectos más resaltantes de las teorías de la administración,
sus enfoques y énfasis.
•• Tabla 45. Teorías más recient

421
Sánchez Marchán, Nanci. (2014).Introducción a la Ingeniería Industrial, Material Instruccional de Apoyo. Caracas: UNA

Tabla 44. Aspectos más resaltantes de las teorías clásicas de la administración,


sus enfoques y énfasis:
Teorías de la
Enfoque Énfasis
Administración
• Se aplican los métodos de la ciencia  a los problemas de
Administración Científica la administración.
o Taylorismo • Los principales métodos son la observación y la medi-
(1890 - 1917) ción.

Henry Towne, Frederick • Se fusionan los intereses de los trabajadores y de la


Winslow Taylor, Frank y administración en un todo mutuamente beneficioso.
Lillian Gilbreth. Yoichi • Entendimiento por parte de gerentes y no gerentes de la

En la Estructura y las Tareas


Ueno cantidad y calidad de trabajo a realizarse en un período
dado.
• Se considera la empresa como un todo, no sólo un seg-
Teoría Clásica mento de ella.
• Divide las actividades industriales en técnicas comer-
Henry Fayol
ciales, financieras, de seguridad, contables, técnicas y
(1841-1925) gerenciales.
• Formulación de catorce principios (ver página anterior).
Teoría Neoclásica • Atención a los aspectos prácticos de la administración
por el realismo y la búsqueda de resultados concretos.
Peter Ferdinan Druker • Busca demostrar una relación causa-efecto.
Cyril J. O´Donnell • Los objetivos son valores investigados o resultados
William Newman deseados por la organización.
• Recogen el contenido de casi todas las teorías adminis-
(1909 -1960) trativas.
• Describe especialmente las organizaciones en función a
los conceptos de jerarquía y autoridad.
Teoría Burocrática • Estudia la manera de cómo se ejerce el poder, así como
las relaciones con los subordinados.
Max Weber
• Establece normas y relaciones estrictas, con registros
(1864-1920) por escrito.
• Selección del personal según capacidades, manteniendo En la Estructura
líneas jerárquicas de autoridad.
• Analiza las relaciones que tienen las organizaciones con
el sistema social que las rodea.
Teoría Estructuralista • Identifica los conflictos internos y externos y su influen-
cia en los cambios e innovaciones que ocurren en las
Ralf Dahrendorf, Renate organizaciones.
Maintz y Amitai Etzioni
• Se considera los intercambios y las influencias del con-
(1930 - 1947 ) texto en la organización.
• Los grupos formales e informales dentro de la empresa,
así como, la influencia en su contexto.

422
Sánchez Marchán, Nanci. (2014).Introducción a la Ingeniería Industrial, Material Instruccional de Apoyo. Caracas: UNA

Teorías
Enfoque Énfasis
Administrativas

Teoría de Sistemas • Estudio interdisciplinario que trata de encontrar las


propiedades comunes a entidades.
Ludwig von Bertalanffy • Los sistemas, que se presentan en todos los niveles
(1951-1972) de la realidad, son objeto de disciplinas diferentes.

En la Tecnología
• La toma de decisión es un proceso y por lo tanto pue-
Teoría Matemática de la de ser traducido a fórmulas matemáticas.
Administración • Establece la relación de la investigación de operacio-
nes con la ciencia administrativa.
(1947- 1954)
• Las técnicas matemáticas son utilizadas para modelar,
Von Neumann y Morgenstern, analizar y solucionar problemas de administración.
Wald Savage • Utiliza las calculadoras, computadoras y grupo de
expertos para  la solución de problemas.
Teoría de las • Si la gerencia mejora el ambiente de trabajo y los su-
pervisores les prestan atención especial, los emplea-
Relaciones Humanas
dos trabajarían con más tesón.
(1924 -1933) • Los grupos informales de trabajo (el entorno social
Elton Mayo Fritz J. Roethlisber- de los empleados) tienen una influencia positiva en la
ger y William J. Dickson productividad.
Teoría Conductista • Las personas procuran obtener algo más que recom-
pensas o placer, dado sus formas de vidas complejas.

En las personas y el ambiente


(1950- 1970)
• Considera los factores que motivan o desmotivan al
Abraham Maslow, Douglas Mc- trabajador dentro de la organización.
Gregor y Frederick Herzberg,
entre otros
Teoría del Comportamiento • Se fundamenta en el comportamiento individual de las
personas, para explicar su comportamiento.
Organizacional
• Lograr la motivación humana,
(1947-1960)
Herbert Alexander Simon,
Chester Barnard, Douglas
McGregor, Renis Likert, entre
otros
Teoría del Desarrollo • Desarrollo planeado de las organizaciones.
Organizacional • Cambio organizacional planeado y un enfoque de
sistema abierto.
(1960-1970)
Grupo de científicos sociales

423
Sánchez Marchán, Nanci. (2014).Introducción a la Ingeniería Industrial, Material Instruccional de Apoyo. Caracas: UNA

Tabla 45.Teorías más recientes de la administración


Teoría de la Dependencia de • Enfoque económico y político de las organizaciones.
Recursos • Intercambio a través de funciones, alianzas, entre otras rela-
(1970 – 1975) ciones interorganizacionales para disminuir la dependencia y
contingencia.
Aldrich y Pfeffer, Pfeffer y
• Contexto político interno para el análisis y la toma de decisio-
Salancik, Aldrich, Pfeffer,
nes estratégicas.
Wamsley y Zald, Benson,Haselfeld
y Jacobs, entre otros
• Estudia las formas organizacionales y las poblaciones de orga-
nizaciones.
Teoría Ecológica Evolucionista • Las organizaciones no se adaptan al entorno, sino que son
seleccionadas por éste, en un proceso natural que tiene tres
(1977 - 1989) etapas: 1) Variación en la forma o modo de operar, planeada
Hannan, Freeman, Campbell o no; 2) Selección de las formas que funcionan con éxito y
3) Retención de las formas seleccionadas, conservándolas y
reproduciéndolas.
• Capacitación gerencial y la formación profesional sobre la base
de los modelos que funcionan con éxito.
Teoría de los Costos de • Proporciona una visión limitada de la organización.
Transacción • La unidad de análisis es la transacción que se lleva a cabo en
(1975- 1988) contextos de relaciones sociales más que de relaciones econó-
micas.
Williamson, Granovenen,
Lazerson

Actividad propuesta
Revise las teorías clásicas de la administración y realice una comparación entre
ellas, considerando el énfasis y el enfoque que las identifica. A su vez, compare
las teorías clásicas con las teorías más recientes.

El término administración debe describirse más bien como una actividad con largos
períodos de cambios, interrumpidos ocasionalmente por momentos breves de estabilidad.
Los puestos que ocupan los trabajadores hoy día se encuentran en un estado de flujo perma-
nente, de modo que los empleados tienen que actualizar continuamente sus conocimientos
y habilidades para realizar nuevos cometidos laborales. Los administradores y empleados
deben aprender a enfrentar la temporalidad, a vivir con flexibilidad, espontaneidad e impre-
visibilidad.

Una de las funciones más importantes en la administración de una empresa es la direc-


ción del recurso humano, en este sentido se introduce a continuación este concepto.

LA ADMINISTRACIÓN DE RECURSOS HUMANOS

La dirección del recurso humano, es una de las funciones clásicas de la administración,


que permite determinar las actividades necesarias para dotar a la organización de personal
y mantener un alto desempeño de éstos.

424
Sánchez Marchán, Nanci. (2014).Introducción a la Ingeniería Industrial, Material Instruccional de Apoyo. Caracas: UNA

Surge de la necesidad de crear un departamento en la organización que asumiera la


misión exclusiva de solucionar los problemas concernientes al personal, esto demandaba un
equipo especializado que eliminara la improvisación existente en este campo.

El término Recursos Humanos (abreviado como RRHH, RH), también conocido como
capital humano, se originó en el área de economía política y ciencias sociales, donde se
utilizaba para identificar a uno de los tres factores de producción, también conocido como
trabajo (los otros dos son tierra y capital).

En la administración de empresas, se denomina administración de recursos humanos a:

La función que se ocupa de seleccionar, contratar, formar, emplear y retener a los tra-
bajadores de las organizaciones. Estas tareas las puede desempeñar una persona o un
departamento determinado (los profesionales en recursos humanos) junto a los directivos
de estas organizaciones.

Generalmente la función de recursos humanos está compuesta por áreas tales


como: reclutamiento-selección, compensaciones-beneficios, formación-desarrollo y admi-
nistración-operaciones. Dependiendo de la empresa o institución donde la función de re-
cursos humanos opere, pueden existir otros grupos que desempeñen distintas responsabi-
lidades que tengan que ver con aspectos tales como la administración de la nómina de los
empleados, el manejo de las relaciones con sindicatos, entre otros. Para poder ejecutar la
estrategia de la organización es fundamental la administración de los recursos humanos,
considerando conceptos tales como:
•• Comunicación organizacional.
•• Liderazgo.
•• Trabajo en equipo.
•• Negociación.
•• Cultura.
•• Sistema de administración de recursos humanos.

En la tabla siguiente se muestran las principales funciones de cada una de las áreas de
recursos humanos (RRHH):

Tabla 46.Funciones de las áreas de RRHH

Principales funciones de las áreas de RRHH

Formación- Reclutamiento- Compensaciones - Administración -


Desarrollo Selección Beneficios operaciones
• Capacitación. • Atracción. • Bonos. • Aspectos administrati-
• Entrenamiento. • Selección. • Ganancias por ventas.
vos en general.
• Planes de carrera. • Incorporación e • Beneficios no moneta-
• Liquidación de habe-
• Planes de sucesión. inducción de perso- rios. res.
• Estudios. nas. • Políticas salariales.
• Control de ausentismo,
etc.
Estas funciones deben considerar las relaciones con los gremios (relaciones industriales)

425
Sánchez Marchán, Nanci. (2014).Introducción a la Ingeniería Industrial, Material Instruccional de Apoyo. Caracas: UNA

El objetivo de las funciones consiste en desarrollar y administrar políticas, programas


y procedimientos para proveer una estructura administrativa eficiente, empleados capaces,
trato equitativo, oportunidades de progreso, satisfacción en el trabajo y una adecuada segu-
ridad en el mismo, cuidando el cumplimiento de sus objetivos que redundará en beneficio de
la organización, los trabajadores y la colectividad.

El departamento de RRHH debe preparar adecuadamente el desarrollo de las personas


en las organizaciones, a través de un sistema propio que considere:

Reclutamiento de personal: es un conjunto de procedimientos orientados a atraer


candidatos potencialmente calificados y capaces de ocupar cargos en la organización. El
proceso de reclutamiento comienza cuando la empresa coloca el aviso de vacante y termina
cuando el interesado deposita su currículo en las oficinas de RRHH.

Selección de personal: consiste en escoger entre los candidatos reclutados, los más
adecuados para ocupar cargos, manteniendo la eficiencia y eficacia. El objetivo es escoger
personal idóneo atendiendo al perfil del candidato

Descripción y análisis de puestos: a través del análisis y descripción de puestos,


se puede ubicar el puesto de un individuo en la organización, describir su misión, funciones
principales y tareas necesarias para desempeñar de modo completo dichas funciones.

Evaluación del desempeño: toda evaluación es un proceso para estimular o juzgar el


valor, la excelencia y las cualidades de alguna persona.

Los objetivos fundamentales de la evaluación del desempeño son:

1. Establecer condiciones de medición del potencial humano en el sentido de pre-


cisar su plena aplicación.

2. Distinguir el recurso humano como un recurso básico de la empresa y cuya


productividad puede desarrollarse indefinidamente, dependiendo de la forma de
administración.

3. Fomentar oportunidades de crecimiento y condiciones de efectiva participación


a todos los miembros de la organización, teniendo en cuenta, por una parte, los
objetivos empresariales y por la otra, los objetivos individuales.

Capacitación y entrenamiento: define acciones dirigidas a poner a una persona en


condiciones de ejecutar satisfactoriamente una tarea particular especificada de antemano.
Ahora bien, en la realización de cualquier tarea intervienen los conocimientos y las habilida-
des. La capacitación se centra en el componente de conocimientos y el entrenamiento en el
de habilidades.

Administración de sueldos y salarios: precio del trabajo efectuado por cuenta y or-
den de un patrono. El concepto del salario ha evolucionado con el progreso y constituye uno
de los problemas más complejos de la organización económica y social de los pueblos. Los

426
Sánchez Marchán, Nanci. (2014).Introducción a la Ingeniería Industrial, Material Instruccional de Apoyo. Caracas: UNA

desequilibrios son capaces de provocar las más graves perturbaciones (huelgas, alzamien-
tos, revoluciones, entre otras.)

Planes de beneficios: conjunto de facilidades, comodidades, ventajas y servicios que


la empresa brinda a sus empleados, con el propósito de mantener o recuperar la salud y
evitar el deterioro de su capacidad económica. Estos beneficios pueden ser financiados en
forma parcial o total, o en algunos casos gratuitos, además constituyen medidas indispensa-
bles para mantener la fuerza laboral satisfecha y productiva. Ejemplo: bonificaciones, seguro
de vida, premios por producción, servicio de transporte, recreación, subsidios familiares,
entre otros.

Higiene y seguridad: es uno de los puntos clave de cualquier organización. Es parte


de su responsabilidad social cuidar a sus empleados, protegiéndolos de accidentes y asegu-
rarles un ambiente saludable. Dentro de las necesidades que el empleador debe satisfacer
durante la vida laboral de un trabajador, se encuentran las necesidades de seguridad física
y emocional. Ver tema Higiene y Seguridad Industrial.

AUTORIDAD DE LÍNEA EN LA ADMINISTRACIÓN DE RRHH


La autoridad de línea en la administración de RRHH se encarga de:
•• Colocar a la persona indicada en el puesto adecuado.
•• Iniciar a los nuevos empleados en la organización (inducción).
•• Capacitar a los empleados para los puestos.
•• Obtener cooperación creativa y desarrollar relaciones de trabajo.
•• Mejorar el desempeño en el trabajo de los empleados.
•• Interpretar las políticas y procedimientos de la empresa.
•• Controlar los costos de mano de obra.
•• Desarrollar las capacidades potenciales de cada persona.
•• Mantener la moral de la empresa.
•• Proteger las condiciones de salud de los empleados.

Se puede afirmar que la mayoría de las organizaciones consideran sus recursos huma-
nos como elementos valiosos y en muchas ocasiones irremplazables. Es por ello, que su
gestión se convierte en un asunto estratégico y crucial, vinculándose directamente con los
resultados de la gestión empresarial.

427
Sánchez Marchán, Nanci. (2014).Introducción a la Ingeniería Industrial, Material Instruccional de Apoyo. Caracas: UNA

ECONOMÍA Y FINANZAS
ECONOMÍA
•• Introducción a la economía.
•• Definición de la economía.
•• Campos de la economía.
- Microeconomía.
- Macroeconomía.

INTRODUCCIÓN A LA ECONOMÍA
Desde el siglo XV hasta el siglo XVII, se escribieron numerosos ensayos sobre econo-
mía, pero fue Adam Smith (considerado el padre de la economía) quien escribió el tratado
más completo sobre esta materia, denominado “la naturaleza y causas de las riquezas de
las naciones”, investigación que generó lo que más tarde se llamó (a pesar de que Smith
era escocés) la Escuela de Economía Política Inglesa.

El objetivo de estudio de la economía es la actividad humana, por tanto, es una ciencia


social. Usa una modalidad diferente del método científico, un conjunto de técnicas desarro-
lladas por algunos economistas investigadores.

DEFINICIÓN DE ECONOMÍA

La economía se deriva del griego: οἰκονομία que significa “administración de una casa
o familia”, de οἶκος que representa la “casa en el sentido de patrimonio” y νέμω que quiere
decir “administrar”.

La economía se puede definir como:

Una ciencia social que estudia de forma racional los procesos de producción, intercam-
bio, distribución, costos, beneficios y consumo de bienes y servicios.

Otra definición propuesta por Lionel Robbins en 1932 establece que: La economía es
una ciencia que analiza el comportamiento humano como una relación entre fines
dados y medios escasos que tienen usos alternativos.

La materia prima de trabajo de los economistas es un conjunto de recursos escasos


que pueden satisfacer usos alternativos. Para asignar los recursos a la producción de bienes
y servicios, deben existir ciertos criterios que permitan hacerlo de la manera más racional y
eficiente posible.

429
Sánchez Marchán, Nanci. (2014).Introducción a la Ingeniería Industrial, Material Instruccional de Apoyo. Caracas: UNA

CAMPOS DE LA ECONOMÍA
La económica se divide en dos grandes campos: la microeconomía y la macroecono-
mía.

La microeconomía

Estudia el comportamiento económico de pequeñas unidades de decisión: personas, em-


presas, trabajadores, propietarios de tierras, consumidores, productores, así como, de
los mercados. Explica cómo se determinan variables referidas a los precios de bienes
y servicios, el nivel de salarios, el margen de beneficios y las variaciones de las rentas.

Variables de la microeconomía

Las variables microeconómicas son:

1. El consumidor: anticipar la elección partiendo de sus preferencias individuales,


ante la gama de bienes y servicios que se le ofrecen, y entre los que puede optar
según los recursos disponibles.
2. La demanda: cantidad y calidad de productos, servicios y bienes en general,
que se pueden adquirir de manera individual o en conjunto, a precios que varían
en el mercado en un momento dado. La demanda constituye una función mate-
mática y puede ser individual, total o de mercado.
3. El productor: desarrollada por las empresas cuando planifican, coordinan y
supervisa la tarea de producción, eligiendo la combinación de estos elementos
para aumentar los beneficios.
4. La estabilidad: trata de explicar las relaciones desde la unidad hasta el con-
junto. En la actualidad, requiere el uso de computadoras, para trabajar en la
búsqueda de soluciones más eficientes en este campo.
5. Elección racional (último aporte): las personas se inclinan por elegir lo bueno
y descartar lo que las perjudica, guiándose por la intuición, traducida luego en
razonamiento. Este principio se adapta tanto al escenario social como al político;
en ambos la tendencia es evitar los riesgos y maximizar los beneficios.
6. Mercados de activos financieros: se presentan cuatro tipos o modelos de
estructuras de mercado:
•• Monopolio: una sola firma que produce un bien o servicio determinado.
•• Oligopolio: varias firmas que producen el mismo bien o servicio.
•• Competencia monopolística: las empresas pueden competir libremen-
te con productos semejantes pero no iguales, es decir, cada una pro-
duce su propia versión.
•• Competencia perfecta: producción y oferta de un mismo bien o servi-
cio por varias empresas que ejercen influencia en el usuario y compi-
ten por su demanda.

430
Sánchez Marchán, Nanci. (2014).Introducción a la Ingeniería Industrial, Material Instruccional de Apoyo. Caracas: UNA

Indicadores microeconómicos

En al área microeconómica, los principales indicadores son:


•• Base monetaria: suma de dinero en manos del público más reservas
bancarias.
•• Oferta monetaria: cantidad de dinero del que dispone la economía.
•• Tasas de interés: porcentaje al cual esta invertido un capital en una
unidad de tiempo.
•• Precios: pago asignado a la obtención de un bien o servicio.
•• Índice de precios al consumidor (IPC): se valoran los precios de un con-
junto de productos (canasta familiar).
•• Índice de precios al productor (IPP): medida en el porcentaje de cam-
bio a través del tiempo del costo de los productos de la canasta familiar.
•• Salario mínimo: límite fijado por un gobierno para asegurarle al
trabajador un ingreso mínimo.
•• Tasa de desempleo: es la proporción de la población que se encuentra
desocupada.
•• Producto Interno bruto per cápita (PIBpc): relación que existe entre el
producto interno bruto (PIB) y la cantidad de habitantes de un país.

La microeconomía es un medio fundamental para el estudio de la teoría económica,


puesto que aporta estudio y conocimiento acerca de las unidades económicas individuales,
estudios que sirven como base para que otras áreas, como la macroeconomía desarrollen
sus principios.

La macroeconomía

Se encarga del estudio global de la economía en términos del monto total de bienes y
servicios producidos, el total de los ingresos, el nivel de empleo, de recursos productivos
y el comportamiento general de los precios.

Este campo no realiza estudios sobre decisiones individuales, como lo hace la mi-
croeconomía, pero es fundamental que sea coherente con estas decisiones individuales,
puesto que las tendencias globales de las cuales se encarga resultan de la suma de las
tendencias individuales.

Para alcanzar estos objetivos, la macroeconómica se basa en los siguientes variables


e indicadores:

Variables:
•• Política fiscal: considera los ingresos (impuestos, tasas, entre otros) y
gastos (servicios públicos, salarios, jubilación, subsidios) de una nación.
Cuando los gastos son mayores a los ingresos, se genera un déficit fis-
cal.

431
Sánchez Marchán, Nanci. (2014).Introducción a la Ingeniería Industrial, Material Instruccional de Apoyo. Caracas: UNA

•• Política monetaria: regula la oferta monetaria o cantidad de dinero del


que dispone la economía (responsabilidad del banco central).
•• Política exterior: se fundamenta en la política comercial y el tipo de
cambio que determina los precios de los bienes y servicios extranjeros
en comparación con los nacionales, así como los precios nacionales a
través de la importación
•• Política de rentas: persigue la distribución equitativa de la renta, así
como el control de la inflación, para mejorar la calidad de vida.

Indicadores macroeconómicos:

Producción:
•• El Producto Interno Bruto (PIB): lo que se genera dentro del país, en
términos geográficos y es igual a la producción nacional bruta (PNB) –
pagos netos a factores en el exterior.
•• Producto Nacional Bruto (PNB): son medidas importantes relaciona-
das con la producción, la tasa de desempleo. PNB = Ingresos + depre-
ciaciones + impuestos indirectos – subsidios.
•• Producto Nacional Neto (PNN): mide el rendimiento real o neto de la
actividad económica de un país, permite conocer el crecimiento de la
producción eliminando las depreciaciones para reponer el capital fijo que
ya existe. PNN = PNB – Depreciaciones.

Inflación:
•• La tasa de inflación, facilita el estudio de los precios, es decir, índice ge-
neral de precios al consumo (IPC).

Comercio internacional
•• La balanza comercial, permite desarrollar un análisis sobre el comercio
internacional (importaciones, exportaciones).
•• Tipo de cambio, precio de la moneda propia, en términos de otras mo-
nedas ($, €, entre otras).
•• Divisa, moneda en billetes de otro país admitida en el nuestro a cotiza-
ción oficial.
•• Mercado de divisas, por efecto de la oferta y la demanda se determina
un precio de las divisas (el tipo de cambio).
Uno de los principales objetivos de la macroeconomía de cualquier país es buscar la
satisfacción de las necesidades de la población, lo cual se logra por medio de la producción
de bienes y servicios.

Partiendo de la teoría macroeconómica se establece el ambiente dentro del cual se


ejerce la función financiera.

432
Sánchez Marchán, Nanci. (2014).Introducción a la Ingeniería Industrial, Material Instruccional de Apoyo. Caracas: UNA

FINANZAS
•• Introducción a las finanzas.
•• Tipos de finanzas.
•• Planeación financiera.
•• Objetivos finales de la política fiscal.
•• Mecanismos de control para la política fiscal.
•• Tipos de política fiscal.
•• Valor del dinero en el tiempo.
•• Costos de la inversión.
•• Estados financieros.

INTRODUCCIÓN A LAS FINANZAS


Las finanzas formaron parte durante mucho tiempo de la economía, se separaron como
un campo de estudios independiente a principios del siglo XX. Inicialmente se relacionaron
solamente con los documentos, instituciones y aspectos de procedimiento de los mercados
de capital. Con el avance de las innovaciones tecnológicas y las nuevas industrias surgió la
necesidad de mayor cantidad de fondos, impulsando el estudio de las finanzas para destacar
la liquidez y el financiamiento de las empresas. La atención se centró más bien en el funcio-
namiento externo que en la administración interna.

Una empresa requiere de muchos activos tangibles como estructuras físicas, máquinas,
vehículos, entre otras y de activos intangibles como conocimiento técnico, marcas comercia-
les, patentes, entre otros. Para obtener estos activos requiere de una inversión o presupues-
to de capital de la empresa que es obtenido generalmente a través de financiamiento.

Se puede definir las finanzas como:

Una rama de la administración y económica, que se preocupa del tema relacionado con
la obtención y gestión del dinero, recursos o capital por parte de una persona, empresa o
del estado. Se refieren a la forma en como se obtienen los recursos, a la forma en como
se gastan o consumen, a la forma en como se invierten, pierden o rentabilizan.

El término finanzas proviene del latín “finis”, que significa acabar o terminar. Las finan-
zas tienen su origen en la finalización de una transacción económica, es decir, con la trans-
ferencia de dinero se acaba la transacción financiera.

Las finanzas están íntimamente relacionadas con economía, administración, así


como, con la contabilidad.

433
Sánchez Marchán, Nanci. (2014).Introducción a la Ingeniería Industrial, Material Instruccional de Apoyo. Caracas: UNA

TIPOS DE FINANZAS
Cuando las finanzas son utilizadas por las personas se denominan finanzas personales;
por los gobiernos, finanzas públicas y por los negocios, finanzas corporativas.

Las metas de cada uno de los tipos de finanzas antes mencionados, se alcanzan con
el uso de instrumentos financieros apropiados, con la consideración a su ajuste institucional.
Las finanzas son uno de los aspectos más importantes de la gerencia de negocio. Sin el
planeamiento financiero apropiado es poco probable que una nueva empresa tenga éxito.

Recientemente se está investigando con gran alcance, un nuevo tipo de finanzas lla-
madas, finanzas prospectivas que tratan de verificar la eficacia de los modelos de gestión
actuales sobre la base de las riquezas futuras de las empresas, los gobiernos y las personas.

Dentro de los servicios financieros se encuentran la banca e instituciones relacionadas.

PLANEACIÓN FINANCIERA

La planeación financiera se fundamenta en la reunión, clasificación, análisis e interpre-


tación de la información financiera, con el objeto de crear un plan combinado de las necesi-
dades que genera éste, así como las posibles consecuencias de sus operaciones.

Las finanzas son un reflejo de la realidad operativa de la empresa, se man-


tienen sanas y tienden a ser cada vez mejores, cuando se consume con
atención, se ahorra con eficacia y se invierte con cuidado.

Planeación financiera personal

Proceso a través del cual una persona analiza su situación financiera, establece sus ob-
jetivos y diseña planes de acción, que le permitan elaborar un presupuesto personal ajustado
a sus necesidades.

Entre los planificadores financieros personales se encuentran:

Inversiones:
•• Corredores de bolsa. Ayudan a los clientes a elegir, comprar y vender
valores.
•• Analista de valores. Investiga las obligaciones y acciones, por lo gene-
ral de industrias específicas, proporcionando asesoría a las empresas de
valores y de seguros.
•• Gerentes de cartera. Establecen y administran la cartera de valores
para empresas e individuos.

434
Sánchez Marchán, Nanci. (2014).Introducción a la Ingeniería Industrial, Material Instruccional de Apoyo. Caracas: UNA

•• Ejecutivos de banco de inversión. Asesoran en las emisiones de va-


lores y actúan como intermediarios entre éstos y los compradores de
obligaciones.
Bienes raíces: se refiere a la compra, venta y arrendamiento de propiedades. Se en-
cargan de ello:
•• Agentes o corredores de bienes raíces. Informan respecto a la venta
y arrendamiento de propiedades residenciales y comerciales, realizando
las negociaciones pertinentes.
•• Valuadores. Calculan el valor en el mercado de todo tipo de propieda-
des.
•• Prestamistas en bienes raíces. Son aquellos que recomiendan y to-
man decisiones con respecto a solicitudes sobre préstamos.
•• Banqueros hipotecarios. Son los individuos que buscan y arreglan fi-
nanciamiento para proyectos de bienes raíces.
•• Gerentes de propiedades. Son las personas que dirigen operaciones
diarias de las propiedades, con el fin de obtener los máximos rendimien-
tos para el propietario.

Seguros: representan el resguardo de la salud y la protección de los bienes materiales,


contra un siniestro, a través de un monto estipulado de dinero, que puede cubrir el aconte-
cimiento.
•• Agentes o corredores de seguros. Son los profesionales que crean
programas de seguros y venden pólizas para satisfacer las necesidades
de los clientes. Además se encargan de cobrar las primas y colaboran en
el procesamiento y la liquidación de deudas.

Finanzas para la administración: se relacionan con las obligaciones del gerente de fi-
nanzas en una empresa. Administran de manera activa los asuntos financieros de empresas
públicas o privadas, entre ellos se encuentran:
•• Analista financiero. Es el responsable principalmente de preparar los
planes y los presupuestos financieros de cualquier tipo de empresa.
•• Analista o gerente de presupuesto. Es el que se encarga de la evalua-
ción y recomendación de activos propuestos, participa en los aspectos
financieros de la ejecución de inversiones aprobadas.
•• Gerente de proyectos de finanzas. Arregla el financiamiento para las
inversiones de activos aprobados y coordina a los consultores, a los ban-
queros de inversiones y a los asesores legales.
•• Gerente de administración de efectivo. Responsable del manteni-
miento y control de los saldos diarios de efectivo de la empresa, dirige
el cobro de efectivo, las actividades de desembolso y las inversiones y

435
Sánchez Marchán, Nanci. (2014).Introducción a la Ingeniería Industrial, Material Instruccional de Apoyo. Caracas: UNA

préstamos a corto plazo de la empresa. Además coordina las relaciones


bancarias.
•• Analista o gerente de crédito. Es el que administra las políticas de
crédito de la empresa, por medio de la evaluación de las solicitudes de
crédito, la concesión de créditos, la supervisión y recaudación de las
cuentas por cobrar.
•• Gerente del fondo de pensiones. Individuo responsable de la vigilancia
y administración de los activos y pasivos del fondo de pensiones de los
empleados.

Actividad propuesta N° 1
De acuerdo con Piñango (2003) “La diferencia entre el éxito y el fracaso financiero
depende, no sólo de la información disponible, sino también de la sabiduría que
se utilice al lidiar con esa información”.
Considerado está aseveración, el autor plantea que existen cuatro tipos de perso-
nas financieras y son las siguientes:
a. Los que se preocupan por sacar el mayor rendimiento, se ocupan de hacerlo
y saben hacerlo.
b. Los que se preocupan por su dinero y se ocupan, pero no saben hacerlo.
c. Los que se preocupan, pero no se ocupan ni saben hacerlo.
d. Los que no se preocupan, no se ocupan ni saben hacerlo.
¿Con cuál tipo de persona financiera se identifica?

Actividad propuesta N° 2
¿Cuáles son los diferentes instrumentos financieros utilizados por los inversionistas
venezolanos?

Planeación financiera pública

Para realizar la planeación financiera pública se debe considerar la política fiscal, tam-
bién conocida como finanzas públicas. Es una política económica que usa el gasto público y
los impuestos como variables de control para asegurar y mantener la estabilidad económica,
cuando existe un déficit o superávit. Por lo tanto, es una política en la cual el Estado participa
activamente, a diferencia de otras como la política monetaria.

Objetivos finales de la política fiscal:


•• Plena ocupación (mayor nivel de empleo posible).
•• Control de la demanda agregada mediante los impuestos y el gasto pú-
blico.
•• Controlar un déficit o un superávit.

436
Sánchez Marchán, Nanci. (2014).Introducción a la Ingeniería Industrial, Material Instruccional de Apoyo. Caracas: UNA

Mecanismos de control para la política fiscal: Los dos mecanismos de control suge-
ridos para la política fiscal son:
•• Variación del gasto público.
•• Variación de los impuestos.

En este sentido, es de suma importancia el control de la inversión pública. Pero si hay


que elegir entre hacer que el Estado gaste más o bajar los impuestos, los políticos suelen
preferir lo segundo, porque se revierte rápidamente.
•• T: tasas (impuestos), dinero que pagan los ciudadanos
•• G: gasto público, dinero que gasta el Estado para obras, etc.
•• T=G: punto de equilibrio en el cual hay plena ocupación.

Los partidarios de la política fiscal creen que en vez de intentar estar en el punto de
equilibrio P, es recomendable aumentar el gasto público para incentivar la economía. Por
tanto cuando G>T hay déficit.

Tipos de política fiscal

Puede ser expansiva o restrictiva:


•• Política fiscal expansiva: cuando el objetivo es estimular la demanda
agregada, en el momento que la economía está atravesando un período
de recesión y necesita un impulso para expandirse. Como resultado se
tiende al déficit o incluso se puede contraer la economía.

Demanda agregada: representa la cantidad de bienes y servicios que los habitantes,


las empresas, las entidades públicas y el resto del mundo desean y pueden consumir en el
país, un nivel determinado de precio. La curva de demanda agregada tiene pendiente nega-
tiva. Si suben los precios la gente querrá comprar menos y si bajan querrán comprar más.

•• Política fiscal restrictiva: cuando el objetivo es frenar la demanda agre-


gada, cuando la economía está en un período de excesiva expansión y
tiene necesidad de frenarse por la enorme inflación que está creando.
Como resultado se tiende al superávit.

Planeación de las finanzas corporativas

Las finanzas corporativas (en inglés Corporate Finance) son un área de las finanzas
que se centra en la forma en la que las empresas pueden crear valor y mantenerlo a través
del uso eficiente de los recursos financieros.

437
Sánchez Marchán, Nanci. (2014).Introducción a la Ingeniería Industrial, Material Instruccional de Apoyo. Caracas: UNA

Para iniciar un plan de finanzas corporativas se debe establecer la misión, es decir, lo


que se quiere alcanzar y para qué. Posteriormente definir los objetivos a largo o corto plazo,
lo cual permitirá dar sentido a las actividades financieras que se van a realizar. El plan debe
ser realista para aprovechar las oportunidades y evitar las amenazas que se presenten en
el entorno.

Las finanzas corporativas se centran en cuatro tipos de decisiones:


•• Las decisiones de inversión: que se centran en el estudio de los acti-
vos reales (tangibles o intangibles) en los que la empresa debería invertir.
•• Las decisiones de financiación: que estudian la obtención de fondos
(provenientes de los inversores que adquieren los activos financieros
emitidos por la empresa) para que la compañía pueda adquirir los activos
en los que ha decidido invertir.
•• Las decisiones sobre dividendos: deben balancear aspectos cruciales
de la entidad. Por un lado, implica una remuneración al capital accionario
y por otro supone privar a la empresa de recursos financieros.
•• Las decisiones directivas: que atañen a las decisiones operativas y
financieras del día a día.

Ejemplo Ilustrativo N° 1
Cuando se realiza la venta de acciones de una compañía a inversionistas insti-
tucionales como los bancos de inversión, que luego la venden generalmente al
público. La acción le proporciona a la persona que la adquiere una parte de esa
compañía.
Es decir, si una persona compra una parte de la compañía BACI C.A., la cual
tiene 100 partes originales (sostenidas por los inversionistas), esa persona es
1/100 dueño de esa compañía.

El objetivo básico de las finanzas corporativas es maximizar el valor o la riqueza


para los accionistas o propietarios, en tal sentido uno de los temas fundamenta-
les se concentra en medir la contribución de una determinada decisión al valor
del accionista. Para responder a esta cuestión se han creado las técnicas de
valoración de activos

Se puede decir entonces, que las finanzas son uno de los aspectos más importantes
de la gerencia de negocio. Sin el planeamiento financiero apropiado una nueva empresa es
poco probable que sea acertada. El manejo de dinero (un activo líquido) es esencial para
lograr un futuro seguro, para el individuo y una organización.

El inversionista prudente diversifica su inversión total, repartiendo sus recursos entre


varias inversiones distintas. El efecto de diversificar es distribuir el riesgo y así reducir el
riesgo total.

438
Sánchez Marchán, Nanci. (2014).Introducción a la Ingeniería Industrial, Material Instruccional de Apoyo. Caracas: UNA

Otros ejemplos ilustrativos de finanzas:

Ejemplo Ilustrativo N° 2
¿Cuáles son los inconvenientes de los modelos de planeación financiera que se
deben tomar en cuenta?
Respuesta
Los inconvenientes más importantes que pueden surgir en los modelos de
planeación financiera son:
• No indican cuales políticas financieras son los mejores, es decir no toma la
decisión de emitir capital nuevo para lograr una tasa de crecimiento mayor.
• Considerados modelos muy sencillos, ya que los costos no siempre son pro-
porcionados a las ventas.
• Sin alguna clase de planeación financiera a largo plazo, se producirá un cam-
bio de dirección.

Ejercicio Ilustrativo N° 3
¿Por qué se genera la inflación?
Respuesta
Las causas de la inflación son variadas y dependen de muchísimos factores. Por
otro lado, no todos los economistas se ponen de acuerdo en este tema. Se expli-
carán de manera sencilla las causas más importantes:
• Que el nivel de ingresos del país aumente, con lo cual la gente gasta más di-
nero en consumir. Así, como hay más compradores, mientras los productores
fabrican más para satisfacer esa demanda, los vendedores suben los precios
de los productos.
• Que los empresarios quieran aumentar su cuota de beneficios, para lo cual,
si hay una buena demanda, pueden aumentar los precios y por lo tanto ganar
más sin disminuir sus ventas.
• Que algunos factores necesarios para la producción incrementen su precio,
por ejemplo, que suba la gasolina o que los trabajadores exijan subida de
salarios. El empresario repercutirá esas subidas en los precios de bienes o
servicios.

Ejercicio Ilustrativo N° 4
¿Cuál es la Importancia de las finanzas internacionales y mencione algunos
ejemplos de la globalización?
Respuesta
Las finanzas internacionales son de gran importancia ya que ayudan a:
1. Decidir la manera en como los eventos internacionales pueden perturbar a
una empresa.
2. Considerar los pasos que pueden tomarse para explotar los desarrollos positi-
vos que pueden aislar a la empresa de los cambios dañinos, como por ejemplo
la variación del tipo de cambio, el valor de los activos, entre otros.

439
Sánchez Marchán, Nanci. (2014).Introducción a la Ingeniería Industrial, Material Instruccional de Apoyo. Caracas: UNA

Ejercicio Ilustrativo N° 5
¿Por qué el patrón Oro favorece la estabilidad de los precios?
Respuesta
Porque al ser considerado como base monetaria internacional, cada país esta-
blece el valor de su unidad monetaria en términos del oro y existe una paridad
(equivalencia).
¿Por qué la acumulación del oro no aumenta la riqueza del país?
Respuesta
Debido a que este metal constituye la base monetaria del país, por lo tanto la
acumulación de oro desencadena una reducción en la cantidad de dinero en la
economía para mantener un equilibrio.

Ejemplo Ilustrativo N° 6
Suponga que durante una semana entran a Venezuela 217.000.000 millones de
dólares de la inversión extranjera en cartera.
a) ¿Qué sucedería con la oferta monetaria si el banco Central de Venezuela no
hiciera nada?
Respuesta
Habría un aumento considerable en el dinero circulante en la economía, lo que
desencadenaría una inflación, aumentando los precios internos y reduciendo las
tasas de interés devaluándose la moneda.
b.) ¿Cómo puede el Banco Central de Venezuela controlar esta entrada de capital?
Respuesta
Vendiendo bonos internos en el mercado abierto para recoger el exceso de circu-
lante y mantener la base monetaria sin cambio.

Ejercicio Ilustrativo N° 7
Explique por qué las políticas de empobrecimiento de un país vecino pueden per-
turbar a los demás.
Respuesta
Estas políticas de empobrecimiento tienen que ver con el sistema de restricciones
al comercio e incluye también altos aranceles, cuotas y restricciones administra-
tivas además de medidas proteccionistas que desalientan el comercio internacio-
nal, es decir, el comercio en el resto de los países se estanca originando pérdidas
considerables por la disminución del intercambio comercial.

Ejercicio Ilustrativo N° 8
Describa el ajuste real bajo el patrón oro.
Respuesta
Al haber un déficit en la balanza de pagos o salida del oro, el banco Central vende
activos internos y reduce la base monetaria, de esta manera aumentan las tasas
de interés y reducen los precios internos haciendo favorable la inversión y capta-
ción de capitales.

440
Sánchez Marchán, Nanci. (2014).Introducción a la Ingeniería Industrial, Material Instruccional de Apoyo. Caracas: UNA

Ejercicio Ilustrativo N° 9
¿Cuál fue el papel original del FMI?
Respuesta
Su labor era la de garantizar el cumplimiento de las normas acordadas en lo refe-
rente al comercio y las finanzas internacionales y establecer facilidades de crédito
para los países con dificultades temporales de balanza de pagos.

Ejercicio Ilustrativo N° 10
Enumere las ventajas de la inversión extranjera directa para una economía
abierta basada en el mercado.
Respuesta
a) Genera empleo.
b) Crecimiento económico.
c) Transferencia de tecnología.
d) Mayor competitividad internacional.
e) Mejora la infraestructura.

Todas las organizaciones, empresas, grupos o personas necesitan en al-


gún momento obtener fondos para financiarse, estos fondos comúnmente
son muy difíciles de conseguir y cuando se logran alcanzar, las entidades o
personas que realizan estos préstamos cobran una retribución por el tiem-
po que el dinero está en manos de sus deudores.

Las finanzas trabajan con activos financieros o títulos valores e incluso bonos, acciones
y préstamos otorgados por instituciones financieras, que forman parte de los elementos fun-
damentales de la matemática financiera, la cual se puede definir como:

La aplicación de la matemática a las finanzas, centrándose en el estudio del valor del


dinero en el tiempo, a través de métodos de evaluación que permitan tomar decisiones.

La matemática financiera se deriva de la matemática aplicada, que estudia el valor del


dinero en el tiempo, combinando el capital, la tasa y el tiempo para obtener un rendimiento o
interés, a través de métodos de evaluación que permiten tomar decisiones de inversión. De
esta matemática surge también el término Ingeniería Económica.

Se puede definir la Ingeniería Económica como:

Un conjunto de técnicas para tomar decisiones de índole económica en la industria pro-


ductora de bienes y servicios, considerando siempre el valor del dinero a través del tiem-
po.

441
Sánchez Marchán, Nanci. (2014).Introducción a la Ingeniería Industrial, Material Instruccional de Apoyo. Caracas: UNA

VALOR DEL DINERO EN EL TIEMPO


El dinero representa un papel elemental en la vida diaria de las personas. Se trabaja
entre otras razones por dinero, se compra con dinero y se vende por dinero. Se asevera que
casi todo se puede comprar con el dinero. El dinero solventó las dificultades que tenía el
trueque para poder realizar los intercambios de bienes y servicios en la sociedad. Se puede
definir entonces el dinero como el sistema de cambio aceptado para el pago de bienes y
servicios, así como para la amortización de deudas.

El dinero también sirve como medida del valor para estimar el precio económico relativo
de los distintos bienes y servicios. El número de unidades monetarias requeridas para com-
prar un bien se denomina precio del bien.

La función del dinero como medio de cambio y la medida del valor facilitan
el intercambio de bienes y servicios y la especialización de la producción.

El significado de valor del dinero en el tiempo, revela que una unidad de dinero hoy
vale más que una unidad de dinero en el futuro. Esto sucede porque el dinero de hoy puede
ser invertido, ganar intereses y aumentar en valor nominal.

El valor del dinero en el tiempo es denominado interés, esto es una medida del incre-
mento entre la suma originalmente prestada o invertida y la cantidad final debida o acumu-
lada. Es decir, es el costo pagado por su uso en un período determinado y expresado como
un índice porcentual.

Cuando se invierte dinero el Interés se expresa como sigue:

Interés = cantidad acumulada - cantidad original.

Si se presta dinero el interés que se genera es:

Interés =cantidad debida - préstamo original

En cualquiera de los dos casos el aumento sobre la cantidad de dinero original es el


interés y la inversión o préstamo original se llama capital.

Considerando lo antes expuesto, se puede definir el interés como la cuantificación del


valor del dinero en el tiempo.

Cuando el interés se expresa como un porcentaje de la cantidad original por unidad de


tiempo, el resultado es una tasa de interés.

Tasa de Interés

interés acumulado por unidad de tiempo x 100


Tasa de Interés (TIR) = cantidad original

442
Sánchez Marchán, Nanci. (2014).Introducción a la Ingeniería Industrial, Material Instruccional de Apoyo. Caracas: UNA

Ejemplo Ilustrativo
La Compañía KASA S.A., invirtió Bs. 10.000 el 01 de junio y retiró Bs.10.600
exactamente (1) año más tarde. Calcule el interés obtenido y la tasa de interés de
la inversión original.
Solución:
Interés = cantidad acumulada - inversión original
Interés= 10.600 - 10.000= Bs. 600
Tasa de interés = interés acumulado por unidad de tiempo x 100
cantidad original

600 x 100 = 6%
Tasa de Interés = 10.000

La tasa de interés constituye el precio que la empresa o persona debe pagar por dispo-
ner de cierta suma de dinero en el presente, para devolver una suma mayor en el futuro, o la
inversión en el presente compensará en el futuro una cantidad adicional invertida.

La unidad de tiempo usada para expresar la tasa de interés debe estar identificada y
denominada como período de interés.

CLASIFICACIÓN DE LAS TASAS DE INTERÉS


Considerando el valor del dinero en el tiempo

Cuando en una economía se producen cambios en los niveles generales de precios


es importante diferenciar entre variables nominales y variables reales, concibiéndose las
siguientes tasas de interés: Tasa de interés (nominal, real, efectiva)

Tasa de interés nominal. Se refiere al regreso de los ahorros en términos de la can-


tidad de dinero que se obtiene en el futuro (un tiempo determinado) para un monto dado de
ahorro reciente.

Tasa de interés real. Mide el regreso de los ahorros en términos del volumen de bienes
que se pueden adquirir en el futuro con un monto dado de ahorro reciente.

Tasa efectiva: es aquella que se obtiene a partir de una tasa nominal y considera el
efecto de la capitalización, con respecto a la inflación.

La inflación, es la subida generalizada del precio de los bienes y servicios, tanto de los
utilizados para el consumo, como los que genera la producción. El dinero pierde poder de
compra, es decir, que con el mismo dinero, se pueden comprar menos cosas. Una forma de
medir la inflación o subida de precios es a través del IPC: Índice de Precios al Consumi-
dor (ver tópico de microeconomía y macroeconomía), el cual se organiza sobre productos
de varios sectores seleccionados, sobre el volumen de importancia en los presupuestos
familiares.

443
Sánchez Marchán, Nanci. (2014).Introducción a la Ingeniería Industrial, Material Instruccional de Apoyo. Caracas: UNA

La fórmula simplificada para obtener la tasa de interés real es:

Tasa de Interés Real (TIR)= Tasa de interés nominal – tasa de inflación

Ejemplo Ilustrativo Nº 1
Supóngase que un ahorrista realiza un depósito de 100 Bs en un banco que se
compromete a pagar un interés nominal anual del 10% al final del período. Para el
mismo tiempo, se espera que la inflación anual sea baja, digamos del 0,5%. Dicho
ahorrista espera obtener a fin de año 110 Bs. Dado que la inflación es baja, la tasa
nominal (10%) es prácticamente igual a la tasa real (tasa nominal–inflación), es de-
cir (9,5%), por lo tanto, la suma que obtendrá el ahorrista en términos de capacidad
de compra no se verá prácticamente afectada.

Otra Variedad de Tasas:

De acuerdo a la nomenclatura bancaria:


•• Tasa activa: es aquella que se aplica a las operaciones de colocación
de fondos vía prestamos (descuentos, créditos ordinarios, créditos hipo-
tecarios, etc.)
•• Tasa pasiva: es la tasa que el banco paga a los depositantes o ahorris-
tas por la captación de depósitos (ahorros, cuentas corrientes, depósitos
a plazo, etc.)

De acuerdo al momento en que se cobran los intereses:


•• Tasa vencida: es aquella tasa que se aplica al vencimiento del plazo de
la operación pactada, es un cálculo racional pues presupone el paso del
tiempo como requisito para el cobro de intereses.
•• Tasa adelantada: es la tasa que se descuenta del capital antes del
transcurso del tiempo. Determina en cuanto disminuye un valor nominal
de un título valor (valor actual)

De acuerdo al cumplimiento de la obligación:


•• Tasa compensatoria: es la contraprestación por el uso del dinero, es la
tasa corriente tanto para operaciones de crédito como de captación de
fondos.
•• Tasa moratoria: es aquella que se aplica al incurrir el prestatario en
atraso con respecto al pago de sus obligaciones.

444
Sánchez Marchán, Nanci. (2014).Introducción a la Ingeniería Industrial, Material Instruccional de Apoyo. Caracas: UNA

TIPOS DE INTERÉS (INTERÉS SIMPLE E INTERÉS COMPUESTO)


Interés simple. Es el que se obtiene cuando los intereses producidos, durante todo el
tiempo que se mantenga una inversión, se deben únicamente al capital inicial.

El interés simple se calcula usando sólo el capital, ignorando cualquier interés que pue-
de haberse generado o acumulado en períodos anteriores. El interés simple puede calcular-
se usando la siguiente expresión.

Interés Simple (IS) = c * t * i


C= Capital t (Nº de Periodos) i (Tasa de Interés)
i  está expresado en tanto por uno y  t  en años

El interés (I)  que produce un capital es directamente proporcional al capital inicial (C), al
tiempo  (t), y a la tasa de interés (i).

Ejemplo Ilustrativo Nº 2
Calcular el interés simple de:
a) 2.500 Bs durante 8 meses al 8%.
b) 6.000 Bs durante 63 días al 9%.

Solución
a) C = 2.500 Bs., t = 8 meses e i= 8% / 100 = 0.08 (tanto por 1)
t = 8meses x1año /12 meses =0,66 años
IS = 2.500 * 0,66 * 0.08 =132 Bs.
b) I S =6.000 Bs., t = 63 días, i =0,09.
t = 63 días x1año / 365 días = 0,17 años.
IS =6.000 * 0.17 * 0.09 = 918 Bs.

Ejemplo Ilustrativo Nº 2
Compañía HRP invirtió 1000 Bs a 3 años con un interés anual del 14%. Determi-
nar el interés simple.

Solución
IS = C * i  * t
IS = (1000) *(3) * (14%)
IS = 1420 Bs.

445
Sánchez Marchán, Nanci. (2014).Introducción a la Ingeniería Industrial, Material Instruccional de Apoyo. Caracas: UNA

Ejemplo Ilustrativo Nº 3
Calcular a cuánto asciende el interés simple producido por un capital de 25. 000
bolívares invertidos durante 4 años a una tasa del 6 % anual.

Solución
IS = C * i  * t
IS = 25 000 * 0,06 * 4 = 6 000 Bs.

Ejemplo propuesto
Al cabo de un año, un banco ha ingresado en una cuenta de ahorro, por concepto
de intereses, 970 Bs. La tasa de interés de una cuenta de ahorro es del 2 %. ¿Cuál
es el saldo medio (capital) de dicha cuenta en ese año?

INTERÉS COMPUESTO
Interés compuesto. Es el que se logra cuando al capital se le suman periódicamente
(en general, los períodos son anuales) los intereses producidos. Así, al final de cada período,
el capital que se tiene es el capital anterior más los intereses producidos por éste.

Cuando se calcula el interés compuesto, el interés de un período se calcula sobre la


cantidad más elevada que se obtiene del interés ganado o acumulado, en períodos anterio-
res.

Interés Compuesto (IC)


Al transcurrir n  años el capital inicial (C) o Valor Presente (VP), invertido en la modalidad
de interés compuesto se convertirá en un capital final (Cn) o Valor Futuro (VF).

VF= capital final (Cn) = C (1+i)n


Puesto que el interés es la diferencia entre el capital final y el inicial:

Cn
La tasa de interés compuesta es: Ic = n −1
C

El capital, cuando los períodos son diferentes a 1 año se calcula como sigue: Cn= C(1+i)n

446
Sánchez Marchán, Nanci. (2014).Introducción a la Ingeniería Industrial, Material Instruccional de Apoyo. Caracas: UNA

Ejemplo Ilustrativo Nº 4
Averiguar en qué se convierte un capital de 1.200.000 Bs al cabo de 5 años a una
tasa de interés compuesto anual del 8 %.

Solución:
Aplicando la fórmula Cn = C (1 + i )n
C5 = 1.200.000 (1 + 0,08)5 = 1.200.000 x 1,4693280 = 1 763 193,6
El capital final es de 1 763 194 Bs.

Ejemplo Ilustrativo Nº 5
Un cierto capital invertido durante 7 años a una tasa de interés compuesto anual
del 10 % se ha convertido en 1.583.945 bolívares. Calcular el capital inicial, sa-
biendo que los intereses se han pagado semestralmente.

Solución:

n= 7, C= 1.583.945

Como los intereses se han pagado semestralmente, la fórmula que se ha de apli-


car es: Cn= c ( n
Cn=  C ( 7x2
                        
Cn= 1.583.945= C (1+0.05)14
                                        
1.583.945
Despejando C, se tiene: C = 1.979 = 800.376,45

El capital inicial fue de 800.376,45 Bs.

La diferencia fundamental entre el interés simple y el interés compuesto


estriba en que en el primero el capital permanece constante y en el segun-
do el capital cambia al final de cada período. 

EQUIVALENCIA
El valor del dinero en el tiempo y la tasa de interés acumulada conjuntamente generan
el concepto de equivalencia, esto significa que diferentes sumas de dinero en diferentes
tiempos pueden tener igual valor económico. Por ejemplo si la tasa de interés es de 12% por
año, 100 Bs hoy pueden ser equivalentes 112 Bs en un año.

447
Sánchez Marchán, Nanci. (2014).Introducción a la Ingeniería Industrial, Material Instruccional de Apoyo. Caracas: UNA

Esta equivalencia o cambio del valor del dinero en el tiempo es producto de la incorpo-
ración o influencia de la tasa de interés. Por ello hablar del valor agregado del dinero en el
tiempo, implica hablar de tasas de interés nominales, reales y efectivas de períodos, de las
fechas en las que se dan los movimientos de dinero y de la naturaleza de estos movimientos
iniciándose siempre con un valor presente, para llegar a un valor futuro.

El primero, valor presente (VP) se refiere a la cantidad de dinero que será invertida o
tomada en préstamos al principio de un período determinado. Su fórmula es: VP= VF (1 +
i)n. El segundo, valor futuro (VF) involucra la cantidad de dinero que será obtenida por el
inversionista o pagada por el solicitante en una fecha futura, al final del plazo. Su fórmula es:
VF = VP/ (1 + i)n.

Ejemplo Ilustrativo Nº 6
Hallar el valor futuro de 1 millón de Bs., invertido a una tasa del 5% trimestral al
cabo de 2 años. Los valores de las variables se definen como:
VP = 1.000.000
i = 5%
n = 8 periodos trimestrales.
Nota: la periodicidad de la tasa de interés debe coincidir con la periodicidad del
plazo de tiempo, en este caso trimestres y donde la tasa de interés determina
este ciclo.
Luego se elabora el diagrama de flujo de caja y se define la fórmula que deter-
mina el valor futuro:
VF = 1.000.000 * (1+ 0.05)8 = 1.477.455 Bs.
El valor futuro de 1.477.455 Bs. es equivalente al valor presente de 1.000.000
Bs., siempre y cuando los rendimientos generados al 5% trimestral se reinviertan
a la misma tasa.

Ejemplo Ilustrativo Nº 7
Hallar la cantidad de dinero que se debe invertir hoy para disponer de 2.000.000
Bs al final de 3 años, si la tasa de interés es del 2% mensual.
F = 2.000.000 Bs.
i = 2% mensual
n = 36 meses.
VF = VP * (1+ i)n, despejando el valor de VP.
VP = VF / (1+i)n: es el factor que convierte un pago único futuro de valor F, en un
pago único presente de valor P equivalente.
VP = 2.000.000 / (1+.02)36 = 980.446 Bs.

448
Sánchez Marchán, Nanci. (2014).Introducción a la Ingeniería Industrial, Material Instruccional de Apoyo. Caracas: UNA

DIAGRAMA DE FLUJO DE CAJA


El diagrama de flujo de dinero en caja, es un método gráfico que permite representar
cada alternativa de inversión económica mostrando toda la información requerida para ana-
lizar una propuesta de inversión.

El diagrama de flujo de caja representa cada ingreso recibido durante el período con
una flecha hacia arriba (incremento de dinero) ubicada a fin del período. La longitud de la
flecha es proporcional a la magnitud de los ingresos recibidos durante ese período. En forma
similar, los egresos se representan por una flecha hacia abajo (una disminución en dinero).
Estas flechas son ubicadas en una escala de tiempo que representan la duración de la al-
ternativa.

Ejemplo Ilustrativo Nº 8
Un banco presta 1.000 Bs a un cliente, el cual debe cancelar su deuda efectuan-
do una serie de 5 pagos anuales, al 8% de interés. Hallar los valores acumulados
al final de cada año, para el prestatario (cliente) y para el prestamista (Banco).

Solución:
Año Suma adeuda- Interés adicionado a Suma adeuda- Suma a ser
da a comienzo la deuda a fin de año da a fin de año pagada a fin de
del año (A) (B) (A+B) año
1 1 000 1 000 × 0,08 = 80 1 080 0,00
2 1 080 1 080 × 0,08 = 86,4 1 166,4 0,00
3 1 166,4 1 166,4 × 0,08 = 93,3 1 259,7 0,00
4 1 259,7 1 259,7 × 0,08 = 100,8 1 360,5 1 360,5

En la figura 72, se observan los diagramas de flujo para el prestatario y el prestamista


del ejemplo anterior:
1.000 1.360,5

0 1 2 3 4 0 1 2 3 4

1.000
1.360,5

Figura 72. Diagrama de flujo de caja: (a) prestatario y (b) prestamista.

449
Sánchez Marchán, Nanci. (2014).Introducción a la Ingeniería Industrial, Material Instruccional de Apoyo. Caracas: UNA

RESUMEN DE SIMBOLOGÍA Y FÓRMULAS UTILIZADAS


Los símbolos y fórmulas matemáticas utilizadas en la Ingeniería Económica se mues-
tran en la tabla 47:

Tabla 47. Simbología y fórmulas utilizadas en la Ingeniería Económica


Simbología y fórmulas utilizadas en la Ingeniería Económica
Símbolo Significado Fórmula
I = cantidad acumulada – cantidad original.
i Interés. Si se presta dinero el interés que se genera es:
I =cantidad debida – préstamo original
i Tasa de interés por período. i = Tasa nominal / Nº períodos por año
IS Interés simple. IS = c * t * i
Capital, cuando los períodos
C= Cn = C (1 + i )n
son diferentes a 1 año.

VF
IC Interés compuesto. Ic = n −1 = ( VF / VP) 1/n – 1
VP

n # de períodos n = log(VF/VP)/log(1+i)
Valor o suma de dinero en Es igual al capital inicial (C) o (VP) = V F / (1+i)n
VP
un tiempo presente.
Valor o suma de dinero en Capital final = VF =(Cn) = VF = VP * (1+ i)n
VF
un tiempo futuro.

A
Serie consecutiva igual de i
dinero al final del período. Anualidad = A = VF
(1 + i ) n − 1

fórmulas del interés compuesto


Equivalencia: la fórmulas VP y VF, se obtienen de la siguiente forma:
VF = VP * (1+ i)n, despejando el valor de VP se tiene:
VP =V F / (1+i)n o VP= VF (1+n) -n, factor que convierte un pago único futuro de Valor VF,
en un pago único presente de valor (VP) equivalente.
Los valores VP y VF representan valores sencillos, los cuales ocurren una vez en el
tiempo.
También se muestra a continuación la relación de factor de valor presente y futuro con
la anualidad:
Dado A, encontrar Dado VP, encontrar Dado A, encontrar Dado VF, encontrar
VP: A: VF: A:
VP= A(AP,i,n) A= VP(AP,i,n) VF= A(FA,i,n) A= VF(A,F,i,n)
n
(1+ i) n −1 A = VP ix(1 + i ) (1 + i ) n − 1 i
VP = AX VF = A x A = VF
ix(1+ i) n (1 + i ) n − 1 i (1 + i ) n − 1
El factor (1+i)n es el factor de monto compuesto con pago simple.

450
Sánchez Marchán, Nanci. (2014).Introducción a la Ingeniería Industrial, Material Instruccional de Apoyo. Caracas: UNA

El principio económico más importante que utilizan las finanzas para la administración
en la toma de decisiones es el análisis marginal, el cual establece que se debe intervenir
en la economía sólo cuando los beneficios adicionales excedan a los costos agregados,
generando que todas las decisiones financieras se orienten a un cálculo de sus beneficios y
costos marginales. 

Las finanzas proporcionan información financiera a partir de la información contable, es


decir, la contabilidad genera información medible en términos monetarios, presentándola en
forma estructurada y sistemática para reflejar las operaciones de una empresa, aquí se ubica
la contabilidad de costos.

La contabilidad de costos  se puede definir como:

Un sistema de información para predeterminar, registrar, acumular, distribuir, controlar,


analizar, interpretar e informar de los costos de inversión (producción, distribución, ad-
ministración y financiamiento), para el uso interno de los directivos de la empresa en el
desarrollo de las funciones de planeación, control y toma de decisiones

COSTOS DE LA INVERSIÓN
El fin de toda empresa es satisfacer las necesidades y deseos del cliente vendiéndole
un producto o servicio por más dinero de lo que cuesta fabricarlo, para cubrir los costos y
obtener una utilidad. Conocer los costos de la empresa es un mecanismo clave de la co-
rrecta gestión empresarial, el cálculo de costo es uno de los instrumentos más importantes
para la toma de decisiones y es necesario considerar la incidencia de cualquier decisión en
este sentido, además de los posibles resultados que pueda generar.

El cálculo de costos, es importante en la planificación de productos y pro-


cesos de producción, la dirección y el control de la empresa así como,
para la determinación de los precios.

En atención a lo antes expuesto se puede definir el costo como:

El resultado económico que representa la elaboración de cualquier bien, o la prestación


de cualquier servicio.

Conociendo el costo de un bien o servicio se puede determinar el precio de venta al


público de los mismos, ya que el P.V.P. (Precio de Venta al Público) es la suma del costo más
el beneficio.

Los objetivos de un costo son aquellos de tipo operativos, como por ejemplo pagar los
sueldos al personal de producción, comprar materiales, fabricar un producto, venderlo, pres-
tar un servicio, obtener fondos para financiarnos, administrar la empresa, etc. Si no se logra
el objetivo deseado, se dice que existe una pérdida.

451
Sánchez Marchán, Nanci. (2014).Introducción a la Ingeniería Industrial, Material Instruccional de Apoyo. Caracas: UNA

Clasificación de los costos

La clasificación de los costos se puede realizar de acuerdo a jerarquías o grupos, de for-


ma tal que tengan ciertas características comunes para poder efectuar los cálculos, realizar
el correspondiente análisis y presentar la información que puede ser utilizada para la toma
de decisiones. Los costos se pueden clasificar como sigue:

a. Costo de producción
También llamados costos de operación. Son los gastos necesarios para mantener un
proyecto, línea de procesamiento o un equipo en funcionamiento. Entre los cuales se
encuentran:
• Costo de la materia prima y materiales que intervienen en el proceso productivo.
• Sueldos y cargas sociales del personal de producción.
• Depreciaciones del equipo productivo.
• Costo de los servicios públicos que intervienen en el proceso productivo.
• Costo de envases y embalajes.
• Costos de almacenamiento, depósito y expedición, entre otros.
2. Clasificación según la función que cumplen

b. Costo de comercialización (gastos)


Es el costo que facilita el proceso de venta de los bienes o servicios a los clientes, como
por ejemplo:
• Sueldos y cargas sociales del personal del área comercial.
• Comisiones sobre ventas.
• Fletes, hasta el lugar de destino de la mercadería.
• Seguros por el transporte de mercadería.
• Promoción y publicidad.
• Servicios técnicos y garantías de post-ventas.

c. Costo de administración (gastos)


Son aquellos costos necesarios para la gestión de la empresa, entre los cuales se puede
citar los siguientes:
• Sueldos y cargas sociales del personal del área administrativa y general de la em-
presa.
• Honorarios pagados por servicios profesionales.
• Servicios públicos correspondientes al área administrativa.
• Alquiler de oficina.
• Papelería e insumos propios de la administración.

d. Costo de financiación (gastos)


Es el correspondiente a la obtención de fondos aplicados al negocio. Por ejemplo:
• Intereses pagados por préstamos.
• Comisiones y otros gastos bancarios.
• Impuestos derivados de las transacciones financiera.

452
Sánchez Marchán, Nanci. (2014).Introducción a la Ingeniería Industrial, Material Instruccional de Apoyo. Caracas: UNA

a. Costos fijos
Son aquellos costos cuyo importe permanece constante, independientemente del nivel
de actividad de la empresa, es decir que se realice o no la producción, se venda o no
la mercancía o servicio, dichos costos igual deben ser solventados por la empresa. Por
2. Clasificación según su grado de variabilidad

ejemplo:
• Alquileres.
• Amortizaciones o depreciaciones.
• Seguros.
• Impuestos fijos.
• Servicios públicos (Luz, Tlf., Gas, etc.).
• Sueldo y cargas sociales de encargados, supervisores, gerentes, etc.

b. Costos variables
Son aquellos costos que varían en forma proporcional, de acuerdo con el nivel de pro-
ducción o actividad de la empresa. Son generados por la producción o las ventas, tal es
el caso de:
• Mano de obra directa (a destajo, por producción, por tiempo trabajado, por obra,
etc.).
• Materias Primas directas.
• Materiales e insumos directos.
• Impuestos específicos.
• Envases, embalajes y etiquetas.
• Comisiones sobre ventas.
3. Clasificación según su asignación

a. Costos directos
Son aquellos costos que se asigna directamente a una unidad de producción. Por lo
general son absorbidos por los costos variables.

b. Costos indirectos
Son aquellos que no se pueden fijar directamente a un producto o servicio, sino que se
distribuyen entre las diversas unidades productivas a través de algún criterio de reparto.
En la mayoría de los casos los costos indirectos son costos fijos.

453
Sánchez Marchán, Nanci. (2014).Introducción a la Ingeniería Industrial, Material Instruccional de Apoyo. Caracas: UNA

a. Costo variable unitario


Es el costo que se asigna directamente a cada unidad de producto. Comprende la uni-
dad de cada materia prima o materiales utilizados para fabricar una unidad de producto
terminado, así como la unidad de mano de obra directa, la unidad de envases y embala-
jes, la unidad de comisión por ventas, etc.

b. Costo variable total


Es el costo que resulta de multiplicar el costo variable unitario por la cantidad de produc-
4. Clasificación según su comportamiento

tos fabricados o servicios vendidos en un período determinado; sea éste mensual, anual
o cualquier otra periodicidad. La fórmula del costo variable total es la siguiente:
Costo variable total = costo variable unitario x cantidad

Para el análisis de los costos variables, se parte de los valores unitarios para llegar a
los valores totales. En los costos fijos el proceso es inverso, se parte de los costos fijos
totales para llegar a los costos fijos unitarios.

c. Costo fijo total


Es la suma de todos los costos fijos de la empresa.

d. Costo fijo unitario


Es el costo fijo total dividido por la cantidad de productos fabricados o servicios brinda-
dos.
Costo fijo unitario = costo fijo total / cantidad

e. Costo Total
Es la suma del costo variable más el costo fijo. Se puede expresar en valores unitarios
o en valores totales
Costo total unitario = costo variable unitario + costo fijo unitario
Costo total = costo variable total + costo fijo total

Los costos espirados generan los gastos, es decir, todos aquellos costos que pueden
deducirse de los ingresos, tales como gastos de operación, de ventas o administrativos, in-
tereses e impuestos.

A continuación se observan algunas diferencias entre costos y gastos:

454
Sánchez Marchán, Nanci. (2014).Introducción a la Ingeniería Industrial, Material Instruccional de Apoyo. Caracas: UNA

Tabla 48. Diferencias entre costos y gastos


Diferencias entre costos y gastos
Costos Gastos
Costo del producto o costos inventariables. Gastos del período o gastos no inventariables.

Constituyen el valor monetario de materia Se identifican con intervalos y no con los pro-
prima directa, mano de obra directa y los ductos elaborados.
cargos indirectos.
Se incorporan a los inventarios de materias Se relacionan con las funciones de distribución,
primas, producción en proceso y artículos administración y financiamiento de la empresa.
terminados, y se reflejan dentro del balance
general.
Los costos totales del producto se llevan al Se llevan al estado de resultados a través del
estado de resultados a medida que los pro- renglón de gastos de ventas, gastos de admi-
ductos elaborados se venden. nistración y gastos financieros.

Ejemplo Ilustrativo N° 1
Una empresa fabrica pistones para los cilindros de motores. La empresa ha de-
sarrollado un nuevo modelo de pistones que mejora la velocidad del motor. José
Castro, gerente de ingeniería de procesos, está intentando decidir si la empresa
debe fabricar o comprar una biela que requiere el nuevo pistón. Sus ingenieros
han realizado las siguientes estimaciones:
Fabricar Fabricar
Comprar
(Proceso A) (Proceso B)
Volumen anual 10.000 UNIDADES 10.000 UNIDADES 10.000 UNIDADES
Costos fijos/año 100.000 Bs 300.000 Bs
Costos variable/ 75 Bs 70 Bs 80 Bs
unidad

¿Debe la empresa fabricar la biela requerida por el nuevo pistón utilizando el pro-
ceso A, fabricar la biela utilizando el proceso B o comprar la biela?
Solución:
Se desarrolla el costo anual para cada una de las alternativas:
Costo anual total = Costo fijo +costo variable x volumen
Proceso A = 100.000 Bs. + 10.000(75 Bs) = 850.000 Bs.
Proceso B = 300.000 Bs. + 10.000(70 Bs) = 1.000.000 Bs.
Comprar = 0 Bs + 10.000(80 Bs.) = 800.000 Bs.

Si el volumen anual se estima como estable en 10.000 unidades, la empresa de-


berá comprar la biela.

455
Sánchez Marchán, Nanci. (2014).Introducción a la Ingeniería Industrial, Material Instruccional de Apoyo. Caracas: UNA

También se puede hablar de costos capitalizables, los cuales de acuerdo a la situa-


ción que los genera, pueden actuar como costos o como gastos, es decir, se capitalizan
como activo fijo o cargos diferidos, que se deprecian o amortizan a medida que se usan o
expiran, dando origen a cargos inventariables (costos) o del período (gastos)

Ejemplo Ilustrativo N° 2
Costos Plan 1 Plan2
Costo inventario estacional 411.220 265.720
Costo de reposición de trabajadores 0 155.086
Pago de horas extra 0 36.000
Subcontratación adicional 0 0

De acuerdo a los datos mostrados en la tabla realice un análisis comparativo de


los planes 1 y 2. Seleccione el más conveniente. Justifique su respuesta
Solución:
Costos totales:
Costo del Plan 1 = 411.220
Costo del Plan 2 = 265.720 + 155.086 + 36.000 = 457.306

El costo total del Plan 1 es inferior al costo correspondiente al Plan 2, por lo tanto
la selección del Plan 1 es la más conveniente. Desde el punto de vista práctico se
debe evitar la rotación del personal.

CONTRIBUCIÓN MARGINAL Y PUNTO DE EQUILIBRIO


Contribución marginal o margen de contribución

Se llama contribución marginal a la diferencia entre el precio de venta y el costo variable


unitario.

Contribución marginal = precio de venta - costo variable unitario

Existe un margen de contribución que muestra cómo ayudan los precios de los pro-
ductos o servicios, a cubrir los costos fijos y a generar utilidad, finalidad que persigue toda
empresa.

Recomendaciones referentes a la contribución marginal:

1. Si la contribución marginal es positiva, contribuye a absorber el costo fijo y a dejar


un margen para la utilidad o ganancia.

2. Cuando la contribución marginal es igual al costo fijo y no deja margen para la ga-
nancia, se dice que la empresa está en su “punto de equilibrio”. No gana, ni pierde.

456
Sánchez Marchán, Nanci. (2014).Introducción a la Ingeniería Industrial, Material Instruccional de Apoyo. Caracas: UNA

3. Cuando la contribución marginal no alcanza para cubrir los costos fijos, la empresa
puede seguir trabajando en el corto plazo, aunque la actividad dé resultado negati-
vo. Porque esa contribución marginal sirve para absorber parte de los costos fijos.

4. La situación más crítica se da cuando el precio de venta no cubre los costos va-
riables, o sea que la contribución marginal es negativa. En este caso extremo, es
cuando se debe tomar la decisión de no continuar con la elaboración de un producto
o servicio.

El concepto de contribución marginal o margen de contribución es muy importante


en la decisión de conservar, retirar o incorporar nuevos productos de la empresa, debido a
la incidencia que pueden tener estos en la absorción de los costos fijos y la capacidad de
generar utilidades.

Ejemplo Ilustrativo N° 3
Una empresa tiene una facturación anual de 200 millones de bolívares.
Entre las diversas gamas de su fabricación existe un tipo de piezas que se com-
pran al exterior a un precio total de 100 Bs., incluidos todos los gastos. El con-
sumo anual de esta pieza es de 100.000 unidades. Para el caso de fabricar el
producto en la empresa, se conocen los siguientes datos.

Costo de mano de obra 25,00 Bs. / unidad


Costo de materiales 30,00 Bs. / unidad
Costo de energía eléctrica 2,5 Bs. / unidad
Costo anual de amortización de maquinarias e instalaciones
2.000.000 Bs. / año
necesarias
Costo de almacenaje necesario 60.000 Bs. / año
Gastos generales totales 20.000.000 Bs. / año

Para fabricar este producto sería preciso suprimir la producción de otras piezas,
cuyo beneficio bruto supone 800.000 Bs. al año. Usted como gerente, ¿qué deci-
sión se podría tomar, comprar o fabricar?
El costo anual de comprar 100.000 piezas es: 100.000 x 100 Bs = 10.000.000 Bs
Si se entiende que costo marginal es el que varía con la alternativa que se consi-
dere, tendremos que el costo marginal de fabricar 100.000 piezas sería:
Mano de obra 100.000 x 25 = 2.500.000 Bs.
Materiales 100.000 x 30 = 3.000.000 Bs.
Energía 100.000 x 2,50 = 250.000 Bs.
Amortización de Instalaciones 2.000.000 Bs.
Almacenaje 60.000 Bs.
Total 7.810.000 Bs

457
Sánchez Marchán, Nanci. (2014).Introducción a la Ingeniería Industrial, Material Instruccional de Apoyo. Caracas: UNA

Se supone que los gastos de la empresa no varían por el hecho de montar esta
nueva instalación. El beneficio marginal bruto anual al fabricar en vez de comprar
sería:
10.000.000 Bs. - 7.810.000 Bs. = 2.190.000 Bs.
Quedaría disminuido en 800.000 Bs., por pérdida de beneficio al dejar de fabricar
otras piezas. Por lo tanto el beneficio marginal neto seria:
2.190.000 Bs. - 800.000 Bs. = 1.390.000 Bs.
A la vista de los datos obtenidos, la decisión a tomar seria lógicamente fabricar
estas piezas.

PUNTO DE EQUILIBRIO
Es conocido que una empresa se encuentra en su Punto de Equilibrio cuando no genera
ni ganancias ni pérdidas; es decir cuando el beneficio es igual a cero. Para un determinado
costo fijo de la empresa y conocida la Contribución Marginal de cada producto, se pueden
calcular las cantidades de productos o servicios y el monto total de ventas necesarios para
no ganar ni perder; es decir para estar en equilibrio.

La fórmula es la siguiente:

Costo fijo total


Punto de equilibrio= = Cantidades
Margen de contribución

Punto de equilibrio en Bs = cantidades * precio de venta

Gráfico del punto de equilibrio

P (Precio Bs) Ingreso total

(b)
Costo total

(a)= área de pérdida


(a)
(b) = área de ganancia

Q (cantidad vendida)
0 Pe

Figura 73. Gráfico del punto de equilibrio.

458
Sánchez Marchán, Nanci. (2014).Introducción a la Ingeniería Industrial, Material Instruccional de Apoyo. Caracas: UNA

(a) Área de pérdida.

(b) Área de ganancia.

En el caso (a) el punto de equilibrio está expresado en cantidades de producto y en el


caso (b) en montos de venta.

Elasticidad de la demanda

La elasticidad de la demanda, también conocida como la elasticidad precio de la deman-


da, es un concepto que en economía se utiliza para medir la sensibilidad o capacidad de res-
puesta de un producto, respecto a un cambio en su precio, como se muestra en la figura 74.

A. Demanda elástica! B. Demanda inelástica C. Demanda normal


Precio Precio Precio

P0
P0 P0
P* P* P*

Q
Q0 Q *
Q 0 Q* Q Q0 Q* Q

Figura 74. Elasticidad de la demanda.

Ante una variación en el precio, el producto A tiene una respuesta mayor que el pro-
ducto B. Esto indica que la demanda de A es elástica, mientras que la de B es inelástica y la
demanda de C es normal.

El precio de venta

El precio de venta es el valor de los productos o servicios que se venden a los clientes.
La determinación de este valor, es una de las decisiones estratégicas más importantes, ya
que el precio es uno de los elementos que los consumidores tienen en cuenta a la hora de
comprar lo que necesitan.

El cliente estará dispuesto a pagar por los bienes y servicios, lo que considera un pre-
cio justo, es decir, aquel que sea equivalente al nivel de satisfacción de sus necesidades o
deseos al accionar la compra.

459
Sánchez Marchán, Nanci. (2014).Introducción a la Ingeniería Industrial, Material Instruccional de Apoyo. Caracas: UNA

La ganancia se calcula utilizando la siguiente fórmula:

Ganancia o margen de utilidad = ventas - costos

Ejemplo Ilustrativo N° 4
Entre los productos de Enlatados Carolina se encuentra el atún, el cual es vendido
en el mercado nacional e internacional. La empresa desea aumentar las ganan-
cias en la venta del producto, para lo cual presenta dos alternativas con base a los
costos. Se le pide a usted como analista en producción, recomendar la alternativa
más viable a la situación planteada. Cada alternativa tiene su fortaleza y debilidad.
Se ha preparado la siguiente información como ayuda para la comparación y se-
lección de la alternativa:

Factores Alternativa 1 Alternativa 2


Costo anual de operación en Bs. 8.500.000 9.250.000
Unidades producidas al año 25.000 28.000
Precio de venta por unidad en Bs. 1.800 1.600
Costo de almacenamiento Bs./año 12.000 12.500

Solución:
Ganancia = Ventas – Costos

Alternativa 1
Ventas = 25.000 unidades/año x 1.800 Bs./ unidad
Ventas = 45.000.000 Bs./ año
Costo Total = Costo de operación + Costo almacenamiento
Costo Total = 8.500.000 + 12.000 = 8.512.000 Bs./año
Ganancias = 45.000.000 – 8.512.000 = 36.488.000 Bs./año

Alternativa 2
Ventas = 28.000.000 unidades/año x 1.600 Bs./ unidad
Ventas = 44.800.000 Bs./año
Costo Total = 9.250.000 Bs./año + 12.500 = 9.262.500 Bs./año
Ganancias = 44.800.000 – 9.262.500 = 35.537.500 Bs./año

Se debe elegir la alternativa 1 por proporcionar ésta mayores ganancias.

460
Sánchez Marchán, Nanci. (2014).Introducción a la Ingeniería Industrial, Material Instruccional de Apoyo. Caracas: UNA

Ejemplo Ilustrativo N° 5
Una empresa está considerando la adquisición de una máquina para ser utiliza-
da en la elaboración de un nuevo producto, el precio de venta de este producto
será de 4.700 Bs./unidad. Se están tomando en consideración tres máquinas
diferentes para la producción de éste. Sobre la base de los datos de la tabla que
se muestra a continuación:
Máquina Costo de operación Unidades produ- Desechos al
Bs/mes cidas al mes mes
Máquina A 2.800.000 15.000 2%
Máquina B 3.400.000 20.000 1.5%
Máquina C 2.600.000 18.000 1.8%
¿Cuál máquina seleccionaría usted para optimizar la producción?

Solución:
Ganancias = Ventas - Costos

Máquina “A”
Desechos generados = 2% de 15.000 = 300 unidades/mes
Unidades producidas = 15.000 – 300 = 14.700 unidades/mes
Ventas = 14.700 unidades/mes x 4.700 Bs./ unidad
Ventas = 69.090.000 Bs./mes
Ganancia = 69.090.000 – 2.800.000 = 66.290.000 Bs./mes

Máquina “B”
Desechos generados = 1.5% de 20.000 = 300 unidades/mes
Unidades producidas = 20.000 – 300 = 19.700 unidades/mes
Ventas = 19.700 unidades/mes x 4.700 Bs./ unidad
Ventas = 92.590.000 Bs./mes
Ganancia = 92.590.000 – 3.400.000 = 89.190.000 Bs./mes

Máquina “C”
Desechos generados = 1.8% de 18.000 = 324 unidades/mes
Unidades producidas = 18.000 – 324 = 17.676 unidades/mes
Ventas = 17.676 unidades/mes x 4.700 Bs./ unidad
Ventas = 83.077.000 Bs./mes
Ganancia = 83.077.200 – 2.600.000 = 80.477.200 Bs./mes

En función a la ganancia obtenida la máquina a seleccionar es la “B”.

461
Sánchez Marchán, Nanci. (2014).Introducción a la Ingeniería Industrial, Material Instruccional de Apoyo. Caracas: UNA

Ejemplo Ilustrativo N° 6
Hallar el punto de equilibrio (P.E.) de una empresa que tiene unos costos fijos de
24 millones de Bs. y trabaja con un margen sobre ventas del 27%.

Solución:
En el P.E. el volumen de ventas produce un Margen de Contribución Total que
iguala a los costos Fijos. Siendo éstos de 24 millones de Bs., es decir, el 27% de
las ventas debe ser 24 millones de bolívares.
P.E. * 0,27 = 24.000.000 → P.E. = 24.000.000 / 0,27 = 88,89 millones de bolíva-
res.

Ejemplo Ilustrativo N° 7
Industrias CORINCA S.A., trabajó el año pasado con un margen medio del 25%
sobre ventas. Sus gastos fijos ascendieron a 18,5 millones de euros. Tras una
reestructuración, los gastos fijos se han comprimido hasta 16,8 millones. Las
nuevas inversiones realizadas van a permitir elevar el margen medio hasta el
28%. Determinar el P.E. del año anterior y el nuevo P.E. tras la nueva situación.
Para el primer año, tenemos que, con unos costos fijos de 18,5 millones de bolí-
vares, el margen de contribución es del 25%.

Solución:
P.E. * 0,25 = 18,5 → X = 18,5 / 0,25 = 74 millones de bolívares para el año anterior.
Para el segundo año, se tiene una situación similar, salvo que los costos fijos han
pasado a 16,8 millones de bolívares y el Margen de Contribución medio ha subido
hasta el 28%. Luego:
P.E. = 16,8 / 0,28 = 60 millones de bolívares para la nueva situación.

Ejemplo Ilustrativo N° 8
Los siguientes son datos referentes a la demanda del bien X:

Precio Cantidad
A 25 0
B 20 20
C 15 40
D 10 60
E 5 80
F 0 100

Grafique la curva de demanda. b) ¿Qué signo tiene el coeficiente de elasticidad?

462
Sánchez Marchán, Nanci. (2014).Introducción a la Ingeniería Industrial, Material Instruccional de Apoyo. Caracas: UNA

Solución:

b) Negativo. Esto como consecuencia de la ley de la demanda (cuando aumenta


el precio, disminuye la cantidad, y viceversa).

Además de la información presentada hasta el momento, es importante estudiar el pre-


cio de venta, el cual es igual al costo total del producto más la ganancia.

Precio de venta = Costo total de venta(Costos variables + Costos fijos)


+ Ganancia (margen de utilidad)

En la figura que se muestra a continuación se presenta una fórmula de cómo se genera


el precio de venta:

Figura 75. Fórmula para generar el precio de venta.

463
Sánchez Marchán, Nanci. (2014).Introducción a la Ingeniería Industrial, Material Instruccional de Apoyo. Caracas: UNA

La empresa espera, a través del precio, cubrir los costos y obtener ganancias. En la
determinación del precio, es necesario tomar en cuenta los objetivos de la empresa y las
expectativas del cliente.

La evaluación de costos permite a los gerentes como también a las em-


presas, la posibilidad de analizar si son económicamente eficientes en la
producción.

Ejemplo Ilustrativo
La gerencia de producción de la empresa CARAMEL C.A., debe determinar sus
pérdidas o utilidades resultantes de las operaciones productivas. La gerencia
cuenta con los siguientes datos:
• Los costos fijos ascienden a 40 Bs.
• Los costos variables son de 1,20 Bs. por unidad
• El ingreso por venta es de 2,00 Bs. por unidad
• El nivel actual de producción es de 25.000 unidades

Sobre la base de los datos mostrados:


a) ¿Cuál debe ser el volumen de producción para lograr el punto de equilibrio?
b) ¿Qué volumen de producción sería necesario para lograr una producción de
56.000 Bs.?

Solución:
a) Ingreso por ventas=25.000 unidades x 2.00 Bs./unidad =50.000 Bs.
Costo total=CF+CV= 40.000 Bs. +1.20Bs. / unidad x 25.000 unidades = 70.000 Bs.
Los costos (70.000 Bs) exceden al ingreso (50.000 Bs.)
Las pérdidas ascienden a 20.000 Bs., si el nivel de producción se mantiene en
25.000 unidades el volumen de producción deberá ser:

PE =CF / Precio unitario – Costo variable por unidad



40.000 Bs.
PE = = 50.000 unidades
2, 00 Bs. − 1, 20 Bs.

b) Utilidad +CF=(Precio unitario–Costo variable/ Unidad)


56.000Bs. + 40.000 Bs. = X (2, 00 Bs. - 1, 20 Bs.)
96000 Bs. = (0, 80) X
Es necesario por tanto operar la planta a un nivel de 120.000 unidades para obte-
ner utilidades de 56.000 Bs.

464
Sánchez Marchán, Nanci. (2014).Introducción a la Ingeniería Industrial, Material Instruccional de Apoyo. Caracas: UNA

La escasez conlleva economizar y elegir el máximo beneficio al mínimo costo, a veces


se conoce a la economía como la ciencia de la elección, que predice en qué forma los cam-
bios de circunstancias afectan las elecciones de las personas.

Las finanzas informan en muchos casos de las decisiones que toma una empresa y que
tienen incidencia en el cambio de los activos, pasivos y el patrimonio, por lo cual, toda deci-
sión que se tome en el área financiera debe estar soportada por los estados financieros que
emite la contabilidad, apoyada por las operaciones microeconómicas y macroeconómicas.

ESTADOS FINANCIEROS
Los estados financieros son los documentos que debe preparar la empresa al terminar
el ejercicio contable, con el fin de conocer la situación financiera y los resultados económicos
obtenidos en las actividades de su empresa a lo largo de un período. Son informes sobre la
situación financiera y económica de una empresa en un período determinado.

La información presentada en los estados financieros interesa a:


•• La administración, para la toma de decisiones, después de conocer el
rendimiento, crecimiento y desarrollo de la empresa durante un período
determinado.
•• Los propietarios para conocer el progreso financiero del negocio y la
rentabilidad de sus aportes.
•• Los acreedores, para conocer la liquidez de la empresa y la garantía de
cumplimiento de sus obligaciones.
•• El Estado, para determinar si el pago de los impuestos y contribuciones
está correctamente liquidado.

Los estados financieros básicos son:


•• El balance general.
•• El estado de resultados.
•• El estado de cambios en el patrimonio.
•• El estado de cambios en la situación financiera.
•• El estado de flujos de efectivo.

Balance general

Es el documento contable que comunica en una fecha determinada la situación finan-


ciera de la empresa, mostrando en forma clara el valor de sus propiedades y derechos, sus
obligaciones y su capital; valorados y elaborados de acuerdo con los principios de contabi-
lidad generalmente admitidos.

465
Sánchez Marchán, Nanci. (2014).Introducción a la Ingeniería Industrial, Material Instruccional de Apoyo. Caracas: UNA

Estado de resultados (ganancias o pérdidas)

Es un documento complementario en la que se informa detallada y ordenadamente


como se obtuvo la utilidad del ejercicio contable. El estado de resultados está compuesto por
las cuentas nominales, transitorias o de resultados, o sea las cuentas de ingresos, gastos y
costos.

Estado de cambios en el patrimonio (estado de superávit)

Es el estado financiero que muestra en forma detallada los aportes de los socios y la
distribución de las utilidades obtenidas en un período, además de la aplicación de las ganan-
cias retenidas en períodos anteriores. También muestra la diferencia entre el capital contable
(patrimonio) y el capital social (aportes de los socios), determinando la diferencia entre el
activo total y el pasivo total, incluyendo en el pasivo los aportes de los socios.

El estado de cambios en la situación financiera

El estado de cambios de la situación financiera tiene como esencia dar a conocer el


movimiento financiero de una empresa ajustado a un período. Su base es el capital de traba-
jo, el cual corresponde a la diferencia entre el activo circulante y el pasivo circulante, es decir,
entre los recursos de corto plazo y las obligaciones de corto plazo.

Lo antes expresado indica que puede reconocerse un ingreso de fondos, tanto cuando
se produce una entrada de dinero como cuando se hace un cargo a una cuenta por cobrar,
a una cuenta de existencias o a otros rubros del activo circulante; o bien, cuando disminuye
una cuenta del pasivo circulante. Asimismo, existe un egreso de fondos no sólo cuando ocu-
rre una salida de disponibilidades o una disminución de otros recursos del activo circulante,
esto también puede suceder cuando se contrae una obligación a corto plazo.

El estado de flujos de efectivo

Es un instrumento financiero que representa el impacto que tienen las actividades ope-
racionales, de inversión y de financiamiento de una empresa, sobre sus flujos de efectivo a
lo largo de un período contable. Se trata de un estado que informa sobre la utilización de los
activos monetarios representativos de efectivo y otros activos líquidos equivalentes, clasifi-
cando los movimientos por actividades e indicando la variación neta de dichas actividades
en el ejercicio financiero.

Aunque se trata de una nueva obligación legal, es un estado financiero ampliamente tra-
tado en la literatura contable, generalmente se menciona como estado de flujos de tesorería
o estado de cash flow.

466
Sánchez Marchán, Nanci. (2014).Introducción a la Ingeniería Industrial, Material Instruccional de Apoyo. Caracas: UNA

En este texto, se profundizará un poco más respecto al balance general, considerado


el más básico de los estados financieros. Se ampliará su contenido y presentarán algunos
ejemplos.

El balance general es un estado financiero básico que informa en una fecha determi-
nada la situación financiera de la empresa. Es decir, indica los inventarios realizados por la
empresa en forma de activos y los medios a través de los cuales se financiaron estos activos.
El financiamiento puede ser a través de préstamos (pasivos) o mediante la forma acciones
(capital contable).

Componentes del balance general:


Patrimonio: obligaciones que la empresa tiene con los accionistas (capital, utilidades
acumuladas).
Capital: es la inversión inicial y aportes posteriores hechos por dueños o accionistas,
para la constitución y operación de la empresa.
Utilidades acumuladas: ganancias que la empresa va obteniendo y acumulando con
el tiempo, las cuales se pueden capitalizar (convertirse en capital) o repartirse entre los due-
ños a través del pago de dividendos.
Factores activos: se expresan en términos de unidades monetarias, sólo el efectivo
representa dinero real.

Tipos de activos:
Activos circulantes: lo que se debe invertir en el corto plazo para realizar un negocio,
generalmente corresponden a lo que se tiene en bancos y créditos a favor.
Activos fijos: es la inversión en infraestructura, tales como equipos, maquinarias, he-
rramientas, vehículos, terrenos, oficinas, entre otros.
Otros activos: se incluyen en estos activos las inversiones que se realizan y que no
corresponden a las partidas antes mencionadas o no están directamente relacionadas con
el negocio.
El pasivo lo constituye tanto las obligaciones como las cargas o gravámenes, es decir,
los derechos contra los activos.
Factores pasivos: todos los que son usados por los elementos activos y ayudan a con-
seguir los objetivos de la empresa. Son las obligaciones o deudas de la empresa.
Pasivos circulantes: lo que se debe en el corto plazo y se usó para financiar el negocio.
Pasivos de largo plazo: las fuentes de financiamiento de largo plazo, más de un año
para el plazo de vencimiento (proveedores, letras, acreedores, entre otros).

Las decisiones en una empresa, una vez conocidos los valores totales de activos y pa-
sivos se puede tomar a través de la ecuación patrimonial:

PATRIMONIO =ACTIVO - PASIVO

467
Sánchez Marchán, Nanci. (2014).Introducción a la Ingeniería Industrial, Material Instruccional de Apoyo. Caracas: UNA

Las decisiones individuales se toman considerando los resultados de la siguien-


te ecuación:

ACTIVOS (RIQUEZAS) – PASIVOS (DEUDAS) = CAPITAL

Conocidos cada uno de los aspectos a considerar en un balance general, se pre-


senta a continuación un formato que puede ser utilizado para elaborar este estado
financiero, en una empresa.

ESTRUCTURA DE UN BALANCE GENERAL

BALANCE GENERAL
Nombre de la Empresa:
Fecha:
ACTIVOS
Activo corriente
Disponible
Deudores
Inventarios
Activo no corriente
Propiedades, planta y equipo
Intangibles
Diferidos
Valoraciones
TOTAL DE ACTIVOS
PASIVO Y PATRIMONIO
PASIVOS:
Pasivo corriente
Obligaciones financieras
Proveedores
Cuentas por pagar
Impuestos, gravámenes y tasas
Obligaciones laborales
Diferidos
Pasivo no corriente
Otros pasivos de largo plazo
Bonos y papeles comerciales
TOTAL DE PASIVOS
Capital de accionistas
Superavit de capital
Utilidades retenidas
TOTAL DEL PATRIMONIO

468
Sánchez Marchán, Nanci. (2014).Introducción a la Ingeniería Industrial, Material Instruccional de Apoyo. Caracas: UNA

Ejemplo Ilustrativo N° 1
BALANCE GENERAL
Nombre de la empresa: IASESAN C.A
Fecha: diciembre 31/12/XX
ACTIVOS Terrenos 5.000
Mobiliario 500
Maquinaria 2.000
Bancos 1.000
Existencias 1.500
Edificios 3.000
Total de activos 13.000 Bs
Pasivos Proveedores 1.000
Hacienda 1.000
Efectos a pagar 2.000
Préstamos 2.000
Total de pasivos 6.000 Bs
Patrimonio Capital de accionistas 7.000
Total de pasivos y patrimonio 13.000

Activo = Pasivos + Patrimonio →13.000 = 13.000

Ejemplo Ilustrativo N° 2
BALANCE GENERAL
Nombre de la empresa: BAKUM C.A. (fabricante de lencería)
Fecha: Diciembre 31/12/ 2012
ACTIVOS Efectivo 21.900
Cuentas por cobrar 132.000
Materia prima 3.000
Edificio y terreno 7.000
Maquinaria y equipo 9.400
Automóvil 15.000
TOTAL DE ACTIVOS 208.500 Bs
PASIVOS: Cuentas por pagar 100.000
Salarios por pagar 50.000

TOTAL DE PASIVOS 150.000 Bs


PATRIMONIO Capital de accionistas 48.000
Intereses de capital 10.500

TOTAL DE PATRIMONIO 58.500 Bs


TOTAL DE PASIVOS Y PATRIMONIO 208.500 Bs
PASIVOS + PATRIMONIO = ACTIVOS → 208.500 = 208.500 Bs

469
Sánchez Marchán, Nanci. (2014).Introducción a la Ingeniería Industrial, Material Instruccional de Apoyo. Caracas: UNA

Actividad propuesta
Investigue cómo funciona el equilibrio en un balance general.

Algunos activos fijos (mobiliario, maquinaria, entre otros) pueden sufrir un desgaste, lo
cual los conduce a una depreciación.

DEPRECIACIÓN
La depreciación de un activo fijo se refiere a una reducción anual del valor de una plan-
ta, propiedad o equipo debido a factores como el desgaste por el uso, el paso del tiempo
y la obsolescencia, entre otros. En el cálculo de la depreciación están involucrados cuatro
conceptos: el valor a depreciar, el método a utilizar, la vida útil y el valor de salvamento o
recuperación.

El valor a depreciar. Está fundamentado sobre el costo de adquisición, teniendo en


cuenta siempre el costo de recuperación.

La vida útil. Es la duración estimada que un objeto puede tener cumpliendo correcta-
mente con la función para la cual ha sido creado. Normalmente se calcula en horas de dura-
ción, o en años cuando se refiere a vehículos, maquinarias o equipos u obras de ingeniería
como: carreteras, puentes, represas, entre otros.

Valor de recuperación. La empresa estima que una vez terminada la vida útil del acti-
vo, éste puede aún tener algún tipo de valor.

Método a utilizar. Existen dos métodos para calcular la depreciación, que permite es-
timar el gasto por desgaste de los activos fijos: método lineal de depreciación y el método
acelerado de depreciación.

El Método lineal de depreciación: este método presenta a su vez dos variantes, el


método de línea recta y el de unidades producidas.
•• El Método de la línea recta: es el método más sencillo y más utilizado
por las empresas, consiste en dividir el valor del activo entre la vida útil
de éste, es decir, (costo del activo) - (valor de desecho)/Vida útil. Supone
este método que el activo se desgasta por igual en cada periodo conta-
ble.

Ejemplo Ilustrativo Nº 1
Calcular utilizando el método de la línea recta el costo de depreciación de un tor-
no que cuesta 22.000 Bs., el cual se utilizará aproximadamente durante 5 años y
cuyo valor de desecho es de 2.000 Bs.
Solución:
Costo de depreciación por año=

470
Sánchez Marchán, Nanci. (2014).Introducción a la Ingeniería Industrial, Material Instruccional de Apoyo. Caracas: UNA

•• Método de Unidades Producidas. Considera la depreciación en fun-


ción de la utilización o de la actividad y no del tiempo. La fórmula a utilizar
es: Costo–Valor de desecho = Costo de depreciación de una unidad /
Número de unidades.

Ejemplo Ilustrativo Nº 2
Calcular utilizando el método de unidades producidas la depreciación del torno
utilizado en el ejemplo anterior, al producir 100 piezas. Utilice la tabla mostrada
para determinar la depreciación en cada año:

Año Piezas
1 30
2 30
3 15
4 15
5 10

Solución:
Costo por pieza=

Año Costo por Cantidad (Costo por pieza)(cantidad de


pieza (Bs.) de Piezas pieza) (Bs)
1 0.20 30 6.000
2 0.20 30 6.000
3 0.20 15 3.000
4 0.20 15 3.000
5 0.20 10 2.000

El método de unidades producidas distribuye el costo por depreciación de ma-


nera equitativa, siendo el mismo para cada unidad producida durante todo el
período fiscal.

Método acelerado de depreciación

El método acelerado de depreciación en contraste con el lineal produce un gasto por


depreciación más grande en los primeros años del uso del activo fijo, que en los últimos años
de su vida útil. El método acelerado de depreciación se divide en:

•• Método de depreciación suma de dígitos. En cada año se disminuye


el costo de desecho por lo que el resultado no será equitativo a lo largo

471
Sánchez Marchán, Nanci. (2014).Introducción a la Ingeniería Industrial, Material Instruccional de Apoyo. Caracas: UNA

del tiempo o de las unidades producidas, sino que irá disminuyendo pro-
gresivamente. La suma de dígitos anuales no es otra cosa que sumar el
número de años de la siguiente forma: Para una estimación de 6 años: 1
años + 2 años + 3 años + 4 años + 5 +6 años  = 21.
•• Método de depreciación doble saldo decreciente: en éste método no
se deduce el valor de desecho o recuperación de costo del activo para
tener la cantidad a depreciar, se busca un porcentaje para aplicarlo cada
año. El valor decreciente coincide con el doble del valor obtenido median-
te el método de la línea recta.

En este texto sólo se ejemplificará el método lineal de depreciación. El método acele-


rado de depreciación se estudiará en una de las asignaturas del área economía y finanzas,
cuando se avance en la carrera.

La depreciación de un año varía de acuerdo al método seleccionado pero la de-


preciación total a lo largo de la vida útil del activo no puede ir más allá del valor
de recuperación.

El conocimiento de la economía y la contabilidad deben considerarse como insumos


necesario de las finanzas, los cuales logran generar las condiciones y oportunidades nece-
sarias para obtener el capital, darle el mejor uso y cumplir con los pagos e intereses que se
deben cargar a las transacciones en dinero.

472
Sánchez Marchán, Nanci. (2014).Introducción a la Ingeniería Industrial, Material Instruccional de Apoyo. Caracas: UNA

MERCADO Y MARKETING INDUSTRIAL

•• El mercado.
•• Origen del mercado.
•• Definición de marketing.
•• Otras definiciones de marketing.
•• Objetivos del estudio de mercado.
•• Tipos de mercado Segmentación de mercado.
•• Mercado meta.
•• Origen del término marketing.
•• Fases del proceso de marketing.
•• Nueva visión del marketing.
•• Algunos elementos que han contribuido al desarrollo del marketing.

ORIGEN DEL MERCADO


El origen del mercado se asocia a la época en la cual el hombre primitivo se percató de
que podía obtener objetos que no producía, efectuando el cambio o trueque con otros pue-
blos o tribus. El mercado existió en los pueblos y tribus más antiguas que poblaron la tierra y
a medida que fueron evolucionando se fue desarrollando el comercio. 

Por ello con el devenir, el desarrollo de los pueblos obliga al aumento y expansión del
mercado, hasta convertirse en una actividad económica de gran importancia para el progre-
so de la humanidad. El lugar que ocupa el mercado en la sociedad moderna es cada vez más
importante y debido al logro de las comunicaciones rápidas entre las diversas regiones del
mundo se han extendido aún más las relaciones comerciales.

DEFINICIÓN DE MERCADO

El mercado se puede definir según algunos autores como sigue:


•• El mercadeo es el proceso de:
1. Identificar las necesidades del consumidor.
2. Conceptualizar tales necesidades en función de la capacidad de la empresa
para producir.
3. Comunicar dicha conceptualización a quienes tienen la capacidad de tomar
decisiones en la empresa.
4. Conceptualizar la producción obtenida en función de las necesidades previa-
mente identificadas del consumidor.
5. Comunicar dicha conceptualización al consumidor “.

473
Sánchez Marchán, Nanci. (2014).Introducción a la Ingeniería Industrial, Material Instruccional de Apoyo. Caracas: UNA

John A. Howard, de la Universidad de Columbia:


•• El término mercadeo significa “guerra”. Ambos consultores, consideran que una
empresa debe orientarse a competir; es decir, dedicar mucho más tiempo al aná-
lisis de cada “participante” en el mercado, exponiendo una lista de debilidades y
fuerzas competitivas, así como un plan de acción para explotarlas y defenderse
de ellas: Al Ries y Jack Trout, autores del libro “La Guerra del Mercadeo”.
•• “Es un proceso social y administrativo mediante el cual grupos e individuos ob-
tienen lo que necesitan y desean a través de generar, ofrecer e intercambiar pro-
ductos de valor con sus semejantes”: Philip Kotler, autor del libro “Dirección
de Mercadeo.
•• “Es la realización de aquellas actividades que tienen por objeto cumplir las metas
de una organización, al anticiparse a los requerimientos del consumidor o cliente
y al encauzar un flujo de mercancías aptas a las necesidades y los servicios que
el productor presta al consumidor o cliente”: Jerome McCarthy, Premio Trailbla-
zer de la American Marketing Asociation.

TIPOS DE MERCADO

Según el tamaño las preferencias y necesidades de los clientes:

Mercado total. Conformado por el universo con necesidades que pueden ser satisfe-
chas por la oferta de una empresa.

Mercado potencial. Integrado por todos los entes del mercado total que además de
desear un bien o servicio, están en condiciones de adquirirlos.

Mercado meta. Está constituido por los segmentos (se define más adelante) del mer-
cado potencial que han sido seleccionados en forma específica, como destinatarios de la
gestión de marketing, es el mercado que la empresa desea y decide captar.

Mercado real. Representa el mercado con el cual se ha logrado llegar a los consumi-
dores de los segmentos captados del mercado meta.

Según el monto de la mercancía:


•• Mercado mayorista
Son aquellos donde se vende mercaderías al por mayor y en grandes cantidades. Allí
acuden generalmente los intermediarios y distribuidores a comprar en cantidad, los produc-
tos que después han de revender a otros comerciantes, a precios mayores o caprichosamen-
te elevados.
•• Mercado minorista
Llamados también de abastos, donde se venden en pequeñas cantidades directamente
a los consumidores. Una modalidad de este tipo de mercado, son los llamados Supermerca-
do, de origen norteamericano, constituyen grandes cadenas u organizaciones  que mueven

474
Sánchez Marchán, Nanci. (2014).Introducción a la Ingeniería Industrial, Material Instruccional de Apoyo. Caracas: UNA

enormes capitales. Se estila en esta modalidad el autoservicio, es decir, el consumidor elige


los artículos que va a comprar, eliminándose el empleado dependiente y el pequeño comer-
ciante que vende personalmente sus artículos.

SEGMENTACIÓN DE MERCADO

Es el proceso que consiste en dividir el mercado total de un bien o servicio en varios


grupos más pequeños e internamente homogéneos. Todos los mercados están compuestos
de segmentos y éstos a su vez están formados usualmente por subsegmentos.

No existe una sola forma de segmentar un mercado, es por eso que se deben probar
diversas variables, solas y combinadas, con la expectativa de encontrar la manera óptima
de concebir la estructura del mercado. A continuación se detallan las principales variables
utilizadas en la segmentación de mercado:

1. Segmentación geográfica: Requiere que el mercado se divida en varias unidades


geográficas como naciones, estados, ciudades o barrios, es decir:
•• Mercado internacional: es aquel que se encuentra en uno o más países
en el extranjero.
•• Mercado nacional: es aquel que abarca todo el territorio nacional para
el intercambio de bienes y servicios.
•• Mercado regional: es una zona geográfica determinada libremente, que
no coincide necesariamente con los límites políticos.
2. Segmentación demográfica: Es la división en grupos basados en variables demo-
gráficas como sexo, edad, ingresos, educación, etnias, religión y nacionalidad. Lo
más común es segmentar un mercado combinando dos o más variables demográfi-
cas.

3. Segmentación psicográfica: Aquí los clientes se dividen en grupos según su cla-


se social, estilo de vida o personalidad.

4. Segmentación por conducta: En esta segmentación los clientes se dividen en gru-


pos según sus conocimientos, actitudes, costumbres o sus respuestas a un producto.

MERCADO META

El significado de mercado meta se relaciona con las necesidades que tienen las empre-
sas de seleccionar de un segmento de mercado, la población o grupo de consumidores a
los cuales se quiere llegar. Al mercado meta también se le conoce como mercado objetivo o
target group (público objetivo).

475
Sánchez Marchán, Nanci. (2014).Introducción a la Ingeniería Industrial, Material Instruccional de Apoyo. Caracas: UNA

Un mercado meta es el grupo de clientes al que se captará, servirá y se dirigirán los


esfuerzos de mercadeo. Para localizar posibles clientes, de los cuales se conozcan as-
pectos como edad, sexo, estado civil e ingresos, entre otros. Esto significa segmentar el
mercado.

Lograr un posicionamiento es uno de los objetos del mercado meta, al reunir a consu-


midores con las mismas características psicográficas y de actitudes, necesidades y gustos,
es más fácil delimitar las características del producto o marca, así como las necesidades
que va a cubrir. Cuando se realiza el plan de marketing y se selecciona el mercado meta, es
necesario delimitar el mercado, en los términos que permitan tomar decisiones.

Selección del mercado meta

Se analizarán los perfiles de los segmentos del mercado, se escogerán los más rele-
vantes y se fijará como meta uno o más de estos segmentos, para desarrollar productos, así
como programas de mercadeo a la medida de cada segmento seleccionado; para cubrir las
necesidades y deseos de grupos específicos de clientes sobre una base de mercadeo local
(por área geográfica, por cadena de almacenes, entre otros).

Tendencias de Mercado

Se refiere a los movimientos al alza o a la baja del mercado durante un periodo.

Es más complicado determinar el tamaño del mercado si se está comenzando con algo
completamente nuevo. En este caso se deberá obtener el número de clientes potenciales o
segmentos de clientes.

ORIGEN DEL TÉRMINO MARKETING

Mercado en ingles se expresa como market, que representa un grupo de vendedores


y compradores deseosos de intercambiar bienes y/o servicios por algo de valor. Cuando la
palabra market se expresa en gerundio, es decir, Marketing significa comercialización.

La revolución del mercado, revolución tecnológica, poder de compra de los clientes y la


globalización, marcan las nuevas necesidades de la función directiva en la industria, sobre la
base de estas necesidades surge el Marketing. El término aparece en el ámbito académico
a comienzos de la primera década de 1900. Así en 1902 Jones impartió en la Universidad
de Michigan un curso titulado “La industria distributiva y reguladora” y en 1905 bajo la direc-
ción de Kreusi se ofrece un curso titulado “Marketing de Productos” en la Universidad de
Pennsylvania (Bartels, 1988). Desde entonces se ha producido una evolución del concepto,
y al mismo tiempo una extensión del marketing a otros campos.  En Venezuela, el término
más utilizado es mercadeo, quizás con mayor fuerza en el ámbito académico y en medios
especializados.

476
Sánchez Marchán, Nanci. (2014).Introducción a la Ingeniería Industrial, Material Instruccional de Apoyo. Caracas: UNA

DEFINICIÓN DE MARKETING

El diccionario de la Encyclopaedia Británica, define el marketing como comercialización,


Mercadotecnia o técnica de estudio de los mercados.

Definición amplia de marketing (comercialización) se define como:

La disciplina que se preocupa de estudiar, reflexionar y dar soluciones sobre los mercados
de los clientes o consumidores en cuanto a sus necesidades, deseos y comportamiento,
también sobre la gestión comercial de las organizaciones. Su objetivo fundamental es
retener a los clientes mediante la satisfacción de sus necesidades.

OTRAS DEFINICIONES
•• “El marketing es un proceso social y administrativo mediante el cual gru-
pos e individuos obtienen lo que necesitan y desean a través de generar,
ofrecer e intercambiar productos de valor con sus semejantes” (Philip
Kotler)
•• “El marketing es la realización de aquellas actividades que tienen por
objeto cumplir las metas de una organización, al anticiparse a los reque-
rimientos del consumidor o cliente y al encauzar un flujo de mercancías
aptas a las necesidades y los servicios que el productor presta al consu-
midor o cliente” (Jerome McCarthy)
•• “El marketing es un sistema total de actividades de negocios ideado para
planear productos satisfactores de necesidades, asignarles precios, pro-
mover y distribuirlos a los mercados meta, a fin de lograr los objetivos de
la organización (Stanton, Etzel y Walker)

Con el marketing una empresa puede presentarse en el mercado (consumidores fina-


les), con una visión de rentabilidad a corto o largo plazo. Y ¿qué puede hacer una empresa
para que se tenga una visión y opinión positivas de ella y de sus productos? Puede gestionar
su propio producto, su precio, su relación con los clientes y con los proveedores, así como,
con sus empleados, la publicidad en diversos medios y soportes, relacionarse con los me-
dios de comunicación (relaciones públicas), entre otros. Todo esto es marketing.

Como disciplina de influencias científicas, el marketing es un conjunto de principios,


metodologías y técnicas a través de las cuales se busca conquistar un mercado, colaborar
en la obtención de los objetivos de la organización y satisfacer las necesidades y deseos de
los consumidores o clientes. El marketing busca clientes fieles, mediante herramientas y es-
tratégicas posicionando en la mente del consumidor un producto, marca, etc. De esta forma
busca ser la opción principal en su mente.

En una empresa, normalmente, el área comercial abarca el área de marketing y el área


de ventas para brindar satisfacción al cliente.

477
Sánchez Marchán, Nanci. (2014).Introducción a la Ingeniería Industrial, Material Instruccional de Apoyo. Caracas: UNA

Los conceptos de marketing, mercadotecnia, mercadeo y comercialización


se utilizan como sinónimos. Sin embargo el término marketing es el más
extendido.

ALGUNOS ELEMENTOS QUE HAN CONTRIBUIDO AL DESARROLLO DEL


MARKETING
Canales de distribución: la distribución ha evolucionado y ha marcado las pautas de
consumo. Los supermercados han adquirido el mayor protagonismo mientras que el comer-
cio tradicional está siendo en gran medida perjudicado, al estar perdiendo peso a grandes
pasos, sin hacer nada por retenerlo.
Fidelización: el cliente se sitúa como el eje fundamental de las empresas y el mercado,
por ello, su fidelización es uno de los principales retos del siglo XXI. El marketing se ha con-
vertido en el gran protagonista dentro del entorno empresarial como estrategia que no sólo
ayuda a conocer a los clientes en profundidad, sino a conservarlos. No se debe confundir un
cliente fiel con un cliente cautivo, el cliente cautivo sólo se atrae a través de las promociones.
La marca: se percibe como una de las mejores estrategias para proteger y potenciar el
patrimonio de las empresas. La marca, además de ser el principal identificador del producto,
es un aval que le otorga una garantía y lo sitúa en un plano superior, al construir una verda-
dera identidad y relación emocional con los consumidores.
Patrocinio: es considerado como una variable de la comunicación que está teniendo
un gran auge debido a la rentabilidad cualitativa y cuantitativa que aporta a las empresas.
Franquicias: se han convertido en los últimos años en una clara alternativa al comercio
minorista. Si se cuenta con una marca reconocida en el mercado de un producto o servicio,
se puede buscar una expansión rápida con una inversión pequeña a través de las franqui-
cias.
Innovación: se ubica como una herramienta imprescindible para ser competitivo, lo
cual es generado por el ciclo de vida tan corto de los productos. Como ejemplo se puede
citar al sector de la informática y las telecomunicaciones, que evolucionan a pasos agigan-
tados en el mercado.
Posicionamiento: es decir, lo que importa verdaderamente es lo que percibe el mer-
cado de la empresa y de la marca. Por ello es preciso utilizar todas las herramientas que el
marketing pone al alcance para posicionarse en la mente del consumidor y así la perciba de
manera positiva, para ubicarse en el mercado de forma más competitiva.
Internet: ha sido la gran revolución, el boom que ha obligado a adaptar las estrategias
de marketing en las empresas. Internet se perfila como un magnífico canal de distribución,
que ayuda a comercializar la empresa, situándose como un importante medio de comuni-
cación.

478
Sánchez Marchán, Nanci. (2014).Introducción a la Ingeniería Industrial, Material Instruccional de Apoyo. Caracas: UNA

NUEVA VISIÓN DEL MARKETING


La visión del marketing, expone la necesidad de pasar de la orientación producto a la
orientación cliente, las estrategias se inclinan a incrementar los niveles de satisfacción de los
consumidores. Las relaciones con los éstos se estructuran, pensando a largo plazo; uno de
los instrumentos clave para lograr lo antes expuesto lo constituye la gestión de relaciones.

Las necesidades, deseos y expectativas de los clientes condicionan todos los pensa-
mientos y dirigen todos los procesos de planificación de la organización. La cultura, las estra-
tegias, las competencias básicas, la estructura, el personal, los procesos y procedimientos
así como el estilo de gestión están claramente alineados con los mejores intereses de los
clientes.

Para lograr la alineación antes mencionada la empresa necesita:


•• Conocer a fondo en todas sus dimensiones y características, las necesi-
dades, deseos y expectativas de los clientes.
•• Mantener en toda la organización una actitud permanente de trabajar
para satisfacer esas necesidades, deseos y expectativas, creando el
máximo valor para el cliente.
•• Trabajar siempre al lado del cliente, con una total coordinación interfun-
cional centrada en su satisfacción como el mayor valor que es enfocado
para lograr máxima rentabilidad a largo plazo.
•• Incorporar en toda la estructura de la empresa las expectativas del clien-
te. Cualquiera incidente con el cliente es un asunto de la alta dirección,
siendo estratégico el manejo de quejas.

MARKETING OPERATIVO O MARKETING MIX

El marketing mix son los instrumentos que utiliza la empresa para implantar las estra-
tegias de marketing y lograr los objetivosfijados. Estas herramientas son conocidas también
como las P’s del marketing. El Marketing Mix consiste en la perfecta mezcla de las 4 P (pro-
ducto, precio, plaza, promoción) de tal manera que formen un conjunto coordinado:

1. Producto: es todo aquello tangible o intangible (bien o servicio) que se va a


introducir en un mercado anterior al desarrollo, análisis y distribución por parte
de la empresa. Es importante también el desarrollo específico de marca y las
características del empaquetado, etiquetado y envase.

2. Precio: se ha de fijar en función de los beneficios que se desean obtener. Aquí


empieza el proceso de intercambio con los agentes del mercado por lo que a la
hora de fijar un precio hay que tener en cuenta los competidores, el gasto medio
de los agentes.

479
Sánchez Marchán, Nanci. (2014).Introducción a la Ingeniería Industrial, Material Instruccional de Apoyo. Caracas: UNA

3. Plaza: es lo que actualmente se llama distribución y se trata de los lugares don-


de va a ser comercializado el producto. Un ejemplo de marketing operativo o mix,
centrado en la plaza, es el de las grandes franquicias las cuales hacen estudios
rigurosos antes de abrir una nueva tienda y siempre con las mismas caracterís-
ticas de triunfo de la primera.

4. Promoción: todas las funciones realizadas para que el mercado se entere de la


existencia del producto/marca, incluidas la venta y apoyos a la venta, sea ésta la
gestión de los vendedores, oferta del producto o servicio por teléfono, Internet u
otros medios similares, los anuncios publicitarios y la publicidad mediante otros
vehículos.

Se refieren las cuatro P, a las variables de decisión sobre las cuales una compañía tiene
mayor control. Estas variables se construyen alrededor del conocimiento profundo de las
necesidades del consumidor.

Ejemplo Ilustrativo
¿En qué medida podría ser afectada la empresa VIÑEDO AZUL C.A., en cuanto
a los objetivos de comercialización de sus vinos con años de maduración, para
los cuales se utiliza la misma marca y establecimientos donde se comercializan
los vinos comunes. Mediante sistemas de distribución masiva, con niveles de
precio y calidad bajos.
Se conoce además que el segmento de la población que está incrementando su
consumo es el medio-alto, que prefiere vinos de alta calidad. ¿Qué razones po-
drían llevar a la empresa a adoptar una estrategia de marca/producto?

Al tratar de igual manera en cuanto a la distribución masiva los vinos comunes (de baja
calidad) y los vinos con años de maduración, es decir, los de calidad superior, VIÑEDO AZUL
C.A., corre el riesgo de que estos últimos sean desestimados por parte del cliente, o bien,
que su precio pueda parecer excesivo, tratándose de una empresa que trabaja tanto con
vinos de alta calidad, como con vinos de mesa de menor calidad.

Además, teniendo en cuenta que los hábitos de consumo que se están incrementando
son los de un segmento medio-alto, que prefiere vinos de alta calidad con años de madura-
ción, parece peligroso comercializar estos, bajo la misma marca y en los mismos estableci-
mientos que los vinos comunes, los cuales pueden deslucir los que tienen un alto grado de
maduración. Sin embargo, si comercializa los vinos comunes con distinta marca que los de
mayor calidad, no tiene porque tener problemas, ya que la diferenciación puede ser fácilmen-
te captada.

PLAN DE MARKETING, MERCADEO O MERCADOTECNIA


En el marketing, como en cualquier actividad gerencial, la planificación constituye un
factor clave para minimizar riesgos y evitar el desperdicio de recursos y esfuerzos.

480
Sánchez Marchán, Nanci. (2014).Introducción a la Ingeniería Industrial, Material Instruccional de Apoyo. Caracas: UNA

El plan de marketing, mercado o mercadotecnia es lo que le permite a cada empresa al-


canzar sus objetivos de maximización de beneficios, siempre y cuando dicho modelo sea el
adecuado. Los objetivos deben ser específicos e influir en una meta mensurable, a un costo
definido, con fecha de terminación precisa.
Razones fundamentales que justifican el uso de objetivos medibles:
•• Si no se establece una medida, nunca se sabría si los objetivos fueron
alcanzados.
•• Se constituye en un banco de datos para la preparación de futuros planes
de mercadeo.
Un plan de mercadeo, marketing o mercadotecnia debe ser ante todo una herramienta
de comunicación:
•• Hacia arriba con sus jefes para lograr su cooperación con el plan.
•• Hacia abajo con sus subalternos para que comprendan la importancia de
trabajar en equipo.
•• Hacia los lados para lograr la cooperación de las otras unidades planifi-
cadoras de la empresa.

CONTENIDO DE UN PLAN DE MARKETING, MERCADEO O MERCADOTEC-


NIA
El contenido de un plan de marketing, mercadeo o mercadotecnia, se muestran en el
siguiente esquema:
PLANIFICACIÓN
ESTRATEGICA:
identificación del negocio,
análisis de situación actual,
selección de estrategias, esta-
blecimiento de controles.

EJECUCIÓN DEL PLAN DE MERCADEO CONTROL


PROGRAMA DE MARKETING O • control de plan anual.
MARKETING MERCADOTECNIA • control de rentabilidad.
Ejecución de las acciones pla- • control de eficiencia.
neadas. Se fijan los medios, • control estratégico.
procedimientos y técnicas • control de ventas.
para llevarlas acabo.

INVESTIGACIÓN DE
MERCADO:
• Estudio de Mercado.
• Investigación de comu-
nicación.
• Gerencia de ventas

Figura 76. Contenido de un plan de mercadeo, marketing o mercadotecnia.

481
Sánchez Marchán, Nanci. (2014).Introducción a la Ingeniería Industrial, Material Instruccional de Apoyo. Caracas: UNA

PLANIFICACIÓN ESTRATÉGICA
Analiza el mercado (de la demanda, de la oferta, de la competencia). Este aspecto pro-
cede directamente de la dirección, con el conocimiento de las necesidades de la población.
Tras un estudio riguroso del mercado se analizan las oportunidades y la relación con los
posibles competidores. Deberá llevar un estudio de los ingresos, los costos, la maximización
de los beneficios y el modo de distribución de los productos.

Todos los planes de mercadeo siguen el patrón de la planeación estratégica y se deben


formular preguntas como:
•• ¿Cuál es la ubicación actual?
•• ¿Hacia donde se quiere ir?
•• ¿Cómo se hace para llegar allí?

INVESTIGACIÓN DE MERCADO
•• Estudio de mercado: se utiliza para averiguar cuáles son los cambios
que se están produciendo en el mercado y para conocer como se en-
cuentra la empresa con relación a éste. Se deben considerar siguientes
pasos:

Paso 1. Identificación del problema: se deberá identificar el principal problema que


se quiere resolver en la empresa, por ejemplo: conocer los competidores, sus precios de
venta, dónde distribuir el producto, entre otras cosas.

Paso 2. Establecer los objetivos: es decir, a lo que se quiere llegar con la investiga-
ción. Los objetivos que se podrían plantear para ingresar el producto al mercado pueden ser:
•• Conocer cuál es la demanda potencial.
•• Identificar a la competencia.
•• Definir el segmento de mercado que se quiere alcanzar.
•• Conocer los mejores puntos de ventas.
•• Describir que cambios se deben realizar al producto.

Paso 3. Buscar fuentes de información: las fuentes de información son los recursos
que se van a utilizar para averiguar la información que se necesita. Existen distintos tipos de
datos que pueden dar información. Las fuentes pueden ser primarias o secundarias. 

Los datos primarios son los que se obtienen a través de:


•• Investigación de campo.
•• Entrevistas.
•• Reuniones de grupo.
•• Encuestas.
•• Observación directa.

482
Sánchez Marchán, Nanci. (2014).Introducción a la Ingeniería Industrial, Material Instruccional de Apoyo. Caracas: UNA

Los datos secundarios son aquellos que ya existen y están disponibles al público en
distintos medios, Por ejemplo:
•• Internet.
•• Libros.
•• Revistas especializadas.
•• Embajadas.
•• Estudios realizados (Universidades, Organismos de Investigación).
•• Otros.

Paso 4. Diseñar la forma de recolección de datos: una vez que se han decidido los
medios de investigación para recoger toda la información posible sobre el problema de inves-
tigación y sus objetivos, entonces se debe diseñar cómo se van a recoger los datos. Esto es
el diseño de la investigación. Se consideran dos tipos de investigación:
•• Cuantitativa: permite contestar las preguntas en una forma medible, por
ejemplo: ¿Cuál es la producción anual de café?
•• Cualitativa: permite averiguar preferencias, opiniones y críticas hacia un
producto o servicio. Los datos obtenidos no son fácilmente medibles. Por
ejemplo: ¿Le gusta el sabor de este producto?  

Paso 5. Determinar el grupo de interés para el estudio: determinación de la pobla-


ción y muestra. La muestra es una parte de la población total (universo) que quiere ser estu-
diada; por ejemplo, todas las mujeres adultas del área metropolitana.

Paso 6. Selección y preparación de instrumentos: con los objetivos en mente, se


saca la lista de datos que se quieren conseguir y que fuentes son las más adecuadas para
proporcionarlos. Según lo anterior se decide qué instrumentos convienen: entrevistas, en-
cuestas, observación, grupos de discusión, etc.

Paso 7. Análisis de datos: después de recoger los datos se busca darle significado a
la información o sea interpretarlos. Para eso se organizan, utilizando diferentes técnicas,
entre las que se pueden mencionar las tablas, gráficos. Para analizar los datos se usan va-
rios tipos de métodos estadísticos tales como medias, frecuencias y otros tipos de análisis
más complejos.

Paso 8. Presentación de resultados y conclusiones: el último paso del proceso de


investigación es preparar y presentar un informe final.

Estos aspectos los puede ubicar también en el tema de estadística, que se encuentra
en la unidad 4.
•• Investigación de comunicaciones
El propósito principal de la investigación de comunicaciones es determinar si las acti-
vidades específicas en el campo de las comunicaciones publicidad, promoción de ventas y
relaciones públicas están alcanzando sus objetivos.

483
Sánchez Marchán, Nanci. (2014).Introducción a la Ingeniería Industrial, Material Instruccional de Apoyo. Caracas: UNA

EJECUCIÓN DEL PROGRAMA DE MARKETING


Una vez realizado el programa de actuación sobre el nuevo producto es hora de llevarlo
a cabo. En esta etapa todo el personal de la empresa ha de actuar de acuerdo con lo esta-
blecido para lograr el éxito del producto y conseguir la maximización de los beneficios desea-
dos. Se le asigna al departamento correspondiente la ejecución de las acciones planeadas
y se fijan los medios para llevarlas a cabo, así como los procedimientos y las técnicas que
se utilizarán. Igualmente deben crearse mecanismos que permitan evaluar los resultados del
plan establecido y determinar cuán efectivo ha sido.

CONTROL

Con el fin de modificar el programa de marketing, si fuera necesario, se han de llevar a


cabo unos mecanismos de control una vez llevado al mercado el producto. Algunos de los
mecanismos de control usados por las empresas son:
•• Control de plan anual.
•• Control de rentabilidad.
•• Control de eficiencia.
•• Control estratégico.
•• Control de ventas.

El control establece mecanismos de retroalimentación y evaluación con los que se pue-


de comprobar el grado de cumplimiento de los objetivos y establecer las correcciones que
correspondan

Sobre la base de la información anterior, se muestra a continuación una guía para el


desarrollo de un plan de mercadeo:

GUÍA PARA LA ELABORAR UN PLAN DE MARKETING,


MERCADEO O MERCADOTECNIA

Portada: Nombre de la organización, título del documento, año


fiscal del Plan de Mercadeo

484
Sánchez Marchán, Nanci. (2014).Introducción a la Ingeniería Industrial, Material Instruccional de Apoyo. Caracas: UNA

Antecedentes: Presenta los acontecimientos hasta el momento


de provisión del producto o servicio, como serían las condiciones
del mercado y la experiencia de la compañía.

Definición del Producto o Servicio: Especifica el producto y/o


servicio propuesto por el Plan de Mercadeo.

Mercado Meta, Mercado Objetivo o Target: Describe al com-


prador típico del producto y/o servicio. El mercado se puede defi-
nir según el tipo de cliente, región geográfica, sector de la indus-
tria o cualquier otro tipo de segmentación significativa.

Aspectos Sociales y Gubernamentales:


• Clima legislativo. Está fundamentado en leyes que proceden
de relaciones derivadas de la naturaleza de las cosas y las re-
laciones sociales. Permite conocer toda la normativa que afec-
ta una actividad.
• Política fiscal y monetaria. La política fiscal afecta el gasto
público y los impuestos, que impactan en la economía de mer-
cado de bienes y servicios. La política monetaria a través del
BCV, en el caso de Venezuela, controla la canidad de dinero en
circulación y los tipos de interés orientados hacia el crecimiento
de la economía.
• Protección al consumidor. Defensa, protección y salvaguar-
da de los derechos e intereses de los consumidores y usuarios.
• Clima general del entorno. Se refiere a las condiciones eco-
nómicas, políticas y sociales reinantes en un país. Un clima
adecuado que genere estabilidad en las políticas y en las re-
glas que rigen la inversión.

Análisis Competitivo. Se provee una declaración de las activida-


des de las organizaciones competidoras para identificar la fuente
y naturaleza de la competencia. Debe identificarse los nombres y
las ofertas de las principales organizaciones competidoras.

485
Sánchez Marchán, Nanci. (2014).Introducción a la Ingeniería Industrial, Material Instruccional de Apoyo. Caracas: UNA

Matriz FODA: Análisis de Fortalezas, Oportunidades (facto-


res internos), Debilidades y Amenazas (factores externos).
Especifica los puntos fuertes y débiles de la organización del pro-
ducto/servicio ofrecido. Discute las futuras oportunidades y posi-
bles amenazas de los competidores y otras fuente (p. ej., cambios
reglamentarios).

Posicionamiento del Mercado. Detalla cómo se está promo-


viendo a la organización y al producto/servicio frente a potencia-
les clientes, con énfasis en los atributos únicos de la organización
y del producto/servicio.

Consideraciones Estratégicas. Describe aspectos a largo pla-


zo que enfrenta la organización y el producto/servicio y cómo se
avanzará con el tiempo.

Plan Táctico. Define las actividades y herramientas específicas


que se desarrollarán y llevarán a cabo durante el período del plan
de mercadeo.

Herramientas de Venta. Describe las herramientas que se desa-


rrollarán para el uso del personal de ventas (p. ej., página Web de
demostración, folletería, descripción de servicios, etc.).

Generación de Contactos. Representa la forma en como se ge-


nerarán nuevos negocios (p.ej., llamando a potenciales clientes
por teléfono, asistiendo a ferias, conversando con clientes actua-
les y solicitando referencias, publicidad, etc.).

Investigación y Análisis Vertical / Competitivo. Refiere la in-


vestigación y el análisis que se hará de la segmentación del mer-
cado y de las actividades de la competencia, para filtrar mejor las
actividades de venta y mercadeo.

Investigación de Nuevos Mercados. Describe las actividades


que se llevarán a cabo para encontrar nuevos mercados y clien-
tes.

486
Sánchez Marchán, Nanci. (2014).Introducción a la Ingeniería Industrial, Material Instruccional de Apoyo. Caracas: UNA

Calcular el presupuesto: Se convierte en números todo lo an-


terior, según las estrategias y prioridades, es decir, presupuestar
cada acción y totalizar.

Ejecución y Control: Para cada objetivo que se haya determina-


do, se debe tener una cifra de referencia para luego poder com-
parar. Es decir, los resultados de la implementación del plan de
mercadeo serán monitoreados y medidos a través de contactos
que resultan de eventos; contactos que resultan de publicidad;
contactos que resultan de una página Web; cobertura lograda en
los medios; asistencia a seminarios de información; relación con-
tactos/ventas (medida de calidad).

Resumen ejecutivo: Por último se realiza un resumen de los pun-


tos principales del plan y las conclusiones esenciales que se han
obtenido de todo el trabajo anterior. Con la finalidad de entregarlo
a los diferentes gerentes de la empresa.

Ejemplo Ilustrativo

Elaboración del Plan de Mercadeo del producto: Té Natural de limón con hojas de menta.
Portada: Nombre de la organización, título del documento, año fiscal del Plan de Mer-
cadeo

Antecedentes: Presenta los acontecimientos hasta el momento de provisión del pro-


ducto o servicio, como serían las condiciones del mercado y la experiencia de la
compañía.

Definición del Producto o Servicio: Es un producto elaborado a base de menta, esta


bebida contienen una extraordinaria capacidad para saciar la sed en los meses más
calurosos. La menta posee propiedades antiespasmódico y es un poderoso digestivo,
estomacal. Por otra parte, es bactericida y antiséptico. También sirve para bajar de
peso, debe realizar una dieta baja en calorías. Los tés son de mucha utilidad como
complemento.

Mercado Meta, Mercado Objetivo o Target: Hombres y mujeres entre 14 y 65 años.


Por el clima tropical de Venezuela, es un producto que se consumirá en todo el año.

487
Sánchez Marchán, Nanci. (2014).Introducción a la Ingeniería Industrial, Material Instruccional de Apoyo. Caracas: UNA

Aspectos Sociales y Gubernamentales:


•• Clima legislativo: conocer las regulaciones en el ejercicio del comercio
de las bebidas refrescantes y todas aquellas regulaciones que afecten
este negocio, antes de tomar decisiones.
•• Política fiscal y monetaria: Las políticas fiscal y monetaria en Vene-
zuela enfrentan limitaciones para lograr que la economía sea estable. No
existe un libre juego entre la oferta y la demanda de bienes y servicios, ni
una política monetaria clara.
•• Protección al consumidor: existe un ente gubernamental en el país,
con la competencia para regular y controlar el comercio interior de la be-
bida té natural de limón con hojas de menta, teniendo atribuciones para
fijar su precio al mayor y al detal.
•• Clima general del entorno: la empresa productora de té puede consi-
derarse como un sistema abierto, en el que influye y recibe influencias de
otras empresas de la competencia.

Análisis Competitivo: Competencia directa. Está compuesta por la gran variedad


de té fríos que se encuentran en el mercado venezolano.

Competencia indirecta. Está compuesta por todos aquellos productos que satisfacen
la misma necesidad, como lo son jugos, agua, refrescos, etc.

Análisis de Fortalezas, Oportunidades, Debilidades y Amenazas (FODA).

Factores internos:

Fortalezas:
•• Única bebida de té frío de limón con un toque de menta.
•• Es más refrescante.
•• Es beneficiosa para la salud.
•• Es adelgazante.

Debilidades:
•• Debe añadirse poca menta para no alterar el sabor.
•• No es una bebida tan dulce como la que produce la competencia.

Factores Externos:

Oportunidades.
•• Los consumidores prefieren productos naturales.
•• Los consumidores buscan verse bien y estar en forma.
•• El clima tropical favorece el lanzamiento de bebidas refrescantes.

488
Sánchez Marchán, Nanci. (2014).Introducción a la Ingeniería Industrial, Material Instruccional de Apoyo. Caracas: UNA

•• Las ventas en el mercado nacional de té frio listo para tomar son de 30


millones de litros anuales.

Amenazas:
•• Alta cantidad de bebidas refrescantes en el mercado, como son Nestea
y Lipton Ice Tea, entre otros.
•• Competimos con marcas reconocidas que tienen años de experiencia en
el mercado.
•• El consumidor venezolano no está acostumbrado a una bebida a base
de menta.
•• La Competencia tiene una buena red de distribución.

Posicionamiento del Mercado: Los canales de distribución y a su vez comercializa-


ción serán a nivel nacional.

Destacan los siguientes: Supermercados, Hipermercados, Tiendas de mayoristas,


Tiendas de conveniencia, Auto farmacias.

Plan de Promoción y Publicidad: Televisión, radio, revistas, vallas publicitarias, publi-


cidad en autobuses. Se pretende adquirir también un plan de circuitos con autobuses en las
principales rutas de este medio de tranporte del país.

Consideraciones Estratégicas: Describe aspectos a largo plazo que enfrenta la orga-


nización y el producto/servicio y cómo se avanzará con el tiempo.

Plan Táctico: Define las actividades y herramientas específicas que se desarrollarán y


llevarán a cabo durante el período del plan de mercadeo.

Herramientas de Venta: Página Web de demostración, folletería, descripción de ser-


vicios, etc.).

Generación de Contactos: Se contactarán a través de llamadas telefónicas a clientes


potenciales, asistiendo a ferias, conversando con clientes actuales y solicitando referencias,
publicidad, etc.

Investigación y Análisis Vertical/Competitivo: Fresh tea no tiene una participación


de mercado representativa por tratarse de un producto nuevo. Entre sus competidores di-
recto, el líder es Neste con una participación de 65% del mercado de té frio en Venezuela,
el porcentaje restante se divide entre las otras marcas. El Precio es más bajo que el de sus
competidores directos e indirectos.

Investigación de Nuevos Mercados. Fresh tea lograría entrar en nuevos mercados,


recurriendo al precio de penetración, que consiste en fijar desde el principio unos precios
suficientemente bajos con el objetivo de incrementar la demanda; el incremento de ventas

489
Sánchez Marchán, Nanci. (2014).Introducción a la Ingeniería Industrial, Material Instruccional de Apoyo. Caracas: UNA

repercutiría en unas economías de escala que provocarían una reducción de costes y el


precio podría seguir manteniéndose bajo.

Calcular el presupuesto:
Presupuesto de Televisión = 418.206,00 Bs.
Presupuesto de Radio = 67.950,00 Bs.
Presupuesto de revistas = 90.650,00 Bs.
Presupuesto de Vallas = 85.400,00 Bs.
Presupuesto de Publicidad en Autobuses = 285.000,00 Bs.
Presupuesto total = 1.047.206 Bs.
Ejecución y Control: Los resultados de la implementación del plan de mercadeo se-
rán monitoreados y medidos a través de contactos que resultan de eventos; Contactos que
resultan de publicidad; contactos que resultan de una página Web; cobertura lograda en
los medios; asistencia a seminarios de información; relación contactos/ventas (medida de
calidad).

Resumen ejecutivo: Por último se realiza un resumen de los puntos principales del
plan y las conclusiones esenciales que se han obtenido de todo el trabajo anterior.

Ejercicio propuesto
Investigue en medio impreso o electrónico las siguientes interrogantes:
1. ¿Cuáles son los factores que deben considerarse a la hora de diseñar y
seleccionar los canales de distribución?

2. ¿Cuál puede ser la estrategia para la elección de mercados para los próxi-
mos años?

En conclusión con el Marketing, Mercadotecnia o Mercadeo se gestiona el mercado


o la comercialización del producto o productos de una organización, se busca fidelidad de
clientes, mediante herramientas y estrategias, posicionando en la mente del consumidor un
producto, marca, etc., que sea la opción principal. Parte de las necesidades del cliente o con-
sumidor para diseñar, organizar, ejecutar y controlar la función comercializadora o mercadeo
de la organización.

490
Sánchez Marchán, Nanci. (2014).Introducción a la Ingeniería Industrial, Material Instruccional de Apoyo. Caracas: UNA

COMPORTAMIENTO ORGANIZACIONAL

•• Historia del C.O


•• Disciplinas que han contribuido a desarrollar el Comportamiento
Organizacional.
•• Distribución del Comportamiento Organizacional:
- Individuo
- Grupo
- Estructura

ORIGEN DEL COMPORTAMIENTO ORGANIZACIONAL


El Comportamiento Organizacional ha recibido diferentes nombres desde su origen. Sus
antecedentes se pueden precisar desde el mismo momento en que apareció la necesidad
de organizar las grandes industrias que surgieron con la Revolución Industrial (ver unidad 1,
historia de la Ingeniería Industrial). También se le denominó sicología industrial o relaciones
industriales, este último término fue bastante aceptado y aún sigue vigente.

Esta ciencia aplicada ha recibido aportaciones de diversas disciplinas conductuales ta-


les como: sicología, antropología, sociología, ciencia política, entre otras. Se fundamenta en
un grupo de teorías tales como:
•• Teoría organizacional antes de 1900 (Adam Smith, Charles Babbage, en-
tre otros) ver tabla Nº 2.
•• Teoría organizacional entre 1900 y mediados de la década del 30 (Frede-
rick, Henry Gantt y los esposos Gilberth Taylor, Henry Fayol, entre otros)
ver tabla Nº 2.
•• Teoría organizacional entre las décadas de los 30 y los 50 (Elton Mayo,
Abraham Maslow, Douglas McGregor, entre otros).
•• Teoría organizacional de los 60 a la fecha (teoría de sistemas, Teoría
situacional y contingencial, entre otras).

En atención a lo antes expuesto se puede definir el Comportamiento Organizacional


como:

Un campo de estudio en donde se investiga el impacto que individuos, grupos y estruc-


turas tienen en la conducta interna de las organizaciones, con la finalidad de aplicar un
conjunto de conocimientos para lograr la eficacia y eficiencia de dichas organizaciones.

Esta disciplina científica agrega constantemente a su base de conocimientos, una gran


cantidad de investigaciones y desarrollos conceptuales. El Comportamiento Organizacional

491
Sánchez Marchán, Nanci. (2014).Introducción a la Ingeniería Industrial, Material Instruccional de Apoyo. Caracas: UNA

se interesa particularmente en las situaciones que atañen al empleo, por lo que se destaca el
comportamiento en lo que se refiere al trabajo, puestos, ausentismo, rotación, productividad,
desempeño humano, administración, entre otros.

DISTRIBUCIÓN DEL COMPORTAMIENTO ORGANIZACIONAL

El comportamiento organizacional se distribuye como se observa en la figura mostrada:

INDIVIDUO GRUPO Y EQUIPOS ESTRUCTURA

Figura 77. Distribución del comportamiento organizacional.

EL INDIVIDUO Y LAS BASES DE SU CONDUCTA


La personalidad de un individuo se refiere a una serie de características particulares,
con las cuales se muestran motivos, emociones, valores, intereses, actitudes y competen-
cias. Organizadas en el entorno social, cultural y familiar, se puede decir que este elemento
influye de manera determinante en el desempeño laboral y el éxito de la organización.

Es posible prever el comportamiento de las personas en las organizaciones en cuanto a


la orientación a la realización, el autoritarismo, el maquiavelismo, autoestima, el autocontrol
y la tendencia a correr riesgos. No de manera inmediata o sencilla, pero existen factores
que son fáciles de identificar en todas las personas, es decir, las características biográficas,
como por ejemplo edad, género, estado civil, antigüedad, entre otras.

Existen diferencias entre los individuos, en cuanto a sus habilidades y actitudes. La ha-
bilidad es la capacidad que un individuo tiene para realizar las diversas tareas de su trabajo.
Básicamente, las habilidades generales están compuestas por dos grupos de factores: habi-
lidades intelectuales y habilidades físicas.

Las habilidades intelectuales. Son aquellas que se utilizan para realizar las activida-
des mentales, éstas se pueden medir a través de test o pruebas para organizaciones, escue-
las, dependencias gubernamentales. Existen siete dimensiones: aptitud numérica, compren-
sión verbal, velocidad perceptual, razonamiento inductivo, visualización espacial y memoria.
A continuación se presenta un cuadro donde se muestran cada una de estas dimensiones,
con una descripción y un ejemplo:

492
Sánchez Marchán, Nanci. (2014).Introducción a la Ingeniería Industrial, Material Instruccional de Apoyo. Caracas: UNA

Tabla 49. Dimensiones de las habilidades intelectuales de la conducta de los individuos


Dimensión Descripción Ejemplo en el Trabajo
Aptitud Habilidad para la velocidad y la preci- Contable: cálculo del impuesto so-
numérica sión aritmética. bre ventas en una serie de artículos.
Comprensión Habilidad para entender lo que se Gerente de Planta: sigue las políti-
verbal lee o escucha y la relación entre las cas de las corporativas.
palabras.
Velocidad de Habilidad para identificar las similitu- Investigador de Incendios: identi-
percepción des visuales y diferencias rápidamen- ficar pistas para apoyar un caso
te y con precisión. de incendio (Higiene y Seguridad
Industrial)
Razonamiento Habilidad para identificar una se- Investigador de mercados: pronosti-
inductivo cuencia lógica en un problema para ca la demanda de un producto en el
resolverlo. siguiente período.
Razonamiento Habilidad para usar la lógica y evaluar Supervisor: elegir entre dos suge-
deductivo las implicaciones de un argumento. rencias ofrecidas por los empleados.
Visualización Habilidad para imaginar cómo se vería Decorador de interiores: redecora
espacial un objeto si se le cambiara la posición una oficina.
de los objetos en el espacio.
Memoria Capacidad para retener y recordar Vendedor: recordar los nombres de
experiencias. los clientes.

Las habilidades físicas. Son requerimientos necesarios para hacer tareas que deman-
dan fuerza, vigor, destreza, donde la capacidad física es la que será identificada por la ge-
rencia. Se han identificado nueve habilidades básicas involucradas en la realización de las
tareas físicas; las cuales son descritas a continuación en la tabla 50:

Tabla 50. Habilidades físicas

Habilidades Físicas
FACTORES DE FUERZA
Habilidad para aplicar fuerza muscular repetida o continua du-
Fuerza dinámica
rante un tiempo.
Habilidad para aplicar fuerza muscular usando el músculo del
Fuerza torácica
torso (en particular los abdominales)
Fuerza estática Habilidad para ejercer fuerza contra objetos externos.
Habilidad para gastar un máximo de energía en una serie de
Fuerza explosiva
actos explosivos.
FACTORES DE FLEXIBILIDAD
Flexibilidad de extensión Habilidad para alargar los músculos del tronco y la espalda.
Flexibilidad dinámica Habilidad para hacer flexiones rápidas y repetidas.
OTROS FACTORES
Habilidad para coordinar acciones simultáneas con partes dife-
Coordinación corporal
rentes del cuerpo.
Habilidad para mantener el equilibrio a pesar de las fuerzas des-
Balance
equilibradoras.
Habilidad para continuar el máximo esfuerzo prolongado reque-
Vigor
rido.

493
Sánchez Marchán, Nanci. (2014).Introducción a la Ingeniería Industrial, Material Instruccional de Apoyo. Caracas: UNA

Todas las conductas complejas son aprendidas. Cualquier cambio observable en la


conducta es prueba directa de que ha ocurrido un aprendizaje. Desde luego, los conceptos
de aprendizaje proveen de conocimientos para explicar y pronosticar la conducta.

Se han postulado algunas teorías para explicar el proceso por el cual se adquieren
pautas de conducta, entre las que se encuentran las teorías del conocimiento clásico, el
condicionamiento operante y el aprendizaje social:
•• Teoría del condicionamiento clásico. El individuo responde a un esti-
mulo que de ser ordinario, no produciría respuesta.
•• Teoría del condicionamiento operante. Reconoce a un tipo de condi-
cionamiento deseado, el cual lleva a la compensa o a la prevención del
castigo
•• Teoría del aprendizaje social. Afirma esta teoría que las personas
aprenden por observación y experiencia directa.

El término satisfacción en el trabajo siempre será un punto clave a tratar, ya que de


ello depende que el empleado se sienta conforme y demuestre qué tan productivo es, pero
existen factores que pueden influir en su desempeño. A continuación se observa un resumen
(adaptado del texto Comportamiento Organizacional, Stephen P. Robbins), donde se consi-
deran estos aspectos:

Valores y Actitudes

Los valores. Son objeto de estudio pues son la base para entender las actitudes y las
motivaciones, porque influyen en la percepción. Todos los valores de cada persona tienen
una fuente de procedencia que son: la familia, los amigos, la escuela con los maestros, la
cultura del país donde se viva; pero los valores que regularmente se muestran son los que
adquieren en los primeros años de la vida.

Las actitudes. Son los juicios sobre objetos, personas o acontecimientos, es decir, es
la forma de representar cómo se siente una persona. Al igual que los valores las actitudes
se adoptan de los padres, grupos sociales, maestros. Se nace con cierta predisposición y
a medida que el individuo crece va copiando lo que observa en las personas que respeta,
admira o incluso de los temores. Las actitudes son más inestables que los valores, ya que
son moldeables a la conveniencia de personas o empresas, obteniendo de ellas un compor-
tamiento deseable. Algunas de estas actitudes son:

Satisfacción en el trabajo: es la actitud que un empleado asume respecto a su trabajo.


De forma lógica, aquellas personas que obtienen un alto nivel de satisfacción con sus activi-
dades establecen actitudes muy positivas y benéficas.

Compromiso con el trabajo: es un término que establece el grado en el cual la perso-


na se valora a sí mismo, a través de la identificación sociológica en su puesto dentro de la

494
Sánchez Marchán, Nanci. (2014).Introducción a la Ingeniería Industrial, Material Instruccional de Apoyo. Caracas: UNA

empresa. A los trabajadores plenamente identificados realmente les importa el trabajo que
realizan.

Compromiso organizacional: se refiere a la identificación del empleado con la em-


presa, metas, objetivos, es decir, identificarse con el servicio personal a la organización y el
compromiso con su labor específica.

Personalidad y emociones

La personalidad es el resultado de factores hereditarios y ambientales, influenciados


por las condiciones situacionales.

Existe un indicador, el Meyers - Briggs (MBTI por sus siglas en inglés) que es un examen
de personalidad de 100 preguntas, se clasifica a los individuos en cuatro tipos de persona-
lidad: extrovertidos o introvertidos, sensibles o intuitivos, racionales o pasionales y percepti-
vos o juiciosos.

Los atributos de la personalidad de mayor influencia en el comportamiento organizacio-


nal son denominados pronosticadores, entre los que se pueden mencionar:
•• Locus de control o lugar de control: hay personas que piensan que
ellos son responsables de su estilo de forma de vida y su destino. De
forma interna, controlan lo que les pasa o de forma externa, a través de
fuerzas exteriores.
•• Maquiavelismo: toma el nombre por Maquiavelo, quien escribió acerca
de cómo ganar y usar el poder. Este tipo de personas cree que el fin jus-
tifica los medios, les gusta manipular más y ganar más por lo que no son
fáciles de persuadir.
•• Autoestima: es el grado de aceptación de uno mismo, esta caracte-
rística determina en muchas ocasiones el éxito de las personas, todos
aquellos que tengan una autoestima alta serán capaces de enfrentar re-
tos fuera de sus miedos o condicionantes, además de que no serán tan
susceptibles a las situaciones del exterior.
•• Auto monitoréo: ser capaz de adaptar el comportamiento a las situacio-
nes que se presentan en la vida cotidiana. Establece que las personas
con alto auto monitoréo pueden mostrar diversas caras de sí mismos,
según como se requiera, aunque algunas veces existen contradicciones,
es posible su consideración para lograr ascensos dentro de la empresa
u otras organizaciones.
•• Toma de riesgos: se refiere a que en los puestos de gerencia dentro de
una empresa deben buscarse y aceptar las responsabilidades al tomar
decisiones. Según los estudios, el grado en el que se adopten los riesgos
depende en algunas ocasiones del puesto que se desempeña.

Las emociones son una parte natural de la composición de los individuos y se encuen-
tran estrechamente relacionados con los términos de afecto y estado de ánimo. Los estados

495
Sánchez Marchán, Nanci. (2014).Introducción a la Ingeniería Industrial, Material Instruccional de Apoyo. Caracas: UNA

de ánimos son sentimientos menos intensos que las emociones y carecen de estímulos
contextuales.

Percepción y toma de decisiones

La percepción. Es un proceso a través del cual los individuos organizan e interpretan


las impresiones sensoriales, con el fin de darle un sentido al entorno. Entre los factores que
afectan la percepción se tienen: Factores que la persona percibe: actitudes, motivos, intere-
ses, experiencias, expectativas. Factores en la situación, como por ejemplo: tiempo, entorno
en el trabajo, entorno social. Factores en el objetivo, tal es el caso de: novedad, movimiento,
sonidos, tamaño, antecedentes, proximidad.

El proceso de toma de decisiones. Describe el comportamiento de los individuos


para que un resultado sea el máximo o el óptimo, es decir, se eligen opciones constantes de
máximo valor en el contexto de sus restricciones, siguiendo un modelo de toma racional de
decisiones de seis etapas (ver tema toma de decisiones):

1. Definir el problema.

2. Identificar los criterios de decisión.

3. Dar peso a los criterios.

4. Desarrollar alternativas.

5. Evaluar las alternativas.

6. Elegir la mejor alternativa.

El Comportamiento Organizacional también ha de considerar el concepto de motiva-


ción, el cual puede definirse como:

La voluntad que tienen los individuos para realizar esfuerzos hacia las metas planteadas
por las organizaciones, satisfaciendo al mismo tiempo sus necesidades particulares.

Se dice que las personas motivadas están en constante tensión y liberan esta sensación
a través del esfuerzo. Las necesidades individuales deben también ser compatibles con las
de la empresa para que logren juntarse y obtener el mayor aprovechamiento mutuo.

Durante el estudio del cual ha sido objeto la motivación se han desarrollado varias teo-
rías, las cuales en su contexto han sido muy atacadas y cuestionadas en la actualidad, la
mayoría realizadas durante los años cincuenta del siglo XX y que se deben conocer, ya que
son la base de las teorías actuales. A continuación se hace referencia a cada una de ellas:

Teoría de la Jerarquía de las necesidades (Abraham Maslow): una de las teorías más
conocida, estipula que el hombre tiene 5 necesidades jerarquizadas:
•• Fisiológicas: necesidades físicas hambre, sed, sexo.
•• Seguridad: protección del daño físico y emocional.
•• Social: la aceptación, los amigos, el afecto.

496
Sánchez Marchán, Nanci. (2014).Introducción a la Ingeniería Industrial, Material Instruccional de Apoyo. Caracas: UNA

•• Estima: interna, el ser autónomo, los logros y el entorno que rodea a la


persona.
•• Autorrealización: Convertirse en lo que es capaz de volverse por su
mismo esfuerzo, la satisfacción que uno provoca, la forma eficaz de ha-
cer las cosas.

Maslow separó estas necesidades en dos categorías: 1) de orden bajo (las necesidades
fisiológicas y de seguridad) y 2) De orden alto (social, la estima y la autorrealización). Estas
dos categorías se diferencian por el hecho de que las de nivel alto se satisfacen internamente
y las de nivel bajo se satisfacen de manera externa. Se muestra en la figura 78, cada una de
las necesidades expresadas por Maslow, además del nivel que ocupan según su prioridad.

Autorrealización

Estima

Social

Seguridad

Fisiológicas

Figura 78.Teoría de la jerarquía de las necesidades (Abraham Maslow).

Teoría X y Teoría Y: creada por Douglas McGregor el cual propuso dos posiciones
del ser humano extremadamente opuestas, la teoría X (negativa) y otra (positiva) teoría Y.
McGregor estableció de acuerdo con la teoría X e Y, cuatro premisas que todos los gerentes
adoptaban.

Premisas de la teoría X:

1. A los empleados les disgusta trabajar por lo que tratarán de evitarlo.


2. Ya que no les gusta trabajar hay que controlarlos, reprimirlos y ser amenazados.
3. Los empleados buscarán evitar la responsabilidad, así como la dirección formal,
siempre que les sea posible.
4. La mayoría de los que trabajan consideran la seguridad ante todo y no muestran
una buena motivación.

Premisas de la teoría Y:

1. Los empleados ven en el trabajo la manera de descansar.

497
Sánchez Marchán, Nanci. (2014).Introducción a la Ingeniería Industrial, Material Instruccional de Apoyo. Caracas: UNA

1. Si los empleados se comprometen con sus objetivos buscarán su propia direc-


ción, a través del apoyo de la gerencia.
1. La mayoría de las personas buscan encontrarse con la responsabilidad.
1. El hecho de que no se forme parte de los puestos gerenciales no excluye la toma
de decisiones innovadoras, que beneficien a la empresa.

La teoría de la motivación-higiene. (Frederick Herzberg) concluyó que la gente cuan-


do se siente bien, responde de una manera y por lo contrario cuando se siente mal lo hace de
manera distinta, es decir, a veces la gente no está satisfecha con su trabajo, estado de ánimo
generado por situaciones no deseadas por el individuo. Los gerentes en su afán de arreglar
la situación modifican esos errores, pero no necesariamente logran motivar a los empleados.

Teorías contemporáneas: se les llama así porque son el fundamento de estudio que
se toma en cuenta en la actualidad, no porque se hayan descubierto hace poco, estas son:

1. Teoría ERG: (Clayton Alderfer) contempla tres tipos de necesidades: existencia


(las mismas que Maslow, fisiológicas y de seguridad), relación (el deseo de rela-
cionarse con personas en sociedad) y las de crecimiento (desarrollo personal).
Hay necesidades de nivel bajo que satisfacen las de nivel alto semejante a la
jerarquía de necesidades de Maslow.
2. Teoría de las necesidades de David McClelland: son tres tipos de necesidades
las que explican la motivación:
•• De logro: el de las personas que buscan el éxito a través de sus es-
fuerzos.
•• De poder: hacer que otras personas se comporten como uno lo quie-
re a través de un control.
•• De afiliación: las personas buscan relacionarse con otras de manera
amistosa y cordial.
3. Teoría de la evaluación cognoscitiva: expresa que las recompensas que se
dan de manera extrínseca, como el salario, pueden provocar que el nivel de moti-
vación disminuya porque no se toma en cuenta el nivel de satisfacción intrínseca
que brinda el propio trabajo al empleado.
4. Teoría del establecimiento de metas: sostiene que el establecer metas difíciles
y específicas provocan en el individuo mayor grado de satisfacción y por consi-
guiente una buena motivación para seguir adelante creyendo en sí mismo.
5. Teoría del reforzamiento: la situación interna del individuo no es tomada en
cuenta, sino lo que pasa con el individuo respecto a las acciones que toma dentro
de la empresa. Una forma de impulsar o estimular una conducta es con un reco-
nocimiento, en concordancia con la teoría del reforzamiento, premiar inmediata-
mente una conducta con un elogio estimula su repetición.
6. Teoría de la equidad: se basa en el hecho de que un empleado observa las re-
tribuciones que se le dan y las compara con la de otras personas, buscando ob-

498
Sánchez Marchán, Nanci. (2014).Introducción a la Ingeniería Industrial, Material Instruccional de Apoyo. Caracas: UNA

tener la mayor cantidad de premios o bonos elaborando un juicio de lo que ellos


aportan a la organización.
7. Teoría de las expectativas: una persona tiene expectativas y busca cumplirlas.
El hecho de cumplirla y el grado de recompensa que obtenga será lo atractivo
para ella. Una forma de vincular esta teoría con el desarrollo actual es con el
pago variable. Específicamente los individuos deben percibir una relación estre-
cha entre su desempeño y las remuneraciones que reciben para llevar al máximo
la motivación.

BASES DE LA CONDUCTA DE GRUPOS Y EQUIPOS


Se define grupo como:

El conjunto de dos o más individuos que se relacionan de forma interdependiente y que


se reunen para conseguir objetivos particulares.

Sobre esta base se puede definir grupo de trabajo como:

La reunión de individuos que interactúan para compartir información y tomar decisiones


que ayuden a cada miembro del conjunto a desempeñarse en su área de responsabilidad.

Los grupos pueden ser formales o informales. Los grupos formales son grupos de
trabajo designados, definidos por la estructura de la organización. El grupo informal es un
grupo que no está estructurado formalmente ni está determinado por la organización, apare-
cen como respuesta a la necesidad del contacto social.

En general, los grupos pasan en su evolución por una secuencia básica, que se deno-
mina modelo de las cinco etapas. Estas etapas son: formación, conflicto, regulación, desem-
peño y desintegración.
•• La etapa de formación. Es la primera etapa del desarrollo de los gru-
pos, caracterizada por mucha incertidumbre.
•• La etapa de conflicto. Es la etapa del desarrollo de los grupos caracte-
rizada por los conflictos internos.
•• Etapa de regulación. Se caracteriza por las relaciones estrechas y la
cohesión.
•• Etapa del desempeño. Etapa del desarrollo de los grupos, en la que la
asociación funciona a cabalidad.
•• Etapa de la desintegración. Última etapa del desarrollo de los grupos
temporales, caracterizada por la conclusion de las actividades, más que
por la realización de tareas.

El desempeño potencial de un grupo depende, en buena medida de los recursos que


aporten los miembros en lo individual. Los dos principales recursos de los miembros de un

499
Sánchez Marchán, Nanci. (2014).Introducción a la Ingeniería Industrial, Material Instruccional de Apoyo. Caracas: UNA

grupo son por un lado, conocimiento, capacidades y habilidades, por el otro, características
de personalidad.

Los grupos de trabajo tienen una estructura que da forma al comportamiento de sus
miembros y hace posible explicar y predecir una buena parte del comportamiento de los
individuos en estos. Las variables estructurales son:
•• Liderazgo formal. En general los grupos de trabajo tienen un líder for-
mal señalado con un título como gerente, supervisor, capataz, líder de
proyectos, etc. Este líder cumple con un papel importante para el éxito
del grupo.
•• Roles o papeles. Conjunto de pautas de conductas esperadas y atri-
buidas a alguien que ocupa determinada posicion en una unidad social.
•• Identidad de los roles. Ciertas actitudes y conductas congruentes con
un papel.
•• Percepción de los roles. Punto de vista de un individuo sobre cómo
debe actuar en determinada situación.
•• Expectativa de los roles. Acuerdo no escrito que asienta lo que la ad-
ministración espera del empleado o viceversa.
•• Conflicto de roles. Situación en la que un individuo es confrontado por
expectativa de papeles divergentes.

Equipos de trabajo:

Existe una diferencia marcada entre los conceptos de grupo y equipo. Un equipo se
definine como:

La reunión de individuos que a través de un esfuerzo coordinado, generan una sinergia


en la que el aporte de sus esfuerzos en conjunto, proporciona un desempeño mayor a la
suma de los aportes individuales.

Existen diferentes tipos de equipos entre los que podemos citar los siguientes:

Equipos de solucion de problemas (grupo de cinco a 12 empleados del mismo depar-


tamento que se reúnen para analizar las formas de mejorar la calidad, eficiencia y ambiente
de trabajo)

Equipo de trabajo autodirigidos (grupo de 10 a 15 personas que asumen la resposa-


bilidadades de su antiguo supervisor).

Equipos multidisciplinarios (grupo de empleados de más o menos el mismo nivel


jerárquico pero de diferentes áreas que se reúnen para cumplir una tarea).

Equipos virtuales (equipo que se vale de la tecnologia de computación para enlazar a


sus miembros dispersos, con el fin de alcanzar una meta común).

500
Sánchez Marchán, Nanci. (2014).Introducción a la Ingeniería Industrial, Material Instruccional de Apoyo. Caracas: UNA

Se han hecho muchos esfuerzos por identificar los factores relacionados con la eficacia
de un equipo y se ha llegado a un modelo en el que se agrupan las caracteristicas que per-
miten establecer la eficacia de un equipo. Este modelo es:

Diseño de Trabajo
-Autonomía
-Variedad de habilidades
-Identidad de la tarea
-Significado de la tarea

Proceso Composición
-Proposito común -Capacidad
-Objetivos especificos -Personalidad
-Eficacia de grupo -Funciones y diversidad
-Conflicto -Tamaño
-Ocio Social -Flexibilidad

Contexto
-Recursos adecuados
-Liderazgo
-Clima de confianza
-Evaluación: desempe-
ño y recompensas

Figura 79. Modelo de grupo de características que muestra la eficacia de un equipo.

Otro factor fundamental para la eficacia de cualquier grupo u organización es la comu-


nicación. Es un término que refleja el interés de transmitir información, ideas, sentimientos,
pensamientos, conceptos con el fin de que sean entendidos y que tengan la posibilidad de
ser aplicados en algo de interés común o particular.

501
Sánchez Marchán, Nanci. (2014).Introducción a la Ingeniería Industrial, Material Instruccional de Apoyo. Caracas: UNA

El proceso de la comunicación se da a través de una fuente (información), la codifica-


ción, el mensaje, el canal, la decodificación, el receptor y la retroalimentación.

Existen direcciones en la comunicación, puede realizarse de manera descendente que


va de los puestos gerenciales a los niveles más bajos. De manera ascendente, contraria-
mente a la anterior, las personas de un puesto inferior hacen llegar sus quejas o sugerencias
a su jefe inmediato o a la dirección general si así lo requiere el caso. Otra forma de comuni-
carse es de manera lateral y es la que se da entre los mismos miembros del grupo.

Existen canales de comunicación que se presentan dentro de una organización: las


redes formales, o aquellas que se relacionan con las actividades de la empresa y se llevan
a cabo de manera descendente. Y la red informal que comúnmente se le llama rumor o
chisme.

Con la llegada de nuevos medios electrónicos (teléfono, computadoras, e-mail, ser-


vidores), se facilita la comunicación y por ende las barreras se han minimizado. Esto ha
revolucionado la obtención de información de manera ágil y rápida, la localización de las
personas (celulares, localizadores, entre otros) en cualquier momento y las comunicaciones
en línea (teléfonos con pantalla digital o videoconferencias) tanto dentro del país como con
el extranjero.

Estos canales permiten cerrar contratos, realizar inversiones, tener información opor-
tuna y veraz acerca de lo que se requiere, además de ser considerados necesarios para la
empresa u organización.

Con los grupos y equipos surge el liderazgo. El liderazgo cumple una funcion central
para entender el comportamiento de los grupos y equipos, ya que el líder es quien general-
mente señala la dirección para cumplir una meta.

Se puede definir el liderazgo como:

La influencia que una persona puede ejercer sobre un grupo o equipo, para alcanzar las
metas u objetivos trazados .

Max Weber (1864-1920), economista y sociólogo alemán, distingue tres tipos de lideraz-
go: el líder carismático, al que sus seguidores le atribuyen condiciones y poderes superiores
a los de otros dirigentes; el líder tradicional, que hereda el poder, ya sea por la costumbre de
que ocupe un cargo destacado o porque pertenece a un grupo familiar que ha tenido el poder
por largo tiempo; y el líder legal, que asciende al poder por los métodos reglamentarios, ya
sean las elecciones o votaciones, o por demostrar su experiencia sobre los demás.

Además deMax Weber, otros estudiosos de la influencia de algunos individuos sobre


otros, han presentado algunas teorías del liderazgo, que se exponen a continuación:

502
Sánchez Marchán, Nanci. (2014).Introducción a la Ingeniería Industrial, Material Instruccional de Apoyo. Caracas: UNA

Teoría de los Sistemas de Likert

Sugiere que los estilos de administración se dividen en 4 sistemas con sus extremos en
el sistema 1 (autoritario) y el sistema 4 (participativo).

Sistema 1. Autoritario explotador: Los directivos son muy autoritarios, confían poco
en los subordinados, motivan mediante el temor y el castigo, ofrecen recompensas ocasio-
nales y sólo participan en la comunicación descendente. Las decisiones se toman en los
niveles superiores de la organización.

Sistema 2. Autoritario pero paternal: Las personas directivas son condescendientes


con los subordinados, motivan con recompensas, en parte, con el temor y el castigo; permi-
ten alguna comunicación ascendente, solicitan algunas ideas u opiniones a los subordinados
y existe una cierta delegación de la toma de decisiones, pero controlan con políticas.

Sistema 3. Consultivo con derecho a tener la última palabra: Los directivos tienen
una cierta confianza en los subordinados, pero no completa. Están en los flujos de informa-
ción ascendente y descendente; toman decisiones generales y de política amplia en el nivel
superior; pero permiten la toma de decisiones en niveles inferiores.

Sistema 4. Participativo y democrático: En este caso, los directivos tienen una con-
fianza completa en los subordinados, recompensan económicamente de acuerdo con la par-
ticipación e integración del grupo en la fijación de objetivos y según lo que se ha conseguido.
Participan en la comunicación ascendente y descendente, promueven la toma de decisiones
en toda la organización.

Rensis Likert y su grupo de colaboradores, llegaron a la conclusión de que el sistema


más eficiente era el 4. Las críticas a este sistema se fundamentan, en el hecho de que los
estudios toman normalmente a pequeños grupos, pero no a toda la empresa, y por otra par-
te, en el hecho de que se han realizado en momentos de prosperidad.

Teoría de contingencia

La palabra contingencia significa algo incierto o eventual, que bien puede suceder o no.
Se explica aquí que existe una relación funcional entre las condiciones del ambiente y las
técnicas administrativas apropiadas para el alcance eficaz de los objetivos de la organiza-
ción.

Para la teoría de la contingencia no existe una universalidad de los principios de ad-


ministración ni una única mejor manera de organizar y estructurar las organizaciones. El
ambiente impone desafíos externos a la organización, mientras que la tecnología impone
desafíos internos. Para enfrentarse con los desafíos externos e internos, las organizaciones
se diferencian en tres niveles organizacionales:
•• Nivel institucional o nivel estratégico.
•• Nivel intermedio.
•• Nivel operacional.

503
Sánchez Marchán, Nanci. (2014).Introducción a la Ingeniería Industrial, Material Instruccional de Apoyo. Caracas: UNA

Liderazgo situacional
Uno de los modelos de liderazgo más extensamente seguidos es la teoría del liderazgo
situacional de Paul Hersey y Kenneth Blanchard.

Esta teoría se basa en dos variables críticas del comportamiento del líder: cantidad de
dirección (conducta de tarea) y cantidad de apoyo socioemocional (conducta de relación) que
el líder debe proporcionar en una situación a raíz del nivel de madurez de sus subordinados.
•• La conducta de tarea: grado en el que el líder explica lo que deben hacer
sus seguidores, cuándo, dónde y cómo realizar la tarea.
•• La conducta de relación: grado en el que el líder proporciona apoyo so-
cioemocional.

Modelo camino-meta

La teoría del camino-meta es un modelo de contingencia del liderazgo que se inspira


en los estudios de la Ohio State, referentes a la consideración e iniciación de estructura.
Expresa que un buen líder aclara el camino para que sus seguidores lleguen al logro de sus
objetivos de trabajo y recorran el camino con menos dificultades, al reducir las desviaciones
y errores. La iniciación de la estructura sirve para aclarar el camino y la consideración facilita
el recorrido.

House identificó cuatro comportamientos de liderazgo:

1. El líder directivo. Permite a los subordinados saber lo que se espera de ellos, pro-
grama el trabajo a realizarse y da guías específicas de cómo lograr las tareas.
2. El líder que apoya. Es amistoso y muestra interés por las necesidades de sus su-
bordinados.
3. El líder participativo. Consulta con los subordinados y utiliza sus sugerencias an-
tes de tomar una decisión.
4. El líder orientado al logro. Establece metas de desafío y espera que los subordi-
nados se desempeñen a su nivel más alto.

Liderazgo transformador

El liderazgo transformador es un tipo de liderazgo que considera los intereses del gru-
po, de la organización y de la sociedad. Busca potenciar el desarrollo de sus colaboradores,
de sus capacidades, motivaciones y valores, y no sólo su desempeño.

Este tipo de liderazgo ocurre cuando el líder cambia a sus subordinados en 3 formas:
•• Hacerlos conscientes de qué tan importante es su trabajo para la organi-
zación para que se alcancen las metas.
•• Hacerlos conscientes de sus propias necesidades para su crecimiento
personal, desarrollo y logro.

504
Sánchez Marchán, Nanci. (2014).Introducción a la Ingeniería Industrial, Material Instruccional de Apoyo. Caracas: UNA

•• Motivarlos para que trabajen bien y que piensen no sólo en su beneficio


personal sino en el de toda la organización.

Liderazgo transaccional

En este modelo el liderazgo reconoce lo que se quiere conseguir con el trabajo y lo ga-
rantiza si se consiguen los resultados requeridos. Intercambia premios por esfuerzos. Res-
ponde a los intereses de los trabajadores si éstos responden con su trabajo.

Los líderes transaccionales centran su interés en las demandas y condiciones del mo-
mento y no en asuntos a largo plazo, se preocupan porque se hagan las cosas. Elevan la je-
rarquía de las necesidades de los trabajadores, les hacen trascender sus propios intereses.

PODER

Otro punto necesario en una organización, lo representa el poder. Es por ello impor-
tante saber, cómo se adquiere y se ejerce para comprender totalmente el comportamiento
organizacional.

El concepto de liderazgo y poder se encuentran muy relacionados. Los lideres usan el


poder como medio para alcanzar las metas de los grupos. Estos logran metas y el poder es
una forma de facilitar su consecusión. Se presenta una tabla en la que se puede observar la
diferencia entre los conceptos de poder y liderazgo:

Tabla 51. Diferencia entre los conceptos de poder y liderazgo

Liderazgo Poder
Usa el poder como medio para alcanzar las Es la capacidad que tiene una persona (lider)
metas de los grupos para influir en la conducta de otros, y así ac-
túen en concordancia con su deseo
Diferencias
El liderazgo requiere que haya congruencia El poder no exige compatibilidad de metas,
entre las metas del líder y sus seguidores sólo dependencia
En el liderazgo se influye directamente en los En el poder no es necesario la dirección de la
seguidores y se minimiza la importancia de los influencia
esquemas de influencia lateral ascendente.
La mayor parte de la investigación del lideraz- La investigación en el poder abarca una ex-
go se centra en el estilo. tensión mayor y se enfoca en las tácticas para
ganarse la obediencia de los demás.

505
Sánchez Marchán, Nanci. (2014).Introducción a la Ingeniería Industrial, Material Instruccional de Apoyo. Caracas: UNA

BASES DE LA ESTRUCTURA DE LA ORGANIZACIÓN


La estructura organizacional se define como:

La forma en como se dividen, agrupan y coordinan formalmente las tareas en el trabajo.

Existen seis elementos básicos que se deben abordar cuando se diseña la estructura
de una organización: especialización del trabajo, departamentalización, cadena de mandos,
tramo de control, centralización y descentralización y formalización.

1. Especialización del trabajo. Los individuos se especializan en una parte de la acti-


vidad y no en toda la tarea, es decir, en lugar de que un solo individuo haga todo el trabajo, el
trabajo se divide en varios pasos y cada uno lo completa una persona por su cuenta.

2. Departamentalización. Después que las tareas han sido divididas mediante la es-
pecialización, se coordina aquellas que sean comunes. La departamentalización se puede
realizar por funciones, por productos, por geografía, por procesos, por clientes.

3. Cadena de mandos. Se consideran dos conceptos complementarios: autoridad y


unidad de mando. La autoridad se refiere al derecho inherente de una posicion gerencial
para dar ordenes y esperar que éstas se acaten. El principio de la unidad de mando sos-
tiene el concepto de línea continua de autoridad. Afirma que una persona debe tener sólo
un superior ante el cual es responsable directo. Estos conceptos tienen hoy mucha menor
importancia por los adelantos tecnológicos de la información y la tendencia a facultar a los
empleados.

4. Tramo de control. Determina el número de subordinados que un gerente puede diri-


gir con eficacia. Cuanto más ancho o largo sea el tramo de control, más eficiente es la orga-
nización. La tendencia actual es hacia los tramos de control más anchos, lo que concuerda
con los esfuerzos recientes de las compañias por reducir costos, suprimir gastos generales,
acelerar la toma de decisiones, aumentar la flexibilidad, acercarse a los clientes y facultar a
los empleados.

5. Centralización, descentralización: una organización está centralizada cuando la


dirección toma las principales decisiones de la organización sin considerar las aportaciones
del personal de niveles inferiores. En cambio en una organización descentralizada, las medi-
das para resolver los problemas son más rápidas, más personas participan en las decisiones
que repercuten en su vida laboral.

6. Formalización: Es el grado en el que las tareas de la organización Están estanda-


rizadas. Las descripciones de los puestos son explícitas, hay muchas reglas internas y se
definen con claridad los procedimientos que abarcan los procesos de trabajo.

Se consideran en este texto tres diseños organizacionales de uso más frecuente: es-
tructura simple, burocracia y estructura matricial.

506
Sánchez Marchán, Nanci. (2014).Introducción a la Ingeniería Industrial, Material Instruccional de Apoyo. Caracas: UNA

•• La estructura simple. Es una estructura caracterizada por un grado es-


caso de departamentalización, tramos amplios de control, autoridad cen-
tralizada en una persona y poca formalización. Por ejemplo, una tienda
de ropa.
•• Burocracia. Estructura de las tareas operativas más rutinarias que se
consiguen mediante especialización, reglas, regulaciones muy formaliza-
das, tareas agrupadas en departamentos funcionales, autoridad centrali-
zada, márgenes estrechos de control y toma de decisiones que sigue la
cadena de mandos. Ejemplo, un hospital.
•• La estructura matricial. Estructura que crea líneas dobles de autoridad
y combina la departamentalización de funciones y productos. Por ejem-
plo, la industria aereoespacial. Ver unidad 2, tabla Nº 10.

En los últimos 10 ó 20 años, los directores de diversas organizaciones han planteado


nuevas opciones estructurales con el fin de que sus empresas compitan mejor. A continua-
ción se describen tres de los modelos de nuevos diseños estructurales:
•• Estructura por equipos. Uso de equipos como medio central para coor-
dinar las actividades de trabajo. Ejemplo, una consultora.
•• Organización virtual. Organización pequeña que subcontrata las princi-
pales funciones. Ejemplo, compañias telefónicas.
•• Organización sin fronteras. Organización que tiene por objetivo elimi-
nar la cadena de mandos, tener tramos ilimitados y sustituir a los depar-
tamentos por equipos facultados. Ejemplo, e-business (comercio electró-
nico).

La estructura de la organización también puede verse afectada por el ambiente, es


decir aquellas instituciones o fuerzas fuera de ella que pueden afectar su desempeño. Entre
estas fuerzas se pueden mencionar: proveedores, clientes, competidores, dependencias gu-
bernamentales reguladoras, grupos de presión públicos, etc.

Algunas organizaciones enfrentan ambientes estáticos, pocas fuerzas cambian en su


ambiente; por ejemplo no hay nuevos competidores, pocos cambios tecnológicos introduci-
dos, etc. Otras organizaciones enfrentan ambientes muy dinámicos, como por ejemplo am-
bientes donde las regulaciones gubernamentales cambian con precipitación y afectan el ne-
gocio, enfrentarse a nuevos competidores, dificultades para conseguir materias primas, etc.

El comportamiento organizacional además está influenciado por los siguientes con-


ceptos:

507
Sánchez Marchán, Nanci. (2014).Introducción a la Ingeniería Industrial, Material Instruccional de Apoyo. Caracas: UNA

DISEÑO Y TECNOLOGÍA DEL TRABAJO


La influencia significativa de la organización electrónica reescribe las reglas de la co-
municación. Las barreras tradicionales se están derrumbando y están siendo sustituidas por
redes.

En los últimos años se han generado varias opciones de trabajo, como el horario flexi-
ble, el trabajo compartido y la telecomunicación. Se han convertido en una herramienta
estratégica importante a medida que las organizaciones tratan de incrementar la flexibilidad
que necesitan sus empleados en un ambiente laboral cambiante, es decir, se utilizan las
bondades de la tecnología para aumentar en gran medida la flexibilidad, haciendo uso del
teletrabajo o realización del trabajo desde el hogar. Existiendo poca interacción fisica entre
los trabajadores, lo que disminuye la generación de conflictos, el liderazgo, los gastos per-
sonales, entre otros.

POLÍTICAS Y PRÁCTICAS DE RECURSOS HUMANOS


Las politicas y prácticas de recursos humanos, como la selección de empleados, capa-
citación, evaluación del desempeño y relaciones obreros-patronales, influyen en la eficacia
de una organización. El objetivo de una selección eficaz es hacer corresponder las caracte-
risticas de un individuo con los requisitos de los trabajos que va a realizar. Cuando la admi-
nistración no consigue la correspondencia adecuada, se reduce tanto el desempeño como
la satisfacción de los empleados.

En las organizaciones, la evaluación del desempeño cumple varios fines. Esta permite
valorar la aportación del empleado como base para tomar decisiones sobre la distribución
de remuneraciones. Si el proceso de evaluación hace hincapié en los criterios equivocados
o valora mal el desempeño real, las remuneraciones de los empleados serán excesivas o
insuficientes..

La presencia de un sindicato en una organización añade otra variable al comporta-


miento de los empleados. Se sabe que un sindicato es un elemento muy importante en las
impresiones, actitudes y conductas de los empleados. El poder de los sindicatos aflora en
los acuerdos de las negociaciones colectivas con la dirección. Ver tema recursos humanos
unidad 3.

CULTURA ORGANIZACIONAL

La cultura organizacional es uno de los pilares fundamentales para apoyar a todas


aquellas organizaciones que quieren hacerse competitivas. Es una nueva óptica que permite
a la gerencia comprender y mejorar las organizaciones, se puede definir como:

Un sistema de significado compartido entre sus miembros que distingue a una organiza-
ción de otras.

508
Sánchez Marchán, Nanci. (2014).Introducción a la Ingeniería Industrial, Material Instruccional de Apoyo. Caracas: UNA

La cultura organizacional tiene la particularidad de manifestarse a través de conductas


significativas de los miembros de una organización, las cuales facilitan el comportamiento
en ella y se identifican básicamente a través de un conjunto de prácticas gerenciales, como
elementos de la dinámica organizacional. Hay prácticas dentro de la organización que re-
flejan que la cultura es aprendida, y por lo tanto, deben crearse culturas con espíritu de un
aprendizaje continuo. Al cultivarse una cultura en la organización sustentada por sus valores,
se persigue que todos los integrantes desarrollen una identificación con los propósitos estra-
tégicos de dicha organización y se desplieguen conductas direccionadas a ser autocontro-
ladas (Schein, 1985).

Características primarias que reflejan la esencia de la cultura de una organización:


•• Innovación y toma de riesgos. Considera el grado de apoyo a los em-
pleados para ser innovadores y tomar riesgos.
•• Atención al detalle. Contempla la demostración de los empleados en
cuanto a la precisión, análisis y atención al detalle.
•• Orientación de los resultados. Grado en el que la gerencia se enfoca
en los resultados y no en los medios para lograr tales resultados.
•• Orientación hacia las personas. La gerencia le da prioridad al efecto
de los resultados en los miembros de la organización.
•• Orientación al equipo. El grado en el que las actividades están organi-
zadas es en función de equipos y no de forma individual.
•• Energía. El grado en que la gente es activa y competitiva.
•• Estabilidad. Se da prioridad al mantenimiento del statu quo en las acti-
vidades organizacionales sobre el crecimiento.

CLIMA ORGANIZACIONAL

El clima organizacional se puede definir como:

El conjunto de factores internos y externos de un ambiente laboral, que pueden ser medi-
bles según la percepción de quienes trabajan en él, utilizando como variables de medición
la estructura, responsabilidad, recompensas, desafío, relaciones personales, flexibilidad,
estándares, formas de recompensa, claridad y compromiso de equipo, etc.

La medición del clima organizacional generalmente se realiza utilizando encuestas apli-


cadas a los trabajadores de toda la organización, o a un área en particular de ésta, que se
quiera medir.

Características del clima organizacional

A continuación se mencionan algunas características del clima organizacional:

509
Sánchez Marchán, Nanci. (2014).Introducción a la Ingeniería Industrial, Material Instruccional de Apoyo. Caracas: UNA

•• Es plural. Debido a que conduce a diferentes opiniones subjetivas indi-


viduales o grupales.
•• Produce satisfacción laboral. Está fundamentada en el valor que se le
asigne al trabajo, así como en el equilibrio de las recompensas.
•• El clima organizacional conjuntamente con la estructura y los in-
tegrantes de las organizaciones. Forman un sistema organizacional
dinámico

Un buen clima organizacional  tendrá consecuencias positivas en la empresa, las


cuales van a estar definidas por cómo las personas perciben el ambiente interno y externo
de la organización.

Actividad propuesta
Relacione cómo influyen las políticas y prácticas de recursos humanos en la cul-
tura y clima organizacional.

DIAGNÓSTICO Y DISEÑO DE ORGANIZACIONES


DIAGNÓSTICO

Se puede definir al diagnóstico como:

Un proceso razonado que permite conocer la situación real de una organización en un


momento dado, para descubrir problemas y corregirlos, así como determinar oportunida-
des de mejoras para aprovecharlas.

En el diagnóstico se examinan y mejoran los sistemas y prácticas de la comunicación


interna y externa de una organización en todos sus niveles.

Elementos del diagnóstico organizacional:

Se puede dividir al Diagnóstico Organizacional en tres etapas principales:

1. Generación de información, la cual abarca a su vez tres aspectos:


•• La forma en como se recolecta la información, las herramientas y los
procesos utilizados.
•• Los métodos utilizados para recopilar la información siguen dos corrien-
tes: los métodos usados para obtener información desde el cliente (en-
trevistas, cuestionarios) y los usados para obtenerla desde el consultor
(observación).
•• La frecuencia con que se recolecta la información, la cual depende de la
estabilidad del sistema.

510
Sánchez Marchán, Nanci. (2014).Introducción a la Ingeniería Industrial, Material Instruccional de Apoyo. Caracas: UNA

2. Organización de la información, en la que es necesario considerar tres aspec-


tos clave:
•• El diseño de procedimientos para el proceso de la información.
•• El almacenamiento apropiado de los datos.
•• El ordenamiento de la información, de modo que sea fácil de consultar.

3. Análisis e interpretación de la información: consiste en separar los elementos bá-


sicos de la información y examinarlos con la intención de responder a los temas planteados
al inicio de la investigación.

DISEÑO ORGANIZACIONAL
El diseño organizacional se da cuando:

Los gerentes desarrollan o cambian la estructura de una organización para alcanzar me-
tas y objetivos, con eficacia y eficiencia que permita coordinar las actividades a través de
un modelo de estructura hacia la mejora del esfuerzo humano.

En el diseño de una estructura se debe considerar lo siguiente:

Complejidad. Entre mayor sea la división del trabajo en una empresa más niveles ver-
ticales habrá en la jerarquía y entre más dispersas estén las unidades de la organización
geográfica más complejas serán las actividades y relaciones personales.

Formalización. Depende del cumplimiento de las reglas y procedimientos para dirigir el


comportamiento de los empleados.

Centralización. Se da cuando los gerentes desarrollan o cambian en función de la


toma de decisiones.

El diseño favorece la buena marcha de las empresas. Las organizaciones que


más crecen son aquellas que tienen al diseño incorporado a su estructura como una
estrategia de acción. Debe diseñarse para determinar quién y cuándo realizará las tareas,
quién será responsable de los resultados. Todo ello permitirá eliminar la incertidumbre res-
pecto a la asignación de actividades y se podrá mejorar la red de información y la toma de
decisiones, que respondan y sirvan de apoyo al logro de los objetivos empresariales.

LA COMUNICACIÓN EN LA ORGANIZACIÓN
El proceso de comunicación es un aspecto importante, que depende de la habilidad
para hablar, escribir, leer, razonar y escuchar. Se puede definir la comunicación como:

511
Sánchez Marchán, Nanci. (2014).Introducción a la Ingeniería Industrial, Material Instruccional de Apoyo. Caracas: UNA

El proceso mediante el cual un emisor envía un mensaje a través de un canal para ser
decodificado y capturado por un receptor, a la espera de respuesta.

Si alguno de estos elementos falla, se dice que se  ha producido una interferencia y no
podrá establecerse la comunicación. En este sentido es necesario colocar de nuevo el men-
saje en el sistema para que surja la retroalimentación, la cual consiste en verificar el éxito al
transferir el mensaje, es decir determinar si el entendimiento se ha logrado.

Ningún grupo o equipo puede existir sin la comunicación. La falta de una comunicación
adecuada puede tener consecuencias adversas, además es la fuente de conflictos más fre-
cuente. La buena comunicación es esencial para la eficacia de cualquier grupo u organiza-
ción.

Estudiados los elementos del comportamiento organizacional, se presenta a continua-


ción un caso, en el que se pueden observar muchos de estos elementos:

Ejemplo Ilustrativo Nº 1
Caso de comportamiento organizacional en empresas corporativas y multinacio-
nales.
En estas empresas prevalece la adopción de nuevas técnicas administrativas que
generan cambios de actitudes y modificación de la cultura administrativa. La cul-
tura organizacional está formalizada, es comunicada a los miembros de la organi-
zación y se da una retroalimentación constante respecto a la interiorización y com-
presión del corazón ideológico que sustenta a la entidad. Capacitación continúa al
personal con el objetivo de que el empleado desarrolle múltiples habilidades y sea
capaz de solucionar problemas.
El conjunto de estas empresas cuentan con departamentos de comunicación, re-
laciones públicas y de mercadotecnia, lo cual refleja una preocupación e interés
por mantener debidamente informados tanto a sus públicos internos como exter-
nos. Sin embargo estas organizaciones se enfocan principalmente en el cuidado
de su imagen, la calidad de sus productos y servicios, y la satisfacción al cliente.
Con respecto a las condiciones y garantías de trabajo se otorgan en algunos ca-
sos de manera parcial. Aquí las características son: rotación constante de perso-
nal, ya que los contratos son por honorarios o eventualidades que impiden acce-
der a las prestaciones de ley y a tener antigüedad. Horarios variables y solicitud
de disponibilidad del empleado según exigencias de la organización con el mismo
sueldo y sin goce de horas extra.

A continuación se presenta un caso para su respectiva interpretación y opinión:

512
Sánchez Marchán, Nanci. (2014).Introducción a la Ingeniería Industrial, Material Instruccional de Apoyo. Caracas: UNA

Actividad propuesta
HILADOS MARÍA S.A., es una empresa que produce diversos hilados y tejidos.
Fue fundada hace más de 50 años y durante un tiempo, fue una empresa exitosa.
A mediados de la década de los 80 su fundador se enfermó y tuvo que retirarse,
lo que generó un deterioro en términos de intercambio que derivaron en la pér-
dida de sus mercados de exportación y comenzó a tener grandes problemas. La
crisis estalló poco tiempo después que en la empresa se decidiera realizar una
importante inversión en maquinaria, con el objetivo de mejorar su productividad.
El crédito que habían tomado resultó imposible de pagar, los hijos del fundador de
la empresa, decidieron venderla.
Una conocida empresa internacional (Tejidos Beta International) adquirió todo el
paquete accionario de HILADOS MARÍA S.A., bajo la responsabilidad de respetar
los compromisos de ésta y mantener a un 80% del personal. La adquisición de la
empresa coincidió, asimismo, con el retiro del gerente general por jubilación.
Los nuevos propietarios rápidamente se hicieron cargo de la empresa. Enviaron a
uno de sus principales ejecutivos Rafael Pérez, como gerente interino. Anunciaron
que una vez que el nuevo gerente analizara la situación organizativa de la empre-
sa y estableciera los primeros ajustes, se nombraría un gerente general local.

513
Sánchez Marchán, Nanci. (2014).Introducción a la Ingeniería Industrial, Material Instruccional de Apoyo. Caracas: UNA

Rafael Pérez entrevistó a tres de los gerentes de la empresa:


Amalia, gerente de ventas: tiene 45 años, es divorciada y tiene a su cargo a dos
hijos ya mayores. Ingresó en la empresa hace 28 años, como vendedora auxiliar.
Al comienzo, estudiaba mientras trabajaba. Llegó a obtener así un título de Licen-
ciada en Publicidad.
Luego logró obtener una licenciatura en comercio. Durante muchos años, Ama-
lia fue la jefa de ventas exterior. Al retirarse el anterior gerente de ventas, hace
ya unos 10 años, fue ascendida. Amalia cumple con su jornada de trabajo, es
cuidadosa y dedicada. La honestidad, la puntualidad y la dedicación al trabajo
fueron los elementos valorados por el Sr Rafael Pérez. Se merece el cargo, estoy
seguro de que no nos defraudará, dijo Pérez. Como gerente, resultó ser justa con
sus subordinados y procuró realizar su trabajo en forma cuidadosa, además de
capacitarse. Pero no obstante a pesar de sus esfuerzos, la caída de las ventas y
la pérdida de los mercados de exportación fueron inevitables. Durante los años
en que fue gerente, sufrió varios problemas de salud y, especialmente, dos que-
brantos nerviosos.

Alberto, el gerente de producción: tiene 32 años, es ingeniero y trabaja hace


5 años en la empresa. Es soltero y vive solo. Ingresó en la empresa al renunciar
el anterior gerente de producción. Su experiencia previa era muy poca y en parti-
cular, nunca había desempeñado un cargo gerencial. Ingresó con la tarea inicial
de cambiar las maquinarias de la empresa, ya obsoletas. Con este motivo, realizó
varios viajes al exterior para analizar distintos tipos de maquinarias y finalmente,
solicitó y obtuvo la ayuda de Amalia para realizar los trámites y solicitudes.
Posteriormente, participó activamente en las negociaciones para la venta de la
empresa.
En ocasión del último pedido logrado por Amalia para Suiza, un desperfecto dejó
sin energía a la planta durante varias horas.

Daniela es contadora: tiene 43 años y es la gerente de Administración desde


hace diez años, anteriormente había sido jefa de contaduría por diez años. Es
casada y tiene tres hijos. Tiene una gran capacidad técnica y, si la empresa logró
sobrevivir durante los momentos más duros de la crisis, fue gracias a su cuida-
dosa planificación financiera. En particular, Daniela no estuvo de acuerdo con la
compra de las nuevas maquinarias, y tras los problemas que esto ocasionó, lo
hizo notar en forma reiterada. Daniela es incansable y exige a todos sus funciona-
rios igual dedicación. En la época en que era jefa de contaduría, por ejemplo, el
cierre de balance era una fecha temida por todos.
Es muy respetada por su capacidad técnica, pero sus colaboradores se quejan de
su negativa a aceptar nuevas ideas y también de su seriedad. De la misma forma,
se revela como una persona muy conservadora, tanto en su profesión como en
su vida personal.

514
Sánchez Marchán, Nanci. (2014).Introducción a la Ingeniería Industrial, Material Instruccional de Apoyo. Caracas: UNA

Rafael Pérez piensa elegir al futuro gerente general entre estas tres personas, ac-
tuales gerentes. Los cambios que se producirán en el próximo año serán muchos:
probablemente se despida al 25% del personal, se prevé que surjan conflictos
gremiales y se realizará una reingeniería de la empresa.
Una tarde, pocos días después de la llegada del Sr. Rafael Pérez, Amalia y Danie-
la se encontraron en el cafetín de la empresa.
Hola Amalia, -dijo Daniela- al fin soplan vientos de cambio en esta empresa. Es
la primera vez en años que me puedo detener a analizar los números sin estar
enloquecida corriendo por los bancos. Y Pérez me dijo que instalarían un sistema
integral de gestión que usará toda la empresa, las cosas han mejorado. A su vez,
también tenemos más responsabilidades.
Lo que sí creo, es que nosotras debemos trabajar juntas. Una de nosotras espero
que sea la gerente general. Nos tendremos que apoyar.
Amalia dice: En el pasillo se dice otra cosa, ayer mi secretaria me dijo que había
escuchado que sería Alberto.
Ah, no, no...Alberto es un acomodado, no tiene experiencia. Seremos una de no-
sotras.
De acuerdo al caso leído, se pide:
1. Analizar las variables de nivel individual y el estilo de liderazgo de Amalia,
Daniela y Alberto.
2. Analizar el diálogo establecido entre Amalia y Daniela.
3. Explicar en qué forma perciben cada uno de los gerentes la situación.
4. Si Ud. estuviera en el lugar del Sr. Pérez ¿a cuál de los tres elegiría? Justifique
su respuesta.
5. Explica si a su juicio existen las condiciones para que surja un conflicto entre
ambas.
6. Mientras no sea nombrado el gerente, establezca cuál será la conducta de
Alberto, Daniela y Amalia.
7. Explique cuáles son las bases del poder de cada uno de los tres gerentes.
8. ¿Se observan las características primarias de la cultura organizacional en la
empresa?
9. ¿Cómo se da el proceso de comunicación en la empresa?
10. ¿Existe un buen clima organizacional?

Los elementos fundamentales del comportamiento organizacional son las personas,


la estructura, la tecnología y el ambiente. Con la adecuada interacción y aprovechamiento
óptimo de estos elementos, la organización puede lograr el éxito.

515
Sánchez Marchán, Nanci. (2014).Introducción a la Ingeniería Industrial, Material Instruccional de Apoyo. Caracas: UNA

RESUMEN DE LA UNIDAD Nº 3
En esta unidad se estudió el área de gerencia, la cual determina el manejo óptimo de los
recursos de una empresa a través de la Administración, Economía y Finanzas, Marketing In-
dustrial, además del Comportamiento Organizacional de las personas en su sitio de trabajo.

Administración

La Administración es la base de todo en una empresa, ya que si no se saben adminis-


trar los recursos (físicos y humanos) no se obtendrán los resultados deseados, Cuando se
habla de administración implica saber mantener el orden de todo, hasta de la vida, tiempo,
etc. La administración está en todos lados hasta en los hogares.

En la administración de empresas, se denomina Recursos Humanos a la función


que se ocupa de seleccionar, contratar, formar, emplear y retener a los colaboradores de la
organización

Los conceptos, principios, teorías y técnicas de la administración se agrupan en cuatros


funciones:

Planeación: implica seleccionar misiones y objetivos.

Organización: es la parte de la administración que supone el establecimiento de una


estructura intencionada de los papeles que los individuos deberán desempeñar en una em-
presa.

Dirección: su principal función es la de influir en los individuos para que contribuyan a


favor del cumplimiento de las metas organizacionales y grupales.

Control: Implica la medición del desempeño con base en metas y planes, la detección
de desviaciones respecto de las normas y la contribución a la corrección de éstas.

El término Administración debe describirse más bien como una actividad con largos
períodos de cambios, interrumpidos ocasionalmente por momentos breves de estabilidad.

Economía y finanzas

Economía es la ciencia que analiza el comportamiento humano como una relación entre
fines dados y medios escasos que tienen usos alternativos, dentro de ésta ciencia se en-
cuentra la microeconómica que se encarga de estudiar el comportamiento económico de las
pequeñas unidades de decisión y la macroeconomía la cual estudia el comportamiento de
variables económicas agregadas.

Las finanzas formaron parte durante mucho tiempo de la economía, se separaron como
un campo de estudios independiente a principios del siglo pasado, se refieren a la forma en
como se obtienen los recursos, a la forma en como se gastan o consumen, a la forma en

516
Sánchez Marchán, Nanci. (2014).Introducción a la Ingeniería Industrial, Material Instruccional de Apoyo. Caracas: UNA

como se invierten, pierden o rentabilizan. También suele definirse como el arte y la ciencia
de administrar dinero.

Mercado y Marketing

El marketing es la orientación con la que se administra el mercadeo o la comercializa-


ción de éste dentro de una organización. En una empresa, normalmente, el área comercial
abarca el área de marketing y el de ventas, para brindar satisfacción al cliente. Los concep-
tos de marketing, mercadotecnia, mercadeo y comercialización se utilizan como sinónimos.
No obstante, el más extendido y utilizado es el término marketing.

Comportamiento Organizacional

Para que exista una organización no basta con el conjunto de personas que posean un
propósito en común. Lo realmente decisivo es que dichas personas organicen o coordinen
sus actividades, con una acción conjunta hacia el logro de resultados que les interesa
conseguir.

Existe un grupo de disciplina que han contribuido al desarrollo del comportamiento orga-
nizacional entre las cuales se encuentran la sicología, sociología, antropología, entre otras.

El comportamiento organizacional se interesa particularmente en las situaciones que


atañen al empleo, por lo que se destaca el comportamiento en lo que se refiere al trabajo,
puestos, ausentismo, rotación, productividad, desempeño humano y administración.

Su distribución se fundamenta en el individuo que posee una serie de características


particulares, con las que se muestran motivos, emociones, valores, intereses, actitudes y
competencias; el grupo en el cual el aporte individual de cada uno de los individuos dan por
resultado un desempeño que es mayor que la suma de los aportes de cada uno y la estruc-
tura que define cómo se dividen, agrupan y coordinan formalmente las tareas en el trabajo.

Seguidamente se presenta la autoevaluación de la unidad 3, en la que encontrará un


conjunto de ejercicios, cuyas respuestas se presentan al final del texto. También se puede
observar una serie de ejercicios propuestos, que le permitirá al estudiante ejercitarse lo sufi-
ciente para la presentación de las pruebas.

517
Sánchez Marchán, Nanci. (2014).Introducción a la Ingeniería Industrial, Material Instruccional de Apoyo. Caracas: UNA

AUTOEVALUACIÓN DE LA UNIDAD Nº 3
Una vez estudiados los contenidos concernientes al área de gerencia, es reco-
mendable resolver los ejercicios correspondientes a la autoevaluación, sobre la
base de los ejemplos ilustrativos. Resueltos estos ejercicios, podrá comprobar sus
resultados con las respuestas aportadas por la autora al final del texto. En caso
de no realizar los ejercicios correctamente, repase de nuevo los contenidos e in-
téntelo una vez más hasta lograr su cometido. Si aún persisten las fallas en sus
respuestas, consulte a su asesor, para que le ayude a despejar sus dudas.

ÁREA DE GERENCIA
Administración

Ejercicio N° 1

a) ¿Cómo se clasifican los administradores gerentes?

b) Según su criterio cuál es el factor de motivación que puede generar cada uno de
los siguientes principios:

Principio Factor de motivación


Suprimir algunos controles manteniendo la responsabi-
lidad del trabajo.
Aumentar la responsabilidad del trabajador respecto a
su trabajo.
Delegar un área de trabajo cerrada en sí misma.
Conceder mayor autoridad y mayor libertad.
Informar al trabajador sobre los informes mensuales.
Reparto de tareas nuevas y más difíciles.
Reparto de tareas especiales, que permitan al trabaja-
dor mejorar profesionalmente.

Ejercicio Nº 2

CASO: Fairchild Industries

Charles Collis, era presidente de Fairchild Republic, división de Fairchild Industries dedi-
cada a la manufactura de aviones. La división tenía un contrato para construir el avión caza
A- 10 Thunderbolt II y Collis se enojó con un celoso analista de la Fuerza Aérea. No permitía
que la gente de la Fuerza Aérea le dijera cómo administrar su negocio. El analista se ofen-
dió y se quejó con sus superiores. La relación negativa con la Fuerza Aérea obligó a que la
empresa lo despidiera.

Doc Grossman fue nombrado para sustituir a Collis y su objetivo prioritario fue restable-
cer las buenas relaciones con la Fuerza Aérea. Grossman también ayudó a resolver algunos

518
Sánchez Marchán, Nanci. (2014).Introducción a la Ingeniería Industrial, Material Instruccional de Apoyo. Caracas: UNA

problemas técnicos, para ello volaba al sitio de prueba. El A-10 perdía potencia en virajes
repentinos y Grossman resolvió el problema colocando una pequeña placa de metal en la
sección derecha de cada ala.

Pero mientras Grossman se concentraba en resolver los problemas del A-10, el desa-
rrollo de otro avión, el Peacemaker, quedó fuera de control. Grossman admite que estaba
tan dedicado al diseño del a-10 que olvidó el mercado para otros tipos de aviones. La Fuerza
Aérea rechazó al Peacemaker, General Dynamics ganó el contrato con el F-16, y este fue el
contrato más importante de los contratos de defensa de la historia.

Analice el caso mostrado y responda las siguientes preguntas:


a) Tomando en cuenta las actividades gerenciales requeridas en este caso ¿Cómo
fue el desempeño de Collis y Grossman?
b) ¿Qué debe tomar en consideración un buen gerente?

Ejercicio Nº 3
¿Cuáles son los elementos importantes que influyen en las decisiones para conceder
ascensos, hacer transferencias y despedir al personal que labora en una organización?

Ejercicio Nº 4
¿Por qué gerenciar el cambio en una organización y cuáles son las palabras clave a
considerar para impulsar este cambio?

Economía y finanzas
Ejercicio N° 5
Responda las siguientes preguntas:
a) ¿Cuál es el punto de contacto entre el estado de situación financiera con el esta-
do de resultados?
b) ¿Qué estados financieros conoce?
c) ¿Por qué la depreciación es una cuenta que se presenta en el estado de
resultados?

Ejercicio Nº 6
Entre los productos de Enlatados Carolina se encuentra el atún, el cual es vendido en el
mercado nacional e internacional. La empresa desea aumentar las ganancias en la venta del
producto, para lo cual presenta dos alternativas con base en los costos. Se le pide a usted
como analista de producción recomendar la alternativa más viable a la situación planteada.
Cada alternativa tiene su fortaleza y debilidad.

Se ha preparado la siguiente información como ayuda para la comparación y selección


de la alternativa:

519
Sánchez Marchán, Nanci. (2014).Introducción a la Ingeniería Industrial, Material Instruccional de Apoyo. Caracas: UNA

Factores Alternativa 1 Alternativa 2


Costo anual de operación en Bs 8.500.000 9.250.000
Unidades producidas al año 25.000 28.000
Precio de venta por unidad en Bs 1.800 1.600
Costo de almacenamiento Bs/año 12.000 12.500

Ejercicio Nº 7
El proceso productivo de la empresa ALFA S.A requiere de 900 kg/día de materia prima.
Usted tiene la responsabilidad de seleccionar un proveedor, para la adquisición de la misma,
para ello se le presentan las siguientes alternativas de compra: Proveedor (1) puede suminis-
trar 700 kg/día a 4.000 Bs./kg, el proveedor (2) puede suministrar 1.000 kg/día a 5.000 Bs./Kg.
De acuerdo a las alternativas planteadas: a)¿Qué propuesta recomendaría a la geren-
cia? b)¿Cuál es el costo mínimo para adquirir los 900 kg/día de materia prima?

Ejercicio Nº 8
¿Qué tiempo (años) serán necesarios para que un capital de Bs. 10.500.000, colocado
al 12% obtenga una ganancia de Bs. 2.700.000?

Ejercicio Nº 9
¿Qué capital será necesario para que colocado al 15%, obtenga un interés de Bs 48.000
durante un año?

Ejercicio Nº 10
A continuación se expresan, en orden aleatorio, los reglones que deben incluirse en el
balance general de la empresa CARTÓN S.A., al 31 de diciembre de 2012:

Equipos 429.200 Bs Edificación 450.000 Bs


Terreno 425.000 Bs Efectivo 321.400 Bs
Cuentas por Pagar 354.800 Bs Muebles 458.700 Bs
Cuentas por Cobrar 510.600 Bs Vehículos 750.000 Bs
Salarios por pagar 433.500 Bs Documentos por pagar 620.00 Bs
Interés por pagar 120.000 Bs

Se pide:

a) Preparar un balance general a 31 de diciembre de 2012. Incluya un encabeza-


miento apropiado y organice su balance general en el formato presentado en el

520
Sánchez Marchán, Nanci. (2014).Introducción a la Ingeniería Industrial, Material Instruccional de Apoyo. Caracas: UNA

texto. Para ello necesitará calcular la cantidad que debe mostrarse como el patri-
monio de los propietarios.

b) Suponga que no se ha vencido ningún pago sobre los documentos por pagar
hasta 2013. ¿Indica este balance general que la compañía está en una posición
financiera sólida al 31 de diciembre de 20012? Explique brevemente.

MARKETING INDUSTRIAL
Ejercicio Nº 11
¿Cuándo se considera que un plan de marketing es eficiente?

Ejercicio Nº 12
Describa los aspectos más resaltantes de un grupo meta.

Ejercicio Nº13
Mencione las actividades que se realizan en el campo de la promoción de ventas.

COMPORTAMIENTO ORGANIZACIONAL
Ejercicio Nº 14

A continuación se presentan varios grupos de ciencias. Seleccione el grupo que ha


aportado al comportamiento organizacional.

1. Sicología, sociología, antropología.


2. Administración, finanzas, contabilidad.
3. Medicina, biología, economía.

Ejercicio Nº 15
Mencione algunas fuentes de actitudes.

Ejercicio Nº 16
Mencione algunos factores que permiten medir la satisfacción laboral.

Ejercicio Nº 17
¿Cómo expresan los empleados su insatisfacción?

521
Sánchez Marchán, Nanci. (2014).Introducción a la Ingeniería Industrial, Material Instruccional de Apoyo. Caracas: UNA

EJERCICIOS PROPUESTOS
Se presenta este grupo de ejercicios con la intención de proporcionar al estu-
diante planteamientos de situaciones que le permitan ejercitarse, antes de la pre-
sentación de las pruebas. Estos ejercicios no tienen respuestas en este texto,
para verificar los resultados de los mismos, puede apoyarse en la lectura de otros
textos o buscar información en la web.

ÁREA DE GERENCIA

Administración

Ejercicio N° 1
Interprete la siguiente aseveración: las demandas de estandarización, calidad flexibili-
dad e innovación, crean presiones a la gerencia para realinear sus sistemas y organización
acorde con un mundo cada vez más interconectado por redes cibernéticas y sociales.

Ejercicio N° 2
Diga dónde cree que terminará el administrador o gerente, si continúan pensando y
haciendo las cosas de la misma forma.

Ejercicio Nº 3
¿Cómo se debe realizar la evaluación del desempeño y cuáles son sus principales ob-
jetivos?

Ejercicio Nº 4
En la actualidad, numerosas  organizaciones empresariales consideran a sus trabajado-
res como la base de su éxito, por lo que en muchas de ellas se han creado los departamentos
de recursos humanos. Estos departamentos asumen la gestión del personal encargándose
de un conjunto de actividades. Describa cada una de esas actividades.

Economía y Finanzas

Ejercicio Nº 5

En la empresa donde usted trabaja, el gerente le presenta ciertas opciones con el fin
de que recomiende la mejor, en función a costos: La maquinaria adecuada disponible en el
mercado para la línea de producción de la empresa es del tipo A, B y C y tiene las siguientes
características:

522
Sánchez Marchán, Nanci. (2014).Introducción a la Ingeniería Industrial, Material Instruccional de Apoyo. Caracas: UNA

Máquina Inversión inicial consumo de energía consumo de lubricante


( Tipo ) ( Bs. ) ( Bs./Año ) ( Bs./Año)
A 50.000.000 10.500.000 1.000.000
B 35.000.000 14.000.000 2.500.000
C 30.000.000 19.000.000 2.000.000

Nota: Asuma 10 años de vida útil.

Analice la situación y en función de las opciones consideradas, ¿Cuál sería su decisión


de compra y por qué?

Ejercicio Nº 6
¿A qué tanto por ciento se deberá colocar Bs. 5.000.000, para que al cabo de un año
obtenga una ganancia de Bs. 450.000?

Ejercicio Nº 7
Si usted deposita 500 Bs. en una cuenta al final de cada año durante los próximos cua-
tro años. ¿Cuánto podría retirar al final de los cuatro años como monto total, si el interés es
del 6%?
Se proporciona un formulario y parte de la tabla de interés compuesto, como medio a
utilizar en la solución del problema.

VP= Valor presente VF= P(FP,i,n)


VF= Valor futuro VP= F(PF,i,n)
A= anualidades A= P(AP,i,n)
i= tasa de interés F= A(FA,i,n)
n= número de periodos de interés

Tabla de interés compuesto

n FP PF AF AP FA PA PG
1 1.060 0.943 1.000 1.060 1.000 0.943 -
2 1.124 0.890 0.485 0.545 2.060 1.833 2.57
3 1.191 0.840 0.314 0.374 3.184 2.673 4.95
4 1.262 0.792 0.229 0.289 4.375 3.465 7.93

523
Sánchez Marchán, Nanci. (2014).Introducción a la Ingeniería Industrial, Material Instruccional de Apoyo. Caracas: UNA

Marketing Industrial

Ejercicio Nº 8

Escenario: PROTECCIÓN empresa de seguros.

¿Cuáles son los factores que debe considerar PROTECCIÓN a la hora de diseñar y
seleccionar sus canales de distribución?

Ejercicio Nº 9

Situación: La estrategia de comunicación seguida por Venezuela Yuc, empresa dedi-


cada a la producción y comercialización de páspalos, en el ámbito nacional e internacional,
está llevada a cabo con el objetivo de conseguir la fidelización del cliente final, mediante
una combinación de diseño y política de promoción como factor empresarial diferenciador.
¿Cuáles deberían ser los medios de difusión empleados por VENEZUELA Yuc en su política
de comunicación?

Comportamiento Organizacional

Ejercicio Nº 10

Caso GEICO Corporation.

Jack Byrne, presidente del consejo de Geico, resume su estilo gerencial como “de todo
un poco”. Ha logrado establecer los fundamentos de una administración por consenso com-
binada con reglas operativas inflexibles. Geico, asegura a aquellos conductores con menos
accidentes que la mayoría y puede ofrecer precios bajos. Antes de que Byrne se hiciera car-
go, la compañía crecía sin control, pero entonces el énfasis cambió hacia la reducción de los
costos internos como fuente de aumento de los ingresos. Byrne cree que en la actualidad se
requiere un crecimiento moderado, pero desea avanzar solamente si existe un consenso. No
toma las decisiones más importantes. Antes de que cualquier aspecto se analice en grupo,
Byrne va de oficina en oficina, explicando y escuchando.

La parte más importante de la participación se lleva a cabo durante la reunión anual


de planeación. Los gerentes de los veintiocho centros principales se reúnen para celebrar
sesiones de debates en las que se analizan el presupuesto propuesto y las metas para el
año siguiente. Los gerentes debaten sus metas hasta que los pocos realistas desaparecen
y los gerentes se ponen de acuerdo. Las metas se establecen mediante la participación de
todos, por lo que los subordinados de Byrne saben exactamente qué deben realizar el año
siguiente.

Después que se han establecido las metas mediante la participación, Byrne es inflexi-
ble acerca de la implantación y las recompensas. Los gerentes que no logran sus metas no
reciben ni bonos simbólicos. Los gerentes tienen un fuerte incentivo para lograr un buen

524
Sánchez Marchán, Nanci. (2014).Introducción a la Ingeniería Industrial, Material Instruccional de Apoyo. Caracas: UNA

desempeño. Byrne insiste en que el desempeño es lo único que cuenta. Se preocupa más
por el fondo que por la forma. Mientras un empleado logre buenos resultados, Byrne dice “no
me importa si limpia sus zapatos con un ladrillo”.

Analice el caso planteado y responda la siguiente pregunta:

¿Qué estilo de liderazgo utiliza Byrne?

Ejercicio Nº 11
Indique los aportes de Abraham H. Maslow a la administración.

Ejercicio Nº 12
Según su criterio cuál es el factor de motivación que puede generar cada uno de los
siguientes principios:

Principio Factor de Motivación


Suprimir algunos controles manteniendo la responsa-
bilidad del trabajo.
Aumentar la responsabilidad al trabajador respecto a
su trabajo.
Delegar un área de trabajo cerrada en sí misma.
Conceder mayor autoridad y mayor libertad.
Informar al trabajador sobre los informes mensuales.
Reparto de tareas nuevas y más difíciles.
Reparto de tareas especiales, que permitan al trabaja-
dor mejorar profesionalmente.

525
Sánchez Marchán, Nanci. (2014).Introducción a la Ingeniería Industrial, Material Instruccional de Apoyo. Caracas: UNA

UNIDAD 4

DISCIPLINAS AFINES A LA INGENIERÍA INDUSTRIAL

Objetivo:
Reconocer los aportes de áreas afines a la ingeniería industrial para potenciar la
productividad de las empresas.

ESTADÍSTICA

INGENIERÍA DE SISTEMAS

CIENCIAS DE LA DECISIÓN

Se debe recordar que la Ingeniería Industrial se ocupa del diseño, mejoramiento e insta-
lación de sistemas integrados por personas, materiales y energía en la producción de bienes
y servicio, implementando procesos que mejoran la productividad y la calidad, así como, la
seguridad e higiene en el trabajo, sobre esta base se relaciona con algunas disciplinas, entre
las cuales se pueden destacar las siguientes:
•• Estadística.
•• Ingeniería de Sistemas.
•• Ciencias de la decisión.

527
Sánchez Marchán, Nanci. (2014).Introducción a la Ingeniería Industrial, Material Instruccional de Apoyo. Caracas: UNA

ESTADÍSTICA

•• Historia de la estadística
•• División de la estadística
•• Etapas del método Estadístico

INTRODUCCIÓN A LA ESTADÍSTICA
Desde los comienzos de la civilización han existido formas sencillas de estadística, se
utilizaban representaciones gráficas y otros símbolos en pieles, rocas, palos de madera y
paredes de cuevas, para contar el número de personas, animales o cosas. Hacia el año
3000 A.C., los babilónicos usaban pequeñas tablillas de arcilla para recopilar datos sobre la
producción agrícola y de las especies vendidas o cambiadas mediante trueque. Los egipcios
analizaban los datos de la población y la renta del país mucho antes de construir las pirámi-
des en el siglo XI A.C. Los libros bíblicos de números y crónicas incluyen, en algunas partes,
trabajos de estadística. En China existían registros numéricos similares con anterioridad al
año 2000 A. C. Los griegos realizaban censos cuya información se utilizaba hacia el 594
A.C. para cobrar impuestos.

La palabra estadística procede del latín statisticum collegium (consejo de estado) y de


su derivado italiano statista (hombre de estado o político). El término alemán Statistik, que
fue primeramente introducido por Gottfried Achenwall (1749), la designaba originalmente
como el análisis de datos del Estado, es decir, la ciencia del Estado.

La estadística se define como:

La ciencia matemática que se refiere a la recopilación, estudio e interpretación de los


datos obtenidos en una situación, para obtener conclusiones válidas. Es aplicable a una
amplia variedad de disciplinas, entre ellas la Ingeniería Industrial, es usada también en la
toma de decisiones por instituciones públicas y privadas.

DIVISIÓN DE LA ESTADÍSTICA
La estadística se divide en dos ramas:
•• La Estadística Descriptiva. Se dedica a los métodos de recolección, descrip-
ción, visualización y resumen de datos originados a partir de los fenómenos en
estudio. Los datos pueden ser resumidos numérica o gráficamente y se calcula

529
Sánchez Marchán, Nanci. (2014).Introducción a la Ingeniería Industrial, Material Instruccional de Apoyo. Caracas: UNA

una población o muestra. Para analizar los datos se usan varios tipos de métodos
estadísticos tales como media, frecuencias y otros tipos de análisis más comple-
jos.

Ejemplos Ilustrativos
1. Según una encuestadora la producción anual de azúcar en el estado Su-
cre, corresponde al 20% de la producción total anual en la región oriental
del país.
2. En los últimos 30 días se han computado un total de veinte accidentes de
tránsito en la autopista regional del centro.
3. Una encuesta informa una ventaja de 18% en el consumo de pasta frente
al consumo de arroz, en la región central del país.
Como puede observarse en cada uno de estos ejemplos, los datos estadísticos
extraídos, corresponden a la muestra de una población y denotan la aplicación
de un instrumento que permite describir, resumir o reducir las características de
un conjunto de datos, es decir, se hace uso de la estadística descriptiva.

•• La Inferencia Estadística. Se ocupa de la generación de modelos, inferencias y


predicciones asociadas a los fenómenos en estudio, teniendo en cuenta la alea-
toriedad e incertidumbre en las observaciones. Generalmente el análisis esta-
dístico inferencial se lleva a cabo para mostrar relaciones de causa y efecto, así
como para probar hipótesis y teorías científicas, estimación, pronósticos de futu-
ras observaciones, correlación o modelamiento entre variables.

Para determinar la confiabilidad de la inferencia de los datos estadísticos de una mues-


tra, se hace necesario comprobar la ésta y así poder asegurar que lo que se observa en ella
se observará también en la población. En este sentido, se requiere utilizar técnicas, cóm-
putos y análisis estadísticos más avanzados con los datos de la muestra, para confirmar la
veracidad de las inferencias que se hagan sobre la respectiva población en estudio.

Ejemplo Ilustrativo
•• Tiempo que tardan las cajeras de un supermercado en cobrar a los
clientes.
•• Número de componentes defectuosos encontrados en una fábrica de
equipos electrónicos.
En estos ejemplos se lleva a cabo una deducción lógica basada en los datos
estadísticos de una muestra, pero la inferencia o deducción que se utiliza para
generalizar la observación requiere de unos cómputos y análisis estadísticos que
conforman los números obtenidos de la muestra. La deducción o inferencia debe
ser comprobada para aceptarse como confiable y válida, por lo tanto, esto re-
quiere un procedimiento estadístico mucho más complejo; el cuál compete a la
estadística inferencial.

530
Sánchez Marchán, Nanci. (2014).Introducción a la Ingeniería Industrial, Material Instruccional de Apoyo. Caracas: UNA

Ambas ramas (descriptiva e inferencial) comprenden la estadística aplicada. Existe tam-


bién una disciplina llamada estadística matemática, la cual se refiere a las bases teóricas de
la estadística.

La estadística aplicada se apoya en la utilización de paquetes estadísticos que ayu-


dan a resolver problemas de esta índole, acortando enormemente los tiempos de resolución.

La estadística tiene una gran importancia científica, debido al amplio campo de apli-
cación que posee. Por ello, todo proyecto de ingeniería Industrial, de Higiene y Seguridad
Industrial o de cualquier otra área en estudio, requiere del método estadístico, para su desa-
rrollo. El método estadístico parte de la observación de un fenómeno, que puede no siempre
mantener las mismas condiciones predeterminadas a voluntad del investigador; por lo cual
se deja que actúe con cierta flexibilidad, pero al mismo tiempo se registran las diferentes
observaciones y se analizan sus variaciones.

ETAPAS DEL MÉTODO ESTADÍSTICO

Para el planeamiento y desarrollo de un proyecto se siguen las siguientes etapas:

1. Planteamiento del problema. Al abordar un proyecto se debe tener bien definido


qué se va a profundizar en el estudio. Es decir, se debe establecer una delimitación clara
sobre el o los fenómenos que se pretenden estudiar, para lo cual hace falta, entre otras
cosas, la revisión bibliográfica del tema, para ver su accesibilidad y consultar los resultados
obtenidos por investigaciones similares, hacer una ubicación histórica y teórica del problema.

2. Enunciación de los objetivos. Luego de tener claro lo que se pretende investigar y


el alcance de dicha investigación, se plantean los objetivos de forma tal que no haya lugar a
confusiones o ambigüedades. Se debe además, establecer la diferenciación entre los tipos
de objetivos: de corto, mediano y largo plazo, así como, entre los objetivos generales y los
específicos.

3. Formulación de las hipótesis. Una hipótesis es ante todo una explicación provisio-
nal de los hechos objeto de estudio y su formulación depende del conocimiento que el inves-
tigador posea sobre la población analizada. Una hipótesis estadística debe poderse probar
para su aceptación o rechazo. Se formula con la ayuda de un parámetro (media, varianza,
entre otros) con el propósito de rechazarla o aceptarla; se le llama Hipótesis de Nulidad y se
representa por Ho; cuando resulta ser negada a su hipótesis contraria se le llama hipótesis
alternativa (H1).

4. Definición de la unidad de observación y de la unidad de medida. La unidad


de observación puede estar constituida por uno o varios individuos u objetos y denominarse
respectivamente simple o compleja. Si se trata de medidas de longitud, volumen, peso, etc.,
debe establecerse bajo qué unidad se tomarán las observaciones (metros, pulgadas, libras,
kilogramos, etc.). Asociado a la unidad de medida, deben establecerse los criterios sobre las
condiciones en las cuales se ha de efectuar la toma de la información.

531
Sánchez Marchán, Nanci. (2014).Introducción a la Ingeniería Industrial, Material Instruccional de Apoyo. Caracas: UNA

5. Determinación de la población y de la muestra. Estadísticamente, la población se


define como un conjunto de individuos o de objetos que poseen una o varias características
comunes. Ejemplo de población puede ser: trabajadores entre 18 y 60 años de Venezue-
la, establecimientos comerciales de una comunidad, unidades de vivienda de una ciudad,
accidentes laborales de la empresa pública o privada, entre otros. Desde el punto de vista
estadístico la población puede ser considerada finita o infinita. Por ejemplo una población
infinita puede ser el conjunto de los números positivos, un ejemplo de población finita es el
número de operarios de una empresa o el número de operarios lesionados en un mes en
esa empresa.

Muestra. Es un subconjunto representativo de la población, a la cual se le efectúa una


medición con el fin de estudiar las propiedades del conjunto al que pertenece.

El muestreo es indispensable para el investigador, en la práctica resulta engorroso estu-


diar todos y cada uno de los elementos que conforman la población, además no es aconse-
jable, ya sea por la poca disponibilidad de recursos, por la homogeneidad de sus elementos
o por ser demasiado grande; por eso se recurre al análisis de los elementos de una muestra
con el fin de hacer inferencias respecto al total de la población.

Población

Muestra

Figura 80. Representación gráfica de la población y muestra.

532
Sánchez Marchán, Nanci. (2014).Introducción a la Ingeniería Industrial, Material Instruccional de Apoyo. Caracas: UNA

Fundamentalmente el muestreo es de dos tipos:


•• Probabilístico o aleatorio: permite calcular el posible error de la muestra, el
más utilizado es el llamado aleatorio o al azar, en el cual la muestra se obtiene
mediante sorteo de los individuos que la forman. Se pueden mencionar también
las muestras obtenidas por azar simple, azar sistemático, conglomerados y es-
tratificada.
•• No probabilístico: es aquel en el cual el cálculo del error no es posible. Pueden
distinguirse tres clases: accidental, es decir, cuando no existe ningún plan precon-
cebido, resultando las unidades escogidas producto de circunstancias casuales;
por cuotas, en el que sólo se especifica el tamaño de la muestra y las característi-
cas que tienen que tener los individuos que la componen; y por juicio, en el que el
investigador selecciona los individuos de la muestra según su representatividad.

En el muestreo es importante considerar los siguientes aspectos:

Tamaño muestral. Es el número de sujetos que forman la muestra extraída de una po-


blación, los cuales deben ser necesarios para que los datos obtenidos sean representativos
de la población en estudio.

Atributo. Es la cualidad o característica propia de una persona o una cosa, especial-


mente algo que es parte esencial de su naturaleza. Son atributos de los seres humanos por
ejemplo, la inteligencia y el lenguaje, entre otros.

Variable. Es una característica que al ser medida en diferentes individuos es capaz


de arrojar diferentes valores. Los tipos de variables se pueden clasificar de acuerdo con su
medición y según su influencia.

Según su medición:
•• Variable cualitativa: es una variable que no se puede medir sino observar. Ex-
presa distintas modalidades, cada modalidad que se presenta se denomina atri-
buto o categoría y la medición consiste en una clasificación de dichos atributos.
Se pueden distinguir dos tipos:
- Nominal: en este tipo de variable los valores no pueden ser sometidos a un
criterio de orden, como por ejemplo los colores, marcas de productos, sexo,
mes de nacimiento, estado civil (casado, soltero, divorciado) entre otros.
- Ordinal: puede tomar distintos valores establecidos, siguiendo una escala fi-
jada, como por ejemplo: accidentes laborales (leve, moderado, grave).
•• Variable cuantitativa: esta variable se asocia con aspectos que pueden ser medi-
dos y los cuales se pueden expresar con un número, como por ejemplo el tiempo,
el precio, etc. Se puede dividir en:

533
Sánchez Marchán, Nanci. (2014).Introducción a la Ingeniería Industrial, Material Instruccional de Apoyo. Caracas: UNA

- Variable discreta: es aquella que toman únicamente valores puntuales. Por


ejemplo, el número de hijos de una familia, el número de trabajadores de una
empresa, son valores enteros: 0, 1, 2, 3,...
- Variable continua: se corresponde con aquella variable que puede adquirir
cualquier valor dentro de un intervalo especificado de valores. Ejemplos: la
altura de los ciudadanos de un determinado colectivo, la medición de tempe-
raturas, etcétera.
•• Variable aleatoria: es una función que asigna un numero real a cada resultado
en el espacio muestral de un experimento aleatorio. Las variables aleatorias se
denotan con una letra mayúscula, tal como X, o con una letra minúscula, como
x. El conjunto de los posibles valores de la variable aleatoria X recibe el nombre
de rango de X.

Según su influencia las variables se pueden dividir en.

Variable dependiente: es el objeto de estudio, sobre el cual se centra la investigación


en general y se puede modificar su condición con el cambio de otras variables.

Variable Independiente: es aquella que no depende de otros factores para estar pre-
sente en la realidad en estudio. Tiene la capacidad para influir, incidir o afectar a otras varia-
bles.

Se puede decir entonces, que la variable independiente en un estudio influye sobre la


variable dependiente. Ejemplo Y= X+2: Y es la variable dependiente, X es la independiente.
Es decir el valor de Y va a depender del valores que tome X.

6. La recolección. Es una de las etapas más importantes de un proyecto, donde se es-


tablecen las fuentes de información, así como la cantidad y complejidad de las interrogantes,
de acuerdo con los objetivos de la investigación. Se debe descubrir dónde está la informa-
ción y cómo y a qué costo (recursos financieros, humanos, entre otros) se puede conseguir.
También se determina el medio a través del cual se debe aplicar el instrumento (teléfono, por
correo, agentes directos que recojan la información) y establecer un número óptimo de datos.

7. Consistencia de los datos. Pueden producirse errores cuando se toman las me-
diciones y registran los datos originales, cuando se transcribe de la fuente original a una
planilla, o cuando se registran para armar la base de datos. Por ello es necesario realizar
una depuración de los datos. El avance tecnológico hace que estas tareas, manualmente
tediosas, puedan ser realizadas en corto tiempo.

8. Tabulación y presentación. Una información estadística adquiere más claridad


cuando se presenta en la forma adecuada. Las tablas y gráficos facilitan el análisis, la elec-
ción de estas herramientas para mostrar los resultados, debe hacerse no sólo en función de
las variables, sino del lector a quien va dirigido el informe. Las herramientas más empleadas
en la organización de los datos son:

534
Sánchez Marchán, Nanci. (2014).Introducción a la Ingeniería Industrial, Material Instruccional de Apoyo. Caracas: UNA

•• Una tabla de frecuencias. Indica el número de unidades de análisis que caen en


cada una de las clases de la variable.
•• Gráfico de barra. Es útil para representar datos categóricos nominales u ordi-
nales. A cada categoría o clase de la variable se le asocia una barra cuya altura
representa la frecuencia o la frecuencia relativa de esa clase, se debe indicar el
número total de datos ya que el gráfico sólo muestra porcentajes o frecuencias
relativas y la fuente de la que se obtuvieron los mismos.
•• Gráfico de torta. Se representa la frecuencia relativa de cada categoría como
una porción de un círculo, en la que el ángulo se corresponde con la frecuencia
relativa correspondiente. Esta representación gráfica es muy simple y permite
comparar la distribución de una variable categórica en 2 o más grupos.

9. El análisis. La técnica estadística ofrece métodos y procedimientos objetivos que


convierten las especulaciones en afirmaciones, cuya confiabilidad puede ser evaluada, ade-
más de ofrecer una premisa medible en la toma de una decisión. Esta es la fase de la deter-
minación de los parámetros y estadísticos muestrales; para las estimaciones e inferencias
respecto a la población, el ajuste de modelos y las pruebas de las hipótesis planteadas, con
el fin de establecer y redactar las conclusiones definitivas. Los parámetros y estadísticos
más usados son:

Medidas de posición o localización


•• La media. Es la medida de posición usada más frecuentemente. Para calcular la
media aritmética o promedio de un conjunto de observaciones se suman todos
los valores y se divide por el número total de observaciones.
•• La mediana. Es el dato que ocupa la posición central en la muestra ordenada de
menor a mayor.
•• La moda. Es el dato que ocurre con mayor frecuencia en un conjunto de ellos, es
útil para identificar la categoría con mayor cantidad de datos.
•• Los percentiles. Son otro modo de resumir una distribución muestral o poblacio-
nal. El percentil p% de un conjunto de datos es la observación que deja a lo sumo
p% de las observaciones debajo de él y a lo sumo (1 − p) % encima de él.

Medidas de dispersión o variabilidad


•• El rango de observaciones X1, X2, ..., Xn. Es la diferencia entre la observación
más grande y la más pequeña, Rango = máx(Xi) – mín(Xi)
•• La desviación estándar. Mide cuán lejos se encuentran los datos de la media
muestral. La desviación estándar es útil para comparar la variabilidad de dos con-
juntos de datos en los que la variable es medida en las mismas unidades.
•• La varianza. Se presenta como promedio de las distancias a la media al cuadra-
do. no tiene las mismas unidades que los datos. Para salvar este inconveniente,

535
Sánchez Marchán, Nanci. (2014).Introducción a la Ingeniería Industrial, Material Instruccional de Apoyo. Caracas: UNA

se define la desviación estándar muestral como la raíz cuadrada positiva de la


varianza.

10. Publicación. Toda conclusión es digna de ser comunicada a un auditorio, también


hay otros estudiosos del mismo problema a quienes se les puede aportar información, cono-
cimientos y otros puntos de vista acerca de él.

Actividad propuesta
Seleccion de los temas que se presentan seguidamente, aquellos relacionados
con la disciplina que estudias. Aplique los pasos para la planeación y desarrollo
de un proyecto e investigue el papel que ha jugado el avance del conocimiento
científico, en el tema seleccionado:
•• Tecnología e innovación
•• Desarrollo económico
•• Combustión
•• Liderazgo
•• Agentes contaminantes

IMPORTANCIA DE LA ESTADÍSTICA PARA LA INGENIERÍA INDUSTRIAL


Sin estadísticas una empresa carece de capacidad para reconocer qué actividades o
productos le generan utilidades, y cuáles sólo pérdidas. La estadística acompañada de las
poderosas herramientas informáticas permiten a los gerentes, asesores, así como al per-
sonal, contar con la suficiente información para mejorar a partir de ella los procesos de la
empresa, tomar mejores decisiones comerciales, mejorar la seguridad y hacer un uso mucho
más productivo, además de provechoso de los recursos.

La estadística es fundamental para la Ingeniería Industrial en una empresa, ya que con-


tribuye tanto para la administración financiera, como para la administración de operaciones,
las ventas, el marketing, las cobranzas, la logística, la gestión de personal, entre otras áreas
y actividades. Una empresa debe procurar ser cada día más productiva, eliminando siste-
máticamente los desperdicios. Hacer posible esto exige de información, la cual se puede
obtener gracias a la estadística.

A continuación se presenta una tabla donde se puede visualizar la aplicación directa de


la estadística en diferentes áreas de Ingeniería Industrial.

536
Sánchez Marchán, Nanci. (2014).Introducción a la Ingeniería Industrial, Material Instruccional de Apoyo. Caracas: UNA

Tabla 52: Algunas aplicaciones de la estadística en diferentes áreas de la


Ingeniería Industrial
Área de la Ingeniería
Tema Algunas aplicaciones Estadísticas
Industrial
Producción Planificación, pronósticos.
Calidad Control estadístico.
Seguridad e Higiene Programas de Higiene y Seguridad.
Industrial
Mantenimiento Confiabilidad.
Operativa Industrial
Investigación de Teoría de juegos.
Operaciones Simulación.
Teoría de cola.
Procesos markovianos.

Administración Programas de política salarial,


Planificación de plantillas.
Gerencia Economía y Finanzas Valor del dinero en el tiempo
Finanzas administrativas.

En la Ingeniería Industrial como en la Higiene y Seguridad Industrial y el resto de las


carreras, se realizan proyectos que dependen del análisis estadístico para fundamentar sus
soluciones y considerar los resultados confiables. Desde la descripción y planteamiento del
problema, hasta la discusión de los resultados, se usan datos estadísticos para sustentar
hechos y puntos de vista que se establecen como fuentes de referencia en la toma de deci-
siones.

537
Sánchez Marchán, Nanci. (2014).Introducción a la Ingeniería Industrial, Material Instruccional de Apoyo. Caracas: UNA

INGENIERÍA DE SISTEMAS

•• Historia de la Ingeniería de Sistemas.


•• Importancia de la computadora en la ingeniería industrial. 
•• Definición de la Ingeniería de Sistemas.
•• Otras Definiciones.
•• La Ingeniería de Sistemas en la actualidad.
•• Influencia de la Ingeniería de Sistemas en la Ingeniería Industrial.
•• Softwares de aplicación más usados en la Ingeniería Industrial.

INTRODUCCIÓN A LA INGENIERÍA DE SISTEMAS


La primera referencia que describe ampliamente el procedimiento de la Ingeniería de
Sistemas fue publicada en 1950 por Melvin J. Kelly, entonces director de los laboratorios
de la Bell Telephone. Esta compañía jugó un papel importante en el surgimiento de esta
disciplina, por tres razones: la apremiante complejidad que planteaba el desarrollo de redes
telefónicas, su tradición de investigación y su salud financiera. Aunque su aplicación en los
comienzos fue en grandes proyectos del sector defensa y aeroespacial, actualmente la diver-
sidad de productos de alta complejidad: teléfonos móviles, sistemas del automóvil y robots
industriales, ha puesto un mayor campo de aplicación de la Ingeniería de Sistemas, cuyos
aspectos comunes con la dirección de proyectos no se debe olvidar.

A medida que los centros industriales se diversificaban y se hacían más grandes, con
un gran número de componentes y relaciones entre sí, el problema de manejar tal diversidad
fue creciendo y surge la Ingeniería de Sistemas, como la disciplina a la que se le encarga la
tarea de ponerse al frente del problema del diseño y manejo de la complejidad. Dada esta
complejidad para manejar volúmenes de datos, se crea la computadora.

La computadora se operaba por medio de tableros enchufables en la primera genera-


ción y luego por medio del trabajo en lote en la segunda generación. El operador realizaba
siempre una secuencia de pasos repetitivos, una de las características contempladas en la
definición de lo que es un programa, a través del tiempo y por su enorme complejidad se le
llamó sistema operativo. Así surge el sistema operativo Fortran Monitor System.

Posteriormente, en la tercera generación de computadoras surge un sistema operativo


con la filosofía de administrar una familia de computadoras: el OS/360 de IBM. Esto puso
en evidencia la problemática del trabajo en equipos de análisis, diseño e implantación de
sistemas grandes. Surge también en esta generación el concepto de la multiprogramación;
debido al alto costo de las computadoras era necesario idear un esquema de trabajo que
mantuviese a la unidad central de procesamiento más tiempo ocupada.

En la cuarta generación la electrónica avanza hacia la integración a gran escala, lográn-


dose crear circuitos con miles de transistores en un centímetro cuadrado de silicón, se puede
hablar de las computadoras personales y las estaciones de trabajo. Surgen los conceptos de

539
Sánchez Marchán, Nanci. (2014).Introducción a la Ingeniería Industrial, Material Instruccional de Apoyo. Caracas: UNA

interfaces amigables, intentando así atraer al público en general al uso de las computadoras
como herramientas cotidianas. Se crean los programas MS-DOS y UNIX en estas máquinas.
También es común encontrar clones de computadoras personales y pequeñas empresas
ensamblándolas por todo el mundo.

Para mediados de los años 80, comienza el auge de las redes de computadoras y la
necesidad de sistemas operativos en red, así como, sistemas operativos distribuidos. La red
mundial Internet se va haciendo accesible a toda clase de instituciones y se comienzan a
dar muchas soluciones. Para los 90, el paradigma de la programación orientada a objetos
cobra auge, también el manejo de objetos desde los sistemas operativos. Las aplicaciones
intentan crearse para ser ejecutadas en una plataforma específica y poder ver sus resultados
en diferentes monitores.

INFLUENCIA E IMPORTANCIA DE LA COMPUTADORA Y LA INGENIERÍA


DE SISTEMAS EN LA INGENIERÍA INDUSTRIAL Y AFINES 
Ningún otro aspecto de la tecnología tiene probablemente mayor impacto potencial en la
ingeniería y profesiones afines como la computadora. Como el resto de los ingenieros, el In-
geniero Industrial y sus adjuntos (TSU en Higiene y Seguridad Industrial, entre otros) mane-
jan la programación de computadoras. Estas especialidades requieren control y simulación
que amplían el papel de los principios de la informática dentro de sus carreras. La mayoría
de las herramientas usadas en la ingeniería industrial son computarizadas, en la actualidad
el análisis y los diseños asistidos por computadora de los sistemas de producción, tienen un
nuevo potencial en la carrera. La simulación por computadora implica el uso de lenguajes
de programación especializados para modelar sistemas de producción, este modelado se
realiza con el uso de herramientas que proporciona la Ingeniería de Sistemas. Luego el com-
portamiento del modelo se analiza en la computadora, antes de comenzar a experimentar
con los sistemas reales.

La Ingeniería de Sistemas no construye productos tangibles, trata con sistemas abstrac-


tos con ayuda de las metodologías de la ciencia de sistemas, y confía en otras disciplinas
para diseñar y entregar los productos tangibles que son la realización de esos sistemas.
Está basada en parámetros funcionales sistémicos, en las interrelaciones más que en los
elementos. Su base ingenieril de enfoque reduccionista y su base de sistemas de enfoque
dominante, permiten entender cómo se debe ejercitar el pensamiento inductivo-deductivo,
aspecto que han adoptado otras disciplinas para mejorar sus procesos, entre ellas, la inge-
niería industrial.

DEFINICIÓN DE LA INGENIERÍA DE SISTEMAS

La Ingeniería de Sistemas se puede definir como:

Un área de conocimiento bajo un enfoque multidisciplinario que se encarga del diseño,


programación, implantación y mantenimiento de sistemas operativos.

540
Sánchez Marchán, Nanci. (2014).Introducción a la Ingeniería Industrial, Material Instruccional de Apoyo. Caracas: UNA

OTRAS DEFINICIONES
Ingeniería de Sistemas es la aplicación de las ciencias matemáticas y físicas para desa-
rrollar sistemas que utilicen económicamente los materiales y fuerzas de la naturaleza para
el beneficio de la humanidad.

Una definición especialmente completa que data de 1974, la ofrece un estándar militar
de las fuerzas aéreas estadounidenses sobre gestión de la ingeniería. Ingeniería de Siste-
mas es la aplicación de esfuerzos científicos y de ingeniería para:

1. Transformar una necesidad de operación en una descripción de parámetros de


rendimiento del sistema y una configuración del sistema a través del uso de un
proceso interactivo de definición, síntesis, análisis, diseño, prueba y evaluación.

2. Integrar parámetros técnicos relacionados para asegurar la compatibilidad de


todos los interfaces de programas y funcionales, de manera que optimice la de-
finición y diseño del sistema total.

3. Integrar factores de fiabilidad, mantenibilidad, seguridad, supervivencia, huma-


nos y otros en el esfuerzo de ingeniería total, a fin de cumplir los objetivos de
coste, planificación y rendimiento técnico.

Las definiciones antes expuestas involucran una serie de aspectos cuyo fin es el desa-
rrollo de sistemas, que permitan el procesamiento de información. Por ejemplo, una com-
putadora es el sistema formado por su hardware y su sistema operativo. También se puede
hablar de un sistema de contabilidad, de facturación, de base de datos, entre otros. En este
sentido se puede definir un sistema como:

Una combinación de programas, procedimientos, datos y equipamiento utilizados en el


procesamiento de información.

Se puede hablar de tres componentes fundamentales en una información: entrada, pro-


cesamiento y salida. La entrada implica la captura de datos que entran al sistema con el fin
de ser procesados. El procesamiento incluye todos los procedimientos de transformación
que convierten las entradas (datos) en salidas (información). En la salida se conducen los
elementos generados por la transformación, hasta su utilización final.

En toda empresa se maneja un volumen alto de información, razón por la cual dicha
información, es considerada como un recurso más para la empresa. Se debe administrar de
la misma forma en como se administra el personal, el dinero y las instalaciones físicas. Para
lograr que la información en una empresa sea fidedigna, oportuna y veraz es recomenda-
ble crear sistemas de información cuyos datos sean medibles, fácil de registrar, organizar,
almacenar y procesar.

Un sistema de información, es el medio que permite la fluidez de la información interna


o externa en una empresa; entre gerentes, empleados, proveedores y clientes, a través de
una base de datos que facilite la toma de decisiones (tema a tratar más adelante). Razón
por la cual la Ingeniería de Sistemas en la actualidad, ha experimentado un avance impor-
tante en los sistemas de información.

541
Sánchez Marchán, Nanci. (2014).Introducción a la Ingeniería Industrial, Material Instruccional de Apoyo. Caracas: UNA

LA INGENIERÍA DE SISTEMAS EN LA ACTUALIDAD


En la actualidad, así como sus inicios, en, la Ingeniería de Sistemas se orienta a la
documentación, es decir, los ingenieros de sistemas al desarrollar un producto documentan
éste, utilizan descripciones textuales y cálculos en un conjunto de documentos de especi-
ficaciones y diseño; complementando éstos con planos para describir la arquitectura física
del producto.

La anterior orientación tiene como desventaja, la dificultad de mantener la documenta-


ción al día, la ambigüedad del lenguaje natural para describir problemas de ingeniería y la di-
ficultad de las evaluaciones rigurosas de las soluciones escogidas en la toma de decisiones.
Todo ello ha generado la necesidad de un cambio que facilite los procesos en la Ingeniería
de Sistemas.

En la actualidad la Ingeniería de Sistemas está cambiando su orientación a un enfoque


dirigido a los modelos. Un modelo se puede definir como:

La representación de un proceso donde se describe, explica o comprende mejor la reali-


dad de situaciones complejas, para simplificarlas y poder predecir su futuro.

Existen diversos tipos de modelos que se diferencian entre ellos según el propósito que
persigan; entre los cuales se pueden mencionar: los modelos físicos, considerados los
más básicos (una maqueta); modelos gráficos, en lo que se dibujan de una forma simpli-
ficada los componentes del sistema; modelos numéricos, los cuales usan magnitudes y
ecuaciones matemáticas para describir con exactitud los distintos componentes del sistema
y las relaciones entre ellos; y los modelos computacionales, considerados como los más
complicados, que sólo pueden utilizarse empleando computadoras avanzadas, que permitan
manejar la gran cantidad de datos que poseen.

Los modelos describen el sistema según diferentes puntos de vista y utilizando notacio-
nes diversas. Este enfoque facilita los procedimientos y genera las siguientes ventajas:
•• Los modelos son un medio de comunicación con los clientes, usuarios y
fabricantes.
•• Permiten mantener la integridad del sistema mediante la coordinación de
las actividades de diseño.
•• Ayudan a diseñar los sistemas suministrando plantillas; organizando y
registrando las decisiones; permitiendo explorar y manipular los paráme-
tros, así como las características de la solución; guiando en la incorpora-
ción y descomposición de las funciones, sus componentes y elementos
constructivos.

Para evitar la existencia de un número excesivo de técnicas y herramientas de mode-


lado, la academia y la industria han tomado la iniciativa de estandarizar dichas técnicas,
proponiendo la notación y semántica para los siguientes tipos de modelos:

542
Sánchez Marchán, Nanci. (2014).Introducción a la Ingeniería Industrial, Material Instruccional de Apoyo. Caracas: UNA

1. Estructurales: describen al sistema de manera detallada, los subsistemas y


sus interrelaciones.
2. De comportamiento: modelos basados en sistemas autónomos y de compor-
tamiento social.
3. De requisitos: delimitan el sistema y capturan la funcionalidad que ofrecerá
desde la perspectiva del usuario. Se pueden mencionar aquí los métodos analí-
ticos, simulación, entre otros.
4. Paramétritos: se fundamentan en una distribución de probabilidad (F) pertene-
ciente a una familia de distribuciones representada por un parámetro de dimen-
sión finita.

También los profesionales de la industria y la academia de este sector, en su incesan-


te búsqueda por mejorar los procesos en la Ingeniería de Sistemas, dada la diversidad de
modelos con los cuales trabajaban, así como, su semántica y notación (símbolos), que de
alguna forma dificultaba el trabajo en equipo; lograron en 2001 adaptar el estándar para la
aplicación de softwares, UML (Unified Modeling Language) a uno más general que pudiera
facilitar el desarrollo de cualquier tipo de sistema. Esta nueva notación se denomina sysML
(System Modeling Language) y proporciona técnicas de modelado de una gran variedad de
sistemas, los cuales incluyen equipos físicos, software, datos, personas, procedimientos e
instalaciones.

En el modelado se contempla la necesidad de disminuir la complejidad de los sistemas,


por ello se consideran aspectos importantes como los elementos y vistas que lo conforman,
para su mejor comprensión y manejo.

Elementos y vistas de un sistema

Un sistema se considera como un conjunto integrado de elementos que realizan una


misión u objetivo específico, por consiguiente lo que caracteriza a un sistema es la alta inte-
gración entre sus elementos constructivos, los cuales pueden ser elementos físicos o lógicos
que inducen a la consecución de un objetivo único o misión, para el cual el sistema ha sido
desarrollado.

Los sistemas requieren que sean representados utilizando diferentes vistas. El conjunto
de vistas (una representación de todo el sistema software desde una determinada perspec-
tiva) utilizadas para describir un sistema ha de ser lo más completa que sea posible, para
poder cubrir todos los aspectos concernientes al mismo, es decir, cada vista captura informa-
ción diferente relativa al sistema. Pero no necesariamente éste tiene que hacer uso de todas
las vistas, el uso de ellas necesariamente dependerá de la función para la cual fue creado
dicho sistema.

Se muestra seguidamente como ejemplo, la representación gráfica de un modelo de


sistema propuesto por Rechtin (ingeniero de sistemas norteamericano, autoridad respetada

543
Sánchez Marchán, Nanci. (2014).Introducción a la Ingeniería Industrial, Material Instruccional de Apoyo. Caracas: UNA

en el sector aeroespacial y en la arquitectura de sistemas), así como la definición de las dife-


rentes vistas que lo componen:

Prestaciones

Propósito/Objetivos Comportamiento

EL SISTEMA
Dirección Estructura

Datos
Datos

Tabla 1 Tabla 2
1111 12 1131 14
221 222 223 224

Figura 81. Sistema propuesto por Rechtin.

Definición de las vistas del sistema propuesto por Rechtin:


•• Propósitos/Objetivos: representa las necesidades del cliente y usua-
rios del sistema.
•• Estructura también llamada de forma: constituye los elementos o par-
tes del sistema y su organización.
•• Comportamiento o vista funcional: describe lo que hace el sistema a
diferencia de la vista estructural que describe como es el sistema.
•• Prestaciones: esta vista describe o predice cómo el sistema va a sa-
tisfacer una funcionalidad, normalmente esta vista es cuantitativa y los
modelos que la describen se aplican a funcionalidades del sistema.
•• Datos: vista importante en aquellos sistemas en los cuales la informa-
ción a gestionar es elevada, es decir, describe los datos y relaciones
entre éstos, de importancia para el sistema.
•• Dirección: representa los aspectos de dirección del proyecto en el siste-
ma, su planificación, programación y presupuesto.

544
Sánchez Marchán, Nanci. (2014).Introducción a la Ingeniería Industrial, Material Instruccional de Apoyo. Caracas: UNA

Cada una de las vistas anteriores puede ser representada por multitud de modelos, pero
sólo se mencionarán algunos de ellos para cada vista. Ver tabla 53:

Tabla 53. Modelos para las vistas del sistema de Rechtin


Vista Modelos
Representación de requisitos, objetivos y casos de uso del
Propósito/Objetivos
sistema.
Se utilizan diagramas de bloques (interconexión de sistemas,
diagramas de flujo, diagramas de procesos, diagramas de
Estructura o forma
clase en UML, diagramas de instalaciones de fabricación) y
modelos a escala y prototipos.
Existen diversa técnicas de modelado como escenarios, redes
de datos y eventos, modelos basados en la teoría de sistemas
Comportamiento o
(sistemas continuos, sistemas discretos) modelos basados en
vista funcional
sistemas autónomos y caóticos, además de modelos de com-
portamiento social.
Utiliza diversas técnicas de modelado que se pueden basar
Presentaciones en métodos analíticos, modelos de simulación o técnicas de
evaluación de las prestaciones por expertos.
Modelos de entidad relación, modelos orientados a objetos,
Datos
filosofías y redes semánticas.
Planificación y programación de proyectos (Gann, PERT,
Dirección de proyectos
Otras)

Actividad propuesta Nº 1
Investigue algunos sistemas planteados por otros personajes respetados en la
arquitectura de sistemas. Cite las diferencias que tienen con el sistema propuesto
por Rechtin.

Considerando todo lo anterior se puede decir, que sólo el hombre ha pasado de la ex-
plosión atómica a la explosión digital y virtual, de ahí le espera un largo camino hacia las
explosiones universales de los sistemas. Y por ello el ingeniero industrial debe dirigir su edu-
cación, conocimiento, entrenamiento, experiencia, etcétera, dentro de los campos sistémicos
y las tecnologías. Debe ser capaz de determinar los factores involucrados en la producción,
en los valores agregados, en los recursos relacionados con el hombre y en cualquier ámbito
económico. Las premisas y prioridades deben ser el bien común, comprendiendo las leyes
que rigen el funcionamiento de los campos sistémicos que proporciona la Ingeniería de Sis-
temas, lo cual se traduce en el logro de una mayor eficiencia y eficacia de su labor.

SISTEMAS DE INFORMACIÓN

En la búsqueda de la eficacia y eficiencia, la Ingeniería de Sistemas se ha preocupado


por mejorar y clasificar sus sistemas de información, en atención a las necesidades de la
organización. Se debe recordar que un sistema de información es aquel que permite la

545
Sánchez Marchán, Nanci. (2014).Introducción a la Ingeniería Industrial, Material Instruccional de Apoyo. Caracas: UNA

fluidez de la información interna o externa en una empresa, a través de una base de


datos que facilite la toma de decisiones. Una organización se caracteriza por poseer inte-
reses, especialidades y niveles distintos, por ello se generan diferentes tipos de sistemas de
información. Se puede hablar de sistemas a nivel operativo, del conocimiento, administrativo
y estratégico. A continuación se presenta una figura que recoge los sistemas citados, así
como su definición:

SISTEMAS A NIVEL ESTRATÉGICO


Ayuda a los directores a enfrentar y resolver as-
pectos estratégicos y tendencias a largo plazo,
SISTEMAS DE INFORMACIÓN tanto en la empresa como en el entorno.

SISTEMAS A NIVEL ADMINISTRATIVO


Contribuye con las actividades de surpevisión,
control, toma de decisiones y administrativas de
los gerentes medios. Proporciona información
periódica.

SISTEMAS A NIVEL DE CONOCIMIENTO


Ayuda a los trabajadores del conocimiento y
obtención de datos de una empresa. Permite
integrar conocimientos nuevos del negocio, así
como a controlar el trabajo de oficina.

SISTEMAS A NIVEL OPERATIVO


Apoya a los gerentes operativos en el segui-
miento de transacciones elementales como:
ventas, ingresos, depòsitos en efectivo, nómina,
flujo de materiales, decisiones de crédito, etc..

Figura 82. Los sistemas de información y sus niveles.

Los sistemas de información son independientes unos de otros, apoyan por separado
los procesos de negocios y las funciones empresariales, lo que los hace importante para la
Ingeniería Industrial. Un sistema de información acopia información sobre una organización y
su entorno. Dentro de este entorno figuran clientes, proveedores, competidores, accionistas
y agentes reguladores que interactúan con la organización y sus procedimientos.

546
Sánchez Marchán, Nanci. (2014).Introducción a la Ingeniería Industrial, Material Instruccional de Apoyo. Caracas: UNA

Ejemplo Ilustrativo Nº 1
A continuación se presentan algunos ejemplos de sistemas a nivel estratégico
que se pueden aplicar dentro de la empresa:
•• Pronósticos de tendencias de ventas a cinco años.
•• Plan Operativo de cinco años.
•• Planeación de utilidades.
•• Planeación de personal.

Ejemplo Ilustrativo Nº 2
Caso: empresa SANAE S.A.
La empresa SANAE S.A., se crea para la fabricación de cartón para la elabora-
ción de cajas, años después establece la primera planta productora de envases
de cartón para diferentes empresas manufactureras.
Al asumir una política de calidad de compromiso con sus clientes tanto nacionales
como extranjeros SANAE S.A., emprendió un proceso de transformación orienta-
do a dar a sus clientes un servicio basado en la calidad personal. Es ahí cuando
implementa el SIO (Sistema de Información Operativa). Comenzó en el área de
ventas de SANAE y a éste módulo se le llamó semáforos, debido a que en él se
muestra información para saber si la empresa está operando correctamente o no.
El objetivo del sistema es presentar información diaria que se actualiza al día de
ayer para todas las plantas de la empresa.

Información de este sistema


Operaciones de la empresa:
-Flujo de efectivo.
-Junta de resultados.
-Noticias.
-Nueva organización.
-Junta de consejo.
-Responsables.

¿Qué tipo de nuevo sistema de información requiere SANAE S.A.?


Analizando detalladamente las características que tiene el sistema de informa-
ción existente, los ejecutivos requieren de un soporte para la toma de decisiones
que ayude a cambiar la forma de operar de la empresa y así tener ventajas com-
petitivas. Necesitan un sistema de información de Nivel Administrativo, el cual
puede apoyar las decisiones no rutinarias y tienden a enfocarse en decisiones
menos estructuradas.

Actividad propuesta Nº 1
Cite algunos otros ejemplos de aplicación de sistemas administrativos utilizados
en una empresa de servicio.

547
Sánchez Marchán, Nanci. (2014).Introducción a la Ingeniería Industrial, Material Instruccional de Apoyo. Caracas: UNA

SISTEMAS DE INFORMACIÓN MÁS UTILIZADOS PARA LA GESTIÓN POR


LA INGENIERÍA INDUSTRIAL
Dentro de la Ingeniería Industrial también se presenta la elección de estrategias, diseño
e  implantación de sistemas de control, desarrollo de controles administrativos y técnicas de
optimización. En atención a esto, se puede observar la aplicación que tiene la Ingeniería de
Sistemas para la Ingeniería Industrial en los siguientes sistemas de gestión:
•• Gestión de operaciones en sistemas productivos
El enfoque sistémico de la gestión. Sistemas productivos. Procesos de transformación:
operaciones. Problemáticas y tendencia de la gestión. Tecnología informática.
•• Predicción y pronóstico
Rol estratégico del pronóstico. Patrones de comportamiento de la demanda. Proceso de
predicción, simulación con base en modelos.
•• Administración de inventarios
Inventarios de demanda dependiente e independiente. Sistemas de control de inventa-
rios: Sistema de revisión continua y sistema de revisión periódica. Principales modelos bási-
cos de inventarios con cantidad fija de pedido. Demanda constante. Cálculo del lote óptimo.
Modelo básico de inventario de período fijo. Tecnología de Información Disponible.
•• Planificación
Proceso de planificación jerárquico. Planificación de capacidad y planificación agregada
de producción. Aplicación de programación Lineal. Softwares de aplicación.
•• Planificación de requerimientos de materiales
Planificación de requerimientos de materiales. Sistemas MRP. Estructura del producto.
La matriz MRP. Plan de órdenes. Plan de requerimientos. Sistemas MRPII: planificación de
los recursos de manufactura. Sistemas ERP limitaciones.
•• Calendarización
Scheduling. La función scheduling por tipos de procesos. Scheduling de operaciones
batch/job shop. Método de asignación de carga de máquinas. Secuenciamiento. Diagrama
de Gantt. Clasificación de los procesos. Métodos matemáticos. Métodos mixtos. Flow shop
scheduling. Manufactura sincrónica (OPT). Algoritmo genético. Tecnología de información
disponible.
•• Sistemas de distribución de productos
Canales de distribución. Gestión de inventarios en un canal de distribución. Sistemas
DRP.
•• Filosofía justo a tiempo (JIT)
Líneas de flujo, sistema de arrastre kanban.
•• Cadena de suministro
Gestión integrada de la cadena de suministro: objetivo, diseño, modelos de integración,
respuesta rápida (QR), Programa de abastecimiento continuo (CRP), inventario administrado

548
Sánchez Marchán, Nanci. (2014).Introducción a la Ingeniería Industrial, Material Instruccional de Apoyo. Caracas: UNA

por el proveedor (VMI), pronóstico, planeamiento y reaprovisionamiento colaborativo (CPFR).


Tecnología de información: Sistemas B2B.
•• Análisis de decisión
Tabla de pagos. Criterios de decisión bajo incertidumbre. Árboles de decisión. Valor
esperado de la información perfecta. Árboles de decisión secuenciales. Softwares de apli-
cación.
•• Gestión de proyectos
Modelo PERT/CPM. Cálculo fechas tempranas y tardías, holguras y camino crítico. Du-
ración aleatoria de las tareas y del proyecto.

La Ingeniería de Sistemas y la Ingeniería Industrial comparten un interés


común en estructuras matemáticas discretas. 

También se puede observar la aplicación que tiene la Ingeniería de Sistemas en la


Ingeniería Industrial, en el número de softwares que se utilizan para agilizar los procesos y
resolver problemas en las diferentes áreas que conforman esta disciplina.

ALGUNOS SOFTWARES DE APLICACIÓN USADOS EN LA INGENIERÍA


INDUSTRIAL

Es un poderoso paquete que integra numerosas herramientas de solu-


WinQSB ción de problemas en áreas como investigación de operaciones, diseño
de sistemas productivos, planeación y control de producción, sistemas
de calidad, planeación, entre otras.
Permite resolver los principales problemas planteados en la dirección
S.D.P de operaciones. Así, incluye resolución de problemas de programación
lineal, gestión de inventarios, productividad, equilibrado de cadena de
montajes, gráficos de control, teoría de colas, teoría de juegos, produc-
tividad, localización, etc.
Es una herramienta informática especialmente diseñada para ayudar
Lingo en el modelado y resolución de problemas de optimización lineal de una
manera rápida, fácil y eficiente.
Es el programa de análisis de riesgo más usado en el mundo. Aunque
@ RISK ningún programa de software puede predecir el futuro, @ RISK permite
seleccionar la mejor estrategia basándose en la información disponible,
ayuda a tomar decisiones de una manera fácil y eficaz.
Premium Ayuda a resolver problemas mucho más complejos (alrededor de 1000
Solver variables)  a una mayor velocidad.

549
Sánchez Marchán, Nanci. (2014).Introducción a la Ingeniería Industrial, Material Instruccional de Apoyo. Caracas: UNA

Es la herramienta de administración de proyectos más usada en el


Microsoft
mundo, la cual ayuda en las etapas de formulación, planeación, admi-
Project
nistración, control y ejecución de diversos tipos de proyectos.
Grafx
FlowCharter
Es una herramienta integrada de software que permite unificar los dife-
Grafx rentes procesos estratégicos de su organización a través de un am-
Process for biente totalmente amigable y especialmente diseñado para el modela-
Six Sigma do, análisis y mejora de los diferentes procesos enfocados a negocios.
Grafx IDEF0
Poderoso programa para la elaboración de presentaciones, sitios web,
MinManager resúmenes, informes y material didáctico por medio de mapas menta-
les. Permite combinar una serie de elementos multimedia como texto,
imágenes, animaciones y videos.
Blender Diseño y modelado de animaciones y gráficos en 3D
Ayuda a comprimir y descomprimir archivos de una manera rápida y
Win Zip
sencilla.
Es un poderoso programa para la resolución de problemas y análisis
de alternativas desde el punto de vista estadístico. Administra datos,
evalúa y compara estadísticas generales (análisis de regresión, aná-
Minitab
lisis de varianza, control de procesos estadísticos, herramientas de
calidad), análisis de sistemas de medición, diseño de experimentos,
gráficas de vanguardia, entre otras aplicaciones.
Es un completo sistema de gestión de la producción que le permite
administrar materias primas, materiales, mano de obra, chequear pro-
cesos, establecer fórmulas de producto, generar planes de producción,
ERP
asignar costos y evaluar requerimientos de equipos, personas y artí-
Manager.
culos para cumplir con sus metas industriales. Sus avanzadas herra-
mientas de trazabilidad lo convierten en un sistema indispensable para
empresas certificadas bajo normas de calidad ISO.
Otros Softwares
Software de la infraestructura de la empresa:
•• Negocio workflow software.
•• Sistemas de Gerencia de Base de Datos de Software(DBMS).
•• Gerencia de activo de Digital Software (DAM).
•• Gerencia del documento software.

Software del trabajador de la información:


•• Tiempo y gerencia de recurso:
- Software de la contabilidad.
- Tarea y Scheduling.
- Gerencia del servicio de campo software.

550
Sánchez Marchán, Nanci. (2014).Introducción a la Ingeniería Industrial, Material Instruccional de Apoyo. Caracas: UNA

•• Gerencia de datos:
- Gerencia del contacto.
- Hoja de balance.
- Base de datos personal.

•• Documentación:
- Procesamiento de textos.
- Software el publicar de escritorio.
- Software de Diagrama Ming.
- Software de la presentación.

•• Software de Colaboración:
- E-mail.
- Blog.
- Wiki.

Software de la ingeniería del producto:


•• Ingeniería del harware:
- Ingeniería automatizada.
- Diseño automatizado (cad).

La Ingeniería de Sistemas abarca más que un conjunto de técnicas, es un enfoque que


ubica principalmente los objetivos globales y  las relaciones entre las partes de una empresa
manufacturera o de servicio, la sociedad, o cualquier otro sistema; razón por la cual la Inge-
niería Industrial se apoya en esta disciplina para obtener mejores soluciones a problemas
complejos, especialmente en la economía de recursos y la sociedad.

551
Sánchez Marchán, Nanci. (2014).Introducción a la Ingeniería Industrial, Material Instruccional de Apoyo. Caracas: UNA

TEORÍA DE LA DECISIÓN
•• Modelos de criterios de decisión.
•• Etapas de la toma de decisión para dar solución a un problema.
•• Componentes de la decisión.
•• Importancia de la toma de decisiones.
•• Técnicas para la toma de decisión en grupo.
•• Modelos y técnicas usadas en la toma de decisiones en las diferentes
áreas de la Ingeniería Industrial.

ORIGEN Y DEFINICIÓN DE LA TOMA DE DECISIONES


Chester Barnard, ejecutivo de una empresa telefónica importó el término toma de deci-
siones desde la administración pública al mundo de los negocios, reemplazándose términos
como asignación de recursos o formulación de políticas. La confianza en la gestión del ries-
go, junto a una mejor comprensión del comportamiento humano y avances que respaldan e
imitan los procesos cognitivos, han mejorado la toma de decisiones.

Está relacionada con las ramas de la ciencia, ingeniería, así como las actividades so-
ciales. Permite resolver los distintos desafíos a los que debe enfrentarse una persona u
organización.

La toma de decisiones se puede definir como:

El proceso que consiste en realizar una elección óptima entre diversas alternativas.

Se consideran un estudio formal sobre la toma de decisiones: los estudios de ca-


sos reales, que se sirven de la inspección y los experimentos, se denominan teoría descripti-
va de decisión; los estudios de la toma de decisiones racionales, que utilizan la lógica y la es-
tadística, se llaman teoría preceptiva de decisión. Estos estudios se hacen más complicados
cuando hay más de un individuo, cuando los resultados de diversas opciones no se conocen
con exactitud y cuando las probabilidades de los distintos resultados son desconocidas.

La toma de decisión es también un proceso durante el cual la persona debe escoger


entre dos o más alternativas. Todos pasamos los días y las horas de la vida teniendo que
tomar decisiones. Algunas decisiones tienen una importancia relativa en el desarrollo de la
vida, mientras otras son pasajeras en ella.

Con frecuencia se dice que las decisiones son algo así como el motor de los negocios y
en efecto, de la adecuada selección de alternativas depende en gran parte el éxito de cual-
quier organización.

553
Sánchez Marchán, Nanci. (2014).Introducción a la Ingeniería Industrial, Material Instruccional de Apoyo. Caracas: UNA

Los administradores consideran a veces la toma de decisiones como su trabajo princi-


pal, sin embargo, es sólo un paso de la planeación, la primera función de la administración.
Los responsables de la toma de decisiones utilizan modelos de predicción, como meca-
nismos de soporte en el proceso de toma de decisiones; de manera de poder investigar el
impacto de acciones regresivas.

MODELOS DE CRITERIOS DE DECISIÓN


Certeza: se sabe con seguridad cuáles son los efectos de las acciones.

Riesgo: no se sabe qué ocurrirá tomando determinadas decisiones, pero sí se conoce


qué puede ocurrir y cuál es la probabilidad de ello.

Incertidumbre estructurada: no se sabe qué ocurrirá tomando determinadas decisio-


nes, pero sí se sabe qué puede ocurrir entre varias posibilidades.

Incertidumbre no estructurada: en este caso no se conoce qué puede ocurrir ni tam-


poco que probabilidades hay para cada posibilidad. Es cuando se desconoce lo qué puede
pasar.

Conflicto: tomar decisiones en función de otros.

COMPONENTES DE LA DECISIÓN
La técnica de tomar decisiones está basada en cinco componentes primordiales:

1. Información: la información se recoge tanto para los aspectos que están a fa-
vor como en contra del problema, con el fin de definir sus limitaciones. Aquí se
puede formular la siguiente pregunta: ¿Qué es lo que se quiere obtener con la
situación planteada?
2. Conocimientos: si quien toma la decisión tiene conocimientos, ya sea de las
circunstancias que rodean el problema o de una situación similar, entonces estos
pueden utilizarse para seleccionar un curso de acción favorable.
3. Experiencia: cuando un individuo soluciona un problema en forma particular, ya
sea con resultados buenos o malos, esta experiencia le proporciona información
para la solución de algún problema similar.
4. Análisis: No puede hablarse de un método en particular para analizar un proble-
ma, debe existir un complemento, pero no un reemplazo de los otros ingredien-
tes. En ausencia de un método para analizar matemáticamente un problema, es
posible estudiarlo con otros métodos diferentes. Si estos otros métodos también
fallan, entonces debe confiarse en la intuición.

554
Sánchez Marchán, Nanci. (2014).Introducción a la Ingeniería Industrial, Material Instruccional de Apoyo. Caracas: UNA

5. Juicio: el juicio es necesario para combinar la información, los conocimientos, la


experiencia y el análisis, con el fin de seleccionar el curso de acción apropiado.
No existen sustitutos para el buen juicio.

ETAPAS DE LA TOMA DE DECISIÓN PARA DAR SOLUCIÓN A UN PROBLEMA

Estas etapas son:

Identificación y diagnóstico del problema. El directivo debe


ser capaz de distinguir entre causa y efecto, entre el síntoma y el
problema; además conocer los antecedentes que estén a la dis-
posición en la institución o empresa, es decir, precisar si se trata
de un problema actual o si se desarrolló anteriormente.

Investigación y recopilación de información. La información


reduce la incertidumbre y con ello se clarifica la toma de deci-
siones, para así poder alcanzar la solución correcta. No se debe
pensar nunca que es suficiente la información que se tiene, es
necesario recabar más información y obtener nuevos datos que
ayuden a distinguir las alternativas de solución que existen.

Generación de alternativas de solución. Desarrollar posibles


soluciones al problema, es decir; conocer los distintos caminos
que se pueden tomar para solucionar el problema. Entre más al-
ternativas existan es probable encontrar una que resulte satisfac-
toria.

Determinar algunos criterios de decisión para evaluar las al-


ternativas. Sobre la base de la investigación realizada para ob-
tener información, se establecen algunos aspectos que son rele-
vantes (criterios) al momento de tomar la decisión, los cuales son
ponderados para facilitar la evaluación.
Evaluación de las alternativas. Consiste en estudiar las venta-
jas y desventajas de las diferentes alternativas; considerando los
criterios seleccionados, así como, la factibilidad de su ejecución,
los recursos necesarios para llevarlas a cabo de acuerdo con el
marco específico de la organización, es decir se aplica la toma de
decisión.

Selección de la mejor alternativa. En esta fase el administra-


dor está en capacidad de considerar las consecuencias de sus
posibles opciones y elegir aquella alternativa que cuenta con los
mejores atributos de calidad y adaptabilidad.

555
Sánchez Marchán, Nanci. (2014).Introducción a la Ingeniería Industrial, Material Instruccional de Apoyo. Caracas: UNA

Implantación y control de la decisión. En este punto se debe


asegurar que el plan se lleve a cabo a tiempo, aplicando los con-
troles adecuados, para asegurar que se encuentre dentro de los
límites deseados.

La forma en la cual los individuos toman decisiones en las organizaciones


y la calidad de las opciones que eligen está influenciada principalmente
por sus percepciones, por sus creencias y sus valores.

Ejemplo Ilustrativo:
Aplicación de los pasos a seguir en la toma de decisiones, en el desarrollo
de una empresa de servicio.

Identificación y diagnóstico del problema:

Identificación:

Elaborar un proyecto que contribuya al desarrollo turístico en el país.

Diagnóstico:
•• No existe una infraestructura adecuada que respalde la actividad turísti-
ca.
•• Existen muchos paisajes naturales para fomentar el turismo, pero con
poca seguridad en ellos.
•• No se evidencia una cultura en la población en este ámbito.
•• Poco personal capacitado en el área de turismo.

Investigación y recopilación de información:

Obtener información referente a la actividad económica de cada región, servicios que


se deben prestar como gerentes en el área de turismo, conocer la infraestructura con la cual
se cuenta en cada región, así como el personal capacitado en el área de turismo. Análisis
de costo–beneficio, etc.

Generación de alternativas:
•• Desarrollar la actividad turística por regiones, utilizando los recursos que
posee cada zona.
•• Asociar a empresarios dedicados a la actividad turística en diferentes
estados del país, para que de manera mancomunada y organizada se
pueda desarrollar el turismo en todo el territorio nacional.

556
Sánchez Marchán, Nanci. (2014).Introducción a la Ingeniería Industrial, Material Instruccional de Apoyo. Caracas: UNA

•• El proyecto puede ser asumido por todos los sectores del país (gobierno,
empresa privada, comunidad, universidades que dicten la carrera Turis-
mo, entre otros)

Determinar algunos criterios de decisión para evaluar las alternativas:


•• Economía y prosperidad de las regiones.
•• Condiciones naturales necesarias para desarrollar el turismo.
•• Infraestructura adecuada para iniciar el proyecto.
•• Regulaciones gubernamentales existentes.
•• Disposición de los factores de la sociedad, para involucrarse en el pro-
yecto.
•• Costos para la implantación.

Evaluación de alternativas: 
•• Iniciar la actividad turística por regiones con los recursos que existan en
cada zona, dándole prioridad a las regiones que necesiten mejorar su
economía y prosperidad. Sobre la marcha, con los resultados y bene-
ficios que se obtengan, servir de agente multiplicador para desarrollar
el turismo en otros estados del país, disminuyendo así los costos de im-
plantación.
•• Que se genere una sociedad de empresarios dedicados a la actividad
turística en diferentes estados del país, para que de manera mancomu-
nada y organizada se pueda desarrollar el turismo en todo el territorio na-
cional. El aporte de capital de los socios permitiría crear las condiciones
necesarias para desarrollar el turismo (infraestructura, capacitación de
personal, seguridad) en todo el país.
•• El proyecto puede ser asumido por todos los sectores del país (gobierno,
empresa privada, comunidad, universidades que tengan en su pensum
de estudio la carrera Turismo, entre otros). Es una forma de involucrar a
los diferentes factores de la sociedad, trabajando por su región, crean-
do conciencia en cuanto a la importancia que tiene el cuidado de los
recursos naturales, asimismo fomentando una cultura, la cual puede ser
heredada por generaciones futuras. Los costos pueden ser asumidos
por el gobierno y la empresa privada. El beneficio lo obtendrían todos los
involucrados en el proyecto, así como, el país.

Selección de la mejor alternativa:

Se ha decidido desarrollar la actividad turística involucrando a todos los sectores de


la sociedad. Esta decisión permitiría crear conciencia y una cultura enfocada al respeto y
cuidado de los recursos naturales existentes en las diferentes zonas del país. El aporte eco-

557
Sánchez Marchán, Nanci. (2014).Introducción a la Ingeniería Industrial, Material Instruccional de Apoyo. Caracas: UNA

nómico para la implantación lo asumiría el gobierno y la empresa privada. La comunidad se


encargaría de resaltar la idiosincrasia de sus regiones.

Implantación y control de la decisión:

Para llevar a cabo la ejecución de la decisión, se deben desarrollar los pasos pertinen-
tes, para la creación de una empresa, además se debe crear la infraestructura requerida,
capacitación del personal, crear una cultura respecto al cuidado y mantenimiento de los
lugares naturales.

Para el monitoreo de las condiciones del proyecto, se decidió realizar reportes del acon-
dicionamiento de la infraestructura existente en la comunidad para el turismo, educación y
capacitación del personal, ganancias que perciben los involucrados en el proyecto cada año.

Actividad propuesta
Considere la situación referente al transporte público en la zona metropolitana de
la región donde vive. Aplique los pasos a seguir en la toma de decisiones.

IMPORTANCIA DE LA TOMA DE DECISIONES


La toma de decisiones es una actividad de gran importancia dentro de cualquier grupo
social: familia, club social, empresa, institución, etc. Esta actividad en muchos casos es asu-
mida por el líder del grupo o de un alto funcionario. Este personaje debe tener características
y conocimientos necesarios y suficientes, que le permitan tomar decisiones sustentadas en
el seguimiento de objetivos, como en la supervivencia del grupo.

En esta actividad se puede considerar un problema, para llegar a una conclusión válida,
significa que se han examinado todas las alternativas y que la elección ha sido correcta. Uno
de los enfoques más competitivos de investigación y análisis para la toma de decisiones es
la investigación operativa. Puesto que ésta, es una herramienta importante para la adminis-
tración de la producción y las operaciones.

La toma de decisiones se considera como parte importante del proceso de planeación,


cuando ya se conoce una oportunidad y una meta. El núcleo de la planeación es realmente
el proceso de decisión.

TÉCNICAS PARA LA TOMA DE DECISIONES EN GRUPO

Técnica de interacción. Aquí el grupo toma las decisiones a través de una comunica-
ción frontal. Es usual que el grupo intercambie opiniones que son criticadas, ejerciéndose
presión entre los miembros individuales, para llegar a un acuerdo.

558
Sánchez Marchán, Nanci. (2014).Introducción a la Ingeniería Industrial, Material Instruccional de Apoyo. Caracas: UNA

Técnica de tormenta de ideas. Proceso de generación de ideas que acepta todas las


alternativas, sin criticarlas. Tiene la intención de vencer las presiones que buscan la con-
formidad del grupo interactuante y que algunas veces retarda el desarrollo de alternativas
creativas, registradas luego para su discusión y análisis.

Técnica de grupo nominal.  Esta técnica limita el debate o comunicación interpersonal


durante el proceso de toma de decisiones, por ello se llama nominal. Todos sus miembros
están presentes físicamente, como en una reunión tradicional de comité, pero éstos operan
de forma autónoma.

Conferencias electrónicas. Se facilita a los participantes los temas a discutir para la


toma de decisiones. Los cuales deben analizarlas y escribir sus respuestas en la pantalla
de las computadoras. A su vez se despliega una pantalla de proyección en la sala, donde
se muestran estos comentarios individuales y la votación que permitirá la selección de la
alternativa más votada. Este enfoque es el más reciente de la toma de decisiones en grupo
y combina la técnica de grupo nominal con la tecnología electrónica.

MODELOS Y TÉCNICAS UTILIZADAS EN LA TOMA DE DECISIÓN, EN SI-


TUACIONES CONCEBIDAS POR DIFERENTES ÁREAS DE LA INGENIERÍA
INDUSTRIAL
En las disciplinas Ingeniería Industrial e Higiene y Seguridad industrial, entre otras, se
deben tomar decisiones en una gran diversidad de situaciones que se presentan, tanto en
el área operativa como en el área de gerencia. Para ello se hace uso de una serie de herra-
mientas, que se sustentan en modelos matemáticos. Un modelo es: Ver definición en el tema
Ingeniería de Sistemas.

Estos modelos hacen uso de algunos contenidos de las áreas antes mencionadas (ope-
rativa, gerencia) incluso de las áreas afines a la Ingeniería Industrial (Ingeniería de Sistemas,
Estadística). Se muestra a continuación una tabla, en la que se puede observar un bosquejo
de algunas herramientas con sus respectivos modelos, que hacen uso de los contenidos de
éstas áreas.

559
Tabla 54. Modelos y técnicas utilizadas en la toma de decisión en las diferentes áreas de Ingeniería Industrial

Área Tema Modelo/ Técnica Campo de aplicación


Operativa
Planificación Modelos de fila de espera. Planificar los niveles de la fuerza laboral, el tiempo extra, el
inventario.
Evaluar alternativas de expansión de la capacidad cuando la
demanda es incierta.
Modelo de árbol de decisión.
Inventario Modelos probabilísticos y determinísticos. Se utilizan para auxiliar a los administradores que enfrentan
los problemas duales de mantener suficiente inventarios
para satisfacer la demanda de bienes y al mismo tiempo de
incurrir en los menores costos posibles por el mantenimiento
de esos inventarios.
Calidad Métodos cualitativos: Se obtiene infor- • Definen la naturaleza del problema y descubren proble-
mación sobre un problema de manera mas ocultos.
estructurada: Gráficos de causa y efecto, • Ayudan a organizar y ordenar ideas confusas.
flujo, afinidad, matriz.
• Muestran la dirección adecuada para implantar mejoras.

560
Se pueden utilizar ante la búsqueda de una causa o ante la
Métodos cuantitativos: Utilizan datos búsqueda de una solución.
numéricos para actuar sobre una situación Encontrar los cambios en inventarios que generaran los
real: Gráfico de Pareto, de Dispersión, de beneficios más altos. Útil donde existen muchos caminos de
Control Estadístico. acción compitiendo para ser seleccionado.

Pronósticos Técnicas cuantivas: Pueden emplearse para predecir aspectos futuros de una
operación de negocios o en el Gobierno.
Series de tiempo. Análisis de Estacionali-
dad Elaboración de presupuestos y de la planeación operativa en
diferentes departamentos de una compañía: mercadotecnia,
Tendencia. producción, finanzas, etc.
Análisis Delphi.
Análisis de sensibilidad. Incorporan factores importantes como la intuición, emocio-
nes, experiencias personales del que toma la decisión así
como sistemas de valores para alcanzar un pronóstico.
Técnicas cualitativas (Subjetivas)
Sánchez Marchán, Nanci. (2014).Introducción a la Ingeniería Industrial, Material Instruccional de Apoyo. Caracas: UNA
Investigación • Modelos de programación lineal. • Organización y gerencia.
de Operaciones • Modelos de toma de decisión con certe- • Ausentismo y relaciones de trabajo.
za o incertidumbre. • Economía.
-Programación
• Modelos de procesos estocásticos. • Decisiones individuales.
lineal. • Modelos de teoría de colas. • Investigaciones de mercado.
-Programación
• Modelos de optimización con redes para • Eficiencia y productividad.
Entera.
la planeación, ejecución y control de
proyectos. • Organización de flujos en fábricas.
-Cadenas de • Modelos con cadenas de Markov • Métodos de control de calidad, inspección y muestreo.
Markov. • Prevención de accidentes.
• Modelos de inventarios.
-Teoría de • Modelos de programación dinámica y • Organización de cambios tecnológicos.
juegos. teoría de juegos. • Diseño de sistemas.
-Simulación. • Modelos de simulación. • Transporte.
-Teoría de
• Almacenamiento, distribución y manipulación.
colas. • Comunicaciones.
• Programación de proyectos.
Gerencia
Pronósticos • Modelos financieros, Series de tiempo y • La adopción de decisiones sobre los costos unitarios, el
econometría. volumen de producción y el precio de venta.
• Mejora de productos.
• Técnica de costo beneficio. • Reemplazo de equipo.

561
Economía • Modelo de insumo-producto. • Introducir o eliminar una línea de producto.
• Análisis de rentabilidad. • Determinar planeación estrategia.
• Modelos de redes. • Fundamentar la estratégica. competitiva.
• Modelos probabilísticos. • Reducción de costos.
• Modelos determinísticos. • Localización y distribución en planta.
Finanzas e • Modelos de estacionalidad y tendencia. • Política de confiabilidad y sustitución.
inversión
• Modelos de autorregresión. • Control de existencias y falta de stock.
• Introducción de nuevos productos.
• El modelo costo-volumen-utilidad. • Selección de mejores procesos de producción.
• Técnica de punto de equilibrio. • Análisis del modelo de costo-volumen-utilidad.
Contabilidad
• Determinación de precios.
de costos • Nivel de ventas para cubrir costos.
Sánchez Marchán, Nanci. (2014).Introducción a la Ingeniería Industrial, Material Instruccional de Apoyo. Caracas: UNA
Mercado y Modelos de compra o adquisición. Investigación de mercado. Convertir los productos en dinero
marketing en efectivo.
Relación entre las ventas y la publicidad.

Modelos de campañas publicitarias. La selección de estrategias de mercado para el lanzamiento


de nuevos productos.
Áreas afines a la Ingeniería Industrial
Estadística Estadística Tabla de frecuencias y gráficos estadísti- Organizar, resumir y presentar datos.
descriptiva cos. Medidas de tendencias central y de
dispersión.
Estadística • Distribución binomial. Control estadístico- utilizado en el control de procesos
inferencial • Distribución hipergeométrica. productivos de bienes y servicios.
• Distribución de poisson. Mercadeo.
• Distribución normal.
- Investigación de mercado.
• Distribución T.
- Combinación de productos y existencias.
• Aproximación binomial a normal.
• Teoría de regresión y correlación. - Publicidad.
Gerencia de Operaciones.

562
• Distribución muestral.
• Estimados y tamaño de muestra. - Pronósticos.
• Distribución Chi cuadrada. - Gestión de calidad Total (TQM).
• El análisis de varianza – ANOVA. - Minimización de costos.
• Prueba de hipótesis. - Eliminación de desperdicios.
• Pruebas no paramétricas. - Localización.
- Ruta crítica.
- Productividad.
- Simulación.
- Teorías de colas.
Finanzas
- Análisis financieros.
Economía.
- Análisis económicos.
- Impuestos y gastos públicos.
- Producción nacional.
- Inflación.
- Macroeconomía.
Sánchez Marchán, Nanci. (2014).Introducción a la Ingeniería Industrial, Material Instruccional de Apoyo. Caracas: UNA
Áreas afines a la Ingeniería
Industria
Ingeniería Análisis de Modelos físicos: representan la entidad • La asignación de los recursos de investigación y desarrollo
de decisiones estudiada en cuanto a su apariencia hasta para productos o procesos.
Sistemas cierto punto, en cuanto a sus funciones. • Aplicaciones en el diseño y la optimización de los progra-
mas de producción, sistemas de distribución, ubicación
Modelos analógicos: Muestra el compor- y distribución de instalaciones, planeación y el control de
tamiento real de la entidad estudiada pero proyectos.  
no tiene el mismo aspecto.
Modelos narrativos: Descripciones en • La selección de estrategias de marketing para el lanzamien-
lenguaje natural de las relaciones existen- to de nuevos productos.
tes entre las variables de un proceso o de • Las decisiones sobre la ampliación o construcción de nue-
un sistema. vas instalaciones.
Modelos gráficos: Describen partes o • Las determinaciones con respecto a reparar o sustituir
equipos.
pasos de una entidad o proceso mediante
una representación gráfica (diagrama de • Selección de opciones de automatización.
flujo). • Determinación del programa de satélites artificiales.
• Selección de sitios para el depósito definitivo de desperdi-
Modelos matemáticos: Se valen de cios radiactivos.
variables cuantitativas (fórmulas) para
representar las partes de un proceso o de
un sistema.

563
Sistemas de apoyo a la Toma de Decisiones: conjunto de programas y herramientas que permiten obtener de manera oportuna
la información que se requiere, durante el proceso de toma de decisiones. Estos sistemas son:
DSS. Sistema de aporte a la Toma de Decisiones: uso de herramientas computacionales.
GDSS. Sistema para la toma de decisión en grupo: participación en anonimato de un grupo de personas para llegar a un con-
senso.
EIS. Sistemas de información para ejecutivos: uso de recursos visuales y fácil interpretación para apoyar la toma de decisiones
de los altos ejecutivos.
EDSS. Sistemas expertos de soporte con inteligencia artificial* para la toma de decisiones: programas de software di-
señados para replicar el proceso de toma de decisiones de un experto humano, sobre una base de conocimiento de un campo
especifico especializado.
*Logran que la computadora efectúe tareas de una manera inteligente, es decir, se trabaja con sistemas que permitan
mejorar y simular las capacidades sensoriales y mecánicas del ser humano.
Sánchez Marchán, Nanci. (2014).Introducción a la Ingeniería Industrial, Material Instruccional de Apoyo. Caracas: UNA
Sánchez Marchán, Nanci. (2014).Introducción a la Ingeniería Industrial, Material Instruccional de Apoyo. Caracas: UNA

El uso de modelos ha sido creado, para interpretar y predecir las dinámicas y


controles en la toma de decisiones operacionales y gerenciales. Estos modelos
ayudan a los gerentes y administradores a realizar un mejor trabajo al momento
de anticipar hechos, y por lo tanto, un mejor manejo de la incertidumbre.

Tomar una decisión de manera individual, en muchos casos es una tarea difícil y es más
difícil aún cuando se ha de tomar en grupo, debido a que cada una de las personas que in-
tegran el grupo tiene criterios diferentes y hasta contradictorios unos con otros. Estos casos
son comunes en las empresas, las administraciones públicas, entes gubernamentales, etc.
Las decisiones que se deben tomar casi siempre se ven afectadas por intereses políticos,
personales, de grupo o de partido, y por ello no responden a los intereses del bienestar social
a los que deben estar destinados. El proceso de la toma de decisiones requiere ser llevado
a cabo de forma rápida y certera. Así lo demanda el mundo empresarial de hoy día, el cual
tiene un entorno socio-económico cambiante y globalizado.

La toma de decisiones representa una herramienta importante y un reto para los inge-
nieros industriales, así como para los profesionales en general; debido a que pueden realizar
aplicaciones efectivas que le permitan lograr una organización más competitiva nacional e
internacionalmente y servir mejor a los clientes.

564
Sánchez Marchán, Nanci. (2014).Introducción a la Ingeniería Industrial, Material Instruccional de Apoyo. Caracas: UNA

RESUMEN DE UNIDAD Nº 4

Esta unidad contempla las áreas afines a la Ingeniería Industrial, entre las cuales se
pueden destacar las siguientes:

Estadística

La palabra estadística procede del latín statisticum collegium (consejo de Estado) y de


su derivado italiano statista (hombre de Estado o político).

La estadística se divide en dos ramas:


•• La Estadística descriptiva, que se dedica a los métodos de recolec-
ción, descripción, visualización y resumen de datos originados a partir de
los fenómenos en estudio.
•• La Inferencia estadística, que se dedica a la generación de los mode-
los, inferencias y predicciones asociadas a los fenómenos en cuestión,
teniendo en cuenta lo aleatorio e incertidumbre en las observaciones

La estadística es fundamental para la Ingeniería Industrial, ya que contribuye tanto para


la administración financiera, como para la administración de operaciones, las ventas, el mar-
keting, las cobranzas, la logística y la gestión de personal entre otras áreas y actividades.

Ingeniería de Sistemas

La Ingeniería de Sistemas, es un modo de acercamiento interdisciplinario que permite


evaluar la estructura de la organización y de los subsistemas que lo integran, con el propósito
de implementar u optimizar los sistemas complejos.

Actualmente la Ingeniería de Sistemas está cambiando su orientación a un enfoque di-


rigido a los modelos, que describen el sistema según diferentes puntos de vista y utilizando
notaciones diversas.

Dentro de la Ingeniería Industrial también se presenta la elección de estrategias, diseño


e implantación de sistemas de control, desarrollo de controles administrativos y técnicas
de optimización, en atención a esto, se puede observar la influencia de la Ingeniería de
Sistemas en la Ingeniería Industrial en los siguientes sistemas de gestión: de operaciones
productivos, predicción y pronósticos, administración de inventarios, planificación, análisis
de decisión, entre otros.

Toma de Decisiones

La Teoría de la Decisión es un área interdisciplinaria de estudio, relacionada con


casi todos los participantes en ramas de la ciencia, Ingeniería, así como, las actividades

565
Sánchez Marchán, Nanci. (2014).Introducción a la Ingeniería Industrial, Material Instruccional de Apoyo. Caracas: UNA

sociales. Concierne a la forma y estudio del comportamiento de fenómenos psíquicos, de


aquellos que toman las decisiones (reales o ficticios), como también a las condiciones por las
que deben ser tomadas las decisiones óptimas.

Las etapas necesarias en la toma de decisión para dar solución a un problema son:

1. Identificación y diagnóstico del problema


2. Investigación y recopilación de información
3. Generación de soluciones alternativas
4. Evaluación de alternativas
5. Selección de la mejor alternativa
6. Implantación de la decisión

En la toma de decisiones se puede considerar un problema y llegar a una conclusión vá-


lida, significa que se han examinado todas las alternativas y que la elección ha sido correcta.

Seguidamente se presenta la autoevaluación de la unidad 4, en la que encontrará un


conjunto de ejercicios, cuyas respuestas aparecen al final del texto. También se puede ob-
servar una serie de ejercicios propuestos, con la finalidad de lograr que el estudiante se
ejercite lo suficiente, para la presentación de las pruebas.

AUTOEVALUACIÓN DE LA UNIDAD 4
Una vez estudiados los contenidos concernientes a las disciplinas afines a la
Ingeniería Industrial, es recomendable resolver los ejercicios correspondientes
a la autoevaluación, sobre la base de los ejemplos ilustrativos. Resueltos estos
ejercicios podrá comprobar sus resultados con las respuestas aportadas por la
autora al final del texto. En caso de no realizar los ejercicios correctamente, revise
de nuevo los contenidos e inténtelo una vez más hasta lograr su cometido. Para
cualquier duda consulte a su asesor, quien le podrá apoyar con la verificación de
las respuestas.

Estadística

Ejercicio Nº 1

¿En qué áreas se aplica la metodología estadística?

Ejercicio Nº 2

Explique cuál es la importancia de la estadística para la Ingeniería Industrial y sus dife-


rentes áreas y de ejemplos de su aplicación en esta disciplina.

566
Sánchez Marchán, Nanci. (2014).Introducción a la Ingeniería Industrial, Material Instruccional de Apoyo. Caracas: UNA

Ejercicio Nº 3
La Estadística proporciona los métodos que ayudan a resolver diferentes problemas,
para ello es necesario conocer los conceptos básicos que conforman los contenidos de esta
materia. Diga cuáles son estos contenidos:

Ingeniería de Sistemas
Ejercicio Nº 4
Existen una serie de metodologías de resolución de problemas asociados a Ingeniería
de Sistemas, que utilizan un importante grupo de técnicas, algoritmos y programas disponi-
bles, mencione algunas de ellas.

Ejercicio Nº 5
Una industria metalmecánica no ha podido producir los bienes de capital que se había
propuesto como meta, para satisfacer las demandas regionales ¿Qué tipos de problemas se
deben corregir en esta industria para lograr el cometido? Seleccione la opción apropiada al
planteamiento realizado.

1. Problemas de mejoramiento.
2. Problemas de corrección del sistema.
3. Problemas de creación de sistemas.
4. Problemas de magnitud.
5. Problemas de expansión.

Ejercicio Nº 6
¿Qué hace un Ingeniero de Sistemas que le permite apoyar a la Ingeniería Industrial en
sus diferentes áreas?

Toma de Decisiones
Ejercicio Nº 7
Describa cómo debe realizarse el proceso de implementación de la decisión.

Ejercicio Nº 8
Mencione las etapas de la toma de decisión para dar solución a un problema.

Ejercicio Nº 9

Según su punto de vista cómo y cuándo se debe decidir.

567
Sánchez Marchán, Nanci. (2014).Introducción a la Ingeniería Industrial, Material Instruccional de Apoyo. Caracas: UNA

Ejercicio Nº 10

La política de emisión de cheques para el pago de proveedores establece un límite de


dinero para el pago y no requiere de la autorización del jefe superior. Sobre la base de esta
afirmación seleccione el tipo de decisión que se corresponde con el escenario:

1. Decisión programada (involucran situaciones repetitivas hasta cierto punto, cotidia-


nas)
2. Decisión no programada (imprevistos que requieren una solución específica)
3. Decisión riesgosa (un grado mayor o menor de incertidumbre)

568
Sánchez Marchán, Nanci. (2014).Introducción a la Ingeniería Industrial, Material Instruccional de Apoyo. Caracas: UNA

Ejercicios Propuestos
Se presenta este grupo de ejercicios con la intención de proporcionar al estudiante
planteamientos de situaciones, que le permitan ejercitarse antes de la presenta-
ción de las pruebas. Estos ejercicios no tienen respuestas en este texto, para la
solución de éstos puede apoyarse en la lectura de otros textos o buscar informa-
ción en la web.

Estadística

Ejercicio Nº 1

Recorte de periódicos o revistas artículos referentes a información estadística. Lo podrá


reconocer por la presencia de gráficos de números, como promedios, porcentajes, entre
otros.
Analice los recortes obtenidos y escriba en su cuaderno las siguientes observaciones:
a) ¿Qué variables se estudian de la población?
b) ¿Qué características se reportan?
c) ¿Cómo cree que se obtuvo la información?
d) ¿Qué inferencia o conclusión se desprende del trabajo estadístico reportado?

Ejercicio Nº 2

En una empresa procesadora de café se aplicó una encuesta a 50 operarios, acerca


del número de tasas de café que ingieren en el día, obteniéndose los siguientes resultados:

Tazas de café consumidas en el día No. de operarios


0 4
1 5
2 6
3 10
4 8
5 7
6 6
7 4

Conteste en la línea algunas preguntas referentes a este ejemplo:

¿Cuántas observaciones se tienen?


¿Cuántos datos son del problema?
¿Cuántos elementos tiene la muestra?
¿Cuál sería la población?

Ejercicio Nº 3
¿Qué sucede si la información que se desea recabar en una investigación no se en-
cuentra en ninguna dependencia?

569
Sánchez Marchán, Nanci. (2014).Introducción a la Ingeniería Industrial, Material Instruccional de Apoyo. Caracas: UNA

Ingeniería de Sistemas

Ejercicio Nº 4
Encuentre la diferencia entre el enfoque mecánico-analítico y el enfoque de sistemas.

Ejercicio Nº 5
En la era de la comunicación e información se reconoce el valor que tienen estos dos
elementos para la industria, y se busca desarrollarlos y administrarlos como un recurso. Esta
era se caracteriza por:

Ejercicio Nº 6
Describa la inteligencia artificial y diga cuál es su importancia para la Ingeniería Indus-
trial.

Ejercicio Nº 7
Diga cuál es Influencia de la computadora en la Ingeniería Industrial y sus áreas (pro-
ducción, higiene y seguridad, calidad, entre otras) 

Toma de Decisiones

Ejercicio Nº 8
Las decisiones que se toman en el área de producción están relacionadas con un grupo
de aspectos. Describa algunos de ellos.

Ejercicio Nº 9
De qué forma se podría visualizar el proceso que conduce a tomar una decisión.

Ejercicio Nº 10
Describa algunos pasos para la identificación de alternativas en la toma de decisiones.

Ejercicio Nº 11
Sofía tiene un gran potencial en el área de Higiene y Seguridad Industrial, pero sus es-
tudios serán largos y difíciles. En este momento le están ofreciendo un empleo no muy bien
remunerado, pero que le servirá para pagar algunas deudas y comprar algunos enseres que
necesita y que se había privado desde hace mucho tiempo; sin embargo, ello le impediría
continuar sus estudios hasta dentro de tres años. En función de la visión, ¿qué consecuen-
cias traería a Sofía escoger una u otra opción?

570
Sánchez Marchán, Nanci. (2014).Introducción a la Ingeniería Industrial, Material Instruccional de Apoyo. Caracas: UNA

BIBLIOGRAFÍA
Acosta Flores J.J. (1989). Cómo mejorar su habilidad para tomar decisiones. México,
DEAC (Desarrollo Integral Empresarial y Consultoría.)
Allen Roy. (1987). Manual de Ingeniería y Organización Industrial. Barcelona Rever-
té.
Beekman, George. (1995). Computación & Informática. Una mirada a la tecnología
del mañana. España, Addison Wesley Iberoamericana.
Buffa Elwood S. Y Taubert William H. (1978). Sistemas de Producción e Iinventario.
México, Limusa.
Buffa, E. S. Y Sarin, R. K. (1992). Administración de la producción y de las operacio-
nes. México, Limusa.
Camacho, Luz Amanda. (1996). Teoría General de Sistemas. Santafé de Bogotá,
Unisur.
Chase, Richard. (2009). Administración de Operaciones, Producción y Cadena de
Suministros. México, Mc Graw Hill.
Chiavenato, Idalberto. (1992). Introducción a la Teoría General de la Administración.
México, McGraw-Hill.
Chiavenato, Idalberto. (1992). Administración de Recursos Humanos. México, Mc-
Graw-Hill.
Cantu Humberto. (2006). Desarrollo de una Cultura de Calidad. México, Mc Graw
Hill.
Cortés Díaz, José María. (2007). Seguridad de Higiene del Trabajo. Barcelona, Te-
bar.
Coss Bu. (2008). Análisis y Evaluación de Proyectos de Inversión. México Limusa
Erosa, Victoria. (2006). Proyectos de Inversión en Ingeniería. México, Limusa
Farrer, Francisco .(1995).Manual de ergonomía. Madrid, Mapfre.
Ficher, Laura. (2004). Mercadotecnia. México, Mc Graw Hill.
García del Junco, C. (2000). Fundamentos de Gestión Empresarial. Madrid, Pirámi-
de.
Grech, Pablo. (2000). Introducción a la Ingeniería Industrial. Bogotá, PrenticeHall.
Gass I., Saúl. (1985). Programación Lineal. México, CESSA
Gibson, J., Ivancevich, J. y Donelly, J. (2001). Las organizaciones: comportamiento,
estructura, procesos. México, Mc Graw Hill.
Hamel, G. (2000). Liderando la Revolución. Gestión 2000, Barcelona.
Handy, C. (1985). Understanding Organisations. Penguin Books, Londres.
Hak J. Eduard. (1988). Justo a Tiempo. México, Norma.
Hicks,PhilipE. (2001). Ingeniería Industrial y Administración. Barcelona, CECSA.
Ramírez, David. (2002). Contabilidad Administrativa.México, Mc Graw Hill
Robbins, S. (2004). Comportamiento organizacional. México, Pearson

571
Sánchez Marchán, Nanci. (2014).Introducción a la Ingeniería Industrial, Material Instruccional de Apoyo. Caracas: UNA

Montgomery Douglas C., RUNGER George C. (1996). Probabilidad y Estadística


Aplicadas a la Ingeniería. México, McGraw Hill.
Mintzberg, H. (1986). La alta dirección: mitos y realidades. Bogota, Harvard de la
Administración.
Niebel Benjamín. (2001). Métodos, Estándares y Diseño del Trabajo. México,Alfa
omega
Oficina Internacional del Trabajo. (2001). Introducción al Estudio del Trabajo. Méxi-
co, Limusa.
O’Guinn Thomas. (2007). Publicidad y Comunicación Integral Marca. México,
Thomson.
Perez Gorostegui, E. (2001). Introducción a la Administración de Empresas. Madrid,
Centro de Estudios Ramón Areces S.A.
Taha H. (1995). Investigación de Operaciones. Alfaomega, Cataluña.
Krick, Edward. (1978). Introducción a la Ingeniería Industrial. Limusa, México
Vaughn, Richard. (1978). Introducción a la Ingeniería Industrial. Reverte.
Schmidt J.W. y Miller David. (1992). Ingeniería Industrial e Investigación de Opera-
ciones. México, Limusa.
Meredith, JackR. (2000). Administración de Operaciones. México, Limusa.
Salvendy, Gavriel. (1991). Manual de Ingeniería Industrial. México, Limusa.

572
Sánchez Marchán, Nanci. (2014).Introducción a la Ingeniería Industrial, Material Instruccional de Apoyo. Caracas: UNA

RESPUESTA A LOS EJERCICIOS DE AUTOEVALUACIÓN DEL MÓDULO 1,


LA UNIDAD Nº 1, UNIDADES 1, 2, 3 Y 4
En este conjunto de argumentos el estudiante podrá encontrar las respuestas a los
ejercicios de autoevaluación, correspondientes a los diferentes módulos que conforman este
texto.

UNIDAD Nº 1

Respuesta Nº 1

Frederick Winslow Taylor es considerado el padre de la Ingeniería Industrial, desarrolla


un concepto basado en la idea de tarea. Propone que la gerencia realice un plan de trabajo
para cada uno de sus empleados, en el que se escriba en detalle la tarea correspondiente
y se le especifique al operario las herramientas que deberá utilizar en la realización de la
misma, así como el tiempo determinado para cada actividad.

Estos conceptos dieron origen a lo que se conoce como la fórmula de Taylor para máxi-
mo rendimiento y consiste en:
•• Definir la tarea.
•• Definir el tiempo.
•• Definir el método.

Determinó un método óptimo para la realización de una tarea, es decir su manejo, rit-
mo, períodos de trabajo y descanso óptimos. Esta situación se denomina ahora diseño del
trabajo o estudio de métodos. Fue pionero en lo que se conoce hoy como medición del tra-
bajo, lo que llevó a estándares de tiempo que constituyen la base para el control de costos
laborales, así como, un insumo necesario para programar y fijar precios de las actividades
en una industria.

Respuesta Nº 2.

Se proponen en este caso, dos métodos que involucran los pasos a seguir para realizar
la tarea presentada:

Método A

Tiempo 1 2 3 2
(hrs)
Actividad Dirigirse hasta Seleccionar la biblio- Buscar la informa- Tomar los apuntes
la biblioteca. gráfica adecuada. ción pertinente. necesarios.

573
Sánchez Marchán, Nanci. (2014).Introducción a la Ingeniería Industrial, Material Instruccional de Apoyo. Caracas: UNA

Método B

Tiempo
1 1/2 2 1/2
(hrs)
Trasladarse hasta Iniciar navega- explorar las páginas con Grabar o imprimir
Actividad
un Cibercafé. ción en Internet. la información requerida. la información.

Una vez desarrollados los métodos, se hacen uso los conceptos que dieron origen a la
fórmula de Taylor, para seleccionar el método más sencillo y que involucre menor tiempo de
realización.
En este caso el método a seleccionar es el B, por ser el que se adapta mejor a la fórmula
de Taylor.

Respuesta Nº 3
Desarrolló estudios sobre economía de movimientos (b)

Respuesta Nº 4
Las ideas aportadas por Henry Ford se basaron en la fabricación en proceso continuo,
en el que el artículo a fabricar recorre un itinerario establecido dentro de la planta. Los ope-
rarios a lo largo del proceso, van ejecutando una tarea única y específica, sobre todos los ar-
tículos que van transitando por su lugar de trabajo. Al final del recorrido se tiene un producto
terminado con todos sus componentes incorporados.

Respuesta Nº 5
Antes de la revolución industrial no era práctica usual el uso de partes intercambiables,
puesto que no existían patrones (estándares) de producción. Por lo que Eli Whitney (1765-
1825) introduce los conceptos sobre partes intercambiables para el momento en el cual se
desarrollan las fábricas textiles, recibiendo éstas un fuerte impulso que aceleró su desarrollo.

Respuesta Nº 6
De acuerdo con la complejidad de los problemas en las organizaciones, es necesario
constituir equipos multidisciplinarios, en los que los especialistas interactúen aportando sus
conocimientos, para generar una solución eficaz y eficiente en los proyectos.

Respuesta Nº 7
La diferencia entre arte y técnica radica para el primer caso, en el hecho de que las co-
sas pueden hacerse de diversas maneras, ejemplo de ello puede ser elaborar una escultura.
Sin embargo la técnica exige orden, seguir reglas para poder construir algo, por ejemplo de
no seguir determinadas reglas en la construcción de un edificio, el mismo puede derrumbar-
se antes de finalizar su construcción.

574
Sánchez Marchán, Nanci. (2014).Introducción a la Ingeniería Industrial, Material Instruccional de Apoyo. Caracas: UNA

Respuesta Nº 8
La relación que existe entre ciencia e Ingeniería Industrial, se evidencia debido a que la
ciencia es la invariable búsqueda del conocimiento exacto y razonado en un todo y/o partes,
el cual se aplica en ingeniería de modo científico con fines concienzudos para obtener los re-
sultados deseados. Es por ello que la base de la ingeniería es la ciencia y en ella se inspiran
los ingenieros para realizar o llevar a cabo la investigación científica.

Respuesta Nº 9
La respuesta correcta es la letra (d) Todas las anteriores. Tanto el Ingeniero Industrial
como el TSU en Higiene y Seguridad Industrial pueden asumir todas las funciones plantea-
das en la pregunta, dada su formación sólida en el área de higiene y seguridad, así como su
formación humanística y sentido de responsabilidad para con una empresa y la sociedad en
su conjunto.

Respuesta Nº 10
La letra correspondiente a cada función señalada es:
•• Realiza el control de inventarios en una fábrica de repuestos para (a)
•• Diseña las estructuras de una planta industrial (e)
•• Determina los requerimientos de energía de maquinarias y equipos ( c)

UNIDAD Nº 2

Ejercicios del área operativa

La empresa
Respuesta Nº 1
1.1. Cuántos trabajadores pueden tener una empresa mediana.
a) 50 a 250
b) 100 a 300
c) 300 a 200
d) 80 a 450
e) 600 a 1500

1.2. Se define como la unidad económica o social en la cual se reúne capital, etc. para
lograr un objetivo común.
a) La Empresa
b) Costos
c) Empresa chica
d) Empresa grande
e) Contabilidad

575
Sánchez Marchán, Nanci. (2014).Introducción a la Ingeniería Industrial, Material Instruccional de Apoyo. Caracas: UNA

1.3. Esta empresa puede tener más de 20 trabajadores:


a) Grande
b) Mediana
c) Pequeña
d) Microempresa
e) Macroempresa

1.4. La empresa_____ puede tener ________ trabajadores


a) Pequeña-300
b) Grande-500
c) 25-Pequeña
d) 1500

1.5. A qué sociedad pertenece las siguientes siglas: SRC


a) Sociedad Anónima.
b) Sociedad de Responsabilidad Limitad.
c) Sociedad Comanditaria.
d) Sociedad Anónima Laboral.
e) Ninguna de las anteriores.

Respuesta Nº 2

La empresa automotriz, aplicó estrategias operacionales tales como: implementación


de planes, políticas de calidad en el proceso, reglas y procedimientos, para disminuir los
costos en cuanto a recursos humanos y materiales, mejora de los procedimientos en los
procesos administrativos, disminuyendo la forma burocrática que generan estos trámites.
Se puede decir entonces que se pudieron alcanzar los resultados, logrando la eficiencia
deseada. También es de notar el alcance de los objetivos, consiguiendo así la eficiencia en
el proceso.

Por todo lo expuesto la empresa logró complementar la eficacia y la eficiencia y así dar
prioridad a las tareas que se deben realizar asegurándose que lo que se realizó, contiene
una adecuada finalidad y muestra la calidad deseada en los resultados.

Respuesta Nº 3.

Los pasos a seguir para la toma de decisiones en la empresa son:

1. Identificación del problema. Consiste en identificar la discrepancia entre el resulta-


do deseado y el real.

2. Análisis del problema. Investigar utilizando métodos que ayuden a identificar y do-
cumentar las posibles causas del problema.

576
Sánchez Marchán, Nanci. (2014).Introducción a la Ingeniería Industrial, Material Instruccional de Apoyo. Caracas: UNA

3. Generación de alternativas. Este paso debe estar encaminado en la oportunidad


de solucionar el problema utilizando la creatividad, utilizando la razón y estableciendo crite-
rios de selección. Mientras más alternativas se conciban mayor será la oportunidad de tener
alguna buena solución.

4. Desarrollo de criterios de selección de alternativas. Los criterios ayudan a definir


lo que se precisa independientemente de la preferencia.

5. Selección de la decisión. Determinados los criterios se procede a escoger la alter-


nativa más viable para solucionar el problema. También se le puede añadir un valor determi-
nado a cada criterio si se quiere hacer la selección de forma cuantitativa.

6. Implementación de la decisión. Una vez seleccionada la mejor alternativa hay que


ponerla en acción.

7. Evaluación de resultados. Por último se evalúan los resultados, para determinar


si se solucionó el problema o se crearon otros adicionales. Este es un paso tan importante
como identificar el problema

Respuesta Nº 4

Organigrama Horizontal Organigrama circular

Respuesta Nº 5

Estructura Jerárquica o Lineal Estructura por


Departamentalización por Producto

Farmacéuticos

Producción Químicos

Colorantes

577
Sánchez Marchán, Nanci. (2014).Introducción a la Ingeniería Industrial, Material Instruccional de Apoyo. Caracas: UNA

PRODUCCIÓN
Respuesta Nº 6
La empresa NANEL S.A., se dedica a la extracción de mineral de hierro, en el territorio
guayanés. Seleccione de acuerdo a la afinidad, el Sector de la producción, donde se encuen-
tra NANEL:
a) Primario
b) Secundario
c) Terciario

Respuesta Nº 7
a) Investigación de mercado, método DELPHI o ambos.
b) Investigación de mercado.
c) Series de tiempo.

Respuesta Nº 8
Utilizando promedio móvil.
MT = 1/N (dt-N+1 +dt –N+2 + …dt)
M13= d8+ d9+ d10+ d11+ d12 / 5 = 288.4 = 288
F13= F14= F20= 288; por ser un modelo constante

INVENTARIOS

Respuesta Nº 9

Entre los principales objetivos requeridos para alcanzar las metas del inventario se en-
cuentran:
•• Disminuir el costo de compra.
•• Reducir los gastos de intereses.
•• Aumentar la disponibilidad de fondos internos.
•• Lograr menores costos de operación.
•• Disminuir el costo unitario de producción.
•• Aumentar la confiabilidad en la entrega por parte de la producción.
•• Mejorar servicio a los clientes en el suministro de productos

Respuesta Nº 10

Tipos de inventario:
•• Materias primas: La compañía adquiere las materias primas sin proce-
sar, las cuales necesitan una mayor elaboración o transformación para
convertirlas en parte de un producto final (petróleo crudo, madera, sus-
tancia químicas, etc.)

578
Sánchez Marchán, Nanci. (2014).Introducción a la Ingeniería Industrial, Material Instruccional de Apoyo. Caracas: UNA

•• El trabajo en proceso: Incluye todos los materiales de producción sobre


los que la compañía ha realizado operaciones de fabricación, procesa-
miento o transformación, pero que no están en la forma terminada. Están
en proceso de transformación y el inventario es la protección contra la
variabilidad en el proceso de producción. Componentes que se almace-
nan para usarse en el ensamble final de los productos o pueden también
venderse como refacciones.
•• Los productos terminados: Cubre todos los productos o bienes termi-
nados, producidos y almacenados, que esperan venderse o enviarse a
los clientes finales.
•• Piezas de servicio: Las partes que suelen llamarse refacciones, piezas
de repuesto, se usan para mantener el producto o el equipo que la com-
pañía vende o atiende. Este inventario puede almacenarse en el sitio de
producción de las piezas terminadas; o distribuirse y almacenarse en los
distribuidores, los lugares de servicio o los sitios que están estrechamen-
te ligados a la reparación o al mantenimiento del producto final.
•• Distribución: Los bienes terminados, así como también las piezas de
servicio, se ubican, se almacenan o se transportan en los depósitos de
toda red de distribución y pueden abarcar los que posee la compañía
y se ubican lejos de la planta central de fabricación, en las sucursales,
bodegas etc.
•• Suministros: Los artículos que se usan para mantener las operaciones,
ya sea en la fábrica o en la oficina, pero que no llegan a formar parte del
producto terminado; son artículos para el mantenimiento de la planta, la
reparación de máquinas, el consumo de la planta o la producción, artícu-
los de oficina, etc.

PRODUCTIVIDAD

Respuesta Nº 11

Indicadores de Productividad Empresa A


Productividad horas- máquina 2.000.000 cuadernos.
Unidades Producidas/Nº de Horas-Máquina trabaja- 10 x 190 horas trabajadas
das en el mes 1053= cuadernos por h-m

Respuesta Nº 12

Indicadores de Productividad Empresa A


Productividad horas- hombre 1.000.000 unidades.
Unidades Producidas / Nº de Horas-Hombre trabajadas 100 x 155 horas trabajadas
en el mes = 65 unidades por h-h

579
Sánchez Marchán, Nanci. (2014).Introducción a la Ingeniería Industrial, Material Instruccional de Apoyo. Caracas: UNA

Respuesta Nº 13

La productividad de la planta A no es igual a la de las plantas B y C, ya que la misma


genera mayores costos en la utilización del recurso mano de obra, para lograr alcanzar su
producción.

ESTUDIO DEL TRABAJO: MÉTODOS, MOVIMIENTOS Y TIEMPOS

Respuesta Nº 14

Símbolo Significado Definición

Se efectúa alguna actividad sobre el objeto, se


Operación
modifica el objeto o sus cualidades originales.

Se realiza cada vez que se mueve un objeto de


Transporte
un lugar a otro.

Ocurre cuando se protege material u objetos ter-


minados de las condiciones climáticas o cuando
Almacenamiento
se acumulan en un lugar determinado para ser
transportados.

Se realiza cuando se mide o comprueba alguna


Inspección de las características del artículo producido o la
cantidad producida del mismo.

Cuando existe un retraso en un proceso, cuando


Demora la actividad siguiente no puede ser ejecutada in-
mediatamente después de terminada la anterior.

580
Sánchez Marchán, Nanci. (2014).Introducción a la Ingeniería Industrial, Material Instruccional de Apoyo. Caracas: UNA

Respuesta Nº 15
Diagrama bimanual actual de la operación cortar tubo.

Diagrama Bimanual
Proceso: Cortar Tubo Fecha: mayo de 2014
Elaborado por: Miguel Fernández N° de Diagrama: 40
Descripción: Trabajo completamente manual Hoja: 1 de 1
Método: Actual ( X ) Propuesto ( )
   
Descripción de las operación   Descripción de las  

Inspección  

Transporte  

Demora  

o  
Sostenimient

operación  

Inspección  

Transporte  

Demora  
o  
Sostenimient
Actividades de la Actividades
mano izquierda de la Mano derecha  

Sostiene tubo             Recoge lima

Llevar tubo hasta                   Sostiene lima


plantilla

            Lleva lima hasta tubo


Meter tubo en
plantilla
                  Sostiene lima
Empujar tubo hasta el
final
Sostiene tubo             Sostiene lima

            Muesca tubo con lima


Retira un poco el tubo

Gira tubo 120-180º                     Sostiene lima    


 
                    Acerca lima a tubo    
Empuja tubo hasta el
final
                    Muesca tubo    
Sostiene tubo
                    Coloca lima en mesa    
Retira tubo
                    Se acerca a tubo    
Pasa tubo a mano
derecha
                    Dobla tubo    
Dobla tubo hasta
partir
                    Suelta tubo cortado    
Sostiene tubo
Corre a otra parte de                     Va hasta lima    
tubo
Cuadro Resumen   Área de Trabajo  
actividad M.  Actual   M.  Propuesto    
Izq Der   Izq   Der    
8 5      
         
  2 5        
        Colocación para
Plantilla    
  4 4     Marcado   Tubo de  vidrio    
Total   14 14      
 

581
Sánchez Marchán, Nanci. (2014).Introducción a la Ingeniería Industrial, Material Instruccional de Apoyo. Caracas: UNA

Diagrama bimanual propuesto de la operación cortar tubo

Para elaborar este diagrama se deben eliminar los Therblings ineficientes, tales como:
buscar, posicionar, seleccionar, planear, sostener, inspeccionar, etc.

Diagrama Bimanual
Proceso: Cortar Tubo Fecha: mayo de 2014
Elaborado por: Miguel Fernández N° de Diagrama: 40
Descripción: Trabajo completamente manual Hoja: 1 de 1
Método: Actual ( X ) Propuesto ( )
   
Descripción de las Descripción de las  
operación  

Inspección  

Transporte  

Demora  

Sostenimiento  

operación  

Inspección  

Transporte  

Demora  

Sostenimiento  
Actividades de la Actividades de la Mano
mano izquierda derecha

Meter tubo hasta tope             Sostener lima

Hacer girar tubo                   Muesca tubo con lima

            Golpea tubo con lima,


Sostener tubo trozo cae en caja

                 
           
           
Cuadro Resumen   Área de Trabajo  
actividad M.  Actual   M.  Propuesto    
Izq Der   Izq   Der    
2 2  
Tope  
       
       
      Tubo de  v idrio   Posición para  
1 Muescar  
   
Total   2 3  
 

CALIDAD EN LA EMPRESA

Respuesta Nº 16

La calidad es importante en el ámbito de la empresa, porque puede afectarla en cuatro


aspectos fundamentales: costos, participación de mercado, reputación de la empresa, segu-

582
Sánchez Marchán, Nanci. (2014).Introducción a la Ingeniería Industrial, Material Instruccional de Apoyo. Caracas: UNA

ridad de los productos e implicaciones internacionales; por ello es preciso que los gerentes
de la misma adopten herramientas y métodos útiles que le permitan responder a la obten-
ción de productos de óptima calidad y de servicios eficientes, para competir internacional-
mente en la aldea global.

Respuesta Nº 17

La figura muestra de una manera gráfica las relaciones entre la perspectiva global y las
perspectivas interna y externa. La perspectiva interna hace énfasis en la eficiencia, lo impor-
tante es elaborar un producto o prestar un servicio con una atención especial a los costos y
a la productividad, respetando lo pactado con el cliente de forma tácita y explícita.

La perspectiva externa asume que en los mercados con un alto grado de rivalidad entre
competidores, fuerte ritmo de cambio tecnológico y cambios en los gustos de los consumido-
res, es necesario centrarse en el cliente, que es quien va a indicar qué productos y servicios
necesitan, con qué características y prestaciones, y a qué precio. Esta perspectiva deja en
un segundo plano la eficiencia para poner énfasis en la eficacia.

La perspectiva global abarca las dos anteriores, demostrando que la empresa excelente
es aquella que satisface las necesidades de todos los grupos de influencia relacionados con
ella y además con criterios de eficacia y eficiencia.

Respuesta Nº 18

Datos
M=5 Σ X 514,8
X= = = 20, 59
N = 25 N 25
Σ X = 514, 8 ΣR 120
R= = = 4,8
Σ R = 120 N 25

UcLx = X + A 2 R = 20, 59 + 0, 58 (4, 8) = 23, 38

LcLx = X - A 2 R = 20, 59 – 0, 58 (4, 8) = 17, 80

UcLx = D4 R = 2, 11 x 4, 8 = 10, 13

LcLr = D3 R = 04, 8 = 0.

583
Sánchez Marchán, Nanci. (2014).Introducción a la Ingeniería Industrial, Material Instruccional de Apoyo. Caracas: UNA

Respuesta Nº 19

SE INSCRIBE ESTUDIA SE ENTREVISTA


COMO CON ASESOR FORMALIZA
OFERTA
ESTUDIANTE ACADÉMICO INSCRIPCIÓN
ACADÉMICA
REGULAR

ESTUDIA ASISTE A
REALIZA
UNIDADES DE ASESORIAS
AUTOEVALUACIÓN
MÓDULOS PREVISTAS

PRESENTA
PRUEBAS

Respuesta Nº 20

Enfoque Orientación del enfoque


Inspección Productos
Control Procesos
Aseguramiento de la calidad Sistemas
Gestión de la calidad total Personas

HIGIENE Y SEGURIDAD INDUSTRIAL

Respuesta Nº 21

Las medidas mínimas que debe exigir Marcos Salazar son las siguientes:
•• Que se coloque una alarma que indique cuándo existe un derrame o fuga
del fluido venenoso. Ésta puede ser una pintura que cambie de color o
un dispositivo que detecte la presencia del fluido en la atmósfera y lo
transmita a un lugar donde se perciba de inmediato (timbre, sirena, luz
roja, entre otros).
•• Se debe obligar al personal que efectúa reparación utilizar trajes especia-
les y un suministro de aire para no respirar el aire atmosférico que rodea
el lugar de trabajo.
•• Se toman medidas para lavar o destruir, apropiadamente el equipo y tra-
jes empleados.
•• Se evita el acceso de personas ajenas a la reparación que se está efec-
tuando.

584
Sánchez Marchán, Nanci. (2014).Introducción a la Ingeniería Industrial, Material Instruccional de Apoyo. Caracas: UNA

Respuesta Nº 22

Vías de entrada Agentes contaminantes


Auditiva Ruido alto y bajo, golpeteo, vibraciones.
Respiratoria Polvos, gases, humos, vapores, neblinas.
Cutánea Agentes que irritan la piel
Visual Poca iluminación, radiaciones, temperatura y brillantes.
Digestiva Cosas que se toman o beben antihigiénicas.

Respuesta Nº 23

El enunciado relacionado con la higiene y seguridad industrial falso, es el que corres-


ponde a la letra (a)

ERGONOMÍA

Respuesta Nº 24

Si se hace uso de la computadora con frecuencia o por períodos prolongados, es ade-


cuado tomar las siguientes precauciones:

• Descansos frecuentes en la jornada


Pararse y alejarse de la computadora a intervalos frecuentes durante la sesión de traba-
jo ayuda a disminuir los riesgos.
• Organizarse:
Procurar organizar las diversas actividades diarias para satisfacer sus necesidades dia-
rias de descanso, trabajo, alimentación y relaciones sociales. Esto le permitirá tener un
equilibrio mental, físico y emocional.
• Descansar los ojos:
La vista se cansa por mirar fijamente a una distancia igual durante mucho tiempo. Aun si
no es posible levantarse, mirar alrededor y enfocar diferentes objetos a diferentes distan-
cias puede ayudar. Cerrar los ojos por aproximadamente un minuto permite relajarlos.
• Ponerse en movimiento:
Durante los lapsos de descanso, hacer ejercicios suaves, como girar el torso o rotar el
cuello para estimular la circulación y relajar la espalda. También es recomendable mover
los pies con giros en la articulación del tobillo y la rodilla.
• Ejercitarse:
El sedentarismo acentúa las lesiones o la propensión a las mismas, así como los trastor-
nos circulatorios y respiratorios. Se recomienda realizar ejercicios al menos tres veces a
la semana.
• Cuidar la postura:

585
Sánchez Marchán, Nanci. (2014).Introducción a la Ingeniería Industrial, Material Instruccional de Apoyo. Caracas: UNA

Debe adoptarse posiciones saludables, sin forzarse ni tensarse, esto hace más cómodo
el trabajo y evita daños y lesiones que se evidencian con el tiempo. Se deben mantener
los pies planos en el piso al frente. Las piernas no deben estar cruzadas, ni en frente ni
por debajo del usuario, por largos periodos
• Ser delicado al teclear:
Evitar golpear las teclas o apretar el ratón en forma muy rígida. No se requiere imprimir
mucha presión para trabajar.
• Asegurarse de que el centro de trabajo es adecuado ergonómicamente:
Adecuar los muebles, equipos y elementos de trabajo a las necesidades del cuerpo. La
silla y el escritorio deberían ser ajustables. La silla debe ofrecer soporte a la espalda
baja y descansos para los brazos.
• Mantener las muñecas derechas:
Las manos deben estar en línea con los brazos al teclear. Si se mantienen las muñecas
inclinadas en alguna dirección, puede ocasionar fatiga en los músculos e incrementar el
riesgo de lesiones de túnel carpiano o tendón.
• Mantener las muñecas levantadas:
Descansar las muñecas al teclear impide el movimiento de los antebrazos para posicio-
nar las manos e impone tensión en manos y dedos. El soporte para las muñecas se usa
cuando se descansan las manos, no mientras se escribe.

Respuesta Nº 25

Si ambos puestos de trabajo están diseñados adecuadamente, el trabajador puede


mantener una postura corporal correcta y cómoda, lo cual es importante porque una postura
laboral incomoda puede ocasionar múltiples problemas, se puede visualizar también en am-
bos casos que se toma en cuenta al trabajador y la tarea que va a realizar a fin de que ésta
se lleve a cabo cómodamente, sin problemas y eficientemente.

MANTENIMIENTO INDUSTRIAL DE MAQUINARIA Y EQUIPO

Respuesta Nº 26

Cuando los bombillos en las casas fallan, lo que se hace es simplemente reponerlos,
en dicho caso se aplica mantenimiento correctivo. En este caso, la falla no produce grandes
molestias y la reposición requiere de corto tiempo y además, los proveedores de bombillos
son accesibles, fácil de ubicar.

Respuesta Nº 27

A este tipo de mantenimiento se le denomina correctivo contingente. Las labores que


en este caso deben realizarse, tienen por objeto la recuperación inmediata de la calidad del
servicio. Es decir que ésta se coloque dentro de los límites esperados por medio de arreglos
provisionales.

586
Sánchez Marchán, Nanci. (2014).Introducción a la Ingeniería Industrial, Material Instruccional de Apoyo. Caracas: UNA

Respuesta Nº 28

Los costos involucrados con la falla de un equipo son los siguientes:


•• Pérdidas por tiempo de inactividad, la planta debe hacer frente a sus gas-
tos mientras deja de percibir dinero por un producto que no puede vender
porque no lo esta fabricando.
•• Pérdidas por materiales dañados por la falta de las máquinas o equipos.
•• El costo de los materiales y las piezas necesarias para la reparación.
•• El costo de la mano de obra necesaria para realizar la reparación.

Para disminuir o evitar este tipo de accidente se hace necesario realizar el mantenimien-
to preventivo a las máquinas con regularidad y de esta manera se puede garantizar el buen
funcionamiento de las mismas y por ende de la empresa.

PREPARACIÓN DE PROYECTOS DE INVERSIÓN

Respuesta Nº 29

Conclusión

Los mercados formales de trabajo no absorbieron la mano de obra que la crisis eco-
nómica y la migración del campo a la ciudad generaron; no se respondió plenamente a la
demanda de nuevos servicios. La relativa facilidad de entrar al mercado informal explica
la mayor expansión de éste. La dificultad de entrar a los mercados formales no es casual,
sino que está dirigida por las políticas gubernamentales del país. Los ciudadanos parecen
estar siguiendo las señales del mercado de manera optimista y ocupándose de las posibles
necesidades de la sociedad, en cuanto al abastecimiento de algunos productos. Mejorar las
políticas en el sector formal de la economía disminuiría esta problemática.

Respuesta Nº 30

Dependerá del tipo de proyecto, considerando si existen alternativas o bien si hay más
de un proyecto, los criterios a considerar son:
•• Período de recuperación: PR < vida útil
•• Tasa de rendimiento contable: TRC > TMR
•• Valor actual neto: VAN > 0
•• Tasa interna de rendimiento: TIR > TMR

587
Sánchez Marchán, Nanci. (2014).Introducción a la Ingeniería Industrial, Material Instruccional de Apoyo. Caracas: UNA

LOCALIZACIÓN Y DISTRIBUCIÓN EN PLANTAS

Respuesta Nº 31

Almacén de Materia Prima Almacén de Productos


Terminados

Área de Amasado Área de Atención al

Baños
Cortadora

Lavadero
Mesón de Amasar 1
Cliente

Mesón de Amasar 2

Comedor
Baños

Horno

Respuesta Nº 32

Los factores básicos que gobiernan la evaluación para la localización de fábricas son:
•• Localización de materiales de producción.
•• Mano de obra.
•• Terrenos disponibles.
•• Combustible industrial.
•• Facilidad de transporte.
•• Mercado.
•• Facilidades de distribución.
•• Energía.
•• Agua.
•• Condiciones de vida.
•• Leyes y reglamentos.
•• Estructura tributaria.
•• Clima.

588
Sánchez Marchán, Nanci. (2014).Introducción a la Ingeniería Industrial, Material Instruccional de Apoyo. Caracas: UNA

INVESTIGACIÓN DE OPERACIONES

Respuesta N° 33

a) Preguntas alusivas a la situación:


•• ¿Cuál es la ganancia por lote de cada tipo de producto?
•• ¿De cuántas horas por semana dispone cada planta para la elaboración
de un lote de cada tipo de producto?
•• ¿Cuál es el requerimiento en horas para producir 1 lote de cada tipo de
producto en cada una de las plantas?

b) Definición de variables:

X1: Número de lotes del producto 1: fabricados por semana.

X2: Número de lotes del producto 2, fabricados por semana.

c) Función Objetivo:

Maximizar la ganancia semanal total (en miles de bolívares) por la producción de los 2
productos

Z = 3x Z = 3x1 + 5x + 5x2

Respuesta N° 34

Datos:
Z = Costos.
X1 = Cantidad de litros de agua abastecidos por el depósito local:
C1 = 300 Bs / millón de litros.
X2 = Cantidad de litros de agua abastecidos por tubería (bombeo):
C2 = 500 Bs/ millón de litros.

a) Función Objetivo: Min Z = 300X1 + 500X2.

b) Establecimiento de restricciones

1) Requerimiento mínimo de abastecimiento de 10 millones de litros de agua diarios.

2) Capacidad máxima del depósito local de 5 millones de litros de agua diarios.

3) Capacidad máxima de tubería de 10 millones de litro de agua diarios.

589
Sánchez Marchán, Nanci. (2014).Introducción a la Ingeniería Industrial, Material Instruccional de Apoyo. Caracas: UNA

Respuesta N° 35

Datos:

Z es la ganancia total o la suma de las ganancias de cada opción


Las inversiones tienen la restricción de ser múltiplos de Bs 1000.
Las fi(x) = Etapa i, (i = 1, 2,3) y, X es la cantidad de dinero invertida en cada opción.

El modelo con el cual se puede determinar el dinero que deberá invertirse en


cada opción para maximizar las ganancias es el siguiente:

Maximizar z = f1(x1) + f2(x2) + f3(x3)


La persona sólo posee Bs 4000 para invertir:
Las condiciones son:
X1 +x2 + x3 4000

Con todas las variables enteras y no negativas.

UNIDAD 3

Administración

Respuesta Nº 1

Los administradores gerentes se clasifican en:

Gerentes de primera línea. Son las personas que ocupan el nivel más bajo de una
organización. Los gerentes de primera línea dirigen a empleados que no son gerentes; no
supervisan a otros gerentes.

Gerentes medio. El término gerencia media incluye varios niveles de una organiza-
ción. Los gerentes de niveles medios dirigen las actividades de gerentes de niveles más
bajos y en ocasiones también las de empleados de operaciones.

Alta gerencia. La alta gerencia está compuesta por una cantidad de personas compa-
rativamente pequeña y es la responsable de administrar toda la organización. Estas perso-
nas reciben el nombre de ejecutivos. Establecen las políticas de las operaciones y dirigen la
interacción de la organización con su entorno. Algunos cargos típicos de la alta gerencia son:
director general ejecutivo, director y subdirector.

590
Sánchez Marchán, Nanci. (2014).Introducción a la Ingeniería Industrial, Material Instruccional de Apoyo. Caracas: UNA

Los factores de motivación que pueden generarse de los principio planteados son:

Principio Factor de motivación


Suprimir algunos controles mante- Responsabilidad y rendimiento personal.
niendo la responsabilidad del trabajo.
Aumentar la responsabilidad el traba- Responsabilidad y reconocimiento.
jador respecto a su trabajo.
Delegar un área de trabajo cerrada en Responsabilidad, rendimiento y reconoci-
sí misma. miento
Conceder mayor autoridad y mayor Responsabilidad, rendimiento y reconoci-
libertad. miento
Informar al trabajador sobre los infor- Reconocimiento del grupo de trabajo
mes mensuales.
Reparto de tareas nuevas y más difí- Aprendizaje y crecimiento.
ciles.
Reparto de tareas especiales, que Responsabilidad, crecimiento, rendimiento y
permitan al trabajador mejorar profe- éxito.
sionalmente.

Respuesta Nº 2

a) Si se tiene en cuenta las funciones y capacidades de la gestión, Collis cometió un


error al no dar la debida importancia a las relaciones con los clientes, la cual es
una de las responsabilidades primarias de los gerentes. Su sustituto, Grossman,
tuvo un desempeño excelente en las relaciones con el cliente y en proporcionar
asesoría técnica a los empleados. Sin embargo fracasó en la planeación a largo
plazo y perdió la oportunidad con el nuevo avión Caza.

b) Todo gerente debe tratar constantemente de balancear su tiempo entre todas


las actividades a realizar y requiere adoptar un paso rápido para manejar las
múltiples demandas y el gran volumen de información, para lograr desempeñar
eficiente y eficazmente sus roles.

Respuesta Nº 3

Se puede considerar la planificación de plantillas como el conjunto de medidas que


basadas en el estudio de antecedentes relacionados con el personal y en los programas y
previsiones de la empresa, tienden a determinar, desde el punto de vista individual y general,
las necesidades humanas de una industria en un plazo determinado, cuantitativa y cualitati-
vamente, así como su coste.

La base de unas adecuadas relaciones humanas en la empresa es el conocimiento del


personal y tal conocimiento debe comenzar desde el mismo momento en que el personal
entra a formar parte de la plantilla de la empresa; es más, antes de admitir al personal, debe
ser conocido.

591
Sánchez Marchán, Nanci. (2014).Introducción a la Ingeniería Industrial, Material Instruccional de Apoyo. Caracas: UNA

Respuesta Nº 4

El cambio se puede gerenciar siempre que éste beneficie a los trabajadores, consumi-
dores y clientes de la empresa. El cambio puede llevarse a cabo en diferentes áreas: nego-
cio, tecnologías utilizadas, actitudes, administración, procesos y organización, entre otras.
Será positivo si todos están alineados a una misma estrategia, a la misión y visión de la em-
presa. En el camino, es muy probable que surjan algunos inconvenientes, pero los directivos
deben estar preparados para asumir y solventar los riesgos.

Las palabras clave en un proceso de cambio son: cambio, manejo del cambio, re-
sistencia al cambio, cambio organizacional, entre otras

Economía y finanzas

Respuesta N° 5

a) El punto de contacto entre el estado de situación financiera con el estado de resulta-


dos es la utilidad o pérdida obtenida del ejercicio.

b) Entre los estados financieros más conocidos se encuentra: el estado de costos de


producción, el estado de resultados y el estado de cambios en la situación financiera.

c) Porque es un gasto de operación, como los gastos indirectos y se refieren a los gas-
tos del periodo (mes, año).

Respuesta Nº 6

Solución

Ganancia = Ventas – Costos


Alternativa 1
Ventas = 25.000 unidades/año x 1.800 Bs/ unidad
Ventas = 45.000.000 Bs/ año
Costo Total = Costo de operación + Costo almacenamiento
Costo Total = 8.500.000 + 12.000 = 8.512.000 Bs/año
Ganancias = 45.000.000 – 8.512.000 = 36.488.000 Bs/año
Alternativa 2
Ventas = 28.000.000 unidades/año x 1.600 Bs/ unidad
Ventas = 44.800.000 Bs/año
Costo Total = 9.250.000 Bs/año + 12.500 = 9.262.500 Bs/año
Ganancias = 44.800.000 – 9.262.500 = 35.537.500 Bs/año
Se debe elegir la alternativa 1por proporcionar la misma mayores ganancias.

592
Sánchez Marchán, Nanci. (2014).Introducción a la Ingeniería Industrial, Material Instruccional de Apoyo. Caracas: UNA

Respuesta Nº 7

Solución:

a) La propuesta a la gerencia seria la de comprar por partes a ambos proveedores.


b) Proveedor (1): 700 kg/ día x 4000 Bs/kg = 2.800.000 Bs/día
Proveedor (2): 200 kg/ día x 5000 Bs/kg = 1.000.000 Bs/día
--------------
Costo mínimo 3.800.000 Bs/día

Respuesta Nº 8

Interés *1año *100 2.700.000 *1*100


X= ⇒X= = 2,14
Capital *12% 10.500.000 *12

Se requiere de 2,14 años, para obtener la ganancia de Bs. 2.700.000 de un capital de


Bs. 10.500.000 al 12% anual.

Respuesta Nº 9
15 100 48.000*100
= = ⇒ X = 320.000
48.000 X 15* X

Se requiere un capital de 320.000 Bs, para obtener un interés de 48.000 Bs al año.

Marketing Industrial

Respuesta Nº 10

Un plan de marketing es eficiente porque permite dirigirse a los clientes y atraer a mu-
chos otros; ayuda también a determinar cualidades tales como a qué público hay que dirigirse,
cómo contactarlos y llegar a ellos. Puede aportar información sobre qué necesidades tienen
los consumidores. Estos datos sumados a muchos más, proporcionarán información sobre
los métodos que contribuirán al aumento de la eficacia y suma de beneficios.

Respuesta N° 11

El grupo meta se compone de consumidores potenciales. Puede definirse por elemen-


tos demográficos, tales como edad, el ingreso, la educación o el tamaño de la familia. Puede
influir también la psicografía que estudia los estilos de vida o las personalidades. Un método
para determinar el grupo meta es dividir el mercado en segmentos con base en los principa-
les beneficios que se pretende buscar, para luego desarrollar el producto y dirigir su publici-
dad hacia uno de estos grupos.

593
Sánchez Marchán, Nanci. (2014).Introducción a la Ingeniería Industrial, Material Instruccional de Apoyo. Caracas: UNA

Respuesta Nº 13

Las actividades que se pueden presentar en el campo de la promoción de ventas, son


las siguientes:
•• Exposiciones.
•• Muestras gratis.
•• Cupones.
•• Rifas y concursos.
•• Rebajas.
•• Reembolsos.
•• Descuentos al comercio.
•• Exhibiciones en el punto de compra, entre otras.

COMPORTAMIENTO ORGANIZACIONAL

Respuesta Nº 14

La respuesta es la número 1, es decir: sicología, sociología, antropología, ciencia polí-


tica.

Respuesta Nº 15

Las fuentes de actitudes pueden ser: padres, maestros, amigos, enemigos, moda, entre
otros.

Respuesta Nº 16

Los factores que permiten medir la satisfacción laboral pueden ser:


•• Tipos de tareas.
•• Compañeros de trabajo.
•• Prestaciones, sueldos y beneficios.
•• Seguridad en el sitio de tareas.
•• Ser tratado con respeto y justicia.
•• Oportunidades de dar ideas y opinar.
•• Reconocimiento del desempeño.
•• Oportunidades de ascenso, entre otros.

594
Sánchez Marchán, Nanci. (2014).Introducción a la Ingeniería Industrial, Material Instruccional de Apoyo. Caracas: UNA

Respuesta Nº 17

Se puede expresar la insatisfacción laboral a través de:


•• Salida: retiro de la organización.
•• Vociferar: acciones concretas de mejora de condiciones.
•• Lealtad: espera resignada de mejora de condiciones.
•• Negligencia: permitir que las condiciones desmejoren.

UNIDAD Nº 4

Estadística

Respuesta Nº 1

La metodología estadística se emplea en muchos campos. Se aplica en los programas


de Gobierno, ingeniería, agronomía, economía, medicina, biología, sicología, pedagogía, so-
ciología, física, entre otras; no hay alguna ciencia que no la use o profesión que no la aplique.
La estadística es una disciplina que ayuda a diseñar el esquema de búsqueda y registro de
información para describirla y analizarla con facilidad y mediante estimaciones, obtener con-
clusiones que enriquecen el conocimiento de la realidad.

Respuesta Nº 2

La estadística es fundamental para la Ingeniería Industrial y sus áreas en una empresa,


ya que contribuye tanto para la administración financiera en el proceso de ventas, el marke-
ting, las cobranzas, la gestión de personal, entre otros; como para la administración de ope-
raciones en cuanto a planeación de la producción, control de calidad, así como la logística
entre otras áreas y actividades. Una empresa debe procurar ser cada día más productiva,
eliminando sistemáticamente los desperdicios. Hacer esto posible exige de información, que
se puede obtener gracias a la estadística.

Respuesta Nº 3

Los conceptos básicos que conforman los contenidos de esta materia son:
a) Población.
b) Muestra.
c) Variable de estudio.
d) Datos u observaciones.
e) Parámetros.
f) Estadígrafos o Estadísticos.

595
Sánchez Marchán, Nanci. (2014).Introducción a la Ingeniería Industrial, Material Instruccional de Apoyo. Caracas: UNA

Ingeniería de Sistemas

Respuesta Nº 4

Existe un importante grupo de técnicas, algoritmos y programas utilizados en la Inge-


niería de Sistemas entre ellos se pueden mencionar el modelo de programación lineal y el
algoritmo simplex, el algoritmo de punto interior, el modelo de programación lineal entera,
entre otros; estos modelos y algoritmos han resuelto durante muchos años, importantes pro-
blemas de optimización del mundo industrial real.

Respuesta Nº 5

Para satisfacer las demandas regionales de la industria metalmecánica se debe elegir


la opción Nº 2, corrección del sistema.

Respuesta Nº 6

Dentro de la Ingeniería Industrial también se presenta la elección de estrategias, diseño


e implantación de sistemas de control, desarrollo de controles administrativos y técnicas de
optimización, en atención a esto el Ingeniero de Sistemas es capaz de apoyar a la ingeniería
industrial y sus áreas, realizando las siguientes tareas:
•• Desarrollando, evaluando y optimizando softwares.
•• Diseñando recursos computacionales.
•• Creando modelos matemáticos, estadísticos y de simulación.
•• Evaluando e instalando equipos.
•• Desarrollando la organización y arquitectura de equipos, entre otras.

Respuesta Nº 8

Las etapas de la toma de decisión para dar solución a un problema son:


1. Identificación y diagnóstico del problema.
2. Generación de soluciones alternativas.
3. Evaluación de las alternativas.
4. Selección de la mejor alternativa.
5. Implantación de la decisión.

Respuesta Nº 9

En la labor administrativa, continuamente ocurren situaciones en las que el estratega,


ya sea por errores estructurales o por el estilo gerencial, toma decisiones que le competen,
ya sea a su superior o a sus subordinados, pero no a él; esto trae como consecuencia un de-

596
Sánchez Marchán, Nanci. (2014).Introducción a la Ingeniería Industrial, Material Instruccional de Apoyo. Caracas: UNA

sastre psicológico, ya que se responsabiliza por cuestiones que no le corresponden; o bien,


dedica su tiempo a resolver situaciones que deben ser resueltas por otros.

En este sentido para poder determinar de quién es la responsabilidad de la decisión a


tomar, se pueden seguir algunas indicaciones como:
•• La decisión afecta áreas que no son de la competencia del decisor.
•• La decisión requiere información disponible sólo en niveles superiores.
•• El problema se encuentra fuera del área de responsabilidad o autoridad
del decisor.
•• La decisión supone una alteración del presupuesto del decisor.

Si el decisor cae en alguna de estas situaciones, lo más probable es que esté tomando
decisiones que no le corresponden.

Respuesta Nº 10

El tipo de decisión que se corresponde con el escenario es una decisión programada,


la cual debe tener identificadas aquellas situaciones que, por su recurrencia o importancia,
puedan ser tipificadas y se tenga decidido lo que se debe hacer.

597

You might also like